Download as pdf or txt
Download as pdf or txt
You are on page 1of 537

St

udy
C
ha
m
pi
on
(Y
ou
Tu
be
)
)
be
Tu
ENGINEERING

ou
ENTRANCE EXAM 2022

Solved Papers
(Y
on
pi
m
ha
C
dy
u
St
)
be
Tu
ou
ENGINEERING
ENTRANCE EXAM 2022
(Y
Solved Papers
on
pi
m
ha
C
dy

Authentic & Explanatory Solution


u

of Each Question
St

With 5 Practice Sets

Arihant Prakashan (Series), Meerut


)
be
Tu
ou
Arihant Prakashan (Series), Meerut
All Rights Reserved (Y
on
© Publisher
No part of this publication may be re-produced, stored in a retrieval system or by any
means, electronic, mechanical, photocopying, recording, scanning, web or otherwise
pi

without the written permission of the publisher. Arihant has obtained all the
information in this book from the sources believed to be reliable and true. However,
m

Arihant or its editors or authors or illustrators don’t take any responsibility for the
absolute accuracy of any information published and the damage or loss suffered
thereupon.
ha

All disputes subject to Meerut (UP) jurisdiction only.

Administrative & Production Offices


C

Regd. Office
‘Ramchhaya’ 4577/15, Agarwal Road, Darya Ganj, New Delhi -110002
dy

Tele: 011- 47630600, 43518550

Head Office
u

Kalindi, TP Nagar, Meerut (UP) - 250002


Tel: 0121-7156203, 7156204
St

Sales & Support Offices


Agra, Ahmedabad, Bengaluru, Bareilly, Chennai, Delhi, Guwahati,
Hyderabad, Jaipur, Jhansi, Kolkata, Lucknow, Nagpur & Pune.

ISBN 978-93-25793-15-6
PO No : TXT-XX-XXXXXXX-X-XX
Published by Arihant Publications (India) Ltd.
For further information about the books published by Arihant, log on to
www.arihantbooks.com or e-mail at info@arihantbooks.com
Follow us on
FIRST &
FOREMOST

)
be
Tu
Whenever a student decides to prepare for any examination
his/her first and foremost curiosity is to know about the type of
questions that are expected in the exams. This becomes more

ou
important in the context of competitive examinations where they
have to face neck-to-neck competition.
We feel great pleasure in presenting before you this book

(Y
containing Error-Free Solutions of previous years’ (2021-2012)
WB JEE (Engineering) questions and 5 Practice Sets for practice
Going through this book, you will get an exact idea of the
on
questions generally asked in West Bengal Joint Entrance Exam and
become exam-ready.
pi

We have made maximum efforts to provide correct solutions to


the best of our knowledge and opinion. Detailed explanatory
m

discussions follow the answers. Discussions are not just sketchy


rather, have been achieved in a manner that the students will
ha

surely be able to solve some other related problems too. In this


way, the students shall be able to judge the extent upto which
they have been able to comprehend the concepts and become
C

skilled.
We hope, these solved papers along with error-free solutions
dy

would be highly beneficial to the students. We would be grateful if


any discrepancy or mistake in the questions or answers is brought
to our notice so that these could be rectified in subsequent
u

editions.
St

PUBLISHER
CONTENTS

)
be
Tu
SOLVED PAPERS
Solved Paper 2021 1-47

ou
Solved Paper 2020 1-45

Solved Paper 2019 (Y 1-46


on
Solved Paper 2018 1-44
pi

Solved Paper 2017 1-47


m

Solved Paper 2016 1-39


ha

Solved Paper 2015 1-38


C

Solved Paper 2014 1-56


dy

Solved Paper 2013 1-54


u

Solved Paper 2012 1-32


St

5 Practice Sets 3-74


SYLLABUS

)
be
Tu
PHYSICS
Physical World, Measurements, Units & Bulk properties of matter Elasticity, Hooke's law,

ou
dimensions Physical World, Measurements, Units & Young's modulus, bulk modulus, shear, rigidity
dimensions Units & Dimensions of physical quantities, modulus, Poisson's ratio elastic potential energy. Fluid

(Y
dimensional analysis & its applications, error in pressure: Pressure due to a fluid column, buoyancy,
measurements, significant figures. Pascal's law, effect of gravity on fluid pressure. Surface
tension: Surface energy, phenomena involving
Kinematics Scalars & vectors, representation of
on
surface tension, angle of contact, capillary rise.
vectors in 3D, dot & cross product & their
applications, elementary differential & integral Viscosity Coefficient of viscosity, streamline &
calculus, time-velocity & relevant graphs, equations of turbulent motion, Reynold's number, Stoke's law,
pi

motion with uniform acceleration. terminal velocity, Bernoulli's theorem.


m

Laws of motion Newton's laws of motion, using Heat & Thermal Physics Heat & temperature, thermal
algebra & calculus, inertial & non inertial frames, expansion of solids. liquids & gases, ideal gas laws,
ha

conservation of linear momentum with applications, isothermal & adiabatic processes; anomalous
elastic & inelastic collisions, impulse centripetal force, expansion of water & its effects, sp. heat capacity.
banking of roads, relative velocity, projectile motion &
Cp, Cv, calorimetry; change of state, specific latent
C

uniform circular motion.


heat capacity. Heat transfer; conduction, thermal and
Work, power, energy Work, power, energy Work, thermometric conductivity, convection & radiation,
dy

work-energy theorem, power, energy, work done by Newton's law of cooling, Stefan's law.
constant & variable forces, PE & KE, conservation of
Thermodynamics Thermal equilibrium (Zeroth law
mechanical energy, conservative and non-
u

of thermodynamics), heat, work & internal energy. 1st


conservative forces, PE of a spring.
law of thermodynamics, isothermal & adiabatic
St

Motion of centre of mass, connected systems, processes, 2nd law of thermodynamics, reversible &
Friction Centre of mass of two-particle system, irreversible processes.
motion of connected system, torque, equilibrium of Kinetic theory of gases Equation of state of a perfect
rigid bodies, moments of inertia of simple geometric gas, kinetic theory of gases, assumptions in Kinetic
bodies (2D) [without derivation] conservation of theory of gases, concept of pressure & temperature;
angular momentum, friction and laws of friction. rms speed of gas molecules; degrees of freedom, law
Gravitation Kepler's laws, (only statement) universal of equipartition of energy (introductory ideas) &
law of gravitation, acceleration due to gravity (g), application to specific heats of gases; mean free path,
variation of g, gravitational potential & PE, escape Avogadro number.
velocity, orbital velocity of satellites, geostationary Oscillations & Waves Periodic motion – time period,
orbits. frequency, time-displacement eqation, Simple
harmonic motion (S.H.M) & its equation; phase; SHM Magnetic effect of current Concept of magnetic

)
in different sytems, restoring force & force const, field, Oersted's experiment, Biot - Savart law & its

be
energy in S.H.M.-KE & PE, free, forced & damped application to current carrying circular loop; Ampere's
oscillations (introductory ideas), resonance wave law & its applications to infinitely long straight wire,

Tu
motion, equation for progressive wave, longitudinal & straight and toroidal solenoids; force on a moving
transverse waves, sound waves, Newton's formula & charge in uniform magnetic & electric fields, cyclotron
Laplace's correction, factors affecting the velocity of frequency; force on a current-carrying conductor in a

ou
sound in air, principles of superposition of waves, uniform magnetic field, force between two parallel
reflection of waves, standing waves in strings & organ current-carrying conductors-- definition of ampere.
pipes, fundamental mode, harmonics & overtones, Torque experienced by a current loop in a uniform
beats, Doppler effect.
Electrostatics Conservation of electric charges,
Coulomb's law-force between two point charges,
(Y
magnetic field; moving coil galvanometer-its current
sensitivity & conversion to ammeter & voltmeter,
Inter-conversion of voltmeter & ammeter & change of
on
their ranges.
forces between multiple charges; superposition
principle & continuous charge distribution. Electric Magnetics Current loop as a magnetic dipole & its
pi

field, & potential due to a point charge & distribution magnetic dipole moment, magnetic dipole moment
of charges, electric field lines electric field due to a of a revolving electron, magnetic field intensity due
m

dipole; torque on a dipole in uniform electric field; to a magnetic dipole bar magnet along its axis &
electric flux, Gauss' theorem & its simple applications, perpendicular to its axis, torque on a magnetic dipole
conductors & insulators, free charges & bound
ha

(bar magnet) in a uniform magnetic field; magnet as


charges inside a conductor; dielectrics & electric an equivalent solenoid, magnetic field lines; Earth's
polarisation, capacitors & capacitance, combination magnetic field & its magnetic elements. para-, dia- &
C

of capacitors in series & in parallel, capacitance of a ferro- magnetic substances, with examples.
parallel plate capacitor with & without dielectric Electromagnets & the factors affecting their
medium between the plates, energy stored in a strengths, permanent magnets.
dy

capacitor.
Electromagnetic induction & alternating current
Current Electricity Electric current, & conductor, drift Electromagnetic induction; Faraday's laws, induced
u

velocity' mobility & their relation with electric current; emf & current; Lenz's Law, eddy currents, self &
St

Ohm's law, electrical resistance, Ohmic and non- mutual induction, alternating currents, peak and rms
Ohmic conductors, electrical energy & power, carbon value of alternating current and voltage; reactance
resistors, colour codes, combination of resistances, and impedance; LR & CR circuits, phase lag & lead,
temperature dependence of resistances, electric cell, LCR series circuit, resonance; power in AC circuits,
emf and internal resistance of an electric cell, pd, wattless current.
combination of cells, secondary cells, (introductory)
Electromagnetic waves Electromagnetic waves and
Kirchoff's laws of electrical network, simple
their characteristics (qualitative ideas only),
applications, principle of Wheatstone bridge, metre
transverse nature of electromagnetic waves,
bridge and potentiometer and their uses,
electromagnetic spectrum, applications of the waves
thermoelectricity; Seebeck effect; Peltier effect,
from the different parts of the spectrum.
thermo emf.
Optics I (Ray optics) Reflection of light, spherical Einstein's photoelectric equation - particle nature of

)
mirrors, mirror formula. Refraction of light, total light, matter waves; wave nature of particles, de

be
internal reflection & its applications, optical fibres, Broglie relation.
refraction at spherical surfaces, lenses, thin lens
Atomic Physics Alpha-particle scattering expt

Tu
formula, lensmaker's formula. Newton's relation:
Rutherford's nuclear atom model of atom; Bohr
Displacement method to find position of images
model of hydrogen atom, energy levels in a hydrogen
(conjugate points) Magnification, power of a lens,
atom, hydrogen spectrum, continuous &

ou
combination of thin lenses in contact, combination of
characteristic x-rays.
a lens & a mirror refraction and dispersion of light
through a prism; optical instruments, human eye, Nuclear Physics Composition & size of nucleus,
image formation & accommodation, correction of eye
defects (myopia, hypermetropia) using lenses,
microscopes & astronomical telescopes (reflecting & (Y
atomic masses, isotopes, isobars; isotones,
radioactivity - alpha, beta & gamma particles/ rays &
their properties; radioactive decay law; mass-energy
on
refracting) & their magnifying powers. relation, mass defect; binding energy per nucleon &
its variation with mass number; nuclear fission &
Optics II (Wave Optics) Scattering of light - blue
fusion.
colour of the sky, elementary idea of Raman effect;
pi

wave optics: wave front & Huygens' principle, Solid state Electronics Energy bands in solids
(qualitative ideas only), conductors, insulators &
m

reflection & refraction of plane wave at a plane


surface using wave fronts. Proof of laws of reflection & semiconductors; semiconductor diode – I-V
refraction using Huygens' principle Interference, characteristics in forward & reverse bias, diode as a
ha

Young's double slit experiment & expression for rectifier; I-V characteristics of LED, photodiode, solar
fringe width, coherent sources, Fraunhoffer cell & Zener diode; Zener diode as a voltage regulator,
C

diffraction due to a single slit. junction transistor (BJT), transistor action,


characteristics of a BJT, BJT as an amplifier
Particle nature of light & wave particle dualism
(CE configuration) & oscillator; logic gates (OR, AND,
dy

Photoelectric effect, Hertz and Lenard's observations;


NOT, NAND & NOR).
u

CHEMISTRY
St

Atoms, Molecules and Chemical Arithmetic mole fraction, molarity, molality and normality.
Dalton's atomic theory; Gay Lussac's law of gaseous Percentage composition, empirical formula and
volume; Avogadro's Hypothesis and its applications. molecular formula; Numerical problems.
Atomic mass; Molecular mass; Equivalent weight; Atomic Structure Concept of Nuclear Atom –
Valency; Gram atomic weight; Gram molecular electron, proton and neutron (charge and mass),
weight; Gram equivalent weight and mole concept; atomic number. Rutherford's model and its
Chemical formulae; Balanced chemical equations; limitations; Extra nuclear structure; Line spectra of
Calculations (based on mole concept) involving hydrogen atom.
common oxidation – reduction, neutralization, and Quantization of energy (Planck's equation E = hJ);
displacement reactions; Concentration in terms of Bohr's model of hydrogen atom and its limitations,
Sommerfeld's modifications (elementary idea); The Directionality of covalent bonds, shapes of poly –

)
four quantum numbers, ground state electronic atomic molecules (examples); Concept of

be
configurations of many electron atoms and mono – hybridization of atomic orbitals (qualitative pictorial
atomic ions; The Aufbau Principle; Pauli's Exclusion approach): sp, sp2, sp3 and dsp2.
Principle and Hund's Rule. Molecular orbital energy diagrams for homonuclear

Tu
Dual nature of matter and light, de Broglie's diatomic species – bond order and magnetic
relationship, Uncertainty principle; The concept of properties.
atomic orbitals, shapes of s, p and d orbitals (pictorial

ou
Valence Shell Electron Pair Repulsion (VSEPR) concept
approach).
(elementary idea) – shapes of molecules. Concept of
Radioactivity and Nuclear Chemistry Radioactivity resonance (elementary idea), resonance structures
a-, b-, g rays and their properties; Artificial
transmutation; Rate of radioactive decay, decay
constant, half-life and average age life period of (Y
(examples). Elementary idea about electronegativity,
bond polarity and dipole moment, inter- and intra-
molecular hydrogen bonding and its effects on
on
radio-elements; Units of radioactivity; Numerical physical properties (mp, bp and solubility); Hydrogen
problems. bridge bonds in diborane.
Stability of the atomic nucleus effect of neutron-
pi

Coordination Compounds Introduction, Double


proton (n/p) ratio on the modes of decay, group salts and complex salts, coordination compounds
displacement law, radioisotopes and their uses (C, P,
m

(examples only), Werner's theory, coordination


Co and I as examples) isobars and isotones (definition number (examples of coordination number 4 and 6
and examples), elementary idea of nuclear fission and
ha

only), colour, magnetic properties and shapes, IUPAC


fusion reactions. nomenclature of mononuclear coordination
The Periodic Table and Chemical Families Modern compounds.
C

periodic law (based on atomic number); Modern Solid State Classification of solids based on different
periodic table based on electronic configurations, binding forces: molecular, ionic, covalent and metallic
groups (Gr. 1-18) and periods. Types of elements –
dy

solids, amorphous and crystalline solids (elementary


representative (s-block and p- block), transition (d- idea). Unit cell in two dimensional and three
block) elements and inner transition (f-block / dimensional lattices, calculation of density of unit cell,
u

lanthanides and actinides) and their general packing in solids, packing efficiency, voids, number of
characteristics. Periodic trends in physical and
St

atoms per unit cell in a cubic unit cell, point defects,


chemical properties – atomic radii, valency, ionization electrical and magnetic properties. Band theory of
energy, electron affinity, electronegativity, metallic metals, conductors, semiconductors and insulators
character, acidic and basic characters of oxides and and n & p type semiconductors.
hydrides of the representative elements (up to Z =
36). Position of hydrogen and the noble gases in the Liquid State Vapour pressure, viscosity and surface
periodic table; Diagonal relationships. tension (qualitative idea only, no mathematical
derivations).
Chemical Bonding and Molecular Structure
Valence electrons, the Octet rule, electrovalent, Gaseous State Measurable properties of gases.
covalent and coordinate covalent bonds with Boyle's Law and Charles Law, absolute scale of
examples; Properties of electrovalent and covalent temperature, kinetic theory of gases, ideal gas
compounds. Limitations of Octet rule (examples); equation – average, root mean square and most
Fajans Rule. probable velocities and their relationship with
)
temperature. Daltons Law of partial pressure, Electrolytic Solutions Specific conductance,

be
Grahams Law of gaseous diffusion. Deviations from equivalent conductance, ionic conductance,
ideal behavior. Liquefaction of gases, real gases, van Kohlrausch's law, Faraday's laws of electrolysis,
der Waals equation; Numerical problems. applications. Numerical problems.

Tu
Chemical Energetics and Chemical Dynamics Non-electrolytic Solutions Types of solution, vapour
Chemical Energetics – Conservation of energy pressure of solutions. Raoult's Law; Colligative

ou
principle, energy changes in physical and chemical properties – lowering of vapour pressure, elevation of
transformations. First law of thermodynamics; boiling point, depression of freezing point, osmotic
Internal energy, work and heat, pressure – volume pressure and their relationships with molecular mass
work; Enthalpy. Internal energy change (ΔE) and
Enthalpy change (ΔH) in a chemical reaction. Hesss
Law and its applications (Numerical problems). Heat
(Y
(without derivations); Numerical problems.

Ionic and Redox Equilibria Ionic equilibria –


ionization of weak electrolytes, Ostwald's dilution law.
on
of reaction, fusion and apourization; Second law of
thermodynamics; Entropy; Free energy; Criterion of Ionization constants of weak acids and bases, ionic
spontaneity. Third law of thermodynamics (brief product of water, the pH – scale, pH of aqueous
pi

introduction). solutions of acids and bases; Buffer solutions, buffer


action and Henderson equation.
m

Chemical Equilibria The Law of mass action,


dynamic nature of chemical equilibria. Equilibrium Acid-base titrations, acid base indicators (structures
constants, Le Chateliers Principle. Equilibrium not required). Hydrolysis of salts (elementary idea),
ha

constants of gaseous reactions (Kp and Kc) and solubility product, common ion effect (no numerical
relation between them (examples). Significance of problems).
C

ΔG and ΔGº.
Redox Equilibria: Oxidation Reduction reactions as
Chemical Dynamics Factors affecting the rate of electron transfer processes, oxidation numbers,
dy

chemical reactions (concentration, pressure, balancing of redox reactions by oxidation number


temperature, catalyst), Concept of collision theory. and ion-electron methods. Standard electrode
Arrhenius equation and concept of activation energy. potentials (E°), Electrochemical series, feasibility of a
u

Order and molecularity (determination excluded); redox reaction. Significance of Gibb's equation: ΔG° =
St

First order reactions, rate constant, half – life – nFΔE° (without derivation), no numerical problems.
(numerical problems), examples of first order and Redox titrations with (examples); Nernst equations
second order reactions. (Numerical problems).
Physical Chemistry of Solutions Colloidal Solutions Hydrogen Position of hydrogen in periodic table,
– Differences from true solutions; Hydrophobic and occurrence, isotopes, preparation, properties and
hydrophilic colloids (examples and uses); Coagulation uses of hydrogen, hydrides-ionic covalent and
and peptization of colloids; Dialysis and its interstitial; physical and chemical properties of water,
applications; Brownian motion; Tyndall effect and its heavy water, hydrogen peroxide – preparation,
applications; Elementary idea of emulsion, surfactant reactions and structure and use; hydrogen as a fuel.
and micelle.
Chemistry of Non-Metallic Elements and their Chemistry in Industry Large scale production

)
Compounds Carbon – occurrence, isotopes, (including physicochemical principles where

be
allotropes (graphite, diamond, fullerene); CO and CO2 applicable, omitting technical details) and uses of
production, properties and uses. Sulphuric acid (contact process), Ammonia (Haber's

Tu
Nitrogen and Phosphorus occurrence, isotopes, process), Nitric acid (Ostwald's process), sodium bi-
allotopes, isolation from natural sources and carbonate and sodium carbonate (Solvey process).
purification, reactivity of the free elements. Polymers Natural and synthetic polymers, methods

ou
Preparation, properties, reactions of NH3, PH3, NO, of polymerization (addition and condensation),
NO2, HNO2, HNO3, P4O10, H3PO3 and H3PO4. copolymerization, some important polymers –
Oxygen and Sulphur Occurrence, isotopes, allotropic natural and synthetic like polythene, nylon polyesters,
forms, isolation from natural sources and purification,
properties and reactions of the free elements. Water,
unusual properties of water, heavy water (production
(Y
bakelite, rubber. Biodegradable and non-
biodegradable polymers.
Surface Chemistry Adsorption – physisorption and
on
and uses). Hydrogen peroxide and ozone (production, chemisorption, factors affecting adsorption of gases
purification, properties and uses). on solids, catalysis, homogenous and heterogenous
activity and selectivity; enzyme catalysis colloidal
pi

Halogens comparative study, occurrence, physical


states and chemical reactivities of the free elements, state distinction between true solutions, colloids and
m

peculiarities of fluorine and iodine; Hydracids of suspension; lyophilic, lyophobic multimolecular and
halogens (preparation, properties, reactions and macromolecular colloids; properties of colloids;
Tyndall effect, Brownian movement, electrophoresis,
ha

uses), inter-halogen compounds (examples); Oxyacids


of chlorine. coagulation, emulsion – types of emulsions.
Chemistry of Metals General principles of metallurgy Environmental Chemistry Common modes of
C

– occurrence, concentration of ores, production and pollution of air, water and soil. Ozone layer, ozone
purification of metals, mineral wealth of India. Typical hole – important chemical reactions in the
dy

metals (Na, Ca, Al, Fe, Cu and Zn) – occurrence, atmosphere, Smog; major atmospheric pollutants;
extraction, purification (where applicable), properties Green House effect; Global warming – pollution due
and reactions with air, water, acids and non-metals. to industrial wastes, green chemistry as an alternative
u

Manufacture of steels and alloy steel (Bessemer, tool for reducing pollution, strategies for control of
environment pollution.
St

Open-Hearth and L.D. process).


Principles of Chemistry involved in electroplating, Chemistry of Carbon Compounds Hybridization of
anodizing and galvanizing. carbon:  – and  – bonds.
Preparation and properties of K2Cr2O7 and KMnO4. Isomerism constitutional and stereoisomerism;
Geometrical and optical isomerism of compounds
Lanthanoids Electronic configuration, oxidation
containing upto two asymmetric carbon atoms.
states, chemical reactivity and lanthanoid contraction
IUPAC nomenclature of simple organic compounds –
and its consequences.
hydrocarbons, mono and bifunctional molecules only
Actinoids Electronic configuration, oxidation states (alicyclic and heterocyclic compounds excluded).
and comparison with lanthanoids. Conformations of ethane and n-butane (Newman
projection only).
Electronic Effects Inductive, resonance and hydrazine, hydroxyl amines, semi carbazides, alcohols;

)
hyperconjugation. Stability of carbocation, carbanion Aldol condensation, Clemmensen and Wolff – Kishner

be
and free radicals; Rearrangement of carbocation; reduction, haloform, Cannizzaro and Wittig reactions.
Electrophiles and nucleophiles, tautomerism in Carboxylic Acids Hydrolysis of esters (mechanism
b-dicarbonyl compounds, acidity and basicity of excluded) and cyanides; Hunsdicker and HVZ

Tu
simple organic compounds. reactions.
Aliphatic Compounds Alkanes – Preparation from Aliphatic Amines Preparation from nitro, cyano and

ou
alkyl halides and carboxylic acids; Reactions — amido compounds. Distinction of 1º, 2º and 3º amines
halogenation and combustion.
(Hinsberg method); Reaction with HNO2; Carbyl
Alkenes and Alkynes Preparation from alcohols; amine reaction.
Formation of Grignard reagents and their synthetic
applications for the preparation of alkanes, alcohols,
aldehydes, ketones and acids; SNl and SN2 reactions
(Y
Aromatic Compounds Benzene – Kekule structure,
aromaticity and Hückel rule. Electrophilic substitution
on
(preliminary concept). Markownikoff's and anti- – halogenation, sulfonation, nitration, Friedel Crafts
Markownikoff's additions; Hydroboration; reaction, ozonolysis. Directive influence of
Oxymercuration-demercuration, reduction of alkenes substituents in monosubstituted benzenes.
pi

and alkynes (H2/Lindler catalyst and Na in liquid NH3), Carcinogenicity and toxicity.
metal acetylides. Amines Preparation from reduction of nitro
m

Haloalkanes and Haloarenes Haloalkanes – compounds; Formation of diazonium salts and their
stability; Replacement of diazonium group with H,
ha

Preparation from alcohols; Nomenclature, nature of C


-X bond, physical and chemical properties, OH, X (halogen), CN and NO2, diazocoupling and
mechanism of substitution reactions, optical rotation. reduction.
C

Formation of Grignard reagents and their synthetic Haloarenes Nature of C-X bond, substitution
applications for the preparation of alkanes, alcohols, reactions; Nucleophilic substitution, cine substitution
aldehydes, ketones and acids; SN1 and SN2 reactions (excluding mechanism, Directive influence of halogen
dy

(preliminary concept). in monosubstituted compounds only).


Uses and environmental effects of - dichloromethane, Phenols halogenation, sulfonation, nitration, Reimer
u

trichloromethane, tetrachloromethane, iodoform, – Tiemann and Kolbe reactions.


St

freons, DDT.
Aromatic Aldehydes Preparation by Gattermann,
Alcohols Preparation of alcohols from carbonyl Gattermann-Koch, Rosenmund and Stephen's
compounds and esters. Reaction – dehydration, method. Reactions – Perkin, Benzoin and Cannizzaro.
oxidation, esterification, reaction with sodium,
Application Oriented chemistry Main ingredients,
ZnCl2/HCl, phosphorous halides.
their chemical natures (structures excluded) and their
Ethers Preparation by Williamson's synthesis; side effects, if any, of common antiseptics, analgesics,
Cleavage with HCl and HI. antacids, vitamin-C.
Aldehydes and Ketones Preparation from esters,
Introduction to Bio-Molecules Carbohydrates –
acid chlorides, gem-dihalides, Ca-salt of carboxylic
Pentoses and hexoses. Distinctive chemical reactions
acids. Reaction – Nucleophilic addition with HCN,
of glucose.
Aminoacids glycine, alanine, aspartic acid, cysteine by dry and wet tests from among: Acid Radicals: Cl-, S2-
(structures). Zwitterion structures of amino acids, , SO42-, NO3– , CO32-. Basic Radicals: Cu2+, Al3+ , Fe3+, Fe2+,
peptide bond. Zn2+, Ca2+, Mg2+, Na+, NH4+ .

)
ADP and ATP structures and role in bioenergetics; Detection of special elements (N, Cl, Br, I and S) in

be
Nucleic acids – DNA and RNA skeleton structures. organic compounds by chemical tests.
Names of essential elements in biological system. Identification of functional groups in: phenols,
aromatic amines, aldehydes, ketones and carboxylic

Tu
Principles of Qualitative Analysis Detection of
water soluble non-interfering Acid and Basic Radicals acids.

ou
MATHEMATICS
Algebra
A.P., G.P., H.P. Definitions of A. P. and G.P.; General (Y
Problems involving both permutations and
combinations.
on
term; Summation of first
n-terms of series nnnArithmetic/Geometric Principle of mathematical induction Statement of
series, A.M., G.M. and their relation; Infinite G.P. series the principle, proof by induction for the sum of
pi

and its sum. squares, sum of cubes of first n natural numbers,


divisibility properties like 22n— 1 is divisible by
Logarithms Definition; General properties; Change
m

3 (n ≥ 1), 7 divides 32n+1+2n+2 (n ≥ 1)


of base.
Binomial theorem (positive integral index)
ha

Complex Numbers Definition and properties of Statement of the theorem, general term, middle term,
complex numbers; Complex conjugate; Triangle equidistant terms, properties of binomial coefficients.
inequality; Square root of complex numbers; Cube
C

roots of unity; De Moivre's theorem (statement only) Matrices Concepts of m x n (m ≤ 3, n ≤ 3) real


and its elementary applications. Solution of quadratic matrices, operations of addition, scalar multiplication
and multiplication of matrices. Transpose of a matrix.
dy

equation in complex number system.


Determinant of a square matrix. Properties of
Quadratic Equations Quadratic equations with real determinants (statement only). Minor, cofactor and
coefficients; Relations between roots and coefficients;
u

adjoint of a matrix. Nonsingular matrix. Inverse of a


Nature of roots; Formation of a quadratic equation, matrix. Finding area of a triangle. Solutions of system
St

sign and magnitude of the quadratic expression of linear equations. (Not more than 3 variables).
ax2+bx+c
(where a, b, c are rational numbers and a ≠ 0). Sets, Relations and Mappings Idea of sets, subsets,
power set, complement, union, intersection and
Permutation and combination Permutation of n difference of sets, Venn diagram, De Morgan's Laws,
different things taken r at a time (r ≤ n). Permutation Inclusion / Exclusion formula for two or three finite
of n things not all different. Permutation with sets, Cartesian product of sets.
repetitions (circular permutation excluded).
Relation and its properties. Equivalence relation —
Combinations of n different things taken r at a time (r definition and elementary examples, mappings,
≤ n). Combination of n things not all different. Basic range and domain, injective, surjective and bijective
properties. mappings, composition of mappings, inverse of a
mapping.
Statistics and Probability Measure of dispersion, Co-ordinate geometry of three dimensions

)
mean, variance and standard deviation, frequency Direction cosines and direction ratios, distance

be
distribution. between two points and section formula, equation of
a straight line, equation of a plane, distance of a point
Addition and multiplication rules of probability,
from a plane.

Tu
conditional probability and Bayes' Theorem,
independence of events, repeated independent trails
and Binomial distribution. Calculus

ou
Trigonometry Trigonometric functions, addition and Differential calculus Functions, composition of two
subtraction formulae, formulae involving multiple functions and inverse of a function, limit, continuity,
and sub-multiple angles, general solution of derivative, chain rule, derivative of implicit functions
trigonometric equations.
Properties of triangles, inverse trigonometric (Y
and functions defined parametrically.
Rolle's Theorem and Lagrange's Mean Value theorem
on
functions and their properties. (statement only). Their geometric interpretation and
elementary application. L'Hospital's rule (statement
Coordinate geometry of two dimensions Distance
only) and applications. Second order derivative.
formula, section formula, area of a triangle, condition
pi

of collinearity of three points in a plane. Integral calculus Integration as a reverse process of


differentiation, indefinite integral of standard
m

Polar coordinates, transformation from Cartesian to


functions. Integration by parts. Integration by
polar coordinates and vice versa. Parallel
substitution and partial fraction.
ha

transformation of axes, concept of locus, elementary


locus problems. Definite integral as a limit of a sum with equal
subdivisions. Fundamental theorem of integral
Slope of a line. Equation of lines in different forms,
C

calculus and its applications. Properties of definite


angle between two lines. Condition of integrals.
perpendicularity and parallelism of two lines.
Differential Equations Formation of ordinary
dy

Distance of a point from a line. Distance between two


differential equations, solution of homogeneous
parallel lines. Lines through the point of intersection
differential equations, separation of variables
of two lines.
u

method, linear first order differential equations.


Equation of a circle with a given center and radius.
St

Application of Calculus Tangents and normals,


Condition that a general equation of second degree
conditions of tangency. Determination of
in x, y may represent a circle. Equation of a circle in
monotonicity, maxima and minima. Differential
terms of endpoints of a diameter . Equation of
coefficient as a measure of rate.
tangent, normal and chord. Parametric equation of a
circle. Intersection of a line with a circle. Equation of Motion in a straight line with constant acceleration.
common chord of two intersecting circles. Geometric interpretation of definite integral as area,
calculation of area bounded by elementary curves
Definition of conic section, Directrix, Focus and
and Straight lines. Area of the region included
Eccentricity, classification based on eccentricity.
between two elementary curves.
Equation of Parabola, Ellipse and Hyperbola in
Vectors Addition of vectors, scalar multiplication, dot
standard form, their foci, directrices, eccentricities
and cross products, scalar triple product.
and parametric equations.
WB JEE
Engineering Entrance Exam

)
Solved Paper 2021

be
Tu
Physics

ou
Category-I (Q. Nos. 1 to 30) 5.
Carry 1 mark each and only one option is correct.
In case of incorrect answer or any combination of
more than one answer, 1/4 mark will be deducted.
(Y 9V
1kΩ
on
10 V

1. A spherical convex surface of power 5 D


separates object and image space of refractive What is the value of current through the
pi

indices 1.0 and 4/3, respectively. The radius diode in the circuit given?
of curvature of the surface is
m

(a) Zero (b) 1 mA


(a) 20 cm (b) 1 cm (c) 4 cm (d) 5 cm (c) 19 mA (d) 9 mA
ha

2. In Young’s double slit experiment, light of 6. A


wavelength λ passes through the double slit B
and forms interference fringes on a screen 1.2
C

Y
m away. If the difference between 3rd order
maximum and 3rd order minimum is 0.18 cm
and the slits are 0.02 cm apart, then λ is
dy

(a) 1200 nm (b) 450 nm For the given logic circuit, the output Y for
(c) 600 nm (d) 300 nm
inputs (A = 0, B = 1) and (A = 0, B = 0)
u

3. A 12.5 eV electron beam is used to bombard respectively are


St

gaseous hydrogen at ground state. The (a) 0, 0 (b) 0, 1


energy level upto which the hydrogen atoms (c) 1, 0 (d) 1, 1
would be excited is
7. From dimensional analysis, the Rydberg
(a) 2 (b) 3 (c) 4 (d) 1
constant can be expressed in terms of electric
4. Let r , v , E be the radius of orbit, speed of charge (e), mass (m) and Planck constant (h) as
electron and total energy of electron 1
[consider ≡ 1 unit]
respectively in H-atom. Which of the 4 πε 0
following quantities according to Bohr theory, h2 me 4
is proportional to the quantum number n? (a) (b)
r r me 2 h2
(a) vr (b) rE (c) (d) 2 4
E v me me 2
(c) 2
(d)
h h
2 WB JEE (Engineering) Solved Paper 2021

8. 12. In case of projectile motion, which one of the


F 3m following figures represent variation of
m 2m
horizontal component of velocity (ux ) with
Three blocks are pushed with a force F across time t ? (Assume that air resistance is
a frictionless table as shown in figure above. negligible)
ux ux
Let N 1 be the contact force between the left
two blocks and N 2 be the contact force

)
between the right two blocks. Then, (a) (b)

be
(a) F > N1 > N2 (b) F > N2 > N1 t t
(c) F > N1 = N2 (d) F = N1 = N2 ux
ux
V

Tu
9. K
(c) (d)
m m

t t

ou
A block of mass m slides with speed v on a
frictionless table towards another stationary
block of mass m. A massless spring with 13. A uniform thin rod of length L, mass m is

(Y
spring constant k is attached to the second lying on a smooth horizontal table. A
block as shown in figure. The maximum horizontal impulse P is suddenly applied
distance, the spring gets compressed through perpendicular to the rod at one end. The
on
is total energy of the rod after the impulse is
m m k k P2 7 P2 13P 2 2 P2
(a) v (b) v (c) v (d) v (a) (b) (c) (d)
k 2k m 2m m 8m 2m m
pi

10. a (in m/s2)


14. Centre of mass (CM) of three particles of
m

masses 1 kg, 2 kg and 3 kg lies at the point


10 (1, 2, 3) and CM of another system of
particles of 3 kg and 2 kg lies at the point
ha

(− 1, 3 , − 2). Where should we put a particle of


5 mass 5 kg, so that the CM of entire system
C

lies at the CM of the first system?


x (in m) (a) (3, 1, 8) (b) (0, 0, 0)
0 20 35
(c) (1, 3, 2) (d) (− 1, 2, 3)
dy

The acceleration versus distance graph for a


. × 10 3 kg/m 3 is dropped
15. A body of density 12
particle moving with initial velocity 5 m/s is
shown in the figure. The velocity of the from rest from a height 1 m into a liquid to
u

particle at x = 35 m will be density 2.4 × 10 3 kg/m 3 . Neglecting all


St

(a) 20.62 m/s (b) 20 m/s dissipative effects, the maximum depth to
(c) 25 m/s (d) 50 m/s which the body sinks before returning to
float on the surface is
11. A simple pendulum, consisting of a small ball (a) 0.1 m (b) 1 m (c) 0.01 m (d) 2 m
of mass m attached to a massless string
hanging vertically from the ceiling is 16. Two solid spheres S1 and S2 of same uniform
oscillating with an amplitude such that density fall from rest under gravity in a
Tmax = 2Tmin , where Tmax and Tmin are the viscous medium and after sometime, reach
maximum and minimum tension in the terminal velocities v1 and v2 , respectively. If
string, respectively. The value of maximum m v
ratio of masses 1 = 8 , then 1 will be equal to
tension Tmax in the string is m2 v2
3 mg 3 mg
(a) (b) mg (c) (d) 3 mg 1 1
2 4 (a) 2 (b) 4 (c) (d)
2 4
WB JEE (Engineering) Solved Paper 2021 3

17. p q  1 − 1 q 1
(a)   (b)
4 π ε0  a b 4 π ε0 a
q  1 − 1 q 1
1 (c)   (d)
p0 4 π ε0  R a 4 π ε0 R
2

3 22. Three infinite plane sheets carrying uniform


charge densities − σ, 2σ, 4 σ are placed
V
V0 2V0 parallel to XZ-plane at Y = a, 3 a , 4 a,

)
respectively. The electric field at the point

be
In the given figure, 1 represents isobaric, 2 (0, 2a, 0) is
represents isothermal and 3 represents 5σ $ 7σ $
adiabatic processes of an ideal gas. If ∆U1 , (a) j (b) − j
2 ε0 2 ε0

Tu
∆U 2 and ∆U 3 be the changes in internal σ $ 5σ $
energy in these processes respectively, then (c) j (d) j
2 ε0 − 2 ε0
(a) ∆U1 < ∆U 2 < ∆U 3 (b) ∆U1 > ∆U 3 < ∆U 2

ou
(c) ∆U1 = ∆U 2 > ∆U 3 (d) ∆U1 > ∆U 2 > ∆U 3 23. Two point charges +q1 and + q2 are placed a
18. If pressure of real gas O 2 in a container is finite distance d apart. It is desired to put a

(Y
RT a third charge q3 in between these two charges,
given by p = − , then the mass of so that q3 is in equilibrium. This is
2 V − b 4 b2
(a) possible only, if q 3 is negative
the gas in the container is
on
(b) possible only, if q 3 is positive
(a) 32 g (b) 16 g (c) possible irrespective of the sign of q 3
(c) 4 g (d) 64 g (d) Not possible at all
pi

19. 300 g of water at 25°C is added to 100 g of ice 24. Y


at 0°C. The final temperature of the mixture
m

is
L
(a) 12.5°C (b) 0°C (c) 25°C (d) 50°C
ha

X
EZ M O
20.
C

Z/a
Consider two infinitely long wires parallel to
OM
dy

Z-axis carrying same current I in the positive


z-direction. One wire passes through the
point L at coordinates (− 1, + 1) and the other
u

wire passes through the point M at


The variation of electric field along the Z-axis coordinates (− 1, − 1). The resultant magnetic
St

due to a uniformly charged circular ring of field at the origin O will be


radius a in XY -plane as shown in the figure. µ 0I $j µ 0I $j
The value of coordinate M will be (a) (b)
2 2π 2π
(a)
1
(b) 2 (c) 1 (d)
1 µ 0I $i µ 0I $i
(c) (d)
2 2 2 2π 4π
21. A metal sphere of radius R carrying charge q 25. A thin charged rod is bent into the shape of a
is surrounded by a thick concentric metal small circle of radius R, the charge per unit
shell of inner and outer radii a and b, length of the rod being λ. The circle is rotated
respectively. The net charge on the shell is about its axis with a time period T and it is
zero.The potential at the centre of the sphere, found the magnetic field at a distance d away
when the outer surface of the shell is (d >> R) from the centre and on the axis,
grounded will be
4 WB JEE (Engineering) Solved Paper 2021

varies as R m / d n. The values of m and n 30. Y


respectively are
(a) m = 2, n = 2 (b) m = 2, n = 3 R
(c) m = 3, n = 2 (d) m = 3, n = 3 X

26. 1/χ
A

)
B The cross-section of a reflecting surface is

be
T represented by the equation x 2 + y 2 = R 2 as
For two types of magnetic materials A and B, shown in the figure. A ray travelling in the
variation of 1/ χ (χ : susceptibility) versus positive x-direction is directed toward

Tu
temperature T is shown in the figure. Then, positive y-direction after reflection from the
surface at point M. The coordinate of the
(a) A is diamagnetic and B is paramagnetic
point M on the reflecting surface is

ou
(b) A is ferromagnetic and B diamagnetic
(a) 
R R 
(b)  − ,− 
R R
(c) A is paramagnetic and B is ferromagnetic , 
(d) A is paramagnetic and B is diamagnetic  2 2  2 2

(Y (c)  −
R R 
(d) 
R R 
27. V ,  ,− 
 2 2  2 2
on
V0 Category-II (Q. 31 to 35)
t Carry 2 marks each and only one option is
0 T/2 T correct. In case of incorrect answer or any
pi

The rms value of potential difference V combination of more than one answer, 1/2 mark
will be deducted.
m

shown in the figure is


V0 V0 V0
(a) (b) V0 (c) (d) 31. For a plane electromagnetic wave, the electric
ha

2 3 2
field is given by
28. . × 10 11 t) k$ V/m. The
E = 90 sin(0 .5 × 10 3 x + 15
C

corresponding magnetic field B will be


(a) B = 3 × 10−7 sin(0.5 × 103 x + 15
. × 1011t ) $i T
−7
. × 1011t ) $j T
dy

Red Yellow Orange (b) B = 3 × 10 sin(0.5 × 10 x + 15


3

The carbon resistor with colour code is (c) B = 27 × 109 sin(0.5 × 103 x + 15. × 1011t ) $j T
shown in the figure. There is no fourth band
(d) B = 3 × 10−7 sin(0.5 × 103 x + 15
u

in the resistor. The value of the resistance is . × 1011t ) k$ T


St

(a) 24 M Ω ± 20% (b) 14 kΩ ± 5% 32. Two metal wires of identical dimensions are
(c) 24 k Ω ± 20% (d) 34 k Ω ± 10%
connected in series. If σ1 and σ 2 are the
29. L electrical conductivities of the metal wires
respectively, the effective conductivity of the
combination is
σ1σ 2 2σ1σ 2 σ + σ2
(a) σ1 + σ 2 (b) (c) (d) 1
~
V=V0 sin ωt
σ1 + σ 2 σ1 + σ 2 2 σ1σ 2

Consider a pure inductive AC circuit as 33. A uniform rod of length L pivoted at one end
shown in the figure. If the average power P is freely rotated in a horizontal plane with
consumed is P, then an angular velocity ω about a vertical axis
(a) P > 0 (b) P < 0 passing through P. If the temperature of the
(c) P = 0 (d) P is infinite system is increased by ∆T, angular velocity
WB JEE (Engineering) Solved Paper 2021 5

ω (a) Magnetic field at the centre O of the circular coil


becomes . If coefficient of linear expansion M
2 due to the bar magnet is 3 .
of the rod is α(α << 1),then ∆T will be x
1
1 1 1 (b) Induced emf is proportional to 4 .
(a) (b) (c) (d) α x
α 2α 4α
(c) The magnetic moment µ due to induced current
34. An ideal gas of molar mass M is contained in in the coil is proportional to a4 .
1
a very tall vertical cylindrical column in the (d) The heat produced is proportional to 6 .

)
uniform gravitational field. Assuming the gas x

be
temperature to be T, the height at which the 37. Electric field component of an EM radiation
centre of gravity of the gas is located is varies with time as E = a (cos ω 0 t + sin ωt
(R→universal gas constant) cos ω 0 t), where a is a constant and ω = 10 15 s −1 ,

Tu
RT RT
(a)
g
(b)
Mg ω 0 = 5 × 10 15 s −1 . This radiation falls on a
(c) MgR (d) RTg metal whose stopping potential is − 2 eV.

ou
Then, which of the following statement(s)
35. Under isothermal conditions, two soap is/are true? (h = 6 .62 × 10 −34 J-s)
bubbles of radii a and b coalesce to form a

(Y
(a) For light of frequency ω , photoelectric effect is
single bubble of radius c. If the external
not possible.
pressure is p, then surface tension of the
(b) Stopping potential versus frequency graph will
bubbles is be a straight line.
on
p(c 3 − a3 + b 3 ) p(c 3 − a3 − b 3 ) (c) The work function of the mutual is − 2 eV.
(a) (b)
4(a + b − c )
2 2 2
4(a2 + b 2 − c 2 ) (d) The maximum kinetic energy of the
p(c 2 + a3 − b 2 ) p(a3 + b 3 − c 3 ) photoelectrons is − 2 eV.
pi

(c) (d)
4(a + b − c )
3 3 3
4(a2 + b 2 − c 2 ) 38. p
m

2p0 C
Category-III (Q. Nos. 36 to 40)
ha

Carry 2 marks each and one or more option(s) p0 A


is/are correct. If all correct answers are not marked B
and also no incorrect answer is marked, then score
C

V
= 2 × number of correct answers marked ÷ actual V0 2V0
number of correct answers. If any wrong option is
dy

marked or if any combination including a wrong Consider the p - V diagram for 1 mole of an
option is marked, the answer will be considered ideal monatomic gas shown in the figure.
wrong, but there is no negative marking for the Which of the following statement(s) is/are
u

same and zero marks will be awarded. true?


St

(a) The change in internal energy for the whole


36. process is zero.
x a (b) Heat is rejected during the process.
O (c) Change in internal energy for process A → B is
v 3
− p0 V0 .
A small bar magnet of dipole moment M is 2
moving with speed v along x-direction (d) Work done by the gas during the entire process
is 2 p0 V0 .
towards a small closed circular conducting
loop of radius a with its centre O at x = 0 (see 39. The potential energy of a particle of mass
figure). Assume x >> a and the coil has a 0.02 kg moving along X -axis is given by
resistance R. Then, which of the following V = Ax (x − 4) J, where x is in metre and A is a
statement(s) is/are true? constant. Which of the following
statement(s) is/are correct?
6 WB JEE (Engineering) Solved Paper 2021

(a) The particle is acted upon by a constant A particle of mass m and charge q moving with
force. velocity v enters region-b from region-a along the
(b) The particle executes simple harmonic normal to the boundary as shown in the figure.
motion.
Region-b has a uniform magnetic field B
(c) The speed of the particle is maximum at
x = 2 m.
perpendicular to the plane of the paper. Also,
π region-b has length L.
(d) The period of oscillation of the particle is s.
5
Choose the correct statement.

)
b qLB
40. (a) The particle enters region- c only if v >

be
a .
c m
× × ×
qLB
(b) The particle enters region- c only if v < .
× × × m

Tu
v
× × × (c) Path of the particle is a circle in region-b.
L (d) Time spent in region-b is independent of velocity v.

ou
Chemistry
Category-I (Q. Nos. 41 to 70) (Y
44. p-nitro–N, N–dimethylaniline cannot be
on
Carry 1 mark each and only one option is correct. represented by the resonating structures.
In case of incorrect answer or any combination of O O

more than one answer, 1/4 mark will be deducted.
pi

Me2N N Me2N N
O O
41. The exact order of boiling points of the
m

(I) (II)
compounds n-pentane, isopentane, butanone
and 1-butanol is
ha

⊕ ⊕ O ⊕ O
(a) n-pentane < isopentane < butanone <
Me2N N Me2N N
1-butanol O
O
(b) isopentane < n-pentane < butanone <
C

1-butanol (III) (IV)


(c) butanone < n -pentane < isopentane < (a) I and II (b) II and IV
dy

1-butanol (c) I and III (d) III and IV


(d) 1-butanol < butanone < n-pentane <
isopentane 45. CO2H CH3
u

42. The maximum number of atoms that can be 1. H OH and HO H


St

in one plane in the molecule


p-nitrobenzonitrile are
CH3 COOH
(a) 6 (b) 12
(c) 13 (d) 15 CH3 CH3
43. Cyclo [18] carbon is an allotrope of carbon
with molecular formula C18 . It is a ring of 18 and
2.
carbon atoms, connected by single and triple
bonds. The total number of triple bonds NO2 NO2
present in this cyclocarbon are
(a) 9 (b) 10 H Cl H 3C Cl
3. and
(c) 12 (d) 6
CH3 Cl Cl H
WB JEE (Engineering) Solved Paper 2021 7

The relationship between the pair of The compounds A and B above are
compounds shown above are respectively respectively.
(a) homomer (identical), enantiomer and Ph OMe CO2H
constitutional isomer (a) and Ph
(b) enantiomer, enantiomer and diastereomer
(c) homomer (identical), homomer (identical) and Cl
constitutional isomer Ph OMe
(d) enantiomer, homomer (identical) and (b) and PhCOCH

)
3
geometrical isomer

be
Cl
46. The exact order of acidity of the compounds OH
p-nitrophenol, acetic acid, acetylene and Ph OMe CO2H

Tu
ethanol is (c) and Ph
(a) p-nitrophenol < acetic acid < acetylene < Cl
ethanol

ou
(b) acetic acid < p-nitrophenol < acetylene < Ph OMe CO2H
ethanol and Ph
(d)
(c) acetylene < p-nitrophenol < ethanol < acetic OMe

(Y
acid
(d) acetylene < ethanol < p-nitrophenol < acetic 49. For a spontaneous reaction at all temperatures
acid which of the following is correct?
on
47. NH2 H (a) Both ∆H and ∆S are positive
(b) ∆H is positive and ∆S is negative
N CO2H
1. Me (c) ∆H is negative and ∆S is positive
pi

(d) Both ∆H and ∆S are negative


O
50. A given amount of Fe2+ is oxidised by x mol of
m

NH2 H
MnO −4 in acidic medium. The number of moles
N CO2H
ha

2. Me of Cr2O72− required to oxidise the same amount


O
CH3 of Fe2+ in acidic medium is
(a) x (b) 0.83 x
C

H
(c) 2.0 x (d) 1.2 x
N CO2H
51. An element crystallises in a body centered
dy

3. H2N
cubic lattice. The edge length of the unit cell is
O Me
200 pm and the density of the element is 5.0 g
cm −3 . Calculate the number of atoms in 100 g
u

H
of this element.
St

N CO2H
4. H2N (a) 2.5 × 1023 (b) 2.5 × 1024
O (c) 5.0 × 10 23
(d) 5.0 × 1024

The dipeptides which may be obtained from 52. Molecular velocities of two gases at the same
the amino acids glycine and alanine are temperature (T) are u1 and u2 . Their masses are
(a) Only 1 (b) Only 2 m1 and m2 respectively. Which of the following
(c) Both 1 and 2 (d) All of them expressions is correct at temperature T?
m1 m2
48. Benzaldehyde + methanol (a) = (b) m1u1 = m2u 2
u12u 22
1. dil. HCl m1 m2
dry
→ A 2  → B (c) = (d) m1u12 = m2u 22
HCl .( CH CO) O, 3 2 u1 u2
CH 3 COONa
8 WB JEE (Engineering) Solved Paper 2021

53. When 20 g of naphthoic acid (C11H 8O 2) is 59. Solubility products (K sp) of the salts of types
dissolved in 50 g of benzene, a freezing point MX , MX 2 and M3 X at temperature T are
depression of 2K is observed. The van’t Hoff 4 .0 × 10 −8 , 3.2 × 10 −14 and 2.7 × 10 −15
respectively. Solubilities (in mol dm −3 ) of the
. K kg mol −1 ]
factor (i) is [K f = 172
salts at temperature T are in the order.
(a) 0.5 (b) 1.0 (c) 2.0 (d) 3.0 (a) MX > MX 2 > M 3 X (b) M 3 X > MX 2 > MX
(c) MX 2 > M 3 X > MX (d) MX > M 3 X > MX 2
54. The equilibrium constant for the reaction
2NO( g ) is 4 × 10 −4 at

)
N 2( g )+ O 2( g ) c 60. The reduction potential of hydrogen half-cell

be
2000 K. In presence of a catalyst the will be negative if
equilibrium is attained 10 times faster. (a) p(H2 ) = 1 atm and [H+ ] = 10
. M
Therefore, the equilibrium constant, in (b) p(H2 ) =1atm and [H+ ] = 2.0 M

Tu
presence of the catalyst at 2000 K is (c) p(H2 ) =2 atm and [H+ ] = 10
. M
(d) p(H2 ) =2 atm and [H+ ] = 2.0 M
(a) 4 × 10−4 (b) 4 × 10−3
−5
(d) 2.5 × 10−4

ou
(c) 4 × 10 61. A saturated solution of BaSO 4 at 25°C is
4 × 10 −5 M. The solubility of BaSO 4 in 0.1 M
55. Under the same reaction conditions, initial
concentration of 1.386 mol dm −3 of a Na 2SO 4 at this temperature will be
substance becomes half in 40 s and 20 s
through first-order and zero-order kinetics
k 
(Y . × 10−9 M
(a) 16
(c) 4 × 10 −6
M
. × 10−8 M
(b) 16
(d) 4 × 10−4 M
on
respectively. Ratio  1  of the rate constants 62. A solution is made by a concentrated solution
 k0 
of Co(NO 3 )2 with a concentrated solution of
for first-order (k1) and zero-order (k0) of the NaNO 2 is 50% acetic acid. A solution of a salt
pi

reactions is containing metal M is added to the mixture,


(a) 0.5 mol −1 dm 3 (b) 0.5 mol dm −3 when a yellow precipitate is formed. Metal ‘
m

(c) 1.0 mol dm −3 (d) 2.0 mol −1 dm 3 M’ is


56. Which of the following solutions will have (a) magnesium (b) sodium
ha

(c) potassium (d) zinc


highest conductivity?
(a) 0.1 M CH3COOH (b) 0.1 M NaCl 63. Extraction of a metal (M) from its sulphide
C

(c) 0.1 M KNO 3 (d) 0.1 M HCl ore (M2S) involves the following chemical
57. Indicate the products (X ) and (Y ) in the reactions
2 M2S + 3O 2 Heat

→ 2 M2O + 2SO 2 ↑
dy

following reactions
Na 2S + nS( n = 1 − 8 ) → (X ) M2S + 2 M2O Heat
→ 6 M + SO 2 ↑
Na 2SO 3 + S → (Y )
u

(a) Zn (b) Cu (c) Fe (d) Ca


St

(X) (Y) 64. The white precipitate (Y ), obtained on


(a) Na 2S2O 3 Na 2S2 passing colourless and odourless gas (X )
(b) Na 2S( n + 1) Na 2S2O 3 through an ammoniacal solution of NaCl,
(c) Na 2Sn Na 2S2O 3 loses about 37% of its weight on heating and
(d) Na 2S5 Na 2S2O 4 a white residue (Z) of basic nature is left.
Identify (X ), (Y ) and (Z) from following sets.
58. 2.5 mL 0.4 M weak monoacidic base
(K b = 1 × 10 −12 at 25°C) is titrated with 2/15 M (X) (Y) (Z)
HCl in water at 25°C. The concentration of (a) N2 (NH4 )2 CO 3 NH4Cl
H + at equivalence point is (K w = 1 × 10 −14 , at (b) O2 NaNH4CO 3 NaHCO 3
25°C)
(c) CO 2 NH4HCO 3 (NH4 )2 CO 3
. × 10−13 M
(a) 37 . × 10−7 M
(b) 32
(d) CO 2 NaHCO 3 Na 2CO 3
. × 10−2 M
(c) 32 (d) 2.7 × 10−2 M
WB JEE (Engineering) Solved Paper 2021 9

65. Which structure has delocalised π-electrons? 1. Methoxymethyl chloride


(a) O 3 (b) CO 2. Benzyl chloride
(c) HCN (d) O 3 and HCN 3. Neopentyl chloride
66. The H 3O+ ions has the following shape 4. Propyl chloride
(a) 1 and 3 (b) 2 and 3
(a) tetrahedral (b) pyramidal
(c) 2 and 4 (d) 3 and 1
(c) triangular planar (d) “T” shaped
72. The products X and Y which are formed in

)
67. For the reaction 14 N(α , p) 17O, 1.16 MeV

be
the following sequence of reactions are
(Mass equivalent = 0 .00124 amu) of energy is respectively.
absorbed. Mass on the reactant side is
1. Zn / HCl, ∆
18.00567 amu and proton mass = 100782
. dil. HNO 3

Tu
Phenol → X 2.(CH
    → Y
amu. The atomic mass of 17O will be CO) O(1 equiv.)
3 2

(a) 17.0044 amu (b) 16.9991 amu OH OH

ou
(c) 17.0114 amu (d) 16.9966 amu COCH3

68. A solution of NaNO 3, when treated with a (a) and


mixture of Zn dust and ‘A’ yields ammonia. ‘
A’ can be
(a) caustic soda
(Y NO NH2
on
(b) dilute sulphuric acid OH OCOCH3
(c) concentrated sulphuric acid
(d) sodium carbonate
pi

(b) and
69. Indicate the number of unpaired electrons in
m

K 3[Fe(CN)6] and K 4[Fe(CN)6].


NO2 NH2
K 3[Fe(CN)6 ] K 4[Fe(CN)6 ]
ha

(a) 1 0 OH OH

(b) 5 6
C

(c) 6 5 (c) and


(d) 0 1
dy

NO2 NHCOCH3
70. Which of the following compounds have
magnetic moment identical with OH OH
u

[Cr(H 2O)6]3+ ?
St

(a) [Cu(H2O)6 ]2+ (b) [Mn(H2O)6 ]3+


(d) and
(c) [Fe(H2O)6 ]3+ (d) [Mn(H2O)6 ]3 +
COCH3
Category-II (Q. Nos. 71 to 75)
NO NH2
Carry 2 marks each and only one option is
correct. In case of incorrect answer or any 73. The atomic masses of helium and neon are
combination of more than one answer, 1/2 mark 4.0 and 20.0 amu respectively. The value of
will be deducted. the de-Broglie wavelength of helium gas at
− 73 °C is M times the de-Broglie wavelength
71. Among the following chlorides the of neon at 727°C. The value of M is
compounds which will be hydrolysed most (a) 5 (b) 25
easily and most slowly in aqueous NaOH 1 1
(c) (d)
solution are respectively: 5 25
10 WB JEE (Engineering) Solved Paper 2021

74. The mole fraction of a solute in a binary 77. The compounds X and Y are respectively
solution is 0.1 at 298 K, molarity of this 1. Mg, ether
solution is same as its molality. Density Br
2. Acetaldehyde
CH3 X
of this solution at 298 K is 2.0 g cm −3 . 3. Br2 / NaOH
The ratio of molecular weights of the 4. H2O+
solute and the solvent (Msolute / Msolvent) is 1. SOCl2
1 1 2. NH3
(a) 9 (b) (c) 4.5 (d) Y
9 4.5

)
3. Br2 / NaOH

be
75. 5.75 mg of sodium vapour is converted CH3 CH3
to sodium ion. If the ionisation energy of (a) H3C and H3C
OH NH2
sodium is 490 kJ mol −1 and atomic

Tu
weight is 23 units, the amount of energy
CH3
needed for this conversion will be (b) H3C NH2
and H3C

ou
(a) 1.96 kJ (b) 1960 kJ O
(c) 122.5 kJ (d) 0.1225 kJ
CH3

(Y
Category-III (Q.Nos. 76 to 80) (c) H3C and H3C CONH2
OH
Carry 2 marks each and one or more
option(s) is/are correct. If all correct answers
on
are not marked and no incorrect answer is (d) H3C CO2H and H3C NH2
marked, then score = 2 × number of correct
answers marked ÷ actual number of correct 78. Aqueous solution of HNO 3, KOH, CH 3COOH and
pi

answers. If any wrong option is marked or if CH 3COONa of identical concentration are


any combination including a wrong option is provided. The pair(s) of solutions which form a
m

marked, the answer will be considered buffer upon mixing is (are)


wrong, but there is no negative marking for (a) HNO 3 and CH3COOH
ha

the same and zero mark will be awarded. (b) KOH and CH3COONa
(c) HNO 3 and CH3COONa
76. The product(s) in the following sequence
C

(d) CH3COOH and CH3COONa


of reactions will be
1. Na/NH 3 (liq.) 79. Reaction of silver nitrate solution with
ethanol, − 33° C
dy

Me  C ≡≡ C  Me 2  → phosphorus acid produces


. dil.alkaline KMnO 4 (a) silver phosphite
Product(s) (b) phosphoric acid
u

Me Me (c) metallic silver


St

H OH H OH (d) silver phosphate


(a) (b)
H OH HO H
80. N 2H 4 and H 2O 2 show similarity in
Me Me
(a) density
Me Me (b) reducing nature
HO H HO H (c) oxidising nature
(c) H OH (d) HO H (d) hybridisation of central atoms
Me Me
WB JEE (Engineering) Solved Paper 2021 11

Mathematics
Category-I (Q. Nos. 1 to 30) 6. The value of the integral
1
  x + 1 2  x − 1 2  2
1
Carry 1 marks each and only one option is 2

correct. In case of incorrect answer or any ∫   x −1  x + 1  dx is equal to


+ − 2
combination of more than one answer, 1/4 mark −1 2  

)
will be deducted.
(a) loge   (b) 4loge  

be
4 3
 3  4
 x x 2
e − x −1−  (c) 4loge   (d) loge  
4 3

Tu
1. If I = lim sin  2  , then limit  3  4
x→0  x2 
  x
  3
7. If ∫ (e x − 1)−1 dx = log e then the value of x

ou
(a) does not exist log e 2
2
(b) exists and equals 1 is
(c) exists and equals 0 1

(Y
1 (a) 1 (b) e 2 (c) log 4 (d)
(d) exists and equals e
2
8. The normal to a curve at P(x , y) meets the
2. Let f : R → R be such that f (0) = 0 and
on
X -axis at G. If the distance of G from the
| f ′ (x)| ≤ 5 for all x. Then f ()
1 is in
origin is twice the abscissa of P then the
(a) (5, 6) (b) [− 5, 5] curve is
pi

(c) (− ∞, − 5) ∪ (5, ∞ ) (d) [− 4, 4]


(a) a parabola (b) a circle
sin 2 x
3. If ∫ (c) a hyperbola (d) an ellipse
m

dx
(a + bcos x)2
9. The differential equation of all the ellipses
ha

 a  centred at the origin and have axes as the


= α log e|a + bcos x| + + c,
a + bcos x 
co-ordinate axes is

(a) y2 + xy′2 − yy′ = 0
then α is equal to
C

(b) xyy′ ′ + xy′2 − yy′ = 0


2 2
(a) (b) (c) yy′ + xy′2 − xy′ = 0
b2 a2
dy

2 2 (d) x2 y′ + xy′ ′ − 3 y = 0
(c) − 2
(d) − 2
b a dy d2y
where y′ ≡ , y′ ′ ≡ 2
u

2x dx dx
f ()t
4. Let g(x) = ∫ dt where x > 0 and f be
St

t dy xf (xy)
x 10. Ifx + y= , then| f (xy)| is equal to
continuous function and f (2 x) = f (x), then dx f ′ (xy)
2 2
(a) g ( x) is strictly increasing function (a) ke x /2
(b) ke y /2

(b) g ( x) is strictly decreasing function x2 y2


(c) ke (d) ke
(c) g ( x) is constant function
(d) g ( x) is not derivable function 11. The straight the through the origin which
3 divides the area formed by the curves
| x − 1|
5. ∫ | x − 2| + | x − 3| dx is equal to y = 2 x − x 2 , y = 0 and x = 1 into two equal
1 halves is
4 3 (a) y = x (b) y = 2 x
(a) 1 + loge 3 (b) 1+ loge 3
3 4 3 2
(c) y = x (d) y = x
4 3
(c) 1 − loge 3 (d) 1 − loge 3 2 3
3 4
12 WB JEE (Engineering) Solved Paper 2021

5
18. Consider the real valued function h : {0, 1, 2,
12. The value of ∫ max{ x 2 , 6 x − 8} dx is
0
..... 100} → R such that h(0) = 5, h(100) = 20
1
(a) 72 (b) 125 and satisfying h(p) = { h(p +1) + h(p – 1)} for
(c) 43 (d) 69 2
every p = 1, 2 ……99. Then the value of h() 1 is
13. A bulb is placed at the centre of a circular (a) 5.15 (b) 5.5
track of radius 10 m. A vertical wall is erected (c) 6 (d) 6.15

)
touching the track at a point P. A man is
19. If|z| = 1 and z ≠ ± 1, then all the points

be
running along the track with a speed of 10
m/sec. Starting from P the speed with which z
representing lie on
his shadow is running along the wall when 1 − z2

Tu
he is at an angular distance of 60° from P is
(a) a line not passing through the origin
(a) 30 m/sec (b) 40 m/sec
(b) the line y = x
(c) 60 m/sec (d) 80 m/sec

ou
(c) the X -axis
14. Two particles A and B move from rest along a (d) the Y-axis
straight line with constant accelerations f

(Y
and f ′ respectively. If A takes m sec. more 20. Let C denote the set of all complex numbers.
than that of B and describes n units more Define A = {(z , w)| z , w ∈ C and|z| = |w|},
than that of B in acquiring the same velocity,
B = { z , w)|z , w ∈ C and z 2 = w 2 }. Then
then
on
(a) (f + f ′ ) m2 = ff ′ n (b) (f − ff ′ ) m2 = ff ′ n (a) A = B (b) A ⊂ B
1 1 (c) B ⊂ A (d) A ∩ B = ϕ
(c) (f ′ − f ) n = ff ′ m2 (d) (f + f ′ ) m = ff ′ n2
pi

2 2 21. Let α , β be the roots of the equation


15. Let α , β , γ be three non-zero vectors which x 2 − 6 x − 2 = 0 with α > β. Ifa n = α n − β n for
m

are pairwise non-collinear. If α + 3β is a − 2a 8


n ≥ 1, then the value of 10 is
collinear with γ and β + 2 γ is collinear with α, 2a 0
ha

then α + 3β + 6 γ is (a) 1 (b) 2


(a) γ (b) 0 (c) 3 (d) 4
C

(c) α + γ (d) α
22. For x ∈ R , x ≠ − 1, if
16. Let f :R → R be given by f (x) = | x 2 − 1|, x ∈ R.
(1 + x)2016 + x (1 + x)2015 + x 2 (1 + x)2014
dy

Then 2016
(a) f has a local minimum at x = ± 1but no local + .... + x 2016 = Σ a i . x i , then a17 is equal to
i =0
maximum.
u

(b) f has a local minimum at x = 0 but no local 2016! 2016!


(a) (b)
St

minimum. 17 !1999! 16!


(c) f has a local minima at x = ± 1and a local 2017 ! 2017 !
(c) (d)
maxima at x = 0. 2000! 17 !2000!
(d) f has neither a local maxima nor a local minima
at any point. 23. Five letter words, having distinct letters, are
17. Let a , b, c be real numbers, each greater than to be constructed using the letters of the
2 3 5 word ‘EQUATION’ so that each word
1, such that log b a + log c b + log a c = 3. contains exactly three vowels and two
3 5 2
consonants. How many of them have all the
If the value of b is 9, then the value of ‘a’
vowels together?
must be
27 (a) 3600 (b) 1800
(a) 3 81 (b) (c) 18 (d) 27
2 (c) 1080 (d) 900
WB JEE (Engineering) Solved Paper 2021 13

24. What is the number of ways in which an 31. Let T and U be the set of all orthogonal
examiner can assign 10 marks to 4 questions, matrices of order 3 over R and the set of all
giving not less than 2 marks to any question? non-singular matrices of order 3 over R
(a) 4 (b) 6 respectively. Let A = { − 1, 0 , 1}, then
(c) 10 (d) 16 (a) there exists bijective mapping between A and T,
U.
25. The digit in the unit’s place of the number (b) there does not exist bijective mapping between
1! + 2 ! + 3 ! + .... + 99 ! is A and T, U.

)
(a) 3 (b) 0 (c) there exists bijective mapping between A and T

be
(c) 1 (d) 7 but not between A and U.
(d) there exists bijective mapping between A and U
26. If M is a 3 × 3 matrix such that (0, 1, 2) but not between A and T.

Tu
M = (1 0 0), (3, 4 5) M = (0, 1, 0), then
(6 7 8) M is equal to 32. Four persons A , B, C and D throw and
(a) (2 1 −2 ) (b) (0 0 1) unbiased die, turn by turn, in succession till

ou
(c) (−1 2 0) (d) (9 10 8) one gets an even number and win the game.
What is the probability that A wins the game
1 0 0  if A begins?
 
27. Let A = 0 cos t sin t
0 − sin t cos t
  (Y (a)
1
4
(b)
1
2
(c)
7
15
(d)
8
15
on
Let λ 1 , λ 2 , λ 3 be the roots of det(A − λI 3) = 0, 33. The mean and variance of a binomial
where I 3 denotes the identity matrix. If distribution are 4 and 2 respectively. Then
the probability of exactly two successes is
λ 1 + λ 2 + λ 3 = 2 + 1, then the set of possible
pi

7 21 7 9
values of t, − π ≤ t < π is (a)
64
(b)
128
(c)
32
(d)
32
π
(b)  
m

(a) a void set


4
π π
(c) − , 
π π
(d) − , 
34. Let Sn = cot −1 2 + cot −1 8 + cot −118
ha

 4 4  3 3
+ cot −1 32 + ... to nth term. Then lim Sn is
n→ ∞
28. Let A and B two non singular skew
C

π π π π
symmetric matrices such that AB = BA, then (a) (b) (c) (d)
A 2 B2 (A T B)−1 (AB−1)T is equal to 3 4 6 8
dy

(a) A 2 (b) − B2 35. If a > 0 , b > 0 then the maximum area of the
(c) − A 2
(d) AB parallelogram whose three vertices are O(0 , 0),
A(a cos θ , bsin θ) and B(a cos θ , − bsin θ) is
u

29. If a n(> 0) be the nth term of a G.P. then (a) ab when θ =


π
(b) 3ab when θ =
π
St

4 4
log a n log a n + 1 log a n + 2 π
(c) ab when θ = − (d) 2ab
log a n + 3 log a n + 4 log a n + 5 is equal to 2
log a n + 6 log a n + 7 log a n + 8 36. Let A be the fixed point (0, 4) and B be a
(a) 1 (b) 2 (c) −2 (d) 0 moving point on X -axis. Let M be the
midpoint of AB and let the perpendicular
30. Let A , B, C be three non-void subsets of set S. bisector of AB meets the Y -axis at R. The
Let (A ∩ C) ∪ (B ∩ C ′) = φ where C ′ denote the locus of the midpoint P of MR is
complement of set C in S. Then 1
(a) y + x2 = 2 (b) x2 + ( y − 2 )2 =
(a) A ∩ B = φ (b) A ∩ B ≠ φ 4
1
(c) A ∩C = A (d) A ∪C = A (c) ( y − 2 )2 − x2 = (d) x2 + y2 = 16
4
14 WB JEE (Engineering) Solved Paper 2021

37. A moving line intersects the lines x + y = 0 44. The locus of the centre of a variable circle
and x − y = 0 at the points A , B respectively which always touches two given circles
such that the area of the triangle with externally is
vertices (0,0 ), A and B has a constant area C. (a) an ellipse (b) a hyperbola
The locus of the mid-point AB is given by the (c) a parabola (d) a circle
equation
45. A line with positive direction cosines passes
(a) ( x2 + y2 )2 = C 2 (b) ( x2 − y2 )2 = C 2
through the point P(2 , −1, 2) and makes equal

)
(c) ( x + y)2 = C 2 (d) ( x − y)2 = C 2 angle with co-ordinate axes. The line meets

be
the plane 2 x + y + z = 9 at point Q. The length
38. The locus of the vertices of the family of of the line segment PQ equals.
parabolas 6 y = 2 a 3 x 2 + 3 a 2 x − 12 a is

Tu
(a) 1 unit (b) 2 unit
105 64 (c) 3 unit (d) 2 unit
(a) xy = (b) xy =
64 105
35 16  5 x + 12 1 − x 2 
(c) xy = (d) xy = 46. For y = sin −1 

ou
16 35  ;| x| ≤ 1, if
 13 
39. A ray of light along x + 3 y = 3 gets a(1 − x ) y 2 + bxy1 = 0 then (a , b) =
2

reflected upon reaching X -axis, the equation


of the reflected ray is
(a) y = x + 3 (b) 3 y = x − 3
(Y (a) (2, 1)
(c) (− 1, 1)
(b) (1, − 1)
(d) (1, 2)
on
(c) y = 3 x − 3 (d) 3 y = x − 1 47. f (x) is real valued function such that
2 f (x) + 3 f (− x) = 15 − 4 x for all x ∈ R. Then
40. Two tangents to the circle x 2 + y 2 = 4 at the
f (2) =
pi

points A and B meet at M(− 4 , 0). The area of


(a) −15 (b) 22 (c) 11 (d) 0
the quadrilateral MAOB, where O is the origin
m

is 48. Consider the functions f1(x) = x ,


(a) 4 3 sq units (b) 2 3 sq units f 2 (x) = 2 + log e x , x > 0 . The graphs of the
ha

(c) 3 sq units (d) 3 3 sq units functions intersect


41. From a point (d , 0) three normals are drawn (a) once in (0, 1) but never in (1, ∞ )
(b) once in (0, 1) and once in (e 2 , ∞ )
C

to the parabola y = x , then


2
(c) once in (0, 1) and once in (e, e 2 )
1 1
(a) d = (b) d > (d) more than twice in (0, ∞ )
dy

2 2
1 1 49. The equation 6 x + 8 x = 10 x has
(c) d < (d) d =
2 3
u

(a) no real root.


42. If from a point P(a , b, c), perpendicular PA and (b) infinitely many rational roots
St

PB are drawn to YZ and ZX -planes (c) exactly one real root


respectively, then the equation of the plane (d) two distinct real roots
OAB is 50. Let f : D → R where D = [ −0 , 1] ∪ [2, 4] be
(a) bcx + cay + abz = 0 (b) bcx + cay − abz = 0 defined by
(c) bcx − cay + abz = 0 (d) bcx − cay − abz = 0
 x, if x ∈[0 , 1]
43. The co-ordinate of a point on the auxiliary f (x) =  Then,
circle of the ellipse x 2 + 2 y 2 = 4  4 − x , if x ∈[2 , 4 ]
(a) Rolle’s theorem is applicable to f in D.
corresponding to the point on the ellipse
(b) Rolle’s theorem is not applicable to f in D.
whose eccentric angle is 60° will be
(c) there exists ξ ∈ D for which f′(ξ) = 0 but Rolle’s
(a) ( 3, 1) (b) (1, 3 ) theorem is not applicable.
(c) (1, 1) (d) (1, 2) (d) f is not continuous in D.
WB JEE (Engineering) Solved Paper 2021 15

Category-II (Q.Nos. 51 to 65) 58. If the tangent at the point P with


Carry 2 marks each and only one option is co-ordinates (h, k) on the curve y 2 = 2 x 3 is
correct. In case of incorrect answer or any perpendicular to the straight line 4 x = 3 y,
combination of more than one answer, 1/2 mark
will be deducted. then
(a) (h, k ) = (0, 0) only
(b) (h, k ) =  , −  only
51. Let f (x) be continuous periodic function with 1 1
a+T  8 16 

)
period T. Let I = ∫ f (x) dx. Then (c) (h, k ) = (0, 0) or  , 
1 1

be
a  8 16 
(a) I is linear function in ‘a ’ (d) no such point P exists
(b) I does not depend on ‘a’

Tu
59. The co-efficient of a 3b4c 5 in the expansion of
(c) 0 < I < a2 + 1 where I depends on ‘a’
(bc + ca + ab)6 is
(d) I is quadratic function in ‘a’
12 ! 6!

ou
(a) (b)
e− t
1 a
et 3! 4! 5! 3!
52. If b = ∫ dt, then ∫ is
(d) 3. 
t +1 t − a −1 6! 
a −1 (c) 33 
0
 3! 3!
(a) be a (b) be − a (c) − be − a (d) − be a

53. The differential of f (x) = log e (1 + e10x ) − tan −1 (Y


60. Three unequal positive numbers a , b, c are
such that a , b, c are in G.P. while
on
(e 5x ) at x = 0 and for dx = 0.2 is  5c   7 b  2a 
log   , log   , log   are in A.P. Then
(a) 0.5 (b) 0.3 (c) − 02
. (d) − 0.5  2a   5c   7 b
a , b, c are the lengths of the sides of
pi

54. Given that f :S → R is said to have a fixed


(a) an isosceles triangle
point at c of S if f (c) = c.
m

(b) an equilateral triangle


Let f : [1, ∞) → R be defined by f (x) = 1 + x. (c) a scalene triangle
Then (d) a right-angled triangle
ha

(a) f has no fixed point in [1, ∞ )


(b) f has unique fixed point in [1, ∞ )
61. The determinant
(c) f has to fixed points in [1, ∞ ) a2 + 10 ab ac
C

(d) f has infinitely many fixed points in [1, ∞ ) ab b 2 + 10 bc is


4x ac bc c 2 + 10
 3 x − 1
dy

55. The lim   equals


x → ∞  3 x + 1 (a) divisible by 10 but not by 100
(b) divisible by 100
(c) e −8 / 3 (d) e −4 / 9
u

(a) 1 (b) 0 (c) not divisible by 100


(d) not divisible by 10
St

56. The area bounded by the parabolas


x2 62. Let R be the real line. Let the relations S and T
y = 4 x2, y = and the straight line y = 2 is
9 or R be defined by
20 2 S = {( x, y) : y = x + 1, 0 < x < 2}, T = {( x, y) : x − y is
(a) sq. unit (b) 10 5 sq. unit an integer}. Then
3
10 3 (a) both S and T are equivalence relations on R
(c) sq. unit (d) 10 2 sq. unit (b) T is an equivalence on R but S is not
7
(c) neither S nor T is an equivalence relation on R
57. If a (α × β) + b(β × γ) + c(γ × α) = o, where (d) S is an equivalence relation on R but T is not
a , b, c are non-zero scalars, then the vectors 63. The plane lx + my = 0 is rotated about its line
α , β , γ are
of intersection with the plane z = 0 through
(a) parallel (b) non-coplanar
(c) coplanar (d) mutually perpendicular an angle α. The equation changes to
16 WB JEE (Engineering) Solved Paper 2021

7
(a) lx + my ± tanα l 2 + m2 = 0 7K
68. The remainder when 7 7 (22 time 7) is
(b) lx + my ± z tanα l 2 + m2 + 1 = 0 divided by 48 is
(c) lx + my ± z tanα l 2 + 1 = 0 (a) 21 (b) 7
(c) 47 (d) 1
(d) lx + my ± z tanα l + m = 02 2

69. Whichever of the following is/are correct?


64. The points of intersection of two ellipses 2
dx 1
x 2 + 2 y 2 − 6 x − 12 y + 20 = 0 and (a) To evaluate I1 = ∫4+ , it is possible to x =

)
2
−2
x t

be
2 x 2 + y 2 − 10 x − 6 y + 15 = 0 lie on a circle. 1
The centre of the circle is (b) To evaluate I2 = ∫ ( x2 + 1) dx, it is possible to
(a) (8, 3) (b) (8, 1) 0

Tu
put x = sec t
(c)  , 3
8
(d) (3, 8) 1
3 
(c) To evaluate I2 = ∫ ( x2 + 1) dx, it is not possible

ou
π 0
3
sin x to put x = cosec θ
65. Let I =
π
∫ x
dx . Then
(d) To evaluate I1, it is not possible to putx =
1
4

(Y
t
3 2 3 2 3
(a) ≤ I≤ (b) ≤ I≤
8 6 2π π 70. A plane meets the co-ordinate axes at the
3 2 4π points A , B, C respectively such a way that
≤ I≤ (d) π ≤ I ≤
on
(c)
9 16 3 the centroid of ∆ABC is (1, r , r 2 ) for some
real r. If the plane passes through the point
Category-III (Q. Nos. 66 to 75) (5, 5, − 12) then r =
pi

3
Carry 2 marks each and one or more option(s) (a) (b) 4
2
m

is/are correct. If all correct answers are not 3


marked and also no incorrect answer is marked, (c) − 4 (d) −
2
then score = 2 × number of correct answers
ha

marked ÷ actual number of correct answers. If any 71. Let P be a variable point on a circle C and Q
wrong option is marked or if any combination be a fixed point outside C. If R is the
C

including a wrong option is marked, the answer midpoint of the line segment PQ, then locus
will considered wrong, but there is no negative of R is
marking for the same and zero marks will be
dy

(a) a circle
awarded. (b) a circle and a pair of straight lines
(c) a rectangular hyperbola
66. If|z + i| − |z − 1| = |z| − 2 = 0 for a complex
u

(d) a pair of straight lines


number z, then z is equal to
St

 n n n
(a) 2 (1 + i ) (b) 2 (1 − i ) 72. lim  + +
(c) 2 (− 1 + i ) (d) 2 (− 1 − i ) n→ ∞
 (n )
3
(n + 4) 3
(n + 8)3
x 3x + 2 2 x −1
n 
67. 2 x −1 4x 3 x + 1 = 0 is true for + ..... +  is
7 x − 2 17 x + 6 12 x − 1 [ n + 4 (n −1)]3 
(a) only one value of x 5− 5 5+ 5
(a) (b)
(b) only two value of x 10 10
(c) only three values of x 2+ 3 2− 3
(c) (d)
(d) infinitely many value of x 2 2
WB JEE (Engineering) Solved Paper 2021 17

0 , if − 1≤ x < 0 (a) fmin = 3 −1


 (b) fmax =
π 1
+ ln 3
73. Let f (x) = 1, if x =0 and let
6 4
2 , if 0 < x ≤1 π 1
 (c) fmin = − ln 3
x 3 4
F(x) = ∫ f ()t dt, − 1 ≤ x ≤ 1, then (d) fmax =
π
12
+ ln 5
−1
(a) F is continuous function in [− 1, 1]
75. Let f and g be periodic functions with the

)
(b) F is discontinuous function in [− 1, 1]

be
(c) F ′( x) exists at x = 0 periods T1 and T2 respectively. Then f + g is
(d) F ′( x) does not exist at x = 0 (a) periodic with period T1 + T2
(b) non-periodic

Tu
74. The greatest and least value of
(c) periodic with the period T1
1 1 
f (x) = tan −1 x − ln x on  , 3  are (d) periodic when T1 = T2
2  3 

ou
Answers
(Y
on
Physics
1. (d) 2. (c) 3. (c) 4. (a) 5. (a) 6. (c) 7. (*) 8. (a) 9. (a) 10. (c)
11. (a) 12. (b) 13. (d) 14. (a) 15. (b) 16. (b) 17. (d) 18. (b) 19. (b) 20. (d)
pi

21. (a) 22. (b) 23. (c) 24. (b) 25. (d) 26. (d) 27. (d) 28. (c) 29. (c) 30. (c)
m

31. (b) 32. (c) 33. (b) 34. (b) 35. (b) 36. (b,c) 37. (a,b) 38. (b,c) 39. (b,c) 40. (a,d)

Chemistry
ha

41. (b) 42. (d) 43. (a) 44. (b) 45. (c) 46. (d) 47. (d) 48. (d) 49. (c) 50. (b)
51. (d) 52. (d) 53. (a) 54. (a) 55. (a) 56. (d) 57. (b) 58. (d) 59. (d) 60. (c)
C

61. (d) 62. (c) 63. (b) 64. (d) 65. (a) 66. (b) 67. (b) 68. (a) 69. (a) 70. (d)
71. (a) 72. (c) 73. (a) 74. (a) 75. (d) 76. (b,c) 77. (d) 78. (c, d) 79. (b, c) 80. (b,c,d)
dy

Mathematics
1. (c) 2. (b) 3. (c) 4. (c) 5. (b) 6. (c) 7. (c) 8. (c) 9. (b) 10. (a)
u

11. (d) 12. (c) 13. (b) 14. (c) 15. (b) 16. (c) 17. (d) 18. (a) 19. (d) 20. (c)
St

21. (c) 22. (d) 23. (c) 24. (c) 25. (a) 26. (c) 27. (c) 28. (c) 29. (d) 30. (a)
31. (b) 32. (d) 33. (a) 34. (b) 35. (a) 36. (a) 37. (b) 38. (a) 39. (b) 40. (a)
41. (b) 42. (b) 43. (b) 44. (b) 45. (c) 46. (b) 47. (c) 48. (c) 49. (c) 50. (b)
51. (b) 52. (c) 53. (a) 54. (b) 55. (c) 56. (a) 57. (c) 58. (b) 59. (d) 60. (c)
61. (b) 62. (b) 63. (d) 64. (c) 65. (a) 66. (b, c) 67. (d) 68. (b) 69. (c,d) 70. (a,c)
71. (a) 72. (a) 73. (a, d) 74. (b, c) 75. (d)

(*) None of the option is correct.


Answer with Explanations
Physics
1. (d) The situation is as shown in the figure below 1 12.5
⇒ 1− =
n2 136 .
µ1 =1 µ2 = 4 125
. 1

)
3 ⇒ 1− = 2

be
136
. n
1
⇒ = 0.080
n2

Tu
⇒ n = 353
. ≈4
So, n2 = n = 4 th excited state.
Given, P = 5D
1 1 n2

ou
∴ f = = m = 20 cm 4. (a) Radius (r) of nth orbit in H-atom = × 0.529 Å
P 5 Z
Let u = − ∞ (object distance) Speed (v) of electron in nth orbit in H-atom

(Y = 2.19 × 106   m/s


Then, v = f (by sign convention) Z
Using formula for refraction at spherical surface,  n
µ 2 µ1 µ 2 − µ1 Energy (E) of electron in nth orbit in H-atom
− =
−13.6
on
v u R = eV
4 n2
−1
4 1
⇒ − = 3 [Q v = 20 cm] So, the quantity which is proportional to quantum
pi

3 × (20) (−∞) R number n is rv.


1 1 5. (a) The net voltage in the circuit is (10 − 9) V = 1 V
⇒ =
m

15 3R This leads to reverse bias of diode in the circuit.


⇒ R = 5cm Hence, current through the diode will equal
ha

to 0 mA.
2. (c) Given, D = 1.2 m, d = 0.02 cm = 0.02 × 10−2 m
According to question, 6. (c) The given logic circuit is
C

3Dλ 5Dλ AB
− . × 10−2
= 018 A
d 2d
B
dy

3 × 1.2 5 × 1.2
λ −  = 0.18 × 10−2
Y
 0.02 × 10−2 2 × 0.02 × 10−2 
AB
⇒ λ3 −
5
u

. × 10−2 × 0.02 × 10−2


× 1.2 = 018
 2
St

. × 10−4 × 0.02 × 2
018 Output is written as
⇒ λ= m Y = AB + AB
1.2
When A = 0, B = 1,
⇒ λ = 0.006 × 10−4 m
then Y = 0 ⋅ 1 + 0 ⋅1
⇒ λ = 600 × 10−9 m
= 0 ⋅ 0 + 1 ⋅1
= 600nm = 0+1
3. (c) Let initially the electron in hydrogen atom be =1
in ground state, i.e. n1 = 1 and after bombarding, it When A = 0 , B = 0,
jumps to nth energy level i.e. n2 = n, then then Y = 0⋅ 0 + 0 ⋅ 0
136. . 
136 = 0 ⋅1 + 1 ⋅ 0
12 . 5 eV =  2 − 2  eV
 ()
1 n  =0+ 0=0
WB JEE (Engineering) Solved Paper 2021 19

7. (*) According to question, from dimensional From Newton’s equation of motion,


analysis,
N 2 = 3ma = 3m ×
F Q a = F 
 
R ∝ e ambhc 6m  6m 
⇒ R = ke ambhc F
⇒ N2 =
2
Using dimensional formula of R, e, m and h,
Hence, F > N1 > N 2.
[M 0L−1 T0 ] = [M1 / 2L3/ 2T−1 ]a[M]b[ML2T−1 ]c
9. (a) At maximum compression of spring,
1 e2 e2

)
{Since, F = ⋅ 2 = 2 total kinetic energy of first block = potential
4 π ε0 r

be
r energy of spring
⇒ e 2 = Fr 2 ⇒
1 1 2
mv = kx
2

[e] = [Fr 2]1 / 2 = [MLT−2 ⋅ L2]1 / 2 = [M1 / 2L3/ 2T−1 ]} 2 2

Tu
m
⇒ [M 0L−1 T0 ] = [M a/ 2L3a/ 2T− a][M b][M cL2c T− c ] ⇒ x= ⋅v
k
a 3a
+b + c + 2c
⇒ [M 0L−1 T0 ] = [M 2 T− a− c ] 10. (c) During first half of the motion (i.e. upto x = 20

ou
L2
a m) acceleration in increasing linearly is given by
∴ + b+ c=0 …(i) 5 x
2 a= x+5 ⇒ a= + 5

(Y
3a 20 4
+ 2c = − 1 …(ii)
2 ⇒
dv x
= + 5 Q a = dv 
 
dt 4  dt 
−a − c = 0 …(iii)
on
v ⋅ dv x Q dx = v ⇒ dt = dx 
On solving Eqs. (i), (ii) and (iii), we get ⇒ = +5  
a = 2, b = 1, c = − 2 dx 4  dt v
v 20
∴ R ∝ e 2mh−2 x 
pi

2
⇒ ∫ v ⋅dv = ∫  4 + 5 ⋅ dx
e m 5 0
⇒ R∝
m

h2 v 2 v
 x2 
2

Hence, no option is correct. ⇒   = + 5x 


2
 5  8 0
ha

8. (a) Given,
v2 25 400
⇒ − = + 100
5 2 8
C

3m
F 2m v2 25
m ⇒ = 150 + ⇒ v2 = 325
2 2
Since, the table is frictionless, hence acceleration of
dy

Now, in second half of motion, acceleration is


the whole system is given as
constant, i.e. a = 10m / s2
F F
a= = ⇒ v2 − u2 = 2as
m + 2m + 3m 6m
u

Free body diagram at 1st block (left most) ⇒ v ′2 − v2 = 2as (take, v = v′, u = v)
St

a ⇒ v ′ − 325 = 2 × 10 × (35 − 20)


2
(Q v2 = 325)
F m N1 ⇒ v′ = 625 = 25m/s
From Newton's equation of motion, 11. (a) The given situation is shown below
F − N1 = ma = m ×
F Qa = F 
 
6m  6m 
θ
5F Tmin
⇒ N1 = Tmax
6 l
O
Free body diagram of 3rd block (right most)
a h θ
F 3m v
N2 mg cosθ
mg
20 WB JEE (Engineering) Solved Paper 2021

Tmin = mg cosθ …(i) Kinetic energy


mv2 =  mvCM + I CMω2 
1 2 1
Tmax − mg = …(ii)  2 
l 2
By law of conservation of energy, 1 2
1 mL2 P 2L2 × 144 
=  × m ×   + ×
P
1 × 
mgh = mv2  m 4 × m2 × L4 
2 2 12
2
1 P2
⇒ mg(l − l cosθ) = mv2 [Q h = l − l cosθ] = [1 + 3]
2 2m

)
2P 2

be
2mg(l − l cosθ) = mv2 =
m
⇒ 2mgl(1 − cosθ) = mv2 …(iii)
14. (a) Centre of mass of three particles ( i.e. 1kg, 2kg

Tu
From Eqs. (ii) and (iii), we get
and 3kg)
2mgl(1 − cosθ)
Tmax − mg = 1r1 + 2r2 + 3r3
l = [1$i + 2$j + 3k$]
6
⇒ Tmax − mg = 2mg(1 − cosθ)

ou
 m1 r1 + m2r2 + m3r3 
⇒ 2Tmin − mg = 2mg(1 − cosθ) [QTmax = 2Tmin]
QCM = m + m + m 
⇒ 2mg cosθ − mg = 2mg(1 − cosθ) [from Eq. (i)]  1 2 3 



2cosθ − 1 = 21
( − cosθ)
2cosθ − 1 = 2 − 2cosθ
4 cosθ = 3
(Y where, r1 , r2 and r3 are position vector of mass 1kg
2kg and 3kg, respectively.
Now, centre of mass of two particles ( i.e. 3kg and
on
3 2kg)
⇒ cosθ = 3r4 + 2r5
4 $]
= [− $i + 3$j − 2k
3 3 5
∴ Tmax = 2Tmin = 2mg cosθ = 2mg × = mg
pi

4 2 where, r4 and r5 are position vector of mass 1 kg and


3 2 kg.
⇒ Tmax = mg
m

2 Now, according to question, if 5kg mass is added to


whole particle system, then entire mass lies on CM
ha

12. (b) In projectile motion, horizontal component at of first system,


velocity remains constant with time. 1r1 + 2r2 + 3r3 + 3r4 + 2r5 + 5r6 $]
= [1$i + 2$j + 3k
Hence, correct representation of graph between t 16
C

and ux is shown in option (b).


where, r6 is position vector of 5 kg mass.
13. (d) L
⇒ 6$i + 12$j + 18k $ − 5$i + 15$j − 10k$ + 5r
6
dy

$
= 16$i + 32$j + 48k
P
⇒ $
5r6 = 15$i + 5$j + 40k
u

The given impulse acts as both linear and an $


⇒ r = 3$i + $j + 8k
6
angular impulse.
St

Linear impulse = P = mvCM (where, vCM = velocity Hence, position of 5 kg mass is (3, 1, 8).
of centre at mass of rod) 15. (b) Given, density of body = 1.2 × 103 kg / m 3 = ρb
L
Angular impulse = P × = I CMω Density of liquid = 24
. × 103 kg / m 3 = ρl
2
mL2 Height of fall = 1m = H
where, I CM =
12 Let the volume of the body be V.
and ω is angular velocity at rod about centre of Then, buoyant force acting on the body when it is
mass, totally immersed in liquid = (ρl Vg)
PL Weight at the body = (ρbVg)
i.e ω=
2I CM Net upward force acting on the body = (ρl − ρb)Vg
WB JEE (Engineering) Solved Paper 2021 21

Net deceleration produced 17. (d) Given figure is


Net upward force p
(a) =
Mass of body
(ρl − ρb)Vg 1
(a) = p0
2
ρbV
(ρl − ρb) 3
(a) = g
ρb V

)
V0 2V0
Let initial velocity of the body be u, then

be
1 Process 1 is isobaric expansion (p = constant).
(ρbV)u2 = ρbVgH Hence, temperature of gas will increase, therefore
2
∆U1 will be positive.
⇒ u2 = 2gH

Tu
Process 2 is an isothermal process, ∆U 2 = 0
Final velocity of the body will be zero. Process 3 is an adiabatic expansion, hence
⇒ v2 − u2 = 2as temperature of gas will fall.

ou
(ρ − ρb) g ∴ ∆U 3 = negative
⇒ (0)2 − 2gH = − 2 l ×s
ρb Therefore, ∆U1 > ∆U 2 > ∆U 3
 

(Y
ρb 1.2 × 103 18. (b) The van der Waals’ equation for real gas is as
⇒ s= ⋅H=  ⋅1
ρl − ρb  2.4 × 10 3
− 1.2 × 10 3
 follows
an2
=
1.2
=1 m p + 2 (V − nb) = nRT
on
1.2 V
where, p is the pressure, a and b are constants, V is
16. (b) Given densities of both spheres are same. Let the volume, T is the temperature and n is the
it be ρ unit and density of liquid be σ and η be its
pi

number of moles.
viscosity.
van der Waals’ equation can be rearranged as
Let radius of sphere S1 be r1 and that of sphere S2 be
m

nRT an2
r2 and volumes are V1 and V2 , respectively. p= − 2
Now, V − nb V
ha

m1 m2 Dividing first part with n and the 2nd part with n2,
= (Q density is same)
V1 V2 we get
nRT an2
C

m1 m2
⇒ = 2
4 3 4 3 p= n − n2
πr1 πr2 V nb
3 3 − V
dy

3 n n n2
 r1  m  8
⇒   =  1 = RT a
 2
r  m2  1 ⇒ p= − 2 …(i)
V V
−b
u

r1 2
⇒ = n n2
St

r2 1
The equation given in question is as follows
Now, terminal velocity is given by RT a
p= − …(ii)
2r 2(ρ − σ) g 2V − b 4b 2
v=
9η By comparing Eqs. (i) and (ii), we get
2r12(ρ − σ) g V
= 2V

v1
= 9η n
v2 2r22(ρ − σ) g ⇒ n=
V
=
1 1
(i.e. n = mole)
9η 2V 2 2
2 By using mole formula to determine the amount of
v1  r1 
⇒ =   = 22 = 4 O2 as follows
v2  r2  Mass
Mole = …(iii)
Molar mass
22 WB JEE (Engineering) Solved Paper 2021

The molar mass of O2 is 32g/mol. Substituting in Eq. 21. (a) The given situation is shown below
(iii), we get
1 Mass +
⇒ = +
2 32g / mol b

⇒ Mass of O2 = 16 g –
+q +
19. (b) We know that, latent heat of fusion at ice is – r a –
79.7 cal per gram. – –
+ –

)
Let final temperature be T. +
P

be
Then, +
m1 s∆T = m2L
⇒ 300 × 1 × (25 − T) = 100 × 75 A radial electric field E exists in the region

Tu
⇒ 25 − T = 25 between the two shells due to charge on inner
shell only.
⇒ T = 0° C
Electric field at point P is calculated according to
After that total energy left, Q = 4.7 × 100 = 470 cal

ou
Gauss’s law,
Total mass of water = 400 g q
Amount of water again converted into ice, i.e φE = E ⋅ 4 πr 2 =
ε0

(Y
Q 470
m= = q
Lice 79.7 ⇒ E= …(i)
4 π ε0 r 2
⇒ m = 59
. g
The potential at the centre of sphere,
on
Thus, whole mass is converted into water at 0°C and b b b q
about 5.9 g water is again converted into ice whose V = − ∫ E ⋅ dr = ∫ E ⋅ dr = ∫ dr
a a a 4 πε r 2
temperature is also 0°C. 0
pi

After achieving the temperature of 0°C, latent heat [from Eq. (i)]
of fusion is required firstly for conversion of water q 1 1 
⇒ V=  − 
m

into ice, then further lowering of temperature as 4 π ε0  a b 


possible.
22. (b) The given situation is shown in the figure
ha

So, the final temperature will be 0°C.


20. (d) The electric field along Z-axis due to uniformly X
–σ 2σ 4σ
charged circular ring of radius a in XY-plane is
C

given by
kqz
E= 2 2a
,0
(z + a 2)3/ 2 0,
dy

O Y
a P
where, q is the net charge on the ring and z is axial
distance on Z - axis from the centre of the ring. 3a
u

Now, at point M in the graph shown, electric field is


Z 4a
St

maximum.
So, We have to calculate electric field at
3
kq(z 2 + a 2)3/ 2 − (z 2 + a 2)3/ 2 ⋅ 2z 2 point P (0, 2a, 0).
dE 2
= Net electric field at point P,
dZ (z 2 + a 2)
−σ $ 2σ $ 4σ $
dE E= j− j− j
For maxima, =0 2 ε0 2 ε0 2 ε0
dZ
−7σ $
3 E= j
⇒ (z 2 + a 2)3/ 2 = × 2z 2(z 2 + a 2)1 / 2 2 ε0
2
⇒ z + a = 3z 2 ⇒ 2z 2 = a 2
2 2 23. (c) The given situation is shown below
z 1 +q1 q3 +q2
⇒ =
a 2
d
WB JEE (Engineering) Solved Paper 2021 23

Case 1 If charge q 3 is taken as positives. ∴ From Eq. (i), we get


In this situation, force on charge q 3 due to charges q = 2πRλ
q1 and q 2 will be repulsive and in opposite direction. Current associated,
Hence, at a certain location between + q1 and + q 2, q
I=
charge q 3 will be in equilibrium, when both T
repulsive forces will be equal in magnitude. 2πRλ
I= …(ii)
Case 2 If charge q 3 is taken as negative. T
In this situation, force on charge q 3 due to charges Magnetic field on the axis at a distance d away from

)
q1 and q 2 will be attractive and in opposite direction. the centre,

be
Hence, at a certain location between + q1 and + q 2,
µ 0 IR2
charge q 3 will be in equilibrium, when both B=
attractive forces will be equal in magnitude. 2(R + d 2)3/ 2
2

Tu
24. (b) The given situation is shown below For d >> R,
Y µ 0 IR2
B=
2(d 2)3/ 2

ou
µ 0 IR2
L(–1,1) BL =
2d 3
BM
O
X
(Y =
µ 0 
2πRλ  2
 T 
2d 3
R
on
M (–1,–1) µ πλ R3
= 0 . 3
T d
R3 Rm
pi

⇒ B∝ 3 [given, B ∝ ]
Length, OL = 1 + (−1) =
2 2
2 d dn
m

∴ m = 3, n = 3
and OM = (−1)2 + (−1)2 = 2
26. (d) Since, susceptibility χ of diamagnetic material
ha

QMagnetic field at point O due to wire L,


is independent of temperature.
µ I  $i + $j  Q B = µ 0 ⋅ I 
BL = 0 .   1/χ
2π 2  2   2π r 
A
C

Magnetic field at point O due to wire M,


B
µ I  − $i + $j 
BM = 0 .  
dy

2π 2  2  T
1
∴Resultant magnetic field at the origin O, Hence, is also independent of temperature.
χ
u

B = BL + B M
Therefore, material B is diamagnetic. For
µ I  $i + $j  µ 0 I  − $i + $j 
St

= 0   + .   paramagnetic substance, susceptibility varies


2π 2  2  2π 2  2  inversely as temperature.
1 1
µ 0 I  $i + $j − $i + $j  i.e. χ∝ or ∝T
= χ
2 2π   T
2 
Therefore, material A is paramagnetic.
µ 0I µ I
= .( 2$j) = 0 $j
27. (d) V
2 2 2π
25. (d) Charge per unit length of the thin rod = λ
V0
If l be the length of the rod, the charge on the rod,
q = lλ …(i) t
0 T/2 T 3T
According to given situation, 2
l = 2πR
24 WB JEE (Engineering) Solved Paper 2021

Rms value of potential difference is given as Here, i=r


V02 + 0 2 Since, i + r = 90°
Vrms = ⇒ i + i = 90°
2
⇒ i = 45°
V0
= i = r = 45°
2
Coordinate at point P is
28. (c) (R cosθ, Rsinθ) = (R cos135° , Rsin135°)
 −1 1 

)
=  R  , R  

be
  2  2 
−R R 
= 
Red Yellow Orange
, 
 2 2

Tu
The colours of the four bands of carbon resistors are
red, yellow, orange and no fourth band. 31. (b) For a plane electromagnetic wave, the electric
∴ Red Yellow Orange No colour field is given by

ou
↓ ↓ ↓ ↓ $ V/m
E = 90sin(0.5 × 103 x + 1.5 × 1011 t) k
2 4 3 20% Here, E 0 = 90 V/m
∴ R = 24 × 103 Ω ± 20 % E

(Y
∴ B0 = 0
= 24kΩ ± 20% c
90
29. (c) L =
3 × 108
on
= 30 × 10−8 T
= 3 × 10−7 T
~
pi

V=V0 sin ωt Since, electric field vector and magnetic field


vector, both are perpendicular to each other and
m

In pure inductive circuit, alternating voltage leads


π also perpendicular to direction of propagation of
angle from alternating current. electromagnetic wave. Hence,
2
ha

π B = B0 sin(0.5 × 103 x + 1.5 × 1011 t)$jT


i.e. Phase difference, φ =
2 B = 3 × 10−7 sin(0.5 × 103 x + 1.5 × 1011 t)$jT
C

Average power consumed, 32. (c) Since, both wires are identical, so they have
π
P = Vrms × I rms × cos same cross-section A and same length l. Also, in
2 series combination of wires, effective resistance,
dy

⇒ P=0 Q cos π = 0 R = R1 + R2
 2  (l + l) 1 1
⇒ = +
σ eff A σ1 A σ 2 A
u

30. (c) The given situation is shown below


2 1 1
St

Y ⇒ = +
σ eff σ1 σ 2
2σ1 σ 2
r ⇒ σ eff =
i σ1 + σ 2
P R
33. (b) Initial length of rod = L
135°
X Initial angular velocity, ωi = ω
O
When rod pivoted at one end is freely rotated in
horizontal plane, then its angular momentum,
Li = I1ω1
 ML2 
Reflecting surface (x 2 + y 2 = R 2 ) Li =   ⋅ω
 3 
WB JEE (Engineering) Solved Paper 2021 25

∴Putting the value of ρ in Eq. (i), we have


∞ ( − Mgh / RT )
∫ hρ0 e
h = 0∞
dh
=
RT
− (Mgh/RT )
∫0 ρ0 e
L dh Mg

35. (b) Under isothermal conditions,


p1 V1 + p2V2 = pV
When temperature of the system is increased with
temperature ∆T, then change in length of rod ∆L is 
⇒ p −
4T  4 3  4T  4 3
 . πa +  p −  ⋅ πb

)
given as  a  3  b  3

be
=  p −
∆L = αL∆T 4T  4 3
 πc
Now, new length of the rod,  c 3

Tu
L ′ = ∆L + L = αL∆T + L = L(α∆T + 1) where, T is surface tension of soap bubble.
⇒ L ′ = L(1 + α∆T) …(i) ⇒ p[c 3 − (a 3 + b 3)] − 4T(a 2 + b 2 − c 2) = 0
Now, new angular momentum, p(c 3 − a 3 − b 3)

ou
⇒ T=
ML ′2 ω 4(a 2 + b 2 − c 2)
L f = I 2ω2 = .
3 2
By the law of conservation of angular momentum, 36. (b,c)

(Y
M
L f = Li O
a
ML2 x
I 2ω2 = ⋅ω
on
3
ML ′2 ω ML2
⋅ = ⋅ω Magnetic field at the centre O of the circular coil
3 2 3
due to bar magnet (axial position)
pi

L ′2 µ 2M
⇒ = L2 B= 0. 3
2 4π x
m

L2(1 + α∆T)2
⇒ = L2 [from Eq. (i)] Hence, option (a) is not true.
2 dφ d
ha

Induced emf, e = − = − ⋅ BA
⇒ (1 + α∆T)2 = 2 dt dt
⇒ 1 + α 2∆T 2 + 2α∆T = 2 d µ 2M
= − ⋅ 0 ⋅ 3 πa 2
dt 4 π x
C

Since, α is very small, hence the term α 2∆T 2 may be


µ Ma 2 d 1
neglected. =− 0 ⋅
∴ 1 + 2α∆T = 2 ⇒ 2α∆T = 1 2 dt x 3
dy

−3 dx
⇒ ∆T =
1 = − µ 0 Ma 2.  4 
2α  x  dt
u

3µ 0 Ma 2 dx
34. (b) As the gravitational field is uniform, therefore e= ⋅
St

centre of gravity and the centre of mass are at x4 dt


same location. 1
⇒ e∝ 4
∴The location of centre of mass is x
∞ Hence, option (b) is true.
∫ hdm
h= 0∞ e 3µ 0 Ma 2 dx
Induced current, I = = …(i)
∫0dm R x 4 R dt
∞ 3µ 0 Ma 2 dx 2
∴ Magnetic moment, µ = IA = πa
∫ hρdh
h = 0∞ …(i) x 4 R dt
3 π µ 0 M dx 4
∫0 ρdh ⇒ µ= (a )
x 4 R dt
But from barometric formula and gas law,
µ ∝ a4
ρ = ρ0 e − Mgh/ RT
⇒ Hence, option (c) is true.
26 WB JEE (Engineering) Solved Paper 2021

Net heat produced, p


(V0 , 2p0)
H ∝ I2
2p0 C
2
H ∝  4 
1 (2V0 , p0)
⇒ [from Eq. (i)]
x  p0 A
B
1
⇒ H∝
x8 V
V0 2V0
Hence, option (d) is not true.

)
Heat rejected by the gas in path AB during isobaric
37. (a,b) Given, E = a(cosω0 t + sinωt + cosω0 t)

be
compression,
ω = 1015s−1 dQ AB = nCp ∆T
5
ω0 = 5 × 1015 s−1

Tu
= 1 ⋅ R(TB − TA) [Q n = 1]
2
Since, ω < ω0 5R  p0 V0 2p0 V0  p V
=  −  = −5 0 0
⇒ 2πν < 2πν0 2 1 × R 1 × R  2

ou
⇒ ν < ν0 [ν0 = threshold frequency] Heat absorbed by the gas in path BC of isobaric
Hence, for light of frequency ω, process,

(Y
photoelectric effect is not possible. dQBC = nCV ∆T
3R
According to Einstein’s equation, = 1 × (TC − TB)
2
eV0 = hν − hν0
3R  2p0 V0 p0 V0  3p0 V0
on
eV0 = hν − φ0 =  − =
2  1 × R 1 × R 2
Hence, stopping potential versus frequency graph
will be straight line. Net heat = dQ AB + dQBC
pi

−5p0 V0 3p V
= + 0 0 = − p0 V0
Stopping (V0) 2 2
m

potential
∴Heat is rejected during the process.
Change in internal energy in process A → B,
ha

3
∆U = nCV ∆T = 1 × R(TB − TA)
2
3R  p0 V0 2p0 V0 
C

Frequency = −
 
(ν 0 ) 2 1 × R 1 × R 
3
= − p0 V0
dy

Work-function of metal surface, 2


ω
φ0 = hν0 = h 0  Work done by the gas during entire process is W,
 2π  then
u

5 × 1015 W = W AB + WBC
= 6.62 × 10−34 ×
St

2π = p0 (V0 − 2V0) + 0 = − p0 V0
. × 10−19J
= 527 39. (b,c) Mass, m = 0.02 kg

. × 10
527 −19 Potential energy,
= eV U = Ax(x − 4) J
1.6 × 10−19
⇒ U = Ax 2 − 4 Ax
= 329
. eV
dV d
∴Maximum kinetic energy of photoelectron, Force, F=− = − (Ax 2 − 4 Ax)
dx dx
K max = − (eV0) = − [e(−2)] = 2eV
⇒ F = − 2Ax + 4 A …(i)
38. (b,c) Since, given p-V diagram is not a cyclic This force is dependent on x, hence particle is not
process, hence the change in internal energy for acted upon by a constant force.
the whole process is not zero.
From Eq. (i), it is clear that f ∝ − x
WB JEE (Engineering) Solved Paper 2021 27

Hence, particle executes simple hormonic motion. Since, charge particle enters into magnetic field B
Speed of particle is maximum at equilibrium perpendicular to it, hence it performs a circular
position, i.e. when F = 0 path in magnetic field. Radius of circular path,
−2Ax + 4 A = 0 mv
i.e. r=
⇒ x = 2m Bq
Period of oscillation, Charge particle will enter in region c, when
m L < 2r
T = 2π …(i) mv
⇒ L< 2

)
k
Bq

be
From Eq. (i), F = −2Ax + 4 A
LBq qLB
Since, value of k will be obtained in terms of A. ⇒ < v ⇒ v>
Therefore, the value of time period will be also 2m 2m

Tu
obtained in terms of A, hence option (d) is not ⇒ 
v>  
1  qLB
correct.  2 m
qLB
40. (a,d) The given situation is shown below i.e. When v > , then particle surely will inter

ou
b m
a qLB qLB
× × × c into region c, because >
m 2m

(Y
× × ×
Hence, option (a) is correct.
× × ×
Since, charge particle enters into region c, hence
× × × path of the particle is a circle not in region b.
on
× × × Hence, option (c) is not correct.
× × × Time spent in region b is given as
v 2πm
pi

× × × T= which is independent of v.
Bq
m

L Hence, option (d) is correct.


ha

Chemistry
41. (b) The exact order of the boiling points of the Therefore, the maximum number of atom in one
C

given compound is isopentane < n-pentane plane in this case are 15.
< butanone < 1-butanol. 43. (a) The total number of triple bonds present in
dy

The highest boiling will be of butanol because in cyclo (18) carbon are 9. Its structure is shown
butanol, molecules are associated due to extensive below:
intermolecular hydrogen bonding. C ≡≡ C — C≡≡
u

Now butanone has higher boiling point from —


C C—
≡≡

n-pentane and iso-pentane because butanone has


St

stronger dipole-dipole interaction and n-pentane C C


≡≡

has higher boiling point than isopentane because of C


C

larger surface area, due to which n-pentane has


≡≡

greater van der Waals’ force of attraction. C


C
≡≡

42. (d) All the atoms present in p-nitrobenzonitrile C≡ — C


are in one plane as show in the structure ≡ C
C≡≡
C
C ≡≡ C
— —
14 15
H H
13 O 5
Structure of cyclo (18) carbon
4
11
N 6 2 1 44. (b) Resonating structures II and IV of p-nitro-N,
== + 7 3 C ≡≡ N N-dimethyl aniline are incorrect. In structure II
O12 8 and IV, valencies of N are not satisfied correctly.
H 10 H9
28 WB JEE (Engineering) Solved Paper 2021

All the correct resonating structures of p-nitro-N, The general formula of amino acid is
N-dimethyl aniline are as follows H 2 N  CH(R) COOH, R is CH 3 for alanine and H
+ + + for glycine. The structure are as follows
NMe2 NMe2 NMe2 NMe2 O O
 
– – H 2N  CH  C  OH + H 2N  C H  C OH →
 
CH 3 H

NH2 NH2

)
+ +
N N N+ N+

be
NH COOH + NH COOH +
O O– O O– O O– O O– CH3 CH3
CH3
45. (c) COOH CH3 O O

Tu
H H

H OH and HO H are monomers. N COOH N COOH


H2N HN
+ 2

ou
CH3 COOH O Me O
When the same compound is represented in 48. (d) Benzaldehyde reacts with 2 moles of methanol

(Y
different ways, then all the representations are OMe
in presence of dry HCl to give Ph which
known as homomers of each other. on reaction with dil. HCl OMe
CH3 CH3 gives respective aldehyde and alcohol. Aldehyde
on
on reaction with (CH 3CO)2O,
and CH 3COONa gives COOH
are also homomers. Ph
pi

NO2 O2N OMe dil. HCl


Dry
PhCHO + CH3OH Ph
CH3 HCl
m

H Cl Cl Benzaldehyde OMe
and are constitutional
H3C Cl Cl H COOH (CH3CO)2O
PhCHO+2CH3OH
ha

cis trans Ph
CH3COONa
isomers.
Constitutional isomers are the compounds that 49. (c) The free energy change of a reaction is
C

have the same molecular formula and different ∆G° = ∆H ° − T∆S °


connectivity. Now, a spontaneous reaction is one that releases
46. (d) The correct order of acidity of the given free energy and therefore, the sign of ∆G must be
dy

compounds is acetylene < ethanol < negative.


p-nitrophenol < acetic acid. So, when ∆H is negative and ∆S is positive, the sign
of ∆G will be negative and the reaction will be
u

Ethanol is stronger acid than acetylene. This is


because after the loss of acidic proton (H + ) the spontaneous at all temperature.
St

corresponding anion formed by ethanol is stabilised 50. (b) In acidic medium,


by the electronegative oxygen atom. Now +7
p-nitrophenol is more acidic than ethanol because Mn O−4 + 8H+ + 5e − → Mn2+ + 4H 2 O
the phenoxide ion obtained after the deprotonation and Fe 2+ → Fe 3+ + e −
is stabilised by resonance effect and also by EWG 1
effect (which is caused by NO2 group). So, number of moles of MnO−4 = moles …(i)
5
Likewise, acetic acid is more acids than all of the
given compounds as the formed carboxylate ion is Also, in acidic medium,
+6
stabilised by resonance which is due to the presence Cr2 O 27− + 14H+ + 6e − → 2Cr 3+ + 7H 2O
of electronegative oxygen atom.
and Fe2+ → Fe3+ + e −
47. (d) All the given dipeptides can be obtained from
1
the amino acids glycine and alanine. 7 =
So, number of moles of Cr2O 2− moles …(ii)
6
WB JEE (Engineering) Solved Paper 2021 29

Now, from Eq. (i) For gas A,


1
moles of MnO−4 = x moles of MnO −4 u1 =
3RT
5 m1
1
∴ = x 3RT
5 u12 =
m1
5x = 1 …(iii)
u12m1
Put this value in Eq. (ii). T= …(i)
1 5x 3R

)
will be For gas B,

be
6 6
So, the number of moles of Cr2 O72− required to 3RT
u2 =
oxidise the same amount of Fe 2+ in acidic medium m2

Tu
5 3RT
is x or 0.83x. u22 =
6 m2
51. (d) Density of the element (d) = 5.0 g cm−3 u22m2
T=

ou
…(ii)
Edge length of unit cell (a) = 200 pm 3R
= 200 × 10−10 cm From Eq. (i) and (ii),

(Y
Z× M u12m1 u22m2
d= 3 =
a × NA 3R 3R
[where M = mass of element ∴ m1 u12 = m2u22
on
N A = Avogadro number 53. (a) ∆Tf = i × k f × m
Z = number of atoms per unit cell]
∆Tf = depression in freezing point of solvent
For body centred cubic lattice, Z = 2
pi

i = van’t Hoff factor


2× M
5= k f = freezing point depression constant
(200 × 10−10)3 × 6.022 × 1023
m

m = molality
5 × (200 × 10−10)3 × 6.022 × 1023 ∆ Tf
⇒ M= i= =
2
ha

2 m × kf  wsolute × 100 
5 × (200)3 × 1030 × 6.022 × 1023   × 1.72
=  M solute × Wsolvent 
2
C

2
= 12. 044 g =
 20 × 1000  × 1.72
Now, 12.044 g of element have → 6.022 × 1023  
 172 × 50 
dy

atoms
 Msolute = M( C11 H 8 O 2) 
6.022 × 1023  
1 g will have → atoms
12. 044  = 12 × 11 + 8 × 1 + 2 × 16 
u

 = 172 
6.022 × 1023  
100 g will have → × 100 atoms
St

2
12. 044 ⇒ i= = 0.5
4
= 0.5 × 10 25
atoms or
54. (a) The presence of catalyst does not change the
= 5 × 10 atoms
24
value of equilibrium constant. It only increases
the rate of forward and backward reaction to
52. (d) Let two gases are A and B.
equal extent. So, the equilibrium constant in
u1 = molecular velocity of gas A presence of catalyst at 2000 K is same as
u2 = molecular velocity of gas B equilibrium constant in the absence of catalyst at
m1 = molecular mass of gas A 2000 K, i.e. 4 × 10−4 .
m2 = molecular mass of gas B 55. (a) For 1st order kinetic, t1 / 2 =
ln 2
= 40 …(i)
3RT k1
The root mean square velocity, urms =
M [A ]
For 2nd order kinetics, t1 / 2 = 0 = 20 …(ii)
(where, M = molecular mass, R = gas constant) 2k0
30 WB JEE (Engineering) Solved Paper 2021

From Eq. (i), 0.1 × h2


= 10−2
ln 2 1−h
40 =
k1
On calculating, h = 0.27 which is significant, not
ln 2 negligible.
⇒ k1 = …(iii)
40 ∴ [H + ] = Ch = 0.1 × 0.27 = 2.7 × 10−2 M
From Eq. (ii),
[A0 ] 59. (d) For salt MX, K sp = [S] [S]
20 = …(iv)
2k0 K sp = 4 × 10−8 (given)

)
be
[A0 ] ∴ 4 × 10−8 = [S]2
k0 =
40
∴ S = 2 × 10−4
 k1  ln 2 40
  = ×

Tu
Now, ratio For salt MX 2, K sp =[S] [2S]2
 k0  40 [A0 ]
ln 2 0.693 . × 10−14
K sp = 32
= =
[A0 ] 1.386 mol dm −3

ou
1/ 3
. × 10−14 
 32
−1
= 0.5 mol dm 3 [S] =  
 4 

(Y
56. (d) Among the given solutions, NaCl, KNO3 and = (8 × 10−15)1 / 3 = 2 × 10−5
HCl are strong electrolytes but conductivity also For salt MX 3, K sp = [3S]−3[S]
depends upon size of ions. Smaller the size of
ions, greater its conductance, hence greater is the K sp = 2.7 × 10−15
on
conductivity than larger ions. So, out of NaCl, 1/ 4
KNO3 and HCl ions, H + is the smallest and  2.7 × 10−15 
[S] =  
therefore, 0.1 M HCl will have highest  27 
pi

conductivity.
= (10−16)1 / 4 = 10−4
57. (b) Na2 S + nS → Na2 S( n + 1)
m

∴Solubility of MX order is
Na2 SO3 + S → Na2 S2 O3 MX > MX 3 > MX 2
ha

X = Na2 S( n + 1) 60. (c) Reduction hydrogen half-cell is


Y = Na2 S2 O3 H+|(xM)|Pt (H 2 ); Pressure pH 2
C

58. (d) Let a weak monoacidic base is BOH. BOH is


Half-cell reaction is
neutralised as follow
2H + (aq) + 2e − → H 2(g)
BOH + HCl → BCl + H 2 O
dy

At point of equivalence, all BOH get converted into n= 2


salt and the concentration of H + ions is due to the Reaction quotient ⇒ Q =
pH 2
hydrolysis of resultant salt, BCl. [H + ]2
u

B+ + H 2 O q BOH + H + ° 0.0591
E red = E red − log Q
St

C(1 − h ) Ch Ch
n
Volume of HCl used 0.0591
NV 2.5 × 2 × 15 E red = − log Q
Va = b b = = 7.5mL 2
Na 2× 5 1
Case (a), log Q = log = 0 ∴ E red = 0
Now, the concentration of salt, [BCl] 1
Conc. of base 2 × 2.5 1 1
= = = = 0.1 Case (b), log Q = log = − ve ∴ E red = + ve
Total volume 5 (7.5 + 2.5) 10 4
Ch2 K 10−14 2
Case (c), log Q = log = + ve ∴ E red = − ve
So, K h = = w = −12 = 10−2
1 − h K b 10 1
2
Ch2 Case (d), log Q = log = − ve ∴ E red = + ve
or = 10−2 4
1−h
∴E red is negative for option (c).
WB JEE (Engineering) Solved Paper 2021 31

61. (d) Solubility of saturated solution = Solubility Its structure is as follows:


product
∴ K sp = 4 × 10−5 (for BaSO 4 ) O+
Let the solubility of BaSO4 in 0.1 M Na2 SO4 be x.
BaSO4 c Ba2+ + SO24 − H H
x x H
Na2 SO4 c 2Na+ SO24 − Hybridisation = sp 3 and shape = pyramidal
2 × 0. 1 ( 01
67. (b) Mass on the reactant side = 18.00567 amu

)
.)
( 0. 2)
Mass of the proton = 1.00782 amu

be
∴Total (SO24−) = x + 01
.
Energy absorbed in the reaction = 116
. MeV
≈ 0.1 M = 0.00124 amu
K sp = [Ba2+ ] [SO24− ]

Tu
Thus, 14
N(α, p ) 17O
4 × 10−5 = [x] (0.1) This reaction can be simplified as
−4 ∆E
∴ x = 4 × 10 14
+ 42α → 17
+ 11 H

ou
7 N 8 O
−4
∴ Solubility = 4 × 10 M where, ∆E is the energy absorbed.
In this reaction, highly energetic alpha particle with
62. (c) The given reaction is as follows:
Co(NO 3)2 + 7Na 2NO 2 + 4CH 3COOH →
Na 3[Co(NO 2)6] + 3NO 2 + 2H 2O + 4CH 3COONa (Y kinetic energy 1.16 MeV are absorbed by 14
nucleus to form 17
Hence,
8 O isotope and a proton.
7 N
on
Sodium cobalt nitrile 1.16MeV
With the salt of potassium, sodium cobalt nitrile
14
7 N + 42α → 17
8 O + 11 H
forms insoluble double salt K 2Na[Co(NO 2)6]⋅ H 2O.
18.00567 amu 1.00782 amu
pi

This salt is yellow coloured ppt.


According to conversation of mass-energy.
63. (b) The metal (M) is copper and the reactions are 18.00567 amu = ( 17
8 O + 1.0078 − 0.00124) amu
m

as follows: 17
8 O = 18.00567 − 1.0078 + 0.00124

2Cu 2 S + 3O2 → 2Cu 2 O + SO2 ↑ = 16.99914 amu
ha


Cu 2 S + 2Cu 2 O → 6Cu + SO2 ↑ 68. (a) A solution of NaNO3, when treated with a
mixture of Zn dust and NaOH (caustic soda)
C

64. (d) When CO2 is passed through ammoniacal yields ammonia gas.
solution of NaCl, sodium bicarbonate (NaHCO3) Complete reaction is as follows
and (NH 4 Cl) are formed.
Zn + NaOH + NaNO 3 → Na 2ZnO 2 + H 2O + NH 3 ↑
dy

NH 4OH + NaCl +CO 2 → NaHCO 3 + NH 4Cl Zinc Caustic Sodium Sodium Ammonia
( X) Ammonium metal soda nitrate zincate
Sodium
bicarbonate chloride
(Y) 69. (a) In K 3[Fe(CN)6 ]Fe present in +3 oxidation state
u

On further heating NaHCO3, we get Na2 CO3 . outer electronic configuration of Fe 3+ =


St

2NaHCO 3 → Na 2CO 3 + H 2O + CO 2 3d 4s 4p
Sodium
carbonate
(white ppt.)
( Z) As CN is a strong field ligand. So, it causes the
pairing of electrons.
65. (a) Structure of ozone (i.e. O3) has delocalised Therefore, K 3[Fe(CN)6 ] =
π-electrons. Its resonance structure are as follows: 3d 4p 4s
O⊕ O⊕ O
O O O

 128 pm
O 117°
O O
CN CN CN CN CN CN
66. (b) The structure of the hydronium ion is 2 3
pyramidal because it has three bonding pairs of d sp
electrons and one lone pair of electrons.
32 WB JEE (Engineering) Solved Paper 2021

Thus, in this complex, number of unpaired electron is h


73. (a) de-Broglie wavelength, λ =
one. mv
In K 4[Fe(CN)6 ]Fe, present in +2 oxidation state λ He M Ne 20
∴ = =
outer electronic configuration of Fe 2+ λ Ne M Ne 4
3d 4s 4p
∴ λ He = 5λ Ne
74. (a) Given,
As in this case also the ligand is CN (which is a x solute = 0.1
strong field). So, it again causes the pairing of the

)
electron. This can be shown in following way. Molarity = Molality

be
K 4[Fe(CN)6 ] = Density of solution = 2g cm −3
3d 4s 4p 2 × 10−3
= kg
10−6

Tu
Thus, in this case the number of unpaired electron = 2 × 103 kg/m 3
is zero. = 2 kg/L

ou
70. (d) When two complexes have same number of
unpaired electrons, they have same value of msolute msolute
magnetic moment. The number of unpaired Msolute Msolute
=
electrons present in[Cu(H 2 O)6 ]2+ , [Mn(H 2 O)6 ]4+ ,
[Mn(H 2 O)6 ]3+ , [Fe(H 2 O)6 ]3+ and [Cr(H 2 O)6 ]3+ are
0, 3, 4, 5 and 3 respectively.
(Y Volume Mass of solvent (kg)
Volume of solution = Mass of solvent (kg)
Mass of solution
= Mass of solvent
on
Thus, the magnetic moment of [Mn(H 2 O)6 ]3+ and d
[Cr(H 2 O)6 ]3+ are identical as they have same Mass of solution
∴ =2
Mass of solvent
pi

number of unpaired electrons (i.e., 3).


Mass of solute+ Mass of solvent
71. (a) Methoxymethyl chloride (CH 3 OCH 2 Cl) ∴ =2
Mass of solvent
m

hydrolysed most easily and neopentyl chloride Mass of solute


[(CH 3)3 C  Cl] hydrolysed most slowly. Primary ∴ =1 …(i)
Mass of solvent
ha

halide readily undergoes SN 2 mechanism and


ηsolute
forms intermediate whereas tertiary halides due = 0.1
to steric hinderance reacts slowly with water. ηsolute + ηsolvent
C

72. (c) Phenol on nitration with dil. HNO3 gives msolute


p-nitro phenol (X) which on heating with Zn/HCl, M solute
= 0.1
dy

form p-amino phenol, which on acetylation with msolute m


+ solvent
[CH 3 (CO)2 O] gives p-hydroxy acetanilide. Reaction M solute M solvent
is as follows :
1
u

OH OH OH M solute {Q M solute = msolvent


= 01
.
St

1 1 using (i)}
+
dil.HNO3 Zn/HCl, ∆ M solute M solvent
M
(CH3CO)2O ∴ solute = 9
M solvent
NO2 NH2
( X) 75. (d) IE Na = 490 kJ/mol
OH
m (g)
ηNa = Na
M Na
. × 10−3
575
=
23
NHCOCH3 ∴ IE Na = 490 × 0.25 × 10−3
p-hydroxy acetanilide = 0.1225 kJ
WB JEE (Engineering) Solved Paper 2021 33

76. (b,c) Butyne (Me  C ≡≡ C  Me) on reaction with Na / NH 3(liq.) undergoes reduction to give but-2-ene which
on reaction with dil. alkaline KMnO 4 gives diol.
H H OH H H OH
Na/NH3(liq.) Dil. alk. KMnO 4
Me—C== C—Me Me—C== C—Me Me—C—C—Me + Me—C—C—Me
Ethanol, –33ºC
But-2-ene
H OH HO H
Butan-2, 3-diol

)
be
77. (d) Complete reaction is as follows :
O
OMg+Br–

Tu
Mg, ether – + CH3C—H
Br CH3 BrMg CH3 H3C—C CH3
H
4-bromotoluene

ou
Br2/NaOH,
H3O+ SOCl2
HOOC CH3 ClOC CH3

4-methyl benzoic acid


( X) (Y 4-methyl benzoylchloride

NH3
on
Br2/NaOH O
H2N CH3 C CH3
Hoffmann H2N
pi

bromamide
(Y) degradation 4-methyl benzamide
m

78. (c, d)Buffer solutions are made by mixing weak acid with its salt of strong base or mixing weak base with its
salt of strong acid. It is never formed by mixing strong acid and strong base. The pair(s) of solutions which
ha

− + − +
form a buffer upon mixing are (c) HNO3 and CH 3 COONa, (d) i.e. CH 3 COOH and CH 3 COONa.

79. (b, c) Reaction of silver nitrate solution with phosphorus acid produces phosphoric acid and metallic silver.
C

Complete reaction is as follows:


H 3 PO3 + AgNO3 + H 2 O −→ H 3 PO4 + Ag ↓ + 2HNO3
dy

Phosphorus Phosphoric
acid acid

80. (b, c, d) N 2 H 4 and H 2 O2 show similarity in reducing and oxidising nature. They have same hybridisation of
central atoms i.e. sp 3. The structure of hydrazine and hydrogen peroxide as follows.
u

H
St

H H
N N O O
H
H H
Hydrazine Hydrogen peroxide
(N2H4 ) (H2O2 )
34 WB JEE (Engineering) Solved Paper 2021

Mathematics
 x x2  2  1
dt − ∫ 2 dt 
a
b 2  ∫ t
 e − x −1 −  =−
t 
1. (c) Given, I = lim  2
x→ 0  2 

x
 2  t− 2 + 1 
  = − 2 ∫  log() t −a + C
b  − 2 + 1

)
 x x2 
= − 2  log | t | +  + C
 e − x −1 −  2 a

be
sin 2 b  t 
 x 2  x2
  ex − x − 1 − 2  a 
  2 ∴ I=−  log | a + b cos x | + a + b cos x  + C

Tu
= lim × b2
x→ 0 x2 x2  
e − x −1 −
x
2
2 ⇒ α=−
x2 b2

ou
x2 4. (c) Given, g(x) = ∫ f ()
2x t
dt, x > 0
ex − x − 1 −
= lim 2  0 form x t
 
0 

(Y
x→ 0 x2 ⇒ g′(x) =
f (2x) d
(2x) −
f (x) d
(x)
Q lim sin x
= 1
2x dx x dx

 x→0 x  (Using Leibnitz Rule for Differentiation)
on
ex − 1 − 0 − x f (2x) f (x)
= lim  0 form = (2) −
  2x x
x→ 0 2x 0 
f (2x) − f (x)
ex − 1 =
pi

= lim x
x→ 0 2 f (x) − f (x)
= (Q f (2x) = f (x))
ex − 1
m

1
= lim × x x
2x→0 x ⇒ g′(x) = 0
ha

1 ex − 1 ⇒ g(x) is constant function


= lim × lim x
2 x → 0 x x→ 0
5. (b) We have,
=0
| x + 1|
C

3
=∫ dx
2. (b) Given, | f ′(x) |≤ 5 for all x 1| x − 2| + | x − 3|
1 1 1 2 x −1 3 x −1
⇒ ∫0 − 5dx ≤ ∫0 f ′(x)dx ≤ ∫0 5 dx = ∫1 − x + 2 − x + 3 dx + ∫2 x − 2 − x + 3 dx
dy

⇒ − 5 ≤ f()
1 − 0≤ 5 (Q| x | = x, x ≥ 0, − x, x < 0)
⇒ − 5 ≤ f()1 ≤5
u

2x −1 3
=∫ dx + ∫2(x − 1)dx
⇒ 1 ∈ [− 5, 5]
f() 1 − 2x + 5
St

sin 2x 3
3. (c) Let I = ∫ dx 1 2− 2x + 2  x2 
(a + b cos x)2 =− ∫
2 − 2x + 5
1
dx + 
 2
− x
2
2 sin x cos x
⇒ I=∫ dx 1 2− 2x + 5 − 3
dx +  − 3 − (2 − 2)
9
2 ∫1 − 2x + 5
(a + b cos x)2 =−
2 
Put a + b cos x = t
t−a 1  2− 2x + 5 2 3 3
2  ∫1 − 2x + 5 ∫1 − 2x + 5 dx +
⇒ cos x = ⇒ − b sin x dx = dt =− dx −
b 2
t − a 1
I = − ∫  1 2
dx + log [− 2x + 5]1  +
2 3 3
2  ∫1
2
 dt =−
b  b  t2 2  2
2 t−a 3
= − 2 ∫ 2 dt = log 3 + 1
b t 4
WB JEE (Engineering) Solved Paper 2021 35

1 1
= [log e t − log e (t + 1)]1t
2 
2
 
2
2
6. (c) ∫  x + 1  +  x − 1  − 2 dx  t 
t

1  x − 1   x + 1
 =  log e
− 
2  t + 1 1
1 1
 t 1
2 x + 1 x − 1  2  2 =  log e − log e 
= ∫  −   dx  t + 1 2
1  x − 1 x + 1 
−   2t
2 = log e
t +1

)
1

be
2 
4x  2t 3
∴ =
= ∫  2  dx
− 1 2t + 1 2
1 x

⇒ 4t = 3t + 3

Tu
2
1 ⇒t = 3
0
 4x  4x  2
Now, ex − 1 = t
= ∫  2  dx − ∫  2  dx

ou
1 x − 1  0  x − 1 ⇒ ex − 1 = 3

2
1
⇒ ex = 4
⇒ x = log 4

(Y
0 2
x x
=−4 ∫ 1 − x 2 dx + 4∫ 1 − x 2 dx 8. (c) Equation of normal
1 0

2 −1
Y − y= (X − x)
on
1
dy / dx
= 2[log(1 − x 2)]0 1 − 2 [log(1 − x 2)]02

2
When y = 0
G =  x + y , 0
dy
pi

= − 2 log 1 −  − 2 log 1 − 


1 1 Then,
  dx 
4  4
m

3 4 According to the question,


= − 4 log = 4 log dy dy
4 3 x+ y = 2x ⇒ y = x
ha

x dx dx
1
7. (c) Let I = ∫ e x
−1
dx ⇒ ∫ y dy = ∫ x dx
loge 2
y2 x 2 y2 − x 2
C

Put ex − 1 = t ⇒ ex = t + 1 ⇒ = + C ⇒ =C
2 2 2
e x dx = dt
⇒ which is hyperbola.
dy

dt dt
⇒ dx = x = 9. (b) We know that the equation of an ellipse
e t +1
x2 y2
When x = x, t = e − 1 x whose centre is at origin is + =1
u

a2 b2
and when x = log e 2, t = eloge 2
−1= 2−1 =1
St

Differentiate both sides w.r.t x, we get


t 2x 2y dy
1 + 2 =0
I= ∫ t(t + 1) dt a2 b dx
1
2y dy 2x
1 A B ⇒ =− 2
= + b 2 dx a
t(t + 1) t t +1
y dy b2
1 = A(t + 1) + B()
t ⇒ =− 2
x dx a
t = 0, A = 1
Again differentiate both sides w.r.t x, we get
t + 1 = 0 ⇒ t = − 1, B = − 1
y d  dy  d  y

1 1
= −
1   + y′   =0
x dx dx   dx  x 
t(t + 1) t t + 1
xy′ − y × 1 
y ′′ + y′ 
y
t
1 1  ⇒  =0
⇒ I = ∫ −  dt x  x2 
1
 t t + 1
36 WB JEE (Engineering) Solved Paper 2021

xy′− y 
y ′′ + y′ 
y
⇒  =0 13. (b) Given, radius of circular track is 10 m.
x  x2 
⇒ xyy ′′ + x(y′)2 − yy′ = 0
y
Which is the required differential equation.

10. (a) Given, x dy + y = x f (xy) C


θ
P
dx f ′(xy) r
d f (xy)
⇒ (xy) = x

)
dx f ′(xy)

be
f ′(xy)
⇒ d(xy) = x dx
f (xy) dθ
and speed of man = 10 m/sec. i.e. v = r = 10

Tu
f ′(xy) dt
⇒ ∫ f (xy) d(xy) = ∫ x dx y
Now, tan θ =
x 2 r

ou
⇒ log | f (xy) | = + C ⇒ y = r tan θ
2
dy dθ
x 2
⇒ = r sec2 θ ⋅
⇒ | f (xy) | = e 2 + C dt dt

=
x2
ke 2 (Y = sec θ ⋅ r
2 dθ
dt
= sec2 60° ⋅ (10)
on
11. (d) − y = (x − 1) − 1 2
= (2)2 × 10
y = 1 − (x − 1)2 = 40 m/sec.
pi

Hence the required area is


1 14. (c) Let B travels x units,
1
 (x − 1)3  v = u + at
m

∫ [1 − (x − 1) ] dx =  x − 3 
2

0 0 According to the problem,


f ′ t = f (t + m)
ha

 (− 1)3 
= 1 − −  f′ − f m
 3  ⇒ =
f t
1 2
C

=1 − = 2
3 3  f 
⇒ t 2 = m2   ...(i)
Hence, y =
2
x will divide the entire area in 2 equal  f′ − f 
dy

3 1
parts. Again, n + x = f (t + m)2
2
5
1 1
u

12. (c) ∫ max {x 2 , 6 x − 8} dx ⇒ n + f ′ t 2 = f (t + m)2 ...(ii)


2 2
St

0
2 4 5 Now from Eqs. (i) and (ii), we get
= ∫ x dx + ∫ (6 x − 8)dx + ∫ x dx
2 2
m 2( f ′ − f )
=
0 2 4
n mff ′
x 3 2
 6x  2  x3 
4 5
1
=  +  − 8x +   ⇒ ( f ′ − f)n = ff ′ m2
 3 0  2 2  3 4 2
8 125 64 15. (b) Given, α + 3 β is collinear with γ
= + [3x 2 − 8 x]42 + −
3 3 3
∴ α + 3β = λ1 γ
69
= (48 − 32 − 12 + 16) + λ α
3 ⇒ β= 1γ− ...(i)
3 3
= 20 + 23
and β + 2γ is collinear with α
= 43
∴ β + 2γ = λ 2α
WB JEE (Engineering) Solved Paper 2021 37

⇒ β = λ 2α − 2γ ...(ii) 1
⇒ a = (36) 2 = 33 = 27
From Eqs. (i) and (ii), we get
λ1 α
γ − = λ 2α − 2γ 18. (a) Given, h(p) = 1 {h(p + 1) + h(p − 1)}
3 3 2
λ for every p = 1, 2, ..., 99
α  λ 2 +  = γ  1 +
 2
1

 3  3  2h(p) = h(p + 1) + h(p − 1)
1 λ ⇒ h(p − 1), h(p), h(p + 1) are in AP.
⇒ λ 2 + = 0 and 1 + 2 = 0
h(100) = 20

)
3 3 Now,
λ1

be
1
⇒ λ 2 = − and =−2 ⇒ h(0) + 99d = 20
3 3 ⇒ 5 + 99d = 20 (Qh(0) = 5)
1 15 5
⇒ λ1 = − 6 and λ 2 = − ⇒ d= =

Tu
3 99 33
α ∴ 1 = h(0) + d
From Eqs. (i) and (ii), β = − 2γ − h()
3 5
= 5+ = 5 + 0.15

ou
 α 33
∴ α + 3β + 6γ = α + 3  − 2γ −  + 6γ
 3 =5.15
=0

16. (c) Given, f (x) = | x − 1 |, x ∈ R.


2

The graph of f (x) is clearly, from graph.


(Y
19. (d) Let z = e iθ
Also, let w =
e iθ
=
1
1 − e 2iθ e − iθ − e iθ
on
f(x)=x2–1 1 i
= =
− 2 ⋅ i sin θ 2 sin θ
pi

⇒ w is purely imaginary
⇒ Locus of point is Y-axis.
m

x=–1 x=1
20. (c) We have, z 2 = w2
ha

⇒ z 2 − w2 = 0
f (x) has a local minima at x = ± 1 and has local
maxima at x = 0 ⇒ (z − w)(z + w) = 0
⇒ z = w, z = − w
C

17. (d) Given,


2 3 5 ⇒ |z|= |w|
log b a + log c b + log a c = 3
3 5 2 ∴B⊂ A
dy

2 log a 3 log b 5 log c


⇒ + + =3 21. (c) Given, x 2 − 6 x − 2 = 0
3 log b 5 log c 2 log a
⇒ x n − 2(x 2 − 6 x − 2) = 0
u

2 3 5
log b a + log c b + log a c
⇒ x n − 6 x n − 1 − 2x n − 2 = 0
St

Since, A.M = 3 5 2
3
3 When, n = 10,
= =1
3 Then, x10 − 6 x 9 − 2x 8 = 0
1
⇒ x10 − 2x 8 = 6 x 9
G.M =  log b a ⋅ log c b ⋅ log a c 
2 3 5 3
and
3 5 2  Since, α and β are roots of (i).
2 log a 3 log b 5 log c
= × × =1 ∴ α10 − 2α 8 = 6α 9
3 log b 5 log c 2 log a
and β10 − 2β 8 = 6β 9
∴ A.M. = G.M

2 log a
= 1 ⇒ a2 = b3 ∴ (α10 − β10) − 2(α 8 − β 8) = 6(α 9 − β 9)
3 log b
38 WB JEE (Engineering) Solved Paper 2021

⇒ a10 − 2a 8 = 6a 9 (Qa n = α n − β n ) 27. (c) Given,


a10 − 2a 8 1 0 0 
⇒ =3
2a 9 A =  0 cos t sin t 
 
 0 − sin t cos t 
22. (d) For x ∈R, x ≠ − 1
Since, det(A − λI 3) = 0
(1 + x)2016 + x(1 + x)2015 + x 2(1 + x)2014 + K 1−λ 0 0
2016
⇒ cos t − λ =0
+ x = ∑ ai ⋅ x 0 sin t
2016 i

)
− sin t cos t − λ

be
i=0 0
Here, coefficient of x17 ⇒ (1 − λ) {(cos t − λ)2 + (sin t)2} = 0
2015 2014
= 2016
C17 + C16 + C15 + K + 1999
C0 ⇒(1 − λ) (cos2 t + λ2 − 2λ cos t + sin2 t) = 0

Tu
2015 2014 1999
= 2016
C1999 + C1999 + C1999 + K + C1999 ⇒ (1 − λ) (λ2 − 2λ cos t + 1) = 0
n+1
(Q Cr + Cr − 1 =
n n
C r) ⇒ λ2 − 2λ cos t + 1 − λ3 + 2λ2 cos t − λ = 0

ou
(2017)!
= 2017
C2000 = ⇒ − λ3 + λ2(1 + 2 cos t) − λ(1 + 2 cos t ) + 1 = 0
(2000)! (2017 − 2000)!
⇒ λ3 − λ2(1 + 2 cos t) + λ(1 + 2 cos t ) − 1 = 0

(Y
2017!
= ∴ λ1 + λ 2 + λ 3 = (1 + 2 cos t )
(2000)! (17)!
⇒ 1 + 2 cos t = 2 + 1 (Given)
23. (c) Given, word “EQUATION” 1 π π
on
⇒ cos t = ⇒t = − ,
Here, consonants—Q, T , N 2 4 4
and Vowel—E , U , A, I , O
∴Number of word = 5C3 × 3! × 3C2 × 3!
28. (c) Given A and B be two non-singular skew
pi

symmetric matrices.
= 10 × 6 × 3 × 6 Such that AB = BA
m

= 1080 ⇒ AT = − A and BT = − B ...(i)


24. (c) According to the question, Since matrices are non-singular
ha

Consider, x1 + x 2 + x 3 + x 4 = 10, where x i ≥ 2 ∴ order of these matrices are even.


⇒(x1 − 2) + (x 2 − 2) + (x 3 − 2) + (x 4 − 2) = 2 Now, A2B2(AT B)− 1 (AB− 1)T
C

⇒ y1 + y2 + y3 + y4 = 2, where yi ≥ 2, = A2B2(− AB)− 1 (AB− 1)T


(where, x1 − 2 = y1 , x 2 − 2 = y2, x 3 − 2 = y3 and
x 4 − 2 = y4 ) = − A2B2(B− 1 A− 1) (BT)− 1 AT
dy

∴Number of ways = 2 + 4 − 1 C4 − 1 = 5C3 = 10 = − A2(B2B− 1) A− 1 (− B)− 1 (− A)


= − A2B ⋅ A− 1 B− 1 A
25. (a) We have,
u

= − A2B(BA)− 1 A
1! + 2! + 3! + 4! + 5! + K + 99!
St

Since unit digit of 5!, 6!, 7!, ... 99! are zero. = − A2B(AB)− 1 A (QAB = BA)
∴1! + 2! + 3! + 4! + 5! + K + 99! −1 −1
= − A BB2
A A
= 1 + 2 + 6 + 24 = 33 = − A2(BB− 1) (A− 1 A)
∴Unit digit = 3
= − A2
26. (c) Given,
(01 2) M = (1 0 0) ...(i) 29. (d) We have, an = ar n − 1
and (3 4 5)M = (01 0) ⇒ log a n = log(ar n − 1)
⇒ (6 810) M = (0 2 0) ...(ii) ⇒ log a n = log a + (n − 1) log r
On subtracting Eq. (i) from Eq. (ii), we get log a n log a n + 1 log a n + 2
(6 810) m − (0 1 2) M = (0 2 0) – (1 0 0) Now, log a n + 3 log a n + 4 log a n + 5
⇒ (6 7 8) M = (−1 2 0) log a n + log a n + log a n +
6 7 8
WB JEE (Engineering) Solved Paper 2021 39

log a + (n − 1) log r log a + n log r 2 6


P(x = 2) = 8 C2    
1 1

= log a + (n + 2) log r log a + (n + 3) log r  2  2
log a + (n + 5) + log r log a + (n + 6) log r 8
= 8 C2  
1
log a + (n + 1) log r  2
log a + (n + 4) log r 8×7 1 7
= × =
log a + (n + 7) log r 2 × 1 256 64
On applying R2 → R2 − R1 , R3 → R3 − R1

)
34. (b) Given,
log a + (n − 1) log r log a + n log r log a + (n + 1) log r

be
Sn = cot − 1 2 + cot − 1 8 + cot − 1 18
3 log r 3 log r 3 log r
+ cot − 1 32 + K to nth term.
6 log r 6 log r 6 log r
= cot − 1 2 × 12 + cot − 1 2 × 22 + cot − 1 2 × 32

Tu
= 0 (Since R2 and R3 are proportional)
+ cot − 1 2 × 42 + K
30. (a) Given, (A ∩ C) ∪ (B ∩ C ′) = φ ∴ nth term = cot − 1 2n2

ou
⇒ A∩C=φ ...(i)
tn = tan− 1  2 
1
and B ∩ C′ = φ ...(ii) ⇒
 2n 

(Y
From Eqs. (i) and (ii), we get
 (2n + 1) − (2n − 1) 
A∩B=φ = tan− 1  
 1 + (2n + 1) (2n − 1) 
31. (b) Since, = tan− 1 (2n + 1) − tan− 1 (2n − 1)
on
n(A) ≠ n(T)
∴ Sn = tan− 1 (2n + 1) − tan− 1 ()
1
and n(A) ≠ n(U)
∴There does not exist bijective mapping between A and ∴ lim Sn = tan− 1 (∞) − tan− 1 ()
1
pi

n→ ∞
T, U.
π π π
= − =
m

32. (d) Even numbers on die are 2, 4, 6. 2 4 4


and odd numbers on die are 1, 3, 5.
35. (a) We have vertices of parallelogram O(0, 0),
ha

3 1
∴ P(even) = = A(a cos θ, b sin θ) and B(a cos θ, − b sin θ)
6 2
A (a cos θ,b sin θ)
3 1
and P(odd) = =
C

6 2
O(0,0) P
According to the question,
4 8 12
dy

1 1 
+   × +   × +  
1 1 1 1 1
P(A(wins)) = × + K∞
2  2 2  2 2  2 2
5 9 3
B (a cos θ,–bsin θ)
1 1 
+   +   +   + K ∞
1 1
u

=
2  2  2  2
St

1 1 Here, OA = OB
2 1 16 8
= = 2 = × = ∴Area of parallelogram = 2 × area of ∆OAB
 1
4 15 2 15 15
1−  = 2 ×  × OP × AB
1
 2 16
2 
33. (a) We have, 1
= 2 × × a cos θ × 2b sin θ
Mean, np = 4 2
= 2 × ab (sin θ cos θ)
and variance, npq = 2
= ab sin 2θ
npq 2 1 π
∴ = = ∴Area will be maximum when θ = .
np 4 2 4

1 1
q = ∴ p = and n = 8 π
⇒ Maximum Area = ab when θ =
2 2 4
40 WB JEE (Engineering) Solved Paper 2021

36. (a) Let point B = (2p , 0) ⇒ {(β + α)2 − (β − α)2}2 = 16c 2

Y ⇒ (4h2 − 4k2)2 = 16c 2


⇒ h2 − k2 = c 2
A (0,4)
M ∴Locus of the mid point P of MR is (x 2 − y 2)2 = c 2
B
O(0,0) P
X 38. (a) Given equation of parabola
6 y = 2a 3 x 2 + 3a 2 x − 12a

)
R
6 y + 12a = 2a  a 2 x 2 + ax 

be
3

 2 
∴Coordinates of M are (p , 2) 
= 2a  a x + 2⋅ ax +  − a
2 2 3 9 9

Tu
4−0 2  4 16  8
Now, Slope of AB = =−
0 − 2p p 2
a = 2a  ax + 
105 3
p ⇒ 6y +
∴Slope MR = 8  4

ou
2
p Let vertices are (h, k)
∴Equations of MR = y − 2 = (x − p) −3 − 35a
2 ∴ h= and k =
On putting x = 0, we get
y − 2= −
p2
2
⇒ y = 2−
p2
2
(Y ∴ hk =
4a
105
64
16
on
∴locus of the vertices is
 p2 
∴Coordinates of R is  0, 2 −  105
 2 xy =
64
pi

 p2 
 2− + 2 39. (b) Given, equation of ray of light x + 3y = 3
 p 2 
∴Coordinates of P is ,
m

2 2  x+ √3y =√3
 
  B
ha

p p 2
i.e.  , 2 − 
2 4 X–axis
A
C

∴y = 2 − x ⇒ y + x = 2
2 2

Y B′
37. (b) x–y=0 On putting y = 0, we get x = 3
dy

B
∴Coordinates of A are ( 3, 0)
Now, let B(0, 1) then B′ = (0, − 1)
u

O
X y−0 x− 3
∴Equation of AB′ is =
St

A 0+1 3−0
⇒ 3y = x − 3
x+y=0
Let coordinates of A are (α, − α) and coordinates of B 40. (a)
are (β, β) A
∴β + α = 2h and β − α = 2k
1
Now, Area of ∆AOB = × OA × OB M O
x2+y2=4
2 (–4,0)
1 B
= 2α 2
2β 2 = |αβ | = c
2
⇒ α 2β 2 = c 2 Given, equation of circle x 2 + y 2 = 4
⇒ 16α 2β 2 = 16c 2 Centre = (0, 0) and radius = 2
WB JEE (Engineering) Solved Paper 2021 41

∴ MA = (4)2 − (2)2 = 12 = 2 3 44. (b) According to the question, we have.


∴Area of quadrilateral MAOB
= 2 ×area of ∆MAO
C
1
= 2 × × MA × OA r r
2
=2 3×2 r1 r2
= 4 3sq. units.

)
C2
C1

be
41. (b) Given, equation of parabola y = x 2

We know that for y 2 = 4ax


CC1 = r + r1

Tu
if we draw a normal from (h, 0) on it then condition
and CC2 = r + r2
for three normal is h > 2a
1 ∴ CC2 − CC1 = (r + r2) − (r + r1) = r2 − r1
∴d> ⇒ Locus of centre is Hyperbola.

ou
2

42. (b) A 45. (c) Let direction cosines of line is (l , l , l).


∴ l2 + l2 + l2 = 1
O
B

According to question the coordinates of A and B are


(Y ⇒ 3l 2 = 1 ⇒ l =
1


3
1 1 1 
on
(0, b , c) and (a , 0, c) respectively. ∴ Direction cosines of line is  , , 
 3 3 3
Now, perpendicular vector OA × OB
$i $j k$ Now, equation of a line passing through P(2, − 1, 2)
pi

x − 2 y+1 z− 2
= 0 b c = = =k … (i)
1/ 3 1/ 3 1/ 3
m

a 0 c
⇒ x = k + 2, y = k − 1and z = k + 2
= $i(bc − 0) − $j(0 − ac) + k$ (0 − ab) Since line (i) meets the plane
ha

= bc$i + ac$j − abk$ 2x + y + z = 9 at Q


∴ 2(k + 2) + (k − 1) + (k + 2) = 9
∴Required equation of plane is bcx + acy − abz = 0
⇒ 2k + 4 + k − 1 + k + 2 = 9
C

43. (b) Given, equation of ellipse x 2 + 2y2 = 4 ⇒ 4k + 5 = 9


x2 y2 ⇒ 4k = 4 ⇒ k = 1
dy

⇒ + =1 … (i)
4 2 ∴Coordinates of Q are (3, 0, 3)
Y ∴ PQ = (3 − 2)2 + (0 + 1)2 + (3 − 2)2
u

= 1+1+1= 3 units.
St

P (2 cos 60°
2 sin 60°)  5x + 12 1 − x 2 
60° 46. (b) Given, y = sin− 1  
X  13 
O
 5x 12 1 − x 2 
= sin− 1  + 
13 13
 
5
Now, equation of auxiliary circle is x 2 + y 2 = 4. Let sinθ1 = and cosθ2 = x
13
∴Point on the auxiliary circle with eccentric angle ∴ y = sin− 1 (sinθ1 ⋅ cosθ2 + cosθ1 ⋅ sinθ2)
60º is P(2cos 60º , 2sin 60º)
= sin− 1 {sin(θ1 + θ2)}
i.e., P(1, 3)
5
= θ1 + θ2 = sin− 1 + cos− 1 x
13
42 WB JEE (Engineering) Solved Paper 2021

1 Clearly f (x) is increasing in [0, 1] and decreasing in


⇒ y1 = −
1 − x2 [2, 4]
− x ∴ f (x) is neither continuous and nor differentiable
and y2 = in D.
(1 − x ) 1 − x 2
2
∴Rolles theorem is not applicable to f in D
⇒ y2(1 − x 2) = xy1
51. (b) f (x) be a continuous periodic function with
⇒ y2(1 − x 2) − xy1 = 0 period T

)
∴ a = 1, b = − 1 a+ T
I= ∫ f (x) dx

be
∴ (a , b) = (1, − 1) Given,
a
47. (c) Given, 2 f (x) + 3 f (− x) = 15 − 4 x … (i) T
I= ∫ f (x) dx

Tu
On replacing x by − x in Eq. (i), we get
0
2 f (− x) + 3 f (x) = 15 + 4 x … (ii)
Hence, I does not depend on ‘a’.
On solving Eqs. (i) and (ii), we get

ou
1 t
f (x) = 4 x + 3 52. (c) We have, b = ∫ e dt
∴ f(2) = 4 × 2 + 3 = 11 0
t +1

e− t

(Y
a
48. (c) Given, f1 (x) = x, f2(x) = 2 + log e x Let I= ∫t − a −1
dt
Now, Let h(x) = f2(x) − f1 (x) a −1

= (2 + log e x) − x a
e − ( a + a − 1 − t)
on
= 2 + log e x − x
I= ∫a + a −1 − t − a −1
dt
a −1
Here, h(0+ ) < 0, h()
1 > 0, h(e) > 0 and h(e 2) < 0 and
 b b 
value of h(x) for all x ≥ e 2 is negative.
pi

Q ∫ f (x) dx = ∫ f (a + b − x) dx 
∴h(x) = 0 has two roots is (0, 1) and (e , e 2)  a a 
m

a t − 2a + 1
e
49. (c) Given equation, 6x + 8x = 10x I= ∫a − 2− t
dt
a −1
ha

x x
⇒  6  +  8  =1
    put t − (a − 1) = x ⇒ dt = dx
 10   10 
x x when t = a, x = 1 and t = a − 1 ⇒ x = 0
 3 +  4  = 1
C

⇒ 1 + a − 1 − 2a + 1
    ex
 5  5 ∴ I= ∫ a − 2 − x − a + 1 dx
3 4 0
= sinθ ⇒ = cosθ
dy

Let
ex ⋅ e− a
1
5 5
∴ (sinθ)x + (cosθ)x = 1
⇒ I= ∫ − (x + 1) dx
0
u

it is possible only x = 2 1
ex
∴given equation has exactly one real root. ⇒ I = − e− a∫ dx
St

0
x +1
50. (b) We have, f : D → R where D = [0, 1] ∪ [2, 4]  1 et 
if x ∈ [0, 1] ⇒ I = − e − a(b) Q ∫ dt = b 
 x, +
f (x) =   0 t 1 
4 − x , if x ∈ [2, 4]
⇒ I = − be − a
Y
0,2 53. (a) We have,
2
f (x) = log e (1 + e10 x ) − tan− 1 (e 5x )
1 0,1
10e10 x 5e 5x
X′ X ⇒ f ′(x) = −
O (1,0) (2,0) (3,0) (4,0) 1+ e 10 x
1 + e10 x
10e10 x − 5e 5x
⇒ f ′(x) =
1 + e10 x
Y′
WB JEE (Engineering) Solved Paper 2021 43

10 − 5 5 2 3/ 2 2
⇒ f ′(0) = = = 5× [y ]0
1+1 2 3
10 3/ 2
We know that, = [2 − 0]
f (x + ∆x) − f (x) = f ′(x) dx 3
10
5
⇒ f (x + ∆x) − f (x) = × 0.2 [Qdx = 0.2] = ×2 2
2 3
20
⇒ ∆f (x) = 0.5 = 2sq unit.
3

)
54. (b) We have,

be
57. (c) We have,
f (x) = 1 + x
a(α × β) + b(β × γ) + c(γ + α) = 0
f (c) = 1 + c
Clearly a(α × β), (β × γ ) and (γ + α) are coplanar

Tu
⇒ c =1 + c [Q f (c) = c] vector∴α , β and γ are also coplanar
⇒ (c − 1)2 = ( c)2
58. (b) Given equation
⇒ c 2 − 2c + 1 = c

ou
y 2 = 2x 3
⇒ c 2 − 3c + 1 = 0 dy
⇒ 2y = 6x2
3± 5 dx
⇒ c=


2
∴ f has unique fixed point in [1, ∞]

55. (c) Let L = lim  3x − 1 
4x
(Y ⇒

 dy 
dy 3x 2
dx
=
4
3h2
on
⇒   =
 dx  ( h , k) k
x → ∞ 3x + 1 
 3x − 1  Slope of tangent is perpendicular to line
lim  − 1 4x
pi

L=e
x → ∞  3x + 1  4 x = 3y
 3x − 1 − 3x − 1 
 3h2   4 
∴     = −1
m

lim   4x
L=e
x→∞  3x + 1   k   3
− 2× 4 ⇒ 4h2 = − k
ha

lim …(i)
x→∞ 1
3+
L=e x and k = 2h
2 3
…(ii)
From Eqs. (i) and (ii)
C

L = e− 8/ 3
(4h2)2 = 2h3
2
56. (a) Given curve, y = 4 x 2, y = x and y = 2 ⇒ 16h4 = 2h3
dy

9
1
Graph of given curve ⇒ h=
y=4x2 8
u

2
k = − 4 
–1 1 1
,2 2 ∴
,2
√ 2 y=2 √ 2 y=x /9  8
St

1 −1
(–3/√2,2)
0,2 (3/√2,2) ⇒ k=−4× =
64 64
1 − 1
∴ (h, k) =  ,  only
 8 16 

59. (d) Given, (bc + ca + ab)6


6!
General term = (bc)p (ca)q(ab)r
Area of shaded region p!q !r !
2 y 6!
= 2∫  3 y −  dy = (a)q + r (b)p + r (c)p + q
0 2  p!q !r !
0 5 y Coefficient of a 3b 4 c 5 in the expansion (bc + ca + ab)6
= 2∫ dy
2 2 ∴ q + r = 3, p + r = 4, p + q = 5
44 WB JEE (Engineering) Solved Paper 2021

= 2(p + q + r) = 12 1 b2 c2
=p+ q+ r=6 = (a + b + c + 10) 1 b + 10
2 2 2 2
c2
∴p = 3, q = 2, r = 1 1 b 2
c + 10
2

= 3
6! 6! 
∴Coefficient of a 3b 4 c 5 is 
3! 21
! !  3! 3!  R2 → R2 − R1 and R3 → R3 − R1
1 b2 c 2
60. (c) Given a , b , c are in G.P and log  5c  , log  7b  . = (a + b + c + 10) 0 10
2 2 2
0
 2a   5c 

)
log   and A.P
2a 0 0 10

be
 7b 
= (a 2 + b 2 + c 2 + 10) (100)
∴b 2 = ac
QIf is divisible by 100.
2log   = log   + log  

Tu
7b 5c 2a
and
 5c   2a   7b  62. (b) We have,
2 S = {(x , y) : y = x + 1, 0 < x < 2}
49b 5c
⇒ = For reflexive (x , x) ∈ S

ou
25c 2 7b
x = x + 1 ∉S
⇒ (7b)3 = (5c)3 ∴S is not reflexive
7 ∴S is not equivalence relation

(Y
⇒ 7b = 5c ⇒ c = b
5 T = {(x , y) : x − y is an integer}
and b 2 = ac For reflexive (x , x) ∈ T
x = x − x = 0∈R
b 2 = a  b 
on
7
⇒ T is reflexive
5 
For symmetric
7
⇒ b= a (x , y) = x − y is an integer
pi

5
(y, x) = y − x is also integer
5b 7
Sides are , b, b ∴T is symmetric
m

7 5
For Transitive
∴ a , b , c are the length of the sides of scalene
(x , y) ∈ T, (y, z) ∈ T a⇒(x , z) ∈ T
ha

triangle.
(x , y) = x − y is an integer
61. (b) We have, (y, z) = y − z is also integer
C

a 2 + 10 ab ac ∴(x − y) + (y − z) = x − z is an integer
ab b 2 + 10 bc ∴ (x , z) ∈ T
c 2 + 10
dy

ac bc Hence, T is an equivalence relation.

a(a 2 + 10) ab 2 ac 2 63. (d) Given plane,


1 P1 = lx + my = 0and P2 = z = 0
u

= a b 2
b(b + 0)
2
bc 2
abc Plane through common line of P1 and P2
a c 2
b c 2
c(c + 10)
2
St

∴ P3 = lx + my + nz = 0
Taking common a , b , c from R1 , R2 and R3 angle between P1 and P3 = α
respectively
l 2 + m2
a 2 + 10 b2 c2 ∴ cosα =
abc l + m + n2 l 2 + m2
2 2
= a2 b 2 + 10 c2
abc l 2 + m2
a2 b2 c 2 + 10 ⇒ cosα =
l + m2 + n2
2
Applying C1 → C1 + C2 + C3, we get
a 2 + b 2 + c 2 + 10 b2 c2 l 2 + m2
⇒ cos2 α =
l + m2 + n2
2
= a 2 + b 2 + c 2 + 10 b 2 + 10 c2
a + b + c + 0
2 2 2 2
c + 10
2 l 2 + m2 + n2 n2
b ⇒ sec2 α = =1 + 2
l + m
2 2
l + m2
WB JEE (Engineering) Solved Paper 2021 45

⇒ (sec2 α − 1) (l 2 + m2) = n2 |z + i |− |z − 1|= 0


⇒ n = tan α( l + m )
2 2 2 2 ⇒ |z + i |= |z − 1|
x 2 + (y + 1)2 = (x − 1)2 + y 2
= n= ± (l 2 + m2) tanα
⇒ x 2 + y 2 + 2y + 1 = x 2 − 2x + 1 + y 2
∴Equation becomes
= y=− x …(i)
lx + my ± z tanα l 2 + m2 = 0

64. (c) Given ellipse

)
x 2 + 2y 2 − 6 x − 12y + 20 = 0

be
and 2x 2 + y 2 − 10 x − 6 y + 15 = 0
The point of intersection of the given ellipse is y=–x

Tu
(x 2 + 2y 2 − 6 x − 12y + 20) +
λ(2x 2 + y 2 − 10 x − 6 y + 15) = 0
∴ | z | = 2 ⇒ x 2 + y2 = 4 …(ii)

ou
(1 + 2λ) x 2 + (2 + λ) y 2 − (6 + 10λ) x
From Eqs. (i) and (ii)
− (12 + 6λ) y + 20 + 5λ = 0
x2 + x2 = 4
The points lie on circle
∴Equation becomes circle
∴ 1 + 2λ = 2 + λ ⇒ λ = 1
∴Equation of circle is
(Y ⇒
and

2x 2 = 4 x = ±
y=m
z = x + iy
2
2
on
3x 2 + 3y 2 − 16 x − 18 y + 35 = 0 z=± 2m 2i

∴Centre of circle is  , 3 z=±


8 21
( m i)
3 
pi

z= ( − i)
21
π / 3 sin x
65. (a) We have, I = ∫ z= 2(− 1 + i)
m

dx and
π /4 x
sin x 67. (d) We have,
Let f (x) =
ha

x x 3x + 2 2x − 1
f (x) is decreasing functions 2x − 1 4x 3x + 1 = 0
⇒ f (π / 3) < f (x) < f (π / 4) 7 x − 2 17 x + 6 12x − 1
C

3 3 2 2
⇒ < f (x) < R3 → R3 − 3R1 − 2R2, we get
2π π
x 3x + 2 2x − 1
dy

3 3 π /3 2 2 π /3
2π ∫π / 4 π ∫π / 4
⇒ dx < I < dx 2x − 1 4x 3x + 1 = 0
0 0 0
3 3  π π
u

2 2
⇒  − <I< (π / 3 − π / 4)
2π  3 4  π ∴infinite value of x is possible.
St


3
<I<
2 68. (b) We have
8 6 7... 7
77 (22 time 7)
7
66. (b, c) have | z + i | − | z − 1 | = | z | − 2 ⇒ 7 77
is an odd number
| z | − 2⇒| z | = 2represent a circle. ∴ 7( 2m + 1) = 72m × 7
= (72)m × 7
= (49)m × 7
|z|=2
= (48 + 1)m × 7
= 71
( + 48k)
= 7 + (48 × 7) k
∴ remainder = 7
46 WB JEE (Engineering) Solved Paper 2021

2 2 2
α β r2
⇒ Locus of R is  x −  +  y −  =
dx
69. (c, d) (a) I1 = ∫4+ x 2  2  2 4
−2

It is possible to put x = tan t Which represents a circle the locus of R is circle.


Option (a) is incorrect. 
1 72. (a) Let I = lim  n + n
+
n
n→ ∞ 3
 n (n + 4)3 (n + 8)3
(b) I 2 = ∫ x + 1 dx
2

0
n 
It is possible to put x = tan t + …+

)

[n + 4 (n − 1)]3 

be
∴ Option (b) is also incorrect
n −1
Hence, option (c) and (d) are correct. n
⇒ I = lim
n→ ∞

(n + 4r)3

Tu
70. (a, c) Let equation of plane be r=0
x y z n −1
+ + =1 1 n n
a b c ⇒ I = lim
n→ ∞ n

r = 0 (n + 4r)
3

ou
Centroid =  , , 
a b c
n −1
 3 3 3 1 1
a b c
⇒ I = lim
n→ ∞ n
∑ 3/ 2
∴ = 1, = r, = r 2 r=0 1 + 4r 

(Y
 
3 3 3  n
⇒ a = 3, b = 3r c = 3r 2 1 dx
Equation of plane be
⇒ I= ∫0 (1 + 4 x )3/ 2
on
x y z
+ + =1  (1 + 4 x)− 3/ 2 + 1 
1
3 3r 3r 2 ⇒ I= 
Now plane is passes through (5, 5, − 12)  4(− 3 / 2 + 1)  0
pi

5 5 12 −1
∴ + − =1 ⇒ I= [(1 + 4 x) −1 / 2]10
3 3r 3r 2
m

2
⇒ 5r 2 + 5r − 12 = 3r 2 − 1 − 1/ 2
⇒ I= [5 − 1]
2
ha

⇒ 2r 2 + 5r − 12 = 0
−1  1
⇒ 2r 2 + 8r − 3r − 12 = 0 ⇒ I= − 1
2  5 
C

= (2r − 3) (r + 4) = 0 − 1 1 − 5 
⇒ I= 
⇒ r = 3 / 2, − 4 2  5 
dy

71. (a) Let the equation of circle is 5 −1 5( 5 − 1) 5 − 5


⇒ I= = =
(x − α)2 + (γ − β)2 = r 2 2 5 10 10
∴ x = α + r cosθ −1≤ x < 0
u

0, if
y = β + r sinθ 73. (a, d) We have, f (x) = 1, if x=0
St

∴ P ≡ (α + r cosθ, β + r sinθ)  2, if 0< x ≤1



θ = (a , b) x
Let R = (h, k) F(x) = ∫ f ()t dt
α + r cosθ β + r sinθ −1
∴ h= ,k =
2 2 0 1

r cosθ = 2h − α and r sinθ = 2k − β


F(x) = ∫ 0 dt + ∫ 2 dt
−1 0
∴ r 2(cos2 θ + sin2 θ) = (2h − α)2 + (2k − β)2
 0, − 1 ≤ x ≤ 0
2 2 2 F(x) = 
α β
=  h −  +  k −  =   2x , 0 < x ≤ 1
r
 2  2  2
WB JEE (Engineering) Solved Paper 2021 47

Graph of i(x) is − (x − 1)2


f ′(x) =
(0,2) 2x(1 + x 2)
f ′(x) is decreasing function ∀ x > 0
1
fmax at x = and fmin at x = 3
(1,0) 3
f   = tan− 1
1 1 1 1
∴ − log
 3 3 2 3

)
π 1
= + log 3

be
Clearly F(x) is continuous in [− 1, 1] 6 4
F ′(x) does not exist at x = 0 1
f( 3) = tan− 1 3 − log 3
74. (b, c) We have, 2

Tu
π 1
log x on  , 3 = − log 3
1 1
f (x) = tan− 1 −
2 
 3  3 4
π 1

ou
1 1 ∴ fmax = + log 3
f ′(x) = − 6 4
1 + x 2 2x
π 1
fmin = − log 3
2x − 1 − x 2

(Y
f ′(x) = 3 4
2x(1 + x 2)
75. (d) We have, f and g be periodic function with
− (x 2 − 2x + 1)
f ′(x) = periods T1 and T2 respectively
2x(1 + x 2)
on
f + g is periodic if T1 = T2
pi
m
ha
C
u dy
St
WB JEE
Engineering Entrance Exam

Solved Paper 2020

)
be
Tu
Physics

ou
Category I (Q. Nos. 1 to 30) the β-particle emitted is (where, me is the
Carry 1 marks each and only one option is mass of an electron and c is the velocity of

(Y
correct. In case of incorrect answer or any light)
combination of more than one answer, 1/4 mark (a) (M x − M y − me ) c 2 (b) (M x − M y + me ) c 2
will be deducted. (c) (M x − M y ) c 2 (d) (M x − M y −2 me ) c 2
on
1. The intensity of light emerging from one of 5. For nuclei with mass number close to 119
the slits in a Young’s double slit experiment and 238, the binding energies per nucleon are
is found to be 1.5 times the intensity of light
pi

approximately 7.6 MeV and 8.6 MeV,


emerging from the other slit. What will be respectively. If a nucleus of mass number 238
the approximate ratio of intensity of an
m

breaks into two nuclei of nearly equal masses,


interference maximum to that of an what will be the approximate amount of
interference minimum? energy released in the process of fission?
ha

(a) 2.25 (b) 98 (c) 5 (d) 9.9 (a) 214 MeV (b) 119 MeV
(c) 2047 MeV (d) 1142 MeV
2. In a Fraunhofer diffraction experiment, a
C

single slit of width 0.5 mm is illuminated by 6. A common emitter transistor amplifier is


a monochromatic light of wavelength connected with a load resistance of 6 kΩ.
When a small AC signal of 15 mV is added to
dy

600 nm. The diffraction pattern is observed


on a screen at a distance of 50 cm from the the base-emitter voltage, the alternating base
slit. What will be the linear separation of the current is 20 µA and the alternating collector
current is 1.8 mA. What is the voltage gain of
u

first order minima?


the amplifier?
St

(a) 1.0 mm (b) 1.1 mm


(a) 90 (b) 640 (c) 900 (d) 720
(c) 0.6 mm (d) 1.2 mm
7. VCC=5V
3. If R is the Rydberg constant in cm −1, then
hydrogen atom does not emit any radiation 500 W
of wavelength in the range of Y
1 4 7 19
(a) to cm (b) to cm
R 3R 5R 5R lB=200 mA
4 36 9 144
(c) to cm (d) to cm
R 5R R 7R

4. A nucleus X emits a β-particle to produce a


nucleus Y . If their atomic masses are Mx and In the circuit shown, the value of β of the
M y respectively, then the maximum energy of transistor is 48. If the supplied base current
2 WB JEE (Engineering) Solved Paper 2020
is 200 µA, what is the voltage at the terminal 13. The bob of a swinging second pendulum (one
Y? whose time period is 2 s) has a small speed v0
(a) 0.2 V (b) 0.5 V (c) 4 V (d) 4.8 V at its lowest point. Its height from this lowest
point 2.25 s after passing through it, is given
8. The frequency ν of the radiation emitted by by
an atom when an electron jumps from one
v 02 v 02
orbit to another is given by ν = k δE, where k (a) (b)
is a constant and δE is the change in energy 2g g
level due to the transition. Then, dimension v2 9v 02
(c) 0 (d)

)
of k is 4g 4g

be
(a) [ML2 T−2 ]
14. A steel and a brass wire, each of length 50 cm
(b) the same dimension of angular momentum
(c) [ML2 T−1 ]
and cross-sectional area 0.005 cm 2 hang from

Tu
a ceiling and are 15 cm apart. Lower ends of
(d) [M−1L–2 T]
the wires are attached to a light horizontal
9. Consider the vectors A = $i + $j − k$ , bar. A suitable downward load is applied to

ou
1 $ $ the bar, so that each of the wires extends in
B = 2 i$ − $j + k$ and C = (i − 2j + 2k$ ). What is length by 0.1 cm. At what distance from the
5 steel wire, the load must be applied?
the value of C ⋅(A × B) ?
(a) 1
(c) 3 2
(b) 0
(d) 18 5
(Y [Young’s modulus of steel = 2 × 10 12 dyne/cm 2
and that of brass = 1 × 10 12 dyne/cm 2 ]
on
(a) 7.5 cm (b) 5 cm
10. A fighter plane, flying horizontally with a (c) 10 cm (d) 3 cm
speed 360 km/h at an altitude of 500 m drops
pi

a bomb for a target straight ahead of it on the 15. Which of the following diagrams correctly
ground. The bomb should be dropped at shows the relation between the terminal
m

what approximate distance ahead of the velocity vT of a spherical body falling in a


target? Assume that acceleration due to liquid and viscosity η of the liquid?
gravity (g) is 10 ms−2 . Also, neglect air drag.
ha

vT vT
(a) 1000 m (b) 50 5 m (a) (b)
(c) 500 5 m (d) 866 m
C

11. A block of mass m rests on a horizontal table h h


with a coefficient of static friction µ. What
dy

minimum force must be applied on the block vT vT


to drag it on the table? (c) (d)
µ µ −1
u

(a) mg (b) mg
1 + µ2 µ+1
St

h h
µ
(c) mg (d) µ mg
1 − µ2 16. An ideal gas undergoes the cyclic process abca
12. A tennis ball hits the floor with a speed v at as shown in the given p - V diagram.
an angle θ with the normal to the floor. If the p
collision is inelastic and the coefficient of c
restitution is ε , what will be the angle of
reflection?
tanθ  sinθ 
(a) tan−1   (b) sin−1  
b a
 ε   ε 
2ε V
(c) θε (d) θ
ε+1
WB JEE (Engineering) Solved Paper 2020 3
It rejects 50 J of heat during ab and absorbs In that case, which of the following graph(s)
80 J of heat during ca. During bc, there is no shows most correctly the electric field vector
transfer of heat and 40 J of work is done by in E x in x-direction?
the gas. What should be the area of the Ex Ex
closed curve abca?
(a) 30 J (b) 40 J (c) 10 J (d) 90 J
(a) x (b) x
17. A container AB in the shape of a rectangular O (10,0) O (10,0)
parallelopiped of length 5 m is divided

)
internally by a movable partition P as shown

be
in the figure. Ex Ex
5m

Tu
(10,0)
(c) x (d) x
O O (10,0)
A P B

ou
The left compartment is filled with a given 20. Four identical point masses, each of mass m
and carrying charge + q are placed at the
mass of an ideal gas of molar mass 32 while
the right compartment is filled with an equal
mass of another ideal gas of molar mass 18 at (Y corners of a square of sides a on a frictionless
plane surface. If the particles are released
simultaneously, the kinetic energy of the
on
same temperature. What will be the distance
of P from the left wall A when equilibrium is system when they are infinitely far apart is
established? q2 q2
(a) (2 2 + 1) (b) ( 2 + 2 )
pi

(a) 2.5 m (b) 1.8 m (c) 3.2 m (d) 2.1 m a a


q2 q2
18. When 100 g of boiling water at 100°C is (c) ( 2 + 4) (d) ( 2 + 1)
m

a a
added into a calorimeter containing 300 g of
cold water at 10°C, temperature of the 21. A very long charged solid cylinder of radius a
ha

mixture becomes 20°C. Then, a metallic block contains a uniform charge density ρ.
of mass 1 kg at 10°C is dipped into the Dielectric constant of the material of the
C

mixture in the calorimeter. After reaching cylinder is K. What will be the magnitude of
thermal equilibrium, the final temperature electric field at a radial distance x (x < a) from
becomes 19°C. What is the specific heat of the axis of the cylinder?
dy

the metal in CGS unit? (a) ρ


x
(b) ρ
x
(a) 0.01 (b) 0.3 (c) 0.09 (d) 0.1 ε0 2 kε0
u

x2 x
19. As shown in the figure, a point charge (c) ρ (d) ρ
−6
q1 = + 1 × 10 C is placed at the origin in 2 aε0 2k
St

xy-plane and another point charge 22. A galvanometer can be converted to a


q2 = + 3 × 10 −6 C is placed at the coordinate voltmeter of full scale deflection V0 by
(10, 0). connecting a series resistance R1 and can be
Y converted to an ammeter of full scale
deflection I 0 by connecting a shunt resistance
R2 . What is the current flowing through the
galvanometer at its full scale deflection?
q2 V0 − I0 R 2 V0 + I0 R 2
(a) (b)
q1
X R1 − R 2 R1 + R 2
(10,0)
V0 − I0 R1 V0 + I0 R1
(c) (d)
R 2 − R1 R1 + R 2
4 WB JEE (Engineering) Solved Paper 2020
23. B 26. I
A

G O r a
t

When a DC voltage is applied at the two ends


of a circuit kept in a closed box, it is observed
a
that the current gradually increases from

)
be
As shown in the figure, a single conducting zero to a certain value and then remains
wire is bent to form a loop in the form of a constant. What do you think that the circuit
circle of radius r concentrically inside a contains?
square of side a, where a : r = 8 : π. A battery

Tu
(a) A resistor alone
B drives a current through the wire. If the (b) A capacitor alone
battery B and the gap G are of negligible sizes, (c) A resistor and an inductor in series

ou
determine the strength of magnetic field at (d) A resistor and a capacitor in series
the common centre O.
µ 0I µ 0I 27. Consider the circuit shown.
(a) 2 ( 2 − 1) (b) ( 2 + 1)
2 πa
µ I
(c) 0 2 2 ( 2 + 1)
πa
2 πa
µ I
(d) 0 2 2 ( 2 − 1)
πa (Y 2W 8W

8W
on
4W 2 µF
24. × × × × × ×
2V
× × × × R× ×
pi

O
× × × × × × 2V 2V
m

As shown in the figure, a wire is bent to form If all the cells have negligible internal
a D-shaped closed loop, carrying current I, resistance, what will be the current through
ha

where the curved part is a semi-circle of the 2 Ω resistor when steady state is reached?
radius R. The loop is placed in a uniform (a) 0.66 A (b) 0.29 A (c) 0 A (d) 0.14 A
C

magnetic field B, which is directed into the


28. Consider a conducting wire of length L bent
plane of the paper. The magnetic force felt by
in the form of a circle of radius R and another
the closed loop is
conductor of length a (a < < R) is bent in the
dy

1
(a) zero (b) IRB (c) 2IRB (d) IRB form of a square. The two loops are then
2
placed in same plane such that the square
u

25. 2R 2R 2R
loop is exactly at the centre of the circular
loop. What will be the mutual inductance
St

A
between the two loops?
R R R πa2 πa2
(a) µ 0 (b) µ 0
L 16 L
B πa2 a2
(c) µ 0 (d) µ 0
4L 4 πL
What will be the equivalent resistance
between the terminals A and B of the infinite 29. An object, is placed 60 cm in front of a
resistive network shown in the figure? convex mirror of focal length 30 cm. A plane
( 3 + 1)R ( 3 − 1)R mirror is now placed facing the object in
(a) (b)
2 2 between the object and the convex mirror
R
(c) 3 (d) ( 3 + 1) R such that it covers lower half of the convex
2 mirror. What should be the distance of the
WB JEE (Engineering) Solved Paper 2020 5
plane mirror from the object, so that there block, if the specific heat of the block is 0.1
will be no parallax between the images CGS unit? Assume g =10 ms−2 and uniform
formed by the two mirrors? rise in temperature throughout the whole
(a) 40 cm (b) 30 cm (c) 20 cm (d) 15 cm block. [Ignore absorption of heat by the
table]
30. A thin convex lens is placed just above an
empty vessel of depth 80 cm. The image of a (a) 0.0025°C (b) 0.025°C
(c) 0.001°C (d) 0.05°C
coin kept at the bottom of the vessel is thus
formed 20 cm above the lens. If now water is 34. Consider an engine that absorbs 130 cal of

)
poured in the vessel upto a height of 64 cm, heat from a hot reservoir and delivers 30 cal

be
what will be the approximate new position of heat to a cold reservoir in each cycle. The
the image? Assume that refractive index of engine also consumes 2 J energy in each
water is 4/3. cycle to overcome friction. If the engine

Tu
(a) 21.33 cm, above the lens works at 90 cycles per minute, what will be
(b) 6.67 cm, below the lens the maximum power delivered to the load?
(c) 33.67 cm, above the lens

ou
[Assume the thermal equivalent of heat is
(d) 24 cm, above the lens 4.2 J/cal]
(a) 816 W (b) 819 W
Category II (Q. Nos. 31 to 35)

(Y
(c) 627 W (d) 630 W
Carry 2 marks each and only one option is correct.
In case of incorrect answer or any combination of 35. Two pith balls, each carrying charge + q are
more than one answer, 1/2 mark will be deducted. hung from a hook by two strings. It is found
on
that when each charge is tripled, angle
31. A conducting circular loop of resistance 20 Ω between the strings double. What was the
and cross-sectional area 20 × 10 −2 m 2 is initial angle between the strings?
pi

placed perpendicular to a spatially uniform (a) 30° (b) 60° (c) 45° (d) 90°
m

magnetic field B, which varies with time t as


B = 2 sin(50 πt) T. Find the net charge flowing Category III (Q. Nos. 36 to 40)
ha

through the loop in 20 ms starting from t = 0. Carry 2 marks each and one or more option(s)
(a) 0.5 C (b) 0.2 C (c) 0 C is/are correct. If all correct answers are not marked
(d) 0.14 C
and also no incorrect answer is marked then score
C

32. A pair of parallel metal plates are kept with a = 2 × number of correct answers marked ÷ actual
separation d. One plate is at a potential + V number of correct answers. If any wrong option is
and the other is at ground potential. A
dy

marked or if any combination including a wrong


narrow beam of electrons enters the space option is marked, the answer will be considered
between the plates with a velocity v0 and in
wrong, but there is no negative marking for the
u

a direction parallel to the plates. What will be


same and zero marks will be awarded.
the angle of the beam with the plates after it
St

travels an axial distance L? 36. A point source of light is used in an


−1 
eVL  −1  eVL  experiment of photoelectric effects. If the
(a) tan   (b) tan  2

 mdv 0   mdv 0  distance between the source and the
 eVL   eVL  photoelectric surface is doubled, which of the
(c) sin−1   (d) cos −1  2
 following may result?
 mdv 0   mdv 0 
(a) Stopping potential will be halved.
(b) Photoelectric current will decrease.
33. A metallic block of mass 20 kg is dragged
(c) Maximum kinetic energy of photoelectrons will
with a uniform velocity of 0.5 ms−1 on a decrease.
horizontal table for 2.1 s. The coefficient of (d) Stopping potential will increase slightly.
static friction between the block and the
table is 0.10. What will be the maximum 37. Two metallic spheres of equal outer radii are
possible rise in temperature of the metal found to have same moment of inertia about
6 WB JEE (Engineering) Solved Paper 2020
their respective diameters. Then, which of 39. A 400 Ω resistor, a 250 mH inductor and a
the following statement(s) is/are true? 2.5 µF capacitor are connected in series with
(a) The two spheres have equal masses. an AC source of peak voltage 5 V and angular
(b) The ratio of their masses is nearly 1.67 : 1. frequency 2kHz. What is the peak value of
(c) The spheres are made of different materials. the electrostatic energy of the capacitor?
(d) Their rotational kinetic energies will be equal
(a) 2 µJ (b) 2.5 µJ
when rotated with equal uniform angular speed
(c) 3.33 µJ (d) 5 µJ
about their respective diameters.
38. A simple pendulum of length l is displaced, 40. A charged particle moves with constant

)
so that its taught string is horizontal and velocity in a region, where no effect of gravity

be
then released. A uniform bar pivoted at one is felt but an electrostatic field E together
end is simultaneously released from its with a magnetic field B may be present. Then,
which of the following cases are possible?

Tu
horizontal position. If their motions are
synchronous, what is the length of the bar? (a) E ≠ 0, B ≠ 0
(a)
3l
(b) l (c) 2l (d)
2l (b) E ≠ 0, B = 0

ou
2 3 (c) E = 0, B = 0
(d) E = 0, B ≠ 0

(Y
Chemistry
on
Category-I (Q. Nos. 41 to 70) OH OH OH COOH
Carry 1 mark each and only one option is correct.
pi

In case of incorrect answer or any combination of


more than one answer, 1/4 mark will be
m

NO2
deducted.
NO2 CH3 OCH3
ha

41. (I) O2N CO2CH3 I II III IV


(a) II < IV < III < I (b) II < III < I < IV
C

(c) II < III < IV < I (d) III < II < I < IV
(II) MeO CO2CH3
44. For the following carbocations, the correct
order of stability is
dy

(III) Me CO2CH3 I. ⊕ CH 2  COCH 3 II. ⊕ CH 2  OCH 3


III. ⊕ CH 2  CH 3
u

For the above three esters, the order of rates


(a) III < II < I (b) II < I < III
St

of alkaline hydrolysis is
(c) I < II < III (d) I < III < II
(a) I > II > III (b) II > III > I
(c) I > III > II (d) III > I > II 45. The reduction product of ethyl
È 3-oxobutanoate by NaBH 4 in methanol is
50% aq.NaOH
42. Ph  CDO → Ph  COOH + an OH OH O
Warm
alcohol.
(a) OEt (b) OH
This alcohol is
OH O O
(a) Ph  CHD  OH (b) Ph  CHD  OD
(c) Ph  CD2  OH (d) Ph  CD2  OD
(c) OEt (d) OH
43. The correct order of acidity for the following
compounds is :
WB JEE (Engineering) Solved Paper 2020 7
46. What is the major product of the following The equilibrium constant for the reaction
reaction ? 1
P - A + B at 25°C is
O O 2
CHO 1. NaOCOEt 1 1
(a) (b) 20 (c) (d) 21
+ C C 2. H3O+ 20 42
Et O Et
O2N
50. Among the following, the ion which will be
H more effective for flocculation of Fe(OH)3
solution is

)
C COOH
(a) PO 34 − (b) SO 24 −

be
(a) C
(c) SO 23 − (d) NO −3
O2N CH3
51. The mole fraction of ethanol in water is 0.08.

Tu
H
Its molality is
C COOH (a) 6.32 mol kg − 1 (b) 4.83 mol kg − 1
(c) 3.82 mol kg − 1 (d) 2.84 mol kg − 1

ou
(b) C
O2N
H 52. 5 mL of 0.1 M Pb(NO 3 )2 is mixed with 10 mL

(Y of 0.02 M KI. The amount of PbI2 precipitated


CH3
will be about
(c)
C COOH (a) 10− 2 mol (b) 10− 4 mol
C (c) 2 × 10− 4 mol (d) 10− 3 mol
on
O2N H 53. At 273 K temperature and 76 cm Hg pressure,
g L− 1 . The gas is
pi

CH3 the density of a gas is 1964


.
C (a) CH4 (b) CO (c) He (d) CO 2
m

(d) H
C
54. Equal masses of ethane and hydrogen are
ha

O2N COOH
mixed in an empty container at 298 K. The
fraction of total pressure exerted by hydrogen is
47. The maximum number of electrons in an (a) 15:16 (b) 1:1 (c) 1:4 (d) 1:6
C

atom in which the last electron filled has the


quantum numbers n = 3, l = 2 and m = − 1 is
55. An ideal gas expands adiabatically against
vacuum. Which of the following is correct for
dy

(a) 17 (b) 27
the given process?
(c) 28 (d) 30
(a) ∆S = 0 (b) ∆T = − ve
48. In the face centred cubic lattice structure of (c) ∆U = 0 (d) ∆P = 0
u

gold the closest distance between gold atoms


. K kg mol − 1 . The temperature
56. K f (water) = 186
St

is (‘a’ being the edge length of the cubic unit


cell) at which ice begins to separate from a
a a mixture of 10 mass % ethylene glycol is
(a) a 2 (b) (c) (d) 2 2a
2 2 2 (a) − 186
. °C (b) − 372
. °C
(c) − 3.3° C (d) − 3° C
49. The equilibrium constant for the following
reactions are given at 25°C 57. The radius of the first Bohr orbit of a
hydrogen atom is 0 .53 × 10 − 8 cm. The velocity
2A - B + C , K 1 = 10
.
of the electron in the first Bohr orbit is
2B - C + D, K 2 = 16 (a) 2.188 × 108 cm s − 1 (b) 4.376 × 108 cm s − 1
2C + D - 2 P, K 3 = 25 (c) 1.094 × 108 cm s − 1 (d) 2.188 × 109 cm s − 1
8 WB JEE (Engineering) Solved Paper 2020
58. Which of the following statements is not true 66. To a solution of a colourless efflorescent
for the reaction, 2F2 + 2 H 2O → 4 HF + O 2 ? sodium salt, when dilute acid is added, a
(a) F2 is more strongly oxidising than O 2 colourless gas is evolved along with
(b) F  F bond is weaker than O == O bond formation of a white precipitate. Acidified
(c) H  F bond is stronger than H  O bond dichromate solution turns green, when the
(d) F is less electronegative than O colourless gas is passed through it. The
sodium salt is
59. The number of unpaired electrons in the
(a) Na 2SO 3 (b) Na 2S
uranium (92 U) atom is
(c) Na 2S2O 3 (d) Na 2S4O 6

)
(a) 4 (b) 6 (c) 3 (d) 1

be
67. The reaction for obtaining the metal (M)
60. How and why does the density of liquid from its oxide ( M2O 3) ore is given by
water change on prolonged electrolysis?

Tu
Heat
(a) Decreases, as the proportion of H2O increases M2O 3(s) + 2 Al()l → Al 2O 3 () l + 2 M(s),
(b) Remains unchanged (s = solid, l = liquid) in that case, M is
(c) Increases, as the proportion of D2O increases (a) copper (b) calcium

ou
(d) Increases, as the volume decreases (c) iron (d) zinc
61. The difference between orbital angular 68. In the extraction of Ca by electro reduction of

(Y
momentum of an electron in a 4 f -orbital and molten CaCl 2 some CaF2 is added to the
another electron in a 4 s-orbital is electrolyte for the following reason :
(a) 2 3 (b) 3 2 (a) To keep the electrolyte in liquid state at
on
(c) 3 (d) 2 temperature lower than the m.p. of CaCl 2
(b) To effect precipitation of Ca
62. Which of the following has the largest
(c) To effect the electrolysis at lower voltage
number of atoms?
pi

(d) To increase the current efficiency


(a) 1 g of Ag (b) 1 g of Fe
69. The total number of alkyl bromides
m

(c) 1 g of Cl 2 (d) 1 g of Mg
(including stereoisomers) formed in the
63. Indicate the correct IUPAC name of the reaction Me3C  CH == CH 2 + HBr → will be
ha

coordination compound shown in the figrue. (a) 1 (b) 2


NH3 (c) 3 (d) No bromide forms
NH3
C

Cl Cr NH3 Cl 70. 1. Mg/diethyl ether


Cl Br Product
Cl 2. CH2O
NH3
dy

+
3. H2O
(a) Cis-dichlorotetraminochromium (III) chloride The product in the above reaction is
(b) Trans-dichlorotetraminochromium (III) chloride
u

(c) Trans-tetraminedichlorochromium (III) chloride (a) Br CH2OH


St

(d) Cis-tetraamminedichlorochromium (III) chloride


CH2OH
64. What will be the mass of one atom of 12C?
(a) 1 amu (b) 19923
. × 10− 23 g (b) Cl CH2OH
− 22
(c) 16603
. × 10 g (d) 6 amu
CH3
65. Bond order of He2, He+2 and He22 + are
respectively (c) Cl CH2OH
1 1
(a) 1, , 0 (b) 0, , 1
2 2 (d) HOH2C CH2OH
1 1
(c) , 1 , 0 (d) 1, 0,
2 2
WB JEE (Engineering) Solved Paper 2020 9
Category-II (Q. Nos. 71 to 75) aq.È OH
75. CH 3  O  CH 2  Cl →
Carry 2 marks each and only one option is ∆

correct. In case of incorrect answer or any CH 3  O  CH 2  OH


combination of more than one answer, 1/2 mark Which information below regarding this
will be deducted. reaction is applicable?
71. Which of the following compounds is (a) It follows SN2 pathway, because it is a primary
asymmetric? alkyl chloride
Br Cl Cl (b) It follows SN1 pathway, because the intermediate

)
carbocation is resonance stabilised

be
(a) (b)
(c) SN1 pathway is not followed, because the
intermediate carbocation is destabilised by
Cl
−I-effect of oxygen

Tu
Br
Br Cl Cl (d) A mixed SN1 and SN2 pathway is followed

(c) (d) Category-III (Q. Nos. 76 to 80)

ou
Cl Carry 2 marks each and one or more option(s)
Br is/are correct. If all correct answers are not
72. For a reaction 2 A + B → P, when
concentration of B alone is doubled, t1 / 2 does (Y
marked and no incorrect answer is marked, then
score = 2 × number of correct answers marked ÷
actual number of correct answers. If any wrong
on
not change and when concentrations of both option is marked or if any combination including
A and B is doubled, rate increases by a factor a wrong option is marked, the answer will be
of 4. The unit of rate constant is, considered wrong, but there is no negative
pi

(a) s − 1 (b) L mol − 1s − 1 marking for the same and zero mark will be
(c) mol L− 1s − 1 (d) L2mol − 2s − 1 awarded.
m

73. A solution is saturated with SrCO 3 and SrF2.


76. Which of the following reactions give(s) a
. × 10 − 3 M. The
The [CO 23 − ] is found to be 12
ha

meso-compound as the main product?


concentration of F− in the solution would be
. × 10− 6 M
Br2 H2
(a) 37 (a) (b)
C

CH2Cl2
. × 10− 3 M
Pd-C
(b) 32
. × 10− 7M
(c) 51
dy

. × 10− 2 M
(d) 37 (c) H2 (d) Br2

Given: Ksp (SrCO 3 ) = 7.0 × 10−10 , Lindlar's catalyst CCl4


u

Ksp (SrF2 ) = 7.9 × 10−10


77. For spontaneous polymerisation, which of
St

74. A homonuclear diatomic gas molecule shows the following is (are) correct?
2-electron magnetic moment. The (a) ∆G is negative (b) ∆H is negative
one-electron and two-electron reduced (c) ∆S is positive (d) ∆S is negative
species obtained from above gas molecule
can act as both oxidising and reducing 78. Which of the following statement(s) is/are
agents. When the gas molecule is incorrect?
one-electron oxidised the bond length (a) A sink of SO 2 pollutant is O 3 in the atmosphere
decreases compared to the neutral molecule. (b) FGD is a process of removing NO 2 from
The gas molecule is atmosphere
(c) NO x in fuel gases can be removed by alkaline
(a) N2 (b) Cl 2
scrubbing
(c) O 2 (d) B 2
(d) The catalyst used to convert CCl 4 to CF4 by HF
is SbF5
79. SiO 2 is attacked by which one/ones of the The product(s) from the above reaction will
following? be
(a) HF (b) Conc.HCl Me Me
(c) Hot NaOH (d) Fluorine H OH H OH
(a) (b)
HO H H OH
Na/NH 3 (liq.)
80. Me C ≡≡ C  Me →
EtOH, − 33 ° C
X Me Me

dil.alkaline KMnO 4
Me Me
→ Product(s)
(c) HO H (d) HO H

)
H OH

be
HO H
Me Me

Tu
Mathematics

ou
Category-I (Q. Nos. 1 to 50) (b) cos − 1 (f( x))2 − (φ( x))2 + c

(Y
Carry 1 mark each and only one option is correct. f( x) φ( x) − 1
(c) 2 tan− 1 +c
In case of incorrect answer or any combination of 2
more than one answer, 1/4 mark will be deducted. f ( x) φ( x) + 1
(d) 2 tan− 1 +c
on
n 2
 y  x
1. Let cos − 1   = log   . Then
 b  n 4. The value of
− 2n 10 2 n + 1
pi

10
(a) x y2 + xy1 + n y = 0
2 2

(b) xy2 − xy1 + 2 n2 y = 0


∑ ∫ sin x dx +
27
∑ ∫ sin27 x dx is equal
n = 1 − 2n − 1 n=1
m

2n
(c) x2 y2 + 3 xy1 − n2 y = 0 to
(d) xy2 + 5 xy1 − 3 y = 0
ha

(a) 27 (b) 54
 d2y dy  (c) − 54 (d) 0
 Here, y2 = 2 , y1 = 
 dx dx  2
5. ∫ [ x 2]dx is equal to
C

2. Let φ(x) = f (x) + f (1 − x) and f ′′(x) < 0 in 0


[0, 1], then (b) 5 − 2 −
dy

(a) 1 3
(a) φ is monotonic increasing in  0,  and (c) 3 − 2
1 (d) 8/3
 2 
6. If the tangent to the curve y 2 = x 3 at (m 2 , m 3)
monotonic decreasing in  , 1
u

1
 2  is also a normal to the curve at (M 2 , M 3), then
St

(b) φ is monotonic increasing in  , 1 and


1 the value of mM is
 2  1 2
(a) − (b) −
monotonic decreasing in  0, 
1 9 9
 2  1 4
(c) − (d) −
(c) φ is neither increasing nor decreasing in any 3 9
sub-interval of [0, 1] 2
a
(d) φ is increasing in [0, 1]  dy 
7. If x 2 + y 2 = a 2, then ∫ 1 +   dx =
f (x) φ ′ (x) + φ(x) f ′ (x)  dx 
∫ ( f (x) φ(x) + 1)
0
3. dx =
f (x) φ(x) − 1 (a) 2 πa (b) πa
1 1
f ( x) (c) πa (d) πa
(a) sin− 1 +c 2 4
φ( x)
WB JEE (Engineering) Solved Paper 2020 11
8. Let f , be a continuous function in [0, 1], 14. If 2 log(x + 1) − log(x 2 − 1) = log 2, then x =
n
1  j (b) − 1and 3
∑ n f  n is
(a) only 3
then lim
n→ ∞ (c) only − 1 (d) 1 and 3
j =0
1 15. The number of complex numbers p such that
1
12 | p | = 1 and imaginary part of p 4 is 0, is
(a) ∫ f( x) dx (b) ∫ f( x) dx
2 0 1
(a) 4 (b) 2
2
1 (c) 8 (d) infinitely many

)
1 2
16. The equation zz + (2 − 3i) z + (2 + 3i)z + 4 = 0

be
(c) ∫ f( x) dx (d) ∫ f( x) dx
0 0 represents a circle of radius
(a) 2 unit (b) 3 unit

Tu
9. Let f be a differentiable function with
(c) 4 unit (d) 6 unit
lim f (x) = 0. If y ′ + yf ′ (x) − f (x) f ′ (x) = 0,
x→∞
dy 17. The expression ax 2 + bx + c (a , b and c are
lim y(x) = 0, then (where y ′ ≡ )

ou
x→∞ dx real) has the same sign as that of a for all x if
(a) y + 1 = e f( x ) + f( x) (a) b 2 − 4ac > 0
(b) y − 1 = e f( x ) + f( x) (b) b 2 − 4ac ≠ 0
(c) y + 1 = e
(d) y − 1 = e
− f( x )

− f( x )
+ f ( x)
+ f ( x) (Y (c) b 2 − 4ac ≤ 0
(d) b and c have the same sign as that of a
on
 y   y  18. In a 12 storied building, 3 persons enter a lift
10. If x sin   dy =  y sin   − x  dx , x > 0 and
 x  x  
cabin. It is known that they will leave the lift
at different floors. In how many ways can
π
pi

 y
1 =
y() , then the value of cos   is they do so if the lift does not stop at the
2  x second floor?
m

(a) 1 (b) log x (c) e (d) 0 (a) 36 (b) 120


(c) 240 (d) 720
11. Let f (x) = 1 − (x ), where the square root is
2
ha

to be taken positive, then 19. If the total number of m-element subsets of


(a) f has no extrema at x = 0 the set A = { a1 , a 2 , …, a n} is k times the
C

(b) f has minima at x = 0 number of m element subsets containing a 4 ,


(c) f has maxima at x = 0 then n is
(a) (m − 1) k
dy

(d) f′ exists at 0 (b) mk


(c) (m + 1) k (d) (m + 2 ) k
12. If the function f (x) = 2 x − 9 ax + 12a x + 1 3 2 2

[ a > 0 ] attains its maximum and minimum at 20. Let I (n) = nn, J (n) = 13
. .5 ........ (2 n − 1) for all
u

p and q respectively such that p 2 = q, then a is (n > 1), n ∈ N , then


St

equal to (a) I(n) > J(n) (b) I(n) < J(n)


1
(a) 2 (b)
1
(c)
1
(d) 3 (c) I(n) = J(n) (d) I(n) = J(n)
2 4 2

13. If a and b are arbitrary positive real numbers, 21. If c0 , c1 , c2, ....., c15 are the binomial
then the least possible value of
6 a 10 b
+ is coefficients in the expansion of (1 + x)15, then
5b 3a c c c c
6 the value of 1 + 2 2 + 3 3 + … + 15 15 is
(a) 4 (b) c0 c1 c2 c14
5
10 68 (a) 1240 (b) 120
(c) (d) (c) 124 (d) 140
3 15
12 WB JEE (Engineering) Solved Paper 2020
3 − t 1 0 29. Four persons A , B, C and D throw an unbiased
  die, turn by turn, in succession till one gets
22. Let A =  − 1 3 − t 1 and det A = 5, then
 0 − 1 0 
an even number and win the game. What is
 the probability that A wins if A begins?
(a) t = 1 (b) t = 2 (c) t = − 1 (d) t = − 2 (a)
1
(b)
1
(c)
7
(d)
8
 125 24 4 2 12 15
 
23. Let A =  x 6 2 . The value of x for 30. A rifleman is firing at a distant target and
 − 1 − 2 3 has only 10% chance of hitting it. The least
 

)
number of rounds he must fire to have more
which the matrix A is not invertible is

be
than 50% chance of hitting it at least once, is
(a) 6 (b) 12 (c) 3 (d) 2
(a) 5 (b) 7 (c) 9 (d) 11
 a b
24. Let A =   be a 2 × 2 real matrix with

Tu
 c d 31. cos(2 x + 7) = a(2 − sin x) can have a real
solution for
det A = 1. If the equation det(A − λI 2) = 0 has
(a) all real values of a (b) a ∈[2, 6]

ou
imaginary roots (I 2 be the identity matrix of
(c) a ∈ (− ∞, 2 ) \ {0} (d) a ∈ (0, ∞ )
order 2), then
(a) (a + d )2 < 4 (b) (a + d )2 = 4 32. The differential equation of the family of
(c) (a + d ) > 4 (d) (a + d )2 = 16

(Y curves y = e x (A cos x + Bsin x) where A , B are


2

arbitrary constants is
a2 c 2 + ac
bc
d2y d2y dy
− 9 x = 13 −2 + 2y = 0
on
25. If a 2 + ab b2 ca = ka 2 b2 c 2 , (a)
dx 2
(b)
dx2 dx
ab b + bc
2
c 2
d2y
2
(d)   +
dy dy
(c) + 3y = 4 − xy = 0
 dx 
pi

then k = dx2 dx
(a) 2 (b) − 2 (c) − 4 (d) 4  π
33. The equation rcos θ −  = 2 represents
m

26. If f : S → R, where S is the set of all  3


non-singular matrices of order 2 over R and
ha

(a) a circle (b) a parabola


 a b  (c) an ellipse (d) a straight line
f    = ad − bc, then
  c d  34. The locus of the centre of the circles which
C

(a) f is bijective mapping touch both the circles x 2 + y 2 = a 2 and


(b) f is one-one but not onto x 2 + y 2 = 4 ax externally is
dy

(c) f is onto but not one-one


(d) f is neither one-one nor onto (a) a circle (b) a parabola
(c) an ellipse (d) a hyperbola
27. Let the relation ρ be defined on R by a ρ b holds
u

35. Let each of the equations x 2 + 2 xy + ay 2 = 0


if and only if a − b is zero or irrational, then
St

(a) ρ is equivalence relation and ax 2 + 2 xy + y 2 = 0 represent two straight


(b) ρ is reflexive and symmetric but is not transitive lines passing through the origin. If they have
(c) ρ is reflexive and transitive but is not symmetric a common line, then the other two lines are
(d) ρ is reflexive only given by
28. The unit vector in ZOX plane, making angles (a) x − y = 0, x − 3 y = 0 (b) x + 3 y = 0, 3 x + y = 0
(c) 3 x + y = 0, 3 x − y = 0 (d) (3 x − 2 y) = 0, x + y = 0
45° and 60° respectively with α = 2 $i + 2 $j − k$
and β = $j − k$ is 36. A straight line through the origin O meets the
1
parallel lines 4 x + 2 y = 9 and 2 x + y + 6 = 0
(a) $i + 1 $j (b)
1 $i − 1 k$ at P and Q respectively. The point O divides
2 2 2 2
1 the segment PQ in the ratio
(c) $i − 1 $j (d)
1 $i + 1 k$
2 2 2 2 (a) 1 : 2 (b) 3 : 4 (c) 2 : 1 (d) 4 : 3
WB JEE (Engineering) Solved Paper 2020 13
37. Area in the first quadrant between the (a) 8 x + 14 y + 13 z + 37 = 0
(b) 8 x − 14 y − 13 z − 37 = 0
ellipses x + 2 y = a and 2 x + y = a is
2 2 2 2 2 2
(c) 8 x − 14 y − 13 z + 37 = 0
a2 1 3a2 1 (d) 8 x − 14 y + 13 z + 37 = 0
(a) tan− 1 (b) tan− 1
2 2 4 2
5a2 − 1 1 9 πa2 44. The sine of the angle between the straight
(c) sin (d) x −2 y−3 z− 4
2 2 2 line = = and the plane
3 4 5
38. The equation of circle of radius 17 unit, 2 x − 2 y + z = 5 is

)
with centre on the positive side of X -axis and 2 3 2
(a) (b)

be
through the point (0, 1) is 5 10
(a) x2 + y2 − 8 x − 1 = 0 4 5
(c) (d)
(b) x2 + y2 + 8 x − 1 = 0 5 2 6

Tu
(c) x2 + y2 − 9 y + 1 = 0
45. Let f (x) = sin x + cos ax be periodic function.
(d) 2 x2 + 2 y2 − 3 x + 2 y = 4
Then,

ou
39. The length of the chord of the parabola (a) a is any real number
y 2 = 4 ax (a > 0) which passes through the (b) a is any irrational number
vertex and makes an acute angle α with the (c) a is rational number
axis of the parabola is
(a) ± 4acotα cosec α
(c) − 4acotα cosec α
(b) 4acotα cosec α
(d) 4a cosec 2 α
(Y (d) a = 0

46. The domain of f (x) = 


 1 
− (x + 1) is
on
 x 
40. A double ordinate PQ of the hyperbola (a) x > − 1 (b) (− 1, ∞ ) \ {0}
x2 y2  5 − 1 1 − 5 
pi

− = 1 is such that ∆OPQ is equilateral, (c)  0, (d)  , 0


a 2 b2  2   2 
m

O being the centre of the hyperbola. Then the


eccentricity e satisfies the relation 47. Let y = f (x) = 2 x 2 − 3 x + 2. The differential of
ha

2 2 y when x changes from 2 to 1.99 is


(a) 1 < e < (b) e =
3 3 (a) 0.01 (b) 0.18 (c) − 0.05 (d) 0.07
3 2
(c) e = (d) e > 1/ x 1/ x
C

2 3 1 + cx  1 + 2 cx 
48. If lim   = 4, then lim   is
x → 0 1 − cx  x → 0 1 − 2 cx 
41. If B and B′ are the ends of minor axis and
dy

x 2 y2 (a) 2 (b) 4 (c) 16 (d) 64


S and S′ are the foci of the ellipse + = 1,
25 9 49. Let f : R → R be twice continuously
then the area of the rhombus SBS′ B′ will be
u

differentiable (or f ′ ′ exists and is continuous)


(a) 12 sq units (b) 48 sq units
such that f (0) = f ()
1 = f ′ (0) = 0 . Then
St

(c) 24 sq units (d) 36 sq units


(a) f ′ ′(c ) = 0 for some c ∈ R
42. The equation of the latusrectum of a parabola (b) there is no point for which f ′ ′( x) = 0
is x + y = 8 and the equation of the tangent (c) at all points f ′ ′( x) > 0
at the vertex is x + y = 12. Then, the length of (d) at all points f ′ ′( x) < 0
the latusrectum is
50. Let
(a) 4 2 units (b) 2 2 units
(c) 8 units (d) 8 2 units f (x) = x 13 + x 11 + x 9 + x 7 + x 5 + x 3 + x + 12.
Then
43. The equation of the plane through the point (a) f( x) has 13 non-zero real roots
(2 , − 1, − 3) and parallel to the lines (b) f( x) has exactly one real root
x −1 y + 2 z x y −1 z − 2
= = and = = is (c) f( x) has exactly one pair of imaginary roots
2 3 −4 2 −3 2 (d) f( x) has no real root
14 WB JEE (Engineering) Solved Paper 2020
Category-II (Q. Nos. 51-65) 57. Let A = { x ∈ R : − 1 ≤ x ≤ 1} and f : A → A be a
Carry 2 marks each and only one option is mapping defined by f (x) = x | x |. Then f is
correct. In case of incorrect answer or any (a) injective but not surjective
combination of more than one answer, 1/2 mark (b) surjective but not injective
will be deducted. (c) neither injective nor surjective
(d) bijective
51. The area of the region
{(x , y) : x 2 + y 2 ≤ 1 ≤ x + y} is 58. Let f (x) = x 2 − 3 x + 2 and g(x) = x be two

)
π2 π π 1 π2 given functions. If S be the domain of fog and

be
(a) (b) (c) (d)
2 4 4 2 3 T be the domain of gof , then
 π (a) S = T (b) S ∩ T = φ
52. In open interval 0 ,  , (c) S ∩ T is a singleton (d) S ∩ T is an interval

Tu
 2
(a) cos x + xsin x < 1 59. Let ρ1 and ρ2 be two equivalence relations
(b) cos x + xsin x > 1 defined on a non-void set S. Then

ou
(c) no specific order relation can be ascertained (a) both ρ1 ∩ ρ2 and ρ1 ∪ ρ2 are equivalence
between cos x + xsin x and 1 relations
1 (b) ρ1 ∩ ρ2 is equivalence relation but ρ1 ∪ ρ2 is not
(d) cos x + xsin x <
2

53. If the line y = x is a tangent to the parabola (Y so


(c) ρ1 ∪ ρ2 is equivalence relation but ρ1 ∩ ρ2 is not
so
on
y = ax 2 + bx + c at the point (1, 1) and the (d) neither ρ1 ∩ ρ2 nor ρ1 ∪ ρ2 is equivalence relation.
curve passes through (− 1, 0), then
x2 y2
(a) a = b = − 1, c = 3
1
(b) a = b = , c = 0 60. Consider the curve +
= 1. The portion
pi

2 a 2 b2
1 1 1 of the tangent at any point of the curve
(c) a = c = , b = (d) a = 0, b = c =
m

4 2 2 intercepted between the point of contact and


the directrix subtends at the corresponding
54. If the vectors α = $i + a$j + a 2k$ , β = $i + b$j + b2k$
ha

focus an angle of
and γ = $i + c$j + c 2 k$ are three non-coplanar π π π π
(a) (b) (c) (d)
4 3 2 6
a a2 1 + a3
C

vectors and b b2 1 + b3 = 0 , then the 61. A line cuts the X -axis at A(7 , 0) and the
2
1+ c 3 Y -axis at B(0 , − 5). A variable line PQ is drawn
dy

c c
perpendicular to AB cutting the X -axis at
value of abc is P(a , 0) and the Y -axis at Q(0 , b). If AQ and BP
(c) − 1
u

(a) 1 (b) 0 (d) 2 intersect at R, the locus of R is


(a) x2 + y2 + 7 x + 5 y = 0
St

55. Let z1 and z 2 be two imaginary roots of


(b) x2 + y2 + 7 x − 5 y = 0
z 2 + pz + q = 0 , where p and q are real. The
(c) x2 + y2 − 7 x + 5 y = 0
points z1 , z 2 and origin form an equilateral
(d) x2 + y2 − 7 x − 5 y = 0
triangle if
(a) p2 > 3q (b) p2 < 3q 1 /( k + α )
dx
(c) p = 3q
2
(d) p = q
2 62. Let 0 < α < β < 1. Then, lim
n→ ∞ ∫ 1 +x
is
1 /( k + β)
56. If P(x) = ax 2 + bx + c and Q(x) = − ax 2 + dx + c, (a) loge
β
(b) loge
1+ β
where ac ≠ 0 [a , b, c , d are all real], then α 1+ α
P(x) ⋅ Q(x) = 0 has 1+ α
(c) loge (d) ∞
(a) atleast two real roots (b) two real roots 1+ β
(c) four real roots (d) no real root
WB JEE (Engineering) Solved Paper 2020 15
 1 1  (log 3 x ) 2 −
9
log 3 x + 5
63. lim  −  68. The equation x 2 = 3 3 has
x → 1  lnx (x − 1)
(a) at least one real root
(a) Does not exist (b) 1
1 (b) exactly one real root
(c) (d) 0 (c) exactly one irrational root
2
(d) complex roots
1
64. Let y = , then 69. In a certain test, there are n questions. In this
1 + x + ln x test 2 n − i students gave wrong answers to at
dy dy least i questions, where i = 1, 2 , …, n. If the

)
(a) x + y = x (b) x = y( yln x − 1)

be
dx dx total number of wrong answers given is 2047,
2
then n is equal to
(d) x  = y − x
dy dy
(c) x2 = y2 + 1 − x 2
dx  dx  (a) 10 (b) 11 (c) 12 (d) 13

Tu
65. Consider the curve y = be − x / a , where a and b 70. A and B are independent events. The
1
are non-zero real numbers. Then probability that both A and B occur is and

ou
x y 20
(a) + = 1is tangent to the curve at (0, 0) the probability that neither of them occurs is
a b
x y 3
(b) + = 1is tangent to the curve, where the

(Y
. The probability of occurrence of A is
a b 5
curve crosses the axis of y 1 1 1 1
(a) (b) (c) (d)
x y
(c) + = 1is tangent to the curve at (a, 0) 2 10 4 5
on
a b
x y 71. The equation of the straight line passing
(d) + = 1is tangent to the curve at (2 a, 0)
a b through the point (4, 3) and making intercepts
on the coordinate axes whose sum is − 1 is
pi

x y x y
Category-III (Q. Nos. 66 to 75) (a) − =1 (b) + =1
m

2 3 −2 1
Carry 2 marks each and one or more option(s) x y x y
is/are correct. If all correct answers are not (c) − + = 1 (d) − = 1
ha

3 2 1 2
marked and no incorrect answer is marked, then
score = 2 × number of correct answers marked ÷ x 2 y2
72. Consider a tangent to the ellipse + =1
actual number of correct answers. If any wrong 2 1
C

option is marked or if any combination including at any point. The locus of the mid-point of
a wrong option is marked, the answer will be the portion intercepted between the axes is
dy

considered wrong, but there is no negative x2 y2 x2 y2


(a) + =1 (b) + =1
marking for the same and zero marks will be 2 4 4 2
1 1 1 1
awarded. (c) 2 + =1 (d) 2 + =1
u

3x 4 y2 2x 4 y2
St

66. The area of the figure bounded by the x2 d2y


parabola x = − 2 y 2 , x = 1 − 3 y 2 is 73. Let y = . Then, 2 is
(x + 1) (x + 2)
2
dx
1 4
(a) sq unit (b) sq unit  3 3 4 
3 3 (a) 2  − + 
(c) 1 sq unit (d) 2 sq units  ( x + 1)4
( x + 1 )3
( x + 2 )3 
 2 4 5 
67. A particle is projected vertically upwards. If it (b) 3  + − 
has to stay above the ground for 12 sec, then  ( x + 1)3
( x + 1) 2
( x + 2 )3 
6 4 3
(a) velocity of projection is 192 ft/sec (c) − +
(b) greatest height attained is 600 ft ( x + 1)3 ( x + 1)2 ( x + 1)3
(c) velocity of projection is 196 ft/sec 7 3 2
(d) − +
(d) greatest height attained is 576 ft ( x + 1)3 ( x + 1)2 ( x + 1)3
16 WB JEE (Engineering) Solved Paper 2020
1 (a) the differential equation of the curve is
74. Let f (x) = x sin x − (1 − cos x). The smallest dy
3 3x + y = 0
f (x) dx
positive integer k such that lim k ≠ 0 is (b) the differential equation of the curve is
x→ 0 x dy
3x − y = 0
(a) 4 (b) 3 (c) 2 (d) 1 dx
(c) the curve passes through  , 2 
1
75. Tangent is drawn at any point P(x , y) on a 8 
curve, which passes through (1, 1). The (d) the normal at (1, 1) is x + 3 y = 4
tangent cuts X -axis and Y -axis at A and B

)
respectively. If AP : BP = 3 :1, then

be
Tu
Answers

ou
Physics

(Y
1. (b) 2. (d) 3. (b) 4. (a) 5. (a) 6. (d) 7. (a) 8. (d) 9. (b) 10. (a)
11. (a) 12. (a) 13. (c) 14. (b) 15. (c) 16. (a) 17. (b) 18. (d) 19. (a) 20. (c)
21. (b) 22. (a) 23. (d) 24. (a) 25. (d) 26. (c) 27. (c) 28. (b) 29. (a) 30. (a)
on
31. (c) 32. (b) 33. (a) 34. (c) 35. (b) 36. (b) 37. (d) 38. (a) 39. (d) 40. (a,c,d)

Chemistry
pi

41. (c) 42. (c) 43. (b) 44. (d) 45. (c) 46. (a) 47. (d) 48. (b) 49. (a) 50. (a)
m

51. (b) 52. (b) 53. (d) 54. (a) 55. (c) 56. (c) 57. (a) 58. (d) 59. (a) 60. (c)
61. (a) 62. (d) 63. (d) 64. (b) 65. (b) 66. (c) 67. (c) 68. (a) 69. (c) 70. (d)
ha

71. (d) 72. (b) 73. (d) 74. (c) 75. (d) 76. (a, b) 77. (a, b,d) 78. (a, b, d) 79. (a, c, d) 80. (a, c)

Mathematics
C

1. (a) 2. (a) 3. (c) 4. (d) 5. (b) 6. (d) 7. (c) 8. (c) 9. (c) 10. (b)
11. (c) 12. (a) 13. (a) 14. (a) 15. (c) 16. (b) 17. (c) 18. (d) 19. (b) 20. (a)
dy

21. (b) 22. (d) 23. (c) 24. (a) 25. (d) 26. (d) 27. (b) 28. (b) 29. (d) 30. (b)
31. (c) 32. (b) 33. (d) 34. (d) 35. (b) 36. (b) 37. (a) 38. (a) 39. (b) 40. (d)
u

41. (c) 42. (d) 43. (*) 44. (b) 45. (c) 46. (c) 47. (c) 48. (c) 49. (a) 50. (b)
St

51. (c) 52. (b) 53. (c) 54. (c) 55. (c) 56. (a) 57. (d) 58. (d) 59. (b) 60. (c)
61. (c) 62. (b) 63. (c) 64. (b) 65. (b) 66. (b) 67. (a, d) 68. (a, c) 69. (b) 70. (c, d)
71. (a, b) 72. (d) 73. (a) 74. (c) 75. (a, c)

Note (*) None of the option is correct.


Answer with Explanations
Physics
1. (b) Let intensity from one slit = I1 The range of wavelength in this series is given by
= R  2 − 2  =
1 1 1 5R
Intensity from other slit = I 2 cm−1
λmax 2 3  36

)
According to question,
I1 = 1.5I 2 = R  2 − 2  = cm−1
1 1 1

be
R
and
I1 3 λmin 2 ∞  4
⇒ = 1.5 =
I2 2
Thus, for Balmer:  to cm
4 36

Tu
I ( I1 + I 2)2  R 5R 
∴Ratio of intensity, max =
Imin ( I1 − I 2)2 For Paschen series,
= R  2 − 2  , where n = 4, 5, 6, …
 3+
2 1 1 1
2

ou
=  λ 3 n 
 3− 2
2 The range of wavelength in this series is given by
 1.732 +1.414 

(Y = R  2 − 2  =
=  1 1 1 7R
cm−1
 1.732 − 1.414  3
λmax 4  144
2
=   = (9.89)2 ~
.
3146
− 98 λmin = R  2 − 2  = cm−1
 1 1 R
 0.318  and
on
3 ∞  9
2. (d) Given, slit width, a = 0.5 mm = 0.5 × 10−3 m  9 144 
Thus, for Paschen :  to cm
Wavelength, λ = 600 nm = 600 × 10−9 m  R 7 R 
pi

Distance of screen, D = 50 cm = 50 × 10−2 m For Brackett series,


m

2Dλ = R  2 − 2  , where n = 5, 6, 7, …
1 1 1
∴Linear separation =
a λ 4 n 
ha

2 × 50 × 10−2 × 600 × 10−9 The range of wavelength in this series is given by


=
0.5 × 10−3 1
= R  2 − 2  =
1 1 9R
cm−1
= 12 × 10−4 m λmax 4 5  400
C

= 1.2 × 10−3 m = 1.2 mm = R  2 − 2  =


1 1 1 R
and cm−1
λmin 4 ∞  16
dy

3. (b) Wavelength of emitted radiation is given by


Thus, for Brackett :  to cm
16 400
1 1 1  R 
= R 2 − 2  9R
λ  n1 n2 
u

For Pfund series,


For Lyman series,
= R 2 − 2  , where n = 6, 7, 8, ….
1 1 1
St

= R 2 − 2  , where n = 2, 3, 4, ....


1 1 1 5 n 
λ
λ 1 n 
The range of wavelength in this series is given by
The range of wavelength in this series is given by
= R  2 − 2  =
1 1 1 11R
cm−1
= R  2 − 2  =
1 1 1 3R
cm−1 λmax 5 6  900
λmax 1 2  4
= R  2 − 2  =
1 1 1 R
cm−1
= R  2 − 2  = R cm−1
1 1 1 and
and λmin 5 ∞  25
λmin 1 ∞ 
Thus, for Pfund:  to cm
25 900
Thus for Lyman :  to cm
1 4  R 
11R
 R 3R 
Hence, wavelength range of  cm does
7 19
For Balmer series, to
 5R 5R 
= R 2 − 2  , where n = 3, 4, 5, …
1 1 1
not emit.
λ 2 n 
18 WB JEE (Engineering) Solved Paper 2020
4. (a) The process involved is as VCC=5V

Z X A
→ Z + 1 Y A
+ −1 e
0
+ ν+ Q RC=500 W
IC
∴ Mass defect, mx = M x − Zme Y
and m y = M y − (Z + 1)me
where, M x is atomic mass of X, M y is atomic mass lB=200 mA
VCE
of Y and me is mass of an electron.
Total mass defect, ∆m = mx − m y = (M x − M y − me)
∴Binding energy, E = ∆mc 2

)
IC
So, maximum energy of β-particle emitted We know that, current gain, β =

be
IB
= (M x − M y − me) c 2
IC
⇒ 48 =
5. (a) Given, binding energy per nucleon of nuclei 200 × 10−6

Tu
with mass number 119 = 7.6 MeV
⇒ I C = 48 × 200 × 10−6
Binding energy per nucleon of nuclei with mass
number 238 = 8.6 MeV = 96 × 10−4 A

ou
According to question, Now, VCC = I C RC + VCE
fission of nucleus 238 =119 + 119 ⇒ VCE = VCC − I C RC

(Y From figure, VCC = 5 V and RC = 500 Ω


Hence, total energy of nucleus before fission,
E i = 238 × 8.6 MeV ⇒ VCE = 5 − 96 × 10−4 × 500
Total energy of nucleus after fission,
= 5 − 4.8 = 0.2 V
on
E f = 238 × 7.6 MeV
∴Energy released in the process of fission 8. (d) Given, frequency, ν = kδE
= Ei − E f where, k is constant and δE is change in energy.
pi

= 238 × 8.6 − 238 × 7.6 ν ν


⇒ k= = [QδE = hν]
= 238 MeV δE hν
m

Hence, released energy is closest to 214 MeV. 1


=
h
6. (d) Given, load resistance, RL = 6 kΩ = 6000 Ω
ha

QWe know that, energy, E = hν


Base voltage, VB = 15 mV = 15 × 10−3 V E ML2 T −2
⇒ h= =
Base current, I B = 20µA = 20 × 10−6 A ν T −1
C

2 −1
Collector current, I C = 1.8 mA = 1.8 × 10−3 A h = [ML T ]
∴Voltage gain = β (Current gain) × Resistance 1 1
∴Dimension of k = = = [M−1 L−2 T]
dy

gain h [ML2 T −1 ]
I R
= C × L 9. (b) Given, A = $i + $j − k$ , B = 2i$ − $j + k$ ,
u

I B RB
1 $ $ $)
I R I R C = (i − 2 j + 2k
St

= C L = C L [QVB = I B RB ] 5
I B RB VB
$i $j k $
1.8 × 10−3 × 6000
= Q A×B=1 1 $
−1 = − 3$j − 3k
15 × 10−3
2 −1 1
= 720
1 $ $ ) (− 3$j − 3k
$)
7. (a) Given, β of transistor = 48 ∴ C ⋅ (A × B) = (i − 2$j + 2k
5
Base current, I B = 200 µA = 200 × 10−6 A 6 6
= − =0
5 5
WB JEE (Engineering) Solved Paper 2020 19
µ mg µ mg
10. (a) Given, speed of fighter plane, v = 360 km/h = =
1 + µ2 1 + µ2
5
= 360 × m/s = 100 m/s
18 1 + u2
Altitude, h = 500 m µ ⋅ mg
⇒ Fmin =
v=100m/s 1 + µ2

12. (a) Given, a tennis ball hits the floor with a speed
h=500 m v making an angle θ with the normal as shown in
the figure below.

)
be
ev cos q
ev
Now, from equation of motion,

Tu
1 v q f
h = ut + gt 2
2
v sin q O e
1 Q u = 0 and

ou
⇒ 500 = 0()t + × 10 × t 2  g =10 m/s2  f
2  
⇒ t 2 =100 ⇒ t = 10 s ev cos q
A
∴The bomb should be dropped at the distance,
x = vt = 100 × 10 = 1000 m

11. (a) The block diagram is as shown below,


(Y v sin q B
The coefficient of restitution = ε
Now, from the above diagram,
on
vsinθ tanθ
F sin q In ∆AOB, tan φ = =
εv cosθ ε
tanθ 
φ = tan−1 
N
pi

F
∴Angle of reflection, 
f= mN  ε 
q
F cos q
m

13. (c) Time period of second pendulum,T = 2 s


Displacement equation of simple pendulum,
ha

mg x = Asinωt … (i)
Given, t = 2. 25 s = 2 + 0.25
According to above diagram, 1
C

= 2+
mg = N + F sinθ 4
⇒ N = mg − F sinθ … (i) ∴ t=T+
T
dy

and F cosθ = µ N 8
⇒ F cosθ = µ (mg − F sinθ) [using Eq. (i)] Velocity of second pendulum,
µmg dx
u

⇒ F= v=
cosθ + µ sinθ dt
St

d
For Fmin , = ⋅ (Asinωt) [from Eq. (i)]
dt
d
(cosθ + µ sinθ) = 0 v = Aω cosωt

Velocity at t = T,
⇒ − sinθ + µ cosθ = 0
2π 
⇒ tanθ = µ v = A ω cos  ⋅ T  = Aω = v0 … (ii)
 T 
µ 1
Q sinθ = and cosθ = T
1 + µ2 1 + µ2 Velocity at t = T + ,
8
µmg
v1 = Aω cosω  T + 
∴ Fmin = T
1 µ ⋅µ  8
+
1+µ 2
1 + µ2 2π  T
= Aω cos T + 
T  8
20 WB JEE (Engineering) Solved Paper 2020
9 From given figure,
= Aω cos 2π ⋅
8 FS × x = FB × (15 − x)
π
= Aω cos  2π +  ⇒ 2 × 107 × x = 107 × (15 − x)
 4
⇒ 2x = 15 − x
π
= Aω cos ⇒ 3x = 15 ⇒ x = 5 cm
4
Aω 15. (c) We know that,
=
2 2r 2
terminal velocity, vT = (ρS − ρL) g
v0 9η

)
v1 = [from Eq. (ii)]

be
2 1
⇒ vT ∝ , where η is viscosity of the liquid.
By equation of motion, η
v12 = v02 − 2gh Hence, graph between terminal velocity vT and

Tu
2 2 viscosity of liquid η is as shown below.
⇒  v0  = v2 − 2gh ⇒ h = v0
  0
 2 4g vT

ou
14. (b) Let x be the distance from steel wire, where
load is applied as shown in the figure below.

(Y
h
Steel FS FB Brass
15 cm 16. (a) Given, during ab, heat rejected, Qab = − 50 J
50 cm 50 cm
on
During ca, heat absorbed, Qca = 80 J
During bc, heat transferred, Qbc = 0 J
x 15–x Work done by the gas, W = 40 J
pi

∴Area of closed curve abca = Qab + Qbc + Qca


W Area = − 50 + 0 + 80
m

Given, Young’s modulus of steel, Area = 30 J


Ysteel = 2 × 1012 dyne/cm 2
ha

17. (b) Given, length of rectangular container, l = 5 m


Young’s modulus of brass, Ybrass = 1 × 1012 dyne/cm 2 The number of moles of two gases are
Cross-sectional area of wires, A = 0.005 cm 2 µ1 =
M
and µ 2 =
M
C

32 18
Extension in wires, ∆l = 01. cm
The given situation is shown in the figure given
We know that,
dy

below.
stress FS / A FS ⋅ l
Young’s modulus, Ysteel = = =
strain ∆l ∆l A 5m
l
u

∆l
⇒ FS = Ysteel × × A
St

l A P B
.
01
= 2 × 1012 × × 0.005
50
x 5–x
⇒ FS = 2 × 107 dyne
where, x is the distance of movable partition P
Similarly,
from the left wall A.
∆l
FB = Ybrass × × A At equilibrium, p1 = p2 = p
l
.
01 and T1 = T2 = T
= 1 × 1012 × × 0.005 By ideal gas equation,
50
p1 V1 p2V2
= 107 dyne = = RT
µ1 µ2
WB JEE (Engineering) Solved Paper 2020 21
V1 V
⇒ = 2 [Qp1 = p2] 20. (c) The given system of charges is as shown
µ1 µ 2 below,
a ⋅ x a ⋅ (5 − x) q a q
= [Qa = area of each wall A and B]
µ1 µ2
x 5− x
=
µ1 µ2

a
a a

2
x 5− x
⇒ =
M / 32 M / 18

)
⇒ 32x = 18 (5 − x)

be
q a q
⇒ 32x = 90 − 18 x
1 q1 q 2 k q1 q 2
⇒ 50 x = 90 QPotential energy, PE = =
90 4 πε0 r r

Tu
⇒ x= = 1.8 m
50 Here, q1 = q 2 = q
and r=a
18. (d) Given, mass of boiling water = 100 g k q q kq 2

ou
⇒ PE = =
Mass of cold water = 300 g a a
Fall in temperature = 100 − 20 = 80°C Due to four identical point masses,
Let heat capacity of calorimeter = ms

(Y
4kq 2 2kq 2
PE i = +
Specific heat of water = 1 cal g −1 °C −1 a 2a
∴Heat lost by boiling water = ms∆T = 100 × 1 × 80 According to law of conservation of energy,
on
and heat gained by cold water KE i + PE i = KE f + PE f
= 300 × 1 × 10 + ms × 10 Initially, KE i = 0
Now, according to the principle of calorimetry, and at infinity, PE f = 0
pi

heat lost = heat gained 4kq 2 2kq 2


∴ KE f = PE i = +
100 × 1 × 80 = 300 × 1 × 10 + ms × 10 2a
m

a
⇒ ms = 500 cal/°C kq 2
= (4 + 2)
Let specific heat of metal is s b.
ha

a
Mass of block = 1 kg =1000 g QIn CGS unit, k =1
Rise in temperature of metallic block, q2
∆t = 19 − 10 = 9°C ∴ KE = (4 + 2)
C

a
Then, again from principle of calorimetry,
mmixture × s∆T + ms∆T = mblock × s b × ∆t 21. (b) Given, radius of solid cylinder = a
dy

(100 + 300) × 1 × (20 − 19) + 500 × (20 − 19) Charge density of cylinder = ρ
= 1000 × s b × 9 Dielectric constant of material of cylinder = k
u

⇒ s b = 01
. cal/g°C The electric field at a radial distance x(x < a) as
shown is calculated as
19. (a) Given, q1 = 1 × 10−6 C, q 2 = 3 × 10−6 C
St

∴ q 2 = 3q1 ; q 2 > q1
1 a
As we know that, electric field, E ∝
x2
So, the graph showing variation of E x in
x
x-direction is as given below. l
Ex

x
O (10,0)
22 WB JEE (Engineering) Solved Paper 2020
From Gauss’ law, 23. (d) According to given figure,
Q
∫ E ⋅ dA = k εin0 , where k is dielectric constant of B

material.
I
ρ(πx 2l)
⇒ E(2πxl) = [Qcharge, Qin = ρ(πx l )]
2
O r
k ε0 G I
a
ρx
⇒ E=
2k ε0

)
a

be
22. (a) A galvanometer can be converted to a
voltmeter of full scale deflection V0 by connecting Given, a : r = 8 : π
a 8 πa
a series resistance R1 , then = ⇒r = … (i)
π

Tu
V r 8
R1 = 0 − G µ 0I
Ig Magnetic field due to square, B1 = × 2,
a
π
⇒ V0 = I g (G + R1) … (i)

ou
2
where, I g = current through the galvanometer outward
µ 0I
and G = resistance of galvanometer. Magnetic field due to circular loop, B2 = ,

(Y
2r
Similarly, a galvanometer can be converted to an
ammeter by connecting a shunt resistance R2, inward
then Strength of magnetic field at the common centre
on
 Ig  O,
R2 =  G µ I 2 2µ 0 I
I − I  B = B1 + B2 = 0 −
 0 g
2r πa
pi

 R2  Substituting value of r from Eq (i), we get


⇒ I g = I0   … (ii)
 G + R2  µ I × 8 2 2 µ 0I
B= 0 −
m

where, I 0 = total current. 2πa πa


Now, from Eq. (i), we get µ 0I
= (4 − 2 2)
ha

V0 πa
− R1 = G … (iii) µ I
Ig = 0 2 2 ( 2 − 1)
πa
C

From Eq. (ii), we get


I0 24. (a) A wire is bent to form a D-shaped loop
G + R2 = × R2 carrying current I, where the curved part is
dy

Ig
semi-circle of radius R. The loop is placed in a
I0 uniform magnetic field B which is directed into
⇒ G= × R2 − R2 … (iv)
the plane of the paper. As single current by flowing
u

Ig
in the loop, so net magnetic force on a closed
St

From Eqs. (iii) and (iv), we get current loop in a uniform magnetic field B is zero.
V0 I
− R1 = 0 × R2 − R2 25. (d) For equivalent resistance, between the
Ig Ig terminals A and B of the infinite resistance
V0 I 0 network, we redraw the given circuit as
⇒ − × R2 = R1 − R2
Ig Ig 2R In parallel combination
1 1 1 = X+R
⇒ (V0 − I 0 R2) = R1 − R2 +
R X RX
Ig X R X
V − I 0 R2
⇒ Ig = 0
R1 − R2
RX
Now the 2R and is in series, so
X+ R
WB JEE (Engineering) Solved Paper 2020 23
RX QIn steady state, the combination of resistance
X = 2R +
R+ X and capacitance in loop CEFD will not work. So,
we can eliminate this combination and hence the
X 2 − 2RX − 2R2 = 0
equivalent circuit is as shown below.
2R ± 4R2 − 41
() (− 2R2) 2W i
⇒ X= i
2 B C
2R ± 12R2 2R ± 2 3R
⇒ X= = = (1 ± 3)R
2 2 4W 8W
⇒ X = (1 + 3) R

)
2V

be
26. (c) When a DC voltage is applied at the two ends A D
of a circuit kept in a closed box, it is observed that 2V
the current gradually increases from zero to a

Tu
QIn the above circuit, the current i will flow from
certain value and then remains constant. As B to C and same current i will flow from C to B.
capacitor blocks DC. So, the circuit must contains Hence, total current through the 2Ω resistor is zero.
a resistor and an inductor in series as shown

ou
below. 28. (b) A conducting wire of
length L bent in the form
of a circle of radius R and

(Y
S
another conductor of R
a\4
length a is bent in the form
of a square. The two loops
are then placed in same
on
R L
From Graph plane such that the square
loop is exactly at the centre
Growth of current, I = I 0 (1 − e − t / τ L )
of circular loop as shown in the figure
pi

At t = 0, I = 0 For circular loop,


At t = τ L, 2πR = L
m

… (i)
I = I 0 1 −  = 0.693 I 0
1 For square of side S,
 e 4S = a
ha

… (ii)
I µ 0 IS 2
Flux linked, φ =
2R
C

I0 φ µ S2
Mutual inductance, M = = 0
I 2R
0.693 I0
dy

Substituting R from Eq. (i) and S from Eq. (ii), we get


t µ a 2 2π µ 0 a 2 π
M= 0 =
L 216
( ) L 16 L
u

where, τ L = = time constant and


R
29. (a) The given situation is as shown in figure below.
St

E
I0 = = maximum current.
R

27. (c) The given circuit is as shown below.


2W 8W
C x
B E
Focal length
=30 cm
Object Plane
4W 8W 2 mF
mirror
2V Convex mirror
Object distance from convex mirror, u = − 60 cm
A D 2V F
2V Focal length of convex mirror, f = 30 cm
24 WB JEE (Engineering) Solved Paper 2020
Using mirror formula, 31. (c) Given, resistance of the circular loop = 20 Ω
1 1 1 1 1 1
+ = ⇒ − = Electric charge flowing due to the induced
v u f v 60 30 ∆φ φ2 − φ1
current, q = I ⋅ dt = = , where φ1 , φ2 are
1 1 1 2+ 1 3
⇒ = + = = R R
v 30 60 60 60 magnetic flux at t1 = 0 and t2 = 20 ms.
⇒ v = 20 cm As the loop is perpendicular to the magnetic field
Distance between the object and image = 60 + 20 B, the flux passing through the loop is,
= 80 cm φ = B ⋅ A = BA cos 90° = 0
∆φ φ2 − φ1

)
As there is no parallex between the image formed Hence, q = = =0

be
by the two mirrors, i.e. the images are formed at R R
the same place. ∴ q = 0C
So, the plane mirror is at a distance of 40 cm from
32. (b) Distance between the parallel plates = d

Tu
the object.
The axial distance of beam from the centre of
30. (a) A thin convex lens is placed just above an parallel plates = L
empty vessel of depth 80 cm. The image (I) of a distance

ou
L
coin kept at the bottom of the vessel is thus As, time, t = =
velocity v0
formed 20 cm above the lens. If now water is
poured in the vessel upto a height of 64 cm as vy
shown in the figure below.

20 cm
Image (I)
(Y d L
v0
on
Convex lens

16 cm
y-component of velocity, v y =   t = ⋅ ×
qE e V L
pi

80 cm  m m d v0
64 cm Object
eVL
⇒ vy =
m

mdv0
Given, object distance, u = − 80 cm v eLV eVL
∴ tanθ = y = =
ha

Image distance, v = + 20 cm vz mdv0 . v0 mdv02


Using lens formula, we get  eVL 
1 1 1 ⇒ θ = tan−1  
C

= − 2
 mdv0 
f v u
1 1 1 5 33. (a) Due to dragging movement of the block, work
⇒ = + =
dy

f 20 80 80 (W f ) will be done against force of friction ( f).


⇒ f = 16 cm Now, force of friction, f = µmg
where, µ = coefficient of static friction = 0.1,
u

Now, water is poured in the vessel upto height of


64 cm, then object distance, mass, m = 20 kg and g = 10 ms−2.
St

64 64 × 3 ⇒ f = 0.1 × 20 × 10 = 20 N
u = 16 + = 16 + = 64 cm
µ 4 This work done (W f ) will produce heat energy,
(given, refractive index, µ = 4 / 3) H = mc∆T
Again, using lens formula, Hence, W f = H
1 1 1
= − ⇒ Force × Displacement = mc∆T
f v u ⇒ f × vt = mc∆T [Qs = v ⋅ t]
1 1 1 Here, v = 0.5 ms−1 , t = 2.1 s
⇒ = + (Qu = −64 cm)
16 v 64 . CGS unit = 0.1 × 4.2 × 103 SI unit
and c = 01
1 3
⇒ = ⇒ 20 × 0.5 × 2.1 = 20 × 0.1 × 4.2 × 103 × ∆T
v 64
20 × 0.5 × 2.1
∴ v = 21.33 cm, above the lens. ⇒ ∆T = = 0.0025° C
20 × 0.1 × 4.2 × 103
WB JEE (Engineering) Solved Paper 2020 25
2 tan θ
34. (c) We have, power = work done As, tan 2θ =
time 1 + tan2 θ
Given, time = 1 min = 60 s and sin 2θ = 2 sin θ cos θ
Now, work done per cycle = (130 − 30) 2 9sin2 θ 9
= 100 cal = 100 × 4.2 J ⇒ = =
1 − tan θ 4sin θ cos θ 4 cos2 θ
2 2 2

As 2J of energy is consumed due to friction by the 2 9


engine, hence work done per cycle ⇒ = sec2 θ
1 − tan2 θ 4
= 100 × 4.2 − 2
2 9
= 420 − 2 = 418 J ⇒ = (1 + tan2 θ)

)
1 − tan2 θ 4

be
Total work done for 90 cycles = 418 × 90 J
418 × 90 [Q1 + tan2 θ = sec2 θ]
∴Power = = 627 W
60 Let tan2 θ = x

Tu
35. (b) Let force acting due to electrostatic attraction 2 9
= (1 + x)
of charges be Fe as shown in the figure below. 1−x 4

ou
⇒ 8 = 9 − 9x2
q q ⇒ 9x2 = 1
L L
1 1

(Y
T cos q ⇒ x2 = ⇒ x=
9 3
q T
Fe 1
q ⇒ tan θ =
2

q T sin q 3
on
O
1
⇒ tanθ = = tan 30°
L sin q L sin q 3
⇒ θ = 30°
pi

mg

Now, T sinθ = Fe …(i) Hence, the angle between the strings


m

T cosθ = mg …(ii) = 2θ = 2 × 30° = 60°


On dividing Eq. (i) by Eq. (ii), we get 36. (b) The photoelectric current depends on the
ha

T sinθ F q ×q intensity of incident radiation by relation,


= e =k 1 2 2
T cosθ mg r i ∝I
Here, q1 = q 2 = q
C

But the intensity of radiation is inversely


Distance, r = 2L sinθ proportional to the square of distance between the
kq 2 source and the photoelectric surface,
⇒ tanθ =
dy

… (iii) 1
(2L sinθ)2 ⋅ mg i.e. I∝ 2
d
Now, in the second case, when the charges are 1
∴ i∝ 2
u

tripled, new charge q ′ = 3q d


As the angle between the strings are doubled, θ
St

So, when the distance(d) is doubled, the


becomes 2θ, hence from Eq. (iii), we get photoelectric current will decrease. The stopping
k(3q)(3q)
tan 2θ = potential and maximum kinetic energy are
(2L sin 2θ)2 ⋅ mg independent of distance, so they remain same.
9kq 2 37. (d) As, inner radii are not given, so we cannot
tan 2θ = … (iv)
(2L sin 2θ)2 mg calculate their masses. Also, it is not given about
On dividing Eq. (iv) by Eq. (iii), we get the nature of material of spheres.
tan 2θ 9kq 2 4L2 sin2 θ ⋅ mg The rotational kinetic energy of sphere,
= 2 2 × 1
tanθ 4L sin 2θ ⋅ mg kq 2 (KE)rot = Iω2
2
tan 2θ 9sin2 θ where, I = moment of inertia
=
tanθ sin2 2θ and ω = angular speed.
26 WB JEE (Engineering) Solved Paper 2020
It is given that, the two metallic spheres have As the motions of pendulum and bar are
same moment of inertia (I) about their respective synchronous, so their angular velocities will be
diameters. So, their rotational kinetic energies will equal, i.e.
be equal when rotated with equal uniform ω1 = ω2
angular speed (ω) about their respective diameters. 2gsinθ 3gsinθ
⇒ =
38. (a) The velocity of the particle at extreme l l′
position, 2 3 3
⇒ = ⇒ l′ = l
v = 2gh [from v 2 − u 2 = 2gh] l l′ 2
3
∴The length of the bar is l.

)
Let when released from extreme position, the

be
pendulum traverse an angular distance of θ as 2
shown below. 39. (d) Given, R = 400 Ω, L = 250 mH = 250 × 10−3 H,
String C = 2.5µF = 2.5 × 10−6 F, VS = 5 V

Tu
I
and ω = 2 kHz = 2000 rad/s = 2 × 103 rad/s
q
∴ Inductive reactance,

ou
w1 I sin q X L = ωL = 2 × 103 × 250 × 10−3 = 500 Ω
Capacitive reactance,

(Y
1 1
XC = = = 200 Ω
v ωC 2 × 103 × 2.5 × 10−6
Here, h = l sin θ Impedance, Z = R2 + (X L − X C)2
on
Then, linear velocity, v = 2gl sinθ
= (400)2 + (500 − 200)2
v 2gl sinθ
and angular velocity, ω1 = = = 250000 = 500 Ω
l l
pi

2gsinθ The peak voltage across capacitor,


= V 5
m

l (VP)C = I P X C = S X C = × 200 = 2 V
For a bar moved through the same angular Z 500
distance θ as shown below. ∴Peak value of electrostatic energy of capacitor,
ha

1
Bar (U P) C = C(VP)C2
I¢ 2
C

1
q = × 2.5 × 10−6 × (2)2
I¢/2 sin q 2
w2 = 5 × 10−6 J
dy

or (U P)C = 5µJ
l′ 40. (a, c, d)
Here, distance travelled, h = sinθ, where l′ is the
u

2 It is given that the charged particle is moving with


St

length of bar. constant velocity in a region of no gravity, so


Potential energy = Rotational kinetic energy following cases are possible
1
mgh = Iω22 (i) The particle may move in a straight line in any
2 direction, when E = 0 and B = 0.
l′ 1 m(l ′)2  ml 2  (ii) The particle may move in a circle with constant
⇒ mg sinθ = × ω22 Q for rod, I = 
2 2 3 3  velocity. The centripetal force for circular

motion is provided by magnetic field.
(l ′)2 2
⇒ gl ′sinθ = ω2 So, E = 0 and B ≠ 0.
3
(iii) The particle may move in a helical path with
3gl ′sinθ 3gsinθ
⇒ ω2 = = constant velocity. For helical motion, the
(l ′)2 l′ condition is that both fields must be present,
i.e. E ≠ 0 and B ≠ 0.
WB JEE (Engineering) Solved Paper 2020 27

Chemistry
41. (c) Alkaline hydrolysis of an ester (carboxylic acid acids are more acidic than phenol due to more
derivative) follows acyl SN 2 mechanism. It is as stability of carboxylate ion than phenoxide ion.
follows: So, compound IV is the most acidic among the
given compounds.
O O
Further, acidicity of phenols increases by the

OH presence of electron withdrawing groups (EWG)
C C + RO–

)
OR OH such as NO 2. The effect of EWG is more

be
pronounced at ortho- and para-positions than at
E

m-position due to combined effect of −I and −R


O effect of —NO 2 group at p-position. On the other

Tu
hand, electron donating group (EDG) such as
C O– + ROH CH 3 decreases the acidity of phenols.
Rate of SN 2 mechanism depends on the polar Therefore, the correct order of acidity is as follows :

ou
nature of C == O group of COOR group.


COOH OH OH OH
Electron withdrawing group (− R > − I) increases
the rate of SN 2 as it promotes the nucleophilic
attack whereas electron donating group (+ R > + I)
decreases the rate of SN 2 reaction as it demotes
the nucleophilic attack.
(Y OCH3
>

NO2
> >
NO2
CH3
on
Here, the nature and order of functional groups
attached para to benzene ring are: (IV) (I) (III) (II)
NO 2(I) >  Me(III) >  OMe(II)
pi

( − R) ( + I) ( + R) 44. (d) Carbocations are stabilised by the presence of


electron donating group (+ R, + I , + H) and
So, the order of rate of alkaline hydrolysis is
m

destabilised by presence of electron withdrawing


I > III > II group(− R, − H , − I).
O
ha

42. (c) Given reaction proceed via shift of D − .


+ 
It is shown as follows : I. CH 2  C CH 3
( − I)
C

O
O O– O
OH – Ph C D 
Ph C q Ph C OH − I-effect of  C CH 3 destabilises the
dy

r.d.s.
carbocation. So, it is least stable carbocation.
D D
II. + +
u


O O [CH2—O CH3 CH2==OCH 3]
St

Ph C OH + Ph C D It is stabilised by + R-effect. So, it is the most


stable carbocation.
D +
III. CH 2  CH 3
O OH Hyperconjugation of —CH 3 group stabilises the
ion to some extent but not as effective as —OCH 3.
Ph C O– + Ph C D Therefore, its stability is intermediate between
I and II.
D Hence, the correct order of stability of
The alcohol formed in is Ph  CD 2  OH. carbocations is II > III > I.
So, correct option is (c). So, the correct option is (d).

43. (b) Among the given compounds, compound IV is 45. (c) NaBH 4 reduces aldehydes, ketones and acid
a carboxylic acid whereas rest are phenols. Carboxylic chlorides into alcohols. It cannot reduce ester and
28 WB JEE (Engineering) Solved Paper 2020
amides. Therefore, in the given reaction, NaBH 4 Since, five orientations of d-orbitals are
reduces only the keto group without affecting the degenerated, therefore m = − 1 can be assigned to
ester group. any one of them which means all the 3d-orbitals
The reaction takes place as: are filled. Therefore, the expected maximum
number of electrons in an atom of the given
O O O
OH quantum numbers are 30.
NaBH4
C So, the correct option is (d).
C C CH
H 3C OC2H5 Methanol
H 3C OC2H5
48. (b) The given crystal lattice is face centred cubic
Ethyl-3- oxobutanoate Ethyl-3- hydroxybutanoate
lattice, which has Zeff = 4.

)
a
Also, for fcc, atomic radius (r) =

be
Thus, correct option is (c). .
2 2
46. (a) The given reaction proceeds as follows: The arrangement of Au-atom in a fcc lattice is

Tu
O O – shown below:
O O
C C C C
CH3 –
OOCEt CH3
Et

ou
O CH Et O CH
r
Nucleophile
H
– r

(Y
O O O

O2N C C C

2r
CH O Et
on
r
H
CH3
pi

O O From the above figure, it is clear that the closest


OH H distance between two Au-atoms = 2r
m

C a a
O2N C C O
C = 2× =
Et 2 2 2
ha

H CH3 Hence, option (b) is correct.


–H2O
O O 49. (a) For 2A - B + C, K 1 = 1.0
C

[B][C]
C C K1 = … (i)
[A]2
dy

O2N CH==C O Et
For 2B - C + D, K 2 = 16
[C][D]
CH3 K2 = … (ii)
H2O/H+ [B]2
u

For 2C + D- 2P, K 3 = 25
St

O
[P]2
K3 = … (iii)
C [C]2[D]
O2N CH==C OH Add equations (i), (ii) and (iii), we get
(Product) [P]2
CH3 K1 ⋅ K 2 ⋅ K 3 =
[B][A]2
So, the correct option is (a). [P]2
25 × 16 × 1 = … (iv)
[B][A]2
47. (d) Given,
n= 3 On reversing and taking the square root of
equation (iv), we get the resultant K for the
l=2 1
reaction, P - A + B as follows:
so, subshell = 3d 2
WB JEE (Engineering) Solved Paper 2020 29
1 Thus, amount of PbI 2 precipitated will be about
K =
K1 ⋅ K 2 ⋅ K 3 = 2 × 10− 4 mol
So, correct option is (c).
[B]1 / 2[A]
=
[P] 53. (d) Given,
1 Density = 1.964 g / L
⇒ =
25 × 16 Pressure = 76 cm Hg = 1 atm
1 We know that density,
∴ K =
20 RT
d=

)
RM

be
So, the option (a) is correct answer.
0.0821 L atm / K mol × 273 K
50. (a) According to Hardy-Schulze rule, “greater the ∴ M=
1 atm × 1.964 g / L
valency of coagulating ion or flocculating ion

Tu
(oppositively charged ion) added, the greater is its = 44 g / mol
power to cause coagulation. To coagulate a Among the given gases, CO 2 gas has molecular
positively charged sol, i.e. Fe(OH)3 the maximum mass of 44 g/mol.

ou
power of coagulating of negative ion is PO 34 − . So, option(d) is correct.
So, the correct option is (a).
54. (a) Equal masses, i.e. w kg of ethane and

(Y
51. (b) Let, the number of moles of ethanol = n hydrogen are mixed in an empty container.
w
The number of moles of water = N ∴Moles of C2H 6 =
30
Now, mole fraction of ethanol
w
on
 n  Moles of H 2 =
(χ B) =   = 0.08 2
n + N From Raoult’s law, we have
Mole fraction of water (χ A) = 1-mole fraction of
pi

pH 2 = ptotal × mole fraction of H 2


ethanol = 0.92
w
χ B × 1000
m

Molality = 2
(1 − χ B) × M A or pH 2 = ptotal ×
w w
+
1000 × mole fraction of ethanol
ha

∴m = 2 30
(1 − mole fraction of ethanol) 15
∴ pH 2 = ptotal ×
× molecular mass of H 2O 16
C

1000 × 0.08
= = 4.83 mol / kg pH 2 15
(1 − .08) × 18 =
ptotal 16
dy

So, the correct option is (b).


Thus, fraction of total pressure exerted by
52. (b) The balanced chemical reaction is hydrogen is 15:16.
Pb(NO 3)2(aq) + 2KI(aq) → PbI 2 ↓ + 2KNO 3(aq)
u

So, the correct option is (a).


Initial moles of Pb(NO 3)2 = 5 × 0.1 × 10− 3 55. (c) For the work against vacuum, W = 0
St

= 5 × 10− 4 For an adiabatic process, q = 0


Initial moles of KI = 10 × 0.02 × 10− 3 From first law of thermodynamics, we have
= 2 × 10− 4 ∆U = q + W
Therefore, ∴ ∆U = 0
Pb(NO 3)2(aq) + 2KI(aq) → PbI 2 ↓ + 2KNO 3(aq) Hence, the correct option for the process is (c).
Initially 5 × 10 − 4 2 × 2 × 10 − 4 0 0 56. (c) Given,
It is clear that for the formation of 1 mole of PbI2, mass of ethylene glycol = 10 g
we require 4 × 10− 4 moles of KI. But the given
molecular mass = 62 g / mol
moles of KI are 2 × 10− 4 . Hence, it is a limiting
K f (water) = 1.86 K kg mol − 1
reagent and the amount of PbI 2 formed will
depend upon the moles of KI. As we know that,
30 WB JEE (Engineering) Solved Paper 2020
∆Tf = K f × molality So, the difference between orbital angular
h h
= 1.86 × 
10 1000  momentum = 2 3 − 0= 2 3
× 
 62 90  2π 2π
= 33
. °C Therefore, the difference between orbital angular
momentum of an electron in a 4 f-orbital and
∴Temperature at which the ice begins to separate
another electron in 4s-orbital is 2 3.
= − 33
. ° C.
Thus, the correct option is (a).
So, the correct option is (c).
mass
57. (a) Given, 62. (d) Number of atoms = × NA ×
molar mass

)
Radius of orbit of H-atom = 0.53 × 10− 8 cm number of atoms in 1 mole.

be
For first Bohr orbit, n = 1 The calculation of number of atoms in given
As we know, options are as follow :

Tu
nh (a) Number of atoms in 1 g of Ag
mvr =
2π 1
= × N A × 1 = 0.0092 N A
nh 107.87
v=

ou
2πmr (b) Number of atoms in 1g of Fe
1 × 6.626 × 10− 27 1
v= = × N A × 1 = 0.0179 N A
2 × 314
. × 91 . × 10− 28 × 0.53 × 10− 8 55.85
v = 2.188 × 108 cm / s
So, the option (a) is correct. (Y (c) Number of atoms in 1g of Cl 2
=
1
70
× N A × 2 = 0.0285 N A
on
58. (d) Among the given statements, option (d) is (d) Number of atoms in 1g of Mg
incorrect. It can be corrected as, F is more
1
electronegative than O because it is the most = × N A × 1 = 0.0411 N A
pi

electronegative element in periodic table. 24.30


Rest all the given statements are true. Thus, 1g of Mg contains the maximum number of
m

So, correct option is (d). atoms among the given options.

63. (d) +
ha

59. (a) The electronic configuration of uranium is NH3


86 3 1
: [Rn] 5 f 6d 7s . 2
Cl NH3
92 U –
Cr Cl
It consist of your electrons, i.e. three from f-orbital Cl NH3
C

and one from d-orbital.


NH3
So, the correct answer is option (a).
dy

Let the oxidation state of central atom (Cr) be x.


60. (c) On prolonged electrolysis, the density of liquid
Then,
water increases as the proportion of D 2O increases.
(x × 1) + (4 × 0) + (2 × − 1) = + 1
This is because on prolonged electrolysis, the
u

concentration of heavy water (D 2O) in liquid water x − 2= + 1


St

increases which has higher density than ordinary x=+ 3


water (H 2O). If electrolysis continued, then almost Thus, the correct IUPAC name of complex is
pure D 2O is obtained. cis-tetraamminedichlorochromium (III) chloride.
61. (a) Orbital angular momentum = l(l + 1) h . So, the correct answer is (d).

64. (b) We know,
For 4 f-orbital, l = 3,
mass of N A atoms = 12 g of C
h
∴ Orbital angular momentum = 3(3 + 1) or mass of 6.023 × 1023 atoms = 12 g

12
=2 3
h ∴Mass of 1 atom = g = 1.9923 × 10− 23g
2π 6.023 × 1023
For 4s-orbital, l=0 So, the correct option is (b).
∴ Orbital angular momentum = 0.
WB JEE (Engineering) Solved Paper 2020 31
65. (b) We know that, 69. (c) The given reaction proceeds as follows:
Bond order CH3

¾
1
= [number of electrons in bonding orbitals – H + +
2 Me3C—CH==CH2 CH3—C—CH—CH3

¾
number of electrons in antibonding orbitals] (2°carbocation)
HBr CH3
The electronic configuration and bond order less stable
(B.O.) of given options is as follows: CH3 Methyl shift

(i) He2(4 e −) = σ(1s)2 σ * (1s)2

¾
* CH3 H

)
H3C—C—–CH—CH3
2− 2

¾
¾
¾

be
B.O = =0 CH3—C — C—CH3
2 CH3 Br +

¾
(ii) He+2 (3 e −) = σ(1s)2 σ * (1s)1 (Minor product) CH3

Tu
( Exists in two (3°carbcation)
2−1 1
B.O = = stereoisomers R and S) more stable
2 2
+Br
(iii) He22 + (2 e −) = σ(1s)2

ou
CH3 H
2− 0
B.O = =1

¾
2 CH3—C — C—CH3
Therefore, the bond order of He2, He+2 and He22 + are

(Y

¾
1 Br CH3
respectively 0, , 1.
2 (Major product)
on
66. (c) Colourless, efflorescent sodium salt is Na2S2O 3. Therefore, the total number of alkyl bromides
The colourless gas evolved in SO 2. Complete (including stereoisomers) formed in the reaction
are 3. Thus, the correct option is (c).
pi

reaction is as follows :
Na 2S2O 3(s) + 2H + (dil.) → 70. (d) The given reaction can be completed as
m

(Colourless, efflorescent) follows:


S(s) + 2Na+(aq) + SO 2 ↑ + H 2O()
l Mg
ha

White ppt. Cl Br ClMg MgBr


ether
Now, when the colourless gas is passed through it
acidified dichromate solution turns green.
C

The reaction taking place is (2eq.) HCHO

K 2Cr2O 7(aq) + 3SO 2(aq) + 2H + (aq) →


+d d– –d d+
dy

2K + (aq) + 2Cr 3+ (aq) + H 2O()


l + 3SO 4− 2(aq) ClMg O —CH2 CH2— OMgBr
(Green)

So, the correct option is (c). H+/H2O


u

67. (c) Given reaction is an example of thermite


St

process. In thermite reaction, aluminium metal is HO—CH2 CH2OH


oxidised by an oxide of another metal most
commonly iron oxide. (Product)
So, Fe is the metal (M). So, the correct option is (d).
Hence, option (c) is correct.
71. (d) Compounds which do not possess any type of
68. (a) For the extraction of Ca by electro-reduction symmetry are referred as asymmetric.
of molten CaCl 2 some CaF2 is added to the Among the given options, only compound (d) do
electrolyte because CaF2 decreases the melting not possess any type of symmetry, i.e. absence of
point of CaCl 2. It keep the electrolyte in liquid plane of symmetry and centre of symmetry, so
state of temperature than the m.p. of CaCl 2 compound (d) is asymmetric. Options (a), (b) and
So, option (a) is correct. (c) have plane of symmetry.
Hence, correct option is (d).
32 WB JEE (Engineering) Solved Paper 2020
72. (b) Rate law : r = K [A]α [B]β 74. (c) As the gas molecule shows 2-electron
magnetic moment which means it contains 2
As the t1 / 2 does not depend on the concentration unpaired electrons.
of B. Therefore, order w.r.t. B follows first order
Among the given gases, O 2 contains 2 unpaired
kinetics.
electrons.
∴ B =1
O 2 = 8 + 8 = 16
Now, according to the question,
Electronic configuration of O 2
r2 [2A]α [2B]β = σ1s 2 < σ *1s 2 < σ 2s 2 < σ * 2s 2
=
r1 [A]α [B]β < σ 2pz2 < π 2px2 = π 2p 2y < π * 2p1x = πp1y < σ * 2pz

)
be
⇒ 4 = 2α ⋅ 2β 1 5
∴ Bond order of O2 = (10 − 5) = = 2.5
2 =2
2 α+ β 2 2
Now, in oxidised form, O 2 exists as O +2 ,

Tu
α+β=2 −
O 2 −e→ O 2+ (15 e −)
As, β = 1 because B follows first order kinetic.
α =1 So, bond order of O +2 = 2

ou
∴Overall order = 2 As bond ordder ∝
1
Hence, unit of rate constant for second order Bond length
= L mol − 1s− 1 .

(Y
The bond length, in the oxidised form, of molecule
So, the correct option is (b). gets decreased. Thus, the gas molecule is O 2.

73. (d) Given, 75. (d) For the given reaction statement (d) is correct
on
K sp (SrCO 3) = 7.0 × 10− 10 and applicable. For this reaction, a mixed SN1 and
SN 2 pathway is followed. The given reaction can
− 10
K sp (SrF2) = 7.9 × 10 proceed via SN1 pathway, So as,
pi

[CO 23 − ] = 1.2 × 10− 3 M CH 3  O  CH 2  Cl →


+
Now, for the reaction,
m

[CH3—O —CH2 CH3—O ==CH2]


2+
SrCO 3 - Sr + CO 23 −
Here, the carbocation is stabilised by resonance.
ha

K sp = [Sr 2 + ][CO 23 − ] The reaction can also be proceeded via SN 2 path,


since the transition state is resonance stabilised.
7 × 10− 10 = [Sr 2 + ] × 1.2 × 10− 3

C

d+
7 × 10− 10 OH
∴ [Sr 2 + ] = CH3 — O— CH2—Cl CH3 —O—CH2 —OH
1.2 × 10− 3
dy

Also, SrF2 - Sr 2 + + 2F − So, the correct option is (d).


2+ − 2
K sp = [Sr ] [F ] 76. (a, b) Both reactions (a) and (b) give a meso-
u

K sp compound as the main product the reactions are as


[F − ]2 = follows :
St

2+
[Sr ] Br
K sp H
[F − ] = 2+ (a)
Br2
[Sr ] CH2Cl2
H
− 10 Br
7.9 × 10
= Meso-compound
7 × 10− 10
1.2 × 10− 3 H H
− −2 H2
∴ [F ] = 3.7 × 10 M (b) H3C CH3
Pd-C
So, correct option is (d).
Meso-compound
WB JEE (Engineering) Solved Paper 2020 33
H H 79. (a, c, d) SiO 2 is attacked by HF, hot NaOH and
(c) CººC C==C fluorine. Reactions involved are as follows :
H2
Lindlar’s catalyst (a) SiO 2(s) + 6HF(aq) → H 2SiF6(aq) + 2H 2O()
l
Br (c) SiO 2(s) + (hot) 2NaOH(aq) →
H Na 2SiO 3(s) + H 2O()
l
(d) Br2
H (d) SiO 2(s) + F2(g) → SiF4 (aq) + O 2(g)
CCl4
H Br So, the correct options are (a) (c) and (d).
H

)
80. (a, c) Complete reaction is as follows :
So, options (a) and (b) are correct.

be
Na / NH (liq.)
Me  C ≡≡ C  Me  
3
→
77. (a, b, d) For spontaneous polymerisation, EtOH, − 33° C

∆G = negative Me

Tu
H
Polymerisation process involves the following C==C
reaction: H (x) Me
nA → — ( A− A− A− A— )n

ou
So, ∆S = negative due to association of atoms. Dill. KMnO4

Also, we know that,


H

(Y
∆G = ∆H − T∆S or ∆H = ∆G + T∆S Me Me OH
Hence, ∆H = negative. C C H H C C H
+
So, correct options are a, b and d.
HO
on
H Me HO Me
78. (a, b, d) Statements (a, b and d) are incorrect
whereas statement (c) is correct. The corrected
form are as follows : Me Me
pi

● A sink of SO pollutant is ocean.


2 H OH + OH H
FGD (Flue-gas desulfurisation) is a process of
m

● HO H H OH
removing SO 2 from atmosphere. Me Me
ha

● The catalyst used to convert CCl 4 to CF4 by HF is


Racemic mixture
AgF.
So, the correct options are (a) and (c).
So, the correct options are (a), (b) and (d).
C
dy

Mathematics
u

1. (a) We have, d2y dy


⇒ x2 + x = − n2 y
St

n dx 2 dx
cos− 1   = log  
y x
 b  n d2y dy
⇒ x2 2 + x + n2 y = 0
dx dx
⇒ y = b cos(n log x − n log n)
On differentiating both sides w.r.t.x, we get 2. (a) We have,
dy n φ(x) = f (x) + f (1 − x) and f ′′(x) < 0 in [0, 1]
= − b sin(n log x − n log n) ×
dx x φ′(x) = f ′(x) − f ′(1 − x)
dy For monotonic increasing, f ′(x) − f ′(1 − x) ≥ 0
⇒ x = − nb sin(n log x − n log n)
dx ⇒ f ′(x) ≥ f ′(1 − x)
Again differentiating both sides w.r.t. x, we get Since, f ′(x) is decreasing.
d 2 y dy n2b cos(n log x − n log n) 1
x 2 + =− ∴x ≤ 1 − x ⇒ x ≤
dx dx x 2
34 WB JEE (Engineering) Solved Paper 2020
⇒ φ(x) is monotonic increasing in  0,  and ⇒ I=0+ 2 −1 + 2 3 − 2 2 + 6 − 3 3
1
 2
⇒ I = 5− 2− 3
monotonic decreasing in  , 1
1
 2  6. (d) Given, curve
y2 = x 3
3. (c) Let
f (x)φ′(x) + φ(x) f ′(x) On differentiating both sides w.r.t. x, we get
I= ∫ ( f (x)φ(x) + 1) f (x)φ(x) − 1
dx
2y
dy
= 3x 2
dx
Put f (x)φ(x) − 1 = t 2 dy 3x 2

)
⇒ =
⇒( f (x)φ′(x) + φ(x) f ′(x))dx = 2t dt

be
dx 2y
2tdt
∴ I=∫ 2  dy  3(m4) 3
(t + 2)t ⇒   = = m
 dx  ( m 2 , m 3) 2m3 2

Tu
dt
⇒ I = 2∫ 2 3
t + 2 ∴Slope of tangent at (m2 , m3) is m.
2
tan− 1   + c
2 t

ou
= Now slope of normal at (M 2 , M 3) to the curve
2  2
y 2 = x 3 is
f (x)φ(x) − 1
⇒ I= 2 tan− 1 + c dx − 2y

(Y
2 − =
dy 3x 2
4. (d) We have,  dx  − 2M 3 − 2
− 2n
⇒ −  = =
10 10 2n + 1  dy  ( M 2 , M 3) 3M 4 3M
on
I= ∑ ∫ sin x dx + ∑ ∫ sin
27 27
x dx
n =1 − 2n − 1 n =1 2n Given, slope of tangent = slope of normal
3m − 2
10  − 2n 2n + 1  ∴ =
pi

I = ∑  ∫ sin27 x dx + ∫ sin 27
x dx  2 3M
n =1
 − 2n − 1

 −4
2n ⇒ mM =
m

− 2n 9
I1 = ∫ sin
27
Let x dx 7. (c) Given, x 2 + y2 = a 2
ha

− 2n − 1
On differentiating both sides w.r.t. x, we get
put x = − t ⇒ dx = − dt dy
When x = − 2n − 1 2x + 2y =0
C

dx
⇒ t = 2n + 1 and x = − 2n dy − x
⇒ =
⇒ t = 2n dx y
dy

2n
2 2
∴ I1 = ∫ sin (− t) (− dt)  dy  = x
27
⇒  
2n + 1  dx  y2
u

2n a 2
= ∫ sin 1 +   dx
27 dy

St

t dt Let I=
2n + 1  dx 
0
 2n10 2n + 1  a
x2
Hence, I = ∑  ∫ sin27 x dx + ∫ sin27 x dx  I= ∫ 1+ dx
n =1  y2
 2n + 1 2n  0
a
10  2n + 1 2n + 1  x 2 + y2
I = ∑  − ∫ sin27 x dx + ∫ sin27 x dx  = 0 ⇒ I= ∫ y2
dx
n =1
 2n 2n  0
a
a
⇒ I= ∫ [Q x 2 + y 2 = a 2]
2 dx
5. (b) Let I = ∫ [x 2] dx 0 a − x
2 2

0 a
I = a sin− 1 
x
1 2 3 2 ⇒
I = ∫ 0. dx +  a  0
⇒ ∫ dx + ∫ 2dx + ∫ 3 dx
0 1 2 3
WB JEE (Engineering) Solved Paper 2020 35
I = a sin− 1 − sin− 1 0
a dv dx
⇒ ⇒ − =
 a  cosec v x
⇒ I = a[sin− 1 1 − sin− 1 0] dx
π π
⇒ ∫ − sin v dv = ∫ x
= a  − 0 = a
 2  2 ⇒ cos v = log xc
y
⇒ cos = log cx …(i)
8. (c) We have, x
n
1  j π
n→ ∞
lim ∑ f 
 n
Put x = 1, y = in Eq. (i), we get
2

)
j =0n
π

be
By standard formula, we write cos = log c
n 1 2
1  j
lim ∑ f  =
 n ∫ f (x) dx ⇒ log c = 0

Tu
n→ ∞
j =0n 0 ⇒ c =1
lim   (as j = 0) = 0
j y
Where, cos = log x
n→ ∞  n x

ou
lim   (as j = n) = 1
j
and 11. (c) We have,
n → ∞  n
f (x) = 1 − x2
9. (c) We have,


y′ + yf ′(x) − f (x) f ′(x) = 0
y′ + yf ′(x) = f (x) f ′(x)
(Y f (x) = 1 − | x |
1 + x , x < 0
f (x) = 
1 − x , x ≥ 0
on
Which is a linear differentiable equation of the
dy Graph of f (x)
form + Py = Q
dx Y
pi

∫ f ′( x ) dx
∴ I.F = e =e f ( x)
(0,1)
m

f(x
x
Solution of given differential equation is
1+

y ⋅ e f ( x) =∫ f (x) f ′(x) ⋅ e dx )=
)=

f ( x)
1–
f (x

x
ha

X¢ X
y⋅ e f ( x)
= ∫ te t dt [Qput f (x) = t ⇒ f ′(x) dx = dt] (–1,0) O (1,0)

⇒ y ⋅ e f ( x ) = te t − e t + c
C

⇒ y ⋅ e f ( x ) = f (x)e f ( x ) − e f ( x ) + c

⇒ y = f (x) − 1 + ce − f ( x)
dy

Clearly, f (x) has maxima at x = 0.


⇒ y + 1 = f (x) + ce − f ( x)

When x → ∞, f (x) = 0 and y = 0 12. (a) Given,


u

∴ 0+1= 0+ c f (x) = 2x 3 − 9ax 2 + 12a 2 x + 1


St

⇒ c =1 f ′(x) = 6 x 2 − 18ax + 12a 2


Hence, y + 1 = f (x) + e − f ( x ) For maxima or minima f ′(x) = 0
∴ 6 x 2 − 18ax + 12a 2 = 0
10. (b) We have,
  x 2 − 3ax + 2a 2 = 0
x sin   dy =  y sin   − x  dx
y y
 x   x  (x − 2a) (x − a) = 0
⇒ x = 2a, a
= − cosec  
dy y y

dx x  x f ′ ′(x) = 12x − 18a
dy dv ∴ f ′ ′(2a) = 24a − 18a > 0
put y = vx ⇒ = v+ x
dx dx and f ′ ′(a) = 12a − 18a < 0
∴ v+ x
dv
= v − cosec v ∴ f (x) is maximum at x = a
dx
36 WB JEE (Engineering) Solved Paper 2020
and minimum at x = 2a ∴ Radius of circle = | a |2 − b
∴ p = a, q = 2a
= (4 + 9) − 4
Given, p2 = q
= 13 − 4 = 3
⇒ a 2 = 2a
⇒ a=2 17. (c) The expression ax 2 + bx + c has the same sign
as that of a for all x if D ≤ 0
13. (a) 6a + 10b
5b 3a ∴ b 2 − 4ac ≤ 0
We know that, AM ≥ GM

)
6a 10b 18. (d) In 12 storied building 3 person leave the lift

be
+ any 10 floors.
∴ 5b 3a ≥ 6a × 10b
2 5b 3a ∴Total number of ways
10!
6a 10b =10 P3 = = 10 × 9 × 8 = 720

Tu
⇒ + ≥ 2×2= 4 7!
5b 3a
n −1
Hence, least possible value of
6a 10b
+ is 4. 19. (b) n Cm = k ⋅ Cm − 1

ou
5b 3a n! (n − 1)!
⇒ = k⋅
14. (a) We have, m! (n − m)! (m − 1)! (n − m)!
2log (x + 1) − log(x 2 − 1) = log 2 n(n − 1)! k ⋅(n − 1)!

(Y
⇒ =
(x + 1)2 m(m − 1)! (m − 1)!
⇒ log 2 = log 2
x −1 ⇒ n = mk
on
⇒ (x + 1) = 2 (x − 1)
2 2
20. (a) Since, AM ≥ GM
⇒ x + 2x + 1 = 2x − 2
2 2
1 + 3 + 5 + 7 + K + (2n − 1)
1
∴ > (J(n)) n
pi

⇒ x 2 − 2x − 3 = 0 n
1
⇒ (x − 3)(x + 1) = 0 n2
m

⇒ > (J(n)) n
⇒ x = 3, x ≠ − 1 n
⇒ nn > J(n)
ha

15. (c) Let complex number


P = cos θ + i sin θ ⇒ I(n) > J(n)
∴ | P | = 1and θ∈[0, 2π) 21. (b) (1 + x) = c 0 + c1 x + c 2 x 2 + K + c15 x15
15
C

Now, P 4 = (cos θ + i sin θ)4


c1 c c c
Now, + 2 2 + 3 3 + K + 15 15
⇒ P 4 = cos 4θ + i sin 4θ c0 c1 c2 c14
dy

Imaginary part of P 4 = 0 15 + 1 − 1 215


( + 1 − 2) 315 ( + 1 − 3)
+ +
∴ sin 4θ= 0 1 2 3
( + 1 − 15)
1515
u

nπ + ... +
⇒ 4θ = nπ ⇒θ = 15
4
St

0 ≤ θ ≤ 2π  n
C n+1− r 
Q n r = Here, n = 15
nπ  Cr − 1 r 
∴ 0≤ < 2π
4
∴ 15 + 14 + 13 + K + 1
= 0 ≤ x< 8 15 × 16
⇒ n= 8 = = 120
2
Hence, 8 such complex number.
3 − t 1 0
16. (b) Given, 22. (d) Given, A =  − 1 3 − t 1
 
zz + (2 − 3i)z + (2 + 3i) z + 4 = 0  0 − 1 0
This is the form of
| A |= (3 − t) (0 + 1) − 1(0) + 0
zz + az + az + b ⇒ |A | = 3 − t
Here, a = 2 + 3i, b = 4
WB JEE (Engineering) Solved Paper 2020 37
Given,| A | = 5 0 1 1
∴ 3− t = 5 A = 2abc 2 a + b b a
⇒ t=−2 b b+ c c

 12 24 5 Apply C2 → C2 − C3
23. (c) Given, A =  x 6 2 0 0 1
  A = 2abc 2 a + b b − a a
 − 1 − 2 3
b b c
Since, A is not invertible.
A = 2abc 2 |(a + b)b − (b − a) b |

)
∴ | A| = 0

be
| A | = 12 (18 + 4) − 24 (3x + 2) + 5(− 2x + 6) = 0 = 2abc 2 | ab + b 2 − b 2 + ab|
⇒ 264 − 72x − 48 − 10 x + 30 = 0 A = 4a 2b 2c 2

Tu
⇒ 82x = 246 ⇒ x = 3 ∴ k= 4
a b 26. (d) We have,
24. (a) Given, A =  , | A| = 1 and | A − λI | = 0 has
 c d f :S→ R

ou
imaginary roots.  a b 
f   = ad − bc
|A | = ad − bc = 1
 c d  
a − λ b 
A − λI = 
 c d − λ 
| A − λI | = (a − λ) (d − λ) − bc (Y  4
f 
 2
3 
 = 12 − 6 = 6
3 
  2 6 
on
|A − λI | = ad − (a + d) λ + λ2 − bc f    = 12 − 6 = 6
 1 6 
= λ2 − (a + d)λ + 1 (Q ad − bc = 1)
∴Hence, f is not one-one.
| A − λI |has imaginary roots.
pi

Since, O ∈ R but S does not contain any singular


∴ (a + d)2 − 4 < 0 ⇒ (a + d)2 < 4 matrix.
m

a2 bc c 2 + ab ∴ f is not onto.
25. (d) Let A = a + ab2
b2 ca Hence, f is neither one-one nor onto.
ha

ab b 2 + bc c2 27. (b) Given,


Taking common a, b, c from C1 , C2 and C3 aρb = a − b is zero or irrational.
C

respectively. For reflexive,


a c c+ a (a , a) = a − a = 0. Hence, ρ is reflexive.
dy

∴ A = abc a + b b a For symmetric,


b b+ c c (a , b) ∈ρ ⇒(b , a) ∈ρ
⇒ a − b is zero or irrational then b − a is zero or
u

Apply R1 → R1 + R2 + R3 irrational.
2(a + b) 2(b + c) 2(c + a)
St

∴ ρ is symmetric.
A = abc a+ b b a For transitive,
b b+ c c If a = 3, b = 3 and c = 5
a+ b b+ c c+ a (a , b) ∈ρ, (b , c) ∈ρ ⇒(a , c) ∉ρ
A = 2abc a + b b a ∴ ρ is not transitive.
b b+ c c
28. (b) Let the unit vector in ZOX plane be
Apply R1 → R1 − R2 a = x$i + zk $ ,|a | = 1
0 c c
a ⋅ α = |a ||α | cos 45°
A = 2abc a + b b a
$ ) ⋅(2$i + 2$j − k
⇒ ( x i + zk
$ $ ) =1 × 3 × 1
b b+ c c 2
$]
[Qα = 2$i + 2$j − k
38 WB JEE (Engineering) Solved Paper 2020
3 dy
2x − z = ⇒ = y + e x (− A sin x + B cos x)
2 dx
and a ⋅ β = |a ||β | cos 60° d 2 y dy
1 ⇒ = + e x ( − A sin x + B cos x)
⇒ (x$i + zk$) ⋅($j − k$) = 1 × 2 × [Qβ = $j − k$ ] dx 2 dx
2 + e x (− A cos x − B sin x)
1
−z=
d 2 y dy  dy
2 ⇒ = + − y − y
z=−
1
and x =
1 dx dx  dx 
2 2 d 2 y 2dy
⇒ = − 2y

)
1 $ 1 $
∴ a = i− k dx 2 dx

be
2 2
d 2 y 2dy
⇒ − + 2y = 0
29. (d) Four person A, B, C, D throw an unbiased die, dx 2 dx
turn by turn, in succession till one gets an even

Tu
number and win the game. 33. (d) We have,
π
Probability of an even number =
1 r cos  θ −  = 2
2  3

ou
π π
r  cos θ cos + sin θ sin  = 2
1
∴ P(A) = P(B) = P(C) = P(D) =
2  3 3 

(Y
1 r cos θ r sin θ ⋅ 3
P(A) = P(B) = P(C) = P(D) = + = 2,
2 2 2
Required probability
x+ 3y = 4 [Q x = r cos θ, y = r sin θ]
= P(A) + P(A) P(B) P(C) P(D) P(A)
on
+ P(A) P(B) P(C) P(D) P(A) P(B) P(C) P(D) P(A) + K ∴It represents a straight line.
5 9
1 1 
+   +   + K
1 34. (d) Let (h, k) be the centre and r be the radius of
=
pi

2  2  2 the variable circle.


1/ 2 1/ 2 8 Since, variable circle touches the circle x 2 + y 2 = a 2
= = =
m

1 − (1 / 2)4 15/16 15
∴ h2 + k2 = a ...(i)
30. (b) Given, P = 10 = 1
ha

Also variable circle touches the circle x 2 + y 2 = 4ax


100 10
q =1 − P =1 −
1
=
9 ∴ (h − 2a)2 + k2 = 2a + r ...(ii)
C

10 10
From Eqs. (i) and (ii), we get
50 1
P(X ≥ 1) ≥ = (h − 2a)2 + k2 − h2 + k2 = a
100 2
dy

P(X ≥ 1) = 1 − P(X = 0) ∴Locus of centre of variable circle is


0 n
(x − 2a)2 + y 2 − x 2 + y 2 = a
P(X ≥ 1) = 1 − C0    
n 1 9
u

 10   10  which represents the hyperbola.


n
St

1 −   ≥
9 1
35. (b) We have, x 2 + 2xy + ay2 = 0 and
 10  2
ax 2 + 2xy + y 2 = 0 have common line.
n
 9 ≤1
  y = x satisfies both lines.
 10  2
∴ x 2 + 2x 2 + ax 2 = 0
For n = 7 the least number of round to must so fire.
3x 2 + ax 2 = 0 ⇒ a = − 3
31. (c) By using Sandwich theorem. Equation of lines are
32. (b) We have, x 2 + 2xy − 3y 2 = 0 and − 3x 2 + 2xy + y 2 = 0
y = e x (A cos x + B sin x) (x + 3y) (x − y) = 0 and (3x + y) (x − y)= 0
dy The other lines are x + 3y = 0 and 3x + y = 0.
= e x (A cos x + B sin x) + e x (− A sin x + B cos x)
dx
WB JEE (Engineering) Solved Paper 2020 39
36. (b) Given lines, In ∆OAB
AB
4 x + 2y = 9 and 2x + y + 6 = 0 sin α =
6 9 OA
Lines having distance from origin are and
2 5 5 ⇒ OA = AB cosec α = 2at cosec α
respectively. ∴ OA = 2a(2 cot α) cosec α
The point O divide the segment PQ in the ratio OA = 4a cot α cosec α
=
9
÷
6
= 3: 4 ∴Length of chord = 4a cot α cosec α
2 5 5 2 2
40. (d) A double PQ of the hyperbola x 2 − y2 = 1

)
37. (a) Given curve a b

be
x + 2y = a and 2x + y = a
2 2 2 2 2 2 Such that ∆OPQ is an equilateral where O is origin.
Let P(a sec θ, b tan θ) and Q(a sec θ, − b tan θ)
Y (0, a)
∆OPQ is an equilateral

Tu
(0, a/√2) a/√3, a/√3
∴ OP = PQ = OQ
a sec θ + b tan2 θ = 4b 2 tan2 θ
2 2 2

ou
X′ X
O (a, 0) a 2 sec2 θ = 3b 2 tan2 θ
b2 sec2 θ 1
(a/√2, 0) = = cosec2 θ
2
3 tan2 θ 3

(Y
a
Y′
b2 cosec2 θ 1
 a2 a/ 2  e = 1+ = 1+ = 3 + cosec2 θ
∴ Required area =  + ∫a/ a − 2x dx  × 2
2 2 2
a 3 3
 6 
on
3
2
e> [Qcosec2 θ ≥ 1]
a2
tan−1  
1 3
=
2  2
pi

41. (c) Equation of ellipse


38. (a) Given, the radius of circle = 17 x2 y2
+ =1
m

With centre on positive sides of X-axis 25 9


∴ Equation of circle is (x − a)2 + y 2 = 17
ha

Since, it passes through (0, 1) B (0,3)


∴ a 2 + 1 = 17
C

⇒ a=4
∴ Equation of circle is S¢ S(4,0)
(–4,0)
x 2 + y2 − 8 x − 1 = 0
dy

B¢ (0,–3)
39. (b) Given, parabola y2 = 4ax
u

A (at2,2at)
Here, B = (0, 3), B′ = (0, − 3)
St

S = (4, 0), S ′ = (0, − 4)


1
a Area of rhombus SBS ′ B′ = SS ′ × BB′
2
O B (at2,0) 1
= × 8 × 6 = 24 sq units
2

42. (d) Given, equation of latusrectum of parabola is


x+ y=8
Let the point A on parabola (at 2 , 2at) in ∆OAB and equation of tangent at vertex is x + y = 12
at 2 t Since both are parallel.
cot α = =
2at 2 ∴ The distance between latusrectum and equation
t = 2 cot α of tangent at vertex is ‘ a ’
40 WB JEE (Engineering) Solved Paper 2020
∴ a=
4
=
4
=2 2  5 − 1
∴ Domain of f (x) is  0, .
12 + 12 2  2 
∴ Length of latusrectum = 4a = 8 2 unit
47. (c) Given, y = 2x 2 − 3x + 2
43. (*) The equation of the plane through the points dy
= 4x − 3
(2, − 1, − 3) and parallel to the line, dx
x −1 y + 2 z x y −1 z − 2 x changes from 2 to 1.99
= = and = =
2 3 −4 2 −3 2 ∴ ∆x = 1.99 − 2 = − 0.01
x − 2 y+1 z+ 3

)
The differential of y when x charges 2 to 1.99 is

be
2 3 −4 =0
dy = (4 x − 3) ∆x
2 −3 2 dy = (8 − 3) [− (0.01)] [Qx = 2]
⇒ (x − 2) (6 − 12) − (y + 1) (4 + 8) dy = − 0.05

Tu
+ (z + 3) (− 6 − 6) = 0
⇒ − 6 x + 12 − 12y − 12 − 12z − 36 = 0
48. (c) We have,
1
⇒ − 6 x − 12y − 12z − 36 = 0

ou
 1 + cx  x
lim   =4
⇒ x + 2y + 2z + 6 = 0 x → 0  1 − cx 

lim   1
44. (b) Given line, 1 + cx
x−2 y−3 z−4
3
=
4
=

and plane 2x − 2y + z = 5
5 (Y ⇒


ex → 0 
 1 − cx
lim 
ex → 0 
− 1 × = 4
 x
2 cx  1
 × =4
on
 1 − cx  x
Angle between line and plane is
(3)(2) + 4(− 2) + 51
() ⇒ e 2c = 4
sin θ =
pi

32 + 42 + 52 22 + (− 2)2 + ()
12 1
1 + 2cx  x
6−8+ 5 Now, lim   = e 4 c = (e 2c)2 = (4)2 = 16
sinθ = x→ 0  1 − 2cx 
m

50 × 3
1 2 49. (a) Given,
ha

sin θ = =
5 2 10 f is twice continuously and differentiable and
1 = f ′(0) = 0
f (0) = f ()
45. (c) Given, f (x) = sin x + cos ax ∴ f (x) = Kx 2(x − 1)
C

sin x is periodic with 2π. f (x) = K (x 3 − x 2)


f (x) is periodic only on sin x and cos x both are
f ′(x) = K (3x 2 − 2x)
dy

periodic.
2π 2
∴ cos ax is periodic at . f ′(x) = 0 ⇒ x = 0and
a 3
u

2π Hence, Rolle’s theorem is applicable in f ′(x).


LCM of 2π and is possible only a is rational
St

a
∴ f ′ ′(c) = 0 for some c ∈ R
number.
50. (b) We have,
46. (c) Given, f (x) =  1 − x + 1  f (x) = x13 + x11 + x 9 + x 7 + x 5 + x 3 + x + 12
 x 
1 f ′(x) = 13x12 + 11 x10 + 9 x 8 + 7 x 6 + 5x 4 + 3x 2 + 1
f (x) is defined − x + 1 ≥ 0 and x > 0
x f ′(x) > 0 for all value of x ∈R.
x 2 + x − 1 ≤ 0, x > 0 Hence, f (x) has exactly are real roots.
−1 ± 5 51. (c) Area of region
x= , x>0
2 {(x , y) = x 2 + y 2 ≤ 1 < x + y}
WB JEE (Engineering) Solved Paper 2020 41
∴ x 2 + y 2 = 1 and x + y = 1 54. (c) Given α = $i + a$j + a 2k$ , β = $i + b$j + b 2k$ and
Y γ = $i + c$j + c 2k$

1 a a2
(0, 1) ∴ 1 b b2 ≠ 0
B 1 c c2


O
X a a2 1 + a3
A
Now, b b 2 1 + b 3 = 0

)
(1, 0)

be
c c2 1 + c3

a a2 1 a a2 a3

Tu
Y¢ b b 1 + b b2 b3 = 0
2

c c2 1 c c2 c3
Area of shaded region = Area of quadrant − Area of

ou
triangle
1 a a2
π 2 1
= () 1 − ×1 ×1 1 b b 2 (1 + abc) = 0
4 2
1 c c2

(Y
π 1
= −
4 2
 1 a a2 
52. (b) Let f (x) = cos x + x sin x − 1  
⇒ abc + 1 = 0 Q 1 b b 2 ≠ 0
on
f ′(x) = − sin x + x cos x + sin x  1 c c 2 
 
π
f ′(x) = x cos x > 0; x ∈  0,  ⇒ abc = −1
 2
pi

π
f (x) is increasing on x ∈  0,  .
55. (c) Given, z1 and z2 be two imaginary roots of
m

 2 z 2 + pz + q = 0
∴ cos x + x sin x − 1> 0 ∴ z1 + z2 = − p, z1 z2 = q
ha

⇒ cos x + x sin x > 1


z1 , z2 and origin form an equilateral triangle.
∴ z12 + z22 = z1 z2
C

53. (c) Given parabola


⇒ (z1 + z2)2 − 2z1 z2 = z1 z2
y = ax 2 + bx + C
⇒ p 2 − 2q = q ⇒ p 2 = 3q
x = 1, y = 1, we get
dy

put
1=a+ b+ c ...(i) 56. (a) Given,
y = x is tangent of the parabola at (1, 1). P(x) = ax 2 + bx + c, Q(x) = − ax 2 + dx + c
u

∴  dy  = 2ax + b
  Discriminant for P(x) = b 2 − 4ac
St

 dx  (1 , 1)
and Discriminant for Q(x) = d 2 + 4ac
⇒ 1 = 2a + b ...(ii)
If ac> 0, Q(x) has real roots and ac < 0, P(x) has
Also curve passes through (− 1, 0) real roots.
∴ 0=a−b+ c ...(iii) ∴P(x) ⋅ Q(x) = 0 has atleast two real roots.
From Eqs. (i), (ii) and (iii), we get
1 57. (d) We have,
a=c= f (x) = x| x |, x ∈ [− 1, 1]
4
1 − x 2 , − 1 ≤ x < 0
and b= f (x) =  2
2 0≤ x <1
 x ,
42 WB JEE (Engineering) Solved Paper 2020
Graph of f (x) is Here, P is the orthocentre of ∆ABQ.
Y ∴Slope of BR × Slope of AR = − 1

⇒  k + 5 ×  k  = − 1
   
 h   h − 7
f(x)=x2
⇒ x 2 + y 2 − 7 x + 5y = 0
X¢ X
–1 O 1
62. (b) Let
f(x)=x2 n 1 / ( k + α)
dx
I = lim ∑ ∫

)
n→ ∞
k = 1 (1 / k + β) 1 + x

be

1
n
Clearly f (x) is bijective function.
∑ [log(1 + x)] 1
k+ α
I = lim
n→ ∞

Tu
k =1 k+ β
58. (d) We have,
f (x) = x 2 − 3x + 2
n   1   1 
I = lim
n→ ∞
∑  log 1 +  − log 1 +
k + α 

k + β
g(x) = k =1

ou
x
n  k + α + 1  k + β 
fog = f (g(x)) = x − 3 x + 2 I = lim
n→ ∞
∑ log    
k + α   k + β + 1
k =1
fog is defined x −3 x + 2 ≥ 0 and x ≥ 0
( x − 1) ( x − 2) ≥ 0
x ∈ (− ∞ , 1] ∪ [4, ∞)
(Y n→ ∞
 α + 2  β + 1   α +
I = lim log     
 α + 1   β + 2  α +
 α + n + 1
3  β + 2
 

 β+ n 

2  β + 3
on
gof = g ( f (x)) = x 2 − 3x + 2 ....    
 α+ n   β + n + 1
gof is defined x 2 − 3x + 2 ≥ 0   β + 1   α + n + 1 
pi

I = lim  log    
(x − 1) (x − 2) ≥ 0 n→ ∞   α + 1   β + n + 1  
x ∈ (− ∞ , 1]∪ [2,∞ )
m

 β + 1
∴Domain of fog is (−∞ , 1] ∪ [4, ∞) I = log e  
 α + 1
Domain of gof is (− ∞ , 1] ∪ [2,∞)
ha

∴ S ∩ T is an interval.  1 
63. (c) Let P = lim  1 − 
x→1  log x x − 1
59. (b) Given, ρ1 and ρ2 be two equivalent relation by
C

theorem,  x − 1 − log x 
P = lim  
x → 1  (x − 1) log x 
ρ1 ∩ ρ2 is equivalence relation but ρ1 ∪ ρ2 is not so.
dy

60. (c) Since, the length of tangent between the point Apply L-Hospital rules,
of contact and the point where it meets the  1−
1 
 
u

directrix subtends right angle at the P = lim ,  x 


x →1  x −1
corresponding focus. + log x 
St

 x 
61. (c) Y
x −1
=
(0,b) (x − 1) + x log x
Q
Again apply L-Hospital rule,
R (x,y) 1
P = lim
x→1 x
1 + + log x
X¢ X x
P (a,0) A (7, 0)
1
= lim
x → 1 2 + log x
B (0,–5) 1 1
P= =
Y¢ 2+ 0 2
WB JEE (Engineering) Solved Paper 2020 43
1
64. (b) We have, = 2∫ (1 − y 2) dy
0
1
y=  y3 
1
1 + x + log x = 2 y − 
 3 0
(1 + x + log x) y = 1
= 21 −  = sq unit
On differentiating both sides, we get 1 4
 3 3
dy 
+ 1 +  y = 0
1
(1 + x + log x)
dx  x 67. (a, d) We have, v = u − gt
− (x + 1) y

)

dy
= at t = 6, v = 0

be
dx x(1 + x + log x) = u − gt = 0
dy  1  ⇒ u = 6 g = 192ft/sec (Q g = 32 ft / sec2)
⇒ x = − (x + 1) (y 2) Q y = 1 + x + log x  1

Tu
dx   Now, h = ut − gt 2
2
dy
⇒ x = − y(x + 1) y 1
dx = 192.6 − × 32 × 6 × 6

ou
2
dy
⇒ x = y(y log x − 1) [Q(1 + x) y = 1 − y log x] = 576 ft
dx
x 68. (a, c) Given, equation

(Y

65. (b) Given, curve y = be a
x(log3 x) 29

log 3 (x + 5) = 3 3
x 2
dy b −
⇒  (log 3 x)2 − log 3 x + 5 log 3 x = log 3 3
⇒ =− e a 9 3
on
dx a  2  2
∴  dy  (0, b) = − b put log 3 x = t
 
 dx  a  t 2 − 9 t + 5 t = 3
pi

∴  
∴Equation of tangent  2  2
m

y − b= −
b
(x − 0) ⇒ 2t 3 − 9t 2 + 10t − 3 = 0
a
(t − 3) (t − 1)  t −  = 0
1

ha

⇒ y−b=−
bx  2
a 1
⇒ t = 3, 1,
y − x 2
⇒ −1 =
C

b a 1
⇒ log 3 x = 3, 1,
x y 2
⇒ + =1
dy

1
a b
⇒ x = 33, 31 , 32
66. (b) Intersecting points of both curves are P(− 2, 1)
and Q(− 2, − 1). 69. (b) Given, total number of questions are n and
u

total wrong answers are 2047


St

x =–2y2 P (–2,1)
Also, total wrong answer given
Y = Σk × (2n − k − 2n − ( k + 1))
P (–2,1)
(1,0) ⇒ 2n − 1 + K + 1 = 2047
X¢ X
O ⇒ 2n = 2048 = 211
x =1– 3y2 ⇒ n = 11
Q
Y¢ 70. (c, d) Given, P(A′ ∩ B′) = 3
(–2,–1) 5
3
⇒ 1 − P(A ∪ B) =
∴Area of the bounded region 5
1
= ∫− 1 ((1 − 3y ) − (− 2y )) dy 2
2 2
⇒ P(A ∪ B) =
5
44 WB JEE (Engineering) Solved Paper 2020
2 x2
⇒ P(A) + P(B) − P(A ∩ B) = 73. (a) Given, y =
5 (x + 1)2 (x − 2)
2
⇒ P(A) + P(B) − P(A) P(B) = ...(i) By using partial fraction,
5
x2 4 3 1
(since A and B are independent events) = − +
(x + 1)2 (x + 2) (x + 2) x + 1 (x + 1)2
1
Also, given P(A ∩ B) = 4 3 1
20 ∴ y= − +
1 x + 2 x + 1 (x + 1)2
⇒ P(A) P(B) = ...(ii)
20 On differentiating both sides w.r.t. x, we get

)
dy −4 3 2

be
From Eqs. (i) and (ii), we get = + −
1 1 dx (x + 2)2 (x + 1)2 (x + 1)3
P(A) = or
4 5 Again differentiating w.r.t. x

Tu
d2y 8 6 6
71. (a, b) Equation of straight line having intercepts = − +
on the coordinate axes are a and b, are dx 2 (x + 2)3 (x + 1)3 (x + 1)4
 3 

ou
x y 4 3
+ =1 ...(i) = 2 − + 
a b
 ( x + 2) 3
( x + 1) 3
( x + 1)4 
Given, a + b = − 1 ⇒ b = − (a + 1)
74. (c) Given, f (x) = 1 x sin x − (1 − cos x)
Line (i) passing through the point (4, 3).

4 3
+ =1
a b (Y ∴ lim
x→ 0
f (x)
xk
= lim
x→ 0
3
x sin x − 31
3x k
( − cos x)
on
4 3
⇒ − =1 x x x
a a +1 2x sin cos − 6 sin2
= lim 2 2 2
⇒ 4(a + 1) − 3a = a(a + 1) x→ 0 3x k
pi

⇒ a2 + a − a = 4  sin x   2x cos x − 6 sin x 


1    
⇒ a=± 2
m

= lim  2  ⋅ lim  2 2
→ x → k −1
When a = 2, then b = − 3 3 x 0   x 0  2x 
 2   
and when a = − 2, then b = 1
ha

x y x y  x cos x − 3 sin x 
∴ Equation of straight line are − = 1, + =1 1  
2 3 −2 1 = lim  2 2
k − 1
3x→0  
C

2 2 x
72. (d) Equation of tangent to the ellipse x + y = 1  
2 1 f (x)
For, lim ≠ 0⇒k −1 =1⇒k = 2
dy

at point (x1 , y1) is x → 0 xk


xx1 yy
+ 1 =1 ...(i)
2 1 75. (a, c) Given, AP : BP = 3 : 1
u

Let mid-point of intercept be P(h, k). Y


St

1
∴ h=
x1
B
1
⇒ x1 =
h 1
1
and k= P (x,y)
2y1 3
X¢ X
1 O A
⇒ y1 =
2k
∴Required locus of the mid-point
1 1
= 2 + =1 Y¢
2x 4 y2
WB JEE (Engineering) Solved Paper 2020 45
Equation of tangent AB is Y − y =
dy
(X − x) ⇒ 3 log y + log x = c
dx ⇒ xy 3 = c
 x dy − y  at x = 1, y = 1, we get c = 1
 
∴Coordinates of A are  dx , 0 . ∴ xy 3 = 1
 y′ 
  ⇒ x(3y 2)
dy
+ y3 = 0
dx
and coordinates of B are  0, y − x  .
dy
 dx  dy
⇒ 3x + y=0
dx
1 × 0 + 3 ×  y − x 
dy

)
dy y
  ⇒ =−

be
Now, y = dx
dx 3x
4
∴  dy  =−
1
⇒ 4 y = 3y − 3x
dy  
 dx  (1 , 1) 3

Tu
dx
dy ∴Slope of normal = 3
⇒ 3x + y=0
dx ∴Equation of normal = y − 1 = 3(x − 1)

ou

3dy dx
+ =0 ⇒ y − 3x + 2 = 0
y x

(Y
on
pi
m
ha
C
u dy
St
WB JEE (Engineering) Solved Paper 2019 1

WB JEE
Engineering Entrance Exam

Solved Paper 2019

)
be
Tu
Physics

ou
Category I (Q. Nos. 1 to 30) Assuming that a proton is 2000 times heavier

(Y
than an electron, what will be the relation
Carry 1 marks each and only one option is
between the de Broglie wavelength of the
correct. In case of incorrect answer or any
proton (λ p) and that of electron (λ e)?
combination of more than one answer, 1/4 mark
λe
on
will be deducted. (a) λ p = 2000λe (b) λ p =
2000
λ
1. A ray of light is reflected by a plane mirror. (c) λ p = 20 5 λe (d) λ p = e
pi

e$ 0 , e$ and n$ be the unit vectors along the 20 5


incident ray, reflected ray and the normal to 4. To which of the following the angular
m

the reflecting surface respectively. velocity of the electron in the n-th Bohr orbit
Which of the following gives an expression is proportional?
ha

for e$ ? 1 1 1
(a) n2 (b) (c) (d)
n2 n3 / 2 n3

C

n 5. In the circuit shown, what will be the current


∧ ∧ through the 6V zener?
e0 e
dy

1 kΩ
A

(a) e$ 0 + 2(e$ 0 ⋅ n$ )n$ (b) e$ 0 − 2(e$ 0 ⋅ n$ )n$


u

10 V 1 kΩ 6V
(c) e$ 0 − (e$ 0 ⋅ n$ )n$ (d) e$ 0 + (e$ 0 ⋅ n$ )n$
St

2. A parent nucleus X undergoes α-decay with a B


half-life of 75000 yrs. The daughter nucleus (a) 6 mA, from A to B (b) 2 mA, from A to B
Y undergoes β-decay with a half-life of 9 (c) 2 mA, from B to A (d) Zero
months. In a particular sample, it is found
that the rate of emission of β-particles is 6. Each of the two inputs A and B can assume
nearly constant (over several months) at values either 0 or 1. Then which of the
10 7 /h. What will be the number of α-particles following will be equal to A ⋅ B?
emitted in an hour? (a) A + B (b) A + B
(a) 10 2
(b) 10 7
(c) 10 12
(d) 10 14 (c) A ⋅ B (d) A + B

3. A proton and an electron initially at rest are 7. The correct dimensional formula for impulse
accelerated by the same potential difference. is given by
2 WB JEE (Engineering) Solved Paper 2019
(a) ML2 T− 2 (b) MLT− 1 (c) ML2 T− 1 (d) MLT− 2 13. A compressive force is applied to a uniform
8. The density of the material of a cube can be rod of rectangular cross-section so that its
estimated by measuring its mass and the length decreases by 1%. If the Poisson’s ratio
length of one of its sides. If the maximum for the material of the rod be 0.2, which of
error in the measurement of mass and length the following statements is correct?
are 0.3% and 0.2% respectively, the “The volume approximately ...........”
maximum error in the estimation of the (a) decreases by 1% (b) decreases by 0.8%
density of the cube is approximately. (c) decreases by 0.6% (d) increases by 0.2%

)
(a) 1.1% (b) 0.5% (c) 0.9% (d) 0.7% 14. A small spherical body of radius r and density

be
ρ moves with the terminal velocity v in a fluid
9. Two weights of the mass m1 and m2(> m1) are
of coefficient of viscosity η and density σ.
joined by an inextensible string of negligible

Tu
What will be the net force on the body?
mass passing over a fixed frictionless pulley. 4π 3
The magnitude of the acceleration of the (a) r (ρ − σ ) g (b) 6πηrv
3
loads is

ou
(c) Zero (d) Infinity
m2 − m1
(a) g (b) g
m2 15. Two black bodies A and B have equal surface
m − m1

(Y
(c)
m1
g (d) 2 g
areas and are maintained at temperatures
m2 + m1 m2 + m1 27ºC and 177ºC respectively. What will be the
ratio of the thermal energy radiated per
10. A body starts from rest, under the action of second by A to that by B?
on
an engine working at a constant power and (a) 4 : 9 (b) 2 : 3 (c) 16 : 81 (d) 27 : 177
moves along a straight line. The displacement
s is given as a function of time ()
t as 16. What will be the molar specific heat at
pi

(a) s = at + bt 2 , a and b are constants constant volume of an ideal gas consisting of


rigid diatomic molecules?
m

(b) s = bt , b is a constant
2
3 5
(c) s = at 3 / 2 , a is a constant (a) R (b) R
2 2
ha

(d) s = at , a is a constant (c) R (d) 3R


11. Two particles are simultaneously projected in 17. Consider the given diagram. An ideal gas is
C

the horizontal direction from a point P at a contained in a chamber (left) of volume V


certain height. The initial velocities of the and is at an absolute temperature T. It is
particles are oppositely directed to each other allowed to rush freely into the right chamber
dy

and have magnitude v each. The separation of volume V which is initially vacuum. The
between the particles at a time when their whole system is thermally isolated. What will
position vectors (drawn from the point P) are
u

be the final temperature of the equilibrium


mutually perpendicular, is has been attained?
St

v2 v2
(a) (b)
2g g
4v 2 2v2
(c) (d) V V
g g Ideal gas Vacuum

12. Assume that the earth moves around the sun


in a circular orbit of radius R and there exists
a planet which also move around the sun in a T
circular orbit with an angular speed twice as (a) T (b)
2
large as that of the earth. The radius of the T
orbit of the planet is (c) 2T (d)
4
R
(a) 2 − 2 / 3 R (b) 2 2 / 3 R (c) 2 − 1/ 3 R (d)
2
WB JEE (Engineering) Solved Paper 2019 3

18. Five identical capacitors, of capacitance 20µF 22. An electric current ‘I’ enters and leaves a
each, are connected to a battery of 150 V, in a uniform circular wire of radius r through
combination as shown in the diagram. What diametrically opposite points. A particle
is the total amount of charge stored? carrying a charge q moves along the axis of
the circular wire with speed v. What is the
magnetic force experienced by the particle
when it passes through the centre of the circle?
µ 0i µ 0i
(a) qv (b) qv

)
a 2a
µ i

be
(c) qv 0 (d) Zero
2 πa

(a) 15 × 10− 3 C 23. A current ‘I’ is flowing along an infinite,

Tu
(b) 12 × 10− 3 C straight wire, in the positive Z-direction and
the same current is flowing along a similar
(c) 10 × 10− 3 C

ou
parallel wire 5 m apart, in the negative
(d) 3 × 10− 3 C Z-direction. A point P is at a perpendicular
distance 3m from the first wire and 4m from
19. Eleven equal point charges, all of them

(Y
the second. What will be magnitude of the
having a charge + Q, are placed at all the
magnetic field B of P?
hour positions of a circular clock of radius r,
5 7
except at the 10 h position. What is the (a) (µ 0 I) (b) (µ 0 I)
on
electric field strength at the centre of the 12 24
5 25
clock? (c) (µ 0 I) (d) (µ 0 I)
Q 24 288
(a) from the centre towards the mark 10
pi

4 πε0 r 2 24. A square conducting loop is


Q placed near an infinitely long
m

(b) from the mark 10 towards the centre


4 πε0 r 2 current carrying wire with one
Q edge parallel to the wire as shown
ha

(c) from the centre towards the mark 6


4 πε0 r 2 in the figure. If the current in the
(d) Zero. straight wire is suddenly halved,
which of the following statements I
C

20. A negative charge is placed at the midpoint will be true?


between two fixed equal positive charges, “The loop will .............”.
dy

separated by a distance 2d. If the negative (a) stay stationary


charge is given a small displacement x (x < < d) (b) move towards the wire
perpendicular to the line joining the positive (c) move away from the wire
u

charges, how the force (F) developed on it will (d) move parallel to the wire
St

approximately depend on x?
1 25. What is the current I shown in the given
(a) F ∝ x (b) F ∝ circuit?
x
1 R R R
(c) F ∝ x 2
(d) F ∝
x2

21. To which of the following quantities, the 2R V 2R 2R 2R 2R I


radius of the circular path of a charged
particle moving at right angles to a uniform
magnetic field is directly proportional?
(a) energy of the particle V V
(a) (b)
(b) magnetic field 2R R
(c) charge of the particle V V
(c) (d)
(d) momentum of the particle 16R 8R
4 WB JEE (Engineering) Solved Paper 2019

26. When the value of R in the balanced 30. When the frequency of the light used is
Wheatstone bridge, shown in the figure, is changed from 4 × 10 14 s− 1 to 5 × 10 14 s− 1 , the
increased from 5Ω to 7 Ω, the value of S has to angular width of the principal (central)
be increased by 3Ω in order to maintain the maximum in a single slit Fraunhoffer
balance. What is the initial values of S? diffraction pattern changes by 0.6 radian.
What is the width of the slit (assume that
P Q the experiment is performed in vacuum)?
. × 10− 7 m
(a) 15 (b) 3 × 10− 7 m

)
G −7
(c) 5 × 10 m (d) 6 × 10− 7 m

be
R S
Category II (Q. Nos. 31 to 35)

Tu
Carry 2 marks each and only one option is
correct. In case of incorrect answer or any
(a) 2.5 Ω (b) 3 Ω combination of more than one answer, 1/2 mark
(c) 5 Ω (d) 7.5 Ω will be deducted.

ou
27. When a 60 mH inductor and a resistor are 31. A capacitor of capacitance C is connected in
connected in series with an AC voltage source, series with a resistance R and DC source of
the voltage leads the current by 60º. If the
inductor is replaced by a 0 .5 µF capacitor, the
voltage lags behind the current by 30º.What is (Y emf E through a key. The capacitor starts
charging when the key is closed. By the
time the capacitor has been fully charged,
on
the frequency of the AC supply? what amount of energy is dissipated in the
1 1
(a) × 104 Hz (b) × 104 Hz resistance R?
2π π
pi

3 1
(c) × 104 Hz (d) × 108 Hz C
2π 2π
m

R
28. A point object is placed on the axis of a thin E
ha

convex lens of focal length 0.05 m at a distance


of 0.2 m from the lens and its image is formed 1 E2
(a) CE 2 (b) 0 (c) CE 2 (d)
on the axis. If the object is now made to 2 R
C

oscillate along the axis with a small amplitude


of A cm, then what is the amplitude of 32. A horizontal fire hose with a nozzle of
5
oscillation of the image? cross-sectional area × 10 − 3 m 2 delivers
dy

 1  21
you may assume, 1 + x ≈ 1 − x , where x < < 1 a cubic metre of water in 10s. What will be
  the maximum possible increase in the
u

4A 5A temperature of water while it hits a rigid


(a) × 10− 2 m (b) × 10− 2 m
St

9 9 wall (neglecting the effect of gravity)?


A A
(c) × 10− 2 m (d) × 10− 2 m (a) 1ºC (b) 0.1ºC (c) 10ºC (d) 0.01ºC
3 9
33. Two identical blocks of ice move in opposite
29. In Young’s experiment for the interference of directions with equal speed and collide with
light, the separation between the slits is d and each other. What will be the minimum
the distance of the screen from the slits is D. If speed required to make both the blocks
D is increased by 0.5% and d is decreased by melt completely, if the initial temperatures
0.3% then for the light of a given wavelength, of the blocks were − 8 ºC each?
which one of the following is true? (Specific heat of ice is 2100 Jkg − 1 K − 1 and
“The fringe width ............. latent heat of fusion of ice is 3.36 × 105 Jkg − 1 )
(a) increases by 0.8% (b) decreases by 0.8% (a) 840 ms − 1 (b) 420 ms − 1
−1
(c) increases by 0.2% (d) decreases by 0.2% (c) 8.4 ms (d) 84 ms − 1
WB JEE (Engineering) Solved Paper 2019 5

34. A particle with charge q moves with a velocity 37. The initial pressure and volume of a given
v in a direction perpendicular to the  Cp 
directions of uniform electric and magnetic mass of an ideal gas  with = γ  , taken in
fields, E and B respectively, which are  CV 
mutually perpendicular to each other. Which a cylinder fitted with a piston, are p 0 and V0
one of the following gives the condition for respectively. At this stage the gas has the
which the particle moves undeflected in its same temperature as that of the surrounding
original trajectory? medium which is T0 . It is adiabatically
V

)
y compressed to a volume equal to 0 .

be
2
E
Subsequently the gas is allowed to come to
thermal equilibrium with the surroundings.
v

Tu
x What is the heat released to the
B surrounding?
p0 V0
(a) 0 (b) (2 γ − 1 − 1)

ou
z γ −1
E B E B p0 V0
(a) v = (b) v = (c) v = (d) v = q (c) γp0 V0 ln2 (d)
B E B E 2(γ − 1)

35. A parallel plate capacitor in series with a


resistance of 100 Ω, an inductor of 20 mH (Y
38. A projectile thrown with an initial velocity of
10 ms − 1 at an angle α with the horizontal,
has a range of 5 m. Taking g = 10 ms − 2 and
on
and an AC voltage source of variable
frequency shows resonance at a frequency of neglecting air resistance, what will be the
1250 estimated value of α?
Hz.
pi

π (a) 15º (b) 30º (c) 45º (d) 75º


If this capacitor is charged by a DC voltage 39. In the circuit shown in the figure all the
m

source to a voltage 25 V, what amount of resistance are identical and each has the
charge will be stored in each plate of the value r Ω. The equivalent resistance of the
ha

capacitor? combination between the points A and B will


(a) 0.2 µC (b) 2 mC (c) 0.2 mC (d) 0.2 C remain unchanged even when the following
pairs of points marked in the figure are
C

Category III (Q. Nos. 36 to 40) connected through a resistance R.


Carry 2 marks each and one or more option(s) 1 2 3 4 5
dy

is/are correct. If all correct answers are not marked r r r r r r


and also no incorrect answer is marked then score A B
= 2 × number of correct answers marked ÷ actual r r r
u

6 7
number of correct answers. If any wrong option is
St

marked or if any combination including a wrong (a) 2 and 6 (b) 3 and 6


option is marked, the answer will be considered (c) 4 and 7 (d) 4 and 6
wrong, but there is no negative marking for the 40. A metallic loop is placed in a uniform
same and zero marks will be awarded. magnetic field B with the plane of the loop
perpendicular to B. Under which condition(s)
36. Electrons are emitted with kinetic energy T given an emf will be induced in the loop?
from a metal plate by an irradiation of light
of intensity J and frequency ν. Then, which “If the loop is ............”
of the following will be true? (a) moved along the direction of B
(b) squeezed to a smaller area
(a) T ∝ J
(c) rotated about its axis
(b) T linearly increasing with ν
(d) rotated about one of its diameters
(c) T ∝ time of irradiation
(d) Number of electrons emitted ∝ J
Chemistry
Category I (Q. Nos. 41 to 70) 44. The compound, which evolves carbon dioxide
Carry 1 mark each and only one option is correct. on treatment with aqueous solution of
In case of incorrect answer or any combination of sodium bicarbonate at 25ºC, is
more than one answer, 1/4 mark will be (a) C 6H5OH (b) CH3COCI
deducted. (c) CH3CONH2 (d) CH3COOC 2H5

)
be
45. The indicated atom is not a nucleophilic site
41. One of the products of the following reaction
in
is P.
(a) BH−4 (b) CH3MgI

Tu
O ↑ ↑
(c) CH3 OH (d) CH3 NH2
(i) aq. KOH ↑ ↑
CCl3
(ii) H3O+
P 46. The charge carried by 1 millimole of M n +

ou
ions is 193 coulombs. The value of n is
Structure of P is (a) 1 (b) 2 (c) 3 (d) 4

(Y
O O 47. Which of the following mixtures will have
CO2H H
the lowest pH at 298 K?
(a) (b) (a) 10 mL 0.05 N CH3COOH + 5 mL 0.1 N NH4OH
on
(b) 5 mL 0.2 N NH4Cl + 5 mL 0.2 N NH4OH
O HO (c) 5 mL 0.1N CH 3COOH +10 mL 0.05 N CH 3COONa
O
Cl (d) 5 mL 0.1 N CH3COOH + 5 mL 0.1 N NaOH
pi

OH
(c) (d)
Cl 48. Consider the following two first order
m

reactions occurring at 298 K with same initial


42. For the reaction below, the product is Q. concentration of A :
ha

(1) A → B; rate constant, k = 0 .693 min − 1


CO2H
Acetic anhydride
Q [C9H8O4] (2) A → C ; half-life, t1 / 2 = 0 .693 min
C

Conc.H2SO4(cat.)heat Choose the correct option.


HO
(a) Reaction (1) is faster than reaction (2).
The compound Q is (b) Reaction (1) is slower than reaction (2).
dy

CO2H COOCOCH3 (c) Both reactions proceed at the same rate.


(d) Since two different products are formed, rates
u

(a) (b) cannot be compared.


St

OCOCH3 OH
49. For the equilibrium, H 2O( l ) - H 2O(v),
which of the following is correct?
CO2H CO2H (a) ∆G = 0, ∆H < 0, ∆S < 0
COCH3 (b) ∆G < 0, ∆H > 0, ∆S > 0
(c) (d) (c) ∆G > 0, ∆H = 0, ∆S > 0
COCH3 (d) ∆G = 0, ∆H > 0, ∆S > 0
 ab 
OH OH 50. For a van der Waals’ gas, the term  
represents some  V2
43. Cyclopentanol on reaction with NaH followed
(a) pressure (b) energy
by CS2 and CH 3I produces a/an
(c) critical density (d) molar mass
(a) ketone (b) alkene
(c) ether (d) xanthate 51. In the equilibrium, H 2 + I 2 -
2HI, if at a
given temperature the concentrations of the
WB JEE (Engineering) Solved Paper 2019 7

reactants are increased, the value of the 60. The melting points of (i) BeCl 2 (ii) CaCl 2 and
equilibrium constant, K C , will (iii) HgCl 2 follows the order
(a) increase (a) i < ii < iii (b) iii < i < ii
(b) decrease (c) i < iii < ii (d) ii < i < iii
(c) remain the same
61. Which of these species will have non-zero
(d) cannot be predicted with certainty
magnetic moment?
52. If electrolysis of aqueous CuSO 4 solution is (a) Na + (b) Mg (c) F − (d) Ar +
carried out using Cu-electrodes, the reaction

)
taking place at the anode is 62. The first electron affinity of C, N and O will

be
− be of the order
(a) H + e → H
+

(b) Cu2 + (aq ) + 2e − → Cu (s ) (a) C < N < O (b) N < C < O


(c) C < O < N (d) O < N < C
(c) SO 24 − (aq ) − 2e − → SO 4

Tu
(d) Cu(s ) − 2e − → Cu2 + (aq ) 63. The H N H angle in ammonia is 107.6º,
while the H P H angle in phosphine is
53. Which one of the following electronic 93.5º. Relative to phosphine, the p-character

ou
arrangements is absurd? of the lone-pair on ammonia is expected
(a) n = 3, l = 1, m = − 1 (b) n = 3, l = 0, m = 0 to be

(Y
(c) n = 2, l = 0, m = − 1 (d) n = 2, l = 1, m = 0 (a) Less (b) More
(c) Same (d) Cannot be predicted
54. The quantity hv / K B corresponds to
64. The reactive species in chlorine bleach is
on
(a) wavelength (b) velocity
(a) Cl 2O (b) OCl −
(c) temperature (d) angular momentum
(c) ClO 2 (d) HCl
55. In the crystalline solid MSO 4 ⋅ nH 2O of molar 65. The conductivity measurement of a
pi

mass 250 g mol − 1 , the percentage of coordination compound of cobalt (III) shows
anhydrous salt is 64 by weight. that it dissociates into 3 ions in solution. The
m

The value of n is compound is


(a) 2 (b) 3 (c) 5 (d) 7
(a) hexaamminecobalt(III) chloride
ha

56. At S.T.P. the volume of 7.5 g of a gas is 5.6 L. (b) pentaamminesulphatocobalt(III) chloride
The gas is (c) pentaamminechloridocobalt(III) sulphate
(d) pentaamminechloridocobalt(III) chloride
C

(a) NO (b) N2O


(c) CO (d) CO 2 66. In the Bayer’s process, the leaching of
alumina is done by using
dy

125
57. The half-life period of 53 I
is 60 days. The (a) Na 2CO 3 (b) NaOH (c) SiO 2 (d) CaO
radioactivity after 180 days will be
67. Which atomic species cannot be used as a
u

(a) 25% (b) 12.5% nuclear fuel?


(c) 33.3% (d) 3.0%
St

233 235 239 238


(a) 92 U (b) 92 U (c) 94 Pu (d) 92 U
58. Consider, the radioactive disintegration
68. The molecule/molecules that has/have
82 A
210
→ B→ C → 82 D
206
delocalised lone pair(s) of electrons is/are
The sequence of emission can be O O
(a) β, β, β (b) α, α ,β N
(a) OCH3 (b) C
(c) β, β, γ (d) β , β , α H 3C CH2 H 3C CH2
59. The second ionisation energy of the following OCH3
elements follows the order (c) (d) CH3CH==CHCH2NHCH3
(a) Zn > Cd < Hg (b) Zn > Cd > Hg
(c) Cd > Hg < Zn (d) Zn < Cd < Hg
(a) I, II and III (b) I, II and IV
(c) I and III (d) only III
8 WB JEE (Engineering) Solved Paper 2019

69. The conformations of n-butane, commonly 73. At constant pressure, the heat of formation
known as eclipsed, gauche and of a compound is not dependent on
anti-conformations can be interconverted by temperature, when
(a) rotation around C  H bond of a methyl group (a) ∆C p = 0 (b) ∆C v = 0
(b) rotation around C  H bond of a methylene (c) ∆C p > 0 (d) ∆C p < 0
group
(c) rotation around C1-C2 linkage 74. A copper coin was electroplated with Zn and
(d) rotation around C2-C3 linkage then heated at high temperature until there
is a change in colour. What will be the

)
70. The correct order of the addition reaction

be
resulting colour?
rates of halogen acids with ethylene is (a) white (b) black
(a) hydrogen chloride > hydrogen bromide (c) silver (d) golden

Tu
> hydrogen iodide
(b) hydrogen iodide > hydrogen bromide
75. Oxidation of allyl alcohol with a peracid gives
> hydrogen chloride
a compound of molecular formula C3H 6O 2 ,
which contains an asymmetric carbon atom.

ou
(c) hydrogen bromide > hydrogen chloride
The structure of the compound is
> hydrogen iodide
(d) hydrogen iodide > hydrogen chloride OH

(Y
> hydrogen bromide O
(a) O OH (b) CH3
H
Category II (Q. Nos. 71 to 75)
on
H H
Carry 2 marks each and only one option is O
correct. In case of incorrect answer or any OH
(c) H3C (d) H3C
combination of more than one answer, 1/2 mark
pi

O OH
will be deducted.
m

71. The total number of isomeric linear Category III (Q.Nos. 76 to 80)
dipeptides which can be synthesised from
ha

Carry 2 marks each and one or more option(s)


racemic alanine is is/are correct. If all correct answers are not
(a) 1 (b) 2 (c) 3 (d) 4 marked and also no incorrect answer is marked
C

72. The kinetic study of a reaction like vA → P at then score = 2 × number of correct answers
marked ÷ actual number of correct answers. If
300 K provides the following curve, where
any wrong option is marked or if any combination
dy

−3
concentration is taken in mol dm and time
including a wrong option is marked, the answer
in min.
will considered wrong, but there is no negative
r0 : Initial rate
u

[A0] : Initial
marking for the same and zero mark will be
awarded.
St

concentration of A
√r0
Slope = 4.0 76. Haloform reaction with I2 and KOH will be
responded by
O
O
[A]0 Me
(a) I Ph (b) Ph
Identify the correct order(n) and rate constant(k) OH
(a) n = 0, k = 4.0 mol dm− 3 min− 1 OH O
(b) n = 1 / 2, k = 2.0 mol1/ 2 dm− 3 / 2 min− 1
(c) n = 1, k = 8.0 min− 1 (c) Me
Me
Ph (d) Ph N Me
H
(d) n = 2, k = 16.0 dm3 mol − 1 min− 1
WB JEE (Engineering) Solved Paper 2019 9

77. Identify the correct statement(s): (a) NH4NO 3 (b) NaCl


(a) The oxidation number of Cr in CrO 5 is + 6. (c) (NH4 )2 CO 3 (d) HCOONa
(b) ∆H > ∆U for the reaction N2O 4 (g ) → 2NO 2 (g ), 80. The compound(s), capable of producing
provided both gases behave ideally. achiral compound on heating at 100ºC
(c) pH of 01 . N H2SO 4 is less than that of 0.1 N HCl is/are
at 25ºC. Me Me
(d) 
RT  Et CH2COOH Et CH2CO2H
 = 0.0591 volt at 25ºC.
 F  (a) (b)

)
78. Compounds with spin-only magnetic moment HO2C CO2H

be
HO2C CO2H
equivalent to five unpaired electrons are Me

(a) K 4 [Mn(CN)6 ] (b) [Fe(H2O)6 ] Cl 3 Me H


Et CO2H Et CO2H

Tu
(c) K 3 [FeF6 ] (d) K 4 [MnF6 ] (d)
(c)
79. Which of the following chemicals may be used O Me O Me

ou
to identify three unlabelled beakers containing
conc.NaOH, conc.H 2SO 4 and water?

Mathematics (Y
on
Category I (Q. Nos. 1 to 50) 4. The value of the integration
pi

π /4
Carry 1 mark each and only one option is  µ sin x 
correct. In case of incorrect answer or any ∫  λ |sin x | +
 +
+ γ  dx

m

− π /4
1 cos x
combination of more than one answer, 1/4 mark
will be deducted. (a) is independent of λ only
ha

(b) is independent of µ only


1. lim (x n ln x), n > 0 (c) is independent of γ only
x → 0+ (d) depends on λ, µ and γ
C

1  sin 2 t 
(a) does not exist (b) exists and is zero a a
sin 2 t
(c) exists and is 1 (d) exists and is e − 1 5. The value of lim ∫ e dt − ∫e dt is
x→0x
 y 
dy

x + y
 x
2. If ∫ cos x log  tan  dx equal to
 2 2 2
u

y
 x (a) esin (b) e 2sin y (c) e| sin y | (d) ecosec y

= sin x log  tan  + f (x), then f (x) is equal to


 2
St

x x
6. If ∫ 22 ⋅ 2x dx = A ⋅ 22 + C , then A is equal to
(assuming c is a arbitrary real constant)
1 1
(a) c (b) c − x (c) c + x (d) 2 x + c (a) (b) log2 (c) (log 2 )2 (d)
log 2 (log 2 )2
 1− x
3. y = ∫ cos 2 tan − 1  dx is an equation of 7. The value of the integral
 1+ x
1
 x 2015 1 
a family of ∫  e| x |(x 2 + cos x) + e| x |  dx is equal to
(a) straight lines (b) circles − 1 
(c) ellipses (d) parabolas (a) 0 (b) 1 − e −1
(c) 2e − 1 (d) 2(1 − e − 1 )
10 WB JEE (Engineering) Solved Paper 2019

3  n n n 1  1 
8. lim 1 + + + 14. If log 62 + = log 2 2 x + 8  , then the values
n→ ∞ n
 n+3 n+6 n+9 2x  
 
n  of x are
+ ... + 
n + 3(n − 1)  1 1
(a) ,
1 1
(b) , (c) −
1 1
,
1
(d) , −
1
4 3 4 2 4 2 3 2
(a) does not exist (b) is 1
(c) is 2 (d) is 3 15. Let z be a complex number such that the
principal value of argument, arg z > 0.

)
9. The general solution of the differential
Then, arg z − arg(− z) is

be
 x
 x π
equation 1 + e y  dx + 1 −  e x / y dy = 0 is (a) (b) ± π (c) π (d) − π
   y 2
 

Tu
(C is an arbitrary constant) 16. The general value of the real angle θ, which
x x satisfies the equation,
(a) x − ye = C y
(b) y − xe = C y (cos θ + i sin θ) (cos 2θ + i sin 2θ) ...

ou
x x (cos nθ + i sin nθ) = 1 is given by, (assuming k
(c) x + ye y = C (d) y + xe y = C is an integer)
2 kπ 4kπ
10. General solution of (x + y)2
(C is an arbitrary constant)
dy
dx
= a 2 , a ≠ 0 is
(Y (a)

(c)
n+2
4kπ
n+1
(b)

(d)
n(n + 1)
6kπ
n(n + 1)
on
x y
(a) = tan + C (b) tan xy = C
a a 17. Let a , b, c be real numbers such that
y+C x+ y a + b + c < 0 and the quadratic equation
pi

(c) tan( x + y) = C (d) tan =


a a ax 2 + bx + c = 0 has imaginary roots. Then,
m

11. Let P(4 , 3) be a point on the hyperbola (a) a > 0, c > 0 (b) a > 0, c < 0
x 2
y 2 (c) a < 0, c > 0 (d) a < 0, c < 0
−= 1. If the normal at P intersects the
ha

2
a b2 18. A candidate is required to answer 6 out of
X -axis at (16, 0), then the eccentricity of the 12 questions which are divided into two parts
C

hyperbola is A and B, each containing 6 questions and


5 he/she is not permitted to attempt more than
(a) (b) 2 (c) 2 (d) 3 4 questions from any part. In how many
2
dy

different ways can he/she make up his/her


12. If the radius of a spherical balloon increases choice of 6 questions?
by 0.1%, then its volume increases (a) 850 (b) 800 (c) 750 (d) 700
u

approximately by
St

(a) 0.2% (b) 0.3% (c) 0.4% (d) 0.05%


19. There are 7 greeting cards, each of a different
colour and 7 envelopes of same
13. The three sides of a right angled triangle are 7 colours as that of the cards. The number
in GP (geometric progression). If the two of ways in which the cards can be put in
acute angles be α and β, then tanα and tanβ envelopes, so that exactly 4 of the cards go
are into envelopes of respective colour is,
5+1 5 −1 5+1 5 −1 (a) 7C 3 (b) 2.7 C 3 (c) 3!4 C 4 (d) 3!7 C 3 4C 3
(a) and (b) and
2 2 2 2
1 5 2 20. 7 2n + 16 n − 1 (n ∈ N) is divisible by
(c) 5 and (d) and
5 2 5 (a) 65 (b) 63 (c) 61 (d) 64
WB JEE (Engineering) Solved Paper 2019 11

21. The number of irrational terms in the 28. Let the relation ρ be defined on R as aρb if
84
 1 1 1 + ab > 0 . Then,
expansion of 3 8 + 5 4  is (a) ρ is reflexive only.
 
  (b) ρ is equivalence relation.
(a) 73 (b) 74 (c) 75 (d) 76 (c) ρ is reflexive and transitive but not symmetric
(d) ρ is reflexive and symmetric but not transitive.
22. Let A be a square matrix of order 3 whose all
entries are 1 and let I 3 be the identity matrix 29. A problem in mathematics is given to
4 students whose chances of solving

)
of order 3. Then, the matrix A − 3 I 3 is

be
1 1 1 1
(a) invertible (b) orthogonal individually are , , and . The probability
(c) non-invertible 2 3 4 5
(d) real Skew Symmetric matrix that the problem will be solved at least by

Tu
one student is
23. If M is any square matrix of order 3 over ú 2 3
(a) (b)
and if M′ be the transpose of M, then 3 5

ou
adj(M ′) − (adj M)′ is equal to (c)
4
(d)
3
(a) M (b) M′ 5 4
(c) null matrix (d) identity matrix
30. If X is a random variable such that σ(X ) = 2.6,
5 5 x x 

0 0

24. If A = 0 x 5 x  and| A2 | = 25, then| x | is
5 
(Y then σ(1 − 4 X ) is equal to
(a) 7.8 (b) − 10.4 (c) 13 (d) 10.4
on
 − sin x
31. If e −e
sin x
− 4 = 0, then the number of
equal to real values of x is
1
pi

(a) (b) 5 (c) 52 (d) 1 (a) 0 (b) 1


5
(c) 2 (d) 3
m

25. Let A and B be two square matrices of order 3


32. The angles of a triangle are in the ratio
and AB = O3 , where O3 denotes the null
2 : 3 : 7 and the radius of the circumscribed
ha

matrix of order 3. Then,


circle is 10 cm. The length of the smallest
(a) must be A = O3 , B = O3 side is
(b) if A ≠ O3 , must be B ≠ O3
C

(a) 2 cm (b) 5 cm
(c) if A = O3 , must be B ≠ O3
(c) 7 cm (d) 10 cm
(d) may be A ≠ O3 , B ≠ O3
dy

26. Let P and T be the subsets of k , y-plane 33. A variable line passes through a fixed point
(x1 , y1) and meets the axes at A and B. If the
defined by rectangle OAPB be completed, the locus of P
u

P = {(x , y) : x > 0, y > 0 and x 2 + y 2 = 1} is, (O being the origin of the system of axes).
x1 y
St

T = {(x , y) : x > 0, y > 0 and x 8 + y 8 < 1} (a) ( y − y1 )2 = 4( x − x1 ) (b) + 1 =1


x y
Then, P ∩ T is x2 y2
(a) the void set φ (b) P (c) x2 + y2 = x12 + y12 (d) 2 + 2 = 1
(c) T (d) P − T C 2 x1 y1

34. A straight line through the point (3, − 2) is


x2
27. Let f : R → R be defined by f (x) = x 2 − inclined at an angle 60º to the line 3 x + y = 1.
1 + x2 If it intersects the X -axis, then its equation
for all x ∈ R. Then, will be
(a) f is one-one but not onto mapping (a) y + x 3+2+ 3 3=0
(b) f is onto but not one-one mapping (b) y − x 3+2+ 3 3=0
(c) f is both one-one and onto (c) y − x 3 −2 −2 3=0
(d) f is neither one-one nor onto (d) x − x 3+2−3 3=0
12 WB JEE (Engineering) Solved Paper 2019

35. A variable line passes through the fixed point 42. P is the extremity of the latusrectum of
(α , β). The locus of the foot of the ellipse 3 x 2 + 4 y 2 = 48 in the first quadrant.
perpendicular from the origin on the line is The eccentric angle of P is
π 3π π 2π
(a) x2 + y2 − αx − βy = 0 (a) (b) (c) (d)
8 4 3 3
(b) x2 − y2 + 2αx + 2βy = 0
(c) αx + βy ± (α 2 + β 2 ) = 0 43. The direction ratios of the normal to the
x 2
y 2 plane passing through the points (1, 2 , − 3),
(d) + =1 x − 2 y +1 z
(− 1, − 2 , 1) and parallel to = = is

)
α 2
β 2
2 3 4

be
36. If the point of intersection of the lines (a) (2, 3, 4) (b) (14, − 8, − 1)
2 ax + 4 ay + c = 0 and 7 bx + 3 by − d = 0 lies in (c) (− 2, 0, − 3) (d) (1, − 2, − 3)

Tu
the 4 th quadrant and is equidistant from the 44. The equation of the plane, which bisects the
two axes, where a , b, c and d are non-zero line joining the points (1, 2, 3) and (3, 4, 5)
numbers, then ad : bc equals to at right angles is

ou
(a) 2 : 3 (b) 2 : 1 (c) 1 : 1 (d) 3 : 2 (a) x + y + z = 0 (b) x + y − z = 9
(c) x + y + z = 9 (d) x + y − z + 9 = 0
37. A variable circle passes through the fixed

(Y
point A(p , q) and touches X -axis. The locus of 45. The limit of the interior angle of a regular
the other end of the diameter through A is polygon of n sides as n → ∞ is
(a) ( x − p)2 = 4qy (b) ( x − q )2 = 4 py π 3π 2π
(a) π (b) (c) (d)
3 2 3
on
(c) ( y − p) = 4qx2
(d) ( y − q ) = 4 px
2

46. Let f (x) > 0 for all x and f ′(x) exists for all x.
1 3
38. If P(0 , 0), Q(1, 0) and R  ,  are three given If f is the inverse function of h and
pi

2 2  1
h′ (x) = . Then, f ′(x) will be
points, then the centre of the circle for which 1 + log x
m

the lines PQ, QR and RP are the tangents is


(a) 1 + log(f( x)) (b) 1 + f( x)
 1 3  1 1  (d)  1 , − 1
(a)  ,
1 1 (c) 1 − log(f( x))
ha

 (b)  ,  (c)  ,    (d) log f( x)


2 4  2 4  2 2 3 2 3
47. Consider the function f (x) = cos x 2. Then,
x 2
y 2 (a) f is of period 2 π (b) f is of period 2 π
C

39. For the hyperbola − = 1, which (c) f is not periodic (d) f is of period π
cos α sin α 2 2

of the following remains fixed when α varies? 48. lim (e x + x)1/ x


dy

x → 0+
(a) directrix (b) vertices
(c) foci (d) eccentricity (a) Does not exist finitely (b) is 1
(c) is e 2 (d) is 2
u

40. S and T are the foci of an ellipse and B is the


end point of the minor axis. If STB is 49. Let f (x) be a derivable function, f ′ (x) > f (x)
St

equilateral triangle, the eccentricity of the and f (0) = 0 . Then,


ellipse is (a) f( x) > 0 for all x > 0
1 1 1 2 (b) f( x) < 0 for all x > 0
(a) (b) (c) (d) (c) no sign of f( x) can be ascertained
4 3 2 3
(d) f( x) is a constant function
41. The equation of the directrices of the
hyperbola 3 x 2 − 3 y 2 − 18 x + 12 y + 2 = 0 is
50. Let f :[1, 3] → R be a continuous function
13 6 that is differentiable in (1, 3) an
(a) x = 3 ± (b) x = 3 ± f ′ (x) = | f (x)|2 + 4 for all x ∈(1, 3). Then,
6 13
13 3 (a) f(3) − f(1) = 5 is true
(c) x = 6 ± (d) x = 6 ±
3 13 (b) f(3) − f(1) = 5 is false
(c) f(3) − f(1) = 7 is false
(d) f(3) − f(1) < 0 only at one point of (1, 3)
WB JEE (Engineering) Solved Paper 2019 13

Category II (Q.Nos. 51 to 65) 56. For any non-zero complex number z, the
Carry 2 marks each and only one option is minimum value of| z | + | z − 1| is
correct. In case of incorrect answer or any 1 3
(a) 1 (b) (c) 0 (d)
combination of more than one answer, 1/2 mark 2 2
will be deducted.
57. The system of equations
51. Let a = min{ x 2 + 2 x + 3 : x ∈ R} and λ x + y + 3z = 0
1 − cos θ n
2 x + µy − z = 0
b = lim . Then ∑ a r bn − r is

)
θ→ 0 θ2 r=0
5x + 7 y + z = 0

be
has infinitely many solutions in R. Then,
2n + 1 − 1 2n + 1 + 1
(a) (b) (a) λ = 2, µ = 3 (b) λ = 1, µ = 2
3⋅2n 3⋅2n
(c) λ = 1, µ = 3 (d) λ = 3, µ = 1

Tu
n+1
4 −1 1
(c) (d) (2 n − 1)
3⋅2n 2 58. Let f : X → Y and A , B are non-void subsets
of Y , then (where the symbols have their

ou
52. Let a > b > 0 and I (n) = a1/ n − b1/ n,
usual interpretation)
J (n) = (a − b) 1/ n
for all n ≥ 2, Then
(a) f − 1( A) − f − 1(B) ⊃ f − 1( A − B) but the opposite
(a) I(n) < J(n) (b) I(n) > J(n)

(Y
does not hold.
(c) I(n) = J(n) (d) I(n) + J(n) = 0
(b) f − 1( A) − f − 1(B) ⊂ f − 1( A − B) but the opposite
$ γ$ be three unit vectors such that
53. Let α$ , β, does not hold.
on
1 (c) f − 1( A − B) = f − 1( A) − f − 1(B)
α$ × (β$ × γ$) = (β$ + γ$) where
2 (d) f − 1( A − B) = f − 1( A) ∪ f − 1(B)
α$ × (β$ × γ$) = (α$ ⋅ γ$)β$ − (α$ ⋅ β$)γ$ . If β$ is not parallel
pi

59. Let S, T , U be three non-void sets and


to γ$ , then the angle between α$ and β$ is f : S → T , g : T → U be so that gof : s → U is
m

5π π π 2π surjective. Then,
(a) (b) (c) (d)
6 6 3 3 (a) g and f are both surjective
ha

(b) g is surjective, f may not be so


54. The position vectors of the points A , B, C and
(c) f is surjective, g may not be so
D are 3 $i − 2 $j − k$ , 2 $i − 3 $j + 2 k$ , 5 $i − $j + 2 k$ and (d) f and g both may not be surjective
C

4 $i − $j − λk$ , respectively. If the points A , B, C  π


60. The polar coordinate of a point P is 2, −  .
and D lie on a plane, the value of λ is  4
dy

(a) 0 (b) 1 (c) 2 (d) − 4 The polar coordinate of the point Q which is
55. A particle starts at the origin and moves such that line joining PQ is bisected
u

1 unit horizontally to the right and reaches perpendicularly by the initial line, is
1 π π π π
(a)  2,  (b)  2,  (c)  − 2,  (d)  − 2, 
St

P1 , then it moves unit vertically up and


2  4  6  4  6
1
reaches P2 , then it moves unit horizontally
4 61. The length of conjugate axis of a hyperbola is
1 greater than the length of transverse axis.
to right and reaches P3 , then it moves unit Then, the eccentricity e is
8
vertically down and reaches P4 , then it moves 1
(a) = 2 (b) > 2 (c) < 2 (d) <
1 2
unit horizontally to right and reaches P5
16 x  q
and so on. Let Pn = (x n , y n) and lim x n = α 62. The value of lim is
and lim y n = β. Then, (α , β) is n → ∞ x → 0+ p  x 
n→ ∞
[q ]
(b)  ,  (c)  , 1 (d)  , 3
4 2 2 4 (a) (b) 0 (c) 1 (d) ∞
(a) (2, 3)
 3 5 5  3  p
14 WB JEE (Engineering) Solved Paper 2019

63. Let f (x) = x 4 − 4 x 3 + 4 x 2 + c, c ∈ R. Then, 67. Two particles A and B move from rest along a
straight line with constant accelerations f
(a) f( x) has infinitely many zeros in (1, 2) for all c
and h, respectively. If A takes m seconds more
(b) f( x) has exactly one zero in (1, 2) if −1 < c < 0
than B and describes n units more than that
(c) f( x) has double zeros in (1, 2) if − 1 < c < 0
of B acquiring the same speed, then
(d) whatever be the value of c, f( x) has no zero in
(1, 2) (a) (f + h) m2 = fhn
(b) (f − fh)m2 = fhn
64. The graphs of the polynomial x 2 − 1 and cos x
1
intersect (c) (h − f )n = fhm2

)
2

be
(a) at exactly two points 1
(b) at exactly 3 points (d) (f + h) n = fhm2
2
(c) at least 4 but at finitely many points.
68. The area bounded by y = x + 1 and y = cos x

Tu
(d) at infinitely many points.
10 and the X -axis, is
65. A point is in motion along a hyperbola y = 3
x (a) 1 sq unit (b) sq unit

ou
so that its abscissa x increases uniformly at a 2
rate of 1 unit per second. Then, the rate of 1 1
(c) sq unit (d) sq unit
change of its ordinate when the point passes 4 8
through (5, 2)
1
(a) increases at the rate of unit per second
2
(Y
69. Let x1, x 2 be the roots of x 2 − 3 x + a = 0 and
x 3 , x 4 be the roots of x 2 − 12 x + b = 0 .
on
1 If x1 < x 2 < x 3 < x 4 and x1 , x 2 , x 3 , x 4 are in
(b) decreases at the rate of unit per second
2 GP, then ab equals
2
24
pi

(c) decreases at the rate of unit per second


5 (a) (b) 64
5
2
(d) increases at the rate of unit per second
m

(c) 16 (d) 8
5
1 − i cos θ
70. If θ ∈ ú and is real number, then
ha

Category III (Q. Nos. 66 to 75) 1 + 2 i cos θ


Carry 2 marks each and one or more option(s) θ will be (when I : Set of integers)
π 3nπ
(a) (2 n + 1) , n ∈ I , n∈I
C

is/are correct. If all correct answers are not (b)


marked and also no incorrect answer is marked 2 2
then score = 2 × number of correct answers (c) nπ, n ∈ I (d) 2nπ, n ∈ I
dy

marked ÷ actual number of correct answers. If  3 0 3


any wrong option is marked or if any combination  
71. Let A = 0 3 0 . Then, the roots of the
including a wrong option is marked, the answer  3 0 3
u

will considered wrong, but there is no negative  


St

marking for the same and zero marks will be equation det(A − λI 3) = 0 (where I 3 is the
awarded. identity matrix of order 3) are
(a) 3, 0, 3 (b) 0, 3, 6
1
(c) 1, 0, − 6 (d) 3, 3, 6
66. Let I n = ∫ x tann −1
x dx . If a nI n + 2 + bnI n = cn
0 72. Straight lines x − y = 7 and x + 4 y = 2
for all n ≥ 1, then intersect at B. Points A and C are so chosen
(a) a1, a2 , a3 are in GP on these two lines such that AB = AC . The
(b) b1, b2 , b3 are in AP equation of line AC passing through (2 ,− 7) is
(c) c1, c 2 , c 3 are in HP
(a) x − y − 9 = 0 (b) 23 x + 7 y + 3 = 0
(d) a1, a2 , a3 are in AP
(c) 2 x − y − 11 = 0 (d) 7 x − 6 y − 56 = 0
WB JEE (Engineering) Solved Paper 2019 15

73. Equation of a tangent to the hyperbola (c) If g (a) = 0 = g (b ), the equation g ′( x) + kg ( x) = 0


5 x − y = 5 and which passes through an
2 2 is solvable in (a, b ), k ∈ú
(d) If g (a) = 0 = g (b ), the equation g ′( x) + kg ( x) = 0
external point (2, 8) is
may not be solvable in (a, b ), k ∈ú.
(a) 3 x − y + 2 = 0 (b) 3 x + y − 14 = 0
(c) 23 x − 3 y − 22 = 0 (d) 3 x − 23 y + 178 = 0 x3
75. Consider the function f (x) = − sin π x + 3
74. Let f and g be differentiable on the interval I 4
and let a , b ∈ I , a < b. Then, (a) f( x) does not attain value within the interval [− 2, 2 ]
1

)
(a) If f(a) = 0 = f(b ), the equation f ′( x) + f( x)g ′( x) = 0 (b) f( x) takes on the value 2 in the interval [− 2, 2 ]

be
is solvable in (a, b ) 3
1
(b) If f(a) = 0 = f(b ), the equation f ′( x) + f( x)g ′( x) = 0 (c) f( x) takes on the value 3 in the interval [− 2, 2 ]
4
may not be solvable in (a, b ).
(d) f( x) takes no value p, 1 < p < 5 in the interval [− 2, 2 ].

Tu
ou
Answers
Physics
1. (b) 2. (b) 3. (d) 4. (d) 5. (d) 6. (Y
(b) 7. (b) 8. (c) 9. (d) 10. (c)
on
11. (c) 12. (a) 13. (c) 14. (c) 15. (c) 16. (b) 17. (a) 18. (d) 19. (a) 20. (a)
21. (d) 22. (d) 23. (*) 24. (c) 25. (c) 26. (d) 27. (a) 28. (d) 29. (c) 30. (c)
31. (a) 32. (a) 33. (a) 34. (a) 35. (c) 36. (b, d) 37. (b) 38. (a, d) 39. (a, c) 40. (b, d)
pi

Chemistry
m

41. (c) 42. (a) 43. (d) 44. (b) 45. (a) 46. (b) 47. (c) 48. (b) 49. (d) 50. (b)
ha

51. (c) 52. (d) 53. (c) 54. (c) 55. (c) 56. (a) 57. (b) 58. (d) 59. (a) 60. (b)
61. (d) 62. (b) 63. (a) 64. (b) 65. (d) 66. (b) 67. (d) 68. (d) 69. (d) 70. (b)
71. (d) 72. (d) 73. (a) 74. (d) 75. (a) 76. (a, b) 77. (a, b) 78. (b, c, d) 79. (a, c) 80. (d)
C

Mathematics
dy

1. (b) 2. (b) 3. (d) 4. (b) 5. (a) 6. (d) 7. (d) 8. (c) 9. (c) 10. (d)
11. (b) 12. (b) 13. (b) 14. (b) 15. (c) 16. (b) 17. (d) 18. (a) 19. (b) 20. (d)
(b) (c) (c) (a) (d) (a) (d) (d) (c) (d)
u

21. 22. 23. 24. 25. 26. 27. 28. 29. 30.
31. (a) 32. (d) 33. (b) 34. (b) 35. (a) 36. (b) 37. (a) 38. (c) 39. (c) 40. (c)
St

41. (a) 42. (c) 43. (b) 44. (c) 45. (a) 46. (a) 47. (c) 48. (c) 49. (a) 50. (b, c)
51. (c) 52. (a) 53. (d) 54. (d) 55. (b) 56. (a) 57. (c) 58. (c) 59. (b) 60. (a)
61. (b) 62. (a) 63. (b) 64. (a) 65. (c) 66. (b, d) 67. (c) 68. (b) 69. (b) 70. (a)
71. (b) 72. (a, b) 73. (a, c) 74. (a, c) 75. (d)
Answer with Explanations
Physics
1. (b) According to the question, 3. (d) As we know that de-Broglie wavelength is
h h
given as λ = = …(i)
p mv
n

)
where, h = Planck constant

be
e0 e
θ θ p = momentum of particle
v = velocity of particle
and m = mass of the particle.

Tu
We know that incident ray, reflected ray and
normal lie in the same plane. Eq. (i) can be written as,
and angle of incidence = angle of reflection h h
λ= = [QKE = qv]

ou
$ will be along the angle bisector of e$ and
Therefore n 2m(KE) 2mqv
− e$ 0 , where, KE = Kinetic energy of particle
$ = e + (− e 0)
$ $ 1

(Y
i.e. n …(i) Hence, λ ∝
|e$ − e$ 0 | m
[QBisector will along a vector dividing in same λ proton melectron
ratio as the ratio of sides forming that angle] Now, =
λ electron
on
mproton
$ is a unit vector where|e$ − e$ 0 | = OC
But n
= 2OP = 2|e$ | Given, mass of proton, mproton = 2000 melectron
cosθ = 2cosθ λ proton
pi

1
=
Substituting this value in Eq. (i), we get λ electron 2000
$ = e − e0
m

$ $
n λ proton 1 λe
2cosθ = ⇒ λp =
λ electron 20 5 20 5
e$ = e$ 0 + (2cosθ) n
ha

$
e$ = e$ 0 − 2(n$ ⋅ e$ 0)n $ ⋅ e$ 0 = − cosθ]
$ [Qn 4. (d) According to the Bohr’s atomic model,
nh
2. (b) According to the question, Angular momentum, L = mvr = …(i)
C

α-particle β-particle

X → Y → Z(say) Since, angular velocity ω =
v
r
dy

According to Rutherford Soddy law of radioactive


decay, the rate of decay of radioactive along at any ⇒ v = rω
instant is proportional to the number of atoms From Eq. (i), we get
u

present at that instant nh


dN m(rω) r =

St

= − λN
dt nh
Since, λ = constant (decay constant) and mωr 2 =

N = number of atoms nh
Rate of decay of Y particle is given as, ω= …(ii)
2πmr 2
dN Y
λ X N X − λ Y NY = =0
dt Since, the radius of the electron in nth orbit of
[QDecay rate for β-particle become n2h2ε0
Bohr’s atomic model is given as, r = …(iii)
constant after some time] πmze 2
Given, rate of emission of β-particle = 107/h Squaring the Eq. (iii) and substituting its value in
Eq. (ii), we get
∴ λ X N X = λ Y N Y = 10 /h
7
nh(πmze)2 1
ω= ⇒ω ∝ 3
2πm(n4 h4 ε20) n
WB JEE (Engineering) Solved Paper 2019 17

5. (d) According to the question, 9. (d) According to the question, we can draw the
following diagram
1 kΩ
A

10 V 1 kΩ 6V
T T

B m1
a
m2 a

)
Input voltage VS = 10V m1g

be
Source resistance RS = 1kΩ
m 2g
Zener diode voltage VZ = 6V
Q Breakdown voltage of zener diode is = 6V, and the Here, T is the tension in the string.

Tu
potential difference across the zener diode 5V. In equilibrium condition,
∴Current flow in zener diode I Z = 0 For mass m1 , T − m1 g = m1 a …(i)
For mass m2, m2 g − T = m2a …(ii)

ou
6. (b) By using the de-Morgan’s law
After adding Eqs. (i) and (ii), we get
A + B = A⋅ B
m2 g − m1 g = m1 a + m2a

(Y
Hence, option (b) is the correct. (m2 − m1) g = (m1 + m2)a
(m − m1)
7. (b) As we know that, a= 2 g
Impulse, I = force (F) × small time interval (m1 + m2)
on
I = ma × t [Q F = ma] 10. (c) Given, Power (P) = constant
I = [M] [LT− 2] × [T] 1
Kinetic Energy (KE) = mv2
I = [M L T− 1 ]
pi

2
Hence, the correct dimensional formula for impulse KE mv2
We know that, P = ⇒P=
m

is given by [M L T− 1 ] . ∆t 2
Q P = constant,
8. (c) Given,
ha

Hence, velocity of the body v ∝ t …(i)


Maximum error in the measurement of mass = 0.3%
ds
Maximum error in the measurement of length As, Velocity v = …(ii)
= 0.2% dt
C

We know that, From Eqs. (i) and (ii), we get


ds
Error in density is given as, So, ∝ t
dy

mass (m) m dt
Density, ρ = = Integrating the above equation w.r.t. time ()
t,
volume (V) L3
ds
∫ dt ∝ ∫ t
u

where, L = side of cube


St

Error in density is given as,


we get, displacement of the body s ∝ t 3/ 2
 ∆ρ  ∆m 3∆L
  = + Q Displacement s = at 3/ 2, where a is constant.
 ρ  m L
 ∆ρ  ∆m 3∆L  11. (c) According to the question,
or   × 100 =  +  × 100
 ρ   m L  (r2) v v(r1)
P
Substituting the given values, we get
 ∆ρ 
  = (0.3% + 3(0.2)%) = 0.3% + 0.6%
 ρ  max
O
∴ Maximum percentage error measurement of Representation of position vectors
of two particles (drawn from the point P)
 ∆ρ 
density   = 0.9%.
 ρ  max
18 WB JEE (Engineering) Solved Paper 2019
In two dimension, the position vectors r1 and r2 13. (c) Given, Decrement in the length = 1%.
represented as
Poisson’s ratio for material of the rod, σ = 0.2
1
r1 = vt $i − gt 2$j …(i) As we know that,
2
1 Volume, v = πr 2l ,[where, l is the length of the rod]
r2 = vt(− $i) − gt 2($j) …(ii)
∆V 2∆r ∆l
2 = + …(i)
Q We know that, when the two vectors are V r l
mutually perpendicular, i.e. − ∆D / D
Since, Poisson’s ratio, σ =
θ = 90º ∆L/ L

)
r1 ⋅ r2 = r1 r2 cos 90º − ∆r / r

be
So,
=
r1 ⋅ r2 = 0 ∆L/ L
Substituting the values r1 and r2 in the above So, Eq. (i) can be written as,

Tu
relation, we get ∆V ∆l
1 1 = (1 − 2σ)
((vt)$i − gt 2$j) ⋅ (vt(− $i) − gt 2($j)) = 0 V l
2 2 ∆V  ∆l
× 100 =  × 100 × (1 − 2σ)

ou
1 24  l 
− v t + 4 g t = 0 (where, i ⋅ $i = $j ⋅ $j = k$ ⋅ k$ = 1)
2 2 $ V
4 = − 1 × [1 − 2 × (0.2)]
1
v2t 2 = g2t 4 = − 1 × [1 − 0.4]

⇒ v = gt
2
4
1 22
4 (Y = − 0.6%
Here, negative sign shows the decrement in the
volume.
on
1
∴Magnitude of velocity of the particles, v = gt
2 14. (c) When the spherical body falls with constant
We know that, separation distance between velocity, i.e. terminal velocity then the net force
pi

particles at a time t becomes zero, i.e the weight of body is equal to


∆x = 2vt the buoyancy force.
m

2v 4v2 Hence, Fnet = 0


∆x = 2 × v × ⇒ ∆x =
g g 15. (c) According to the question,
ha

Area of both bodies A and B = A


12. (a) According to the Kepler’s third law
Temperature of body A = 27ºC = 27 + 273 K
T2 ∝ r3
C

Temperature of body B = 177ºC = 177 + 273 K


where, T = time period of revolution
Now, by Stefan-Boltzmann law, thermal energy
r = radius
radiated per second by a body
dy

2 3 2/ 3
T  r  r T  Q = σAT 4 ,
Now,  E  =  E  = E =  E 
 TP   rP  rP  TP  where, A = Area
u

2/ 3
rE  2π /ωE  Q T = 2π  T = temperature
= 
 ω  σ = Stefan-Boltzmam’s constant
St

rP  2π /ωP  and
2/ 3 So, the ratio of thermal energy radiated per second
rE  ωP  by A to that by B is
= 
rP  ωE  4
Q1 σ A  T1 
According to the question, ωP = 2ωE and rE = R =  
Q2 σA  T2 
2/ 3
R  2ωE  4
⇒ =  Q1  T1 
rP  ωE  Now, = 
Q2  T2 
R
= (2)2/ 3 4
rP Q1  273 + 27 
= 
R Q2  273 + 177 
rP = = R(2)− 2/ 3
(2)2/ 3 Q1  300 
4
= 
Q2  450 
WB JEE (Engineering) Solved Paper 2019 19
4
Q1  2
=   =  
16 Similarly, in branch CD,
Q2  3  81  CCD = 10 µF
Ratio of thermal energy radiated per second Now, C AB and CCD are connected in parallel
combination. Hence, the equivalent capacitance of
Q1 : Q2 = 16 : 81 the circuit,
16. (b) For a gas at temperature T, the internal energy, Ceq = 10 + 10 = 20 µF
f We know that, charge Q = CeqV
U= µRT
2 Q = 20 × 10− 6 × 150 V

)
where, f = degree of freedom Amount of charge stored Q = 3 × 10− 3C

be
f
⇒ Change in energy, ∆U = µR ∆T …(i)
2 19. (a) The clock diagram is as shown below
Also, as we know for any gas heat supplied at +Q

Tu
+Q +Q
constant volume 12
11 1
(∆QV) = µCV ∆T = ∆U …(ii) +Q
10 2
where, CV = molar specific heat at constant volume

ou
r
From Eqs. (i) and (ii), we get +Q 9 3 +Q
1 C
CV = fR
8

(Y
2 4 +Q
+Q
For diatomic gas, degree of freedom, f = 5 7 6 5
5 +Q +Q
CV = R +Q
2
on
In the above diagram, charge + Q is not placed at
17. (a) Since, the whole system is isolated, this 10 h position.
means, there is no transfer of heat between the
So, net electric field strength at centre C,
pi

system and the surrounding. Also, the right side


container is initially vacuum, so the gas could E net = E1 + E 2 + E 3 + E 4 + E 5 + E 6
+ E 7 + E 8 + E 9 + E11 + E12
m

easily rush there without any resistive force. As


the right side container has no temperature. Thus, ⇒ E net = E1 + E 2 + E 3 + E 4
the temperature of the ideal gas would remain + E 5 + E 6 + (− E1 − E 2 − E 3 − E 5 − E 6)
ha

same even if it enters right side chamber. Q


⇒ E net = E 4 = , from centre towards the
Therefore, final temperature attained at the 4 πε0 r 2
equilibrium will be T.
C

mark 10.
18. (d) The given circuit shows a balanced 20. (a) According to the question,
Wheatstone bridge. Now, the circuit becomes
dy

Q
20µF 20µF
C D r
d
u

θ θ
St

x E
–q
A B
20µF 20µF d

Q
150 V Force experienced by the charge − q due to charge Q
In the branch AB, both capacitor are arranged in 2kQq  1 
F = − 2 cosθ … (i) where, k = 
the series combination. Hence, its equivalent r  4 πε 0
capacitance is given by x
1 1 1 2 1 From diagram, cosθ =
= + = = r
C AB 20 20 20 10
By substituting the value of cosθ in Eq. (i)
C AB = 10 µF
20 WB JEE (Engineering) Solved Paper 2019

F=−
2kQq x
⋅ Q Net magnetic field at the centre = 0
r2 r So, magnetic force Fmagnetic = q(v × B) = 0
2kQx
or F=− So, the magnetic force experienced by the particle
r3 when it passes through the centre is F = 0
2kQx Q r 2 = x 2 + d 2 
or F=−   23. (c) According to the question,
(x + d 2)3/ 2
2
 r = x 2 + d 2  P
For, x < < d, so x can be neglected
2

)
2kQx
F=− 3 3m 4m

be
d
So, the force developed by negative charge (− q) due
to the system of the charges as shown in the figure A B

Tu
is, 5m
− 4kQqx
F= µ0 × I
d3 Magnetic field due to first wire (A) is B1 =

ou
2π × 3
⇒ F∝x
µ ×I
So, the forced developed by negative charge is Magnetic field due to second wire (B) is B2 = 0
directly proportional to the distance x. 2π × 4

21. (d) If velocity of particle, v is perpendicular to


magnetic field, B i.e. θ = 90º, then particle will (Y Net magnitude of magnetic field B =
µ 0I 1
B= +
1
=
µ 0I × 5
B12 + B22
on
experience maximum magnetic force, i.e. 2π 9 16 2π × 3 × 4
Fmax = qvB. This force acts in a direction
5 µ 0I
perpendicular to the motion of charged particle. Magnetic field B = ×
Therefore the trajectory of the particle is a circle. 24 π
pi

In this case path of charged particle is circular and 24. (c) As we know that, magnetic field due to a long
magnetic force provides the necessary centripetal
m

wire at a distance x from it is given by


force,
µ I
mv2 B = 0 , where, I is the current flowing through
2πx
ha

i.e. qvB =
r the wire β ∝ I
mv ∴Magnetic flux associated with the square loop,
⇒ Radius of path, r =
C

qB φ ∝β ∝ I
r=
p
[Q p = mv] Now, if the current increase, then φ also increases.
qB Direction of long wire will be q.
dy

where, p = momentum of the particle This means, magnetic field due to induced
∴ r ∝ momentum current will be opposite to the existing magnetic
u

field, i.e. according to Lenz’s law,


Hence, option (d) is correct.
The induced current in the loop will be in the
St

22. (d) Given, electric current in circular wire = I anti-clockwise direction. Now,
Radius of wire = r
Charge on particle = q FAD
Speed of the particle = v
The given loop can be as shown in the figure below. D A
I/2 FAB
FCD

r1
I I
C B

FBC
r2
I/2
WB JEE (Engineering) Solved Paper 2019 21
Since, wires attract each other, if current flowing 5 7
=
through them is in same direction and repel each S S+ 3
other, if currents are in opposite direction.
5 (S + 3) = 7S
∴ Part CD of the loop will experience a force of
5 S + 15 = 7S
repulsion, whereas part AB will experience
attraction. Parts BC and AD will not experience any 2S = 15
force. Thus, the overall force will be a force of S = 7.5 Ω.
repulsion because AB is closer to straight. The force
between two current carrying conductors is 27. (a) Given, inductance of inductor, L = 60 mH

)
inversely proportional to the distance between = 60 × 10− 3 H

be
them
Phase difference between voltage and current in
1
F∝ L-R circuit, θ1 = 60º
r
Capacitance of capacitor, C = 0.5 µF = 0.5 × 10− 6F

Tu
Q r1 < r2
Phase difference between voltage and current in
So, FCD > FAB
R-C circuit, θ2 = 30º
Fnet = FCD − FAB

ou
For L-R circuit,
Hence, the loop will moves away from the wire. X
tanθ1 = L
25. (c) According to the question, R

(Y
ωL
I1 R R I2 R R tanθ1 = …(i) [Q X L = ωL]
I=I1/8 R
I0 I1/2 I 1 /4 I 1 /8 I Similarly, for R-C circuit,
2R V 2R 2R 2R 2R
on
X
tanθ2 = C
R
Total resistance of the given circuit 1
pi

…(ii) Q X C =
1 
Req = 2R tanθ2 = ωC
R  ωC 
Now, circuit
m

From Eqs. (i) and (ii), we get


V
2R=I tanθ1 ωL
= = ω2LC
ha

tanθ2 R × 1
I0
ωCR
2R V 2R tan 60º
C

= ω LC
2
tan 30º
B 3
= ω2LC
dy

V / 2R 1/ 3
∴ I= (Q I1 = V / 2R)
8 ω2LC = 3
u

So, current I in the circuit I =


V Q I = I1  3
  ω2 =
16R  8
St

LC
3
26. (d) According to the balanced condition of ω2 =
Wheatstone bridge, 60 × 10− 3 × 0.5 × 10− 6
In the first case, 3
ω2 =
P
=
R 30 × 10− 9
Q S
ω = 108
P 5
= …(i)
Q S ω = 104
In the second case As we know that,
P
=
7
…(ii) ω = 2πf
Q S+ 3 ω 104 1
f = = Hz = × 104 Hz
From Eqs. (i) and (ii), we get 2π 2π 2π
22 WB JEE (Engineering) Solved Paper 2019

28. (d) According to the question, we can draw the 30. (c) Given, initial frequency of light,
following diagram f1 = 4 × 1014 s− 1
f=0.05 m Final frequency of light, f2 = 5 × 1014 s− 1
Change in wavelength , ∆λ = λ1 − λ 2
c c
axis or ∆λ = −
0.2 m
f1 f2
3 × 108 3 × 108
= −

)
4 × 1014
5 × 1014

be
Convex lens
3 × 108  1 1 
Given, u = − 0.2 m and f = 0.05 m =  − 
1014  4 5
As we know that,

Tu
3 × 108 1
1 1 1
= − …(i) = ×
f v u 1014 20
1 1 1 = 1.5 × 10− 7 m

ou
= +
v f u Now, we know that,
1 1 1 Angular width of central maxima,
= −
2λ 2∆λ

(Y
v 0.05 0.2 θ= or ∆θ =
1 100 10 d d
= −
5 2 2∆λ
v d=
1 1 ∆θ
on
= 20 − 5 ⇒ v = m
v 15 Here, ∆θ = 0.6 radian
Now, differentiating eq. (i), we get 2 × 1.5 × 10− 7
d= = 5 × 10−7 m
pi

dv du v2 0.6
− 2 = − 2 ∴ dv = du 2
v u u 31. (a) We know that, energy stored in the capacitor
m

2
2
 1 
A max = A ×   × 
1 1 2
 =CE
 15  (− 0.2)  2
ha

1 and energy supplied by the source of emf


A max = A × × 25
225 = CE 2
C

A ∴ Energy dissipated in resistance R


A max =
9 = Energy supplied by the source of emf E
Here A is in cm. − Energy stored in the capacitor
dy

A
Hence, A max = × 10− 2 m 1
= CE − CE 2
2
9 2
u

29. (c) As we know that,  1 1


= 1 −  CE 2 = CE 2
 2
St

λD 2
fringe width, β =
d
32. (a) Given, cross sectional area of nozzle
where, λ = wavelength of light 5
D = Distance of the screen from the slits A= × 10− 3 m2
21
d = separation between the slits 1
∆β ∆D ∆d and rate of heat transfer Q = = 10− 1 m3 / s
Now, × 100 = × 100 + × 100 10
β D d
Q Heat transfer Q = AV
According to the question, Q 10− 1 m3 / s
∆β ∴ V= = = 2 21 × 10 m/s
× 100 = 0.5 + (− 0.3) = 0.5 − 0.3 = 0.2% A 5
× 10− 3 m2
β 21
Hence, the fringe width increases by 0.2%.
WB JEE (Engineering) Solved Paper 2019 23
When it hits a rigid wall then maximum possible E
So, v = is the condition for which the particle
increase in temperature of water can be expressed B
as moves undeflected in its original trajectory.
1 2
mv = ms∆T …(i) 35. (c) In given, Series R - L - C circuit
2
Resistance R = 100 Ω
where, m = mass, s = specific heat of water
Inductance of Inductor, L = 20 mH = 20 × 10− 3H
= 1 cal / g = 4.2 × 103 J / kg
1250
and ∆T = increase in temperature Resonance frequency f = Hz
π

)
1 v2
From Eq. (i), ∆T = Source voltage VDC = 25V

be
2s
According to resonant frequency,
(2 21 × 101)2
= ω0 = 2πf0 =
1
2 × 4.2 × 103

Tu
LC
84 × 102 1
= or (2πf0) =
2
8.4 × 103 LC

ou
= 1° C 2 1250 × 1250 1
or 4π =
π× π LC
33. (a) Maximum loss in K . E = Total K . E energy
1000

(Y
before collision of two ice blocks or C=
1250 × 1250 × 4 × 20
1 1
K . Emax loss = mv2 + mv2 = mv2
2 2 (QBy substituting L = 20 × 10− 3)
on
This maximum loss in kinetic energy will be equal or C = 8 × 10− 6F
to loss in heat to melt two ice block. We know that, Charge QS = CV
⇒ mv2 = (ms ∆ θ + mL)2 …(i) QS = 8 × 10− 6 × 25 = 0.2 mC
pi

According to question, So, the amount of charge stored in each plate of


. × 105 J kg − 1
L = 336
m

capacitor is 0.2 mC.


S = 2100 J kg − 1 K − 1 36. (b,d) In photoelectric effect, if the incident light
ha

∆θ = 8º C had a frequency less than a minimum frequency


From Eq. (i) v = 2(s ∆ θ + L) ν0 , then no electrons are ejected regardless of the
light’s amplitude. This minimum frequency is also
C

= 2(2100 × 8 + 336
. × 105) called the threshold frequency, and the value of ν0
depends on the metal. For frequency greater than
v = 840 ms −1
ν0 , electrons would be ejected from the metal.
dy

Furthermore, the kinetic energy of the


34. (a) According to the question,
photoelectrons is proportional to the light
Charge on particle is = q frequency (ν).
u

Velocity of particle = v The relationship between photoelectron kinetic


Due to uniform electric field, energy T and light frequency ν is shown in graph
St

electric force on particle below


Felectric = qE …(i)
Due to a uniform magnetic field,
magnetic force on particle is given by, Light
kinetic
Fmagnetic = q(v × B) … (ii) energy
When v is perpendicular to E and B, which are (T)
mutually perpendicular to each other,
Felectric = Fmagnetic
v0
From Eqs. (i) and (ii) Light frequency (n) Hz
qE = qvB
Graph shows that kinetic energy T is linearly
E
∴ v= increasing with light frequency (ν).
B
24 WB JEE (Engineering) Solved Paper 2019
However, for a given photosensitive material and 39. (a, c) The circuit diagram is as shown below,
frequency of incident light, number of
photoelectrons emitted per second is directly r r r r r r
proportional to the intensity of incident light
i.e., Number of electrons emitted ∝ J. A B
Also, photoelectric emission is an instantaneous r r r
process.
According to question, when given pair of point in
37. (b) The equation of state for an ideal gas options are connected through a resistance R, then
undergoing adiabatic process is given as,

)
equivalent resistance between point A and B (R AB)
TV γ −1 = constant

be
remains unchanged. It is only possible when current
does not flows through resistance R and circuit become
Let the temperature after adiabatic compression Wheatstone bridge as shown in the figure below,
as given in the question be T then,

Tu
γ −1 C
T0 V γ − 1 = T  0 
V
 2 P Q

ou
γ −1
⇒ T = T0 2
A G B
V0
Now, heat released at volume to achieve
2 R S
temperature T0 .
The net heat released can be determined by the
equation.
(Y At the balanced condition,
D
P R
=
on
∆Q = µCV ∆T Q S
R
=µ × (T0 2 γ − 1 − T0) The current flow in CD branch will be zero.
γ −1 Now by checking each option,
pi

µRT0 γ − 1 from option (a), circuit is,


= (2 − 1) (QµT0 R = p0 V0)
γ −1
m

2
p V
∴ Heat released = 0 0 (2 γ − 1 − 1) 2r 4r
γ −1
ha

A R B
38. (a, d) Given, initial velocity of particle,
−1
u = 10 ms
C

r 2r
Range, R = 5 m
6
Gravitational acceleration,
dy

g = 10 ms− 2 P R 2r r 1 1
Now,Q = ⇒ = ⇒ =
Q S 4r 2 r 2 2
As we known that,
u

Hence, option (a) is correct.


u2 sin 2θ
Range of projectile, R = From option (b), circuit is
St

g
3
(10)2 sin 2α
5=
10 3r 2r
5
sin 2α =
10 A R B
1
sin 2α = r 2r
2
α = 15º 6
or (90° − 15° = 75º) P R 3r r
Now, = ⇒ ≠
Q S 3r 2r
WB JEE (Engineering) Solved Paper 2019 25
So, option (b) is also incorrect. P R 4r r
Now, = ⇒ ≠
From option (c) circuit is, Q S 2r 2r
4 So, option (d) is also incorrect.
Hence, option (a) and (c) are correct.
4r 2r
40. (b, d) According to the question,
A R B B

r 2r

)
7

be
Metallic loop
P R 4r 2r 2 2
Now, = ⇒ = ⇒ =
Q S 2r r 1 1

Tu
So, option (c) is also correct. Given, A metallic loop is placed in uniform
From option (d), circuit is magnetic field B. Magnetic field B and metallic loop
4 are perpendicular to each other. Then, if metallic

ou
loop is moved along B or rotated about its own axis,
4r 2r the net flux associated with it remains constant.
Thus, no emf will be induced in these cases.
A R B

(Y
However, when the loop is squeezed to a smaller
area or rotated about one of its diameters. Then its
r 2r flux changes. Thus, emf is induced. So, option (b)
7 and (d) both are correct.
on
Chemistry
pi

41. (c) 2, 2, 2-trichloroacetophenone reacts with 42. (a) When salicylic acid reacts with acetic anhydride
aqueous KOH to form potassium Salt of benzoic in the presence of conc.H 2SO 4 and heat, the product
m

acid, which undergoes hydrolysis to form benzoic will be (Q) acetyl salicylic acid (C9 H 8O 4).
acid as the final product. CO2H CO2H
ha

acetic anhydride
O O
Conc.H2SO4 (cat) Heat
C C HO H3COCO
(i) aq KOH 'Q'
C

CCl3 OH
(C9 H8 O4)
(ii) H3O+
Mechanism In presence of acid anhydride,
2, 2, 2-trichloro Benzoic acid nucleophilic acyl substitution reaction takes place.
dy

acetophenone P O O O

Mechanism C C C
u

OH H 3C O CH3
+
O O
Acetic anhydride
St

HO (C4 H6 O3)
C C
CCl3 OK Salicylic acid
+ aq KOH + CHCl3 (C7 H6 O3) Conc. H2SO4 ∆
H3O+ O
2, 2, 2-trichloro
C
acetophenone O OH O
C O +
OH C H
H3C O
C
'P'
Benzoic acid O CH3
Acetyl salicylic acid Acetic acid
The product of the above reaction P is benzoic acid. (C9 H8 O4) (C2 H4 O2)
Thus, option (c) is correct answer. Aspirin
So, the option (a) is correct answer.
26 WB JEE (Engineering) Solved Paper 2019
0.5 / 15
43. (d) When cyclopentanol on reaction with NaH ∴ pH = pK a + log ⇒ pH = pK a
followed by CS2 and CH 3I produces a xanthate. 0.5 / 15
OH The value of pK a is the lowest for the mixture of
NaH CH 3COOH and CH 3COONa (acidic buffer).
Xanthate
CS2/CH3I Therefore, it has lowest pH.
Cyclopentanol
Hence, pH value for option (c) is the lowest.
Mechanism 48. (b) Given, for two first order reactions.
–+ A → B, k = 0.693 min− 1 …(1)

)
OH ONa
NaH S=C=S and A → C, t1 / 2 = 0.693 min …(2)

be
For first order reaction
Cyclopentanol S S 0.693
–+
t1 / 2 = ,

Tu
O—C—SNa O—C—S— CH3 k
CH3-I
+ NaI Thus, for the reaction A → B
0.693 0.693
Xanthate t1 / 2 = = = 1.0 min

ou
k 0.693
So, the option (d) is correct answer.
Also, for first order reaction, lower the t1 / 2 value,
44. (b) CH 3  C Cl hydrolyses to form CH 3COOH faster is the reaction.

(Y
 1
Q t1 / 2 ∝
O k
even at 25ºC, which subsequently reacts with 0.693
NaHCO 3 (sodium bicarbonate) present in the and k for A → C is = 1 min− 1
0.693
on
same medium to form carbon dioxide (CO 2).
Thus, reaction (2) is faster than reaction (1) or
The reaction involved is given below. reaction (1) is slower than reaction (2).
CH 3COCl + NaHCO 3 → CH 3COONa + HCl + CO 2
pi

Hence, option (b) is the correct answer.


Thus, the option (b) is correct answer.
49. (d) For the given relation H 2O()l - H 2O (v).
m

45. (a) In BH −4 indicated atom is not a nucleophilic (i) At equilibrium ∆G = 0



(ii) Entropy change, i.e. ∆S is the measurement of
ha

site because there is no lone-pair on ‘B’ atom.


Hence, the option (a) is correct answr. randomness. It increases in vapour-state,
i.e. ∆S = (+) ve or ∆S > 0
46. (b) One millimole = 0.001 mol (iii) When liquid H 2O changes into vapour H 2O, it
C

i.e.1 × 10− 3 mol. requires heat energy. Thus, ∆H > 0


Hence, correct set is
Q 96500 C is required for 1 mol of electrons.
dy

∆G = 0, ∆H > 0 and ∆S > 0


193 × 1
∴ 193 C give = 0.002 mol, i.e. 2 × 10− 3 mol of So, option (d) is the correct answer.
96500
50. (b) Dimensions of  ab2  are shown below.
u

electrons.
0.002 V 
Thus, value of n in M n + is = =2
St

0.001 a
× b =pressure ×volume
Hence, option (b) is the correct answer. V2
force
47. (c) Key Point = × volume
area
For buffer solutions,
MLT−2
pH = pK a + log
[Salt] = × L3 = ML2T−2
[Acid] / [Base] L2
Moles ofCH 3COOH = 5mL × 0.01 m mol = 0.5 m mol The units of energy (joule) is,
Moles of CH 3COONa = 10 mL × 0.05 m mol 1 J = 1 kg m2s−2 = ML2T−2
= 0.5 m mol Therefore, the dimensions of  2  is same as that
ab
Total volume of solution = 5 + 10 = 15mL V 
of energy. So, the option (b) is correct.
WB JEE (Engineering) Solved Paper 2019 27

51. (c) In the equilibrium H 2 + I 2 - 2HI, if at a In the crystalline solid MSO 4 ⋅ nH 2O, the value of
n is 5.
given temperature, the concentrations of the
reactants are increased, the value of the Hence, option (c) is correct answer.
equilibrium constant K C will remain the same 56. (a) Given, mass of gas (W) = 7.5 g
because equilibrium constant does not depend on
the molar concentration of reactants. Volume of gas at STP (V) = 56. L
V(L) W
The option (c) is correct answer. Q Moles of gas at STP = =
22 . 4(L) M
52. (d) Electrolysis of copper sulphate solution using
Where M = molar mass of gas.

)
Cu-electrodes,
W × 22 . 4

be
Ions present : Cu 2 + , H+ , SO 24 − , OH − ∴ M=
V
At cathode Cu 2 + (aq) + 2e − → Cu (s)
7.5 × 22.4
At anode Cu(s) − 2e − → Cu 2 + (aq) = = 30.00 g mol −1

Tu
56
.
So, the option (d) is correct answer.
Among the given options
53. (c) The three quantum numbers n, l and m that Molar mass(M) of

ou
describe an orbital has integer values of 0, 1, 2, 3 (a) NO = 14 + 16 = 30.00 g mol −1
and so on.
(b) N 2O = 28 + 16 = 44.00 g mol −1
Name
Principal quantum
number
Symbol
n
Range of Values
1≤ n
(Y (c) CO = 12 + 16 = 28.00 g mol −1
(d) CO 2 = 12 + 32 = 44.00 g mol −1
Q Molar mass of NO = 30 g mol −1 .
on
Azimuthal quantum l 0≤ l ≤ n − 1
number Thus, the given gas is NO.
Magnetic quantum m − l ≤ m≤ l Hence, (a) is the correct option.
pi

number 57. (b) Let, initial concentration (a) = 100


m

The electronic arrangement of option (c) is absurd Given, half-life (t1 / 2) = 60 days
because To find, radioactivity,
ha

n= 2 i.e. (a − x) after time T (180 days)


l=0 Q T = n × t1 / 2
m= 0 where, n = no. of half-lives
C

To sum up, where n = 2, l = 0, m = 0 180 = n × 60


Hence, for l = 0, value of m should be zero. 180
n= =3
dy

60
54. (c) The quantity hv / K B corresponds to a 100 100
and (a − x) = n = 3 =
temperature. hv = 3 / 2 K B T 2 2 8
u

where, T = temperature (a − x) = 12 .5%


K B = Boltzmann constant
St

Hence, (b) is the correct answer.


The percentage of H 2O in the solid is
58. (d) Given, 82 A
210
→ B → C → 82D
206
= (100 − 64) = 36%
hv 3 i.e. difference in mass no. between A and D is of 4
= T
KB 2 units i.e., mass no. is decreased by 4 units while
atomic no. remains the same, i.e. 82 for A and D.
3 hv
Since, is a constant, the value of corresponds Also,
2 KB
(i) On emission of each β-particle, mass no.
to temperature. Therefore option (c) is correct.
remains the same but atomic no. is increased
55. (c) Mass of H 2O = 36 × 250 = 90 g by one unit.
100 (ii) On emission of each α-particle, mass no. is
90
Moles of H 2O = = 5mol decreased by 4 units, while atomic no. is
18 decreased by 2 units.
28 WB JEE (Engineering) Solved Paper 2019
Thus, on emission of 2-β and 1-α particle, (ii) Smaller the size, more is value of electron affinity, i.e.
we get 82 D 206, more (−) ve will be the electron gain enthalpy.
−β −β −α On moving across a period, the size of atoms
i.e. 82 A
210
→ 83 A210 → 84 A210 → 82 A206 decreases due to increase in nuclear charge.
Thus, option (d) is the correct answer. (i) Electron configuration of the elements are
shown below.
59. (a) Key Point For similar electronic configuration of
C(Z = 6) = 1s 2 2s 2 2p 2
outer most shell, size will decide the value of ionisation
energy. More the size, lesser is the value of ionisation N(Z = 7) = 1s 2 2s 2 2p 3

)
energy. O(Z = 8) = 1s 2 2s 2 2p 4

be
Electronic configuration IInd I.E (ii) Due to half-filled electronic configuration of
Zn+ ion (Z = 30) = [Ar] 3d 10 4s1, (1734 kJ/mol). nitrogen (i.e. 3 electrons in 3p orbitals), it is
highly stable and has (+) ve value of electron

Tu
Cd + ion (Z = 48) = [Kr] 4d 10 5 s1, (1631 kJ/mol) gain enthalpy (i.e. about 30.9 kJ/mol).
+
Hg ion(Z = 80) = [Xe] 4f ⋅ 5d 14 10 1
6s (1809 kJ/mol) (iii) As size of ‘O’, is smaller than that of ‘C’, ‘O’
atom has higher (−) ve electron gain enthalpy

ou
Q Size of Cd+ > Zn+ , thus it has lower IInd (about − 141.1 kJ / mol) than that of carbon
ionisation energy while due to lanthanoid effect (C-atom) [about − 122.3 kJ / mol].
(i.e., poor screening by 4 f and 5d electrons, Hg + has Hence, correct order is O > C > N and (b) is
higher IInd ionisation-energy.
Hence, correct order is,
Zn > Cd < Hg and option (a) is the correct answer.
(Y the correct option.

63. (a) Key point As the percentage of s character in a


bond increases, the bond angle also increases. Therefore,
on
60. (b) Key Point A compound having more ionic as the bond angle increases, the percentage of
character, has more melting point. p-character decreases.
pi

Be and Ca belong to the same group and ionic In the given options,
character increases down the group. Ammonia (NH 3) has bond angle = 107.6º
m

Thus, BeCl 2 is less ionic than CaCl 2. This means and Phosphine (PH 3) has bond angle = 93. 5º
that melting point of CaCl 2 is higher than that of 1
BeCl 2. Q Bond angle ∝ and ∝ s-character
ha

p - character
Also, Hg is a transition metal, its compound is less
ionic than BeCl 2. Therefore, order of melting point Bond angle of NH 3 > PH 3 due to lone-pair lone-pair
will be repulsion.
C

CaCl 2 > BeCl 2 > HgCl 2 Thus, the lone pair on NH 3 has less p-character.
or (ii) > (i) > (iii) Hence, option (a) is the correct answer.
dy

Hence, option (b) is the correct answer. 64. (b) Chlorine bleach is NaOCl. The hypochlorite
ion in chlorine bleach dissociates to give nascent
61. (d) Key point The species which has one or more oxygen as shown below.
u

unpaired electrons have non-zero magnetic moment.


OCl − → [O] + Cl −
St

Electronic configuration of
Therefore, the reactive species in chlorine bleach is
Na+(Z = 11) = 1s 2 2s 2 2p 6
OCl − .
Mg (Z = 12) = 1s 2 2s 2 2p 6 3s 2 So, the option (b) is correct.
F − (Z = 9) = 1s 2 2s 2 2p 6 65. (d) Co-ordinate compound of Co (III) dissociates
Ar + (Z = 18) = 1s 2 2s 2 2p 6 3s 2 3p 5 into 3 ions in the solution means it has two
ionisable-ions out side the coordination sphere
As, Ar + has an unpaired electron, it has non-zero with oxidation state of cobalt (Co) = + 3
magnetic moment. Hence, (d) is the correct option.
In option (a)
62. (b) Key Point Name : Hexaammine cobalt (III) chloride
(i) Half-filled/fully-filled configuration have (−) ve or Formula : [Co(NH 3)6]Cl 3
low electron affinity i.e., (+) ve or high electron gain Q Oxidation state of Co = (+) 3
enthalpy.
WB JEE (Engineering) Solved Paper 2019 29
In option (b) anti-conformations can be interconverted by
Name : Pentaammine sulphatocobalt (III) chloride rotation around C2-C3 linkage.
Formula : [Co(NH 3)5(SO 4)]Cl CH3 CH3 CH3
CH3
Q Oxidation state of Co = + 3 H H H CH3
In option (c)
Name : Pentaamminechloridocobalt (III) sulphate H H H H H H
Formula : [Co(NH 3)5Cl]SO 4 H H
CH3 H
Q Oxidation state of Co = + 3

)
Anti- Gauche- Eclipsed-
In option (d)

be
conformer conformer conformer
Name : Pentaamminechloridocobalt (III) chloride
Hence, the option (d) is correct answer.
Formula : [Co(NH 3)5Cl]Cl 2

Tu
QOxidation state of Co = + 3 70. (b) In halogen acids, as the size of halogen atom
increases, the bond between halogen and
Dissociation of this compound is shown below:
2+
hydrogen atom weakens. Therefore, the case with
[CO(NH 3)5Cl]Cl 2 → CO(NH 3)5Cl] + 2Cl − which the bond can be broken increases.

ou
144424443
3 ions The correct order of the addition reaction rates of
This compound can give total 3-ions, as follows. halogen acids with ethylene is hydrogen iodide
(HI) > Hydrogen bromide (HBr)
one [Co(NH 3)5Cl]2 + and two Cl − ions.
Hence, option (d) is the correct answer.
66. (b) In the Bayer’s process, the leaching of
(Y > Hydrogen chloride (HCl)
So, the option (b) is correct answer.
on
alumina is done by using NaOH. 71. (d) Total number of isomeric linear dipeptides
which can be synthesised from racemic mixture of
Al 2O 3(s) + 2NaOH(aq)  
→
150º C, 35 atm
alanine are four (4), by (R) type and (S) type.
Bauxite ore
pi

2Na[Al(OH)4 ] +3H 2O (Q Has two chiral centres)


Sodium i.e.
m

aluminate
(soluble in water) O
CH3 H
2Na[Al(OH)4 ] + CO 2 → Al 2O 3 ⋅ xH 2O + 2NaHCO 3
ha

H 2N COOH C
Al 2O 3 ⋅ xH 2O(s) 1470
 
K
→ Al 2O 3(s) + xH 2O(g) H 2N COOH
Pure alumina H 3C H NH
So, the option (b) is correct answer. CH3 H
C

The possible isomers are


67. (d) Among the given options,
238 (RR), (SR), (RS) and (SS).
dy

92 U is an isotope of uranium but cannot be used


Hence, total (4) isomeric linear dipeptides for
as a nuclear-fuel. U-238 is non-fissile, i.e., it does
alanine can be synthesised and option (d) is the
not undergo fission by thermal neutrons. The
correct answer.
u

energy released when U-238 absorbs a neutron is


insufficient to carry out nuclear fission. 72. (d) From the given graph
St

Hence, (d) is the correct option. (rate)1 / 2


Slope = =4
68. (d) The molecule in which a lone pair of electrons [A]
undergoes conjugation with an alternate double Q rate (r) = k[A]n
bond is said to be delocalised. The molecule that Where, k = rate constant
has delocalised lone-pair of electrons is,
[A] = concentration at time t (in min).
O—CH3 O—CH3 n = order.
According to the graph (n = 2)
rate
∴ k= = [4]2 = 16
So, option (d) is correct. [A]n
Hence, n = 2and
69. (d) The conformation of n-butane, commonly k = 16.0 dm3 mol − 1 min− 1
known as eclipsed, gauche and
30 WB JEE (Engineering) Solved Paper 2019
and option (d) is the correct answer. (b)
O O α
73. (a) Key Point Me
I2/KOH
CH3
According to Kirchhoff’s equation Ph Ph
(O) O
∆H( f ) = ∆H( i) + CP (∆T) OH
At constant pressure (2º alcohol)
∆Cp = O
Therefore, heat of formation (∆H f ) does not O O
ONa CI3
depend on temperature and value of ∆Cp ,

)
Ph + CHI3 Ph
i.e. when ∆Cp = 0

be
O
O
Hence, option (a) is the correct answer.

74. (d) A copper coin was electroplated with zinc

Tu
Thus, options (a, b) are correct.
(Zn) and then heated at high temperature until
there is a change in colour. The resulting colour 77. (a, b)
will be golden. (a) In CrO 5 oxidation no. of Cr is + 6.

ou
The golden colour is due to the zinc migrating
–2
through the copper to form the alpha-form of brass –1O O O –1
alloy (percentage of Cu > 65% and that of Zn <
35%).

Zn + Cu → Brass
Thus, the option (d) is correct answer.
(Y –1O
Cr
O –1
on
(b) For the reaction (in ideal case)
75. (a) N 2 O4 (g) → 2NO2 (g)
OH Peracid
O
OH ∆H = ∆U + p. ∆V
pi

Q ∆H = ∆U + ∆n g RT
RCO3H
Allyl (C3H6O2) (Q p∆V = ∆n g RT)
m

[O]
alcohol
Where, ∆n g = (gaseous moles of product)
Oxidation of allyl alcohol with a peracid gives a
− (gaseous moles of reactant)
ha

compound of molecular formula C3H 6O 2, which


contains an asymmetric carbon atom. ∆n g = 2 − 1 = 1
The structure of the compound is Thus, ∆H > ∆U
C

O (c) pH of 0.1 N H 2SO 4 is less than of 0.1 HCl at


OH
25º is a wrong statement.
dy

pH of H 2SO 4
So, the option (a) is correct answer.
Q pH = − log[H + ] = − log[10− 2]
76. (a, b) Haloform reaction with I 2 and KOH
u

Q (N = C × Z) and (Z = 2for H 2SO 4 )


(a)
pH = 2and pH of HCl
St

O O
I Ph I /KOH
2 I
Ph pH = − log[10− 1 ] = 1
α CHI3
I Thus, pH of H 2SO 4 > pH of HCl.
I
O RT
(d) = 0.0591 at 25ºC is a wrong statement.
+ Ph—CH2—C—ONa F
Where, R = Gas constant = 8.314
T = Temperature = 273 + 25 = 298 K
F = Charge over one mole of electrons
= 96500C
WB JEE (Engineering) Solved Paper 2019 31
RT 8.314 × 298 • No reaction with water
Thus, = = 0.02567
F 96500 Thus, NH 4NO 3 can be used to label all three
2.303 RT beakers.
The correct value is = 0.0591
F (c) (NH 4)2CO 3 will evolve pungent smelling gas
Hence, only option (a) and (b) are correct. NH 3 with NaOH.
• Effervescence of CO 2 gas with con.H 2SO 4
78. (b,c,d) Among the given species
• No reaction with water.
(a) H 2O and F are weak field ligands and do not
cause pairing of electrons in the central metal Thus, (NH 4)2CO 3 can be used to label all three

)
atom. As CN is a strong field ligand, it causes beakers.

be
pairing of electrons in the central metal atom. Thus, options (a, c) are correct.
Now,
80. (d) O

Tu
Oxidation state of :
Me Me
(b) Fe in [Fe(H 2O)6] Cl 3 = (+) 3and electronic Et
CO2COOH Et CH2—C— OH
∆ (100ºC) ∗
configuration of Fe3 + ions is 3d 5 4s 0 O
O

ou
Fe3 + =
–CO2 ∗
H—O—C C—OH CH (Me) COOH
3d5 4s0 Me Chiral

i.e. have 5 unpaired electrons


(c) Fe in K 3 [FeF6]
(Y (b)

Et
Me
CH2COOH
∆ (100ºC)
Et
Me
CH2COOH
on
O ∗
Oxidation no. of Fe = (+) 3
–CO2
electronic configuration of Fe3 + is 3d 5 4s 0 HO—C CO2H CH2(COOH)
Chiral
pi

3+ 3d5 4s0
Fe =
(c)
m

Me Me
i.e. have 5 unpaired electrons. Et CO2H Et H
∆ (100ºC) ∗
(d) Oxidation state of Mn in K 4 [MnF6] = (+) 2and
ha

electronic configuration of Mn2 + is 3d 5 4s 0 Me


–CO2
Me
2+ O O
Mn =
Chiral
C

3d5 4s0
(d)
H H
Et CO2H Et H
dy

i.e., have 5 unpaired electrons. ∆ (100ºC) ∗


Hence, options (b) (c) and (d) are the correct –CO2
options. O Me O Me
u

79. (a, c) Achiral


St

(a) NH 4NO 3 will evolve NH 3 pungent smelling gas Thus, compound (d), capable of producing achiral
with NaOH. compound on heating at 100ºC.
• NO 2 brown gas with con.H 2SO 4
Mathematics
π /4
1. (b) We have,  µ sin x 
lim x ln x n
4. (b) Let I = ∫  λ |sin x | +
 1 + cos x
+ γ  dx

π
x → 0+ −
4
ln x π /4 π /4
= lim − n sin x
x → 0+ x = ∫ (λ|sin x | + γ) dx + µ ∫ + cos x
dx
− π /4 π1
1 −
4

)
= lim x [using L’Hospital’s rule] sin x

be
x → 0 + − nx
− n −1 Let f (x) =
1 + cos x
−1  1  sin(− x) − sin x
= lim =0 Q x − n = 0, when x = 0 ⇒ f (− x) = = = − f (x)
+
nx − n

Tu
x→ 0   1 + cos(− x) 1 + cos x

2. (b) Let I = ∫ cos x log  tan x  dx ∴ f (x) is an odd function.


 2 π /4
 sin x 

ou
= log  tan  ⋅ sin x − ∫ sin x ⋅
1 x 1
So, ∫   dx = 0
π  1 + cos x 
x
⋅ sec2 ⋅ dx
 2 tan
x 2 2 −
4
2

(Y
π /4

= sin x log  tan  − ∫ sin x


x 1
dx
∴ I= ∫ λ |sin x | + γ) dx
 2 x
2sin cos
x −
π
4
2 2
on
∴ I is independent of µ.
 x
= sin x ⋅ log  tan  − ∫
sin x
dx
 2 sin x 5. (a) We have,
1  sin2 t 
pi

= sin x ⋅ log  tan  − ∫ 1 dx


x a a
2
 2 ∫ e
lim dt − ∫ esin t dt 
x→ 0 x  
y 
m

x + y
= sin x log  tan  − x + c
x
 2 1  
a x + y
2 2
= lim  ∫ esin t dt + ∫ esin t dt 
ha

∴ f (x) = c − x x→ 0 x  
y a 
 
3. (d) Let I = ∫ cos2tan− 1 1 − x  dx 1
x + y
2
= lim ∫ esin t
1+ x
C

 dt
x→ 0 x
y
Put x = cos 2θ ⇒ dx = − 2sin 2θ dθ
x + y
 1 − cos 2θ 
dy

2
∴ I = ∫ cos2 tan− 1  (− 2sin 2θ) dθ ∫ esin t dt
 1 + cos 2θ 
= lim y
x→ 0 x
 2sin θ 
u

2
= − 2∫ cos2 tan− 1  sin 2θ dθ esin
2
( x + y)
(1 + 0) − 0 2
 2cos2 θ  = lim = e sin y
St

x→ 0 1
= − 2 ∫ cos{2tan− 1 (tanθ)} sin 2θ dθ x
6. (d) Let I = ∫ 22 ⋅ 2x dx
= − 2∫ cos 2θ sin 2θ dθ
Put t = 2x
= ∫ cos 2θ d(cos 2θ)
⇒ dt = 2x log 2 dx
cos 2θ2
2t
= + C
2
∴ I= ∫ log 2 dt
x2 x
= + C 2t 22
2 = 2
+ C= + C
(log 2) (log 2)2
Which is an equation of parabola.
1
∴ A=
(log 2)2
WB JEE (Engineering) Solved Paper 2019 33
1
 x 2015 1  1 1
= 3∫
7. (d) Let I = ∫  e| x |(x 2 + cos x) + | x|
e 
dx 0 1 + 3x
dx
−1
1
1
x 2015 1
1 = 3∫ (1 + 3x)− 1 / 2 dx
= ∫ e| x |(x 2 + cos x)
dx + ∫ e| x |
dx 0
1
−1 −1  2 1 + 3x 
= 3 
x 2015 1  3 0
Let f (x) = and g(x) =
| x|
e (x 2 + cos x) e| x | = [4 − 2]
(− x)2015

)
Again, f (− x) = =2

be
e| − x |((− x)2 + cos(− x))  x 
− x 2015 9. (c) 1 + e y  dx + 1 − x  e x / y dy = 0
= = − f (x)  y
e| x |(x 2 + cos x)  

Tu
1 1  x
and g(− x) = | − x | = | x | = g(x) ⇒ (1 + e x / y)dx = − e x / y 1 −  dy
e e  y

ou
Q f (x) is odd function and g(x) is even function. dx − e x / y (1 − x / y)
⇒ = …(i)
1 1 1
dy (1 + e x / y)
∴ ∫ f (x) dx = 0 and ∫ g(x) dx = 2 ∫ g(x) dx

(Y
−1 −1 0
This is homogeneous differential equation.
1 So, put x = vy
1
∴ I = 2∫ dx dx dv
e| x | =v+ y …(ii)
0 dy dy
on
1 1
= 2∫
1
dx = 2∫ e − x dx dv − e v (1 − v)
Therefore, v + y =
0
ex 0 dy 1 + ev
pi

= − 2 [e − x ]10 [from Eqs. (i) and (ii)]


= − 2 [e − 1 − e 0 ] dv − e v + ve v
m

⇒ y = −v
= − 2 (e − 1 − 1) dy 1 + ev
ha

( − e − 1)
= 21 − e v + ve v − v(1 + e v)
=
1 + ev

8. (c) lim 3 1 + n
+
n
+
n
+ ... = − e v + ve v − v − ve v
C

n→ ∞ n
 n+ 3 n+ 6 n+ 9
dv − (e v + v)
n  ⇒ y =
+  dy 1 + ev
dy

n + 3(n − 1) 
1 + ev dy
⇒ dv = − [variable separation]
3 n n n v + ev
= lim  + + y
u

n→ ∞ n
 n + 3× 0 n + 3×1 n + 3× 2
1 + ev dy
⇒ ∫ v + ev dv = − ∫ [by integration] …(iii)
St

n n  y
+ + K+ 
n + 3× 3 n + 3(n − 1)  Let v + e v = t ⇒(1 + e v) dv = dt
3 n − 1 n  1 + ev dt
= lim ∑  ∴ ∫ v + ev dv = ∫ = log t = log(v + e v) …(iv)
n→ ∞ n
 r = 0 n + 3r  t

  ⇒ log(v + e v) = − log y + log C


3 
n −1
1 [from Eqs. (iii) and (iv)]
= lim  ∑ 
n→ ∞ n  r
 r = 0 1 + 3    ⇒ log(v + e v) + log y = log C
  n  
⇒ y(v + e v) = C
n −1
1 1 x 
= 3 lim ∑ ⇒ y + e x / y  = C
1 + 3 
n→ ∞ n r
r=0 y 
 n
34 WB JEE (Engineering) Solved Paper 2019
 x + ye x / y  11. (b) Given hyperbola equation
⇒ y  =C
 y  x 2 y2
H: − =1
⇒ x + ye x/ y
=C a2 b2
P(4, 3)lie on H
10. (d) (x + y)2 dy = a 2, a ≠ 0 16 9
dx ⇒ − =1
a2 b2
Let x + y = t
dy dt Normal equation at P(4, 3) for H
1+ = a 2 y1 (x − x1) + b 2 x1 (y − y1) = 0

)
dx dx

be

dy dt
= −1 ⇒ 3a 2(x − 4) + 4b 2(y − 3) = 0
dx dx Normal cuts the X-axis at (16, 0)
Therefore, t 2  − 1 = a 2
dt
⇒ 3a 2(16 − 4) + 4b 2(0 − 3) = 0

Tu
 dx 
⇒ 3a 2(12) + 4b 2(− 3) = 0
2 dt
⇒ t − t = a2
2
dx ⇒ 36a 2 − 12b 2 = 0

ou
⇒ t2
dt
= a2 + t2 ⇒ 3a 2 − b 2 = 0
dx
⇒ 3a 2 = b 2
t2


a + t2
2

t2
dt = dx [variable separation]

∫ (t 2 + a 2) dt = ∫ dx [integration]
(Y ∴ Eccentricity of the hyperbola is

e=
a2 + b2
a
on
a + 3a 2
2
4a 2 2a
t2 + a2 − a2 = = = =2
⇒ ∫ (t 2 + a 2) dt = x + C′ a a a
pi

[doing a 2 adding and subtracting] 12. (b) Let V be the volume of spherical ballon of
radius r.
m

 a2 
⇒ ∫ 1 − t 2 + a 2  dt = x + C′ Then, V =
4 3
πr
3
ha

4π 
a2 log V = log 
1 dV 3
⇒  + 3log r ⇒ =0+
⇒ ∫ dt − ∫ t 2 + a 2 dt = x + C′  3 V dr r
1 dV 3 1 ∆V 3
C

dt ⇒ = ⇒ =
⇒ t − a2∫ = x + C′
t2 + a2 V dr r V ∆r r
∆V ∆r
⇒ =3
dy

1 t 
⇒ t − a2 tan− 1   = x + C ′ V r
 a  a  
∆V ∆r
x + y ⇒ × 100 = 3 × 100
(x + y) − a tan− 1 
u

⇒  V r
 a  ∆V
⇒ × 100 = 3 × 01 . = 0.3
St

= x + C ′[on putting the value of t] V


y − C′ x + y
⇒ = tan− 1  
∴ Percentage increase in volume is 0.3%.
a  a 
13. (b) Let ∆ABC be a right angled triangle at B. Let
x+ y y − C′ 
⇒ = tan  ∠A and ∠C be α and β
a  a 
A
x+ y y + C
⇒ = tan 
a  a  α
[where, C = − C ′ is an arbitrary constant] a
ar2
y + C x + y
⇒ tan  =
 a  a
β
B ar C
WB JEE (Engineering) Solved Paper 2019 35
Since, sides are in GP so sides are a, ar, ar 2 15. (c) (Im)
Now, AC 2 = AB2 + BC 2
(z)
⇒ (ar 2)2 = a 2 + (a ⋅ r)2
⇒ a 2r 4 = a 2 + a 2r 2 θ
(Re)
⇒ r − r −1 = 0
4 2
–π+θ
1+ 5
⇒ r2 = [r 2 > 0] (–z)
2

)
5+1

be
⇒ r= ∴ arg(z) − arg(− z) = θ − (− π + θ)
2
BC ar = π
∴ tanα = =

Tu
AB a 16. (b) We have,
5+1 (cosθ + i sinθ) (cos 2θ + i sin 2θ) ...
⇒ r=
2 (cos nθ + i sin nθ) = 1

ou
and tanβ =
AB
=
a 1
= ⇒ e iθ ⋅ e i( 2θ) ⋅ e i( 3θ) ⋅ ... e i( nθ) = 1
BC ar r
⇒ e iθ(1 + 2 + 3 + K + n)
=1
5 −1

(Y
1 i n( n + 1) θ
= =
5+1 2 ⇒ e 2 =1
n(n + 1)  n(n + 1) 
⇒ cos θ + i sin
2
 θ = 1 + 0i
on
 2   2 
14. (b) We have,
n(n + 1) 
1  1  ⇒ cos θ = 1
log 62 + = log 2  2x + 8  2 
pi

2x  
  n(n + 1)
⇒ θ = 2kπ
 1  2
m

1
⇒ 1 + log 32 + = log 2  2x + 8 4k
2x   ⇒ θ= π
  n(n + 1)
ha

1
1
2x + 8 1+ 17. (d) Let f (x) = ax 2 + bx + c
⇒ =2 2x
3 ⇒ 1 =a+ b+ c<0
f ()
C

1 1 Again, f (x) has imaginary zeros. So, a < 0.


⇒ 2x + 8= 3⋅ 2⋅ 22x Also, f (0) = c. Since f (x) is downward parabola. So,
dy

1 1 c < 0.
⇒ 2x + 8 = 6 ⋅ 22x Y
1
u

Let y= 22x
St

⇒ y2 + 8 = 6 y
X′ X
⇒ y2 − 6 y + 8 = 0
⇒ (y − 4) (y − 2) = 0 f (x)
⇒ y = 4, 2
1 1 Y′
⇒ 22x = 4 and 22x = 2
1 1 18. (a)
⇒ = 2 and =1 A B
2x 2x
1 1 4 2
⇒ x= ,
4 2 3 3
2 4
36 WB JEE (Engineering) Solved Paper 2019
∴ Total number of ways 1 1 1  3 0 0
= 6C4 × 6C2 + 6C3 × 6C3 + 6C2 × 6C4 = 1 1 1 −  0 3 0
   
6 × 5 6 × 5 6 × 5× 4 6 × 5× 4 1 1 1  0 0 3
= × + ×
2×1 2×1 3× 2×1 3× 2×1 − 2 1 
1
+
6×5 6×5
× =1 −2 1 
 
2×1 2×1  1 1 − 2
= 15 × 15 + 20 × 20 + 15 × 15 = [− 2(4 − 1) − 1(− 2 − 1) + 11
( + 2)]
= 225 + 400 + 225

)
= [− 2(3) − 1(− 3) + 1(3)]
= 850

be
= [− 6 + 3 + 3] = 0
19. (b) Required number of ways Now, det = 0
= 7C4 × D(3) So, matrix A − 3I 3 is non-invertible matrix.

Tu
 1 1 
= 7C3 × 3!1 − + 23. (c) Since, adj (M ′) = (adj M)′
1
− 
  1! 2! 3!   = adj (M ′) − (adj M)′

ou
Q n Cr = n Cn − r , and  [Q adj (A′) = (adj A)′]
 
 D(n) = n!1 − 1 + 1 − 1 + K   = 0, a null matrix.
  1! 2! 3!  
= 7C3 × 2 = 2(7 C3)
(Y
24. (a) Given,
 5 5x
A = 0 x

x
5x 

on
20. (d) We have,
 0 0 5 
72n + 16n − 1
∴ A2 = A ⋅ A
Put n = 1, we get
pi

72 + 16 − 1  5 5x x  5 5x x
= 0 x 5x  0 x 5x 
= 49 + 15    
m

= 64  0 0 5   0 0 5 
 25 25x + 5x 2 5x + 25x 2 + 5x 
ha

which is divisible by 64.


 
 1 1 
84 =0 x2 5x 2 + 25 
21. (b) Given,  38 + 54  0 0 25 
   
C

 
84 − r r = 25(25x 2 − 0)
 1  1
= 84 Cr  38   54  = 25(25x 2)
dy

here, Tr + 1
   
    Since, given that| A |2 = 25
 84 − r  r
Now, 25(25x 2) = 25
u

= 84
Cr  3 8  ⋅ 54
  ⇒ 25x 2 = 1
 
St

Since, Tr + 1 is rational for r = 4, 12, 20, 28, 36, 44, 1


⇒ x2 =
52, 60, 68, 76, 84 25
∴ no. of rational terms = 11 1
⇒ x=±
Here, total number of terms = 85 5
∴ Number of irrational terms = 85 − 11 1
⇒ x=
= 74 5

22. (c) Given, that A − 3I 3 25. (d) Since, product of two non-null matrix can be
a null matrix.
1 1 1 1 0 0 Therefore, may be
= 1 1 1 − 3  0 1 0 A ≠ O3, B ≠ O3.
   
1 1 1  0 0 1 
WB JEE (Engineering) Solved Paper 2019 37

26. (a) Y x8+y8=1 29. (c) Let P(A) = 1 , P(B) = 1


2 3
1 1
P(C) = , P(D) =
4 5
X′ X Now, P(A ∪ B ∪ C ∪ D)
O
= 1 − P(A ∩ B ∩ C ∩ D)
x2+y 2=1 = 1 − P(A) P(B) P(C) P(D)
= 1 − 1 −  1 −  1 −  1 − 
1 1 1 1
Y′
 2  3  4  5

)
be
= 1 −         = 1 − =
From the graph, it is clear that there is common 1 2 3 4 1 4
region which satisfies x 2 + y 2 = 1 and x 8 + y 8 < 1  2  3  4   5 5 5
Q P∩T = P

Tu
30. (d) Given that,
27. (d) We have, σ(X) = 26
. = var(x)
x2 σ(1 − 4 x) = var(1 − 4 x)
f (x) = x −
2

ou
1 + x2
= 16var(x)
(− x)2
∴ f (− x) = (− x)2 − =4× var(x) = 4 × 26
. = 10.4
1 + (− x)2

= x2 −
x2
1 + x2
= f (x)
(Y
31. (a) Given that, esin x − e − sin x − 4 = 0
Let esin x = t
on
∴ f (− x) = f (x).
∴ t − t− 1 − 4 = 0
So, f (x) is many one
x2 1
Again, f (x) = x 2 − ⇒ t− −4=0
pi

t
1 + x2
⇒ t 2 − 1 − 4t = 0
x2 + x4 − x2 x4
m

= = 4± 2 5
1+ x 2
1 + x2 t= = 2± 5
2
∴ Range of f (x) will be [0, ∞)
ha

Now, esin x = 2 ± 5
But, co-domain of f (x) = R
∴ Range ≠ co-domain. So, f (x) is onto. ⇒ e sin x
= 2+ 5 and esin x = 2 − 5
C

28. (d) We observe the following properties: ⇒ e sin x


= 2+ 5 [e x > 0]
Reflexivity : Let a be an arbitrary element of R, Also, − 1 ≤ sin x ≤ 1
dy

Then, a ∈ R 1
∴ ≤ esin x ≤ e
⇒ 1 + a ⋅ a = 1 + a2 > 0 [Qa 2 > 0 for all a ∈ R] e
⇒ (a , a) ∈ R1 ∴ esin x ≠ 2 + 5
u

[By def. of R1 ]
Thus, (a , a) ∈ R1 for all a ∈ R. So, R1 is reflexive on R. Hence, there is no value of x.
St

Symmetry : Let (a , b) ∈ R. 32. (d) Given, that angles of a triangles are 2x, 3x and
Then, (a , b) ∈ R1 7x.
⇒ 1 + ab > 0 [Qab = b for all] Since, 2x + 3x + 7 x = 180
⇒ 1 + ba > 0 ⇒ (b , a) ∈ R1 ⇒ 12x = 180º ⇒ x = 15º
Thus, (a , b) ∈ R1 ≠ (b , a) ∈ R, for all a , b ∈ R So, angles are 30º, 45º and 105º
So, R1 is symmetric on R. a
Now, = 2R
Transitivity : we observe that 1,  ∈ R1 and
1 sin 30º
 2 a
⇒ = 2 × 10 [Q R = 10 cm]
 1 , − 1 ∈ R but (1, − 1) ∉R because 1
  1 1
2  2
1 + 1 × (− 1) = 0 >/ 0 ⇒ 2a = 20 ⇒ a = 10 cm
So, R1 is not transitive on R.
38 WB JEE (Engineering) Solved Paper 2019

33. (b) Let the equation of line be x + y = 1 − 2= 3(3) + c


a b
− 2= 3 3 + c
Since, the line passing through a fixed point(x1 , y1).
c = − 2− 3 3

B Hence, required equation


(a , b )
(0, b)
P ∴ y = mx + c
y= 3x − 2 − 3 3
y− 3x + 2 + 3 3 = 0

)
be
O (a, 0)
y − x 3 + 2+ 3 3 = 0
A
x1 y 35. (a) Let (α , β) be the given point, let Q(x , y) be the
∴ + 1 =1

Tu
a b foot of the perpendicular, and let O be the origin.
The line can have any direction
Since, OAPB is a rectangle, therefore the coordinate
of P will be (a , b). ∠PQO = 90º

ou
Hence, locus of P is Point Q lies on the circle having diameter OP.
x1 y The locus of point Q.
+ 1 =1
x y (x − 0) (x − α) + (y − 0) (y − β) = 0

34. (b) Given line,


3x + y = 1
(Y ⇒

x 2 − xα + y 2 − yβ = 0
x 2 + y 2 − αx − βy = 0
on
⇒ y=− 3x + c 36. (b) Let coordinate of the point be (α , − α)
We know that, m = − 3 [Q y = mx + c] Since, (α , − α) lie on 2ax + 4ay + c = 0
and 7bx + 3by − d = 0
pi

m1 − m2 ∴ 2aα − 4aα + c = 0
tanθ =
1 + m1 m2 ⇒ − 2aα + c = 0
m

m1 − (− 3) ⇒ α=
c
…(i)
tan 60° = [Qθ = 60º]
ha

1 + m1 (− 3) 2a
Also, 7bα − 3bα − d = 0
m1 + 3
3= m1 [tan 60º = 3] ⇒ 4bα − d = 0
1− 3
C

d
⇒ α= …(ii)
m1 + 3 4b
± 3=
1 − 3m1 From Eqs. (i) and (ii),
dy

c d
=
taking + sign taking − sign 2a 4b
u

m1 + 3 m1 + 3 ⇒ 2ad = 4bc
+ 3= − 3= ad 4
St

1 − 3m1 1 − 3m1 ⇒ =
bc 2
3(1 − 3m1 ) = m1 + 3 ⇒− 3(1 − 3m1 ) ad 2
⇒ =
= m1 + 3 bc 1
⇒ 3 − 3m1 = m1 + 3 ⇒− 3 − 3m1 = m1 + 3 ⇒ ad : bc = 2 : 1

⇒ 3m1 − m1 = 2 3 37. (a) In a circle, AB is a diameter where the


coordinate of A is (p , q) and let the coordinate of
⇒ 4m1 = 0 ⇒ 2 m1 = 2 3 B(x1 , y1).
⇒ m1 = 0 ⇒ m1 = 3 Equation of circle in diameter form is
(x − p) (x − x1) + (y − q) (y − y1) = 0
∴ given point (3, − 2)
∴ y = mx + c
WB JEE (Engineering) Solved Paper 2019 39
⇒ x 2 − (p + x1) x + px1 + y 2 − (y1 + q) y + qy1 = 0 39. (c) Given, equation of hyperbola is
⇒ x 2 − (p + x1) x + y 2 − (y1 + q) y + px1 + qy1 = 0 x2 y2
− = 1,
Since, this circle touches X-axis cos α sin2 α
2

∴ y=0 Here, a 2 = cos2 α and b 2 = sin2 α


⇒ x 2 − (p + x1) x + px1 + qy1 = 0 [i.e. comparing with standard
Also, the discriminant of above equation will be x 2 y2
equation 2 − 2 = 1]
equal to zero because circle touches X-axis. a b
∴ (p + x1)2 = 4(px1 + qy1)

)
We know that, foci = (± ae , 0),

be
⇒ p 2 + x12 + 2px1 = 4px1 + 4qy1 where, ae = a2 + b2 = cos2 α + sin2 α = 1
⇒ x12 − 2px1 + p = 4qy1
2
⇒ foci (± 1, 0), where vertices are (± cosα , 0)

Tu
⇒ (x1 − p) = 4qy1
2 1
Eccentricity, ae = 1 or e = .
cosα
Therefore, the locus of point B is
Hence, foci remains constant with change in α.
(x − p)2 = 4qy

ou
  40. (c) Equation of the ellipse is
38. (c) I =  ax1 + bx 2 + cx 3 , ay1 + by2 + cy3  x2 y2
 a+ b+ c a+ b+ c  + 2 =1

(Y
2
a b
P (0, 0) B (0, b)
on
c b T S
pi

I Foci are S(ae , 0), T(− ae , 0).


B(0, b) is the end of the minor axis.
m

Q R
(1, 0) a (1/2, Ö3) STB is an equilateral triangle.
2
SB = ST
ha

1 3 ⇒ SB2 = ST 2
a=+ = 1, b = 1, c = 1
4 4
⇒ a 2e 2 + b 2 = 4a 2e 2
C

Centre of the circle.


⇒ b 2 = 3a 2e 2
1 × 0 + 1 × 1 + 1 × 1 
 3 / 2
= 2 ,1 × 0 + 1 × 0 + 1 × 
⇒ a 2(1 − e 2) = 3a 2e 2
dy

 1+1+1 1+1+1  ⇒ 1 − e 2 = 3e 2
 
⇒ 4e 2 = 1
 3
u

1
0+1+ 0+ 0+  1
= 2, 2  ⇒ e2 =
St

 3 3  4
  1
  ⇒ e=
2
3 3
  Eccentricity of the ellipse, e =
1
=  2, 2  2
3 3 
 
  41. (a) Given equation is
3 1 3 1 3x 2 − 3y 2 − 18 x + 12y + 2 = 0
= × , × 
2 3 2 3 It can be written as
(x − 3)2 (y − 2)2
=  ,
1 1  − =1
 2 2
 2 2 3  13   13 
   
 3  3
40 WB JEE (Engineering) Solved Paper 2019
13 c
here, a = b = =
3 (2) (− 4) − (− 2) (3)
b2 a b c
∴ e = 1+ ⇒ = =
a2 12 + 16 − 8 − 8 − 8 + 6
a b c
= 1+1 [Q a = b] ⇒ = =
28 − 16 − 2
= 2
a b c
∴ Equation of the directrix are ⇒ = =
14 − 8 − 1

)
13
x = 3± Hence, direction ratios are (14, − 8, − 1).

be
6
44. (c) The given points are A(1, 2, 3) and B(3, 4, 5)
42. (c) Given equation of ellipse is

Tu
The direction ratios of line segment AB is given by
3x 2 + 4 y 2 = 48
(x 2 − x1), (y2 − y1), (z2 − z1)
3x 2 4 y 2 x2 y2 (3 − 1), (4 − 2), (5 − 3) = (2, 2, 2) = (1, 1, 1)
⇒ + =1 ⇒ + =1 i.e.

ou
48 48 16 12 Since, the plane bisects AB at right angles, AB is the
Here, a = 4 and b = 2 3 normal to the plane. Which is n
b2 Therefore, n = $i + $j + k$

(Y
∴ e = 1−
a2 Let c be the mid-point of AB
12 4 1 1  x1 + x 2 , y1 + y2 , z1 + z2 
= 1− = = =  
16 16 4 2  2 2 2 
on
1 + 3 2 + 4 3 + 5
∴ Coordinates of P are = (2, 3) = c  , , 
 2 2 2 
  b2  
Q(4 cosθ, 2 3sinθ) = (2, 3) Q p =  ae ,   = c(2, 3, 4)
pi

  a 
Let this vector be a = 2$i + 3$j + 4k$
m

By comparing, we get
Hence, the required equation of vector
4 cosθ = 2 and 2 3sinθ = 3
{r − (2$i + 3$j + 4k$)} ($i + $j + k$) = 0
ha

1 3
⇒ cosθ =
and sinθ = We know that, r = x$i + y$j + zk$
2 2
π π ⇒ {(x$i + y$j + zk$) − (2$i + 3$j + 4k$)} ($i + $j + k$) = 0
⇒ cosθ = cos and sinθ = sin
C

3 3
⇒ (x$i + y$j + zk$) ⋅ ($i + $j + k$)
π π
⇒ θ = and θ =
= (2$i + 3$j + 4k$) ⋅ ($i + $j + k$)
dy

3 3
π ⇒ x + y + z = 2+ 3+ 4
∴ eccentric angle of P is .
3 ⇒x + y+ z= 9
u

43. (b) Direction ratios of line joining the points 45. (a) It is clear that, the limit of the interior angle
St

(1, 2, − 3) and (− 1, − 2, 1) is (− 1 − 1, − 2 − 2, 1 + 3) of a regular polygon of n sides as n → ∞ is π or


i.e. (− 2, − 4, 4) 180º.
Let the direction ratios of the normal to the plane is 46. (a) Given,
(a , b , c), then
h{ f (x)} = x
2a + 3b + 4c = 0 …(i)
differentiating w.r.t.x, we get
and − 2a − 4b + 4c = 0 …(ii)
h′ { f (x)}. f ′(x) = 1
By Eqs. (i) and (ii), we get 1
⇒ f ′(x) =
h′ { f (x)}
1
a b ⇒ f ′(x) =
= 1
3(4) − (4) (− 4) (− 2) (4) − (2) (4)
1 + log{ f (x)}
WB JEE (Engineering) Solved Paper 2019 41
Q h′(x) = 1  f (3) − f ()
1
⇒ = f ′(c)
 1 + log x  2
  f (3) − f ()
1
(given)  ⇒ = [ f | c |2 + 4]
2
⇒ f ′(x) = 1 + log( f (x))
(Q f ′(c) = | f (c) |2 + 4)
47. (c) We have, ⇒ f (3) − f ()
1 = 2⋅ | f (c) |2 + 8
f (x) = cos x 2
⇒ f (3) − f ()
1 ≥8
Let T be the period of f (x). Then It is clear from the given options that (B) and (C) the

)
f (x + T) = f (x) correct.

be
⇒ cos(x + T)2 = cos x 2
51. (c) Let f (x) = x 2 + 2x + 3
But there is no value of T for which
− D − (4 − 12) 8
a = f (x)min = = = =2

Tu
cos(x + T)2 = cos x 2
4a 4 4
∴ f (x) is not periodic 1 − cosθ 1 − 1 + 2sin2 θ/2
and b = lim = lim
θ→ 0 θ2 θ→ 0 θ2

ou
48. (c) Let L = lim (e x + x)1 / x
x → 0+
2sin θ/2
2
1  sin2 θ/2 
= lim = lim ⋅  2 
log(e x + x) θ → 0 (θ/2) ⋅ 4
 (θ/2) 
2 θ→ 0 2
⇒ log L = lim

(Y
+
x→ 0 x
1 sin2 θ/2
1 = ⋅ lim
⋅ ex + 1 2 θ → 0 (θ/2)2
(e x + x)
⇒ log L = lim 1 1
on
x → 0+ 1 = ⋅1 =
2 2
[using L’ Hospital rule]
1
ex + 1 b=
pi

⇒ log L = lim x 2
x→ 0 e + x
+
n
∑a ⋅ bn − r
r
Now,
m

⇒ log L = 2
r=0
⇒ L = e2 n n−r
r 1 
ha

49. (a) Let g(x) = e −x


f (x)
= ∑ (2)  
 2
r=0
∴ g′(x) = e − x f ′(x) − e − x f (x) n
∑2 ⋅2
( r − n)
=
C

r
−x
=e ( f ′(x) − f (x)) r=0
Since, f ′(x) > f (x), so f ′(x) − f (x) > 0 n
∑2
2r − n
dy

=
∴ e − x ( f ′(x) − f (x)) > 0
r=0
⇒ g′(x) > 0 n
= 2− n ∑2
2r
u

∴ g(x) is an increasing function.


r=0
Now, g(x) > g(0) ∀ x > 0
St

⇒ e − x f (x) > e 0 f (0) = 2− n [1 + 22 + 24 + 26 + K + 22n ]

⇒ e − x f (x) > 0 [Q f (0) = 0] 1 ⋅ (22)n + 1 − 1   a(r n − 1) 


= 2− n   Q Sn = 
 2 −1
2
  r −1 
⇒ f (x) > 0, ∀x > 0
 4n + 1 − 1 
50. (b, c) Given that f : [1, 3] → R be a continuous and = 2− n  
 3 
differentiable in (1, 3).
and f ′(x) = | f (x) |2 + 4 4n + 1 − 1
=
By applying LMVT, there exist at least one point 3⋅ 2n
c ∈(1, 3) such that 52. (a) Let a = 4, b = 1 and n = 2
f (3) − f ()
1  f (b) − f (a) 
= f ′(c) Q f ′(c) = ∴ I(n) = a1 / n − b1 / n
3−1  b − a 
42 WB JEE (Engineering) Solved Paper 2019
= 41 / 2 − 11 / 2 = 2 − 1 = 1
P3 (5/4, 1/2)
and J(n) = (a − b)1 / n
P4 (5/4, 3/8)
= (4 − 1)1 / 2 = 3 P2 (1, 1/2)
∴ J(n) > I(n) (P5 (21/16, 3/8))
Similarly, we can prove for other values of a , b , n. O P1 (1, 0)

53. (d) Given, 5 5 21


Here, x1 = 1, x 2 = 1, x 3 = , x 4 = and x 5 =
|α$ |= |β$ | = | γ$ | = 1 4 4 16

)
1 1 3 3

be
1 y1 = 0, y2 = , y3 = , y4 = and y5 =
and α$ × (β$ × γ$) = (β$ + γ$)
2 2 2 8 8
1 $ $ 1 1 1 1 4
$ $
Now, α × (β × γ) = (β + γ)
$ ∴ xn = 1 + + 2 + K ∞ = = =

Tu
4 4 1 3 3
2 1−
1 1 4 4
$ $
⇒ (α$ ⋅ γ$)β − (α$ ⋅ β) γ$ = β$ + γ$
1 1
2 2 and yn = 0 + − + K ∞

ou
On comparing we get, 2 8
1 1 1 1
− (α$ ⋅ β$) = 2
2 = 2 = 2 = 2=

(Y
1  1 
1 −  −  1 + 
1 5 5
$
⇒ |α ||β | cosθ = −
$  4  4 4
2
∴ (α , β) =  ,
1 4 2
⇒ cosθ = − [Q|α$ | = |β$ | = 1] 
3 5
on
2

⇒ θ = 120º or 56. (a) | z | + | z − 1 | a is complex number
3
pi

| z | + | − (1 − z) | |a|= |− a|
54. (d) The position vectors of the points A, B, C and | z | + |1 − z | {Q| a | = | − a |}
m

D are 3$i − 2$j − k$ , 2$i − 3$j + 2k$ , 5$i − $j + 2k$ and |z| + |1 − z|≥ |z + 1 − z |
4$i − $j − λk$ respectively. {Q|z1| + |z2|≥ |z1 + z2|}
ha

BA = (3$i − 2$j − k$) − (2$i − 3$j + 2k$) = $i + $j − 3k$ | z | + |1 − z |≥ |1 |


| z | + |1 − z |≥ 1
CA = (3$i − 2$j − k$) − (5$i − $j + 2k$) = − 2$i − $j − 3k$
| z | + |−(z − 1) |≥ 1
C

DA = (3$i − 2$j − k$) − (4$i − $j + λk$) | z | + | z − 1|≥ 1


= − $i − $j − (1 + λ)k$ Minimum value of| z | + | z − 1 | = 1
dy

These are coplanar, so 57. (c) Given, system of equations


1 1 −3 λx + y + 3z = 0
u

− 2 −1 −3 =0 2x + µy − z = 0
− 1 − 1 − (1 + λ) 5x + 7 y + z = 0
St

−1 −1 3 System has infinitely many solutions in R, if


λ 1 3
= 2 1 3 =0
2 µ −1 = 0
1 1 1+ λ
5 7 1
− 11
[ + λ − 3] + 1[2 + 2λ − 3] + 3[2 − 1] = 0
⇒ λ(µ + 7) − 1(2 + 5) + 314
( − 5 µ) = 0
⇒ − 1[λ − 2] + 1[2λ − 1] + 3 = 0
⇒ λ(µ + 7) − 7 + 3 (14 − 5 µ) = 0
⇒ − λ + 2 + 2λ − 1 + 3 = 0
⇒ λµ + 7λ − 7 + 42 − 15 µ = 0
⇒ λ+ 4=0
⇒ λ=−4

55. (b) According to the given information in ⇒ λµ + 7λ − 15 µ + 35 = 0


question, we can draw the situation of particle at By checking the options, we get option (c)
different stages as following (λ = 1, µ = 3) satisfies the given equation.
WB JEE (Engineering) Solved Paper 2019 43
2
58. (c) Let A = {1, 2, 3}  b2 
e2 =  1 + 2  > 2
and B = {2, 3, 4}  a 

∴ e> 2
1 3
62. (a) Given, lim + x  q 
2 4 x→ 0 p  x 
3 x  q  q 
= lim  −  
x → 0+ p  x  x

)
be
here, f − 1 (3) = 1 =
[q] x  q 
−  
−1 −1 p p x
f (4) = 2 and f (2) = 3
[q] [q]

Tu
It is clear that option (C) f − 1 (A − B) = f − 1 (A) = − 0=
p p
− f − 1 (B) is correct.
63. (b) Given,

ou
59. (b) We have, f (x) = x 4 − 4 x 3 + 4 x 2 + c
gof : S → U is an onto function.
Q 1 =1 + c
f ()
Let Z be any arbitrary element such that Z ∈ U.

(Y
and f (2) = 24 − 4(2)3 + 4(2)2 + c
Now, gof : S → U is onto
⇒ gof (x) = Z, for x ∈ S = 16 − 32 + 16 + c
⇒ g( f (x)) = Z =c
on
⇒ g(y) = Z, where y = f (x) ∈ T for all Z ∈ U Now, f ()1 ⋅ f (2) = c(1 + c)
∴ For all Z ∈ U, there exists y = f (x) ∈ T such that 1 ⋅ f (2) < 0
Let f ()
g(y) = Z ⇒ c(1 + c) < 0 ⇒ c ∈ (− 1, 0)
pi

∴ g : T → U is an onto function. ∴ For c ∈ (− 1, 0), f (x) has exactly one zero is (1, 2).
m

60. (a) Since, Q is the point for which PQ is bisected 64. (a)
Y
perpendicularly by the initial line (X-axis).
ha

Therefore, Q will be the image of P in X-axis x2 – 1 cos x


∴ Coordinates of Q is (2, π / 4)
C

Y X′ O X
Q (2, π/4)
dy

Y′
X′ X It is clear from the graph that it intersects at exactly
u

two points.
St

65. (c) We have,


Y′
P
) 2, –4π ) dx
= 1and y =
10
dt x
10
Now, y =
61. (b) Let the length of conjugate axis = b x
and the length of transvers axis = a On differentiating both the sides w.r.t. t, we get
Given that, b > a dy − 10 dx
⇒ = 2 ×
Now, b2 > a2 dt x dt
− 10
b2 = ×1
∴ Eccentricity (e) = 1 + (5)2
a2
− 10 − 2  x = 10, dx = 1
= =
25 5  dt 
44 WB JEE (Engineering) Solved Paper 2019
2  f 
2
∴ y decreases at the rate of unit per second.
5 ⇒ t 2 = m2   …(i)
h− f
66. (b,d) We have, 1
1 Again, n + x = f (t + m)2
2
In = ∫x
n
tan− 1 x dx
1 2 1
0 ⇒ h + ht = f (t + m)2 …(ii)
2 2
 n + 1 1 n+1
x 1 x 1
=  tan− 1 x ⋅  − ∫0 n + 1 ⋅ 1 + dx From Eqs. (i) and (ii)
 n + 1 0 x2 m 2(h − f)
=

)
π 1 x2 ⋅ xn − 1 n mfh

be
1
= − ∫
4(n + 1) n + 1 0 1 + x 2
dx
m2 fh
⇒ (h − f )n =
π 1 1 (1 + x 2) x n − 1 − x n − 1 2

Tu
= − dx
4(n + 1) n + 1 0 1 + x2 68. (b) We have,
π 1 1 1 1 x n −1 y = x + 1 and y = cos x
x n − 1 dx +
4(n + 1) n + 1 ∫0 ∫0 1 +
= − dx

ou
n+1 x 2

n 1
π 1 x 1 (0, 1)
= −   +

(Y
4(n + 1) n + 1  n  0 n + 1
[x n − 1 ⋅ tan− 1 x]1 − 1 tan− 1 x ⋅ (n − 1) x n − 2 dx (π/2, 0)
 0 ∫0 (–1, 0)
on
π 1 π n −1
= − + −
4(n + 1) n(n + 1) 4(n + 1) n + 1
1 n−2 π /2
∫0 x tan− 1 x dx
pi

0
∴ Require area = ∫− 1 (x + 1) dx + ∫0 cos x dx
π 1 n −1
⇒ In = − − 0
m

In − 2  x2 
2(n + 1) n(n + 1) n + 2 = + x  + [sin x]π0 / 2
 2 − 1
n −1 π 1
ha

⇒ In + In − 2 = −
 
n+ 2 2(n + 1) n(n + 1) = (0) −  − 1 + [1 − 0]
1
  2  
Put n = n + 2, we get
C

n+1 π 1 3
I n+ 2 + In = −
1 = + 1 = sq unit.
n+ 3 2(n + 3) (n + 2) (n + 3) 2 2
π 1 69. (b) We have,
dy

⇒ (n + 3) I n + 2 + (n + 1) I n = −
2 n+ 2 x1 , x 2 be the roots of equation x 2 − 3x + a = 0
π 1 ∴ x1 + x 2 = 3 and x1 x 2 = a
∴ a n = n + 3, bn = n + 1, c n = −
u

2 n+ 2 Also, x 3, x 4 be the roots of equation


St

∴ a n and bn are in AP. x 2 − 12x + b = 0


∴ x 3 + x 4 = 12 and x 3 x 4 = b
67. (c) Let B travels x units, v = u + at
Again, x1 , x 2, x 3, x 4 are in GP
According to problem, ht = f (t + m)
∴ x1 = A, x 2 = AR, x 3 = AR2, x 4 = AR3
⇒ ht = ft + fm
Now, x1 + x 2 = 3
⇒ ht − ft = fm
⇒ A(1 + R) = 3 …(i)
⇒ t(h − f) = fm
h− f m and x 3 + x 4 = 12
⇒ = ⇒ AR2(1 + R) = 12 …(ii)
f t
 f  From Eqs. (i) and (ii), we have
⇒ t = m  R2 = 4 ⇒ R = ± 2
h− f
When, R = 2, A = 1 and when, R = − 2, A = − 3
WB JEE (Engineering) Solved Paper 2019 45
∴ Numbers are either 1, 2, 4, 8 or − 3, 6, − 12, 24
But x1 < x 2 < x 3 < x 4 . 3
So, x1 = 1, x 2 = 2 , x 3 = 4, x 4 = 8 2
x –y=1
∴ ab = x1 x 2 x 3 x 4 1
1/2
= 1 × 2 × 4 × 8 = 64 1 2 3 4 5 6 7 8

70. (a) Let Z = 1 − i cosθ -1 B (6 –1)


1 + 2i cosθ -2
1 + i cosθ
∴ z= -3

)
1 − 2i cosθ

be
-4
Since, z is a real number, then z − z = 0 -5
1 − i cosθ 1 + i cosθ
⇒ = -6

Tu
1 + 2i cosθ 1 − 2i cosθ m
-7
A (2, –7) C
⇒ (1 − i cosθ) (1 − 2i cosθ) = (1 + i cosθ) (1 + 2i cosθ) -8
⇒ 1 − 2i cosθ − i cosθ − 2cos2 θ

ou
Let the slope of AC be m
= 1 + 2i cosθ + i cosθ − 2cos2 θ then, AB = AC
⇒ 6 i cosθ = 0 1 1
m+ − −1


cosθ = 0
π
θ = (2n + 1) , n ∈ I
2
(Y ∴
1−
m
4
4 =
1−
4
1
4
on
− 23
71. (b) We have, ⇒ m= ,1
7
 3 0 3 23
When m = − , then equation of line
A =  0 3 0
pi

  7
 3 0 3 − 23
y+ 7 = ( x − 2)
m

Again, we have 7
| A − λI | = 0 ⇒ 7 y + 49 = − 23x + 46
ha

3− λ 0 3 ⇒ 23x + 7 y + 3 = 0
⇒ 0 3− λ 0 =0 When m = 1, then equation of line
y + 7 = ( x − 2)
C

3 0 3− λ
⇒ x− y−9 =0
⇒ (3 − λ) [(3 − λ) − 0] − 0 + 3[0 − 3(3 − λ)] = 0
2
dy

73. (a, c) Given equation of hyperbola can be write as


⇒ (3 − λ)3 − 9(3 − λ) = 0
⇒ (3 − λ) [(3 − λ)2 − 9] = 0 x2 y2
− =1
u

⇒ (3 − λ) (9 + λ − 6λ − 9) = 0
2
1 ( 5)2
St

⇒ (3 − λ) (λ2 − 6 λ) = 0 here, a = 1 and b = 5


⇒ (3 − λ) λ(λ − 6) = 0 Since, y = mx ± a 2m2 − b 2
⇒ λ = 3, 0, 6
y = mx ± m2 − 5
72. (a, b) Given equations of lines are
⇒ (8 − 2m)2 = ( m2 − 5)2
x− y=7 …(i)
and x + 4y = 2 …(ii) ⇒ (8 − 2m)2 = m2 − 5
By solving Eqs. (i) and (ii), we get the point B(6, − 1) ⇒ 64 + 4m2 − 32m = m2 − 5
⇒ 3m2 − 32m + 69 = 0
22
⇒ m= ,3 [by solving equation]
3
46 WB JEE (Engineering) Solved Paper 2019
23 From option (c)
When m = , then equation of tangent
3 We have, g(a) = g(b) = 0
23 ⇒ g′(a) ⋅ g′(b) < 0
(y − 8) = (x − 2)
3 Again, let m(x) = g′(x) + kg(x)
⇒ 3y − 24 = 23x − 46 ⇒ m(a) = g′(a) + kg(a) = g′(a)
⇒ 23x − 3y − 46 + 24 = 0 ⇒ m(b) = g′(b) + kg(b) = g′(b)
⇒ 23x − 3y − 22 = 0 ∴ m(a) ⋅ m(b) = g′(a) ⋅ g′(b) < 0
When m = 3, then equation of tangent ∴ m(x) = 0 has root between (a , b).

)
(y − 8) = 3(x − 2) So, option (a) and (c) are correct.

be
⇒ y − 8 = 3x − 6 3
⇒ 3x − y − 6 + 8 = 0 75. (d) Given, f (x) = x − sin πx + 3
4
⇒ 3x − y + 2 = 0

Tu
differentiating w.r.t. x, we get
Hence, option (a) and (c) are correct.
3x 2
f ′(x) = − π cos(πx)
74. (a, c) From option (a) 4

ou
We have, at x = − 2, f(− 2) = 1
f (a) = f (b) = 0 and at x = 2, f(2) = 5
⇒ f ′(a) ⋅ f ′(b) < 0
Again, let h(x) = f ′(x) + f (x) g′(x)
⇒ h(a) = f ′(a) + f (a) g′(a) = f ′(a)
and h(b) = f ′(b) + f (b) g′(b) = f ′(b)
(Y
on
–2 2
∴ h(a) ⋅ h(b) = f ′(a) ⋅ f ′(b) < 0
∴ h(x) = 0 has root between (a, b)
pi
m
ha
C
u dy
St
WB JEE
Engineering Entrance Exam

Solved Paper 2018

)
be
Tu
Physics

ou
Category-I (Q. Nos. 1 to 30) X -axis and perpendicular to the xy-plane.

(Y
The reflected ray passes through the point
Only one answer is correct. Correct answer will
fetch full marks 1. Incorrect answer or any (3, 3). What is the path length of the ray
combination of more than one answer will fetch from (0, 1) to (3, 3)?
on
1 (a) 5 (b) 13
− marks. No answer will fetch 0 marks. (c) 2 3 (d) 1 + 2 3
4
pi

4. Two identical
1. Four resistors, 100 Ω, 200 Ω, 300 Ω and 400 Ω
equiconvex lenses,
are connected to form four sides of a square.
m

each of focal length f


The resistors can be connected in any order.
are placed side by side
What is the maximum possible equivalent
ha

in contact with each


resistance across the diagonal of the square?
other with a layer of
(a) 210Ω (b) 240Ω
water in between
C

(c) 300Ω (d) 250Ω


them as shown in the figure. If refractive
2. What will be current through the 200 Ω index of the material of the lenses is greater
dy

resistor in the given circuit, a long time after than that of water, how the combined focal
the switch K is made on? length F is related to f ?
f
(a) F > f <F<f
u

(b)
100Ω 1µF 400Ω 2
St

f
(c) F < (d) F = f
2
200Ω 300Ω 2µF
5. There is a small air bubble at the centre of a
solid glass sphere of radius r and refractive
6V K index µ. What will be the apparent distance
of the bubble from the centre of the sphere,
(a) Zero (b) 100 mA when viewed from outside?
(c) 10 mA (d) 1 mA r
(a) r (b)
µ
3. A point source is placed at coordinates (0, 1)  1
(c) r  1 −  (d) Zero
in x y-plane. A ray of light from the source is  µ
reflected on a plane mirror placed along the
2 WB JEE (Engineering) Solved Paper 2018

6. If Young’s double slit experiment is done 12. In case of a simple harmonic motion, if the
with white light, which of the following velocity is plotted along the X -axis and the
statements will be true? displacement (from the equilibrium position)
(a) All the bright fringes will be coloured. is plotted along the Y -axis, the resultant
(b) All the bright fringes will be white. curve happens to be an ellipse with the ratio:
(c) The central fringe will be white. major axis (along X )
(d) No stable interference pattern will be visible. = 20 π
minor axis (along Y)
7. How the linear velocity v of an electron in the What is the frequency of the simple harmonic

)
motion?

be
Bohr orbit is related to its quantum number n?
1 1 1
(a) v ∝ (b) v ∝ (a) 100 Hz (b) 20 Hz (c) 10 Hz (d) Hz
n n 2 10

Tu
1
(c) v ∝ (d) v ∝ n 13. A block of mass m2 is placed on a horizontal
n
table and another block of mass m1 is placed
8. If the half-life of a radioactive nucleus is on top of it. An increasing horizontal force

ou
3 days, nearly what fraction of the initial F = αt is exerted on the upper block but the
number of nuclei will decay on the third day? lower block never moves as a result. If the
coefficient of friction between the blocks is µ 1
(Given, 3 0.25 ≈ 0 .63)
(a) 0.63 (b) 0.5 (c) 0.37 (d) 0.13
(Y and that between the lower block and the
table is µ 2 , then what is the maximum
possible value of µ 1 / µ 2 ?
on
9. An electron accelerated through a potential
m2 m2 m1 m1
of 10000 V from rest has a de-Broglie wave (a) (b) 1 + (c) (d) 1 +
length λ. What should be the accelerating m1 m1 m2 m2
pi

potential, so that the wavelength is doubled?


14. In a triangle ABC, the sides AB and AC are
(a) 20000 V (b) 40000 V (c) 5000 V (d) 2500 V
represented by the vectors 3 $i + $j + k$ and
m

10. In the circuit shown, inputs A and B are in $i + 2 $j + k$ , respectively. Calculate the angle
states 1 and 0 respectively. What is the only
ha

∠ ABC.
possible stable state of the outputs X and Y ?
5 6
(a) cos −1 (b) cos −1
A 11 11
C

(1) X
 5  5
(c)  90° − cos −1  (d)  180° − cos −1 
 11   11 
dy

(0)
Y
B 15. The velocity (v) of a particle (under a force F)
(a) X = 1, Y = 1 (b) X = 1, Y = 0 depends on its distance (x) from the origin
u

(c) X = 0, Y = 1 (d) X = 0, Y = 0 1
(with x > 0) v ∝ . Find how the magnitude
St

11. What will be the current flowing through the x


6 kΩ resistor in the circuit shown, where the of the force (F) on the particle depends on x?
breakdown voltage of the Zener is 6 V? 1 1 1
(a) F ∝ (b) F ∝ (c) F ∝ 2
(d) F ∝ x
x3 / 2 x x
6 kΩ
16. The ratio of accelerations due to gravity g1 : g2
10V on the surfaces of two planets is 5 : 2 and the
4 kΩ
ratio of their respective average densities
ρ1 : ρ2 is 2 : 1. What is the ratio of respective
escape velocities v1 : v2 from the surface of the
2 3
(a) mA (b) 1mA (c) 10 mA (d) mA planets?
3 2
(a) 5 : 2 (b) 5 : 2 (c) 5 : 2 2 (d) 25 : 4
WB JEE (Engineering) Solved Paper 2018 3

17. A spherical liquid drop is placed on a 22. For an ideal gas with initial pressure and
horizontal plane. A small disturbance causes volume p i and Vi respectively, a reversible
the volume of the drop to oscillate. The time isothermal expansion happens, when its
period of oscillation (T) of the liquid drop volume becomes V0 . Then, it is compressed to
depends on radius (r) of the drop, density (ρ) its original volume Vi by a reversible
and surface tension (S) of the liquid. Which adiabatic process. If the final pressure is p f ,
among the following will be a possible then which of the following statement(s)
expression for T (where, k is a dimensionless is/are true?

)
constant)? (a) pf = pi (b) pf > pi

be
ρr ρ2 r ρr 3 ρr 3 p p
(a) k (b) k (c) k (d) k (c) pf < pi (d) f = i
S S S S2 V0 Vi

Tu
18. The stress along the length of a rod (with 23. A point charge − q is carried from a point A to
rectangular cross-section) is 1% of the another point B on the axis of a charged ring
Young’s modulus of its material. What is the of radius r carrying a charge + q. If the point

ou
approximate percentage of change of its 4
volume? (Poisson’s ratio of the material of A is at a distance r from the centre of the
3
the rod is 0.3.)

(Y
3
(a) 3% (b) 1% (c) 0.7% (d) 0.4% ring and the point B is r from the centre but
4
19. What will be the approximate terminal velocity on the opposite side, what is the net work
of a rain drop of diameter 1.8 × 10 −3 m, when
on
that need to be done for this?
density of rain water ≈ 10 3 kgm −3 and the 7 q2 1 q2
(a) − ⋅ (b) − ⋅
5 4 πε0 r 5 4 πε0 r
coefficient of viscosity of air ≈ 1.8 × 10 −5 N-sm
pi

−2 7 q2 1 q2
? (Neglect buoyancy of air) (c) ⋅ (d) ⋅
5 4 πε0 r 5 4 πε0 r
m

(a) 49 ms −1 (b) 98 ms −1 (c) 392 ms −1 (d) 980 ms −1


24. Consider a region in free space bounded by
ha

20. The water equivalent of a calorimeter is 10 g


and it contains 50 g of water at 15°C. Some the surfaces of an imaginary cube having
amount of ice, initially at − 10 °C is dropped sides of length a as shown in the figure. A
charge + Q is placed at the centre O of the
C

in it and half of the ice melts till equilibrium


is reached. What was the initial amount of cube. P is such a point outside the cube that
ice that was dropped (when specific heat of the line OP perpendicularly intersects the
dy

surface ABCD at R and also OR = RP = a /2.


ice = 0 .5 cal gm −1 ° C−1 , specific heat of water
A charge + Q is placed at point P also. What is
. cal gm −1 ° C−1 and latent heat of melting
= 10 the total electric flux through the five faces
u

of ice = 80 cal gm −1 )? of the cube other than ABCD?


St

(a) 10 g (b) 18 g (c) 20 g (d) 30 g A


a
B
21. One mole of a monoatomic ideal gas
a/2 a/2
undergoes a quasistatic process, which is O P
depicted by a straight line joining points a +Q R +Q
D
(V0 , T0) and (2 V0 , 3T0) in a V-T diagram. What
is the value of the heat capacity of the gas at
a C
the point (V0 , T0)?
3 Q 5Q
(a) R (b) R (a) (b)
2 ε0 6ε0
(c) 2R (d) 0 10Q
(c) (d) zero
6 ε0
4 WB JEE (Engineering) Solved Paper 2018

25. Four equal charges of value + Q are placed at B at its centre. If instead, a circular loop of
any four vertices of a regular hexagon of side radius 2r , made of same material, having the
‘a’. By suitably choosing the vertices, what same cross-section is connected to the same
can be the maximum possible magnitude of voltage source, what will be the magnetic
electric field at the centre of the hexagon? field at its centre?
Q 2Q B B
(a) (b) (a) (b) (c) 2B (d) B
2 4
4 πε0 a2 4 πε0 a2
3Q 2Q 29. An alternating current is flowing through a

)
(c) (d)
4 πε0 a2 4 πε0 a2 series L-C-R circuit. It is found that the

be
current reaches a value of 1 mA at both
26. A proton of mass m moving with a speed v 200 Hz and 800 Hz frequency. What is the

Tu
(<< c, velocity of light in vacuum) completes resonance frequency of the circuit?
a circular orbit in time T in a uniform (a) 600 Hz (b) 300 Hz
magnetic field. If the speed of the proton is (c) 500 Hz (d) 400 Hz

ou
increased to 2 v, what will be time needed to
30. An electric bulb, a capacitor, a battery and a
complete the circular orbit?
switch are all in series in a circuit. How does
(a) 2 T (b) T the intensity of light vary when the switch is
(c)
T
2
(d)
T
2

27. A uniform current is flowing along the length


(Y turn on?
(a) Continues to increase gradually
(b) Gradually increases for sometime and then
on
of an infinite, straight, thin, hollow cylinder becomes steady
of radius R. The magnetic field B produced at (c) Sharply rises initially and then gradually
decreases
pi

a perpendicular distance d from the axis of


the cylinder is plotted in a graph. Which of (d) Gradually increases for sometime and then
gradually decreases
m

the following figures looks like the plot?


B Category-II (Q. Nos. 31 to 35)
ha

(a) Only one answer is correct. Correct answer will


d
fetch full marks 2. Incorrect answer or any
C

B
R combination of more than one answer will fetch
1
− marks. No answer will fetch 0 marks.
2
dy

(b)

d 31. A light charged particle is revolving in a


R
circle of radius r in electrostatic attraction of
u

B
a static heavy particle with opposite charge.
St

(c) How does the magnetic field B at the centre


of the circle due to the moving charge
d depend on r ?
R
1 1
B (a) B ∝ (b) B ∝
r r2
1 1
(d) (c) B ∝ (d) B ∝
r3/ 2 r5/ 2
d 32. As shown in the figure, a rectangular loop of
R
a conducting wire is moving away with a
28. A circular loop of radius r of conducting wire constant velocity v in a perpendicular
connected with a voltage source of zero direction from a very long straight conductor
internal resistance produces a magnetic field carrying a steady current I. When the
WB JEE (Engineering) Solved Paper 2018 5

breadth of the rectangular loop is very small conductor with uniform surface charge
compared to its distance from the straight density σ is placed below it. What will be the
conductor, how does the emf. E induced in time period of the pendulum for small
the loop vary with time t ? amplitude oscillations?
v
L

I
m

)
1 1 1
(a) E ∝ (b) E ∝ (c) E ∝−ln(t ) (d) E ∝ σ

be
t2 t t3
L L
33. A solid spherical ball and a hollow spherical (a) 2 π (b)
 mq   mqσ 

Tu
ball of two different materials of densities ρ1 g −  g − 
 ε0 σ   ε0 
and ρ2 respectively have same outer radii and
same mass. What will be the ratio, the 1 L L
(c) (d) 2 π

ou
moment of inertia (about an axis passing 2π  qσ   qσ 
g −  g − 
through the centre) of the hollow sphere to  ε0 m   ε0 m 
that of the solid sphere?

(a)
ρ2
ρ1


ρ 3
1 − 2 
ρ1 
5

(b)

ρ2  
ρ1  

1 − 1 − 
5
ρ2  3 
ρ1  

(Y Category-III (Q. Nos. 36 to 40)
One or more answer(s) is (are) correct. Correct
on
answer(s) will fetch full marks 2. Any combination
5  5
containing one or more incorrect answer will fetch
ρ2  ρ 3 ρ  ρ1  3 
(c) 1 − 1  (d) 2 1 −  1 −  0 marks. Also, no answer will fetch 0 marks. If all
ρ1  ρ2  ρ1   ρ2  
pi

  correct answers are not marked and also no


incorrect answer is marked then score
m

34. The insulated plates of a charged parallel = 2 × number of correct answers marked ÷ actual
plate capacitor (with small separation number of correct answers.
ha

between the plates) are approaching each


other due to electrostatic attraction. 36. A non-zero current passes through the
Assuming no other force to be operative and galvanometer G shown in the circuit when
C

no radiation taking place, which of the the key K is closed and its value does not
following graphs approximately shows the change when the key is opened. Then, which
variation with time ()
t of the potential of the following statement(s) is/are true?
dy

difference (V) between the plates?


30
0

V V
0
u

20


St

(a) (b) K
t t
O O G
10
0

V V
(c) (d) 10 V

t t (a) The galvanometer resistance is infinite.


O O
(b) The current through the galvanometer is 40 mA.
35. The bob of a pendulum of mass m, (c) After the key is closed, the current through the
suspended by an inextensible string of length 200 Ω resistor is same as the current through
the 300 Ω resistor.
L as shown in the figure carries a small
(d) The galvanometer resistance is 150 Ω.
charge q. An infinite horizontal plane
6 WB JEE (Engineering) Solved Paper 2018

37. A ray of light is incident R 39. Which of the following statements(s) is/are
on a right angled true?
isosceles prism parallel Q “Internal energy of an ideal gas ……… .”
to its base as shown in S (a) decreases in an isothermal process.
the figure. Refractive (b) remains constant in an isothermal process.
index of the material of (c) increases in an isobaric process.
P
the prism is 2. Then, (d) decreases in an isobaric expansion.
which of the following statement(s) is/are true?
40. Two positive charges Q and 4Q are placed at

)
(a) The reflection at P is total internal.

be
points A and B respectively, where B is at a
(b) The reflection at Q is total internal.
distance d units to the right of A. The total
(c) The ray emerging at R is parallel to the ray
incident at S. electric potential due to these charges is

Tu
(d) Total deviation of the ray is 150°. minimum at P on the line through A and B.
What is (are) the distance (s) of P from A?
38. The intensity of a sound appears to an (a)
d
units to the right of A

ou
observer to be periodic. Which of the 3
following can be the cause of it? d
(b) units to the left of A
(a) The intensity of the source is periodic 3

(Y
d
(b) The source is moving towards the observer (c) units to the right of A
(c) The observer is moving away from the source 5
(d) The source is producing a sound composed of (d) d units to the left of A
on
two nearby frequencies
pi

Chemistry
m

Category-I (Q. Nos. 41 to 70) 44. The ease of hydrolysis in the compounds
ha

Only one answer is correct. Correct will fetch full CH 3COCl(I),CH 3  CO  O  COCH 3 (II),
marks 1. Incorrect answer or any combination of CH 3COOC2H 5 (III) and CH 3CONH 2 (IV) is of
more than one answer will detch − 1/4 marks. No the order
C

answer will fetch 0 marks. (a) I > II > III > IV


(b) IV > III > II > I
dy

41. Cl 2O 7 is the anhydride of (c) I > II > IV > III


(a) HOCl (b) HClO 2 (d) II > I > IV > III
u

(c) HClO 3 (d) HClO 4


45. CH 3  C ≡≡ C MgBr can be prepared by the
St

42. The main reason that SiCl 4 is easily reaction of


hydrolysed as compared to CCl 4 is that (a) CH3  C ≡≡ C Br with MgBr2
(a) Si  Cl bond is weaker than C  Cl bond (b) CH3  C ≡≡ CH with MgBr2
(b) SiCl 4 can form hydrogen bonds (c) CH3  C ≡≡ CH with KBr and Mg metal
(c) SiCl 4 is covalent
(d) CH3  C ≡≡ CH with CH3MgBr
(d) Si can extend its coordination number beyond
four 46. The number of alkene (s) which can produce
43. Silver chloride dissolves in excess of 2-butanol by the successive treatment of
ammonium hydroxide solution. The cation (i) B 2H 6 in tetrahydrofuran solvent and
present in the resulting solution is (ii) alkaline H 2O 2 solution is
(a) 1 (b) 2
(a) [Ag(NH3 )6 ]+ (b) [Ag(NH3 )4 ]+
(c) 3 (d) 4
(c) Ag + (d) [Ag(NH3 )2 ]+
WB JEE (Engineering) Solved Paper 2018 7

47. Identify ‘M’ in the following sequence of 51. If aniline is treated with conc. H 2SO 4 and
reactions heated at 200°C, the product is
CH3
(a) anilinium sulphate
Cl (b) benzenesulphonic acid
NH3 Br2
C8H6Cl2O C8H8ClNO (c) m-aminobenzenesulphonic acid
NaOH
M H2N (d) sulphanilic acid
O O 52. Which of the following electronic
configuration is not possible?

)
Cl C C

be
Cl Cl (a) n = 3, l = 0, m = 0
(a) (b)
(b) n = 3, l = 1, m = − 1
CH3 Cl (c) n = 2, l = 0, m = − 1

Tu
CH3 (d) n = 2, l = 1, m = 0
CHO Cl
53. The number of unpaired electrons in Ni
(c) (d) Cl
(atomic number = 28) are

ou
Cl C
(a) 0 (b) 2 (c) 4 (d) 8
CH2Cl CH3
O
54. Which of the following has the strongest
48. Methoxybenzene on treatment with HI
produces (Y H-bond?
(a) O  H ... S
(c) F  H ... F
(b) S  H ... O
(d) F  H ... O
on
(a) iodobenzene and methanol
(b) phenol and methyl iodide
55. The half-life of C14 is 5760 years. For a
(c) iodobenzene and methyl iodide
200 mg sample of C14 , the time taken to
pi

(d) phenol and methanol


change to 25 mg is
K 2 Cr2 O7 I /NaOH
49. C 4 H10O → C 4 H 8O →
2
CHI3 (a) 11520 years (b) 23040 years
m

H 2 SO 4 Warm
N (c) 5760 years (d) 17280 years
Here, N is
ha

56. Ferric ion forms a prussian blue precipitate


OH
(a) OH (b) due to the formation of
(a) K 4 [Fe(CN)6 ] (b) K 3 [Fe(CN)6 ]
C

(c) Fe(CNS)3 (d) Fe 4 [Fe(CN)6 ]3


(c) O (d)
OH
64
57. The nucleus 29 Cu accepts an orbital electron
dy

50. The correct order of reactivity for the addition to yield,


reaction of the following carbonyl (a) 65
28 Ni (b) 64
30 Zn (c) 64
28 Ni (d) 65
30 Zn
u

compounds with ethylmagnesium iodide is


58. How many moles of electrons will weigh one
St

H H3C
kilogram?
C=
=O C=
=O 1
H H3C (a) 6.023 × 1023 (b) × 1031
(I) (II) 9108
.
6.023 1
(c) × 1054 (d) × 108
H (CH3)3C .
9108 9108
. × 6.023
C=
=O C=
=O
H3C (CH3)3C 59. Equal weights of ethane and hydrogen are
(III) (IV) mixed in an empty container at 25°C. The
(a) I > III > II > IV fraction of total pressure exerted by hydrogen
(b) IV > III > II > I is
(c) I > II > IV > III (a) 1 : 2 (b) 1 : 1
(d) III > II > I > IV (c) 1 : 16 (d) 15 : 16
8 WB JEE (Engineering) Solved Paper 2018

60. The heat of neutralisation of a strong base 66. Which of the following is present in
and a strong acid is 13.7 kcal. The heat maximum amount in ‘acid rain’?
released when 0.6 mole HCl solution is added (a) HNO 3 (b) H2SO 4
to 0.25 mole of NaOH is (c) HCl (d) H2CO 3
(a) 3.425 kcal (b) 8.22 kcal
67. Which of the set of oxides are arranged in the
(c) 11.645 kcal (d) 13.7 kcal
proper order of basic, amphoteric, acidic?
61. A compound formed by elements X and Y (a) SO 2 ,P2O 5 ,CO (b) BaO,Al 2O 3 ,SO 2
crystallises in the cubic structure, where X

)
(c) CaO,SiO 2 ,Al 2O 3 (d) CO 2 ,Al 2O 3 ,CO

be
atoms are at the corners of a cube and Y
atoms are at the centre of the body. The
68. Out of the following outer electronic
configurations of atoms, the highest
formula of the compounds is
oxidation state is achieved by which one?

Tu
(a) XY (b) XY2
(c) X 2 Y3 (d) XY3 (a) (n − 1)d 8 ns 2 (b) (n − 1)d 5 ns 2
(c) (n − 1)d ns3 2
(d) (n − 1)d 5 ns1
62. What amount of electricity can deposit

ou
1 mole of Al metal at cathode when passed 69. At room temperature, the reaction between
through molten AlCl 3 ? water and fluorine produces

(Y
(a) 0.3 F (b) 1 F (a) HF and H2O 2 (b) HF, O 2 and F2O 2
(c) 3 F (d) 1/3 F (c) F − , O 2 and H+ (d) HOF and HF

63. Given the standard half-cell potentials (E°) of 70. Which of the following is least thermally
on
the following as stable?
Zn → Zn 2+ + 2 e − ; E° = + 0 .76 V (a) MgCO 3 (b) CaCO 3
(c) SrCO 3 (d) BeCO 3
pi

Fe → Fe2+ + 2 e − ; E° = 0 .41 V
Then the standard e.m.f. of the cell with the Category-II (Q. Nos. 71 to 75)
m

reaction Fe2+ + Zn → Zn 2+ + Fe is Only one answer is correct. Correct answer will


(a) − 0.35 V (b) + 0.35 V
ha

fetch full marks 2. Incorrect answer or any


(c) + 117
. V (d) − 117
. V combination of more than one answer will fetch −
64. The following equilibrium constants are 1/2 marks. No answer will fetch 0 marks.
C

given Br 2 NaNH 2
N 2 + 3H 2 c
2NH 3 ; K 1 71. [ P] → C2H 4Br2 →
NH
[ Q]
3
dy

N2 + O 2
1
c
2NO;K 2 20% H 2 SO 4
[ Q] → [ R] → [ S]
Zn -Hg/HCl

H2 + O 2 c
H 2O; K 3 Hg 2+ , ∆
u

2
The species P, Q, R and S respectively are
The equilibrium constant for the oxidation of
St

(a) ethene, ethyne, ethanal, ethane


2 mole of NH 3 to give NO is (b) ethane, ethyne, ethanal, ethene
K2 K 33
(a) K1 . (b) K 2 . (c) ethene, ethyne, ethanal, ethanol
K3 K1 (d) ethyne, ethane, ethene, ethanal
K 32 K3
(c) K 2 . (d) K 22 72. The number of possible organobromine
K1 K1
compounds which can be obtained in the
allylic bromination of 1-butene with
65. Which one of the following is a condensation
N-bromosuccinimide is
polymer?
(a) 1 (b) 2
(a) PVC (b) Teflon
(c) 3 (d) 4
(c) Dacron (d) Polystyrene
WB JEE (Engineering) Solved Paper 2018 9
O
73. A metal M (specific heat 0.16) forms a metal
OH
chloride with 65% chlorine present in it. The
(a) (b)
formula of the metal chloride will be
(a) MCl (b) MCl 2 CH2OH
(c) (d) H2C CH2
(c) MCl 3 (d) MCl 4

74. During a reversible adiabatic process, the


77. The major products obtained in the following
pressure of a gas is found to be proportional
reaction is/are

)
to the cube of its absolute temperature. The

be
Cp H3C H
ratio for the gas is C=
=C + Br2
CV
H C2H5
3 7

Tu
(a) (b) CH3 CH3
2 2
H Br H Br
5 9 (a) (b)
(c) (d)

ou
3 7 H Br Br H
C2H5 C2H5
75. [ X ] + dil. H 2SO 4 → [ Y ] :

(Y
Colourless, suffocating gas CH3 CH3
[ Y ] + K 2Cr2O 7 + H 2SO 4 → Br H Br H
(c) (d)
Green colouration of solution H Br Br H
on
Then,[ X ] and[ Y ] are
C2H5 C2H5
(a) SO23 − , SO2
(b) Cl − , HCl 78. Which statements are correct for the
pi

(c) S2 − , H 2 S peroxide ion?


(d) CO23 − , CO2
m

(a) It has five completely filled anti-bonding


molecular orbitals
Category-III (Q. Nos. 76 to 80) (b) It is diamagnetic
ha

(c) It has bond order one


One or more answer(s) is (are) correct. Correct (d) It is isoelectronic with neon
answer(s) will fetch full marks 2. Any combination
C

containing one or more incorrect answer will fetch 79. Among the following, the extensive variables
are
0 marks. Also no answer will fetch 0 marks. If all
(a) H (Enthalpy) (b) p (Pressure)
dy

correct answers are not marked and also no


(c) E (Internal energy) (d) V (Volume)
incorrect answer is marked then score
= 2 × number of correct answers marked ÷ actual 80. White phosphorous P4 has the following
u

number of correct answers. characteristics


(a) 6P  P single bonds
St

76. The possible product(s) to be obtained from (b) 4P  P single bonds


the reaction of cyclobutyl amine with HNO 2 (c) 4 lone pair of electrons
is/are (d) P  P  P angle of 60°
10 WB JEE (Engineering) Solved Paper 2018

Mathematics
Category-I (Q. Nos. 1 to 50) 6. Let f :[ a , b] → R be such that f is
Only one answer is correct. Correct answer will differentiable in (a , b), f is continuous at x = a
fetch full marks 1. Incorrect answer or any and x = b and moreover f (a) = 0 = f (b). Then
combination of more than one answer will (a) there exists at least one point c in (a, b ) such that

)
fetch-1/4 marks. No answer will fetch 0 marks. f ′(c ) = f(c )

be
(b) f ′( x) = f( x) does not hold at any point in (a, b )
1. The approximate value of sin31° is (c) at every point of (a, b ), f ′( x) > f( x)
(a) > 0.5 (b) > 0.6 (d) at every point of (a, b ), f ′( x) < f( x)

Tu
(c) < 0.5 (d) < 0.4
7. Let f : R → R be a twice continuously
2. Let f1(x) = e x , f 2(x) = e f1 (x ), ……, differentiable function such that

ou
f n + 1 (x) = e f n (x ) for all n ≥ 1. Then for any fixed f (0) = f ()
1 = f ′ (0) = 0 . Then
d (a) f ′ ′(0) = 0
n, f n(x) is (b) f ′ ′(c ) = 0 for some c ∈ R
dx
(a) fn ( x)
(b) fn ( x)fn − 1( x)
(Y (c) if c ≠ 0, then f ′ ′(c ) ≠ 0
(d) f ′( x) > 0 for all x ≠ 0
on
 x cos 3 x − sin x 
(c) fn ( x)fn − 1( x)…f1( x) 8. If ∫ esin x ⋅  2  dx = e
sin x
f (x) + c,
(d) fn ( x)… f1( x)e x  cos x 
pi

1 − | x| where c is constant of integration, then f (x) is


3. The domain of definition of f (x) = is equal to
2 − | x|
m

(a) sec x − x (b) x − sec x


(a) (−∞, − 1) ∪ (2, ∞ ) (c) tan x − x (d) x − tan x
(b) [−1, 1] ∪ (2, ∞ ) ∪ (−∞, − 2 )
ha

1
(c) (−∞, 1) ∪ (2, ∞ ) 9. If ∫ f (x) sin x cos x dx = log f (x) + c,
2(b − a 2) 2
(d) [−1, 1] ∪ (2, ∞ )
C

where c is the constant of integration, then


Here (a , b) ≡ { x : a < x < b} and
f (x) is equal to
[ a , b] ≡ { x : a ≤ x ≤ b} 2 2
dy

(a) (b)
4. Let f :[ a , b] → R be differentiable on [ a , b] (b 2 − a2 )sin2 x ab sin2 x
2 2
and k ∈ R. Let f (a) = 0 = f (b). (c) (d)
(b 2 − a2 )cos 2 x
u

ab cos 2 x
Also let J (x) = f ′ (x) + kf (x). Then
St

(a) J( x) > 0 for all x ∈[a, b ] π /2 π /4


cos x sin x cos x
(b) J( x) < 0 for all x ∈[a, b ]
10. If M = ∫ x +2
dx , N = ∫ (x + 1)2
dx , then
0 0
(c) J( x) = 0 has at least one root in (a, b )
the value of M − N is
(d) J( x) = 0 through (a, b ) π 2 2
(a) π (b) (c) (d)
5. Let f (x) = 3 x 10
− 7 x + 5 x − 21 x + 3 x − 7 .
8 6 3 2 4 π−4 π+ 4
f (1 − h) − f ()
1
tan −1 x
2014
Then lim
h→ 0 h + 3h
3 11. The value of the integral I = ∫ x
dx is
1 / 2014
50
(a) does not exist (b) is π π
3 (a) log 2014 (b) log 2014
53 22 4 2
(c) is (d) is 1
3 3 (c) πlog 2014 (d) log 2014
2
WB JEE (Engineering) Solved Paper 2018 11
π /3
sin x 19. Given that n numbers of arithmetic means
12. Let I = ∫ x
dx . Then
are inserted between two sets of numbers
π /4
a , 2 b and 2a , b where a , b ∈ R. Suppose further
1
(a) ≤ I≤1 (b) 4 ≤ I ≤ 2 30 that the mth means between these sets of
2
numbers are same, then the ratio a : b equals
3 2 2 3
(c) ≤ I≤ (d) 1 ≤ I ≤ (a) n − m + 1: m (b) n − m + 1: n
8 6 2
(c) n : n − m + 1 (d) m : n − m + 1
13. The value of 20. If x + log 10(1 + 2x ) = x log 10 5 + log 10 6 , then

)
5π / 2 tan −1 (sin x )
e

be
I= ∫ tan −1 (sin x ) tan −1 (cos x )
dx , is the value of x is
π /2 e +e (a)
1
(b)
1
π 2 3
(b) π

Tu
(a) 1 (c) e (d) (c) 1 (d) 2
2
2 πr 2 πr 10
14. The value of 21. If Z r = sin − i cos , then ∑ Z r is equal

ou
1 2 π 2π nπ  11 11 r=0
lim sec + sec 2 + ... + sec 2  is
n→ ∞ n 4n 4n 4 n to
π 4 (a) −1 (b) 0 (c) i (d) −i
(a) loge 2 (b)
2

15. The differential equation representing the


(c)
π
(d) e

(Y
22. If z1 and z 2 be two non-zero complex
numbers such that
z1 z 2
+ = 1, then the
on
family of curves y 2 = 2 d(x + d), where d is a z 2 z1
parameter, is of origin and the points represented by z1 and z 2
pi

(a) order 2 (b) degree 2 (a) lie on a straight line


(c) degree 3 (d) degree 4 (b) form a right angled triangle
m

(c) form an equilateral triangle


16. Let y(x) be a solution of (d) form an isosceles triangle
dy
(1 + x 2) + 2 xy − 4 x 2 = 0 and y(0) = − 1. Then
ha

dx 23. If b1b2 = 2(c1 + c2) and b1 , b2 , c1 , c2 are all real


1 is equal to
y() numbers, then at least one of the equations
1 1 x 2 + b1 x + c1 = 0 and x 2 + b2 x + c2 = 0 has
C

(a) (b)
2 3 (a) real roots
1
(c) (d) −1 (b) purely imaginary roots
dy

6 (c) roots of the form a + ib (a, b ∈ R, ab ≠ 0)


(d) rational roots
17. The law of motion of a body moving along a
u

1 24. The number of selection of n objects from 2n


straight line is x = vt. x being its distance
2
St

objects of which n are identical and the rest


from a fixed point on the line at time t and v are different, is
is its velocity there. Then (a) 2 n (b) 2 n − 1
(a) acceleration f varies directly with x (c) 2 n − 1 (d) 2 n − 1 + 1
(b) acceleration f varies inversely with x
(c) acceleration f is constant 25. If (2 ≤ r ≤ n), then nC r + 2 ⋅ nC r + 1 + nC r + 2 is
(d) acceleration f varies directly with t equal to
n+1
(a) 2 ⋅n C r + 2 (b) Cr +1
18. Number of common tangents of y = x 2 and n+ 2 n+1
(c) Cr + 2 (d) Cr
y = − x 2 + 4 x − 4 is
(a) 1 (b) 2 26. The number (101)100 − 1 is divisible by
(c) 3 (d) 4 (a) 104 (b) 106
(c) 108 (d) 1012
12 WB JEE (Engineering) Solved Paper 2018

27. If n is even positive integer, then the 33. On the set R of real numbers, the relation ρ is
condition that the greatest term in the defined by xρy , (x , y) ∈ R.
expansion of (1 + x)n may also have the
(a) If| x − y| < 2, then ρ is reflexive but neither
greatest coefficient, is symmetric nor transitive.
n n+2 n n+1 (b) If x − y < 2, then ρ is reflexive and symmetric but
(a) < x< (b) < x<
n+2 n n+1 n not transitive.
n+1 n+2 n+2 n+ 3 (c) If| x| ≥ y, then ρ is reflexive and transitive but not
(c) < x< (d) < x<
n+2 n+1 n+ 3 n+2 symmetric.
(d) If x >| y|, then ρ is transitive but neither reflexive

)
−1 7 0  13 −11 5 

be
nor symmetric.
28. If 2 1 −3 = A, Then −7 −1 25  is

    34. If f : R → R be defined by f (x) = e x and
3 4 1 −21 −3 −15

Tu
g : R → R be defined by g(x) = x 2 . The
(a) A 2 (b) A 2 − A + I3
mapping gof : R → R be defined by (gof ) (x)
(c) A 2 − 3 A + I3 (d) 3 A 2 + 5 A − 4I3
= g[ f (x)] ∀ x ∈ R. Then,

ou
(I 3 denotes the det of the identity matrix of (a) gof is bijective but f is not injective
order 3) (b) gof is injective and g is injective

(Y
(c) gof is injective but g is not bijective
29. If a r = (cos 2rπ + i sin 2rπ)1/ 9, then the value of
(d) gof is surjective and g is surjective
a1 a2 a 3
a 4 a5 a 6 is equal to 35. In order to get a head at least once with
 
on
probability ≥ 0 .9 , the minimum number of
a 7 a8 a 9
times a unbiased coin needs to be tossed is
(a) 1 (b) −1 (a) 3 (b) 4 (c) 5 (d) 6
pi

(c) 0 (d) 2
 2r x n(n + 1)  36. A student appears for tests I, II and III. The
m

student is successful if he passes in tests I, II


30. If Sr = 6 r 2 − 1 y n (2 n + 3), then the
2
 3  or I, III. The probabilities of the student
4 r − 2 nr n3 (n + 1) 
ha

z passing in tests I, II and III are respectively


n
p , q and 1/2. If the probability of the student
value of ∑ Sr is independent of to be successful is 1/2. Then
r =1
C

(a) p(1 + q ) = 1 (b) q (1 + p) = 1


(a) only x (b) only y 1 1
(c) only n (d) x, y, z and n (c) pq = 1 (d) + =1
dy

p q
31. If the following three linear equations have a 37. If sin 6θ + sin 4θ + sin 2θ = 0, then general
non-trivial solution, then
u

value of θ is
x + 4 ay + az = 0 nπ π nπ π
St

x + 3 by + bz = 0 (a) , nπ ± (b) , nπ ±
4 3 4 6
x + 2 cy + cz = 0 nπ π nπ π
(c) , 2 nπ ± (d) , 2 nπ ±
(a) a, b, c are in AP (b) a, b, c are in GP 4 3 4 6
(c) a, b, c are in HP (d) a + b + c = 0 (n is an integer)

32. On R, a relation ρ is defined by xρy if and only π


38. If 0 ≤ A ≤ , then
if x − y is zero or irrational. Then, 4
1 
(a) ρ is equivalence relation tan −1  tan 2 A + tan −1 (cot A) + tan −1 (cot 3 A)
2 
(b) ρ is reflexive but neither symmetric nor transitive
(c) ρ is reflexive and symmetric but not transitive is equal to
π π
(d) ρ is symmetric and transitive but not reflexive (a) (b) π (c) 0 (d)
4 2
WB JEE (Engineering) Solved Paper 2018 13

39. Without changing the direction of the axes, 45. Let the eccentricity of the hyperbola
the origin is transferred to the point (2, 3). x2 y2
− = 1 be reciprocal to that of the ellipse
Then the equation x 2 + y 2 − 4 x − 6 y + 9 = 0 a2
b2
changes to x 2 + 9 y 2 = 9 , then the ratio a 2 : b2 equals
(a) x2 + y2 + 4 = 0
(a) 8 : 1 (b) 1 : 8
(b) x2 + y2 = 4 (c) 9 : 1 (d) 1 : 9
(c) x2 + y2 − 8 x − 12 y + 48 = 0
46. Let A , B be two distinct points on the
(d) x2 + y2 = 9

)
parabola y 2 = 4 x . If the axis of the parabola

be
40. The angle between a pair of tangents drawn touches a circle of radius r having AB as
from a point P to the circle diameter, the slope of the line AB is

Tu
x 2 + y 2 + 4 x − 6 y + 9 sin 2 α + (a) −
1
(b)
1
(c)
2
(d) −
2
r r r r
13 cos 2 α = 0 is 2α. The equation of the locus of
47. Let P (at2 , 2at), Q, R(ar 2 , 2ar) be three points on

ou
the point P is
(a) x2 + y2 + 4 x + 6 y + 9 = 0 a parabola y 2 = 4 ax . If PQ is the focal chord
(b) x2 + y2 − 4 x + 6 y + 9 = 0 and PK, QR are parallel where the co-ordinates
(c) x2 +
(d) x2 +
y2 − 4 x − 6 y + 9 = 0
y2 + 4 x − 6 y + 9 = 0 (Y of K is (2 a , 0), then the value of r is
(a)
1− t
t
2
(b)
1− t2
t
(c)
t2 + 1
t
(d)
t2 − 1
t
on
41. The point Q is the image of the point P(1, 5)
about the line y = x and R is the image of the x 2 y2
48. Let P be a point on the ellipse + = 1 and
point Q about the line y = − x . The 9 4
pi

circumcentre of the ∆PQR is the line through P parallel to the Y-axis


(a) (5, 1) (b) (−5, 1) meets the circle x 2 + y 2 = 9 at Q, where P , Q
m

(c) (1, − 5) (d) (0, 0)


are on the same side of the X -axis. If R is a
PR 1
ha

42. The angular points of a triangle are A(−1, − 7), point on PQ such that = , then the locus
B(5 , 1) and C(1, 4). The equation of the bisector RQ 2
of the angle ∠ABC is of R is
C

(a) x = 7 y + 2 (b) 7 y = x + 2 x2 9 y2 x2 y2
(a) + =1 (b) + =1
(c) y = 7 x + 2 (d) 7 x = y + 2 9 49 49 9
dy

43. If one of the diameter of the circle, given by x2 y2 9 x2 y2


(c) + =1 (d) + =1
the equation x 2 + y 2 + 4 x + 6 y − 12 = 0 , is a 9 49 49 9
u

chord of a circle S, whose centre is (2 , − 3), the 49. A point P lies on a line through Q(1, − 2, 3) and
radius of S is
St

x y z
(a) 41 unit (b) 3 5 unit is parallel to the line = = . If P lies on
1 4 5
(c) 5 2 unit (d) 2 5 unit the plane 2 x + 3 y − 4 z + 22 = 0 , then segment
44. A chord AB is drawn from the point A(0 , 3) on PQ equals
(a) 42 units (b) 32 units
the circle x 2 + 4 x + (y − 3)2 = 0 , and is
(c) 4 units (d) 5 units
extended to M such that AM = 2 AB. The locus
of M is 50. The foot of the perpendicular drawn from the
(a) x +
2
y − 8x − 6y + 9 = 0
2 point (1, 8, 4) on the line joining the point
(b) x2 + y2 + 8 x + 6 y + 9 = 0 (0, −11, 4) and (2, −3, 1) is
(c) x2 + y2 + 8 x − 6 y + 9 = 0 (a) (4, 5, 2) (b) (−4, 5, 2)
(d) x2 + y2 − 8 x + 6 y + 9 = 0 (c) (4, −5, 2) (d) (4, 5, −2)
14 WB JEE (Engineering) Solved Paper 2018

Category-II (Q. No. 51 to 65) 56. From a collection of 20 consecutive natural


Carry 2 marks each if only one option is correct. numbers, four are selected such that they are
In case of incorrect answer or any combination of not consecutive. The number of such
more than one answer, 1/2 mark will be selections is
deducted. (a) 284 × 17 (b) 285 × 17
(c) 284 × 16 (d) 285 × 16
51. A ladder 20 ft long leans against a vertical
57. The least positive integer n such that
wall. The top end slides downwards at the n
 π π

)
rate of 2 ft per second. The rate at which the
 cos sin 

be
lower end moves on a horizontal floor when  4 4  is an identity matrix of
it is 12 ft from the wall is π π
 − sin cos 
−8 6  4 4

Tu
(a) (b)
3 5 order 2 is
3 17 (a) 4 (b) 8
(c) (d)
2 4 (c) 12 (d) 16

ou
52. For 0 ≤ p ≤ 1 and for any positive a , b; let 58. Let ρ be a relation defined on N, the set of
I (p) = (a + b)p , J (p) = a P + bP , then natural numbers, as
(a) I( p) > J( p)
(b) I( p) ≤ J( p)
(c) I( p) < J( p) in  0,  and I( p) > J( p) in  , ∞ 
p p
(Y ρ = {(x , y) ∈ N × N : 2 x + y = 41}. Then
(a) ρ is an equivalence relation
(b) ρ is only reflexive relation
on
 2   2  (c) ρ is only symmetric relation
 p   p (d) ρ is not transitive
(d) I( p) < J( p) in , ∞  and J( p) < I( p) in 0,
 2   2 
pi

59. If the polynomial



$ →
53. Let α = i$ + j$ + k, β = i$ − j$ − k$ and (1 + x)a (2 + x)b 1 
m

→ → f (x) =  1 (1 + x) (2 + x)b, then the


a
γ = − i$ + j$ − k$ be three vectors. A vector δ , in  
(2 + x) (1 + x)a
b
ha

→ → → 1
the plane of α and β , whose projection on γ
1 constant term of f (x) is
is , is given by (a) 2 − 3 ⋅ 2 b + 2 3 b (b) 2 + 3 ⋅ 2 b + 2 3 b
C

3
(c) 2 + 3 ⋅ 2 − 2
b 3b
(d) 2 − 3 ⋅ 2 b − 2 3 b
(a) − $i − 3 $j − 3k$ (b) $i − 3 $j − 3k$
[a and b are positive integers]
dy

(c) − $i + 3 $j + 3k$ (d) $i + 3 $j − 3k$


60. A line cuts the X-axis at A(5, 0) and the Y-axis
→ → →
54. Let α , β , γ be the three unit vectors such that at B(0 , − 3). A variable line PQ is drawn
u

→ → → → →
perpendicular to AB cutting the X-axis at P
St

α ⋅ β = α ⋅ γ = 0 and the angle between β and and the Y-axis at Q. If AQ and BP meet at R,
→ → then the locus of R is
γ is 30°. Then α is (a) x2 + y2 − 5 x + 3 y = 0 (b) x2 + y2 + 5 x + 3 y = 0
→ → → →
(a) 2(β × γ ) (b) −2(β × γ ) (c) x2 + y2 + 5 x − 3 y = 0 (d) x2 + y2 − 5 x − 3 y = 0
→ → → →
(c) ±2(β × γ ) (d) (β × γ ) 61. Let A be the centre of the circle
x 2 + y 2 − 2 x − 4 y − 20 = 0 . Let B(1, 7) and
55. Let z1 and z 2 be complex numbers such that
D(4 , − 2) be two points on the circle such that
z1 ≠ z 2 and|z1| = |z 2|. If Re(z1) > 0 and tangents at B and D meet at C. The area of the
z + z2
Im(z 2) < 0 , then 1 is quadrilateral ABCD is
z1 − z 2 (a) 150 sq units (b) 50 sq units
(a) one (b) real and positive (c) 75 sq units (d) 70 sq units
(c) real and negative (d) purely imaginary
WB JEE (Engineering) Solved Paper 2018 15

 π 67. A particle is in motion along a curve 12 y = x 3.


−2 sin x , if x ≤ −
 2 The rate of change of its ordinate exceeds
 π π that of abscissa in
62. Let f (x) =  Asin x + B, if − < x < . Then,
 2 2 (a) −2 < x < 2 (b) x = ± 2
 cos x , π (c) x < − 2 (d) x > 2
if x ≥
 2 68. The area of the region lying above X-axis, and
(a) f is discontinuous for all A and B included between the circle x 2 + y 2 = 2 ax and
(b) f is continuous for all A = − 1and B = 1 the parabola y 2 = ax ,a > 0 is

)
(c) f is continuous for all A = 1and B = − 1 π 2
(b) a2  − 

be
(d) f is continuous for all real values of A, B (a) 8 πa2
 4 3
63. The normal to the curve y = x 2 − x + 1, drawn (c)
16 πa2
(d) π 
27
+ 3a2 

Tu
at the points with the abscissa x1 = 0 , x 2 = − 1 9  8 
and x 3 = 5 / 2 69. If the equation x 2 − cx + d = 0 has roots equal
(a) are parallel to each other to the fourth powers of the roots of

ou
(b) are pairwise perpendicular x 2 + ax + b = 0 , where a 2 > 4 b , then the roots
(c) are concurrent
(d) are not concurrent of x 2 − 4 bx + 2 b2 − c = 0 will be

64. The equation x log x = 3 − x


(a) has no root in (1, 3) (Y (a) both real
(b) both negative
(c) both positive
on
(b) has exactly one root in (1, 3) (d) one positive and one negative
(c) xlog x − (3 − x) > 0 in [1, 3]
70. On the occasion of Dipawali festival each
(d) xlog x − (3 − x) < 0 in [1, 3]
student of a class sends greeting cards to
pi

65. Consider the parabola y = 4 x . Let P and Q be


2 others. If there are 20 students in the class,
the number of cards sends by students is
points on the parabola where P(4 , − 4) and
m

(a) 20
C2 (b) 20
P2 (c) 2 × C 2 (d) 2 ×
20 20
P2
Q(9 , 6). Let R be a point on the arc of the
parabola between P and Q. Then, the area of 71. In a third order matrix A, a ij denotes the
ha

∆PQR is largest when element in the ith row and jth column.
(a) ∠PQR = 90° (b) R(4, 4) If a ij = 0 for i = j
(c) R  , 1 (d) R  1, 
C

1 1 = 1 for i > j
4   4 = −1 for i < j
dy

Then the matrix is


Category-III (Q. Nos. 66 to 75)
(a) skew symmetric (b) symmetric
Carry 2 marks each and one or more option(s) is/are (c) not invertible (d) non-singular
correct. If all correct answers are not marked and
u

also no incorrect answer is marked then score = 2 × 72. The area of the triangle formed by the
St

number of correct answers marked, ÷ actual number intersection of a line parallel to X-axis and
of correct answer. If any wrong option is marked or passing through P(h, k), with the lines y = x
if, any combination including a wrong option is and x + y = 2 is h2 . The locus of the point P is
marked, the answer will considered wrong, but (a) x = y − 1 (b) x = − ( y − 1)
there is no negative marking for the same and zero (c) x = 1 + y (d) x = − (1 + y)
marks will be awarded.
73. A hyperbola, having the transverse axis of
1
x 3 cos 3 x length 2sinθ is confocal with the ellipse
66. Let I = ∫ 2+ x 2
dx . Then 3 x 2 + 4 y 2 = 12. Its equation is
0
1 1 1 1 (a) x2 sin2 θ − y2 cos 2 θ = 1
(a) − < I < (b) − < I < (b) x2cosec 2θ − y2 sec 2 θ = 1
2 2 3 3
(c) −1 < I < 1
3
(d) − < I <
3 (c) ( x2 + y2 )sin2 θ = 1 + y2
2 2 (d) x2cosec 2θ = x2 + y2 + sin2 θ
16 WB JEE (Engineering) Solved Paper 2018
 π
74. Let f (x) = cos   , x ≠ 0 , then assuming k as 75. Consider the function y = log a (x + x 2 + 1),
 x
an integer, a > 0, a ≠ 1. The inverse of the function
 1 1 (a) does not exist
(a) f( x) increases in the interval  , 
 2k + 1 2k  (b) is x = log1/ a ( y + y2 + 1)
 1 1
(b) f( x) decreases in the interval  ,  (c) is x = sinh ( y log a)
 2k + 1 2k 
(d) is x = cos h  − y log 
1
 1 1  
(c) f( x) decreases in the interval   a

)
,
 2 k + 2 2 k + 1

be
 1 1 
(d) f( x) increases in the interval  , 
 2 k + 2 2 k + 1

Tu
Answers

ou
Physics
1. (d)
11. (a)
21. (c)
2. (c)
12. (c)
22. (b)
3. (a)
13. (b)
23. (b)
4. (b)
14. (a)
24. (a)
5. (d)
15. (c)
25. (c)
(Y
6. (c)
16. (c)
26. (b)
7. (a)
17. (c)
27. (c)
8. (d)
18. (d)
28. (b)
9. (d)
19. (b)
29. (d)
10. (c)
20. (c)
30. (c)
on
31. (d) 32. (a) 33. (d) 34. (a) 35. (d) 36. (b,c,d) 37. (a,c) 38. (a,d) 39. (b) 40. (a)

Chemistry
pi

41. (d) 42. (d) 43. (d) 44. (a) 45. (d) 46. (b) 47. (b) 48. (b) 49. (b) 50. (a)
m

51. (d) 52. (c) 53. (b) 54. (c) 55. (d) 56. (d) 57. (c) 58. (d) 59. (d) 60. (a)
61. (a) 62. (c) 63. (b) 64. (b) 65. (c) 66. (b) 67. (b) 68. (b) 69. (c) 70. (d)
ha

71. (a) 72. (a) 73. (b) 74. (a) 75. (a) 76. (a,c) 77. (a, d) 78. (b,c) 79. (a,c,d) 80. (a,c,d)

Mathematics
C

1. (a) 2. (c) 3. (b) 4. (c) 5. (c) 6. (a) 7. (b) 8. (b) 9. (c) 10. (d)
11. (b) 12. (c) 13. (b) 14. (c) 15. (c) 16. (c) 17. (c) 18. (b) 19. (d) 20. (c)
dy

21. (b) 22. (c) 23. (a) 24. (a) 25. (c) 26. (a) 27. (a) 28. (a) 29. (c) 30. (d)
31. (c) 32. (c) 33. (d) 34. (c) 35. (b) 36. (a) 37. (a) 38. (c) 39. (b) 40. (d)
u

41. (d) 42. (b) 43. (a) 44. (c) 45. (a) 46. (c,d) 47. (d) 48. (a) 49. (a) 50. (d)
51. (a) 52. (b) 53. (c) 54. (c) 55. (d) 56. (a) 57. (b) 58. (d) 59. (a) 60. (a)
St

61. (c) 62. (b) 63. (c) 64. (b) 65. (c) 66. (b) 67. (c,d) 68. (b) 69. (a,d) 70. (b,c)
71. (a,c) 72. (a,b) 73. (b) 74. (a,c) 75. (c)
Answer with Explanations
Physics
1. (d) For maximum equivalent resistance across the 3. (a) Ray diagram for the question,
diagonal of the square, the given resistors
Y
connected as
P(3,3)

)
Q

be

20

S(0,1)
0

0
30

X′ X

Tu
P O M Plane mirror
R

40

I(0,–1)
0

ou
0

10

Y′
S
I is image of source S by the plane mirror placed
Resistance of PQR arm, R1 = 300 + 200 = 500 Ω

(Y
perpendicularly along X-axis.
Resistance of PSR arm, R2 = 400 + 100 Ω = 500 Ω
Q SM = IM
The equivalent resistance between P and R.
1 1 1 ∴PM + MS = PM + MI = PI
= +
on
Req R1 R2
∴ PI = (3 − 0)2 + (3 + 1)2
1 1 1+1
= + = = 9 + 16 = 25 = 5
500 500 500
pi

500 4. (b) The given combination of lenses


∴ Req = = 250 Ω
2
m

L2
2. (c) 100 Ω 1µF 400 Ω
f1
ha

200 Ω 300 Ω 2 µF f1
C

L1 L3
6V K
dy

f2

In steady state, arm having capacitors does not flow Here, f1 = focal length of equiconvex lenses of glass.
f2 = focal length of lens formed by water (concave).
u

current. So, we can neglect them.


∴The given circuit reduces to The focal length of the combination.
St

1 1 1 1 1 1 1 2 1
200 Ω 400 Ω ∴ = + + = − + = −
F f1 f2 f3 f1 f2 f1 f1 f2
1 2 f2 − f1 f1 f2
= ⇒F= [Q f3 = f1 ]
6V F f1 f2 2 f2 − f1
f1
F= ...(i)
f
∴ Rnet = 200 + 400 = 600 Ω 2− 1
f2
∴Current in circuit,
1 2
I= =
V 6
= 0.01 A Here, = (µ g − 1) , for L1 and L3
R 600 f1 R

= (µ w − 1)  −  , for L2
= 10mA 1 2
and
f2  R
18 WB JEE (Engineering) Solved Paper 2018
Given, µ w < µ g Number of nuclei that will decay on the 3rd day,
f
Thus, 1 <1 N 3 = N 2 − N1 = 0.63 N 0 − 0.5N 0 = 013
. N0
f2 In the term, fraction is 0.13.
f1
So, F> ...(ii) 9. (d) QKinetic energy of a electron due to
2 accelerated by a potential V, KE = eV
From Eqs. (i) and (ii), we get 1
f1 f me v2 = eV
< F < f1 or < F< f (Q f1 = f) 2
2 2 1 × p2
⇒ = eV [Qp = mv]

)
5. (d) 2me

be
∴ p = 2eVme
Small air Solid glass
r

bubble O Q de-Broglie wavelength of a particle having

Tu
momentum p,
h
λ=
p

ou
As the object is at centre of the sphere.
∴All rays will fall normally on surface, hence they According to question,
do not deflect. Thus, a virtual image is formed at the λ1 p2 2 eV2me V2
= = =
centre O.
∴ Apparent depth = Real depth
6. (c) Young’s double slit experiment with white
(Y Q


λ 2 p1

λ1
2 eV1 me
λ 2 = 2λ1

=
V2
V1
(given)
on
light which consist wavelength from range 4000 Å 2λ1 10000
to 7000 Å.
104
QAt the centre of the screen, the path difference is ∴ Potential V2 = = 2500 V
pi

zero for all wavelengths. The bright fringes of these 4


wavelengths overlap at the centre. Thus, the white 10. (c) A
m

(1) X
fringe at the centre is formed.
7. (a) QLinear velocity of an electron in a orbit of
ha

H like atom, (0)


Y
Z c Z B
v = 218
. × 10 × m/s = 0 ⋅
6
n 137 n ∴ X = AY = Y = 0
C

where, Z = number of protons in nucleus, Y = BX = 1


n = principal quantum number
dy

and c 0 = speed of light in free space, 11. (a) 6 kΩ


Thus, we have
1 +
v∝
u

10V 6V 4 kΩ
n –
St

8. (d) Given, t1 / 2 = 3 days


Number of active nuclei remaining after time t, In the given circuit, the zener diode is used as a
t voltage regulating device. The voltage across 6 kΩ
N = N 0   1 / 2
1 t resistance is (10 − 6) V = 4 V
 2 Current through 6 kΩ resistor,
After t = 2days, 4V 4 2 2
I= = = × 10− 3 = mA
2/ 3
6k Ω 6 × 103 3
N1 = N 0  
1 N0 N 3
= = 0 = 0.63 N 0
 2 22/ 3 41 / 3
12. (c) In representation of simple harmonic motion
∴ N1 = 0.63N 0 in ellipse as velocity along X-axis and
After t = 3days, displacement along Y-axis.
N0 Major axis (along X - axis)
N2 = = 0.5N 0 Q = 20π
2 Minor axis (along X - axis)
WB JEE (Engineering) Solved Paper 2018 19
ωA = ( 32 + 12 + 12) ⋅( 22 + 12) cos θ
⇒ = 20 π ⇒ ω = 20 π
A
⇒ 6 − 1 = 11 × 5 ⋅ cos θ
⇒ 2π n= 20 π
The frequency of the simple harmonic motion 5 5 5
∴ cos θ = = = θ = cos–1 =
∴ n = 10 Hz 55 11 11

13. (b) According to the question, 15. (c) According to the question,
µ1 1
v∝
m1 x

)
F=α t
k

be
m2 or v= ...(i)
µ2 x
1
FBD of lower block of mass m2, dv d K − 1 − 2 − 1 dx
= ⋅ =k ⋅ ⋅x ⋅

Tu
N1 =m1g dt dt x 2 dt
3
k − 2 k  dx
from Eq.(i)
k
fr1 a =− ⋅x ⋅ Q = v and v =
x  dt 

ou
2 x
fr 2 − k2 1
= ⋅ 2
2 x

(Y
N2
∴Force, F = Mass × |Acceleration |
m2 g
k2 1
∴ N 2 = m2 g + N1 = (m1 + m2) g =m ⋅ 2 (magnitude)
2 x
on
QLower block never moves.
k2 m
∴ fr2 ≥ fr1 F= ⋅
2 x2
µ 2 N 2 ≥ µ 1 N1 ⇒ µ 2(m1 + m2) g ≥ µ 1 m1 g
pi

1
m1 + m2 µ 1 ∴ F∝ 2
⇒ ≥ x
m1 µ2
m

µ1 m 16. (c) QEscape velocity from a planet,


∴ ≤1 + 2
µ2 2GM 2GM
m1 ve = =
ha

R
R R2
µ1 m2
∴ =1 + = 2gR ...(i)
µ2 m1
C

max
According due to gravity
14. (a) C 4
G πR 3 ⋅ρ
GM 3 4
g= = = GπRρ
dy

R2 R2 3
θ 3g
A B ∴ Radius R = ...(ii)
u

4πGρ
$ and AC = $i + 2$j + k
AB = 3$i + $j + k $
St

From Eqs. (i) and (ii), we get


∴ $)
BA = − (3$i + $j + k 3g 3 g2
ve = 2g ⋅ =
∠ABC is angle between BA and BC 4πGρ 2 π Gρ
g
Thus, ve ∝
ρ
θ
A B ve1 g1 ρ 5 1
∴ = × 2 = ×
BC = AC + BA ve 2 ρ1 g2 2 2
= AC − AB (Q BA = − AB) g1 5 ρ
$ $ ) = − 2$i + $j (given, = and 1 = 2 : 1)
= i + 2 j + k − (3i + j + k
$ $ $ $
g2 2 ρ2
∴ BA ⋅ BC =| BA|| BC|cosθ 5
$ ) ⋅ (− 2$i + $j) =
− (3$i + $j + k 2 2
20 WB JEE (Engineering) Solved Paper 2018

17. (c) According to the question, 19. (b) Terminal velocity, v = 2 r 2 (ρ − σ) g


9 η
time period, T ∝ r ρ S
a b c

Neglecting buoyancy effect of the fluid,


T = kr aρbS c ...(i) 2 ρ
v = ⋅ r2g
Thus, putting dimension, we get 9 η
[T] = [L]a [ML− 3]b [MT− 2]c Putting the given values, we get
[T] = [M]b + c ⋅[L]a − 3b ⋅[T]− 2c 2 103 × (0.9 ×10− 3)2
v= × × 9.8 = 98 ms −1
Equating the dimensions of both sides, we get 9 1.8 × 10− 5

)
b + c = 0, a − 3b = 0

be
20. (c) Let the mass of ice = m
and − 2c = 1
1 Applying calorimetry principle,
∴ c =−
heat given = heat taken

Tu
2
mL
∴ b=
1 (m1 + m2) s1 (t1 − t) = + ms 2(t − t2)
2 2

ou
3 Putting values, we get
and a = 3b =
m
2 (10 + 50) × 1 × (15 − 0) = × 80 + m × 0.5[0 − (−10)]
Putting these value of a, b and c into Eq. (i), we get 2

(Y
m
3 1

1
⇒ 60 ×15= 40 m + ×10 = 45m
T = k⋅ r 2ρ 2 ⋅ S 2 2
ρ 3 60 × 15
=k r ∴ m= = 20 g
on
S 45

18. (d) QStress, F = 1% of Y = Y 21. (c) Heat capacity of an ideal gas in a


∆A thermodynamic process,
pi

100
F QIdeal gas is monoatomic,
stress ∆A f R 3R
m

But Young’s modulus, Y = = CV = =


strain ∆l 2 2
l p dV
ha

Cprocess = CV + ⋅
Y n dT
∴ Y = 100  putting F = Y  3 p V
∆l   = R+ ⋅ 0
 ∆A 100 
C

2 n 2T
l 3 nRT
∆l 1 ∴ Cprocess = R + [Q pV = nRT]
∴ = 2 n2T
dy

l 100 3 R
∆r = R + = 2R
− 2 2
Poisson’s ratio, σ = r
u

∆l 22. (b) In p-V diagram, the slope of an adiabatic curve


St

l at any point in steeper than that of isothermal


∆r ∆l − 0.3 curve at that point.
∴ =−σ⋅ =
r l 100
∆r − 0.3 C ( pf,Vi )
= pf
r 100
Adiabatic
∆ V 2∆ r ∆ l compression
∴Change in volume, = + pi A
V r l (pi,Vi)
2 × (− 0.3) 1 p
= +
100 100 p2 B
1 − 0.6 0.4 Isothermal
= = expansion
100 100
O Vi V0
∴ ∆V % = 0.4% V
Thus, p f > pi
WB JEE (Engineering) Solved Paper 2018 21

23. (b) A charged circular ring of radius R is shown 25. (c) E D


in figure.
EB EA
C
d 60°
r +Q +Q
P F EC O EF C
O x
30°

)
From the figure, d = x 2 + r 2 …(i) A+Q M B+Q

be
Q/2
∴ Potential due to a uniform ring of positive charge
q at point P, (Regular hexagon)
1 q AM
In ∆AOM , sin 30° =

Tu
V= ⋅
4 πε0 d AO
1 q a
= ⋅ [from Eq. (i)]
∴ AO = 2 = a

ou
4 πε0 x + r2
2
1/ 2
Now, For maximum electric field at centre O charges
C

(Y
should be placed at F, A , B and C.
r QElectric field due to charges at F and C is equal and
A B opposite at O.
O ∴Net electric field at centre O due to charges at A
on
and B.
4 Angle between E A and E B is 60°.
—r 3
3 —r ∴E net at O,
pi

4
2 E net = E 2A + E B2 + 2E AE B cos 60° (QE A = E B = E)
OC 2 + OB2 = r 2 +   r 2 = r
3 5
∴ CB =
m

 4 4 1 Qcos 60° = 1 
= E 2 + E 2 + 2E 2 ⋅  
5 2  2
ha

Similarly, AC = r
3 1 Q
=E 3= ⋅ 3
∴ Work done in bringing a point charge − q from A 4 πε0 a 2
C

to B,
W = − q(VB − V A) 26. (b) QWhen a charged particle moves at a circular
path in uniform magnetic field, it time period is
 1 q 1 q 
= − q ⋅ − ⋅ 
dy

independent from its speed.


 4 πε 0 CB 4 πε 0 CA  ∴Time period of proton will not change with change
  in its speed.
u

−1 1 1 
= ⋅ q2 −  27. (c) QMagnetic field due to a hollow cylinder of
4 πε0 5 5
St

 r r radius r.
4 3 
− 1 q2 1 1 q2
= ⋅ [4 − 3] = − ⋅ ⋅
4 πε0 5r 5 4 πε0 r

24. (a) For surface ABCD electric flux is zero. Because


at surface ABCD net electric field is zero.
Using Gauss’s law,
Q
∫ E⋅ dS = εin0 O r A

Electric flux through the five faces of the cube, Binside = 0 (for d < r)
φE =
Q µ i
Boutside = 0 ⋅ (for d ≥ r)
ε0 2π d
22 WB JEE (Engineering) Solved Paper 2018
So, graph between B and d. 1 q q
Fcentripetal = mrω2 = ⋅ 1 2
4 πε0 r 2
1 q q q1 q 2
1
B∝— ∴ ω2 = ⋅ 1 2 ⇒ ω=
B r 4 πε0 m r 3 4 πε0 mr 3
1
O R ω∝ ...(i)
r d r 3/ 2
Current produced due to moving q1 ,
28. (b) ω
i=

)
q1 ...(ii)

be
O r
Thus, magnetic field due to motion of q1 in circular
path at point O
µ i µ ωq

Tu
B= 0 ⋅ = 0 ⋅ 1 [using Eq. (ii)]
I 2 r 2 2πr
µ I ω
B= 0 ⋅ Hence, B ∝
2 r

ou
r
V
Q I1 = 1
Magnetic field B ∝ 5/ 2 [using Eq. (i)]
R1 r
Case (I) l1 = 2πr (length of the loop),
l µ I
R1 = ρ 1 and B1 = 0 ⋅ 1
A 2 r
...(i) (Y
32. (a)
b
B2
l
E2=vB2l

v=constant
on
Case (II) l 2 = 2π ⋅ 2r = 2l1 B1
l 2l E1=vB1l
R2 = ρ ⋅ 2 = ρ ⋅ 1 = 2R1 y
A A
pi

V V I
I2 = = = 1
m

R2 2R1 2 I
µ I µ I ∴Motional emf in the loop,
B2 = 0 ⋅ 2 = 0 ⋅ 1 ...(ii)
2 2 ⋅ 2r
ha

2 2r E = E1 − E 2 = vl(B1 − B2)
From Eqs. (i) and (ii), µ I µ 0I 
= vl  0 ⋅ −
µ0

I1
 2π y 2 π( y + b) 
C

B2 2 2⋅ 2r 1
= = µ  y+ b − y µ 0 vlI
µ0 = 0 ⋅ I ⋅ vl   = 2π ⋅ y(y + b) b
B1 I 4
⋅ 1
2 2π  y( y + b) 
dy

r
Magnetic field B2 =
B1 Q b << y
4 µ vlI
∴ E= 0 ⋅ 2 b
u

29. (d) QResonance frequency, f0 = f1 f2 2π y


St

Qv = constant (∴ y = vt)
= 200 × 800 = 400 Hz
µ vlI
Thus, E= 0 ⋅ 2 2 b
30. (c) Initially, there will be no voltage drop across 2π v t
capacitor, so intensity of bulb will rise sharply and 1
gradually voltage drop across capacitor will Hence, E∝ 2
t
increase as a result voltage drop across bulb
decreases, so intensity of bulb will decreases. 33. (d) Solid sphere Hollow sphere
ρ1 ρ2
31. (d)
q1 OA=R O′B=R
r A O B O′ O′C=r
O
q2 C
WB JEE (Engineering) Solved Paper 2018 23
4 3 4
∴ M1 = ρ1 πR and M 2 = ρ2 π( R3 − r 3) 35. (d)
3 3
4 4 σ qE
∴ ρ1 πR3 = ρ2 π(R3 − r 3) E= —
ε0 m
3 3
mg
⇒ ρ1 R3 = ρ2(R3 − r 3) + + + + + + + +
ρ1 r3 σ
⇒ =1 − 3
ρ2 R ∴Apparent weight of the bob, w′ = mg − qE
1 mg′ = mg − qE

)
r  ρ 3
∴ = 1 − 1  ...(i) qE

be
R  ρ2  ∴ g′ = g −
m
Moment of inertia, QTime period of a pendulum,

Tu
2  R5 − r 5  L
M2  3 3
T = 2π
IH
=
5 R − r  geff
IS 2
M1 R2 L L

ou
5 = 2π = 2π
g−
qE  qσ 
4  R5 − r 5  g− 
ρ2 π (R3 − r 3)  3 3 
m  ε 0 m
 (R − r ) 

(Y
3
=
4 3 2 36. (b, c, d)
ρ1 πR R

30

30
3

0
0

20
20


ρ2  R5 − r 5  ρ2   r  


5
on
=  I3
 = 1 −   
ρ1  R  ρ1 
5
R  K IG

 5
pi

I H ρ2   ρ 3

10
G
1 − 1 − 1  
10

∴ = G

0
0

ρ1   ρ2  


IS
m

 

34. (a) A
ha

10 V 10 V
Moving each other 10
∴ IG =
100 + G
C

B
10
Plates of a charged capacitor 300 I 3 = G
moving toward each other 100 + G
dy

due to electrostatic attraction.


 1 
G
1 σ I3 = 100 + G 
Force = ⋅ q 30
2 ε0  
u

 (300 + G)
10 +  =
1 G
∴Relative acceleration of plates, ∴ I3 + IG =
St

2F (100 + G)  30  (100 + G) × 30
a rel =
M According to the problem,
Potential difference, V = Ed  200 300 G   (300 + G) 
...(i) 10 =  + 
1  3 300 + G   (100 + G) 30 
where, d = d0 − a rel t 2 (Qplates are moving)
2 [Q V = RI]
60000 + 1100 G 1 1

V = E d0 − a rel t 2 
1 10 = × ×

 2  3 100 + G 30
This is a equation of a parabola with downward ⇒ 900(100 + G) = 60000 + 1100 G
concavity. ⇒ 30000 = 200 G
∴ G = 150 Ω
24 WB JEE (Engineering) Solved Paper 2018

IG =
10
=
10
= 40 mA ∴Partial reflection and refraction at Q.
100 + 150 250 As SP || QR , emergent ray at R is parallel to incident
200 300 ray at S.
As = ∴Net deviation =180°.
100 150
So, I 200 = I 300 38. (a, d) The intensity of a sound source appears to
be periodic due to
37. (a, c) µ=√2
(i) source intensity is periodic.
(ii) source is producing a sound composed of two
nearby frequencies.

)
be
R 45° 39. (b) QInternal energy of an ideal gas depends only
on the temperature of gas.
30°
E in ∝ T

Tu
Q 15° 15°
15° 45° Q 4Q
40. (a) P B
75° 75° 30° S A r
15°

ou
P d
QVP is minimum.
Critical angle θC = sin−1   = 45° ∴ EP = 0 ⇒ E A = EB
1

(Y
 2 1 Q 1 4Q
⇒ ⋅ = ⋅
Snell’s law at points, (Qn1 sin θ1 = n2 sin θ2) 4 πε0 r 2 4 πε0 (d − r)2
1 sin 45° = 2 sinθ 1 2
⇒ = ⇒ d − r = 2r
on
∴ θ= 30° r d− r
At PQincidence angle at P = 75°> θC . d
∴ Distance of P from the A r = units to the right of A.
∴ TIR at point P. 3
pi

At Q Q incidence angle at Q=15° < θC .


m

Chemistry
ha

41. (d) Cl 2O 7 is the anhydride of HClO 4 . It reacts with The hydrolysis of SiCl 4 occurs due to coordination
of OH with empty 3d-orbitals of Si-atom of SiCl 4
water slowly to give perchloric acid.
molecule.
C

Cl 2O 7 + H 2O → 2HClO 4 OH
Perchloric acid
Cl Cl Cl Cl Cl OH
Its structure is as follows: 2H2O
dy

Si Si Si
O O O –2H+ –2Cl–
Cl Cl Cl Cl Cl OH
Cl 117.6° Cl OH 2H2O
O O O O –2H+
u

It is less reactive than other oxides of chlorine and OH


St

does not react with P, S, coal or paper at room HO OH Cl OH


temperature. Si Si
–2Cl–
HO OH Cl OH
42. (d) The main reason that SiCl 4 is easily OH
hydrolysed as compared to CCl 4 is that Si can
extend its coordination number beyond four 43. (d) Silver chloride dissolves in excess of
because it has vacant d-orbital. On the other hand, ammonium hydroxide solution. The cation
carbon has no d-orbital thus, it cannot extend its present in the resulting solution is [Ag (NH 3)2]+
coordination number beyond four, so its halides In aqueous solution, silver chloride exist as Ag+
are not attacked (hydrolysed) by water. The
vacant d-orbitals of Si can coordinate with water and Cl − . Ag + present in solution reacts with NH 3
molecules and hence their halides are hydrolysed to form [Ag (NH 3)2]+ .
by water. Ag+ Cl − + 2NH 3 → [Ag(NH 3 )2 ]+ Cl –
WB JEE (Engineering) Solved Paper 2018 25

44. (a) The ease of hydrolysis of carbonyl compounds Step-I It involves the formation of an amide on
depend on the group attached to the carbonyl carbon. reaction of ammonia and acetyl chloride so, M must be
Among the given options, chlorine (— Cl) group is O
electron withdrawing group which makes the
carbonyl group electrophilic. Hence, hydrolysis can C
Cl
occur most easily. The order for electron withdrawing
tendency among given options is as follows: Cl
 Cl >  OCOCH 3 >  OC2 H 5 >  NH 2
CH3
So, the correct order for the ease of hydrolysis in

)
the given compounds is as follows: Step-II It involves the Hofmann-bromamide

be
reaction.
CH 3 COCl > CH 3 COOCOCH 3
I II
CH3

> CH 3 COOC2 H 5 > CH 3 CONH 2 Cl

Tu
Br2
III IV C8H8ClNO NaOH
45. (d) CH 3  C ≡≡ CMg Br can be prepared by the H2N
reaction of CH 3  C ≡≡ CH with CH 3 MgBr. This

ou
In this reaction, migration of an alkyl or aryl group
method is used in preparation of higher alkynes
from lower alkynes. The chemical equation for the takes place from carbonyl carbon of the amide to
formation of CH 3  C ≡≡ CMgBr is given below the N-atom. So, C8 H 8 Cl NO must be an amide with

CH 3  C ≡≡ CH +CH 3 MgBr →
− 2+ −

− 2+
CH 3  C ≡≡ CMg X −
(Y structural formula.
CH3
Cl
on
46. (b) The number of alkene(s) that can produce H2N
2-butanol by the successive treatment of B2 H 6 in C
tetrahydrofuran solvent and alkaline H 2O 2 solution
pi

O
is 2. The alkenes are cis-but -2- ene and
trans-but-2-ene. The hydration product is obtained 48. (b) Methoxybenzene on treatment with HI
m

in accordance with opposite Markownikoff’s rule. produces phenol and methyl iodide. This is due
In this reaction, addition takes place through the the more stable aryl oxygen bond. Here,
ha

initial formation of π-complex which changes into methyl phenyl oxonium ion is formed by the
a cyclic four centre transition state with the protonation of ether. The bond between O  CH 3
addition of boron atom to the less hindered carbon is weaker than the bond between O  C6 H 5
C

atom. because the carbon of phenyl group is


(i) B2H6 sp 2 -hybridised and there is a partial double bond
H3C HC CH CH3 – CH3 CH CH2CH3
(ii) H2O2/OH character. Thus, the reaction yields phenol and
dy

But-2-ene
OH alkyl halide.
Butan-2-ol OCH3 OH
u

47. (b) Given sequence of chemical equation is 373K


CH3 + HI + CH3I
St

Methyl
Cl Anisole Phenol Iodide
NH3 Br2
C8H6Cl2O (Step I) C8H8ClNO NaOH (Methoxy benzene)
(M) (Step II) H N
2
49. (b) Given sequence of chemical equation is
The structural formula of C8 H 6 Cl 2 O( M ) will be
K2Cr2O7 I2/NaOH
O C4H10O C4H8O CHI3
H2SO4 Warm
(N) (step I) (step II)
C
Cl OH
Cl Here, N will be (IUPAC name = Butan-2-ol)
CH3 Step I It involves the oxidation of secondary
alcohol to corresponding alcohol in the presence of
The various step involves in the above road map can
K 2 Cr2 O7 and H 2 SO4 .
be explain as follows
26 WB JEE (Engineering) Solved Paper 2018

OH K2Cr2O7 O 52. (c) n = 2, l=0, m= – 1 is not a possible electronic


configuration. With n = 2 and l=0, the orbital is 2s
H2SO4
having only one ml value i.e. 0. As the value of
[C4H10O] [C4H8O] ml = − l to + l
(N )
Other options are correct.
Step II It involves the iodoform reaction. Ketone
formed in step I have one methyl group linked to Orbital
the carbonyl carbon atom are oxidised by sodium n = 3, l = 0, m = 0 3s
hypohalite to sodium salts of corresponding

)
carboxylic acids having one carbon atom less than n = 3, l = 1, m = – 1 3p

be
that of carbonyl compound. n = 2, l = 1, m = 0 2p
O I2/NaOH O

O–Na+ + CHI3
53. (b) The number of unpaired electrons in Ni

Tu
[C4H8O] (Yellow ppt.)
(atomic number=28) are 2. It can be easily
concluded from the electronic configuration of Ni.
50. (a) The correct order of reactivity for the addition Electronic configuration of

ou
reaction of the given carbonyl compounds with Ni=1s 2 2 s 2 2p 6 3s 2 3p 6 4s 2 3d 8
ethylmagnesium iodide is I > III > II >IV.
3d8
H H H3C
Orbitals of Ni in 3d =

(Y
=O >
C= C= =O > C==O
H H3C H3C Unpaired electrons
(I) (III) (II)
(CH3)3C 54. (c) The strongest H-bond is FH------ F because
on
> C==O in this case, a hydrogen is bonded to a most
(CH3)3C electronegative atom i.e. fluorine. In all other
(IV) options, hydrogen is bonded to oxygen and
pi

sulphur that are less electronegative atom than


This order can be explained on the basis of
fluorine.
following two factors:
m

(i) Inductive effect Greater the number of alkyl The correct electronegativity order of elements are
(electron releasing) groups attached to carbonyl as follows: F > O > S
ha

group, greater will be the electron density on


55. (d) Given,
carbonyl carbon. Thus, it lowers the attack of
nucleophile and hence, reactivity decreases. Half-life of C14 , t1 / 2 = 5760 years
C

(ii) Steric effect As the number of alkyl group Initial concentration of sample of C14 , N 0 = 200 mg
attached to carbonyl carbon increases, the attack
of nucleophile on carbonyl group becomes more Final concentration of sample of C14 , N t = 25mg
dy

and more difficult due to steric hinderance. The given decay is radioactive and all radioactive
51. (d) When aniline is treated with conc. H 2SO 4 decay follows first order kinetics.
0.693
t1 / 2 =
u

followed by heating at 200° C, the product obtained is


sulphanilic acid i.e. p-aminobenzene sulphonic acid λ
St

+ 0.693 0.693 −1
NH2 NH3HSO4
– ∴ λ= = yr
t1 / 2 5760
We know that, for first order reaction
+ Conc. H2SO4 2.303 N .
2303 [200]
t= log 0 = log
Aniline Anilinium hydrogen λ N t 0.693 [25]
sulphate 5760
+
NH3 NH2 .
2303 × 5760
= log 8
0.693
200°C
a =17, 286. 78 yrs ≈17, 280 yrs
Therefore, the time taken to change 200 mg to
– 25 mg is 17, 286.78 yr, which is very close to option
SO3 SO3H
Zwitter ion Sulphanilic acid
(d) i.e. 17280 yrs.
WB JEE (Engineering) Solved Paper 2018 27
C2H 6 H2
Alternative method Initial gram weight = w g wg
200 mg sample
w w
t1/2 = 5760 years Number of moles =
30 2
100 mg According to Henry’s law,
w
t1/2 = 5760 years pH 2 nH 2 2
17280 years = χH 2 = =
ptotal nH 2 + nC2 H 6 w
+
w
50 mg 2 30
w
pH 2

)
t1/2 = 5760 years 2 w 30 15
= = × =

be
25 mg ptotal 15w + w 2 16 w 16
30
56. (d) Ferric ion forms a prussian blue precipitate So, the fraction of total pressure exerted by

Tu
due to the formation of Fe4 [Fe(CN)6]3. This hydrogen is 15 : 16.
complex is formed during the determine action of
presence of nitrogen in the given sample. In this 60. (a) Given, the heat of neutralisation of a strong
base and a strong acid is 13.7 kcal.

ou
method, to portion of sodium fusion extract,
freshly prepared ferrous sulphate, FeSO 4 solution The reaction of neutralisation is as follows
is added and warmed. Then about 2 to 3 drops of HCl + NaOH → NaCl + H 2O; ∆H = − 137 . kcal
FeCl 3 solution are added and acidified with

(Y
1 mol 1 mol
conc. HCl. The appearance of a prussian blue According to question,
colour indicate the presence of nitrogen.
HCl + NaOH → NaCl + H 2O … (i)
FeSO 4 + 2NaOH → Fe(OH)2 + Na 2SO 4
on
0.6 mol 0.25 mol
6NaCN + Fe(OH)2 → Na 4 [Fe(CN)6] + 2NaOH
Sodium ferrocyanide
In equation (i), NaOH acts as a limiting reagent. For
1 mole of NaOH and 1 mole of HCl, heat of
3 Na 4 [Fe (CN)6] + 4FeCl 3 → neutralisation = 137
pi

. kcal.
Fe4 [Fe (CN)6]3 +12NaCl ∴For 0.25 mole of NaOH and 0.6 mole of HCl, heat
Ferric ferrocyanide of neutralisation = 137
. × 0.25 ⇒ 3.425 kcal
m

(prussian blue)
61. (a) Number of X atoms at the corners = 8
57. (c) The nucleus 64
ha

29 Cu accepts an orbital electron 1


Number of X atoms per unit cell = 8 × = 1 atom
to yield 64
28 Ni. The atomic number of Cu is 29, 8
which is equal to the number of electrons and also Number of Y atoms at the centre of the body = 1 atom
C

equal to the number of protons. Hence, the formula of the compound is XY.
When 64 29 Cu accepts an orbital electron then
62. (c) The number of electrons involved in the
dy

electrons subtract from the atomic number, i.e.


reaction are three as shown below
29 −1 = 28
64 Cu + e 0 → 64 Ni Al 3+ + 3e − → Al
u

29 −1 28 It means the conversion of every aluminium ion to


58. (d) Mass of an electron = 9.108 ×10−31 kg
St

aluminium atom requires three electrons.


Mass of one mole of electron Therefore, the amount of electricity required for
= (9.108 × 10−31 × 6.023 × 1023) kg one mole of Al 3+ ions = 3F.

Then, number of mole of electron in 1 kg 63. (b) Given,


1
= Zn → Zn2+ + 2e − ; E° = + 0.76 V … (i)
. × 6.023×10−8
9108 −
Fe → Fe 2+
+ 2e ; E° = + 0.41 V … (ii)
1
= ×108 mole of e − On reversing the above equation (i) and (ii), we get
. × 6.023
9108
Zn2+ + 2e − → Zn ; E° = − 0.76 V
59. (d) Given, equal weights of ethane and hydrogen
are mixed in an empty container at 25°C. Fe2+ + 2e − → Fe; E° = − 0.41 V
[where, E° = standard reduction potential]
28 WB JEE (Engineering) Solved Paper 2018
To find, the standard emf of the cell with the 66. (b) H 2SO 4 (sulphuric acid) is present in maximum
reaction.
Oxidation (anode)
amount in ‘acid rain’. Oxides of nitrogen and
sulphur, released into the atmosphere from
2+ 2+
Fe + Zn Zn + Fe thermal power plants, industries and automobiles
Reduction (cathode)
are the main sources of acid rain. These oxides on
°
oxidation followed by hydrolysis give sulphuric
So, E cell = E° − E° acid and nitric acid that alongwith HCl are
Fe2+ / Fe Zn 2+ / Zn
responsible for the acidity of rain. The oxidation
= − 0.41 V + 0.76 V = + 0.35V
reaction is catalysed by particulate matter present

)
64. (b) Given, in the polluted atmosphere.

be
[NH 3]2 l → 2H 2SO 4 (aq)
2SO 2(g) + O 2(g) + 2H 2O()
N 2 + 3H 2 c 2NH 3; K 1 = … (i)
[N 2] [H 2]3 4NO 2(g) + O 2(g) + 2H 2O()l → 4HNO 3(aq)

Tu
[NO]2 67. (b) The correct set of oxides arranged in the
N2 + O 2 c 2NO; K 2 = … (ii)
[N 2] [O 2] proper order of basic, amphoteric, acidic are
BaO, Al 2O 3 , SO 2.
1 [H 2O]

ou
H2 + O 2 c H 2O; K 3 = … (iii) Oxides of non-metallic elements are acidic such as
2 [H 2] [O 2]1 / 2
CO 2, NO 2 , SO 2 etc. Oxides of less electropositive
The chemical equation for the oxidation of 2 mol of elements (such as BeO, Al 2O 3 , Bi 2O 3, ZnO etc.) are

(Y
NH 3 to give NO is amphoteric i.e. these behaves as acids toward strong
5 bases and as bases towards strong acids. Oxides of
2NH 3 + O 2 2NO + 3H 2O
c … (iv)
2 electropositive elements (Na 2O, CaO, Tl 2O, BaO
etc.) are basic and contain discrete O 2− ions.
on
To get the equation (iv) from equation (i), (ii) and
(iii) following steps are followed: 68. (b) Out of the given outer electronic configuration
Reversing equation (i), we get of atoms, the highest oxidation state is achieved
pi

2NH 3 N 2 + 3H 2 so K ′ =
1
… (v) by (n − 1)d 5ns 2 i.e. 7.
c K1 A large number of oxidation state is due to the fact
m

Multiplying equation (iii) by 3, we get the (n − 1)d-electrons may get involved along with ns
3 electrons in bonding as electrons in (n − 1) d- orbitals
3H 2 + O 2c 3H 2O so K ′ = K 33 … (vi)
ha

2 are in an energy state comparable to ns -electrons.


Adding equation (ii) and (vi) Oxidation state of other options are as follows:
5
N 2 + 3H 2 + O 2 2NO + 3H 2O
C

c Electronic configuration Oxidation state


2
(n − 1) d ns
8 2
+ 2, + 3, + 4
so, K ′ = K 2 . K 33 … (vii)
dy

(n − 1) d ns
3 2
+ 2, + 3, + 4, + 5
On combining (v) and (vii), we get the required
K3 (n −1) d ns
5 1
+ 2, + 3, + 4, + 5, + 6
equation having equilibrium constant K ′ = K 2 . 3 .
u

K1
69. (c) At room temperature, the reaction between
St

65. (c) Dacron is a condensation polymer. It is also water and fluorine produces F − , O 2 and H + .
known as terylene. It is a polymer obtained by Fluorine, being non-polar molecule, readily
condensation reaction between ethylene glycol dissolves with water and forms mixture of oxygen
and terephthalic acid at 420-460 K in the presence and ozone as shown below
of zinc acetate-antimony trioxide catalyst.
2F2 + 2H 2O → 4HF + O 2
nHOH2C — H2COH + nHOOC— —COOH 3F2 + 3H 2O → 6HF + O 3
Ethylene glycol HF exist as H + and F − in a solution.
Terephthalic
acid
O O 70. (d) BeCO 3 is least thermally stable. The thermal
stability of carbonates increases down the group
420-460K
—O—CH2CH2—O—C— —C— i.e from Be to Ba.
–nH2O
Dacron or Terylene
n BeCO 3 < MgCO 3 < CaCO 3 < SrCO 3 < BaCO 3
(523 K) (813 K) (1173 K) (1562 K) (1633 K)
WB JEE (Engineering) Solved Paper 2018 29
BeCO 3 is unstable to the extent that it is stable only 73. (b) Given, specific heat = 0.16
in atmosphere of CO 2. These carbonates show
reversible decomposition in closed container. Let metal chloride be MCl x then,
6.4
MCO 3 c
MO + CO 2 = Atomic weight of metal
More stable the oxide is formed, lesser will be specific heat
stability of carbonates. 6.4
= Atomic weight of metal
016
.
71. (a) Given reactions are Atomic weight = 40
Br NaNH
[P] →
2
C2H 4Br2 →
2
[Q] 40 is the atomic weight of calcium. According to

)
NH 3
question, metal chloride (MCl x ) have ≈ 65% chlorine

be
20%/ H 2SO4 Zn -Hg / HCl present in it.
[Q] → [R] → [S]
Hg 2 + , ∆ x × Atomic weight of chlorine
× 100 = 65
40 + x × Atomic weight of chlorine

Tu
[P] is an alkene with two carbon atoms i.e. ethene.
When it reacts with Br2, addition reaction occurs x × 355
.
× 100 = 65
and C2H 4Br2 is formed. 40 + x × 355
.

ou
H H
H H x = 209
. ≈ 2(approx.)
C=
=C + Br2 H—C—C—H So, the formula of metal chloride will be MCl 2.
H

(Y
H 74. (a) For a reversible adiabatic process,
Ethene [P] Br Br
(C2H4) 1,2-dibromoethane γ
(C2H4Br2)
pT 1 − γ = constant … (i)
C2H 4Br2 (1, 2-dibromoethane) on reaction with
on
According to question, during a reversible adiabatic
NaNH 2 / NH 3 gives ethyne (C2H 2).
NaNH 2 / NH 3
process the pressure of a gas is found to be
H 2 C  CH 2 → HC ≡≡ CH proportional to the cube of its absolute
pi

  Ethyne [Q]
(C2 H 2 )
temperature.
Br Br p ∝ T3
m

Ethyne [Q] in presence of 20% H 2SO 4 , Hg 2+ at 333 K


pT −3 = constant …(ii)
gives ethanal.
Equating equation (i) and (ii), we get
ha

Hg2 + / H +
CH ≡≡ CH + H  OH → CH 2 == C  H γ
=−3
333 K
 1−γ
OH
C

Tautomerisation γ = − 3 (1 − γ)
→ CH 3CHO
Ethanal [ R] γ = − 3 + 3γ
dy

Ethanal [R] undergoes reduction in presence of − 2γ = − 3


Zn-Hg / HCl to give ethane [S] 3
γ=
Zn- Hg/ HCl 2
u

CH 3CHO → CH 3  CH 3
Ethanal Clemmensen Ethane
As we know, the ratio of molar heat capacities at
St

[ R] reduction [ S] constant pressure and constant volume is


C 3
72. (a) The number of possible organobromine represented by γ. So, the ratio of p for the gas is .
CV 2
compounds which can be obtained in the allylic
bromination of but-l-ene with N-bromosuccinimide 75. (a) According to question,
is 1.
NBS
[X] + Dil. H 2SO 4 → [ Y ] :
CH2 CH CH2 CH3 CH2 CH CH CH3 Colourless, suffocating gas
But-1-ene Br
[Y ] + K 2Cr2O 7 + H 2SO 4 →
CH2 CH CH CH3 Green colouration of solution.
Br All sulphites when treated with dil. H 2SO 4 gives
3-bromobut-1-ene colourless and suffocating sulphur dioxide gas.
30 WB JEE (Engineering) Solved Paper 2018
SO 23− (s) + Dil.H 2SO 4 (aq) → SO 42− (aq) + SO 2(g) In next step, Br − attacks a carbon atom of the
Sulphite ion Sulphur bromonium ion. This release the strain in the three
[ X] dioxide[ Y ]
membered ring and form a vicinal dibromide.
+ H 2O(l) + CH3 CH3
SO 2 gas [Y ] turns acidified potassium dichromate Br
paper green due to reduction of Cr (VI) to Cr (III). H3C H H—C—Br Br—C—H
s
K 2Cr2O 7(aq) + H 2SO 4 (aq) + 3SO 2(g) → C—C + Br +
[Y] H C2H5 H—C—Br Br—C—H
K 2SO 4 (aq) +Cr2(SO 4)3(aq) + H 2O()
l
C2H5 C2H5

)
Green

be
76. (a, c) The possible product(s) to be obtained from
the reaction of cyclobutyl amine with HNO 2 is/are 78. (b, c) The correct statements for peroxide ion
OH (O 2−
2 ) are that it is diamagnetic and it has bond
order one.

Tu
cyclobutanol = and cyclopropylmethanol
Electronic configuration of O 2−
2 (peroxide ion) is
CH2OH
= . The chemical reactions involved σ1s 2 , σ *1s 2 , σ 2s 2 , σ * 2s 2 , σ 2pz2 , [π 2px2 = π 2p 2y ],

ou
are as follows: [π * 2px2 = π * 2p 2y ], σ * 2pz0
NH2
r QBond order =

(Y
HNO2
N2 No. of electrons present in bonding
–N2 − No. of electrons present in non - bonding
Cyclobutylamine r 2
on
10 − 8 2
r CH2
Ring
∴ BO = = =1
contraction 2 2
Number of unpaired electrons = 0
pi

OH So, O 2−
2 is diamagnetic in nature.
CH2OH
m

79. (a, c, d) Among the given options, the extensive


Cyclobutanol Cyclobutyl variables are H (enthalpy), E (internal energy),
ha

methanol
V (volume).These variables have values that
77. (a, d) The major product(s) obtained in the given depends upon the quantity or size of matter
reaction are present in the system. Other examples are heat
C

CH3 CH3 capacity, entropy, free energy, length and mass.


H3C H H Br Br H 80. (a, c, d) White phosphorus (P4) has 6P  P single
dy

C=
=C + Br2 +
H Br Br H bonds, 4 lone pairs of electrons and P  P  P
H C2H5
C2H5 C2H5
angle of 60°. It is a translucent white waxy solid.
u

It has large atomic size and less electronegativity.


Enantiomers So, it forms single bond instead of pπ - pπ multiple
St

The mechanism for the above given reaction is as bond. It consists of discrete tetrahedral P4
follows. When the π-electrons of the alkene approach molecule as shown in the figure.
a molecule of Br2, one of the bromine atom accepts
them and releases the shared electrons to the other P
bromine atom. A cyclic bromonium ion is formed. 60°
Br+ P P
H3C H H3C H
C=
=C + Br—Br C———C
H C2H5 H C2H5 P
(Cyclic bromonium ion)
WB JEE (Engineering) Solved Paper 2018 31

Mathematics
1. (a) We know that, sin 30° = 1 = 0.5 4. (c) We have, f : [a , b] → R be differentiable
2
on [a , b] and k ∈ R, also f (a) = 0 = f (b)
In 1st quadrant sin x is increasing function.
and J(x) = f '(x) + kf (x)
∴ sin 31° > sin 30°
Let g(x) = kxf (x) which is continuous in [a , b] and
⇒ sin 31° > 0.5 differentiable in (a , b) such that
2. (c) We have, f1 (x) = e x g(a) = 0 = g(b)

)
Then, for every c ∈ (a , b), g'(c) = 0

be
f2(x) = e f1 ( x )
(by Rolle’s theorem)
…… …… …… …… Now, g′(x) = kf (x) + kxf '(x)
…… …… …… ……

Tu
⇒ g'(c) = kf (c) + kcf '(c)
fn + 1 (x) = e f n( x ) ⇒ kf (c) + kcf '(c) = 0
Now, fn (x) = e f n−1 ( x ) ⇒ f (x) = 0,for every x = c ∈(a , b)

ou
On taking log both sides, we get ∴ J(x) = 0 has atleast one root in (a , b).

5. (c) We have,
log { fn (x)} = fn − 1 (x) log e

(Y
f (x) = 3x10 − 7 x 8 + 5x 6 − 21 x 3 + 3x 2 − 7
d d
⇒ log ( fn (x)) = fn − 1 (x) (Qlog e = 1) ∴ f (1 − h) = 31
( − h)10 − 71
( − h)8
dx dx
+ 51
( − h)6 − 211
( − h)3 + 31
( − h)2 − 7
on
1 d
⇒ fn (x) = fn′ − 1 (x)
fn (x) dx = 31
( − 10h + 45h2 − 120h3 + ...... + h10)


d
fn (x) = fn (x) fn′ − 1 (x) ... (i) − 71
( − 8h + 28h2 − 56h3 + ..... + h8)
pi

dx + 51
( − 6h + 15h2 − 20h3 + ..... + h6)
f1′ (x) = e x
− 211
( − 3h + 3h2 − h3)
m

Now,

and f2(x) = e f1 ( x )
+ 31
( − 2h + h2) − 7
ha

⇒ log f2(x) = f1 (x) log e = f1 (x) ⇒ f (1 − h) = − 24 + 53h + h2(−46) + h3(− 47) + ....
1 and 1 = − 24
f()
⇒ ⋅ f2′ (x) = f1′(x)
f (1 − h) − f ()
C

f2(x) 1
∴ lim
h→ 0 h3 + 3h
⇒ f2′(x) = f2(x) ⋅ f1′(x)
−24 + 53h + h2(− 46) + h3(− 47) + ... − ( − 24)
dy

= f2(x) ⋅ e x (Q f1′(x) = e x ) = lim


h→ 0 h(h2 + 3)
= f2(x) ⋅ f1 (x) [Q e x = f1 (x)] ...(ii) 53h + h2(− 46) + h3(− 47) + K
= lim
u

From Eq. (i), h→ 0 h(h2 + 3)


St

d 53 + h (− 46) + h2 (− 47) + ... 53


fn (x) = fn (x) ⋅ fn −1 (x) .... f1 (x) [using Eq. (ii)] = lim =
dx h→ 0 h2 + 3 3

6. (a) Let g(x) = e − x f (x)


3. (b) We have, f (x) = 1 − |x| such that g(a) = 0, g(b) = 0
2 − |x|
and g(x) is continuous and differentiable.
f (x) is defined for all x satisfying
Then, for atleast one value of c ∈(a , b) such that
1 − |x| |x| − 1
≥0 ⇒ ≥0 g'(c) = 0
2 − |x| |x| − 2
Now, g'(x) = e − x f '(x) + ( − e − x ) f (x)
⇒ |x|≤ 1or |x|> 2
⇒ g'(c) = e − c f '(c) + (− e − c) f (c) = 0
⇒ x ∈ [−1, 1]or x ∈ (−∞ , − 2) ∪ (2, ∞)
−c
⇒ x ∈ (−∞ , − 2) ∪ [−1, 1] ∪ (2, ∞) ⇒ e f '(c) = e − c f (c) ⇒ f '(c) = f (c)
32 WB JEE (Engineering) Solved Paper 2018
dt
7. (b) We have, Put 2x = t ⇒ dx =
2
f : R → R be a twice continuously differentiable π 2
function such that f (0) = f () 1 = f ′(0) = 0 sin t
Now, for atleast one value of c1 ∈(0, 1),
∴ N= ∫0 4( t 2 + 1)
2
dt

f ′(c1) = 0 (by Rolle's theorem) π 2 sin t


Again, f ′(0) = 0 = f ′(c1)
= ∫0 (t + 2)2
dt
⇒ f ′′(c) = 0 for some c ∈(0, c1) π 2 1
(by Rolle’s theorem) ∴ M−N= ∫0 (cos x) ⋅
(x + 2)
dx

)
8. (b) We have,

be
π 2 sin x
 x cos3 x − sin x 
− ∫0 (x + 2)2
dx
∫e   dx = esin x f (x) + c
sin x

 cos2 x  π 2
 sin x 

Tu
π 2 sin x
⇒ ∫e
sin x
(x cos x − sec x tan x) dx = e sin x
f (x) + c = 
 x + 2 0
− ∫0 −
(x + 2)2
dx

∫e
⇒ sin x
(x cos x − 1 + 1 − sec x tan x) dx sin x π 2
dx − ∫0

ou
(x + 2)2
= esin x f (x) + c
sin π 2 1 2
= = =
⇒ ∫ [esin x cos x(x − sec x) + esin x (1 − sec x tan x)] dx π 2+ 2 π + 4 π + 4


= esin x f (x) + c
d sin x
∫ dx {e (x − sec x)}dx = e f (x) + c
sin x (Y
11. (b) We have,
2
on
2014 tan−1 x
⇒ esin x (x − sec x) = esin x f (x) + c
I= ∫1 2014 x
dx ... (i)

⇒ f (x) = x − sec x 1
x=
pi

Let
t
9. (c) We have,
−1
⇒ dx =
m

1 dt
∫ f (x) sin x cos x dx = 2(b 2 − a 2) log( f (x)) + c t2
1 2014 tan−1 (1 t)  − 1 
I= ∫2014
ha

⇒ f (x)sin x cos x =
1

1
⋅ f ′(x)
Now,  2 dt 
1t  t 
2(b 2 − a 2) f (x)
f ′(x) cot −1 t
2014
⇒ f (x) sin 2x = 2
1
⋅ = ∫1 2014 dt
C

b − a 2 f (x) t
2014 cot −1 x
1 f ′(x) =∫ dx ... (ii)
⇒ sin 2x =
dy

1 2014 x
b 2 − a 2 ( f (x))2
On adding Eqs. (i) and (ii), we get
1 f ′(x)
⇒ ∫ sin 2x dx = b 2 − a 2 ∫ ( f (x))2 dx 2I = ∫
2014 π 2 π
dx = ( log x )
2014
u

1 2014
1 2014 x 2
− cos 2x  −1 
St

1 π
⇒ = 2 ⋅  = ( log 2014 − log1 2014)
2 b − a 2  f (x)  2
cos 2x (b 2 − a 2) 1 π  1 
⇒ = ⇒ I =  log 2014 − log 
2 f (x) 4  2014 

⇒ f (x) = 2
2 π
= (log 2014 + log 2014)
(b − a 2) cos 2x 4
π 2 π π
10. (d) Given, M = ∫ cos x dx = (2 log 2014) = log 2014
0 (x + 2) 4 2
π 4 sin x cos x
and N=∫ dx 12. (c) We have,
0 (x + 1)2 π 3 sin x
π / 41 sin 2x
I= ∫π 4 dx
=∫ ⋅ dx x
0 2 (x + 1)2
WB JEE (Engineering) Solved Paper 2018 33
sin x 16. (c) We have,
Since, is a decreasing function.
x dy
(1 + x 2) + 2xy − 4 x 2 = 0
π sin π 3 π sin π 4
∴ × ≤I≤ × dx
12 π3 12 π4 dy  2x  4x2
⇒ +  2
y=
3 2 dx  1+ x  1 + x2
⇒ ≤I≤
8 6 ∫
2x
1 + x2 2
Here, IF = e = elog(1 + x )
= 1 + x2
13. (b) Let
−1 2

)
4x
etan y (1 + x 2) = ∫ (1 + x 2) ×
(sin x )
5π 2 ∴ dx + C
I= ∫π 2

be
−1 −1
dx (1 + x 2)
etan (sin x )
+ etan (cos x )

⇒ y (1 + x 2) = ∫ 4 x 2dx + C
5π 2 tan −1 (sin x )
e
= ∫0

Tu
−1 −1
dx 4x3
etan (sin x )
+ etan (cos x ) ⇒ y(1 + x 2) = + C
−1
3
π 2 etan (sin x )
4x3
−∫ dx ...(i) ⇒ y (1 + x 2) = −1 [y(0) = − 1]

ou
0 −1 −1
etan (sin x )
+ etan (cos x )
3
−1
5π 2 etan (cos x ) 4x3 1
⇒ y= −
I= ∫0 dx

(Y
tan −1 (cos x ) −1 ( + x 2) 1 + x 2
31
e + etan (sin x )
−1 4 1 1
π 2 etan (cos x ) ∴ 1 = − =
y()
−∫ −1 −1
dx ...(ii) 6 2 6
0
etan (cos x )
+ etan (sin x )
on
 using, b f (x) dx = b f (a + b − x) dx 
17. (c) We have, x = 1 vt
 ∫a ∫a 
2
⇒ x=
1 dx Q v = dx 
pi

t
On adding Eqs. (i) and (ii), we get 2 dt  dt 
5π 2 π 2
2 dt dx
2I = ∫0 dx – ∫0 ⇒ =
m

dx
t x
⇒ 2I = (x)50π 2 − (x)π0 / 2 dt dx
⇒ 2⋅ ∫ = ∫
ha

5π π t x
⇒ 2I = − = 2π ⇒ I= π
2 2 ⇒ 2log |t| + log |c| = log |x|
⇒ log (t 2 ⋅ c) = log x
C

14. (c) We have,


1  2 π 2π nπ  ⇒ x = t 2c
lim sec + sec2 + .... + sec2 
dy

n→ ∞ n 4n 4n 4n  ⇒
dx
= 2tc
n
rπ πx  dt
sec2   = 2
1 1
= lim
n→ ∞
∑n  4n  ∫0 sec  4 
 d2x
⇒ = 2c
u

r =1
1 dt 2
 πx 
tan  
4
St

= ⇒ acceleration f is constant.
π   4   0
4 4 18. (b) We have,
= ×1 =
π π equation of parabola y = x 2
Let P(α , α 2) is a point on the parabola,
15. (c) Given, y 2 = 2d (x + d) ... (i)
∴ y − α 2 = 2α (x − α)
⇒ 2y y1 = 2d ⇒ d = y y1
From Eq. (i),  
dy dy
Q Q = 2x ⇒ = 2α 
y 2 = 2y y1 (x + y y1 )  dx dx( α , α 2) 
 
⇒ y 2 − 2y y1 x = y y1 ⋅ 2y y1
⇒ y = 2αx − α 2
⇒ (y − 2y y1 x) = 4(y y1)3
2 2
Also, given y = − x 2 + 4 x − 4
So, degree of above equation is 3.
34 WB JEE (Engineering) Solved Paper 2018
∴ − x 2 + 4 x − 4 = 2αx − α 2 or t − 2= 0
⇒ x + 2x (α − 2) + (4 − α ) = 0
2 2 ⇒ t=2 [Q neglect t = − 3]
⇒ 2x = 2 ⇒ x = 1
Discriminant = 0
4(α − 2)2 − 4 (4 − α 2) = 0 21. (b) We have, Zr = sin 2πr − i cos 2πr
11 11
⇒ (α − 2) − (4 − α ) = 0
2 2
 2πr 2πr 
⇒ α 2 − 4α + 4 − 4 + α 2 = 0 = − i  cos + i sin 
 11 11 
⇒ α 2 − 2α = 0 i 2πr

)
= − ie 11
⇒ α = 0, α = 2

be
10 10 i 2πr
19. (d) 2b = (n + 2)th form ∴ ∑ Zr = − i∑ e 11

r =0 r =0
= a + (n + 2 − 1) d

Tu
⇒ 2b = a + (n + 1) d =−i×0 =0
2b − a 22. (c) We know that, if z1 , z2 and z3 are the vertices
⇒ d=

ou
n+1 of an equilateral triangle. Then,
 2b − a  z12 + z22 + z32 − z1 z2 − z2z3 − z3z1 = 0 …(i)
∴ m th mean = a + m  
 n+1 

(Y
Now, but we have
and also, b = (n + 2)th form z1 z
+ 2 =1
= 2a + (n + 2 − 1) d z2 z1
= 2a + (n + 1) d
on
⇒ z12 + z22 = z1 z2
b − 2a
⇒ d= ⇒ z12 + z22 − z1 z2 = 0
n+1
z3 = 0
pi

Here,
 b − 2a 
∴ m th mean = 2a + m   Hence, given points form an equilateral triangle.
 n+1 
m

23. (a) We have equations


According to the question,
x 2 + b1 x + c1 = 0
 2b − a   b − 2a 
ha

a + m  = 2a + m   D1 = b12 − 4c1
 n+1   n+1 
a m and x + b2 x + c 2 = 0
2
⇒ =
C

b n+1− m D2 = b22 − 4c 2
Now, D1 + D2 = b12 + b22 − 4(c1 + c 2)
20. (c) We have,
dy

= b12 + b22 − 2b1 b2 [Qb1 b2 = 2(c1 + c 2)]


x + log10 (1 + 2x ) = x log10 5 + log10 6
= (b1 − b2) ≥ 02
⇒ log10 (1 + 2x ) = x log10 5 + log10 6 − x
u

⇒ At least one of D1 and D2 are non-negative


= log10 5x + log10 6 − x log10 10
St

real roots.
= log10 (5x ⋅ 6) − log10 10x
 5x ⋅ 6 
24. (a) Number of ways of selection of n objects from
⇒ log10 (1 + 2x ) = log10  x  2n objects, where as n objects are identical in out
 10  of 2n objects.
5x ⋅ 6 6 n identical and no different object = 1 ways
⇒ 1 + 2x = = x
10x 2 = n C0
⇒ 2x (1 + 2x ) = 6 n −1 identical and 1 different object = 1 × n C1
⇒ t(1 + t) = 6 (let 2 = t)
x
n − 2 identical and 2 different object = 1 × n C2
⇒ t2 + t − 6 = 0 …………………………………………………
⇒ (t + 3) (t − 2) = 0 0 identical and n different objects = 1 × n Cn
⇒ t + 3= 0 = n C0 + n C1 + n C2 + ....... + n Cn = 2n
WB JEE (Engineering) Solved Paper 2018 35

25. (c) n Cr + 2 ⋅ n Cr + 1 + n
Cr + 13 −11 1
2
= 15 −7 −1 5 (R3 → R3 – R2)
= n Cr + n Cr + 1 + n Cr + 1 + n Cr + 2
0 0 −6
n+1 n+1
= Cr + 1 + Cr + 2 = 15 {0 − 0 − 6 (−13 − 77)}
n+1
(Qn Cr + n Cr + 1 = Cr + 1 ) = 15 {(− 6) (− 90)} = 90 × 90 ... (ii)
n+ 2 From Eqs. (i) and (ii),
= Cr + 2
B = A2
26. (a) (101)100 − 1 = (1 + 100)100 − 1

)
29. (c) We have,

be
= (1 + 100
C1 ⋅100 + 100
C21002 + ......) − 1 2rπi

= 100
C1100 + 100
C2(100)2 + a r = (cos 2rπ + i sin 2rπ)1 / 9 = e 9

2πi 4 πi 6πi

Tu
100
C3(100)3 + ........ + 100
C100 (100)100 e 9 e 9 e 9
a1 a2 a3 8 πi 10 πi 12πi
= 104 (1 + 100
C2 + 100
C3102 + ....
Now, a 4 a5 a6 = e 9 e 9 e 9
+ 100
C100 (100)98 14 πi 16 πi 18 πi

ou
a7 a8 a9
e 9 e 9 e 9
= 10 (1 + an integer multiple of 10)
4

2πi 4 πi
27. (a) For greatest term of (1 + x)n , we have
n n+1 n
<
2 1+ x 2
< +1
(Y =e
2πi
9 ×e
8 πi
9
1
1
14 πi
e
e 9
9
2πi

16πi
e
e 9
9
4 πi

18 πi
on
n n+1 n+1 n
⇒ < and < +1 e 9 e 9 e 9
2 1+ x 1+ x 2
n+1 n+1 2πi 8 πi
⇒ 1+ x< <1 + x
pi

and
n2 n
+1 =e 9 ×e 9 ×0 (Q R1 and R2 are identical)
2 =0
m

n+1− n 2
⇒ x< 2r x n(n + 1)
n2
ha

30. (d) We have, Sr = 6r − 1


2
y n2(2n + 3)
n + 1 − (n 2 − 1)
and < x 4r 3 − 2nr z n3(n + 1)
n+ 2
n
C

2
2∑ r x n(n + 1)
n+ 2 n
⇒ x< and < x r =1
n n+ 2 n n
⇒ ∑ Sr = ∑ (6r − 1) y n2(2n + 3)
dy

2
n n+ 2
⇒ < x< r =1 r =1
n+ 2 n n
∑ (4r − 2nr) n3(n + 1)
3
z
u

28. (a) We have, r =1


St

−1 7 0 n(n + 1) x n(n + 1)
A= 2 1 −3 = n2(2n + 3) y n2(2n + 3)
3 4 1 n3(n + 1) z n3(n + 1)

= − 1 (1 + 12) − 7 (2 + 9) =0 (Q C1 and C3 are identical)


n
= − 13 − 77 = − 90
13 −11 5
... (i) Hence, ∑ Sr is independent of x, y, z and n.
r =1
And let B= −7 −1 25
−21 −3 −15
31. (c) For non-trivial solution,
1 4a a
13 −11 1 We have, 1 3b b = 0
= 3 × 5 −7 −1 5 1 2c c
−7 −1 −1
⇒ 1(3bc − 2bc) − (4ac − 2ac) + 1(4ab − 3ab) = 0
36 WB JEE (Engineering) Solved Paper 2018
⇒ bc − 2ac + ab = 0 35. (b) We have, P(H) = 1 , P(T) = 1
⇒ bc + ab = 2ac 2 2
⇒ b(a + c) = 2ac Now, P(X ≥ 1) = 1 − P(X = 0)
n
2ac
= 1 − n C0 (p)0 (q)n = 1 −  
⇒ b= 1
a+ c  2
⇒ a , b , c are in HP. Q
1
1 − n ≥ 0.9
2
32. (c) On the set R, 1
xρy ⇒ x − y is zero or irrational number. ⇒ 01. ≥

)
2n

be
Now, xρ x 1 1
⇒ x− x=0 ⇒ ≥
10 2n
⇒ρ is reflexive.
⇒ 2n ≥ 10

Tu
If xρy ⇒ x − y is zero or irrational.
⇒ n≥ 4
= − (y − x) is zero or irrational.
∴Minimum number of tossed = 4
⇒ yρx is zero or irrational.

ou
⇒ρ is symmetric. 36. (a) Let X be the event that student will be
And if successful.

(Y
xρy ⇒ x − y is 0 or irrational. X1 be the event that student will be pass in test-I.
yρz ⇒ y − z is 0 or irrational. X 2 be the event that student will be pass in test-II.
Then, (x − y) + (y − z) = x − z may be 0 or rational. X 3 be the event that student will be pass in test-III.
∴ P(X) = P(X1 ∩ X 2 ∩ X ′3) + P(X1 ∩ X ′2 ∩ X 3)
on
⇒ρ is not transitive.
+ P(X1 ∩ X 2 ∩ X 3)
33. (d) On the set R of real numbers ⇒ P(X) = P(X1) ⋅ P(X 2) ⋅ P(X' 3) + P(X1) ⋅ P(X' 2) ⋅ P(X 3)
pi

For reflexive, + P(X1) ⋅ P(X 2) ⋅ P(X 3)


xρx ⇒ (x , x) ∈ R 1 1 1 1
⇒ = p ⋅ q ⋅ + p(1 − q) + pq
m

⇒ x > |x|which is not true. 2 2 2 2


⇒ρ is not reflexive. 1 1 1 1 1
⇒ = p⋅ q⋅ + p⋅ − p⋅ q⋅ + p⋅ q⋅
ha

For symmetric, 2 2 2 2 2
(x , y) ∈ R ⇒ x > |y| ⇒
1 1
= p⋅ + p⋅ q⋅
1
and (y, x) ∈ R ⇒ y > |x| 2 2 2
C

So, x > |y| ≠ y > |x| ⇒ (p + pq) = 1


⇒ ρ is not symmetric. ⇒ p(1 + q) = 1
dy

For transitive, 37. (a) We have, sin 6θ + sin 4θ + sin 2θ = 0


(x , y) ∈ R ⇒ x > |y|, (y, z) ∈ R ⇒ y > |z|
⇒ sin 6θ + sin 2θ + sin 4θ = 0
⇒ x > |z| ⇒ (x , z) ∈ R
u

⇒ 2sin 4θ ⋅ cos 2θ + sin 4θ = 0


⇒ ρ is transitive.
⇒ sin 4θ(2cos 2θ + 1) = 0
St

34. (c) We have, f : R → R, defined by f (x) = e x ⇒ sin 4θ = 0 or 2cos 2θ + 1 = 0


and g : R → R defined by g(x) = x 2 −1 2π
⇒ 4θ = nπ or cos 2θ = = cos
2 3
Now, We have
nπ 2π
(gof) (x) = g( f (x)) ⇒ θ= or 2θ = 2nπ ±
4 3
= g(e x ) nπ π
⇒ θ= or θ = nπ ± , where n ∈ z
= (e x )2 4 3
= e 2x , ∀x ∈ R π
38. (c) We have, 0 ≤ A ≤
⇒ gof is injective and g is neither injective nor 4
−1  1 
surjective. tan  tan 2A + tan−1 (cot A) + tan−1 (cot 3 A)
⇒ gof is injective but g(x) is not bijective. 2 
WB JEE (Engineering) Solved Paper 2018 37
 cot A + cot 3 A 
= tan−1  tan 2A + tan−1 
1 41. (d) Given, point P(1, 5) image of the point P(1, 5)

2   1 − cot A 
4
about the line y = x is Q(5, 1) and image of the
 1 2tan A   tan A  point Q on line y = − x is R(− 1, − 5)
= tan−1  ⋅  + tan−1   P(1,5)
 2 1 − tan 2
A   tan A − 1 
2

 tan A   tan A 
= tan−1   − tan−1  
 1 − tan A   1 − tan A 
2 2

=0 R(–1,–5)

)
Q(5,1)

be
39. (b) Put x = x' + 2 ∴ Required circumcentre = Mid-point of P and R
1 − 1 5 − 5
and y = y' + 3 =  ,  = (0, 0)
∴ x 2 + y2 − 4 x − 6 y + 9 = 0  2 2 

Tu
⇒ (x' + 2)2 + (y ' + 3)2 − 4(x' + 2) 42. (b) Here,AB = (5 + 1)2 + (1 + 7)2 = 36 + 64 = 10
− 6(y' + 3) + 9 = 0 A(–1,–7)

ou
⇒ x' 2 + 4 + 4 x' + y ' 2 + 9
10 P
+ 6 y ' − 4 x' − 8 − 6 y ' − 18 + 9 = 0

(Y
⇒ x ′2 + y′2 − 4 = 0 B(5,1) C(1,4)
5
⇒ x 2 + y2 = 4
BC = (1 − 5)2 + (4 − 1)2 = 16 + 9 = 5
40. (d) We have equation of circle
on
By angle bisector theorem,
x 2 + y 2 + 4 x − 6 y + 9sin2 α + 13cos2 α = 0 AP : CP = 10 : 5 = 2 : 1
Here, C ≡ (− 2, 3)  2 × 1 + 1 × (− 1) 2 × 4 + 1 × (− 7)  1 1 
pi

∴ P ,  = P , 
Radius = (− 2)2 + (3)2 − (9sin2 α + 13cos2 α)  2+ 1 2+ 1   3 3
m

= 4 + 9 − 9sin2 α − 13cos2 α Required equation of BP is


1
= 13 − 131
( − sin2 α) − 9sin2 α −1
ha

y −1 = 3 (x − 5)
1
= 13sin2 α − 9sin2 α −5
3
= 4sin2 α = 2sin α −2
C

⇒ y −1 = (x − 5)
−14
A
⇒ 7y − 7 = x − 5
dy

⇒ 7y = x + 2
α
P(h,k) α C(–2,3) 43. (a) Given, equation of circle is
u

x 2 + y 2 + 4 x + 6 y − 12 = 0
St

Centre C(−2, −3)


AC
Here, sinα = and radius = (−2)2 + (−3)2 + 12 = 5
PC
⇒ PC sin α = AC
S
⇒ PC 2 sin2 α = AC 2 = (2sin α)2
C2
⇒ [(h + 2)2 + (k − 3)2]sin2 α = 4 sin2 α (2,–3)
⇒ (h + 2)2 + (k − 3)2 = 4 C1
P
(–2,–3)
⇒ h + 4 + 4h + k + 9 − 6k = 4
2 2

⇒ h2 + k2 + 4h − 6k + 9 = 0
Hence, locus of a point is
x 2 + y 2 + 4 x + 6 y − 12 = 0
x 2 + y2 + 4 x − 6 y + 9 = 0
38 WB JEE (Engineering) Solved Paper 2018
 2 
46. (c, d) Centre of circle =  t1 + t2 , (t1 + t2)
2
∴ C1 C2 = (2 + 2)2 + (−3 + 3)2
 2 
= (4)2 + (0)2 = 4
2
A(t1,2t1)
∴ Radius of circle, S Y
2
B(t2,2t2)
= (4) + (5) = 16 + 25 =
2 2
41 unit
r
44. (c) Given, AM = 2AB X′ X

y2=4x

)
be
Y′
(x1,y1) Since, circle touch the x-axis, so equation of tangent
A(0,3)
B M is y = 0

Tu
⇒ B is mid-point of AM. Q Radius = Perpendicular distance from centre to
0 + x1 3 + y1  the tangent
∴Coordinate of B is  ,  ⇒ Radius = |t1 + t2| = r
 2 2 

ou
2 2
x y + 3 Slope of AB = =
=  1 , 1  t1 + t2 ± r
 2 2 
Since, B lies on the circle x 2 + 4 x + (y − 3)2 = 0
x
 2
2
x
 2  2
y + 3
∴ 1  + 4  1  +  1 − 3 = 0

2
(Y
47. (d) Y

R(a
r2 ,2
ar) P(at ,2at)
2
on
2
x12 y −
+ 2x1 +  1
3 F K(2a, 0)
⇒  =0 X′ X
4  2 
pi

x12 y 2 + 9 − 6 y1
⇒ + 2x1 + 1 =0
4 4 y 2 = 4ax
m

Q
⇒ x12 + y12 + 8 x1 − 6 y1 + 9 = 0
Y′
ha

Hence, locus of a point is a −2a 


Here, coordinate of Q will be  2 , .
x 2 + y2 + 8 x − 6 y + 9 = 0 t t 
2
C

45. (a) Given equation of ellipses is x 2 + 9 y2 = 9 Slope of QR =


1
x2 y2 r−
⇒ + =1 t
dy

9 1 2at 2t
Slope of PK = 2 =
Here, a = 3, b = 1 at − 2a t 2 − 2
c = (3)2 − (1)2 = 8 Since, Slope of QR = Slope of PK
u

c 2 2t
∴Eccentricity of ellipse, e = ∴ =
St

1 t2 − 2
a r−
8 t
⇒ e=
3 t2 − 1
⇒ r=
3 t
∴Eccentricity of hyperbola =
8 2 2
2 48. (a) Since, point P on the ellipse x + y = 1
b 9 9 4
⇒ 1+ =
a2 8 ∴ P(3cosθ, 2sinθ)
b2 1 Now, equation of line parallel of Y-axis is
⇒ =
a2 8 x = 3cosθ
⇒ a2 : b2 = 8 :1 and above line meets circle at Q
∴ Q(3cosθ, 3sinθ)
WB JEE (Engineering) Solved Paper 2018 39

Given,
PR 1
= ⇒ 4λ + 2 + 64λ − 88 + 9λ + 9 = 0
RQ 2 ⇒ 77λ − 77 = 0
R(h, k) ⇒ λ =1
P Q ∴Required foot of perpendicular,
(3 cos q, 2 sin q) (3 cos q, 3 sin q)
P(4, 5, − 2)
3cosθ + 6 cosθ 3sinθ + 4sinθ
∴h= , k=
3 3 51. (a)
7 20 y
⇒ h = 3cosθ, k = sinθ

)
3

be
3k
⇒ cosθ = h 3, sinθ = x
7 x 2 + y 2 = (20)2 = 400
9k2
Now, cos θ + sin θ = h 9 +
2 2 2
=1 dy

Tu
We have, = 2ft /sec
49 dt
x 2 9 y2 When x = 12
Hence, locus of a point is + =1
9 49 (12)2 + y 2 = 400

ou
then
49. (a) Equation of line through Q (1, − 2, 3) and ⇒ 144 + y 2 = 400
x y z
= = ⇒ y 2 = 400 − 144 = 256

(Y
parallel to the line
1 4 5 ⇒ y = 16
x −1 y + 2 z − 3
is = = =λ (say) Now, 2x
dx dy
+ 2y = 0
1 4 5
on
dt dt
Since, point P lies on above line. dx dy
⇒ x =− y
∴ P(λ + 1, 4λ − 2, 5λ + 3) dt dt
pi

Since, P lies on the given plane.  dx 


⇒ 12  = − 16(2)
∴ 2(λ + 1) + 3(4λ − 2) − 4(5λ + 3) + 22 = 0  dt 
m

⇒ 2λ + 2 + 12λ − 6 −20λ − 12 + 22 = 0 dx −8
⇒ =
⇒ − 6λ + 6 = 0 dt 3
ha

⇒ λ =1 1
52. (b) Here, let p=
∴ P(2, 2, 8) m
∴ PQ = (2 − 1)2 + (2 + 2)2 + (3 − 8)2 then (a p + b p)1 p = (a1 m + b1 m)m
C

⇒ PQ = 1 + 16 + 25 = 42 = a + b + k, k ≥ 0
∴ a p + b p ≥ (a + b)p ≥ (a + b)
dy

50. (d) Equation of line joining points (0, − 11, 4) and


⇒ J(p) ≥ I (p)
(2, − 3, 1) is
x − 2 y + 3 z −1 53. (c) We have,
= = =λ
u

(say)
2 8 −3 → → →
δ = α + λβ
St

Q(1, 8, 4)
= (i + j + k$) + λ (i$ − $j − k$)
$ $

⇒ δ = (1 + λ)i$ + (1 − λ) $j + (1 − λ)k$
→→
δ. γ 1
P(2λ+2, 8λ–3, – 3λ+1) Given, =
→ 3
| γ|
Let P is any point of the above line then coordinate
(1 + λ) (− 1) + (1 − λ) ()
1 + (1 − λ) (− 1) 1
of P is (2λ + 2, 8λ − 3, − 3λ + 1). ⇒ =
∴DR’s of PQ is (2λ + 1, 8λ − 11, − 3λ − 3) 3 3
Now, (2λ + 1) (2) + (8λ − 11) (8) + (− 3λ − 3) ⇒ λ=−2

(− 3) = 0 ∴ δ = − i$ + 3$j + 3k$
40 WB JEE (Engineering) Solved Paper 2018

54. (c) α = λ  β × γ  = λ(|β| |γ| |sin 30°|  k k  where, k = 1 


→ → → → →
⇒ A=   
   − k k  2
→  → 1
 k k  k k  0 2k2 
⇒ |α| = |λ| |β| |γ|⋅  ⇒ A2 =     =  
 2  − k k  − k k  −2k 2
0 
1
⇒ 1 = |λ|⋅ 1 ⋅ 1 ⋅  0 2k2   0 2k2 
2 A4 =    
⇒ |λ| = 2  −2k
2
0   −2k 2
0 
⇒ λ=±2  −4 × 1 0 

)
→ → →   −4k4 0   4
A4 =   =

be
∴ α = ± 2 β × γ   1
   0 −4k4   0 −4 × 
 4
55. (d) Let z1 = x1 + iy1 and z2 = x 2 + iy2  −1 0 

Tu
⇒ A4 =  
Re (z1) > 0 ⇒ x1 > 0  0 −1
and Im (z2) < 0  −1 0   −1 0   1 0
⇒ A8 =  =

ou
⇒ y2 < 0   
 0 −1  0 −1  0 1 
Given, |z1| = |z2|
⇒ |z1|2 = |z22| 58. (d) We have, ρ = {(x , y) ∈ N × N : 2x + y = 41}

Now, 
z1 z1 = z2z2
 z1 + z2   z1 + z2 
 + 
 z1 − z2   z1 − z2 

(Y For reflexive,


xρx ⇒ 2x + x = 41
3x = 41
on
 z + z2   z1 + z2  41
⇒ x= ∉N
= 1  +   3
 z1 − z2   z1 − z2 
pi

So, ρ is not reflexive.


z z + z2z1 − z1 z2 − z2z2 + z1 z1 + z1 z2 − z2z1 + z2z2
= 1 1 For symmetric,
(z1 − z2) (z1 − z2)
m

xρy ⇒ 2x + y = 41
2(| z1|2 − |z2|2)
= =0 (Q|z1|2 = |z2|2) and yρx ⇒ 2y + x = 41
ha

(z1 − z2) (z1 − z2)


⇒ xρy ≠ yρx
z + z2
= 1 is purely imaginary. So, ρ is not symmetric.
z1 − z2
C

For transitive,
56. (a) Given, numbers are 1, 2, 3 ........ 20 xρy ⇒ 2x + y = 41
and yρz ⇒ 2y + z = 41
dy

Here, number of ways of selecting four consecutive


numbers = 17 ⇒ xρ/ z
∴Required number of selecting 4 non-consecutive ⇒ρ is not transitive.
u

numbers = 20 C4 − 17 (1 + x)a (2 + x)b 1


20 × 19 × 18 × 17
St

= − 17 59. (a) Given, f (x) = 1 (1 + x) (2 + x)b


a

4 × 3× 2×1
(2 + x)b 1 (1 + x)a
= 285 × 17 − 17
= 284 × 17 For constant term put x = 0
n 1 2b 1
 cos π 4 sin π 4
57. (b) We have,  π π f (0) = 1 1 2b
 − sin cos 
 4 4  2b 1 1
 1 1 
  = 11
( − 2b) − 2b(1 − 22b) + 11
( − 2b)
Let A =  2 2
− 1 1  = 1 − 2b − 2b + 23b + 1 − 2b
 2 2
= 2 − 3⋅ 2b + 23b
WB JEE (Engineering) Solved Paper 2018 41

60. (a) Equation of line AB is 61. (c) Given, equation of circle is


x
+
y
=1 x 2 + y 2 − 2x − 4 y − 20 = 0
5 −3
Center (1, 2) and radius = ()
1 2 + (2)2 + 20 = 5
⇒ 3x −5y = 15
Y Y y=7
B(1,7) C

=0
Q R(x,

20
y) (1,2)

y–
)
A

–4
A(5

3x
,0)

be
X
X O D(4,–2)
P

Tu
Coordinate of intersecting point of tangents at B
B(0,–3) and D is C(16, 7).
∴ Area of quadrilateral ABCD

ou
= 2 × ar(∆ABC)
Perpendicular line to AB is
1
5x + 3y = λ = 2 × × 15 × 5= 75sq units

(Y
2
λ
Coordinate of P is  , 0 π
5  62. (b) At x=−
2
and coordinate of Q is ( 0, λ 3)
on
LHL = − 2
Now, equation of line AQ is
RHL = − A + B
y
x 5+ =1 For continuity, LHL = RHL = f(− π 2)
pi

λ3
⇒ −A + B = 2 ... (i)

x 3y
+ =1 At x = π 2
m

5 λ LHL = A + B
3y x RHL = 0
ha

⇒ =1 −
λ 5 For continuity, LHL = RHL = f(π 2)
1 1  x ⇒ A+ B=0 ... (ii)
⇒ = 1 −  ... (i)
C

λ 3y  5 On solving Eqs. (i) and (ii), we get


and equation of line BP is A = −1and B = 1
dy

x y 63. (c) Given equation of curve,


+ =1
λ 5 −3 y = x2 − x + 1
5x y dy
u

⇒ − =1 ⇒ = 2x − 1
λ 3 dx
St

1 1 y  1
⇒ =  + 1 ... (ii) Slope of normal =
λ 5x  3  1 − 2x
From Eqs. (i) and (ii), Now, at x1 = 0, y1 = 1
1  x 1 y  ∴Slope of normal at (0, 1) = 1
1 −  =  + 1 ∴ Equation of normal, y − 1 = 1(x − 0)
3y  5  5x  3 
⇒ x − y+1= 0 .. (i)
5x 1 −  = 3y + 1
x y
⇒ At x 2 = − 1, y2 = 3
 5 3 
1
⇒ 5x − x 2 = y 2 + 3y Slope of normal at (−1, 3) =
3
⇒ x 2 + y 2 − 5x + 3y = 0 1
Equation of normal, y − 3 = (x + 1)
which is a circle. 3
⇒ 3y − 9 = x + 1
42 WB JEE (Engineering) Solved Paper 2018
⇒ x − 3y + 10 = 0 ... (ii) 1 1
⇒ ∫0 − x dx < I < ∫0 x dx
2 2
5 19
At x 3 = , y3 = 1 1
2 4  − x3   x3 
⇒   <I< 
 2 19 
Slope of normal at  ,  = −
1  3 0  3 0
5 4 4 −1 1
⇒ <I<
= −  x − 
19 1 5
Equation of normal, y − 3 3
4 4 2
43 67. (c, d) Given, curve,12y = x 3
⇒ x + 4y =

)
dy dx
2 and > ... (i)

be
⇒ 2x + 8 y = 43 ... (iii) dt dt
dy dx
Here, intersecting point of Eqs. (i) and (ii) is  ,  12 = 3x 2
7 9 Now, ... (ii)
 2 2 dt dt

Tu
and normal (iii) passes through it. Hence, normals From Eqs. (i) and (ii), we get
are concurrent. dx dx
3x 2 > 12
dt dt

ou
64. (b) Let f (x) = x log x + x − 3
⇒ 3x 2 > 12
1
⇒ f ′(x) = x ⋅ + log x + 1 ⇒ x2 − 4 > 0
x

⇒ f()
f ′(x) = log x + 2 > 0
1 = − 2 and f (3) = 3log 3, f ()
1 ⋅ f (3) < 0
Hence, exactly one root in x ∈(1, 3) as f (x) > 0
(Y ⇒

(x − 2) (x + 2) > 0
x ∈ (− ∞ , − 2) ∪ (2, ∞)

68. (b) Given, equation of circle


on
65. (c) Equation of PQ is 2x − y = 12 x 2 + y 2 = 2ax
⇒ (x − a)2 + y 2 = a 2
pi

Y
2 ,2t)
Q(9,6) and equation of parabola is y 2 = ax , a > 0
R(t
m

Y
A
X′ X x2 + y2 = 2ax
ha

y2=4x X′ X
P(4,–4) (a,0)
C

Y′
y2 = ax
Perpendicular distance Y′
dy

2t 2 − 2t − 12 2 Intersection points of circle and parabola


AR = = (t − 3) (t + 2)
5 5 ⇒ x 2 + ax = 2ax
u

1 ⇒ x 2 = ax
AR is Maximum, at t =
St

2
⇒ x 2 − ax = 0
∴R is  , 1
1
4  ⇒ x(x − a) = 0
⇒ x = 0, a
1 3
66. (b) I = ∫ x cos 32x dx Intersecting points are (0, 0) and (a , a).
0 2+ x
πa 2 a
Here, −1 < cos 3x < 1 ∴ Required area = − ∫ axdx
4 0
⇒ − x 3 < x 3 cos 3x < x 3 a
πa 2  x3 2 
−x −x
3
−x 3 3
x cos 3x 3 = − a 
⇒ < < < 4  3 2 0
x2 x 2 + x2 2 + x2
πa 2 2a 2 π 2
x3 x3 x3 = − = a 2  − 
< < < 2 4 3  4 3
2+ x 2
x x
WB JEE (Engineering) Solved Paper 2018 43

69. (a, d) Let α and β are the roots of x 2 + ax + b = 0 1 1 1


1
⇒ k k 1 = ± h2
and the roots of x − cx + d = 0 are α and β .
2 4 4
2
2− k k 1
Now, α + β = − a, αβ = b ... (i)
and α 4 + β 4 = c , α 4β 4 = d ... (ii) ⇒ 1(k − k) − 1(k − 2 + k) + 1(k2 − 2k + k2) = ± 2h2

From Eqs. (i) and (ii), ⇒ − (2k − 2) + (2k2 − 2k) = ± 2h2


b 4 = d and α 4 + β 4 = c ⇒ 2 − 2k + 2k2 − 2k = ± 2h2
(α + β ) − 2(αβ) = c
2 2 2 2
⇒ 2k2 − 4k + 2 = ± 2h2

)
⇒ [(α + β) − 2αβ] − 2(αβ) = c
2 2 2

be
⇒ k2 − 2k + 1 = ± h2
⇒ (a − 2b) − 2b = c
2 2 2
Hence, locus of a point is
⇒ 2b 2 + c = (a 2 − 2b)2 ⇒ (k − 1)2 = h2

Tu
⇒ 2b 2 + c = a 4 + 4b 2 − 4a 2b y −1 = ± x
⇒ 2b − c = 4a b − a
2 2 4 ⇒ x = y − 1or x = − (y − 1)

ou
⇒ 2b 2 − c = a 2(4b − a 2) 73. (b) Given, 2a1 = 2 sinθ
and for equation x 2 − 4bx + 2b 2 − c = 0 ⇒ a1 = sinθ
D = (4b)2 − 41
() (2b 2 − c)
= 16b 2 − 8b 2 + 4c = 8b 2 + 4c (Y and


3x 2 + 4 y 2 = 12
x2
+
y2
=1
on
= 4(2b + c) = 4(a − 2b) > 0 = real root
2 2 2
4 3
Now, f (0) = 2b 2 − c = a 2(4b − a 2) < 0 (Qa 2 > 4b) Here, a 2 = 4 and b 2 = 3
= roots are opposite in sign ∴ b 2 = a 2(1 − e 2)
pi

70. (b, c) Required number of ways = 20


C2 × 2! ⇒ 3 = 4 (1 − e 2)
m

= 20
P2 ⇒ e2 = 1 −
3 1
=
4 4
 0 −1 −1
ha

1
71. (a, c) A = 1 0 −1 ⇒ e=
  2
1 1 0 
C

Focus, F (ae , 0) = F  2 × , 0


1
 0 1 1  2 
⇒ A =  −1 0 1  = − A

  = F (1, 0)
dy

 −1 −1 0
For hyperbola foci are same
⇒ A is a skew-symmetric matrix. a1 e1 = ae = 1
u

|A| = 0 + 1(0 + 1) − 11
( − 0)
∴ (sinθ) e1 = 1
= 0 + 1 − 1 = 0 = Singular
St

⇒ e1 = cosec θ
⇒ A is not invertible.
and b12 = a12(e12 − 1) = a12e12 − a12
72. (a, b) Given, ar (∆ABC) = h2
⇒ b12 = 1 − sin2 θ = cos2 θ
x
P(h,k) y= x 2 y2
y =k − =1
C(2–k,k) B(k,k) a12 b12
x2 y2
A(1,1) ⇒ − =1
sin θ cos2 θ
2

x+
y=
⇒ x 2cosec2 θ − y 2 sec2 θ = 1
y 2
44 WB JEE (Engineering) Solved Paper 2018

74. (a, c) f (x) = cos π  75. (c) Given, y = log a (x + x 2 + 1), a > 0, a ≠ 1
 x
π −π π π ⇒ a y = (x + x 2 + 1)
⇒ f ′(x) = − sin    2  = 2 sin
 x  x  x x ⇒ a− y =
1
For increasing function, f ′(x) > 0 x+ x2 + 1
π π
⇒sin  > 0 ⇒ 2kπ < < (2k + 1) π = x2 + 1 − x
 x x
a y − a− y

1
> x>
1 ⇒ a y − a − y = 2x ⇒ x =

)
2k 2k + 1 2

be
e y log a − e − y log a
For decreasing function, f ′(x) < 0 ⇒ f −1 (y) =
π 2
⇒ sin  < 0  e x − e −x 

Tu
 x ⇒ f −1 (y) = sinh (y log a) Q = sinh (x)
π  1 1   2 
⇒ ∈ [(2k + 1) π , (2k + 2) π] ⇒ x ∈  , 
x  2k + 2 2k + 1 

ou
(Y
on
pi
m
ha
C
u dy
St
WB JEE
Engineering Entrance Exam
SOLVED PAPER 2017

)
be
Tu
PHYSICS

ou
Category-I (Q.1 to Q. 30) tension T. It can vibrate at frequencies ( v )
Direction Only one answer is correct. Correct
answer will fetch full marks 1. Incorrect answer or (Y given by the formula (where n = 1, 2, 3, ...)
(a) v =
n
2 ML
T
(b) v =
n T
2L M
on
any combination of more than one answer will
1
fetch − marks. No answer will fetch 0 marks. (c) v =
1 T
(d) v =
n TL
4 2n ML 2 M
pi

Unless otherwise specified in the question, the 3. A uniform capillary tube of length l and inner
m

following values should be used. radius r with its upper end sealed is
Mechanical equivalent of heat, J = 4.2 J cal − 1 submerged vertically into water. The outside
Acceleration due to gravity, g = 9.8 ms− 2
ha

pressure is p 0 and surface tension of water is


Absolute zero temperature = − 273 °C γ. When a length x of the capillary is
The following symbols usually carry meaning submerged into water, it is found that water
C

as given below levels inside and outside the capillary


ε 0 : electric permittivity of free space coincide. The value of x is
dy

µ 0 : magnetic permeability of free space l  p r


(a) (b) l 1 − 0 
R : universal gas constant  p 0r   4γ 
1 + 
u

1. The velocity of a particle executing a simple  4γ 


St

harmonic motion is 13 ms− 1 , when its distance  p r l


(c) l 1 − 0  (d)
from the equilibrium position (Q ) is 3 m and  2γ   p 0r 
its velocity is 12 ms− 1 , when it is 5 m away 1 + 
 2γ 
from Q. The frequency of the simple harmonic
motion is 4. A liquid of bulk modulus k is compressed by
5π 5 applying an external pressure such that its
(a) (b)
8 8π density increases by 0.01%. The pressure
8π 8 applied on the liquid is
(c) (d)
5 5π k k
(a) (b)
10000 1000
2. A uniform string of length L and mass M is
(c) 1000 k (d) 0.01 k
fixed at both ends while it is subject to a
2 WEST BENGAL (Engineering) Solved Paper 2017

5. Temperature of an ideal gas, initially at 27° C, its perpendicular bisector at a distance


is raised by 6° C. The rms velocity of the gas r ( r >> L )
molecules will 1
(a) decreases as
(a) increase by nearly 2% r
(b) decrease by nearly 2% 1
(b) decreases as 2
(c) increase by nearly 1% r
(d) decrease by nearly 1% 1
(c) decreases as 3
6. 2 moles of an ideal monoatomic gas is carried r

)
from a state ( p 0, V 0 ) to state ( 2p 0, 2V 0 ) along (d) approaches a finite limit as r → ∞

be
a straight line path in a p-V diagram. The 12. The magnets of two suspended coil
amount of heat absorbed by the gas in the galvanometers are of the same strength so

Tu
process is given by that they produce identical uniform magnetic
9 3 fields in the region of the coils. The coil of the
(a) 3 p 0V 0 (b) p 0V 0 (c) 6 p 0V 0 (d) p 0V 0
2 2 first one is in the shape of a square of side a

ou
7. A solid rectangular sheet has two different and that of the second one is circular of radius
a
coefficients of linear expansion α1 and α 2 . When the same current is passed through
π

(Y
along its length and breadth respectively. The
coefficient of surface expansion is the coils, the ratio of the torque experienced
(for α1 t << 1, α 2 t << 1 ) by the first coil to that experienced by the
α + α2 second one is
on
(a) 1 (b) 2(α1 + α 2 ) 1
2 (a) 1 : (b) 1 : 1 (c) π : 1 (d) 1 : π
4α1α 2 π
(c) (d) α1 + α 2
α1 + α 2
pi

13. A proton is moving with a uniform velocity of


8. A positive charge Q is situated at the centre of 106 ms− 1 along the Y-axis, under the joint
m

a cube. The electric flux through any face of action of a magnetic field along Z-axis and an
the cube is (in SI units) electric field of magnitude 2 × 104 Vm − 1
ha

Q Q Q along the negative X-axis. If the electric field


(a) (b) 4πQ (c) (d) is switched off, the proton starts moving in a
6ε 0 4πε 0 6πε 0
circle. The radius of the circle is nearly
C

9. Three capacitors of capacitance 1.0, 2.0 and  e 


given : ratio for proton ≈ 108 Ckg − 1 
5.0 µF are connected in series to a 10 V  m 
dy

source. The potential difference across the (a) 0.5 m (b) 0.2 m (c) 0.1 m (d) 0.05 m
2.0 µF capacitor is
14. When the frequency of the AC voltage applied
u

100 20 50
(a) V (b) V (c) V (d) 10 V to a series LCR circuit is gradually increased
17 17 17
St

from a low value, the impedance of the circuit


10. A charge of 0.8 coulomb is divided into two (a) monotonically increases
charges Q1 and Q 2. These are kept at a (b) first increases and then decreases
separation of 30 cm. The force on Q1 is (c) first decreases and then increases
maximum when (d) monotonically decreases
(a) Q1 = Q 2 = 0.4 C 15. Six wires, each of resistance r, are connected
(b) Q1 ≈ 0.8 C, Q 2 negligible so as to form a tetrahedron. The equivalent
(c) Q1 negligible, Q 2 ≈ 0.8 C resistance of the combination when current
(d) Q1 = 0.2 C, Q 2 = 0.6 C enters through one corner and leaves through
11. The magnetic field due to a current in a some other corner is
straight wire segment of length L at a point on r r
(a) r (b) 2r (c) (d)
3 2
WEST BENGAL (Engineering) Solved Paper 2017 3

16. Consider the circuit shown in the figure. (a) infinite distance above the lens
(b) 0.1 m above the center of the lens
R
(c) infinite distance below the lens
R X (d) 0.1 m below the center of the lens
E
21.
n
The value of the resistance X for which the
thermal power generated in it is practically

)
independent of small variation of its R

be
resistance is
R
(a) X = R (b) X =
3 x

Tu
R
(c) X = (d) X = 2R A parallel beam of light is incident on a glass
2 prism in the shape of a quarter cylinder of
radius R = 0.05 m and refractive index

ou
17. A
n = 1.5, placed on a horizontal table as shown
X in the figure. Beyond the cylinder, a patch of
B

Consider the circuit shown in the figure where (Y light is found whose the nearest distance x
from the cylinder is
(a) ( 3 3 − 4) × 10− 2 m
on
all the resistances are of magnitude 1 kΩ. If (b) ( 2 3 − 2) × 10− 2 m
the current in the extreme right resistance X is
(c) ( 3 5 − 5) × 10− 2 m
1 mA, the potential difference between A and
(d) ( 3 2 − 3) × 10− 2 m
pi

B is
(a) 34 V (b) 21 V 22. The de-Broglie wavelength of an electron is
m

(c) 68 V (d) 55 V 0.4 × 10− 10 m when its kinetic energy is


18. The ratio of the diameter of the sun to the 1.0 keV. Its wavelength will be 1.0 × 10− 10 m,
ha

distance between the earth and the sun is when its kinetic energy is
approximately 0.009. The approximate (a) 0.2 keV (b) 0.8 keV
C

diameter of the image of the sun formed by a (c) 0.63 keV (d) 0.16 keV
concave spherical mirror of radius of 23. When light of frequency v 1 is incident on a
curvature 0.4 m is metal with work function W (where hv 1 > W ),
dy

(a) 4.5 × 10− 6 m (b) 4.0 × 10− 6 m then photocurrent falls to zero at a stopping
− 3
(c) 3. 6 × 10 m (d) 1.8 × 10− 3 m potential of V1 . If the frequency of light is
u

19. Two monochromatic coherent light beams A increased to v 2, the stopping potential
L
St

changes to V 2. Therefore, the charge of an


and B have intensities L and , respectively. If
4 electron is given by
these beams are superposed, the maximum W( v 2 + v 1 ) W( v 2 + v 1 )
(a) (b)
and minimum intensities will be v 1 V 2 + v 2V1 v 1 V1 + v 2V 2
9L L
(a) ,
5L
(b) , 0 W( v 2 − v 1 ) W( v 2 − v 1 )
(c) (d)
4 4 4 v 1 V 2 − v 2V1 v 2V 2 − v 1 V1
5L L
(c) , 0 (d) 2L,
2 2 24. Radon-222 has a half-life of 3.8 days. If one
starts with 0.064 kg of radon-222, the
20. A point object is held above a thin equiconvex quantity of radon-222 left after 19 days will
lens at its focus. The focal length is 0.1 m and be
the lens rests on a horizontal thin plane (a) 0.002 kg (b) 0.062 kg
mirror. The final image will be formed at (c) 0.032 kg (d) 0.024 kg
4 WEST BENGAL (Engineering) Solved Paper 2017

25. 1 k = 1 Ns− 1 . The distance the block will travel


A 1
1 in 6 seconds is
B 1
0 Y (a) 36 m (b) 72 m
1 0
0 (c) 108 m (d) 18 m
0
Category-II (Q. 31 to Q. 35)
In the given circuit, the binary inputs at A and
B are both 1 in one case and both 0 in the next
Direction Only one answer is correct. Correct
case. The respective outputs at Y in these two
answer will fetch full marks 2. Incorrect answer or

)
any combination of more than one answer will

be
cases will be 1
(a) 1, 1 (b) 0, 0 fetch − marks. No answer will fetch 0 marks.
(c) 0, 1 (d) 1, 0 2

Tu
26. When a semiconducting device is connected 31. A particle with charge Q coulomb, tied at the
in series with a battery and a resistance, a end of an inextensible string of length
current is found to flow in the circuit. If R metre, revolves in a vertical plane. At the

ou
however, the polarity of the battery is centre of the circular trajectory, there is a
reversed, practically no current flows in the fixed charge of magnitude Q coulomb. The
mass of the moving charge M is such that

(Y
circuit. The device may be
Q2
(a) a p-type semiconductor Mg = . If at the highest position of the
(b) a n-type semiconductor 4πε 0R 2
on
(c) an intrinsic semiconductor particle, the tension of the string just
(d) a p-n junction vanishes, the horizontal velocity at the lowest
27. The dimension of the universal constant of point has to be
pi

gravitation, G is (a) 0 (b) 2 gR (c) 2gR (d) 5gR


(a) [ ML2T − 1 ] (b) [M− 1 L3T − 2]
m

32. A bullet of mass 4.2 × 10− 2 kg, moving at a


(c) [M− 1 L2T − 2] (d) [ML3T − 2]
speed of 300 ms − 1 , gets stuck into a block
ha

28. Two particles A and B (both initially at rest) with a mass 9 times that of the bullet. If the
start moving towards each other under a block is free to move without any kind of
mutual force of attraction. At the instant, friction, the heat generated in the process
C

when the speed of A is v and the speed of B is will be


2v, the speed of the centre of mass is (a) 45 cal (b) 405 cal
dy

(a) zero (b) v (c) 450 cal (d) 1701 cal


3v 3v
(c) (d) − 33. A particle with charge e and mass m, moving
2 2
along the X-axis with a uniform speed u,
u

29. Three vectors A = ai + j + k ; B = i + bj + k enters a region where a uniform electric field


St

and C = i + j + ck are mutually perpendicular E is acting along the Y-axis. The particle starts
(i, j and k are unit vectors along X, Y and Z- to move in a parabola. Its focal length
axes respectively). The respective values of a, (neglecting any effect of gravity) is
b and c are 2mu 2 eE mu mu 2
1 1 1 (a) (b) 2
(c) (d)
(a) 0, 0, 0 (b) − , − , − eE 2mu 2eE 2eE
2 2 2
1 1 1 34. A unit negative charge with mass M resides at
(c) 1, − 1, 1 (d) , , the mid-point of the straight line of length 2a
2 2 2
adjoining two fixed charges of magnitude + Q
30. A block of mass 1 kg starts from rest at x = 0 each. If it is given a very small displacement
and moves along the X-axis under the action x( x << a ) in a direction perpendicular to the
of a force F = kt, where t is time and straight line, it will
WEST BENGAL (Engineering) Solved Paper 2017 5

(a) come back to its original position and (b) 0.2 m from the 1st wire, towards the
stay there 2nd wire
(b) execute oscillations with frequency (c) 0.1 m from the 1st wire, away from the
1 Q 2nd wire
2π 4πε 0Ma 3 (d) 0.2 m from the 1st wire, away from the
2nd wire
(c) fly to infinity
(d) execute oscillations with frequency 38. If χ stands for the magnetic susceptibility of a
1 Q substance, µ for its magnetic permeability and

)
µ 0 for the permeability of free space, then

be
2π 4πε 0Ma 2
(a) for a paramagnetic substance: χ < 0,
35. Consider the circuit given here. The potential µ>0

Tu
difference V BC between the points B and C is (b) for a paramagnetic substance: χ > 0,
1 µF 2 µF µ > µ0
A C D (c) for a diamagnetic substance: χ > 0, µ < 0

ou
B (d) for a ferromagnetic substance: χ > 1,
1kΩ 2kΩ µ > µ0

(Y
39. Let v n and E n be the respective speed and
3kΩ 3V
energy of an electron in the nth orbit of radius
rn , in a hydrogen atom, as predicted by Bohr’s
(a) 1 V (b) 0.5 V (c) 0 V (d) − 1 V
on
model. Then.
E r
Category-III (Q. 36 to Q. 40) (a) plot of n n as a function of n is a
E1 r1
pi

Direction One or more answer(s) is (are) straight line of slope 0


correct. Correct answer(s) will fetch full marks 2. r v
m

Any combination containing one or more (b) plot of n n as a function of n is a


r1 v 1
incorrect answer will fetch 0 marks. Also, no
ha

straight line of slope 1


answer will fetch 0 marks. If all correct answers
r 
are not marked and also no incorrect answer is (c) plot of ln  n  as a function of ln ( n ) is a
marked then score = 2 × number of correct  r1 
C

answers marked ÷ actual number of correct straight line of slope 2


answers. r E 
(d) plot of ln  n 1  as a function of ln ( n ) is
dy

36. If the pressure, temperature and density of an  E n r1 


ideal gas are denoted by p, T and ρ, a straight line of slope 4
u

respectively, the velocity of sound in the gas is 40. A small steel ball bounces on a steel plate held
(a) proportional to p, when T is constant.
St

horizontally. On each bounce the speed of the


(b) proportional to T. ball arriving at the plate is reduced by a factor
(c) proportional to p, when ρ is constant. e (coefficient of restitution) in the rebound, so
(d) proportional to T. that V upward = eV downward
37. Two long parallel wires separated by 0.1 m If the ball is initially dropped from a height of
carry currents of 1 A and 2 A, respectively in 0.4 m above the plate and if 10 seconds later
opposite directions. A third current-carrying the bouncing ceases, the value of e is
wire parallel to both of them is placed in the 2 3
(a) (b)
same plane such that it feels no net magnetic 7 4
force. It is placed at a distance of 13 17
(c) (d)
(a) 0.5 m from the 1st wire, towards the 18 18
2nd wire
CHEMISTRY
Category-I (Q.41 to 70)  1  1
(a)  4, 1, − 1, +  (b)  4, 0 , 0 , + 
Direction Only one answer is correct. Correct  2  2
will fetch full marks 1. Incorrect answer or any  1   1
(c)  3, 2, 0, −  (d)  3, 2, − 2, + 
combination of more than one answer will detch  2  2
− 1/4 marks. No answer will fetch 0 marks.

)
45. 0.126 g of an acid is needed to completely

be
41. neutralise 20 mL 0.1 (N) NaOH solution. The
equivalent weight of the acid is

Tu
(a) 53 (b) 40
(c) 45 (d) 63
log V T=const.
46. In a flask, the weight ratio of CH 4 (g ) and

ou
SO 2(g ) at 298 K and 1 bar is 1 : 2. The ratio of
M1 M2
the number of molecules of SO 2(g ) and
log p CH 4 (g ) is
For same mass of two different ideal gases of
molecular weights M1 and M2, Plots of log V vs (Y (a) 1 : 4 (b) 4 : 1 (c) 1 : 2 (d) 2 : 1
47. C 6H 5F18 is a F18 radio-isotope labelled organic
on
log p at a given constant temperature are compound. F18 decays by positron emission.
shown. Identify the correct option. The product resulting on decay is
(a) M1 > M2 (a) C 6H 5O18 (b) C 6H 5Ar19
pi

(b) M1 = M2 (c) B12C 5H 5F (d) C 6H 5O16


(c) M1 < M2
m

(d) Can be predicted only if temperature is 48. Dissolving NaCN in de-ionised water will
known result in a solution having
ha

(a) pH < 7 (b) pH = 7


42. Which of the following has the dimension if (c) pOH = 7 (d) pH > 7
[ ML0T − 2]?
49. Among Me3N, C 5H 5N and MeCN
C

(a) Coefficient of viscosity


(b) Surface tension (Me = methyl group) the electronegativity of
(c) Vapour pressure N is in the order
dy

(d) Kinetic energy (a) MeCN > C 5H 5N > Me3N


(b) C 5H 5N > Me3N > MeCN
43. If the given four electronic configurations. (c) Me3N > MeCN > C 5H 5N
u

(i) n = 4, l = 1 (ii) n = 4, l = 0 (d) Electronegativity same in all


St

(iii) n = 3, l = 2 (iv) n = 3, l = 1 50. The shape of XeF5− will be


(a) square pyramid
are arranged in order of increasing energy, (b) trigonal bipyramidal
then the order will be (c) planar
(a) (iv) < (ii) < (iii) < (i) (d) pentagonal bipyramid
(b) (ii) < (iv) < (i) < (iii)
(c) (i) < (iii) < (ii) < (iv) 51. The ground state magnetic property of B2 and
(d) (iii) < (i) < (iv) < (ii) C 2 molecules will be
(a) B2 paramagnetic and C 2 diamagnetic
44. Which of the following sets of quantum (b) B2 diamagnetic and C 2 paramagnetic
numbers represents the 19 th electron of (c) Both are diamagnetic
Cr (Z = 24)? (d) Both are paramagnetic
WEST BENGAL (Engineering) Solved Paper 2017 7

52. The number of unpaired electrons in 60. The correct order of acid strengths of
[ NiCl 4 ]2 − , Ni(CO)4 and [Cu(NH 3 )4 ]2 + benzoic acid (X), peroxybenzoic acid (Y) and
respectively are p-nitrobenzoic acid (Z) is
(a) 2, 2, 1 (b) 2, 0, 1 (a) Y > Z > X
(c) 0, 2, 1 (d) 2, 2, 0 (b) Z > Y > X
(c) Z > X > Y
53. Which of the following atoms should have the
(d) Y > X > Z
highest I st electron affinity?
(a) F (b) O (c) N (d) C 61. The yield of acetanilide in the reaction

)
(100% conversion) of 2 moles of aniline with

be
54. PbCl 2 is insoluble in cold water. Addition of
1 mole of acetic anhydride is
HCl increases its solubility due to
(a) 270 g (b) 135 g
(a) formation of soluble complex anions like

Tu
(c) 67.5 g (d) 177 g
[ PbCl 3]−
(b) oxidation of Pb(II) to PB(IV) 62. The structure of the product P of the following
(c) formation of [Pb(H 2O)6]2 + reaction is

ou
(d) formation of polymeric lead complexes OH
55. Of the following compounds, which one is the (i) CO2 (high temperature

(Y
and high pressure)
strongest Bronsted acid in a aqueous + NaOH P
(ii) H3O+
solution?
(a) HClO 3 (b) HClO 2 (c) HOCl (d) HOBr
OMe
on
56. The correct basicity order of the following
lanthanide ions is OH OH
(a) La 3 + > Lu 3 + > Ce3 + > Eu 3 + CO2H CO2H
pi

(b) Ce3 + > Lu 3 + > La 3 + > Eu 3 + (a) (b)


(c) Lu 3 + > Ce3 + > Eu 3 + > La 3 +
m

CO2H HO2C
(d) La 3 + > Ce3 + > Eu 3 + > Lu 3 + OMe OMe
ha

57. When BaCl 2 is added to an aqueous salt


solution, a white precipitate is obtained. The OH OH
anion among CO 23 − , SO 23 − and SO 24 − that was CO2H
C

present in the solution can be (c) (d)


(a) CO 23 − but not any of the other two CO2H
dy

(b) SO 23 − but not any of the other two OMe OMe


(c) SO 24 − but not any of the other two
(d) Any of them 63. ADP and ATP differ in the number of
u

(a) phosphate units


St

58. In the IUPAC system, PhCH 2CH 2CO 2H is (b) ribose units
named as (c) adenine base
(a) 3-phenylpropanoic acid (d) nitrogen atom
(b) benzylacetic acid
(c) carboxyethylbenzene 64. The compound that would produce a
(d) 2-phenylpropanoic acid nauseating smell/odour with a hot mixture of
chloroform and ethanolic potassium
59. The isomerisation of 1-butyne to 2-butyne can hydroxide is
be achieved by treatment with (a) PhCONH 2
(a) hydrochloric acid (b) PhNHCH 3
(b) ammoniacal silver nitrate (c) PhNH 2
(c) ammoniacal cuprous chloride (d) PhOH
(d) ethanolic potassium hydroxide
8 WEST BENGAL (Engineering) Solved Paper 2017

65. For the reaction below The equilibrium constant for the reaction?
(i) PhMgBr, THF
H 2(g ) + CO 2(g ) - CO(g ) + H 2O(g )
(ii) H3O+
CN at 1200 K will be
the structure of the product Q is (a) 0.05 (b) 20
(c) 0.2 (d) 5.0
Ph Ph
(a) (b)
Category-II (Q.71 to Q.75)

)
OH O Direction Only one answer is correct. Correct

be
answer will fetch full marks 2. Incorrect answer or
any combination of more than one answer will
Ph Ph
fetch − 1/2 marks. No answer will fetch 0 marks.

Tu
(c) (d)
CN
NH2 71. In a close-packed body-centred cubic lattice of
potassium, the correct relation between the

ou
66. You are supplied with 500 mL each of 2N HCl atomic radius ( r ) of potassium and the
and 5N HCl. What is the maximum volume of edge-length (a) of the cube is
3M HCl that you can prepare using only these a a

(Y
two solutions? (a) r = (b) r =
2 3
(a) 250 mL (b) 500 nL 3 3
(c) 750 mL (d) 1000 mL (c) r = a (d) r = a
on
2 4
67. Which one of the following corresponds to a
72. Which of the following solutions will turn
photon of highest energy?
violet when a drop of lime juice is added to it?
(a) λ = 300 mm
pi

(b) v = 3 × 108 s− 1 (a) A solution of NaI


(c) v = 30 cm − 1 (b) A solution mixture of KI and NaIO 3
m

(d) ε = 6. 626 × 10− 27 J (c) A solution mixture of NaI and KI


(d) A solution mixture of KIO 3 and NaIO 3
ha

68. Assuming the compounds to be completely


73. The reaction sequence given below given
dissociated in aqueous solution, identify the
product R.
pair of the solutions that can be expected to be
C

CO2Me (i) Ag2O


isotonic at the same temperature. HO2C R
(ii) Br2, CCl4
(a) 0.01 M urea and 0.01 M NaCl
dy

(b) 0.02 M NaCl and 0.01 M Na 2SO 4 The structure of the product R is
(c) 0.03 M NaCl and 0.02 M MgCl 2
(a) Br
(d) 0.01 M sucrose and 0.02 M glucose CO2H
u

69. How many faradays are required to reduce


St

Br
1 mol of Cr2O72 − to Cr 3 + in acid medium?
(a) 2 (b) 3 CO2Me
(c) 5 (d) 6 (b) HO2C

70. Equilibrium constants for the following


reactions at 1200 K are given CO2Me
(c) HO2C
2H 2O(g ) - 2H 2(g ) + O 2(g ), Br
K1 = 6. 8 × 10− 8
2CO 2(g ) - 2CO(g ) + O 2(g ), (d) MeO2C
K 2 = 1. 6 × 10− 6
Br
WEST BENGAL (Engineering) Solved Paper 2017 9

74. Reduction of the lactol S (a) as a catalyst


(b) to make fused mixture conducting
O (c) to lower the melting temperature of the
OH with sodium borohydride gives mixture
(d) to decrease the rate of oxidation of
OH
carbon at anode
(a) O (b) O 78. The reduction of benzenediazonium chloride
to phenyl hydrazine can be accomplished by

)
be
O (a) SnCl 2, HCl (b) Na 2SO 3
(c) CH 3CH 2OH (d) H 3PO 2
(c) OH (d)
OH OH 79. The major product(s) obtained form the

Tu
following reaction of 1 mole of hexad-
75. What will be the normality of the salt solution
euteriobenzene is/are
obtained by neutralising x mL y (N) HCl with D

ou
y mL x(N) NaOH, and finally adding
D D
( x + y ) mL distilled water (i) Br2(1 mole), Fe
2( x + y ) xy (ii) H2O
(a) (b)

(Y
N N D
xy 2( x + y ) D
 2xy  x + y D
(c)   N (d)   N
x + y  xy 
on
D
D D
D
Category-III (Q. 76 to Q.80) D
D Br
pi

D
Direction One or more answer(s) is (are) (a) D
(b)
correct. Correct answer(s) will fetch full marks 2. D
m

D D D D
Any combination containing one or more D
D
incorrect answer will fetch 0 marks. Also no
ha

answer will fetch 0 marks. If all correct answers D D


are not marked and also no incorrect answer is Br Br D H
marked then score = 2 × number of correct (d)
C

(c)
answers marked ÷ actual number of correct D D D D
answers. Br D
dy

76. During electrolysis of molten NaCl, some


water was added. What will happen? 80. The conversion of CH 3 CH 2 COOH to
O CH 2
u

(a) Electrolysis will stop


CH 3 CH 2 CH ƒ  can be
St

(b) Hydrogen will be evolved ‚


(c) Some amount of caustic soda will be O CH 2
formed accomplished by
(d) A fire is likely (a) SOCl 2, LiAlH 4 , ethylene glycol.
77. The role of fluorspar, which is added in small (b) SOCl 2, KMnO 4 , NH 2NH 2
quantities in the electrolytic reduction of (c) SnCl 2, HCl, Na 2SO 3
alumina dissolved in fused cryolite is (d) HCl, SnCl 2 ethylene glycol
MATHEMATICS
Category-I (Q.1 to Q.50) 7. The value of det A, where
Direction Only one answer is correct. Correct  1 cos θ 0 
answer will fetch full marks 1. Incorrect answer or  
A =  − cos θ 1 cos θ , lies
any combination of more than one answer will  −1 − cos θ 1 

fetch-1/4 marks. No answer will fetch 0 marks.

)
(a) in the closed interval [1, 2]

be
1. The number of all numbers having (b) in the closed interval [0, 1]
5 digits, with distinct digits is (c) in the open interval (0, 1)
(a) 99999 (b) 9 × 9P4 (d) in the open interval (1, 2)

Tu
10
(c) P5 (d) 9 P4
8. Let f : R → R be such that f is injective and
2. The greatest integer which divides f ( x ) f ( y ) = f ( x + y ) for ∀ x, y ∈ R. If f ( x ),

ou
( p + 1) ( p + 2) ( p + 3) . . . . ( p + q) for all f ( y ), f (z ) are in G.P., then x, y , z are in
p ∈ N and fixed q ∈ N is (a) AP always
(a) p ! (b) q ! (c) p (d) q (b) GP always
3. Let (1 + x + x 2 )9 = a 0 + a1 x + a 2x 2
+ K + a18x18. Then,
(a) a 0 + a 2 + K + a18 = a1 + a 3 + K + a17
(Y (c) AP depending on the value of x, y , z
(d) GP depending on the value of x, y , z
9. On the set R of real numbers we define xPy if
on
(b) a 0 + a 2 + K + a18 is even and only if xy ≥ 0 . Then, the relation P is
(c) a 0 + a 2 + K + a18 is divisible by 9 (a) reflexive but not symmetric
pi

(d) a 0 + a 2 + K + a18 is divisible by 3 but not (b) symmetric but not reflexive
by 9 (c) transitive but not reflexive
m

(d) reflexive and symmetric but not


4. The linear system of equations
transitive
8x − 3y − 5z = 0
ha

 10. On R, the relation ρ be defined by ‘xρy holds if


5x − 8y + 3z = 0 has and only if x − y is zero or irrational’. Then,
3x + 5y − 8z = 0 (a) ρ is reflexive and transitive but not
C

(a) only zero solution symmetric


(b) only finite number of non-zero solutions (b) ρ is reflexive and symmetric but not
dy

(c) no non-zero solution transitive


(d) infinitely many non-zero solutions (c) ρ is symmetric and transitive but not
u

5. Let P be the set of all non-singular matrices of reflexive


(d) ρ is equivalence relation
St

order 3 over R and Q be the set of all


orthogonal matrices of order 3 over R. Then, 11. Mean of n observations x1 , x 2, . . . , x n is x. If an
(a) P is proper subset of Q observation x q is replaced by x q′ then the new
(b) Q is proper subset of P mean is
(c) Neither P is proper subset of Q nor Q is ( n − 1) x + x ′q
proper subset of P (a) x − x q + x ′q (b)
n
(d) P ∩ Q = φ, the void set ( n − 1) x − x q′ nx − x q + x ′q
(c) (d)
 x + 2 3x  x 0  n n
6. Let A =  , B =   . Then
 3 x + 2  5 x + 2
12. The probability that a non-leap year selected
all solutions of the equation det ( AB) = 0 is at random will have 53 Sunday is
(a) 1, − 1, 0, 2 (b) 1, 4, 0, − 2 (a) 0 (b) 1/7
(c) 1, − 1, 4, 3 (d) − 1, 4, 0, 3 (c) 2/7 (d) 3/7
WEST BENGAL (Engineering) Solved Paper 2017 11

13. The equation sin x(sin x + cos x ) = k has real 19. The common chord of the circles
solutions, where k is a real number. Then, x 2 + y 2 − 4x − 4y = 0 and 2x 2 + 2y 2 = 32
1+ 2 subtends at the origin an angle equal to
(a) 0 ≤ k ≤ π π
2 (a) (b)
(b) 2 − 3 ≤ k ≤ 2 + 3 3 4
π π
(c) 0 ≤ k ≤ 2 − 3 (c) (d)
6 2
1− 2 1+ 2
(d) ≤k≤ 20. The locus of the mid-points of the chords of

)
2 2
the circle x 2 + y 2 + 2x − 2y − 2 = 0, which

be
14. The possible values of x, which satisfy the make an angle of 90° at the centre is
trigonometric equation (a) x 2 + y 2 − 2x − 2y = 0
 x − 1 − 1  x + 1 π

Tu
tan − 1   + tan   = are (b) x 2 + y 2 − 2x + 2y = 0
 x − 2  x + 2 4 (c) x 2 + y 2 + 2x − 2y = 0
1 (d) x 2 + y 2 + 2x − 2y − 1 = 0
(a) ± (b) ± 2

ou
2 21. Let P be the foot of the perpendicular from
1
(c) ± (d) ± 2 x2 y 2
focus S of hyperbola 2 − 2 = 1 on the line

(Y
2
a b
15. Transforming to parallel axes through a point bx − ay = 0 and let C be the centre of the
( p, q), the equation hyperbola. Then, the area of the rectangle
on
2x 2 + 3xy + 4y 2 + x + 18y + 25 = 0 becomes
whose sides are equal to that of SP and CP is
2x 2 + 3xy + 4y 2 = 1. Then,
(a) 2ab (b) ab
(a) p = − 2, q = 3 (b) p = 2, q = − 3
( a 2 + b2 )
pi

a
(c) p = 3, q = − 4 (d) p = − 4, q = 3 (c) (d)
2 b
16. Let A( 2 , − 3) and B( − 2 , 1) be two angular
m

points of ∆ABC. If the centroid of the triangle 22. B is an extremity of the minor axis of an ellipse
moves on the line 2x + 3y = 1, then the locus whose foci are S and S′. If ∠SBS ′ is a right
ha

of the angular point C is given by angle, then the eccentricity of the ellipse is
1 1 2 1
(a) 2x + 3y = 9 (a) (b) (c) (d)
C

(b) 2x − 3y = 9 2 2 3 3
(c) 3x + 2y = 5 23. The axis of the parabola
(d) 3x − 2y = 3
x 2 + 2xy + y 2 − 5x + 5y − 5 = 0 is
dy

17. The point P( 3, 6) is first reflected on the line (a) x + y = 0 (b) x + y − 1 = 0


y = x and then the image point Q is again (c) x − y + 1 = 0 (d) x − y =
1
u

reflected on the line y = − x to get the image 2


St

point Q′. Then, the circumcentre of the


24. The line segment joining the foci of the
∆PQQ′ is
hyperbola x 2 − y 2 + 1 = 0 is one of the
(a) (6, 3) (b) ( 6, − 3)
diameters of a circle. The equation of the
(c) ( 3, − 6) (d) (0, 0)
circle is
18. Let d1 and d 2 be the lengths of the (a) x 2 + y 2 = 4 (b) x 2 + y 2 = 2
perpendiculars drawn from any point of the (c) x 2 + y 2 = 2 (d) x 2 + y 2 = 2 2
line 7 x − 9y + 10 = 0 upon the lines
3x + 4y = 5 and 12x + 5y = 7, respectively. 25. The equation of the plane through (1, 2 , − 3)
Then, and ( 2 , − 2 , 1) and parallel to X-axis is
(a) d 1 > d 2 (b) d 1 = d 2 (a) y − z + 1 = 0 (b) y − z − 1 = 0
(c) d 1 < d 2 (d) d 1 = 2d 2 (c) y + z − 1 = 0 (d) y + z + 1 = 0
12 WEST BENGAL (Engineering) Solved Paper 2017

26. Three lines are drawn from the origin O with 33. Let f ( x ) = x13 + x11
direction cosines proportional to (1, − 1, 1), + x 9 + x7 + x 5+ x 3 + x + 19. Then, f ( x ) = 0
( 2 , − 3, 0) and (1, 0, 3). The three lines are has
(a) not coplanar (a) 13 real roots
(b) coplanar (b) only one positive and only two negative
(c) perpendicular to each other real roots
(d) coincident (c) not more than one real root
27. Consider the non-constant differentiable (d) has two positive and one negative real

)
function f of one variable which obeys the root

be
f (x)  xp π
relation = f ( x − y ). If f ′ ( 0) = p and  , if 0 < x ≤
f (y ) 34. Let f ( x ) =  (sin x )q 2,

Tu
f ′ ( 5) = q, then f ′ ( − 5) is  0 , if x = 0
p2 q ( p, q ∈ R ). Then, Lagrange’s mean value
(a) (b)
q p theorem is applicable tof ( x ) in closed

ou
p interval [ 0, x]
(c) (d) q
q (a) for all p, q
(b) only when p > q
28. If f ( x ) = log 5log 3 x, then f ′ ( e) is equal to
(a) e log e 5
(c)
1
(b) e log e 3
(d)
1
(Y (c) only when p < q
(d) for no value of p, q
on
e log e 5 e log e 3 35. lim(sin x )2 tan x is equal to
x→ 0

− x (a) 2 (b) 1
29. Let F( x ) = e , G( x ) = e
x
and H( x ) = G( F( x )),
(c) 0 (d) does not exist
pi

dH
where x is a real variable. Then, at x = 0
dx 36. ∫ cos(log x ) dx = F( x ) + C, where C is an
m

is arbitrary constant. Here, F( x ) is equal to


1
(b) − 1 (c) − (d) − e (a) x [cos (log x ) + sin (log x )]
ha

(a) 1
e (b) x [cos (log x ) − sin (log x )]
x
30. If f ′ ′ ( 0) = k , k ≠ 0, then the value of (c) [cos (log x ) + sin (log x )]
C

2
2f ( x ) − 3f ( 2x ) + f ( 4x ) x
lim is (d) [cos (log x ) − sin (log x )]
x→ 0 x2 2
dy

(a) k (b) 2k (c) 3k (d) 4k


x2 − 1
31. If y = e m sin − 1 x
then 37. ∫x + 3x 2 + 1
4
dx ( x > 0) is
u

d 2y dy  1
(1 − x ) 2 − x
2
− ky = 0, where k is (a) tan − 1  x +  + C
St

dx dx  x
equal to
 1 
(a) m 2 (b) 2 (c) − 1 (d) − m 2 (b) tan − 1  x −  + C
 x
32. The chord of the curve y = x 2 + 2ax + b, 1
joining the points where x = α and x = β, is x+ −1
(c) log e x +C
parallel to the tangent to the curve at abscissa 1
x is equal to x+ +1
x
a+b 2a + b 1
(a) (b) x− −1
2 3 x
2α + β α +β (d) log e +C
(c) (d) 1
3 2 x− +1
x
WEST BENGAL (Engineering) Solved Paper 2017 13

sin x 19
44. The integrating factor of the first order
38. Let I = ∫
1 + x8
dx. Then,
10 differential equation
dy
(a)| I| < 10− 9 (b)| I| < 10− 7 x 2( x 2 − 1) + x( x 2 + 1) y = x 2 − 1 is
− 5
(c)| I| < 10 (d)| I| > 10− 7 dx
1
n n (a) ex (b) x −
39. Let I1 = ∫0
[ x] dx and I2 = ∫
0
{ x} dx, where [x] x
1 1
and {x} are integral and fractional parts of x (c) x + (d) 2
and n ∈ N − {1}. Then, I1 / I2 is equal to x x

)
1 1

be
(a) (b) (c) n (d) n − 1 45. In a GP series consisting of positive terms,
n−1 n each term is equal to the sum of next two
terms. Then, the common ratio of this
40. The value of

Tu
GP series is
 n n 1
lim + 2 + ... + is 5−1
n → ∞  n 2 + 12 n +2 2
2n  (a) 5 (b)
 2

ou
nπ π 5 5+1
(a) (b) (c) (d)
4 4 2 2
π π

(Y
(c) (d) 46. If (log 5 x ) (log x 3x ) (log 3x y ) = log x x 3, then y
4n 2n
1
equals
41. The value of the integral ∫ ex dx
2
(a) 125 (b) 25
on
0
(c) 5/3 (d) 243
(a) is less than 1
(b) is greater than 1 (1 + i )n
47. The expression equals
(1 − i )n − 2
pi

(c) is less than or equal to 1


(d) lies in the closed interval [1, e]
(a) − i n + 1 (b) i n +1
m

100
(c) − 2 i n + 1

x − [ x] (d) 1
42. e dx is equal to
0

−1 48. Let z = x + iy , where x and y are real. The


ha

100
e
(a) z+i
100 points ( x, y ) in the X-Y plane for which is
e100 − 1 z−i
(b)
C

e−1 purely imaginary, lie on


(c) 100( e − 1) (a) a straight line
e−1
dy

(b) an ellipse
(d)
100 (c) a hyperbola
(d) a circle
u

dy
43. Solution of ( x + y )2 = a2 49. If p, q are odd integers, then the roots of the
dx
St

(‘a’ being a constant) is equation 2px 2 + ( 2p + q) x + q = 0 are


(x + y ) y +C (a) rational (b) irrational
(a) = tan , C is an arbitrary (c) non-real (d) equal
a a
constant 50. Out of 7 consonants and 4 vowels, words are
(b) xy = a tan Cx, C is an arbitrary constant formed each having 3 consonants and 2
x y vowels. The number of such words that can be
(c) = tan , C is an arbitrary constant
a C formed is
(d) xy = tan( x + C ), C is an arbitrary (a) 210 (b) 25200
constant (c) 2520 (d) 302400
14 WEST BENGAL (Engineering) Solved Paper 2017

Category-II (Q.51 to Q.65) Then,


Direction Only one answer is correct. Correct (a) R ∪ S is an equivalence relation
answer will fetch full marks 2. Incorrect answer or (b) R ∪ S is reflexive and transitive but not
any combination of more than one answer will symmetric
fetch -1/2 marks. No answer will fetch 0 marks. (c) R ∪ S is reflexive and symmetric but not
 1 1 1 transitive
  (d) R ∪ S is symmetric and transitive but not
51. Let A =  0 1 1 . Then, for positive integer
 0 0 1

)
reflexive
 

be
n, A n is 54. If one of the diameters of the curve
 1 n n 2 x 2 + y 2 − 4x − 6y + 9 = 0 is a chord of a circle
 

Tu
with centre (1, 1), the radius of this circle is
(a)  0 n 2 n  (a) 3 (b) 2
0 0 n 
  (c) 2 (d) 1

ou
  n + 1  55. Let A( − 1, 0) and B( 2, 0) be two points. A
1 n n 
 2  point M moves in the plane in such a way that
 
(b)  0 1 n  ∠MBA = 2∠MAB. Then, the point M moves
0 0


1 n2 n 
1 

 (Y along
(a) a straight line (b) a parabola
on
(c) an ellipse (d) a hyperbola
 
(c)  0 n n 2 x

 0 0 n 2
56. If f ( x ) = ∫−1
| t| dt, then for any x ≥ 0, f ( x ) is
 
pi

equal to
  1
1 2n − 1 (a) (1 − x 2 ) (b) 1 − x 2
m

n 2
 n+1 
(d)  0 n2  1
(c) (1 + x 2 ) (d) 1 + x 2
ha

 2  2
0 n +1

0
2  57. Let for all x > 0, f ( x ) = lim n( x1 / n − 1), then
n→∞
C

52. Let a, b, c be such that b( a + c) ≠ 0.  1


(a) f ( x ) + f   = 1
 x
a a+1 a−1
dy

If − b b + 1 b − 1 (b) f ( xy ) = f ( x ) + f ( y )
c c−1 c+1 (c) f ( xy ) = xf ( y ) + yf ( x )
u

(d) f ( xy ) = xf ( x ) + yf ( y )
a+1 b+1 c−1
St

100π
+ a−1
n + 2
b−1
n +1
c+1 = 0, 58. Let I = ∫0
(1 − cos 2x ) dx , then
( − 1) a ( − 1) b ( − 1) c
n
(a) I = 0 (b) I = 200 2
then the value of n is (c) I = π 2 (d) I = 100
(a) any integer (b) zero
(c) any even integer (d) any odd integer 59. The area of the figure bounded by the
parabolas x = − 2y 2 and x = 1 − 3y 2 is
53. On set A = {1, 2, 3}, relations R and S are 4 2
given by (a) sq units (b) sq units
3 3
R = {(1, 1), ( 2, 2), ( 3, 3), (1, 2), ( 2 , 1)}, 3 6
S = {(1, 1), ( 2, 2), ( 3, 3), (1, 3), ( 3, 1)}. (c) sq units (d) sq units
7 7
WEST BENGAL (Engineering) Solved Paper 2017 15

x2 y 2 66. If a, b ∈ {1, 2, 3} and the equation


60. Tangents are drawn to the ellipse + =1
9 5 ax 2 + bx + 1 = 0 has real roots, then
at the ends of both latusrectum. The area of (a) a > b
the quadrilateral, so formed is (b) a ≤ b
13 (c) number of possible ordered pairs ( a, b)
(a) 27 sq units (b) sq units
2 is 3
15 (d) a < b
(c) sq units (d) 45 sq units
4 67. If the tangent to y 2 = 4ax at the point

)
( at 2, 2at ) where | t | > 1 is a normal to

be
61. The value of K in order that
f ( x ) = sin x − cos x − Kx + 5 decreases for all x 2 − y 2 = a 2 at the point ( a sec θ, a tan θ ), then
positive real values of x is given by (a) t = − cosec θ

Tu
(a) K < 1 (b) K ≥ 1 (b) t = − sec θ
(c) K > 2 (d) K < 2 (c) t = 2 tan θ
(d) t = 2 cot θ

ou
62. For any vector x , where $i , $j, k$ have their
68. The focus of the conic x 2 − 6x + 4y + 1 = 0 is
usual meanings the value of (a) (2, 3) (b) (3, 2)
( x × $i )2 + ( x × $j )2 + ( x × k$ )2 where $i, $j, k$

(Y
(c) (3, 1) (d) (1, 4)
have their usual meanings, is equal to
69. Let f : R → R be twice continuously
(a)| x |2 (b) 2| x |2 (c) 3| x |2 (d) 4| x |2 differentiable. Let f ( 0) = f (1) = f ′ ( 0) = 0.
on
63. If the sum of two unit vectors is a unit vector, Then,
then the magnitude of their difference is (a) f ′ ′ ( x ) ≠ 0 for all x
(a) 2 units (b) 2 units (b) f ′′( c) = 0 for some c ∈ R
pi

(c) 3 units (d) 5 units (c) f ′′( x ) ≠ 0 if x ≠ 0


(d) f ′ ( x ) > 0 for all x
m

64. Let α and β be the roots of x 2 + x + 1 = 0. If n


be a positive integer, then α n + β n is 70. If f ( x ) = x n , n being a non-negative integer,
ha

2nπ 2nπ then the values of n for which


(a) 2 cos (b) 2 sin f ′ (α + β ) = f ′ (α ) + f ′ (β ) for all α, β > 0 is
3 3
nπ nπ (a) 1 (b) 2
C

(c) 2 cos (d) 2 sin (c) 0 (d) 5


3 3
x 2 + 2x + 4 71. Let f be a non-constant continuous function
dy

65. For real x, the greatest value of is for all x ≥ 0. Let f satisfy the relation
2x 2 + 4x + 9
f ( x ) f ( a − x ) = 1 for some a ∈ R + . Then,
1 1 dx
(a) 1 (b) − 1 (c) (d) I= ∫
a
u

is equal to
2 4 0 1 + f (x)
St

a
Category-III (Q. 66 to Q.75) (a) a (b)
4
Direciton One or more answer(s) is/are a
correct. Correct answer(s) will fetch full marks 2. (c) (d) f ( a )
2
Any combination containing one or more
incorrect answer will fetch 0 marks. Also no 72. If the line ax + by + c = 0, ab ≠ 0, is a tangent
answer will fetch 0 marks. If all correct answers to the curve xy = 1 − 2x, then
are not marked and also no incorrect answer is (a) a > 0, b < 0
marked, then score = 2 × number of correct (b) a > 0, b > 0
answers marked ÷ actual number of correct (c) a < 0, b > 0
answers. (d) a < 0, b < 0
16 WEST BENGAL (Engineering) Solved Paper 2017

73. Two particles move in the same straight line 74. The complex number z satisfying the equation
starting at the same moment from the same | z − i| = | z + 1| = 1 is
point in the same direction. The first moves (a) 0
with constant velocity u and the second starts (b) 1 + i
from rest with constant acceleration f. Then, (c) − 1 + i
(a) they will be at the greatest distance at (d) 1 − i
u 75. On R, the set of real numbers, a relation ρ is
the end of time from the start
2f defined as ‘aρb if and only if 1 + ab > 0’. Then,

)
(b) they will be at the greatest distance at (a) ρ is an equivalence relation

be
u
the end of time from the start (b) ρ is reflexive and transitive but not
f symmetric
u2

Tu
(c) their greatest distance is (c) ρ is reflexive and symmetric but not
2f transitive
u2 (d) ρ is only symmetric
(d) their greatest distance is

ou
f

Physics
ANSWERS (Y
on
1. (b) 2. (a) 3. (d) 4. (a) 5. (c) 6. (c) 7. (d) 8. (a) 9. (c) 10. (a)
11. (b) 12. (b) 13. (a) 14. (b) 15. (d) 16. (c) 17. (a) 18. (d) 19. (a) 20. (b)
pi

21. (c) 22. (d) 23. (c) 24. (a) 25. (b) 26. (d) 27. (b) 28. (a) 29. (b) 30. (a)
31. (b) 32. (b) 33. (d) 34. (*) 35. (b) 36. (b,c) 37. (c) 38. (b,d) 39. (a,b,c,d) 40. (d)
m

Chemistry
ha

41. (a) 42. (b) 43. (a) 44. (b) 45. (d) 46. (c) 47. (a) 48. (d) 49. (a) 50. (c)
51. (a) 52. (b) 53. (a) 54. (a) 55. (a) 56. (d) 57. (d) 58. (a) 59. (d) 60. (c)
C

61. (b) 62. (c) 63. (a) 64. (c) 65. (b) 66. (c) 67. (a) 68. (c) 69. (d) 70. (d)
71. (d) 72. (b) 73. (d) 74. (c) 75. (b) 76. (b,c,d) 77. (b, c) 78. (a,b) 79. (a) 80. (a,d)
dy

Mathematics
1. (b) 2. (b) 3. (b) 4. (d) 5. (b) 6. (b) 7. (a) 8. (a) 9. (d) 10. (b)
u

11. (d) 12. (b) 13. (d) 14. (a) 15. (b) 16. (a) 17. (d) 18. (b) 19. (d) 20. (c)
St

21. (b) 22. (b) 23. (a) 24. (c) 25. (d) 26. (b) 27. (a) 28. (c) 29. (c) 30. (c)
31. (a) 32. (d) 33. (c) 34. (b) 35. (b) 36. (c) 37. (a) 38. (b) 39. (d) 40. (b)
41. (d) 42. (c) 43. (a) 44. (b) 45. (b) 46. (a) 47. (c) 48. (d) 49. (a) 50. (b)
51. (b) 52. (d) 53. (c) 54. (a) 55. (d) 56. (c) 57. (b) 58. (b) 59. (a) 60. (a)
61. (c) 62. (b) 63. (c) 64. (a) 65. (c) 66. (c) 67. (a, c) 68. (c) 69. (b) 70. (b)
71. (c) 72. (b, d) 73. (b,c) 74. (a,c) 75. (c)
HINTS & SOLUTIONS
Physics
1. (b) The speed of a particle executing simple 3. (d) When the sealed capillary tube is
harmonic motion is v = ω a 2 − x 2 submerged vertically into water, the pressure
inside the tube changes

)
where, a = Amplitude

be
ω = Angular frequency For the inside capillary,
x = Displacement p1 V1 = p 2V 2
∴ p 0( lA ) = p ′ ( l − x ) A

Tu
or v 2 = ω 2( a 2 − x 2 )
According to the question, where p′ is pressure in capillary after being
v 12 = ω 2( a 2 − x12 ) submerged

ou
v 22 = ω 2( a 2 − x 22 ) pl
∴ p′ = 0
v 1 − v 22 = ω 2( x 22 − x12 )
2 l−x

ω=
v 12 − v 22
x 22 − x12
v 1 = 13 m/s
(Y According to question, since level of water
inside capillary coincides with outside.
∴ p′ − p0 =

on
Here
r
v 2 = 12 m/s
p 0l 2γ l
x1 = 3 m ∴ − p0 = =
l−x
pi

r  p 0r 
x2 = 5 m 1 + 
 2γ 
m

(13)2 − (12)2
ω= 4. (a) The Bulk modulus k
( 5)2 − ( 3)2
ha

p
169 − 144 25 5 k=− … (i)
= = = ∆V / V
25 − 9 16 4 Where, p = Pressure
C

ω = 2πr ∆V = Change in volume


ω 5/ 4 V = Volume of liquid
v= =
dy

2π 2π From (i)
5 p ∆V ∆p
v= =− =
u

8π k V p
St

2. (a) The formula of frequency of 3 vibrations k∆p


⇒ p=
produced in tensed wire is p
n T ∆p = 0 ⋅ 01% = 0 ⋅ 01 / 100
v=
2 m k
m p=
m = Mass per unit length of the wire = 10000
L
5. (c) Initial temperature of ideal gas,
M = Mass of wire
T1 = 273 + 27 = 300 K
L = Length of wire
when temperature of gas is raised by 6°C, the
n = Number of loops produced in wire final temperature of gas
n T n T T2 = 273 + 6 + 27
v= =
2L M / L 2 ML = 306 K
18 WEST BENGAL (Engineering) Solved Paper 2017

Let initial velocities are v rms1 , and v rms2 The coefficient of linear expansion along its
v rms ∝ T breadth = α 2
v rms1 Increase in length,
T1
= L t = l 0(1 + α1 ∆t )
v rms2 T2
Increase in breadth,
T2 Bt = b0(1 + α 2∆t 2 )
v rms2 = × v rms1
T1 Let coefficient of surface expansion is β

)
306 Area = length × breadth
= × v rms1

be
300 = l 0(1 + α1 ∆t ) × b0(1 + α 2∆t )
= 1 ⋅ 00 × v rms1 = l 0b0(1 + α1 ∆t )(1 + α 2∆t )

Tu
So, it will increase by 1% = S 0(1 + α1 ∆t + α 2∆t + . . . )
6. (c) The internal energy where, S 0 = l 0 ⋅ b0
∆U = nC v∆T = Initial area of surface

ou
C v = Specific heat of gas at constant volume In state of expansion,
3R  4p 0V 0 p 0V 0  S t = Lt × Bt
⇒ ∆U = n ⋅  − 

= n⋅
2  nR

2 

nR 
3R  4p 0V 0 − p 0V 0 


(Y = l 0b0(1 + α1 ∆t ) (1 + α 2∆t )
= S 0(1 + α1 ∆t + α 2∆t + . . . )
S t = S 0(1 + β∆t )
on
nR
3R 3p 0V 0 ∴ S 0(1 + β∆t ) = S 0 (1 + α1 ∆t + α 2∆t + . . . )
= n⋅ ⋅ β ⋅ ∆t = α1 ∆t + α 2∆t
2 nR
pi

9 β = α1 + α 2
= p 0V 0 …(i)
m

2 8. (a) The positive charge Q is situated at the


Work done by the gas centre of a cube
ha

V 3p 0V 0
W = ( 2p 0 + p 0 ) 0 = …(ii)
2 2
From first law of thermodynamics,
C

∆Q = dW + dU
3p 0V 0 9
= + p 0V 0
dy

According to Gauss theorem, the flux from


2 2
any closed surface
[from Eqs. (i) and (ii)] Q
φE =
u

3p 0V 0 9
= + p 0V 0 ε0
St

2 2
12p 0V 0 Cube has 6 faces, so electric flux from any
= = 6p 0V 0 Q
2 face φ E =
6ε 0
7. (d) The coefficient of linear expansion along
its length = α1 9. (c) 1µF 2µF 5µF

C1 C2 C3

α1 L When the capacitors are connected in series,


the resultant capacitance of combination
1 1 1 1
B = + +
α2 C C1 C 2 C 3
WEST BENGAL (Engineering) Solved Paper 2017 19

1 1 1 17 1
= + + = µF So, B∝
1 2 5 10 r2
10 So, the magnetic field due to current
C= µF
17 decreases as inverse square of distance of the
The charge will be same on all the capacitors point of observation.
in series 12. (b) In galvanometer I ∝ θ = Gθ
10 100 k
Q = CV = × 10 = Where, G=
17 17

)
NAB

be
The potential difference across 2.0µF When coil is in square of shape,
capacitor k k
Q 100 / 17 50 G= = … (i)
V′ = = = NAB Na 2B

Tu
volt
C 2 17 When coil is in circular shape of radius
10. (a) Q1 Q2 a/ π

ou
30 cm k k
G= =
Checking the given options one by one NAB Nπr 2 ⋅ B
k k

(Y
(i) Q1 = Q 2 = 0 ⋅ 4 C = = … (ii)
2
The force on Q1 due to Q 2  a  Na 2B
Nπ   ⋅B
Q1 × Q 2  π
F = kp
on
30 × 10− 2 So, value of G is same for both type of shape
Q Q × 100 of coil. So, the torque e = NBIA sin θ will be
=k 1 2
same in both the case
pi

30
0 ⋅ 4 × 0 ⋅ 4 × 100 e: e = 1:1
=k
m

30 13. (a) Velocity of proton = 106 m/s along


k × 0 ⋅ 16 × 100 y-direction
=
ha

30 Y
(ii) When Q1 = 0 ⋅ 8C Q 2 ≈ 0
k × Q1Q 2
C

F= ≈0
30 × 10− 2 X

(iii) when Q1 ≈ 0, Q 2 = 0 ⋅ 8 C
dy

Z
F=0
(iv) Q1 = 0 ⋅ 2 C Q 2 = 0 ⋅ 6 C Electric field = 2 × 104 V/m
u

k ⋅ 0⋅ 2 × 0⋅ 6 e
F= for proton = 10−8C / kg
St

30 × 10− 2 m
The magnetic field,
k × 0 ⋅ 12 × 102
= E
30 B= (Q qp vB = qp E )
v
Thus we find that, in option(a), the force on
2 × 104
Q1 will be maximum. = = 2 × 10− 2 T
106
∴ correct answer is Q1 = Q 2 = 0 ⋅ 4 C
The radius of circular path
11. (b) By Biot-Savart’s law, the magnetic field mv 10− 8 × 106
due to a current carrying wire is γ= =
qp ⋅ B 2 × 10− 2
µ Idl sin θ
B= 0 ⋅ 1
4π r2 = = 0⋅ 5 m
2
20 WEST BENGAL (Engineering) Solved Paper 2017

14. (b) The impedance of LCR circuit 16. (c) R


2
 1
Z = R 2 + ω C − 
 ωC R X
E
Where, R = Resistance
L = Inductance
C = Capacitance For above circuit
1 1 1 R+X

)
When frequency of AC voltage applied to a = + =
R′ R X

be
series LCR circuit, it will first increase and RX
1 RX
resonance state ω L = , it will decrease. In R′ =
ωC R+X

Tu
that state (Z = R ), it will be minimum. The current in the circuit
So, it will first increases and then decreases. E E
i= =

ou
15. (d) Six wires each of resistance r form a R + R′  RX 
R + 
tetrahedron as shown in the following figure  R + X

(Y
RX
1 E⋅
R+X EX
V RX = =
 RX  R + 2X
R + 
on
 R + X
Power dissipated in the circuit
3
pi

V2 E2 ⋅ X 2
2 PX = RX =
4 X X ( R + 2X )2
m

E 2X
The equivalent circuit of this tetrahedron =
( R + 2X )2
ha

3
dPX ( R − 2X )
= E2
dX ( R + 2X )3
C

1 2 E 2( R − 2X )
⇒ dPX = ⋅ dX
( R + 2X )3
dy

( dPX ) will be zero for all ( dX ) if


  R 
u

4  E2  R − 2 ×  
R dPX =  2 
X=
St

 R 
3
2 
It is circuit of Wheatstone the equivalent  R + 2 ×  
  2 
resistance of upper circuit
1 1 1 2 1 =0
= + = =
R 2r 2r 2r r 17. (a) 21mA 8mA 3mA 1mA
R=r A

It will in parallel with outer resistance 34mA 13mA 5mA 2mA X


1 1 1 2
= + = B
R eq r r r
The potential between points A and B
r
R eq = V AB = 34 volt
2
WEST BENGAL (Engineering) Solved Paper 2017 21

18. (d) 20. (b) As the lens is placed on a horizontal thin


plane mirror.
D d O

dSE

The angle subtended by sun on mirror


D d
Q= =

)
d SE f

be
D
∴ d= ×f So due to role of plane mirror the image will
d SE be formed at the same point O above 0.1 m

Tu
According to the question, distance above the center of the lens.
D
= 0 ⋅ 009 21. (c) Given, radius R = 0 ⋅ 05 m
d SE

ou
Refractive index n = 1 ⋅ 5
r 0.4
f= = = 0⋅ 2 m
2 2
∴ d = 0 ⋅ 009 × 0 ⋅ 2 m
= 9 × 2 × 10− 4 m
= 18 × 10− 4 m
(Y R
c
Y
on
= 1 ⋅ 8 × 10− 3 m c
R X
19. (a) For maximum intensity,
pi

1 1
Imax = I1 + I2 + 2 I1 I2 cos δ The critical angle, sin c = =
n 1⋅ 5
m

δ=0
10 2
L L = =
ha

Imax = L + + 2 L × ⋅ cos 0 15 3
4 4 From above figure,
5L 2L R
= + cos c =
C

4 2 R+X
5L + 4L R
= 1 − sin 2 c =
dy

4 R+X
9L
= 4 R
1− =
u

4
9 R+X
Imin = I1 + I2 − 2 I1 I2 cos θ
St

5 R
θ = 0° =
3 R+X
L L 5( R + X ) = 3R
=L+ − 2 L× ×1
4 4
5R + 5X = 3R
5L L
= − L= ∴ X = ( 3 5 − 5) × 10− 2 m
4 4
Thus, maximum and minimum intensities 22. (d) de-Broglie wavelength,
 9L L  h
will be  ,  . λ=
 4 4 2mE
22 WEST BENGAL (Engineering) Solved Paper 2017

λ1 E2 Initially mass of radon = 0 ⋅ 064 kg


∴ =
λ2 E1 T1
0 ⋅ 064  
→ 0 ⋅ 032  
T2
→
λ 1 = Wave length of electron T3
0 ⋅ 016  
→ 0 ⋅ 008
= 0 ⋅ 4 × 10− 10 m
T4 T5
E1 = 1 ⋅ 0keV  → 0 ⋅ 004   → 0 ⋅ 002
λ 2 = 1 ⋅ 0 × 10− 10 m So in five half-life periods, the Radon sample
E2 = ? will reduce to 0.002 kg

)
5
0 ⋅ 4 × 10− 10 E2 N  1

be
= Or = 
1 × 10 − 10
1 ⋅ 0 keV N 0  2
1
4 E2 ⇒ N = 0 ⋅ 064 ×

Tu
= 32
10 1
16 E = 0 ⋅ 002 kg
= 2

ou
100 1 25. (b) 1
A 1
E 2 = 0 ⋅ 16keV B
1
1

(Y
23. (c) From Einstein’s photoelectric equation, 1 0 0 Y
1 0
hv = W + eV 0
W = Work function
on
v = Frequency of incident photon The output of logic gate circuit
V = Stopping potential Y = A ⋅B+ A B
pi

For a photon of frequency v 1 , for A =1


hv 1 = W + eV1 …(i) B = 1⇒Y = 0
m

For photon of frequency v 2, and for A = 0, B = 0


hv 2 = W + eV 2 …(ii) ⇒ Y=0
ha

Divided Eq. (i) by Eq. (ii),


26. (d) When the battery and resistance is
hv 1 W + eV1
= connected with pn diode,
C

hv 2 W + eV 2
v 1 W + eV1 p n
=
dy

v 2 W + eV 2
⇒ Wv 1 + eV 2v 1 = Wv 2 + eV1 v 2
u

Solving above equation, + – R


W( v 2 − v 1 )
St

⇒ e= – +
V 2v 1 − V1 v 2 In case of forward bias that p is connected to
24. (a) Half life of radon = 3 ⋅ 8 days + ve terminal of battery, current flows.
If one reverse the polarity i.e., p-end is made
Total half-lives in 19 days
negative, no current flows in semiconductor
19
= diode.
3⋅ 8
190 27. (b) The dimensions of universal constant G, is
= obtained by Newton’s law of gravitation
38
Gm1 m 2
= 5 half-lives F=
r2
WEST BENGAL (Engineering) Solved Paper 2017 23

F × r2 30. (a) From Newton’s second law


G=
m1 m 2 F = ma
F = Force m = 1 kg
r = Distance dv
F=1× a=
m1 and m 2 are masses dt
[ M1 L1 T − 2][ L2] According to the question,
G= dv
[ M][ M] F = kt =

)
dt
[ M 1 L3T − 2]

be
= = [ M− 1 L3T − 2] dv = kt ⋅ dt
[ M2]
Integration gives
28. (a) In the absence of external force the speed

Tu
kt 2 dx
of centre of mass is always zero i.e. v= =
2 dt
u cm = 0 kt 3 1 × 6 × 6 × 6
x= =

ou
Q Fext = 0 then, 6 6
v cm. = 0 always = 36 m
29. (b) Three vectors are mutually perpendicular,
so the scalar or dot product of two vectors will
zero. So, (Y
31. (b) According to the question, when tension
in the string is zero.
on
kQ2
A ⋅ B = 0, B⋅ C = 0, A ⋅ C = 0 V R2
So, A ⋅ B = ( a$i + $j + k$ ) ⋅ ( $i + b$j + k$ ) = 0
pi

Mg
⇒a + b+1= 0 …(i)
Q
B ⋅ C = ( $i + b$j + k$ ) ⋅ ( $i + $j + ck$ ) = 0
m

⇒1 + b + c = 0 … (ii)
ha

v0
A ⋅ C = ( a$i + $j + k$ ) ⋅ ( $i + $j + ck$ ) = 0
⇒ a+1+ c= 0 … (iii) As T = 0
C

Addition of Eqs. (i), (ii) and (iii), KQ 2 mv 2  1 


Mg − = k = 
2( a + b + c) + 3 = 0 R2 R  4πε 0 
dy

3 mv 2
a+b+c=− = centripetal force required to keep the
2 R
3
u

⇒ −1+ c= − body in a circular path


2  kQ 2 
St

⇒ v=0 Q Mg =
c= − +1
3
 R 2 
2
1 So the required work done to keep charge in
= − = 0⋅ 5 vertical motion
2
1 ∴ Wg = ∆KE = change in K.E.
∴ a=−
2 1
⇒ mg( 2R ) = mv 20
1 2
b=−
2 v 20 = 2g ⋅ 2R = 4gR
1 v 0 = 4gR = 2 gR
c=−
2
24 WEST BENGAL (Engineering) Solved Paper 2017

32. (b) Let mass of bullet = m 34. (*)


+Q
Mass of block = M
Velocity of bullet = v = 300 m/s F
Velocity of combined system M + m = V θ
θ –1C
Here, from momentum conservation F
M+ m
V =v
m +Q

)
300 × 4. 2 × 10−2 From figure the net force

be
⇒ V =
4.2 × 10−2 + 9 ( 4.2 × 10−2 ) Fnet = − F cos θ + ( − F cos θ )
= 30 m/s = − 2F cosθ

Tu
kQ × 1 x
Now, heat produced = Loss in kinetic energy = − 2× 2 ×
of bullet (x + a ) 2
x + a2
2

1 1

ou
= mv − ( M + m )V
2 2 2kQ
=− ⋅x
2 2 ( x 2 + a 2 )3/ 2
1 1
= × 4.2 × 10−2 ( 300)2 − ( 4.2 × 10−2  2kQ 

(Y
2 2 ∴ Fnet = −  3  ⋅ x
 a 
+ 9 × 4.2 × 1) ( 30)2
Frequency of oscillation
= 6.3 × 270
on
1 2kQ
= 1701 J =
2π Ma 3
1701
= Cal
pi

1
4.2 2× ⋅Q
= 405 Cal 1 4πε 0
=
m

2π Ma 3
33. (d) From parabola
1 Q
ha

Y =
2π 2πε 0Ma 3
E
⇒ (* None of the option matches)
C

35. (b)
1 µF 2µF
dy

y
A C D
B
X
u

1kΩ 2kΩ
St

x=ut
3kΩ
1  Ee X 2 3V
y= × ×
2  m  u 2 From circuit the current,
E⋅e E
= ⋅ X2 I=
2mu 2 R
As X = 4ay (for parabola) The equivalent resistance of above circuit
2 = 3+ 2+1
2mu
∴ X2 = ⋅y = 6k Ω
E⋅e
= 6 × 103 Ω
2mu 2 mu 2
a= = E = 3 volt
4E ⋅ e 2E ⋅ e
WEST BENGAL (Engineering) Solved Paper 2017 25

3 that fields due to wire (1) and (2) must act


I=
6 × 103 opposite to each other and must be equal.
= 0.5 × 10− 3 A The magnetic field due to long straight wire,
µ ⋅I
Potential, B= 0
V AD = iR = 0 ⋅ 5 × 10− 3 × 3 × 103 2πx
= 1 ⋅ 5V ∴ B1 = B2
µ 0i1 µ 0i 2
Charge, =
2πx 2π( 0 ⋅ 1 + x )

)
1 1 1 3
= + =

be
C 1 2 2 Here i1 = 1A
2 i 2 = 2A
C=
µ0 × 1 µ0 × 2

Tu
3 =
Q = C ⋅ V AD 2πx 2π( 0 ⋅ 1 + x )
2
= × 1⋅ 5 2x = ( 0 ⋅ 1 + x )

ou
3 x = 0⋅1 m
= 1 µC
38. (b, d) We know that magnetic susceptibility
Applying KVL (Kirchhoff’s Voltage Law) from
B to C
1
V B − 0 ⋅ 5 × 10− 3 × 2 × 103 + = V C
(Y and
χ = µr −1
µr =
µ
µ0
on
2
1 = Relative permeability
V B − V C = 1 − = 0 ⋅ 5V
2 For paramagnetic substance,
pi

χ> 0
γRT γP
36. (b,c) v = = ∴ µr > 1
m

M ρ
∴ u > µ0
ha

(i) Velocity of sound is proportional to T For diamagnetic substance,


∴ v∝ T χ< 0
(ii) When ρ is constant, the velocity of sound ∴ µr < 1
C

is proportional to root of pressure P ∴ µ < µ0


∴ v∝ P For ferromagnetic substance,
dy

37. (c) χ>>1


µ > > M0
u

If 0< µ < µ0
St

B1 1A=i1 2A=i2
Then substance will not be paramagnetic.
Hence option (a) is incorrect and option (b)
and (d) are correct.
1
B2 0.1 m 39. (a, b, c, d) We know that, v ∝ (speed)
n
1
3 1 2
En ∝ (energy)
n2
According to question, the third wire should rn ∝ n 2
not feel magnetic force due to wire (1) and ∴ E n rn ∝ n 0
(2). It can be only possible in the condition
∴ E n rn ∝ E1 r1
26 WEST BENGAL (Engineering) Solved Paper 2017

E n rn 40. (d) Given, steel ball bounces on a steel plate


= constant (Qslope = 0)
E1 r1 held horizontally, so
1 V upward = eV downward
rn v n ∝ n 2 × ∝ n
n According to the relation,
rn v n h n = e2n h
∴ =n (Qslope = 1)
r1 v 1 2h 2he2 2he4
∴ t= +2 +2 + ...
rn ∝ n 2 g g g

)
rn
∴ = n2 2h

be
r1 = [1 + 2e + 2e2 + . . . ]
g
r 
log  n  = 2 log n (Qslope = 2) 2h 1 + e

Tu
 r1  =
g 1 − e
rn
∝ n4 ∴ t = 10 sec

ou
En
2( 0 ⋅ 4)  1 + e
rn E ∴ 10 =  
∴ × 1 = n4 10  1 − e
En r1
r E 
log  n 1  = 4 log( n )(Qslope= 4)
 E n r1  (Y Solving above relation

∴ e=
25 2 − 1 17

25 2 + 1 18
on
pi

Chemistry
m

41. (a)For ideal gas, log


k
, log
k 42. (b) Q Surface tension ( T ) = Work done per
M2 M1 unit area
ha

dW
∴ T=
dA
F ⋅ dx
C

= ( FL− 1 = MT − 2 )
dA
∴Dimension of surface tension = ML0T − 2
dy

M2
M1
43. (a) According to ( n + l ) rule;
(i) More be the sum of ( n + l ) value, more be the
u

log p energy.
St

W
Q pV = RT …(i) (ii) For same value of sum, more be the value of n,
M more be the energy.
Let, WRT = K
For
Taking log of both the sides of Eq. (i)
K (i) n + l → 4 + 1 = 5
log p + log V = log
M (ii) n + l → 4 + 0 = 4
K (iii) n + l → 3 + 2 = 5
or log V = − log p + log …(ii)
M
(iv) n + l → 3 + 1 = 4
[On compairing Eq. (ii) with y = mx + C ]
K K Hence, order of energy will be
log > log or M1 > M2
M2 M1 (iv) < (ii) < (iii) < (i)
WEST BENGAL (Engineering) Solved Paper 2017 27

44. (b) Electronic configuration for Cr(24) CN − + H 2O - HCN + OH −


= [ Ar]18 4s1 3d 5 QCN − ions come from the salt of strong base
∴19th electron enters in 4s-orbital and its set and weak acid.
of quantum numbers is ∴The solution becomes basic and pH > 7.
1 49. (a) Q In Me3N → N is sp 3-hybrid
n = 4, l = 0, m = 0, m s = +
2
45. (d) Q Number of milli equivalents of NaOH M

)
( n ′ ) = NV = 0.1 × 20 = 2

be
Me Me
∴Number of equivalent = 0. 002 Me

For Neutralisation In C 5H 5N → N is sp 2-hybrid

Tu
Number of equivalent of base (NaOH)
= Number of equivalent of acid
W 0.126
0. 002 = = N

ou
E E
∴ E=
0.126
= 63 and in MeCN → N is sp-hybrid
Also electronegativity of any element α%

(Y
0. 002
46. (c) Given, weight ratio : WCH 4 : WSO 2 = 1 : 2 s-character in hybrid orbitals.
Thus, order of electronegativity for nitrogen is
n = no. of moles
on
W w = mass MeCN > C 5H 5N < Me3N
Q n=
m 
 M = molar mass 50. (c) Shape of XeF5− →
pi


n1 W1 M2  n1 = n SO 2
∴ = × 
n 2 M1 W2  n 2 = n CH 4
m

F Xe F

n1 WSO 2 MCH 4 F F F
= ×
ha

n 2 MSO 2 WCH 4 The Xe show sp 3d 3-hybridisation in XeF5− ,


2 16 Thus, its geometry is
= ×
C

64 1 Pentagonal-bipyramidal.
n1 1
= , i.e. 1 : 2 8+ 5+1
But, number of electron pair = =7
dy

n2 2 2
Also n ∝ N Number of bond pair = 52, number of lone
pair = 2
u

∴Ratio of number of molecules is 1 : 2.


A lot of electrons are present at axil position
St

47. (a) 9 F18 → xE y + + 1 e0


and all bonds are in same plane. Hence, the
x = 8, y = 18, shape is planar.
∴ x E y
= 8O18
51. (a) According to molecular orbital theory,
QPosition has one unit of the charge and
outer configuration for
zero mass.
[C 2] = C12 → πp x = πp y
2 2

Thus, F18 changes to O18 therefore resulting


decay product is C 6H 5O18. Thus, is diamagnetic in nature.
48. (d)QOn dissolving NaCN in de-ionised water, [ B2] = B10 − > πp1x = πp1y
following reaction takes place. Thus, is paramagnetic in nature.
28 WEST BENGAL (Engineering) Solved Paper 2017

52. (b) Configuration for metal (M) in various 55. (a) Q Higher be the oxidation number of
complex. central atom in oxo-acids, more strongly it
3d 4s behave as Bronsted-acid.
Ni2+ : Oxidation number for Cl in HClO 3 → + 5
3d Oxidation number for Cl in HClO 2 → + 3
[NiCl4]2–= Oxidation number for Cl in HClO → + 1
4s
Oxidation number for Cl in HBrO → + 1

)
Hence, HClO 3 is the strongest Bronsted acid

be
in aqueous solution.
– – – –
Cl Cl Cl Cl
56. (d) As size of cation decreases, ionic character
Ni 2 + has 2 unpaired electrons

Tu
decreases, thus basicity also decreases
3d
Hence, correct order is
[Ni(CO)4]=
La 3 + > Ce3 + > Eu 3 + > Lu 3 +

ou
4s 4p
57. (d) Q If CO 23 − , SO 23 − and SO 24 − are present
alongwith BaCl 2, these can also show white

(Y
CO CO CO CO precipitate (as precipitate of all these are also
0 white).
Ni has zero unpaired electrons with
sp 3-hybridisation. 58. (a) IUPAC name of Ph ⋅ CH 2CH 2CO 2H is,
on
3d 3-phenylpropanoic acid
[Cu(NH3)4]2+=Cu2+= 3
CH2⋅ CH2⋅ COOH
pi

4s 4p 2 1
m

NH3 NH3 NH3


ha

59. (d) Isomerisation of 1-butyne to 2-butyne can


NH3
4d be achived by treatment with ethanolic KOH
Isomerisation
H3C CH2 C CH
C

KOH (ethanolic)
Cu 2 + has one unpaired electron with 1-butyne
dsp 2-hybridisation. CH3 C C CH3
dy

∴(b) is the correct answer, i.e. 2-butyne


Number of unpaired electron (2, 0, 1)
60. (c)QIn p-nitrobenzoic acid (Z)  NO 2 present
u

53. (a)Q F-atom has 2nd highest electron affinity as electron withdrawing group, thus is most
St

(Cl has highest) in the periodic table acidic


∴Among the given options O C O H O C O
s

F-has highest electron gain affinity


54. (a) PbCl 2 react with HCl as follows + H+
Cold
PbCl 2( s) + Cl −   
→ [ PbCl 3]− ( aq)
N N
Excess
PbCl 2( s) + 2Cl −   → [ PbCl 4 ]2 − ( aq) O O O O
of HCl
(p-introbenzoic acid)
Thus, addition of excess of Cl − ions change
the PbCl 2 as soluble complex of[PbCl 4 ]−2. (X)QBenzoic acid is stabilised via resonance,
Hence, becomes soluble. thus is more acidic than per oxybenzoic acid.
WEST BENGAL (Engineering) Solved Paper 2017 29

62. (c) (i) OH O−


s
O C O H O C O

Acid-base
+ H+ + NaOH reaction + H2O

Benzoic acid OMe OMe


(Y) peroxy benzoic acid does not show any (QAcetic anhydride is limiting reagent)
such resonance. Thus, we get one mole of acetanilide and
molar mass of

)
O C O O H
O

be
No resonance HN C  CH3

Tu
(Peroxy benzoic acid)
is 135.
Thus is least acidic.

ou
(ii) O
Hence, correct order is Z > X > Y. O– O–
61.(b) The reaction between aniline and acetic C

(Y
anhydride is as follows : CO2 O–
NH2 O O OMe
OMe
on
C C
+ Me
O Me 100%
H2O/H+
conversion
Initial moles 2 1
pi

OH
Final moles 1 0 O
COOH
m

NH C Me P=
ha

+ MeCOOH OMe
(Product)
C

0 0
QO − is more reactive than  OMe and
1 1 p-position is occupied by  OMe, the
dy

O O substitution occurs at ortho-position


w.r.t  O − group.
C C is limiting reagent
u

As Me Me 63. (a) ADP is adenosine diphosphate


O
St

O (2-phosphate groups) and ATP is adenosine


triphosphate (3 phosphate groups)
NH C Me
Thus, they differ in number of phosphate units.

i.e. C8H9ON is 135 g/mol 64. (c) When primary amine react with chloroform
M. nt of
and ethanolic potassium hydroxide, they
O produce −isocyamides as main product.
The reaction is also known as carbyl amine
NH C Me reaction.
CHCl 3
Ph − NH 2    → Ph ⋅ N → C
As 1 mole of is formed KOH (ethanolic)
(Nauseating smell)
30 WEST BENGAL (Engineering) Solved Paper 2017

65. (b) The given reaction occurs as follows : (b) → E = 1 . 98 × 10− 25 J


 1 
(i) For (c) E = hc × v Q = v 
(i) Ph. Mg Br/THF  λ 
(ii) H3O+ – +
CN C N, MgBr E = 6. 63 × 10− 34 × 3 × 108 × 30 × 10−2
Ph
(c) ⇒ E = 5. 96 × 10− 26
H2O/H+ For (d) d → E = 6. 62 × 10− 27 J
Hence, highest energy for photon is in (a).

)
68. (c) Q For Isotonic solution

be
C NH π1 = π 2 i.e. i1C1 RT = i 2C 2RT
Ph For option (c)

Tu
i1 = 2, i 2 = 3
(Unstable)
C1 = 0. 03
.

(ii) H2O/H+ C 2 = 0. 02

ou
O
C NH
(On further
hydrolysis) C Thus, i1 × C1 = i 2 × C 2
Ph
Hence, are isotonic, i.e.
Ph

C
Ph
(Y 0. 06 = 0. 06
In all other options, i1 × C1 ≠ i 2 × C 2
Thus, are not isotonic.
on
O 69. (d) Cr2O72 − → Cr 3 + Change in + 3
66. (c) As per given relation,
Oxi no. = + 6
pi

N1 V1 + N 2V 2 = N( V1 + V 2 )
Total change in number of electrons
N1 = 2 N 2 = 5
= 2 × 3 = 6 mole = 6 F
m

V1 = 500 V 2 = say-x
70. (d) For the given reactions,
ha

Thus,
2 × 500 + 5x = 3( x + 500) or 2x = 500 2H 2O - 2H 2 + O 2, K1 = 6. 4 × 10− 8
i.e. x = 250 1
or, H 2O - H 2 + O 2, K ′1 = K1
C

Hence, maximum volume of 2


1
3M HCl = 500 + 250 = 750 mL or H 2 + O 2- H 2O, K1′′ = 1 / K1 … (i)
2
dy

hc  1 
67. (a)QE = hv = = hc ⋅ v Q = v  2CO 2 - 2CO + O 2, K 2 = 1.6 × 10− 6
λ  λ 
1
CO + O 2, K ′ 2 = K 2
u

where E = energy of photon


or CO 2 - 2
… (ii)
St

c = velocity of photon (= light) Form (i) and (ii) [on adding]


λ = wavelength of photon K2 1.6 × 10− 6
h = plank’s constant. K= =
K1 6. 4 × 10− 8
∴ For (a)
102
E = 6 . 63 × 10− 34 × 3 × 108 / 300 × 10− 9 K= = 25 = 5
4
E = 1 . 98 × 10− 25 ⋅ J / 300 × 10− 9
1 . 98 × 10− 25 J 71. (d) Q For be packing,
∴ E= 3
300 × 10− 9 m 3a = 4r ∴ r = ⋅a
4
(a) → E = 6. 6 × 10− 19 J where a = edge length
For (b) E = hv = 6. 63 × 10−34 × 3 × 108 r = radius of lattice sphere
WEST BENGAL (Engineering) Solved Paper 2017 31

72. (b) When drop of line juice is added to (d) Fire can likely take place because
mixture of I− + IO −3 . The following reaction relations given given in (b) is highly
take place and violet colour appears due to exothermic.
formation of I2 ∴(b), (c) and (d) are correct.
I− + IO 3− + H + → I2 + H 2O
77. (b), (c) Among the given statements (b) and
73. (d) The reaction occurs as follows : (c) are correct these are the facts.

COO Me (i) 78. (a, b) Reduction of benzene diazoniums

)
HOOC Ag2O chloride occurs as follows:

be
+ –
N2⋅Cl NH NH2
COOMe
Ag+O−OC (a) SnCl2, 2HCl

Tu
(ii) (b) Na2SO3
Br2/CCl4 Phenylhydrazine

ou
COOMe Hence, (a) and (b) are the correct answers.
Br
79. (a) The reaction for 1 mole of
The above reaction is an example of hexadeuterisobenzene occurs as follows :
Borodine Hunsdiecker reaction.
74. (c) The reaction occurs as follows : (Y D
D
D
(i) Br2(1 mole), Fe
on
OH NaBH4
O D D
CH O
D Br
pi

OH HO D D
(S)
m

HO
D D H2O
ha

75. (b) The reaction for given reaction is D


H2O is added to remove
HCl + NaOH → NaCl + H 2O insoluble bromodueterobenzene
Number of moles xy xy 0 0 as from the solution
C

Added NV NV 80. (a,d) CH 3 CH 2 COOH


Number of moles 0 0 xy xy
dy

(a) SoCl 2
left → CH 3 CH 2  C Cl
(d) HCl
∴N (solution) =
Number of milliequivalent 
u

vol. of solution in mililetre O


St

(a)LiAlH 4
76. (b,c,d) During electroysis of molten NaCl,
→ CH 3 CH 2 CHO
when water is added, the following (d)SnCl 2
statements are true CH  OH
 2

(a) false (a)/(d) 
CH  OH
1 ↓ 2
(b)QNa + H 2O → NaOH + H2
2
O CH 2
and discharge potential of hydrogen is less CH 3 CH 2 CH ƒ 
than of sodium. So, H 2 will evolve. ‚
O CH 2
(c) Some amount of caustic soda (NaOH)
will be formed [As shown in (b)] Hence, (a) and (d) are correct answers.
32 WEST BENGAL (Engineering) Solved Paper 2017

Mathematics
1. (b) Total number of 5-digit numbers having 5. (b) All the orthogonal matrix are non-singular
all the digits distinct = 10P5 − 9P4 matrix.
10 ! 9 ! 10 × 9 ! 9 ! ∴Q is proper subset of P.
= − = −
5! 5! 5! 5! 6. (b) We know that,
9! 9!
= (10 − 1) = ( 9) det ( AB) = det ( A ) det ( B)
5! ( 9 − 4)!

)
∴ det ( AB) = 0

be
= 9 × 9P4 ⇒ det ( A ) ⋅ det ( B) = 0
2. (b) ( p + 1) ( p + 2)( p + 3) . . . ( p + q) is the x + 2 3x x 0
product of q consecutive natural number ⇒ =0
x+2 5 x+2

Tu
3
( p, q ∈ N ). The product of q consecutive
natural number is always divisible by q !. ⇒ {( x + 2)2 − 9x} { x( x + 2) − 0} = 0
⇒ ( x 2 + 4x + 4 − 9x ) x( x + 2) = 0

ou
3. (b) (1 + x + x 2 )9 = a 0 + a1 x + a 2x 2 +
⇒ x( x + 2) ( x 2 − 5x + 4) = 0
. . . + a18x18 ⇒ x( x + 2) ( x − 1) ( x − 4) = 0
Put x = − 1 , we get
(1 − 1 + 1)9 = a 0 − a1 + a 2 + . . . + a18
⇒ 1 = a 0 − a1 + a 2 + . . . + a18 ...(i)
(Y ⇒
7. (a) We have,
x = 0, − 2 , 1, 4

cos θ
on
1 0
Put x = 1, we get
| A | = − cos θ 1 cos θ
(1 + 1 + 1)9 = a 0 + a1 + a 2 + . . . + a18
–1 − cos θ 1
pi

⇒ 39 = a 0 + a1 + a 2 + . . . + a18 ...(ii)
On adding Eqs. (i) and (ii), we get = 1 [1 − ( − cos θ ) (cos θ)]
m

39 + 1 = 2( a 0 + a 2 + . . . + a18 ) − cos θ [ − cos θ + cos θ] +0 (cos2 θ + 1)


39 + 1 = 1 + cos2 θ
ha

⇒ a 0 + a 2 + a 4 + . . . + a18 =
2 Now, we know that
19683 + 1 − 1 ≤ cos θ ≤ 1
=
C

2 ⇒ 0 ≤ cos2 θ ≤ 1
=
19684 ⇒ 1 ≤ 1 + cos2 θ ≤ 2
dy

2 ⇒ 1 ≤ | A |≤ 2
= 9842, which is even number. ∴ | A | ∈[1, 2]
u

4. (d) We have, 8x − 3y − 5z = 0, 8. (a) Since, f : R → R is injective and


5x − 8y + 3z = 0, f ( x ) f ( y ) = f ( x + y ), ∀ x, y ∈ R.
St

3x + 5y − 8z = 0 ∴ f (x) = ax
8 −3 −5 Again, f ( x ), f ( y ), f (z ) are in GP.
∴ D= 5 −8 3 ⇒ ( f ( y ))2 = f ( x ) ⋅ f ( y )
3 5 −8 ⇒ a 2y = a x ⋅ a z
⇒ a 2y = a x + z
= 8( 64 − 15) + 3( − 40 − 9) − 5( 25 + 24)
⇒ 2y = x + z
= 8 × 49 + 3 × ( − 49) − 5 × 49
∴ x, y, z are in AP.
=0
∴The system has infinitely many non-zero 9. (d) For every real number x, x 2 ≥ 0
solutions. ∴ ( x, x ) ∈ P
WEST BENGAL (Engineering) Solved Paper 2017 33

Hence, P is reflexive. Sunday, Monday, Tuesday, Wednesday,


Now, let ( x, y ) ∈ P Thursday, Friday and Saturday.
⇒ xy ≥ 0 Out of these 7 cases, we have Sunday in one
⇒ yx ≥ 0 case.
∴ Total number of outcomes = 7
⇒ (y, x) ∈ P
Number of favourable outcomes = 1
Hence, P is symmetric.
1
Again, ( − 1, 0) ∈ P and ( 0, 2) ∈ P. Hence, required probability =
7

)
But ( − 1, 2) ∉ P

be
13. (d) Let, f ( x ) = sin x (sin x + cos x )
as ( − 1) ( 2) = − 2 < 0
∴ P is not transitive. = sin 2 x + sin x cos x
1 − cos 2x 2 sin x cos x

Tu
10. (b) We have, = +
2 2
xρy ⇒ x − y is zero or irrational. 1 1 1
= − cos 2x + sin 2x

ou
Now, x − x = 0
2 2 2
⇒ ( x, x ) ∈ρ 1 1
∴ ρ is reflexive. = + (sin 2x − cos 2x )

(Y
2 2
Again, if x − y is either zero or irrational,
Now, − (1)2 + (1)2 ≤ sin 2x − cos 2x
then y − x will also be either zero or
irrational. ≤ (1)2 + (1)2
on
⇒ ( x, y ) ∈ρ
[Q a 2 + b2 ≤ a sin x + b cos x ≤ a 2 + b2 ]
⇒ ( y , x ) ∈ρ
pi

∴ ρ is symmetric. ⇒ − 2 ≤ sin 2x − cos 2x ≤ 2


Again, ( 2 , 3 ) ∈ρ and ( 3, 4) ∈ρ 2 sin 2x − cos 2x
m

2
⇒ − ≤ ≤
But ( 2 , 4) ∉ρ 2 2 2
ha

∴ρ is not transitive. 1 2 1 sin 2x − cos 2x 1 2


⇒ − ≤ + ≤ +
11. (d) We have, 2 2 2 2 2 2
x1 + x 2 + . . . + x q + . . . + x n
C

1− 2 1+ 2
x= ⇒ ≤ f (x) ≤
n 2 2
⇒ Σx = nx
dy

...(i) 1− 2 1+ 2
∴ ≤k≤
2 2
If x q is replaced by x ′q , then new total will be
u

14. (a) We have,


Σx ′ = Σx − x q − x q′  x − 1 − 1  x + 1 π
St

∴ New mean will be tan − 1   + tan   =


 x − 2  x + 2 4
Σx ′
x′ =
 x−1 x+1 
 x−2+ x+2  π
n
Σx − x q + x ′q ⇒ −1
x′ = tan  =
n 1 − x − 1 ⋅ x + 1  4
nx − x q + x ′q  x – 2 x + 2 
x′ = [from Eq. (i)]
n ( x − 1) ( x + 2) + ( x − 2) ( x + 1) π
⇒ = tan
12. (b) In a non-leap year, total number of days is ( x − 2) ( x + 2) − ( x − 1) ( x + 1) 4
365. Out of them, there are 52 weeks and 1 x2 + x − 2 + x2 − x − 2
day extra. Thus, a non-leap year always has ⇒ =1
x2 − 4 − x2 + 1
52 Sunday. The remaining 1 day can be
34 WEST BENGAL (Engineering) Solved Paper 2017

2x 2 − 4 17. (d)
⇒ =1 Y
−3
P(3, 6) y=x
⇒ 2x 2 − 4 = − 3
⇒ 2x 2 = 1
1 Q(6, 3)
⇒ x2 = X′ X
2
1
⇒ x=±

)
2

be
15. (b) Given equations are
Q′(–3,–6) y=–x
2x 2 + 3xy + 4y 2 + x + 18y + 25 = 0 ...(i) Y′

Tu
2x + 3xy + 4y + 1 = 0
2 2
...(ii) The coordinates of Q and Q′ will be ( 6, 3) and
Let the origin be transferred to ( p, q) axes ( − 3, − 6), respectively.
6− 3

ou
being parallel to the previous axes; then the Now, Slope of PQ = = −1
equation (i) becomes. 3− 6
2( x ′ + p )2 + 3( x ′ + p ) ( y ′ + q) − 6− 3
and Slope of QQ′ = =1
+ 4( y ′ + q)2 + ( x ′ + p ) + 18( y ′ + q) + 25 = 0
⇒ 2x ′ 2 + 2p 2 + 4x ′ p + 3x ′ y ′ + 3x ′ q + 3py ′
+ 3pq + 4y ′ 2 + 4q2 + 8y ′ q + x ′ + p
(Y − 3− 6
∴ Slope of PQ × Slope of QQ′ = − 1 × 1 = − 1
∴∆PQQ′ is right angled triangle at Q.
on
+ 18y ′ + 18q + 25 = 0
∴Circumcentre will be the mid-point of
⇒ 2x ′ + 4y ′ + 3x ′ y ′ + ( 4p + 3q + 1)x ′
2 2
 − 3 + 3 − 6 + 6
1 + ( 3p + 8q + 18)y ′ + 2p 2 + 3pq hypotenuse PQ′ =  , 
pi

 2 2 
+ 4q2 + p + 25 = 0
= ( 0, 0)
m

From Eq. (ii) coefficient of x′ and y ′ must be


zero. 18. (b) Let ( h, k ) be any point on the line
ha

∴ 4p + 3q + 1 = 0 ...(iii) 7 x − 9y + 10 = 0, then 7 h − 9k + 10 = 0
3p + 8q + 18 = 0 ...(iv) ⇒ 7 h = 9k − 10
9k − 10
C

By solving Eqs. (iii) and (iv), we get ⇒ h= ...(i)


p = 2, q = − 3 7
Now, perpendicular distance from point
dy

16. (a) Let the coordinates of C be (α, β). ( h, k ) to the line 3x + 4y = 5 is d1


∴Coordinates of centroid
3h + 4k − 5
 2 − 2 + α − 3 + 1 + β d1 =
u

= ,  32 + 42
 3 3 
St

3h + 4k − 5
 α β − 2 ⇒ d1 = ...(ii)
= ,  5
3 3 
and perpendicular distance from ( h, k ) to
Since, centroid lie on 2x + 3y = 1. the line 12x + 5y = 7 is d 2
2α  β − 2 12h + 5k − 7
∴ + 3  =1 ∴ d2 =
3  3 
122 + 52
2α 3β − 6 12h + 5k − 7
⇒ + =1 ⇒ d2 = ...(iii)
3 3 13
⇒ 2α + 3β − 6 = 3 ⇒ 2α + 3β = 9 3h + 4k − 5 12h + 5k − 7
Now, d1 − d 2 = −
∴Locus of point C will be 2x + 3y = 9. 5 13
WEST BENGAL (Engineering) Solved Paper 2017 35

⇒ d1 − d 2 From figure,
13 ( 3h + 4k − 5) − 5 (12h + 5k − 7 ) OP = ( h + 1)2 + ( k − 1)2
=
65
In ∆OAP,
39h + 52k − 65 − 60h − 25k + 35
= OP
65 sin 45° =
OA
− 21 h + 27 k − 30
= 1 ( h + 1)2 + ( k − 1)2
65 ⇒ =
 9k − 10 2 2

)
− 21   + 27 k − 30

be
 7  On squaring both sides, we get
= [from Eq.(i)]
65 ( h + 1)2 + ( k − 1)2 = 2
− 27 k + 30 + 27 k − 30 ⇒ h 2 + k 2 + 2h − 2k = 0

Tu
= =0
65 ∴Locus of P will be
⇒ d1 − d 2 = 0 ⇒ d 1 = d 2 x 2 + y 2 + 2x − 2y = 0

ou
19. (d) Given, equation of circles are 21. (b) Given, equation of hyperbola is
x 2 + y 2 − 4x − 4y = 0 and 2x 2 + 2y 2 = 32 x2 y 2
− =1
or x 2 + y 2 − 4x − 4y = 0
and x 2 + y 2 = 16
∴Equation of common chord is
(Y a 2 b2
b
y= a x
on
( x 2 + y 2 − 4x − 4y ) − ( x 2 + y 2 − 16) = 0 P
⇒ − 4x − 4y + 16 = 0
pi

⇒ x+y =4 C S(ae, 0)
m

This common chord passes through (2, 2),


i.e. centre of first circle.
ha

Also, (0, 0) is at the circumference of the first


circle.
π From figure,
∴Common chord will subtent angle at
C

2 abe
(0, 0). SP =
b2 + a 2
dy

20. (c) Given, equation of circle is


abe
x 2 + y 2 + 2x − 2y − 2 = 0 = =b
u

ae
⇒ ( x + 1)2 + ( y − 1)2 = 4
and CS = ae
St

∴Centre ( − 1, 1) and radius = 2


Again, ∆SPC is right angled triangle at P.
Let ( h, k ) be the mid-point of chord.
∴ CP = CS 2 − SP 2
= a 2e2 − b2
 b2 
= a 2 1 + 2  − b2
O(–1, 1)  a 
= a 2 + b2 − b2 = a

45° 45° ∴ Area of rectangle = CP × SP


A P(h, k) B = ab
36 WEST BENGAL (Engineering) Solved Paper 2017

22. (b) Y Since, line joining foci of hyperbola is


diameter of circle.
B (0, b)
 0 + 0 2 − 2
∴Centre of circle =  ,  = ( 0, 0)
 2 2 
X′ X
S′(–ae, 0) S(ae, 0) 1
and radius = ( 0 − 0)2 + ( 2 − ( − 2 ))2
2
1
= ( 2 2 )2 = 2

)
2

be
Y′
b−0 b ∴Equation of circle will be
Slope of SB, m1 = =−
0 − ae ae ( x − 0)2 + ( y − 0)2 = ( 2 )2

Tu
b−0 b ⇒ x2 + y 2 = 2
and slope of S ′ B, m 2 = =
0 − ( − ae) ae 25. (d) Equation of the plane will be

ou
Since, ∠SBS ′ is a right angle. x−1 y − 2 z+3
∴ m1 m 2 = − 1 2 − 1 − 2 − 2 1 − ( − 3) = 0
−b b

(Y
⇒ × = −1 1 0 0
ae ae
x−1 y − 2 z + 3
⇒ b2 = a 2e2
⇒ 1 −4 4 =0
on
b2
⇒ = e2 1 0 0
a2
⇒ 1 − e2 = e2 ⇒ 1[ 4( y − 2) + 4(z + 3)] = 0
pi

⇒ 2e2 = 1 [Expanding along R 3]


⇒ 4y – 8 + 4z + 12 = 0
m

1
⇒ e2 =
2 ⇒ 4y + 4z + 4 = 0
ha

⇒ e= ±
1 ⇒ y +z +1= 0
2 1 −1 1
1 1
C

⇒ e= or e = − 26. (b) ∆ = 2 − 3 0
2 2 1 0 3
dy

23. (a) Given, equation = 1 ( − 9 − 0) − ( − 1) ⋅ ( 6 − 0) + 1 ( 0 − ( − 3))


x 2 + 2xy + y 2 − 5x + 5y − 5 = 0 = − 9 + 6 + 3= 0
⇒ ( x + y )2 = 5x − 5y + 5
u

Since, ∆ = 0
⇒ ( x + y )2 = 5( x − y + 1) ∴Lines are coplanar.
St

∴ Axis of the parabola is x + y = 0


27. (a) We have,
24. (c) Given, equation of hyperbola is f (x)
x2 − y 2 + 1 = 0 = f (x − y )
f (y )
⇒ y 2 − x2 = 1
⇒ f ( x ) = a kx
y 2 x2
On comparing it with 2 − 2 = 1, we get ∴ f ′ ( x ) = ka kx log a
b a
Again, f ′ ( 0) = P
a= b=1
⇒ ka 0 log a = P
a2 1 ⇒ k log a = P
Now, e = 1 + 2
= 1+ = 2
b 1 Also, f ′ ( 5) = q
∴Foci = ( 0, ± be) = ( 0, ± 2 ) ⇒ ka 5k log a = q
WEST BENGAL (Engineering) Solved Paper 2017 37

⇒ a 5k P = q 2f ( x ) − 3f ( 2x ) + f ( 4x )
30. (c) lim
q x→ 0 x2
⇒ a 5k =
P 2f ′ ( x ) − 3f ′ ( 2x ) ⋅ 2 + f ′ ( 4x ) ⋅ 4
= lim
Now, f ′ ( − 5) = ka − 5k log a x→ 0 2x
=
k log a f ′ ( x ) − 3f ′ ( 2x ) + 2f ′ ( 4x )
= lim
a 5k x→ 0 x
p p2 f ′′( x ) − 3f ′′( 2x ) ⋅ 2 + 2f ′′( 4x ) ⋅ 4
= = = lim

)
 q q x→ 0 1
 

be
 p = lim f ′′( x ) − 6f ′′( 2x ) + 8f ′′( 4x )
x→ 0

28. (c) We have, = f ′′( 0) − 6f ′′( 0) + 8f ′′( 0)

Tu
f ( x ) = log 5log 3 x = k − 6k + 8k [Q f ′′( 0) = k ]
log log 3 x = 3k
=

ou
log 5 −1
31. (a) y = em sin x

 log x 
log   dy −1 m
 log 3 ⇒ = em sin x ⋅
=

=
log 5
log log x − log log 3 (Y ⇒
dy
1 − x2
dx
dx
−1
= mem sin x
1 − x2
on
log 5
2 dy
1 1 1 ⇒ 1− x = my …(i)
∴ f ′ (x) = ⋅ ⋅ dx
pi

log 5 log x x d 2y 1 dy dy
1 ⇒ 1 − x2 2
+ ( − 2x ) =m
∴ f ′ ( e) = dx 2 1− x 2 dx dx
m

e log 5 ⋅ log e
d 2y dy dy
1 ⇒ (1 − x 2 ) −x = m 1 − x2
ha

= [Q log e = 1] dx 2 dx dx
e log 5 2
dy dy
1 ⇒ (1 − x 2 ) 2 − x = m ⋅ ( my )
=
C

dx dx
e log e 5
[From Eq. (i)]
29. (c) We have, F( x ) = ex, G( x ) = e− x dy2
dy
dy

⇒ (1 − x 2 ) 2 − x − m 2y = 0
∴ H( x ) = G( F( x )) dx dx
= G( ex ) ∴ k = m2
u

= e− e
x

32. (d) Given curve,


St


dH
= e− e ⋅ ( − ex )
x
y = x 2 + 2ax + b
dx At x = α,
= − exe− e
x

y = α 2 + 2aα + b
dH and at x = β,
= − e0 ⋅ e− e
0

dx x = 0 y = β 2 + 2aβ + b
= –1. e–1 ∴Slope of line joining (α, α 2 + 2aα + b) and
= − e− 1 (β, β 2 + 2aβ + b) is
−1 (α 2 + 2aα + b) − (β 2 + 2aβ + b)
= =
e α −β
38 WEST BENGAL (Engineering) Solved Paper 2017

=
α 2 + 2aα + b − β 2 – 2aβ – b 36. (c) Let I = ∫ cos (log x ) dx
α −β
Put log x = t
(α 2 – β 2 ) + 2a(α − β ) ⇒ x = et
=
α −β ∴ dx = et dt
= (α – β )(α + β ) + 2a(α – β )
∴ I= ∫e
t
cos t dt
α–β
t
e
= α + β + 2a = [cos t + sin t] + C

)
12 + 12
dy

be
Slope of given curve =  
dx e ax
Q ∫ e cos bx dx = a2 + b 2 [ a cos bx + b sin bx] + C
ax

= 2x + 2a  

Tu
Now, according to question, tangent is et
⇒ I= [cos t + sin t] + C
parallel to the chord. Therefore, 2
2x + 2a = α + β + 2a x

ou
= [cos(log x ) + sin(log x] + C
α +β 2
⇒ 2x = α + β ⇒ x =
2 x
∴f ( x ) = [cos(log x ) + sin(log x )]
33. (c) We have,
f ( x ) = x13 + x11 + x 9 + x7 + x 5 + x 3 + x + 19
⇒ f ′ ( x ) = 13x12 + 11x10 + 9x 8
(Y
37. (a) Let I =
2


x2 − 1
x 4 + 3x 2 + 1
dx
on
+ 7 x 6 + 5x 4 + 3x 2 + 1 1 − 1 / x2
∴f ′ ( x ) has no real root.
= ∫x 2
+ 3 + 1 / x2
dx
pi

∴f ( x ) = 0 has not more than one real root. 1 − 1 / x2


= ∫ dx
m

34. (b) Since, Lagrange’s mean value theorem is  2 1


x + 2 + 3
applicable on f ( x ).  x 
ha

xP
∴ lim = f ( 0) 1 − 1 / x2
x → 0 (sin x )q = ∫ 1
2
dx
x +  − 2 + 3
C

P
x
⇒ lim =0  x
x→ 0 (sin x )q
1 − 1 / x2
∫
dy

Above equation holds only when p > q. = 2


dx
1
35. (b) Let y = lim(sin x ) 2 tan x
x +  + 1
x→ 0  x
u

⇒ log y = lim log(sin x )2 tan x 1


x→ 0
Let x + =t
St

= 2 lim tan x log sin x x


x→ 0
log sin x  1
= 2 lim ⇒ 1 − 2  dx = dt
x→ 0 cot x  x 
1 dt
⋅ cos x ∴ I= ∫t 2
+1
= 2 lim sin x
x → 0 − cosec 2x
= tan − 1 t + C
= 2 lim( − sin x cos x )  1  1
x→ 0 = tan − 1  x +  + C Q t = x + x 
= 2 × 0= 0  x
∴ log y = 0 ⇒ y = e0 = 1
WEST BENGAL (Engineering) Solved Paper 2017 39

38. (b) For x > 10, we have  n n 1


40. (b) lim  2 + 2 +K+
| sin x | < 1 and 1 + x 8 > 108 n → ∞ n + 12
 n +2 2
2n 
1
⇒ ≤ 10− 8  n n n 
1 + x8 = lim  2 + 2 +K+ 2
n → ∞ n + 12
 n +22
n + n 2 
19 sin x | sin x |
19
∴ ∫10 1 + x8
dx ≤ ∫ 10 1 + x8
dx
= lim ∑ 2

n
r =1n + r
n→∞ 2
19
≤ ∫ 10− 8 = 9 × 10− 8 < 10− 7

)

1 1
10
= lim ∑

be
n→∞ n 2
39. (d) We have, r =1  r
1+  
n  n
I1 = ∫ [ x] dx

Tu
0 1 1
= ∫
0
1
[ x] dx + ∫1
2
[ x] dx
= ∫0 1 + x2
dx
3 n
∫ [ x] dx + . . . + ∫ [ x] dx
+ = [tan − 1 x]10

ou
2 n −1
1 3 3 = tan − 1 1 − tan − 1 0
= ∫ 0 dx + ∫ 1 dx + ∫ 2 dx π π
= −0 =
0 2 2

(Y
n
+K+ ∫ ( n − 1) dx 4 4
n −1

= 0 + [ x]12 + 2[ x]32 + K + ( n − 1)[ x]nn − 1 41. (d) We know that, 0 ≤ x ≤ 1


⇒ x2 ≤ 1
on
= ( 2 − 1) + 2( 3 − 2) + K + 2

( n − 1) ( n − ( n − 1)) ⇒ ex ≤ e
= 1 + 2 + 3 + K + ( n − 1)
1 1
∫ e ∫
pi

x2
⇒ dx ≤ e dx
0 0
( n − 1) ( n − 1 + 1)  n( n + 1)
= Q Σn =
1
∫ e
x2
 ⇒ dx ≤ e
m

2 2 0

n( n − 1) 1
∫ e
2

= ∴ x
∈[1, e]
ha

0
2
100
∫ ex − [ x]dx
n n
42. (c) Let I =
Now, I2 = ∫ { x} dx = ∫ x − [ x] dx 0
C

0 0
1
= 100 ∫ ex − [ x] dx
n n
=∫ x dx − ∫ [ x] dx 0
0 0
dy

[Qx − [ x] is a periodic function of period 1


n
 x2 
=  2  − I1 mT T

 0 and ∫ f ( x ) dx = m ∫ f ( x ) dx , where T is
0 0
u

n 2 n( n − 1) period of f ( x )]
= −
St

1
2 2 = 100 ∫ ex dx [Qx − [ x] = x for 0 < x < 1]
0
n2 − n2 + n
= = 100 [ ex]10
2
n = 100 [ e1 − e0]
=
2 = 100 ( e − 1)
n( n − 1) 43. (a) We have,
I1 2
∴ = dy
I2 n ( x + y )2 = a2
dx
2
Let x+y =v
= ( n − 1)
40 WEST BENGAL (Engineering) Solved Paper 2017

⇒ 1+
dy dv
= Put x = 0,
dx dx ∴ 1= − A
dy dv ⇒ A = −1
⇒ = −1
dx dx Put x = 1,
 dv  ∴ 2 = 2B
∴ v 2 − 1 = a 2
 dx 
⇒ B=1
dv
⇒ v2 = v 2 + a2 Put x = − 1,

)
dx
∴ 2 = 2C

be
dv v 2 + a 2
⇒ = ⇒ C =1
dx v2
2
x +1
2
−1 1 1

Tu
v ∴ = + +
⇒ dv = dx
v 2 + a2 x( x − 1) ( x + 1) x x−1 x+1
−1 1 1 
On integrating both sides, we get ∫  + +  dx

ou
 x x−1 x + 1
v2 ∴ IF = e
∫ v 2 + a 2 dv = ∫ dx = e[ − log x + log( x − 1 ) + log( x + 1)]

(Y
 x2 − 1 
 a2  log 
x2 − 1
∫ 1 − v 2 + a 2  dv = x + C ′

⇒ =e  x 
=
x
on
a2 v =x−
1
⇒ v− tan − 1 = x + C ′
a a x
−1 x + y 45. (b) Let a n be the general term of a GP whose
⇒ x + y − a tan = x + C′
pi

a first term is a and common ratio is r.


x+y
m

⇒ y = a tan − 1 + C′ Now according to the question,


a an = an + 1 + an + 2
y – C′ x+y
ha

= = tan –1 ⇒ ar n − 1 = ar n + ar n + 1
a a ⇒ rn − 1 = rn + rn + 1
x + y  y + C rn rn + 1
C

⇒   = tan  , ⇒ 1 = n −1 + n −1
 a   a  r r
where − C ′ = C. ⇒ 1= r + r 2
dy

44. (b) We have, ⇒ r2 + r − 1 = 0


dy − 1 ± (1)2 − 4(1) ( − 1)
u

x 2( x 2 − 1) + x( x 2 + 1) y = x 2 − 1 ⇒ r=
dx 2(1)
St

dy x2 + 1 1 −1± 1+ 4 −1± 5
⇒ + y= 2 = =
dx x( x − 1)
2
x 2 2
x2 + 1 Since, GP consists only positive terms
∫ dx
x( x 2 − 1 )
∴ IF = e 5−1
x2 + 1
∴ r=

x( x − 1 ) ( x + 1 )
dx 2
=e
46. (a) We have,
x2 + 1 A B C
Let = + + log 5 x ⋅ log x 3x ⋅ log 3x y = log x x 3
x( x − 1) ( x + 1) x x − 1 x + 1
log x log 3x log y
⇒ × × = 3 log x x
⇒ x 2 + 1 = A ( x − 1) ( x + 1) + Bx( x + 1) log 5 log x log 3x
+ Cx( x − 1)
WEST BENGAL (Engineering) Solved Paper 2017 41

log a x2 + y 2 − 1
[Q log b a = and log a m = m log a] ⇒ =0
log b x 2 + ( y − 1)2


log y
=3 [Q log a a = 1] ⇒ x2 + y 2 = 1
log 5 ∴( x, y ) lies on a circle.
⇒ log y = 3 log 5 49. (a) Given equation,
⇒ log y = log 53 2px 2 + ( 2p + q) x + q = 0
⇒ y = 53 = 125 ∴D = ( 2p + q)2 − 4( 2p ) ( q)

)
= 4p 2 + q2 + 4pq − 8pq

be
(1 + i ) n
(1 + i )n
47. (c) =
(1 − i )n − 2 (1 − i )n (1 − i )− 2 = 4p 2 + q2 − 4pq
n = ( 2p − q)2

Tu
 1 + i
=  (1 − i )
2
= a perfect square
 1 − i
∴Given equation has rational roots

ou
n
 1 + i 1 + i 50. (b) Out of 7 consonants, the number of ways
= ×  (1 + i − 2i )
2

 1 − i 1 + i of selecting 3 consonants = 7C 3

(Y
n
 1 + 2i + i 2  Similarly, number of ways of selecting
=  (1 + i − 2i )
2
2 vowels out of 4 vowels = 4C 2
 1 − i2 
n
∴Total number of words formed
 1 + 2i − 1
on
= = 7C 3 × 4C 2 × 5P5
 (1 − 1 − 2i )
 1 − ( − 1)  7 × 6× 5 4× 3
= × × 5!
[Q i 2 = − 1] 3× 2×1 2×1
pi

n
 2i  = 7 × 5 × 2 × 3 × 120 = 25200
=   ( − 2i )
m

 2 1 1 1
= i ( − 2i ) = − 2i
n n +1
51. (b) We have, A = 0 1 1
ha

 
z + i ( x + iy ) + i 0 0 1
48. (d) =
z − i ( x + iy ) − i ∴ A2 = A ⋅ A
C

x + (1 + y ) i 1 1 1 1 1 1
=
x + ( y − 1) i = 0 1 1 0 1 1
  
dy

x + ( y + 1) i x − ( y − 1) i 0 0 1 0 0 1


= ×
x + ( y − 1) i x − ( y − 1) i
1 + 0 + 0 1 + 1 + 0 1 + 1 + 1
u

=
x 2 − x( y − 1) i + x( y + 1) i − ( y + 1) ( y − 1) i 2 = 0 + 0 + 0 0 + 1 + 0 0 + 1 + 1
 
St

x 2 − ( y − 1)2 i 2
0 + 0 + 0 0 + 0 + 0 0 + 0 + 1
x + i[ − xy + x + xy + x] + ( y 2 − 1)
2
=  2( 2 + 1)
x 2 + ( y − 1)2 1 2 3 1 2
2 
[Q i 2 = − 1 ] = 0 1 2 = 0 1 2 
   
x +y2 2
−1 2x 0 0 1 0 0 1 
= + i
x2 + (y − 1)2 x 2 + ( y − 1)2  
z+ i Again, A 3 = A 2 ⋅ A
Now, is purely imaginary.
z− i 1 2 3 1 1 1
 z + i = 0 1 2 0 1 1
∴   
Re  =0 0 0 1 0 0 1
 z − i
42 WEST BENGAL (Engineering) Solved Paper 2017

1 + 0 + 0 1 + 2 + 0 1 + 2 + 3 53. (c) We have,


= 0 + 0 + 0 0 + 1 + 0 0 + 1 + 2 R = {(1, 1), (2, 2), (3, 3), (1, 2), (2, 1)},
 
0 + 0 + 0 0 + 0 + 0 0 + 0 + 1 S = {(1, 1), (2, 2), (3, 3), (1, 3), (3, 1)}.
 3( 3 + 1) ∴R ∪ S = {(1, 1), (2, 2), (3, 3), (1, 2),
1 3 6 1 3 2  (2, 1), (1, 3), (3, 1)}.
= 0 1 3 = 0 1 3  Since, (2, 1) ∈ R ∪ S, (1, 3) ∈ R ∪ S
   
0 0 1 0 0 1  but (2, 3) ∉ R ∪ S
 

)
∴R ∪ S is reflexive and symmetric but not

be
 n( n + 1) transitive.
1 n 2 
54. (a) Given circle
∴ A = 0 1
n
n 

Tu
  x 2 + y 2 − 4x − 6y + 9 = 0
0 0 1   1 1 
  ∴Centre =  − × ( − 4), − × ( − 6)
 2 2 

ou
52. (d) We have,
= ( 2 , 3)
a a+1 a−1
Radius = ( − 2)2 + ( − 3)2 − 9
− b b+1 b−1
c c−1 c+1
a+1 b+1 c−1
(Y = 4+ 9− 9
=2
on
+ a−1 b−1 c+1 =0
n + 2 n +1
( − 1) a ( − 1) b ( − 1) c
n
pi

a a+1 a−1
A (1, 1)
⇒ − b b+1 b−1
m

c c−1 c+1 √5
ha

n + 2
a + 1 a – 1 ( − 1) a C
2
B(2, 3)
+ b + 1 b − 1 ( − 1)n + 1 b = 0
C

c − 1 c + 1 ( − 1)n c
∴ AB = ( 2 − 1)2 + ( 3 − 1)2 = 1 + 4 = 5
a a+1 a−1
∴ AC = AB2 + BC 2 = ( 5 )2 + ( 2)2
dy

⇒ − b b+1 b−1
= 5+ 4= 9 = 3
c c−1 c+1
u

+ 2 ∴Radius of required circle = 3


( − 1)n a a+1 a−1
St

+ ( − 1)n +1
b b+1 b−1 = 0 55. (d) (h, k)
M
( − 1) c
n
c−1 c+1 Y

a(1 + ( − 1)n + 2 ) a + 1 a − 1
⇒ b( − 1 + ( − 1)n + 1 ) b + 1 b − 1 = 0
c(1 + ( − 1)n ) c−1 c+1 A(–1, 0) θ 2θ
X
(0, 0) B(2, 0)
∴n is any odd integer. Y′
WEST BENGAL (Engineering) Solved Paper 2017 43

Let ∠MAB = θ, then ∠MBA = 2θ xy − 1


∴ f ( x ) = lim
k k y→0 y
tan θ = and tan 2θ = ,
1+ h 2− h = log x
2 tan θ k ∴ f ( xy ) = log( xy )
then tan 2θ = =
1 − tan θ 2 − h
2
= log x + log y
2( k / 1 + h ) k = f (x) + f (y )
⇒ =
1 − (k / 1 + h) 2
2− h 100π
58. (b) I = ∫ 1 − cos 2x dx

)
2 / (1 + h ) 1 0

be
⇒ =
(1 + h )2 − k 2 2 − h 100π

(1 + h )2
= ∫
0
2 sin 2 x dx

Tu
2 (1 + h )2 1 100π
⇒ × = = 2∫ sin x | dx
1 + h (1 + h )2 − k 2 2 − h 0
π
2(1 + h ) = 2 × 100 ∫

ou
1 sin x | dx|
⇒ = 0
(1 + h ) − k
2 2
2− h [| sin x | has period of π]
⇒ 1 + h 2 + 2h − k 2 = 2(1 + h ) ( 2 − h ) π

⇒ 1 + h 2 + 2h − k 2
= 2( 2 − h + 2h − h 2 )
⇒ 1 + h 2 + 2h − k 2 = 2( 2 + h − h 2 )
(Y = 100 2

= 100 2 [ − cos x]π0


0
sin x dx
on
= 100 2[(– cos π ) − ( − cos 0)]
⇒ 1 + h 2 + 2h − k 2 = 4 + 2h − 2h 2 = 100 2 [ − ( − 1) − ( − 1)]
⇒ 1 + 3h 2 − k 2 = 4
pi

= 100 2 × 2
⇒ 3h 2 − k 2 = 3
= 200 2
m

which represents hyperbola (d).


56. (c) We have, 59. (a) We have,
ha

x 0 x x = − 2y 2
f (x) = ∫
−1
t | dt = ∫ −1
| t | dt + ∫ | t | dt
0 1
0 x ⇒ y2 = − x …(i)
∫ ∫
C

=− t dt + t dt 2
−1 0
0 x
and x = 1 − 3y 2
 t   t2
2
1
dy

= −  +   ⇒ y 2 = − ( x − 1) …(ii)
 2− 1  20 3
1 1 From Eqs. (i) and (ii), we get
u

= − ( 02 − ( − 1)2 ) + ( x 2 − 02 ) 1 1
2 2 − x = − ( x − 1)
St

1 1 2 1 2 3
= − ( − 1) + ( x ) = (1 + x 2 )
2 2 2 ⇒ 3x = 2( x − 1)
⇒ 3x = 2x − 2
57. (b) We have, f ( x ) = lim n( x1 / n − 1)
n→∞ ⇒ x=−2
x1 / n − 1 1
= lim ∴ y 2 = − ( − 1) = 1
n→∞ 1 / n
2
Let
1
=y ⇒ y = ±1
n
44 WEST BENGAL (Engineering) Solved Paper 2017

∴Point of intersection of two curves is x× 2 y × 5/ 3


+ =1
( − 2, ± 1) 9 5
2 1
(–2,1) ⇒ x+ y =1
9 3
⇒ 2x + 3y = 9
9
(0,0) (1,0) ∴ OA = and OB = 3
2

)
∴Area of quadrilateral ABCD

be
(–2,–1)
= 4 × Area of ∆OAB
1
= 4 × × OA × OB
1
∴Required Area = 2 ∫ sin x dx

Tu
0 2
1
 y 3 1 9
= 2 y −  = 4× × × 3
2 2
 3 0

ou
= 27 sq units.
 1
= 2 1 −  61. (c) We have,
 3
4
= sq units
3 (Y f ( x ) = sin x − cos x − Kx + 5
⇒ f ′ ( x ) = cos x + sin x − K
For decreasing, f ′ ( x ) < 0
on
60. (a) We have,
⇒ cos x + sin x − K < 0
x2 y 2
+ =1 ⇒ K > cos x + sin x
9 5
pi

 1 1 
⇒ a = 3 and b = 5 ⇒ K > 2 cos x + sin x
 2 2 
m

b2  π 
∴ e= 1− ⇒ K > 2 sin  + x
a2 4 
ha

5 2
= 1− = ∴ K> 2
9 3
C

∴ Foci = ( ± ae, 0) 62. (b) Let x = α$i + β$j + γ k$


 2  Then, x × $i = − βk$ + γ $j
=  ± 3 × , 0 = ( ± 2, 0)
dy

 3 
x × $j = k$ − γ $i
 5
∴ Ends of latusrectum =  ± 2, ± 
u

 3 x × k$ = − α$j + β$i
St

B Now, ( x × $i )2 = ( x × $i ) ⋅ ( x × $i )
5
P 2, 3 = ( − βk$ + γ $j ) ⋅ ( − βk$ + γ $j )
= β2 + γ 2
Similarly, ( x × $j )2 = α 2 + γ 2
C F2(–2, 0) O F1(2, 0) A
and ( x × k$ )2 = α 2 + β 2

D ∴( x × $i )2 + ( x × $j )2 + ( x × k$ )2
Equation of tangent at the end of = β2 + γ 2 + α2 + γ 2 + α2 + β2
latusrectum P is = 2(α 2 + β 2 + γ 2 ) = 2| x |2
WEST BENGAL (Engineering) Solved Paper 2017 45

63. (c) Let a$ and b$ are two unit vectors. ⇒


3
≤y≤
1
Then, | a$ + b$ | = 1 7 2
+ + −
⇒ | a$ + b$ |2 = 1
3 1
⇒ ( a$ + b$ ) ⋅ ( a$ + b$ ) = 1 7 2
⇒| a$ |2 + | b$ |2 + 2a$ ⋅ b$ = 1 1
∴Maximum value of y =
⇒ 1 + 1 + 2a$ ⋅ b$ = 1 2

)
⇒ 2a$ ⋅ b$ = − 1 66. (c, d) We have,

be
ax 2 + bx + 1 = 0
Again, | a$ − b$ |2 = ( a$ − b$ ) ⋅ ( a$ − b$ )
For real roots, D ≥ 0

Tu
= | a$ |2 + | b$ |2 − 2a$ ⋅ b$ ∴ b2 − 4a ≥ 0
= 1 + 1 − ( − 1) ⇒ b2 ≥ 4a
= 1 + 1 + 1= 3 ∴ ( a, b) = (1, 2), (1, 3), ( 2 , 3)

ou
∴ | a$ − b$ | = 3 ∴Number of ordered pairs ( a, b) = 3
and a is always less than b.
64. (a) We have, x 2 + x + 1 = 0


− 1 ± 3i
2
x= (Y
67. (a, c) Equation of tangent to y 2 = 4ax at
( at 2, 2at ) will be
x − yt = − at 2
on
…(i)
− 1 + 3i − 1 − 3i
⇒ α= and β = Also, equation of normal to x 2 − y 2 = a 2 at
2 2
i 2π − 2πi ( a sec θ, a tan θ ) will be
pi

or α=e 3
and β = e 3 x
+
y
=2
2nπi − 2nπi a sec θ a tan θ
m

∴ αn + βn = e 3
+e 3
⇒ x + y cosec θ = 2a sec θ …(ii)
 2nπi − 2nπi

ha

Since, Eqs. (i) and (ii) are identical.


e 3
+e 3

= 2  ∴ t = − cosec θ or t = 2 tan θ
 2 
  68. (c) We have,
C

 2nπ  x 2 − 6x + 4y + 1 = 0
= 2 cos 
 3  ⇒ ( x − 3)2 − 9 + 4y + 1 = 0
dy

x 2 + 2x + 4 ⇒ ( x − 3)2 + 4y − 8 = 0
65. (c) Let y =
2x 2 + 4x + 9 ⇒ ( x − 3)2 = − 4( y − 2)
u

⇒ 2x 2y + 4xy + 9y = x 2 + 2x + 4 It represents parabola whose vertex is (3, 2)


St

⇒ ( 2y − 1)x 2 + ( 4y − 2) x + 9y − 4 = 0 ∴ Focus = ( 3, − 1 + 2) = ( 3, 1)
− ( 4y − 2) ± ( 4y − 2)
2
69. (b) Let function f ( x ) = x 2( x − 1)
− 4( 2y − 1) ( 9y − 4)
⇒x = ⇒ f ′ ( x ) = 3x 2 − 2x
2( 2y − 1)
and f ′′( x ) = 6x − 2
Since, x is real number.
Now, f ( 0) = f (1) = f ′ ( 0) = 0
∴ ( 4y − 2)2 − 4( 2y − 1) ( 9y − 4) ≥ 0
Then, according to question,
⇒ 4( 2y − 1)2 − 4( 2y − 1) ( 9y − 4) ≥ 0 1
⇒ 4( 2y − 1) ( 2y − 1 − 9y + 4) ≥ 0 at x = , f ′′( x ) = 0
3
⇒ 4( 2y − 1) ( 3 − 7 y ) ≥ 0 (i.e. c = 1 / 3 for some C∈R)
⇒ ( 2y − 1) (7 y − 3) ≤ 0
46 WEST BENGAL (Engineering) Solved Paper 2017

70. (b, c) We have, 1 2


73. (b, c) We know, S = ut + at …(i)
f (x) = xn 2
⇒ f ′ ( x ) = nx n − 1 Condition for first,
Now, f ′ (α + β ) = f ′ (α ) + f ′ (β ) As, velocity is constant u = u, v = u
⇒ n(α + β )n − 1 = nα n − 1 + nβ n − 1 (constant)
⇒ (α + β )n − 1 = α n − 1 + β n − 1 ∴ a = 0, S = ut … (ii) [from Eq. (i)]
From options we see that n = 2, satisfy the Condition for second u = 0, a = f (constant)

)
above equation. 1
∴ S = at 2

be
∴ n=2 2
1 2
dx
a S = f t … (iii) [from Eq. (i)]
71. (c) I = ∫ …(i) 2

Tu
1 + f (x)
0
On differentiating Eqs. (ii) and (iii), we get
a 1
=∫ dx dS
0 1 + f (a − x) =u … (iv)

ou
dt
Q b f ( x ) dx = b
f ( a + b − x ) dx
 ∫a ∫ dS 1
a  and = f ( 2t )
dt 2
= ∫
0
1+
dx
a

1
f (x)
[Q f ( x ) f ( a − x ) = 1]
(Y dS
dt
= ft

u = ft
on
[from Eq. (iv)]
a f (x)
I= ∫ dx …(ii) t=
u
… (v)
0 f (x) + 1
f
pi

On adding Eqs. (i) and (ii), we get Thus, they will be at the greatest distance at
a f (x) + 1 u
2I = ∫
m

dx the end of the from the start.


0 f (x) + 1 f
For greatest distance
ha

a
⇒ 2I = ∫
0
1 dx
1
S = ft 2
⇒ 2I = [ x]a0 2
C

⇒ 2I = a u
Put t= ,
a f
⇒ I= 1  u2
dy

2 S = f 2
2 f 
72. (b, d) We have,
u2
S=
u

xy = 1 − 2x
2f
1 − 2x
St

⇒y = Hence, option (b) and (c) are correct.


x
dy − 2x − (1 − 2x ) ⋅ 1 74. (a, c) We have,| z − i | = | z + 1| = 1
⇒ =
dx x2 Let, z = x + iy
− 2x − 1 + 2x − 1 ∴ | z − i| = 1
= = 2 <0
x2 x ⇒ | x + iy − i | = 1
Since, ax + by + c = 0 is tangent to the curve ⇒ | x + ( y − 1) i | = 1
xy = 1 − 2x. ⇒ x 2 + ( y − 1)2 = 1 … (i)
−a a | z + 1| = 1
∴ < 0⇒ > 0 Also,
b b ⇒ | x + iy + 1| = 1
⇒ Either a > 0, b > 0 or a < 0, b < 0. ⇒ |( x + 1) + iy | = 1
WEST BENGAL (Engineering) Solved Paper 2017 47

⇒ ( x + 1)2 + y 2 = 1 … (ii) 75. (c) We know that,


From Eqs. (i) and (ii), we get 1 + a 2 > 0, a ∈ R
x = y = 0 and x = − 1, y = 1 ⇒ ( a, a ) ∈ρ
∴ ρ is reflexive
∴ z = 0, − 1 + i
Again, let ( a, b) ∈ρ
(0, 2) ⇒ 1 + ab > 0
⇒ 1 + ba > 0
(–1, 1)
⇒ ( b, a ) ∈ρ

)
(0, 1)

be
∴ ρ is symmetric
(–2, 0) (–1, 0) (0, 0) Now, (1, − 0 ⋅ 1) ∈ρ
and ( − 0 ⋅ 1, − 9) ∈ ρ

Tu
but (1, − 9) ∉ρ
∴ ρ is not transitive.

ou
(Y
on
pi
m
ha
C
u dy
St
SOLVED PAPER 2016
WB JEE
Engineering Entrance Exam

)
be
Tu
Physics
Category I (Q.1 to Q.30) Only one answer is correct. Correct answer will fetch full mark 1. Incorrect

ou
1
answer or any combination of more than one answer will fetch - mark.
4
1. Equivalent capacitance between A and B in
the figure is
4mF 4mF
(Y
4. The potential difference V required for
accelerating an electron to
de-Broglie wavelength of 1 Å is
have the
on
A B (a) 100 V (b) 125 V
4mF (c) 150 V (d) 200 V
pi

5. The work function of Cesium is 2.27 eV. The


4mF 4mF cut-off voltage which stops the emission of
m

C electrons from a cesium cathode irradiated


with light of 600 nm wavelength is
(a) 20 mF (b) 8 mF
ha

(a) 0.5 V (b) -0. 2 V


(c) 12 mF (d) 16 mF
(c) -0.5 V (d) 0.2 V
2. Two wires of same radius having lengths l 1
6. The number of de-Broglie wavelengths
C

and l2 and resistivities r 1 and r2 are contained in the second Bohr orbit of
connected in series. The equivalent hydrogen atom is
dy

resistivity will be (a) 1 (b) 2


r1l2 + r 2 l1 r1l1 + r 2 l2
(a) (b) (c) 3 (d) 4
r1 + r 2 l1 + l2
u

r1l1 - r 2 l2 r1l2 + r 2 l1 7. The wavelength of second Balmer line in


(c) (d)
hydrogen spectrum is 600 nm. The
St

l1 - l2 l1 + l2
wavelength for its third line in Lyman series
3. A hollow metal sphere of radius R is charged is
with a charge Q. The electric potential and (a) 800 nm (b) 600 nm
intensity inside the sphere are respectively (c) 400 nm (d) 200 nm
Q Q
(a) and 8. A ray of light strikes a glass plate at an angle
4p e0 R 2 4p e0 R
Q
of 60°. If the reflected and refracted rays are
(b) and zero perpendicular to each other, the refractive
4p e0 R
index of glass is
(c) zero and zero
4p e0Q Q 3 3 1
(d) and (a) (b) (c) (d) 3
R 4p e0 R 2 2 2 2
2 WB JEE (Engg.) Solved Paper 2016

9. Light travels through a glass plate of 17. A particle vibrating simple harmonically has
thickness t and having refractive index m. If c an acceleration of 16 cms -2 when it is at a
be the velocity of light in vacuum, time taken distance of 4 cm from the mean position. Its
by the light to travel through this thickness time period is
of glass is (a) 1 s (b) 2.572 s
t tc mt (c) 3.142 s (d) 6.028 s
(a) (b) (c) (d) m tc
mc m c
18. Work done for a certain spring when
2
10. If x = at + bt , where x is in metre ( m ) and t is stretched through 1 mm is 10 joule. The

)
be
in hour ( h ), then unit of b will be amount of work that must be done on the
m2 spring to stretch it further by 1 mm is
(a) (b) m
h (a) 30 J (b) 40 J

Tu
m m (c) 10 J (d) 20 J
(c) (d)
h h2 19. If the rms velocity of hydrogen gas at a
certain temperature is c, then the rms

ou
11. The vectors A and B are such that
| A + B| = | A - B|. The angle between the two velocity of oxygen gas at the same
vectors will be temperature is
(a) 0° (b) 60° (c) 90° (d) 45°
12. At a particular height, the velocity of an
ascending body is u. The velocity at the same
(Y (a)

(c)
c
8
c
(b)

(d)
c
10
c
on
4 2
height while the body falls freely is
(a) 2 u (b) -u (c) u (d) -2 u 20. For air at room temperature, the atmospheric
pi

pressure is 1.0 ´ 105 Nm -2 and density of air


13. Two bodies of masses m 1 and m 2 are
separated by a distance R. The distance of the is 1.2 kgm -3 . For a tube of length 1.0 m,
m

centre of mass of the bodies from the mass closed at one end, the lowest frequency
m 1 is generated is 84 Hz. The value of g
ha

m2 R m1R (ratio of two specific heats) for air is


(a) (b)
m1 + m2 m1 + m2 (a) 2.1 (b) 1.5 (c) 1.8 (d) 1.4
C

m1m2 m + m2
(c) R (d) 1 R 21. A gas bubble of 2 cm diameter rises through a
m1 + m2 m1
liquid of density 1.75 g cm -3 with a fixed
dy

14. The velocity of sound in air at 20°C and 1 atm speed of 0.35 cms -1. Neglect the density of the
pressure is 344.2 m/s. At 40°C and 2 atm gas. The coefficient of viscosity of the liquid is
pressure, the velocity of sound in air is (a) 870 poise (b) 1120 poise
u

approximately (c) 982 poise (d) 1089 poise


St

(a) 350 m/s (b) 356 m/s 22. The temperature of the water of a pond
(c) 363 m/s (d) 370 m/s
is 0 °C while that of the surrounding
15. The perfect gas equation for 4 g of hydrogen atmosphere is -20 °C. If the density of ice
gas is is r, coefficient of thermal conductivity is k
(a) pV = RT (b) pV = 2 RT and latent heat of melting is L, then the
1
(c) pV = RT (d) pV = 4RT thickness Z of ice layer formed increases as a
2 function of time t is
16. If the temperature of the Sun gets doubled, (a) Z 2 =
60k
t (b) Z =
40k
t
the rate of energy received on the Earth will rL rL
increase by a factor of 40k 40k
(c) Z 2 = t (d) Z 2 = t
(a) 2 (b) 4 (c) 8 (d) 16 rL rL
WB JEE (Engg.) Solved Paper 2016 3
23. 1000 droplets of water having 2 mm 27. Two coils of self-inductances 6 mH and
diameter each coalesce to form a single drop. 8 mH are connected in series and are
Given the surface tension of water is adjusted for highest coefficient of coupling.
0.072 Nm -1. The energy loss in the process is Equivalent self-inductance L for the assembly
(a) 8146
. ´ 10-4 J (b) 4.4 ´ 10-4 J is approximately
(c) 2.108 ´ 10-5
J . ´ 10-1 J
(d) 47 (a) 50 mH
(b) 36 mH
24. A Zener diode having break-down voltage (c) 28 mH

)
5.6 V is connected in reverse bias with a (d) 18 mH

be
battery of emf 10 V and a resistance of 100 W 28. A 1 mF capacitor C is connected to a battery of
in series. The current flowing through the
10 V through a resistance 1 MW. The voltage
Zener diode is

Tu
across C after 1 s is approximately
(a) 88 mA (b) 0.88 mA
(c) 4.4 mA (d) 44 mA (a) 5.6 V (b) 7.8 V
(c) 6.3 V (d) 10 V

ou
25. In case of a bipolar transistor b = 45. The
29. Two equal resistances, 400 W each, are
potential drop across the collector resistance
connected in series with a 8 V battery. If the
of 1 kW is 5V. The base current is
approximately
(a) 222 m A (b) 55 m A (c) 111m A (d) 45 m A (Y resistance of first one increases by 0.5%, the
change required in the resistance of the
second one in order to keep the potential
on
26. An electron enters an electric field having difference across it unaltered is to
intensity E = 3 i$ + 6 $j + 2 k$ Vm -1 and (a) increase it by 1W
magnetic field having induction (b) increase it by 2 W
pi

(c) increase it by 4 W
B = 2 i$ + 3 $j T with a velocity v = 2 i$ + 3 $j ms -1.
(d) decrease it by 4 W
m

The magnitude of the force acting on the


electron is 30. Angle between an equipotential surface and
ha

(Given, e = - 1.6 ´ 10 -19 C) electric lines of force is


(a) 0° (b) 90°
(a) 2.02 ´ 10-18 N . ´ 10-16 N
(b) 516 (c) 180° (d) 270°
C

. ´ 10-17 N
(c) 372 (d) 4.41 ´ 10-18 N
dy

Category II (Q. 31 to Q. 35) Only one answer is correct. Correct answer will fetch full marks 2. If correct
1
answer are any combination of more than one answer will fetch - mark.
2
u

31. A current I = I0e - lt is flowing in a circuit (d) Both source and screen should be at finite
St

consisting of a parallel combination of distance


resistance R and capacitance C. The total 33. The temperature of a blackbody radiation
charge over the entire pulse period is enclosed in a container of volume V is
I
(a) 0
2I
(b) 0 increased from 100 °C to 1000 °C. The heat
l l required in the process is
(c) I0 l (d) e I 0 l . ´ 10-4 cal
(a) 479 . ´ 10-5 cal
(b) 921
-4
(c) 2.17 ´ 10 cal (d) 7.54 ´ 10-4 cal
32. For Fraunhoffer diffraction to occur
(a) Light source should be at infinity 34. A mass of 1 kg is suspended by means of a
(b) Both source and screen should be at infinity thread. The system is (i) lifted up with an
(c) Only the source should be at finite distance acceleration of 4.9 ms -2 . (ii) lowered with an
4 WB JEE (Engg.) Solved Paper 2016

acceleration of 4.9 ms -2 . The ratio of tension between the same two points, when the link
in the first and second case is AB is removed, is
(a) 3 : 1 (b) 1 : 2 B
(c) 1 : 3 (d) 2 : 1
A
35. The effective resistance between A and B in
7
the figure is W if each side of the cube has
12 7 5 7 5
(a) W (b) W (c) W (d) W
1W resistance. The effective resistance

)
12 12 5 7

be
Category III (Q. 36 to Q. 40) One or more answer (s) is (are) correct. Correct answers will fetch marks 2.
Any combination containing one or more incorrect answer (s) will fetch 0 mark. If all correct answers are
not marked and also no incorrect answer is marked then score = 2 ´ number of correct answers

Tu
marked/actual number of correct answers.
36. A charged particle of mass m 1 and charge q 1 38. A train moves from rest with acceleration a

ou
is revolving in a circle of radius r. Another and in time t 1 covers a distance x. It then
charged particle of charge q 2 and mass m 2 is decelerates to rest at constant retardation b
for distance y in time t2 . Then,

(Y
situated at the centre of the circle. If the
x b b t1
velocity and time period of the revolving (a) = (b) =
particle be v and T respectively, then y a a t2
x bt 1
on
q1q 2 r (c) x = y (d) =
(a) v = y at 2
4p e0 m1
1 q1q 2 39. A drop of water detaches itself from the exit
pi

(b) v =
m1 4 p e0 r of a tap when (s = surface tension of water,
r = density of water, R = radius of the tap
m

3
16 p e0 m12 r 3
(c) T = exit,r = radius of the drop)
q1q 2 1/ 3
ha

æ 2 Rs ö
16 p e0 m2 r 3
3 (a) r > ç ÷
(d) T = è 3 rg ø
q1q 2
2 s
C

(b) r =
37. The distance between a light source and 3 rg
photoelectric cell is d. If the distance is 2s
(c) > atmospheric pressure
dy

d r
decreased to , then 2/ 3
2 æ 2 Rs ö
(d) r > ç ÷
è 3 rg ø
u

(a) the emission of electron per second will be four


times
40. A rectangular coil carrying-current is placed
St

(b) maximum kinetic energy of photoelectrons will be


four times in a non-uniform magnetic field. On that
(c) stopping potential will remain same coil, the total
(d) the emission of electrons per second will be (a) force is non-zero (b) force is zero
doubled (c) torque is zero (d) torque is non-zero
Chemistry
Category I (Q. 1 to Q. 30). Only one answer is correct. Correct answer will fetch full mark 1. Incorrect
1
answer or any combination of more than one answer will fetch – mark.
4
1. Amongst the following compounds, the one 6. Which of the following reactions will not
that will not respond to Cannizzaro reaction result in the formation of carbon-carbon

)
upon treatment with alkali is bonds?

be
(a) Cl 3CCHO (b) Me 3CCHO (a) Cannizzaro reaction
(c) C 6H5CHO (d) HCHO (b) Wurtz reaction
(c) Reimer-Tiemann reaction

Tu
2. Which of the following compounds would
(d) Friedel-Crafts acylation
not react with Lucas reagent at room
temperature? 7. Point out the false statement.

ou
(a) H2C== CHCH2OH (a) Colloidal sols are homogeneous
(b) C 6H5CH2OH (b) Colloids carry + ve or - ve charges
(c) CH3CH2CH2OH (c) Colloids show Tyndall effect
(d) (CH3 )3 COH
3. Amongst the following compounds, the one
which would not respond to iodoform test is
(Y (d) The size range of colloidal particles is 10-1000 Å
8. The correct structure of the drug paracetamol is
OH OH
on
(a) CH3CH(OH)CH2CH3
(a) (b)
(b) ICH2COCH2CH3
pi

(c) CH3COOH
CONH2 NHCOCH3
m

(d) CH3CHO
Cl Cl
4. Which of the following will be dehydrated
ha

most readily in alkaline medium? (c) (d)


O O OH
C

CONH2 COCH3
(a) (b)
OH 9. Which of the following statements regarding
dy

Lanthanides is false?
OH O
OH
(a) All lanthanides are solid at room temperature
u

(c) (d) (b) Their usual oxidation state is +3


(c) They can be separated from one another by
St

5. The correct order of basicity of the following ion-exchange method


compounds is (d) Ionic radii of trivalent lanthanides steadily
increases with increase in atomic number
NH2 NH
1 2 10. Nitrogen dioxide is not produced on heating
NH2 NH2 (a) KNO 3 (b) Pb(NO 3 )2
(c) Cu(NO 3 )2 (d) AgNO 3
NH H2N NH 11. The boiling points of HF, HCl, HBr and HI
3 4
follow the order
(a) 1 < 2 < 3 < 4 (b) 1 < 2 < 4 < 3 (a) HF >HCl >HBr >HI (b) HF >HI >HBr >HCl
(c) 2 < 1 < 3 < 4 (d) 4 < 3 < 2 < 1 (c) HI >HBr >HCl >H (d) HCl >HF >HBr >HI
6 WB JEE (Engg.) Solved Paper 2016

12. In the solid state, PCl5 exists as 20. The number of s and p bonds between two
(a) [PCl 4 ]- and [PCl 6 ]+ ions carbon atoms in calcium carbide are
(b) covalent PCl 5 molecules only (a) one s, one p (b) one s, two p
(c) [PCl 4 ]+ and [PCl 6 ]- ions 1
(c) two s, one p (d) one s, 1 p
(d) covalent P2Cl10 molecules only 2

13. Which statement is not correct for ortho and 21. An element E loses one a and two b particles
para hydrogen? in three successive stages. The resulting
element will be

)
(a) They have different boiling points

be
(b) Ortho-form is more stable than para-form (a) an isobar of E (b) an isotone of E
(c) They differ in their nuclear spin (c) an isotope of E (d) E itself
(d) The ratio of ortho to para hydrogen changes with 22. An element X belongs to fourth period and

Tu
change in temperature
fifteenth group of the periodic table. Which
14. The acid in which O ¾ O bonding is present of the following statements is true?
is

ou
(a) It has completely filled s-orbital and a partially
(a) H2S2O 3 (b) H2S2O 6 filled d-orbital
(c) H2S2O 8 (d) H2S4O 6 (b) It has completely filled s-and p -orbitals and a

(Y
partially filled d-orbital
15. The metal which can be used to obtain (c) It has completely filled s- and p-orbitals and a half
metallic Cu from aqueous CuSO4 solution is filled d-orbital
(a) Na (b) Ag (d) It has a half filled p -orbital and completely filled
on
(c) Hg (d) Fe s-and d-orbitals
16. If radium and chlorine combine to form 23. Which of the following plots represent an
radium chloride, the compound would be exothermic reaction?
pi

(a) half as radioactive as radium


m

(b) twice as radioactive (a) In Kp (b) In Kp


(c) as radioactive as radium
ha

(d) not radioactive 1/T 1/T


17. Which of the following arrangements is
correct in respect of solubility in water? (c) In Kp (d) In Kp
C

(a) CaSO 4 >BaSO 4 >BeSO 4 >MgSO 4 >SrSO 4


(b) BeSO 4 >MgSO 4 >CaSO 4 >SrSO 4 >BaSO 4 1/T 1/T
dy

(c) BaSO 4 >SrSO 4 >CaSO 4 >MgSO 4 >BeSO 4 24. If p° and p are the vapour pressure of the pure
(d) BeSO 4 >CaSO 4 >MgSO 4 >SrSO 4 >BaSO 4 solvent and solution and n 1 and n2 are the
18. The energy required to break one mole of moles of solute and solvent respectively in
u

hydrogen-hydrogen bonds in H2 is 436 kJ. the solution then the correct relation
St

What is the longest wavelength of light between p and p° is


required to break a single hydrogen- é n1 ù é n2 ù
(a) p° = p ê ú (b) p° = p ê ú
hydrogen bond? ë n1 + n2 û ë n1 + n2 û
(a) 68.5 nm (b) 137 nm é n2 ù é n1 ù
(c) 274 nm (d) 548 nm (c) p = p° ê ú (d) p = p° ê ú
ë n1 + n2 û ë n1 + n2 û
19. The correct order of O ¾ O bond length in
25. Ionic solids with Schottky defect may
O2 , H2O2 and O3 is
contain in their structure
(a) O 2 >O 3 >H2O 2
(a) cation vacancies only
(b) H2O 2 >O 3 >O 2
(b) cation vacancies and interstitial cations
(c) O 3 >O 2 >H2O 2
(c) equal number of cation and anion vacancies
(d) O 3 >H2O 2 > O 2
(d) anion vacancies and interstitial anions
WB JEE (Engg.) Solved Paper 2016 7
26. The condition for a reaction to occur 29. Ozonolysis of an alkene produces only one
spontaneously is dicarbonyl compound. The structure of the
(a) DH must be negative alkene is
(b) DS must be negative
(c) (DH - TDS ) must be negative (a) H3C CH CH CH3
(d) (DH + TDS ) must be negative
(b)
27. The order of equivalent conductances at
infinite dilution for LiCl, NaCl and KCl is

)
(c)

be
(a) LiCl >NaCl >KCl (d) CH3 CH CH CH CH2
(b) KCl >NaCl >LiCl
(c) NaCl >KCl >LiCl

Tu
(d) LiCl >KCl >NaCl 30. From the following compounds, choose the
one which is not aromatic.
28. The molar solubility (in mol L-1) of a
sparingly soluble salt MX4 is ‘S’. The

ou
(a) (b)
corresponding solubility product is Ksp . S in +
terms of ‘Ksp ’ is given by the relation
æK ö
(a) S = ç sp ÷
è 128 ø
1/ 4

(c) S =(256 Ksp )1/ 5


æK ö
(b) S = ç sp ÷
è 256 ø
1/ 5

(d) S =(128 Ksp )1/ 4


(Y (c)

(d)
on
••
Me2N CH3

Category II (Q. 31 to Q. 35) Only one answer is correct. Correct answer will fetch full marks 2. Incorrect
pi

1
answer or any combination of more than one answer will fetch - mark.
2
m

31. Identify X in the following sequence of 32. Compound X is tested and the results are
reactions. shown in the table.
ha

CH3 ¾ C H ¾ C H ¾ CH2 ¾ CH2 ¾ CH3


Test Result
½ ½
C

Br Br (i) NaNH2 Aqueous sodium Gas given off which turns


¾¾¾¾¾¾® X hydroxide is added, damp red litmus paper
(ii) Na in liquid NH3
then heated gently. blue.
dy

(a) CH3 ¾ CH ¾ CH ¾ CH2CH2CH3


Dilute hydrochloric Effervescence, gas given
½ ½
Br NH2 acid is added. off which turns lime water
milky and acidified
u

H 3C H K 2Cr2O 7 paper green.


St

(b) C C Which ions are present in compound X?


H CH2CH2CH3 (a) Ammonium ions and sulphite ions
(b) Ammonium ions and carbonate ions
H3C CH2CH2CH3 (c) Sodium ions and carbonate ions
C C (d) Ammonium ions and sulphate ions
(c)
H
33. The time taken for an electron to complete
H
one revolution in Bohr orbit of hydrogen
(d) CH3 ¾ CH ¾ CH ¾CH2CH2CH3
½ ½
atom is
NH2 NH2 4m2 pr 2 n2 h2 4p 2 mr 2 nh
(a) 2 2
(b) 2
(c) (d)
n h 4mr nh 4p 2 mr 2
8 WB JEE (Engg.) Solved Paper 2016

34. Amongst the following , which should have 35. The major products obtained during
the highest rms speed at the same ozonolysis of 2, 3-dimethyl-1-butene and
temperature? subsequent reductions with Zn and H2O are
(a) SO 2 (a) methanoic acid and 2-methyl-2-butanone
(b) CO 2 (b) methanal and 3-methyl-2-butanone
(c) O 2 (c) methanol and 2, 2 -dimethyl-3-butanone
(d) H2 (d) methanoic acid and 2-methyl-3-butanone

)
Category III (Q. 36 to Q. 40) One or more answer (s) is (are) correct. Correct answers will fetch marks 2.

be
Any combination containing one or more incorrect answer (s) will fetch 0 mark. If all correct answers are
not marked and also no incorrect answer is marked then score = 2 ´ number of correct answers
marked/actual number of correct answers.

Tu
36. Choose the correct statement(s) among the 38. Of the following molecules, which have
following. shape similar to CO2 ?

ou
H 3C H H CH3 (a) HgCl 2 (b) SnCl 2 (c) C 2H2 (d) NO 2
(a) C C and C C are enantiomers 39. In which of the following mixed aqueous
H CH3 H 3C
(b) CH3CHO on reaction with HCN gives
racemic mixture
H
(Y solutions, pH = pKa at equilibrium?
(1) 100 mL of 0.1 M CH3COOH + 100 mL of
0.1 M CH3COONa
on
C2H5 C2H5 (2) 100 mL of 0.1 M CH3COOH + 50 mL of 0.1
M NaOH
(c) CH3 C H and H C OH are enantiomers
(3) 100 mL of 0.1 M CH3COOH + 100 mL of
pi

OH CH3 0.1 M NaOH


m

(d) CH3 CH NOH shows geometrical isomerism (4) 100 mL of 0.1 M CH3COOH + 100 mL of
0.1 M NH3
ha

(a) (1) is correct


37. Which of the following statement(s) is (are) (b) (2) is correct
(c) (3) is correct
correct when a mixture of NaCl and K2Cr2O7
C

(d) Both (1) and (2) are correct


is gently warmed with conc. H2SO4 ?
(a) A deep red vapour is evolved 40. Amongst the following compounds, the
dy

(b) The vapour when passed through NaOH solution, one(s) which readily react with
gives a yellow solution ethanolic KCN.
(c) Chlorine gas is also evolved (a) Ethyl chloride (b) Chlorobenzene
u

(d) Chromyl chloride is formed (c) Benzaldehyde (d) Salicylic acid


St
Mathematics
Category I (Q. 1 to Q. 50) Only one answer is correct. Correct answer will fetch full mark 1. Incorrect
answer or any combination of more than one answer will fetch -1 / 4 mark.
1. If the solution of the differential equation 7. The cosine of the angle between any two
dy diagonals of a cube is
x + y = xe x be xy = e x f( x ) + C , then f ( x )
dx 1 1
(a) (b)

)
is equal to 3 2

be
(a) x + 1 (b) x - 1 2 1
(c) (d)
(c) 1 - x (d) x 3 3

Tu
2. The order of the differential equation of all 8. If x is a positive real number different from 1
parabolas whose axis of symmetry along such that log a x, log b x, log c x are in AP,
X-axis is then

ou
(a) 2 (b) 3 a+c
(a) b = (b) b = ac
(c) 1 (d) None of these 2
(c) c 2 = (ac )log a b (d) None of these

(Y
3. The line y = x + l is tangent to the ellipse
2 x2 + 3 y2 = 1. Then, l is 9. If a, x are real numbers and |a| < 1,| x| < 1,
then 1 + (1 + a ) x + (1 + a + a2 )x2 + K ¥ is
(a) -2 (b) 1
on
equal to
5 2
(c) (d) 1 1
6 3 (a) (b)
(1 - a) (1 - ax) (1 - a) (1 - x)
pi

4. The area enclosed by y = 5 - x2 and (c)


1
(d)
1
(1 - x) (1 - ax) (1 + ax) (1 - a)
m

y = | x - 1| is
5p 5p - 2 ö 10. If log 0.3 ( x - 1 ) < log 0.09 ( x - 1 ), then x lies in
(a) æç - 2 ö÷ sq units (b) æç
ha

÷ sq units
è 4 ø è 2 ø the interval
æ 5p 1 ö p
(c) ç - ÷ sq units (d) æç - 5ö÷ sq units (a) (2, ¥) (b) (1, 2 )
è 4 2ø è2 ø
C

(c) (-2, - 1) (d) None of these


13
5. Let S be the set of points, whose abscissae
and ordinates are natural numbers. Let P Î S,
11. The value of å ( i n + i n + 1 ), i = -1 is
dy

n =1
such that the sum of the distance of P from
(a) i (b) i - 1
(8, 0 ) and (0, 12 ) is minimum among all (c) 1 (d) 0
u

elements in S. Then, the number of such


12. If z 1, z2 , z3 are imaginary numbers such
St

points P in S is
(a) 1 (b) 3 (c) 5 (d) 11 1 1 1
that| z 1| = | z2| = | z3| = + + = 1,
6. Time period T of a simple pendulum of z 1 z2 z3
l then| z 1 + z2 + z3| is
length l is given by T = 2p . If the length is
g (a) equal to 1 (b) less than 1
increased by 2%, then an approximate (c) greater than 1 (d) equal to 3
change in the time period is 13. If p, q are the roots of the equation
(a) 2% (b) 1% x2 + px + q = 0, then
1
(c) % (d) None of these (a) p = 1, q = - 2 (b) p = 0, q = 1
2
(c) p = - 2, q = 0 (d) p = - 2, q = 1
10 WB JEE (Engg.) Solved Paper 2016

14. The number of values of k, for which the æ p pö æ 1 ö


2
equation x - 3 x + k = 0 has two distinct çcos - sin ÷ ç ÷
22. Let Q = ç 4 4 ÷ and x = ç 2 ÷, then
roots lying in the interval (0, 1), are ç sin
p p
cos ÷ ç 1 ÷
è 4 4 ø è 2ø
(a) three
(b) two Q3 x is equal to
(c) infinitely many
æ- 1 ö æ- 1 ö
(d) no value of k satisfies the requirement æ 0ö ç ÷ æ -1ö ç ÷
(a) ç ÷ (b) ç 2 ÷ (c) ç ÷ (d) ç 2÷
15. The number of ways in which the letters of è 1ø ç 1 ÷ è0ø 1 ÷

)
ç-
è 2 ø è 2ø

be
the word ARRANGE can be permuted such
that the R’s occur together, is 23. Let R be a relation defined on the set Z of all
7! 7!
(a) (b) integers and xRy, when x + 2 y is divisible

Tu
2! 2! 2!
by 3, then
6! (a) R is not transitive
(c) (d) 5! ´ 2 !
2!

ou
(b) R is symmetric only
1 1 1 (c) R is an equivalence relation
16. If 5
+ 6
= 4
, then the value of r is (d) R is not an equivalence relation
Cr Cr Cr
(a) 4 (b) 2
17. For positive integer n, n3 + 2 n is always
(c) 5 (d) 3
(Y
24. If A = {5 n - 4 n - 1 : n Î N} and
B = {16 ( n - 1 ) : n Î N), then
(a) A = B (b) A Ç B = f
on
divisible by
(c) A Í B (d) B Í A
(a) 3 (b) 7 (c) 5 (d) 6
25. If the function f : R ® R is defined by
18. In the expansion of ( x - 1 ) ( x - 2 ) . . . ( x - 18 ),
pi

f ( x ) = ( x2 + 1 )35 , " x Î R, then f is


the coefficient of x 17 is
m

(a) 684 (b) - 171 (a) one-one but not onto


(b) onto but not one-one
(c) 171 (d) - 342
(c) neither one-one nor onto
ha

19. 1 + n C 1 cos q + n C 2 cos 2q + K + n C n cos nq (d) both one-one and onto


equals 26. Standard deviation of n observations
C

q nq
n
nq a 1, a2 , a3 , K , a n is s. Then, the standard
(a) æç2 cos ö÷ cos (b) 2 cos 2
è 2ø 2 2 deviation of the observations
la 1, la2 , K , la n is
dy

n
2n q æ 2 qö
(c) 2 cos (d) ç2 cos ÷
2 è 2ø (a) ls (b) -ls (c)|l|s (d) ln s
27. Let A and B be two events such that
u

20. If x, y and z are greater than 1, then the value


1 31 7
and P( B ) =
St

1 log x y log x z P( A Ç B) = , P( A È B) = ,
6 45 10
of log y x 1 log y z is
then
log z x log z y 1 (a) A and B are independent
(a) log x × log y × log z (b) A and B are mutually exclusive
A 1
(b) log x + log y + log z (c) P æç ö÷ <
è Bø 6
(c) 0
B 1
(d) 1 - {(log x) × (log y) × (log z)} (d) P æç ö÷ <
è Aø 6
21. Let A be a 3 ´ 3 matrix and B be its adjoint 1° 1°
28. The value of cos 15° cos 7 sin 7 is
matrix. If| B| = 64, then| A| is equal to 2 2
(a) ± 2 (b) ± 4 1 1 1 1
(a) (b) (c) (d)
(c) ± 8 (d) ± 12 2 8 4 16
WB JEE (Engg.) Solved Paper 2016 11
29. The smallest positive root of the equation 36. The equation of a line parallel to the line
tan x - x = 0 lies in 3 x + 4 y = 0 and touching the circle
p p x2 + y2 = 9 in the first quadrant, is
(a) æç 0, ö÷ (b) æç , p ö÷
è 2ø è2 ø (a) 3 x + 4 y = 15 (b) 3 x + 4 y = 45
3p ö 3p
(c) æç p, ÷ (d) æç , 2 p ö÷ (c) 3 x + 4 y = 9 (d) 3 x + 4 y = 27
è 2 ø è2 ø
37. A line passing through the point of
30. If in a DABC, AD, BE and CF are the altitudes intersection of x + y = 4 and x - y = 2 makes

)
and R is the circumradius, then the radius of æ3 ö
an angle tan -1 ç ÷ with the X-axis. It

be
the circumcircle of DDEF is è4 ø
R 2R
(a) (b) intersects the parabola y2 = 4( x - 3 ) at points
2 3

Tu
R ( x 1, y 1 ) and ( x2 , y2 ), respectively. Then,
(c) (d) None of these | x 1 - x2| is equal to
3
16 32

ou
(a) (b)
31. The points ( -a, - b ), (a, b ), (0, 0 ) and 9 9
(a2 , ab ), a ¹ 0, b ¹ 0 are always (c)
40
(d)
80
9 9

(Y
(a) collinear
(b) vertices of a parallelogram 38. The equation of auxiliary circle of the ellipse
(c) vertices of a rectangle 16 x2 + 25 y2 + 32 x - 100 y = 284 is
(d) lie on a circle
on
(a) x2 + y2 + 2 x - 4 y - 20 = 0
32. The line AB cuts off equal intercepts 2a from
(b) x2 + y2 + 2 x - 4 y = 0
the axes. From any point P on the line AB
(c) ( x + 1)2 + ( y - 2 )2 = 400
pi

perpendiculars PR and PS are drawn on the


axes. Locus of mid-point of RS is (d) ( x + 1)2 + ( y - 2 )2 = 225
m

a
(a) x - y = (b) x + y = a 39. If PQ is a double ordinate of the hyperbola
2
x2 y2
ha

(c) x2 + y2 = 4a2 2 2
(d) x - y = 2 a 2
- = 1 such that DOPQ is equilateral. O
a2 b2
33. x + 8 y - 22 = 0, 5 x + 2 y - 34 = 0,
being the centre. Then, the eccentricity e
C

2 x - 3 y + 13 = 0 are the three sides of a


satisfies
triangle. The area of the triangle is 2 2
(a) 1 < e < (b) e =
(a) 36 sq units
dy

3 2
(b) 19 sq units
3 2
(c) 42 sq units (c) e = (d) e >
2 3
u

(d) 72 sq units
2
34. The line through the points andIf the vertex of the conic y - 4 y = 4 x - 4a
(a, b ) 40.
St

( -a, - b ), passes through the point always lies between the straight lines
x + y = 3 and 2 x + 2 y - 1 = 0, then
(a) (1, 1) (b) (3a, - 2 b )
1
(c) (a2 , ab ) (d) (a, b ) (a) 2 < a < 4 (b) - < a<2
2
1 3
35. The locus of the point of intersection of the (c) 0 < a < 2 (d) - < a <
x y x y 1 2 2
straight lines + = K and - = ,
a b a b K 41. A straight line joining the points (1, 1, 1 ) and
where K is a non-zero real variable, is given (0, 0, 0) intersects the plane 2 x + 2 y + z = 10
by at
(a) a straight line (b) an ellipse (a) (1, 2, 5) (b) (2, 2, 2)
(c) a parabola (d) a hyperbola (c) (2, 1, 5) (d) (1, 1, 6)
12 WB JEE (Engg.) Solved Paper 2016

42. Angle between the planes x + y + 2 z = 6 and log x


47. ò dx is equal to
2 x - y + z = 9 is 3x
p p p p 1
(a) (b) (c) (d) (a) (log x )2 + C
4 6 3 2 3
2
43. If y = (1 + x ) (1 + x2 ) (1 + x4 ) K (1 + x2 n ), (b) (log x )2 + C
3
æ dy ö 2
(c) (log x)2 + C
then the value of ç ÷ at x = 0 is
è dx ø 3
1

)
(a) 0 (b) -1 (d) (log x)2 + C

be
3
(c) 1 (d) 2
x
48. ò2 [ f ¢( x ) + f ( x ) log 2 ] dx is equal to
44. If f ( x ) is an odd differentiable function

Tu
defined on ( -¥, ¥ ) such that f ¢(3 ) = 2, then (a) 2 x f ¢( x) + C
(b) 2 x log 2 + C
f ¢( -3 ) is equal to
(c) 2 x f( x) + C
(a) 0 (b) 1 (d) 2 x + C

ou
(c) 2 (d) 4
1 æ1 ö
(1 - x ) 49. ò0 log ç - 1 ÷ dx is equal to
æ1 +x ö (1 - x) èx ø
45. lim ç
x ® 1è 2

(a) 1
÷
+ xø
is equal to

(b) does not exist


(Y (a) 1
(b) 0
(c) 2
on
2
(c) (d) ln 2 (d) None of the above
3
50. The value of
é æ e öù
pi

ì n + 1 + n + 2 + K + 2n - 1 ü
ê log çè x2 ÷ø ú lim í ý is
-1
46. If f ( x ) = tan ê ú n ®¥ n3 / 2
m

2 î þ
ê log (ex ) ú
2 2
êë úû (a) (2 2 - 1) (b) ( 2 - 1)
ha

3 3
é 3 + 2 log x ù 2
+ tan -1 ê ú, then the value of f ¢¢( x ) (c) ( 2 + 1) (d)
ë 1 - 6 log x û 3
C

2
is equal to (2 2 + 1)
3
(a) x2 (b) x
dy

(c) 1 (d) 0

Category II (Q. 51 to Q. 65) Only one answer is correct. Correct answer will fetch full marks 2. Incorrect
u

1
answer or any combination of more than one answer will fetch - mark.
St

2
51. The sum of n terms of the following series (d) a2 x2 + (b 2 + 2 ac ) x + c 2 = 0
13 + 33 + 53 + 73 + K is 53. If w is an imaginary cube root of unity, then
(a) n2 (2 n2 - 1) (b) n3 (n - 1) the value of
(c) n3 + 8n + 4 (d) 2 n4 + 3n2 (2 - w) (2 - w2 ) + 2(3 - w) (3 - w2 )
52. If a and b are roots of ax2 + bx + c = 0, then + . . . + ( n - 1 ) ( n - w) ( n - w2 ) is
the equation whose roots are a 2 and b2 , is n2 n2
(a) (n + 1)2 - n (b) (n + 1)2 + n
(a) a2 x2 - (b 2 - 2 ac ) x + c 2 = 0 4 4
n2 n2
(b) a2 x2 + (b 2 - ac ) x + c 2 = 0 (c) (n + 1)2 (d) (n + 1) - n
4 4
(c) a2 x2 + (b 2 + ac ) x + c 2 = 0
WB JEE (Engg.) Solved Paper 2016 13
n
54. If C r - 1 = 36, n C r = 84 and n
C r + 1 = 126, (a) 0 (b) 1 (c) 2 (d) 3

then the value of n C 8 is 61. For non-zero vectors a and b, if


|a + b| < |a - b|, then a and b are
(a) 10 (b) 7 (c) 9 (d) 8
(a) collinear
55. In a group of 14 males and 6 females. 8 and 3 (b) perpendicular to each other
of the males and females, respectively are (c) inclined at an acute angle
aged above 40 yr. The probability that a (d) inclined at an obtuse angle
person selected at random from the group is dy
+ by2 = a cos x,

)
62. General solution of y
aged above 40 yr given that the selected dx

be
person is a female, is 0 < x < 1 is
2 1 (a) y2 = 2 a (2 b sin x + cos x) + ce -2 bx
(a) (b)

Tu
7 2
(b) (4b 2 + 1) y2 = 2 a (sin x + 2 b cos x) + ce -2 bx
1 5
(c) (d) (c) (4b 2 + 1) y2 = 2 a (sin x + 2 b cos x) + ce 2 bx
4 6
(d) y2 = 2 a (2 b sin x + cos x) + ce -2 bx

ou
56. The equation x3 - yx2 + x - y = 0 represents
Here, c is an arbitrary constant.
(a) a hyperbola and two straight lines
63. The points of the ellipse 16 x2 + 9 y2 = 400 at

(Y
(b) a straight line
(c) a parabola and two straight lines which the ordinate decreases at the same rate
(d) a straight line and a circle
at which the abscissa increases is/are given by
on
57. The locus of the mid-points of chords of the 16 16
(a) æç 3, ö÷ and æç -3, - ö÷
circle x2 + y2 = 1, which subtends a right è 3ø è 3ø
æ 16 ö æ 16 ö
angle at the origin, is (b) ç 3, - ÷ and ç -3, ÷
pi

1 è 3ø è 3ø
(a) x2 + y2 = 1 1ö 1 1
4
æ æ
(c) ç , ÷ and ç - , - ö÷
m

1 è 16 9 ø è 16 9ø
(b) x2 + y2 = 1 1ö 1 1ö
2 æ æ
(d) ç , - ÷ and ç - , ÷
ha

(c) xy = 0 è 16 9ø è 16 9 ø
(d) x2 - y2 = 0
64. The letters of the word COCHIN are
permuted and all permutations are arranged
C

58. The locus of the mid-points of all chords of


2
the parabola y = 4ax through its vertex is in an alphabetical order as in an English
another parabola with directrix dictionary. The number of words that appear
dy

before the word COCHIN is


(a) x = - a (b) x = a
a (a) 96 (b) 48
(c) x = 0 (d) x = -
u

2 (c) 183 (d) 267


St

59. If [ x ] denotes the greatest integer less than or é2 0 0 ù


65. If the matrix A = ê0 2 0 ú, then
equal to x, then the value of the integral ê ú
2 2 êë2 0 2 úû
ò0 x [ x ] dx equals
éa 0 0 ù
5 7
(a) (b) A = ê 0 a 0 ú, n Î N, where
n
3 3 ê ú
8 4 êë b 0 a úû
(c) (d)
3 3 (a) a = 2 n , b = 2 n (b) a = 2 n , b = 2 n
60. The number of points at which the function (c) a = 2 n , b = n 2 n - 1 (d) a = 2 n , b = n 2 n
f ( x ) = max {a - x, a + x, b}, - ¥ < x < ¥,
0 < a < b cannot be differentiable, is
14 WB JEE (Engg.) Solved Paper 2016
Category III (Q. 66 to Q. 75) One or more answer (s) is (are) correct. Correct answer (s) will fetch full
marks 2. Any combination containing one or more incorrect answer will fetch 0 mark. If all correct
answer are not marked and also no incorrect answer is marked, then score = 2 ´ number of correct
answer marked/actual number of correct answers.
66. On the ellipse 4 x2 + 9 y2 = 1, the points at 71. If A, B are two events such that P( A È B) ³ 3
which the tangents are parallel to the line 4
8 x = 9 y, are 1 3
and £ P ( A Ç B) £ , then
8 8

)
æ2 1 ö
(a) ç , ÷

be
è 5 5ø 11
(a) P( A) + P(B) £
2 1 8
(b) æç - , ö÷ 3
è 5 5ø (b) P( A) × P(B) £

Tu
2 1 8
(c) æç - , - ö÷ 7
è 5 5ø (c) P( A) + P(B) ³
2 1 8
(d) æç , - ö÷

ou
è5 5ø (d) None of the above

ì 1, for 0 £ t < 1 72. If the first and (2 n - 1 )th terms of an AP, GP


67. If f( t ) = í , then

3000 æ
î 0, otherwise
2016 ö
ò-3000 ççè r ¢ =å2014f( t - r¢ ) f ( t - 2016 )÷÷ø dt is
(Y and HP are equal and their nth terms are
respectively a, b, c, then always
(a) a = b = c
on
(b) a ³ b ³ c
(c) a + c = b
(a) a real number (b) 1
(d) ac - b 2 = 0
(c) 0 (d) does not exist
pi

68. If the equation x2 + y2 - 10 x + 21 = 0 has 73. The coordinates of a point on the line
m

1
real roots x = a and y = b, then x + y + 1 = 0, which is at a distance unit
5
(a) 3 £ x £ 7
ha

(b) 3 £ y £ 7 from the line 3 x + 4 y + 2 = 0, are


(c) -2 £ y £ 2 (a) (2, - 3)
(d) -2 £ x £ 2 (b) (-3. 2 )
C

(c) (0, - 1)
69. If z = sin q - i cos q, then for any integer n, (d) (-1, 0)
1 np
= 2 cos æç - nqö÷
dy

(a) zn + 74. If the parabola x2 = ay makes an intercept of


zn è2 ø
1 æ np ö length 40 units on the line y - 2 x = 1, then
(b) zn + n = 2 sin ç - nq÷
u

z è2 ø a is equal to
1 æ np ö
(c) zn - n (a) 1
St

= 2 i sin ç nq - ÷
z è 2 ø (b) -2
1 np
(d) zn - n = 2 i cos æç - nqö÷ (c) -1
z è2 ø (d) 2
70. Let f : X ® X be such that f [ f ( x )] = x, for all 75. If f ( x ) is a function such that
x Î X and X Í R, then f ¢( x ) = ( x - 1 )2 (4 - x ), then
(a) f is one-to-one (a) f(0) = 0
(b) f is onto
(b) f( x) is increasing in (0, 3)
(c) f is one-to-one but not onto
(d) f is onto but not one-to-one (c) x = 4 is a critical point of f( x)
(d) f( x) is decreasing in (3, 5)
Answers
Physics
1. (b) 2. (b) 3. (b) 4. (c) 5. (*) 6. (b) 7. (*) 8. (d) 9. (c) 10. (d)
11. (c) 12. (b) 13. (a) 14. (b) 15. (b) 16. (d) 17. (c) 18. (a) 19. (c) 20. (d)
21. (d) 22. (d) 23. (a) 24. (d) 25. (c) 26. (*) 27. (c) 28. (c) 29. (b) 30. (b)

)
31. (a) 32. (b) 33. (*) 34. (a) 35. (c) 36. (*) 37. (a,c) 38. (a,b) 39. (*) 40. (a,d)

be
Chemistry
1. (a) 2. (c) 3. (c) 4. (b) 5. (c) 6. (a) 7. (a) 8. (b) 9. (d) 10. (a)

Tu
11. (b) 12. (c) 13. (b) 14. (c) 15. (d) 16. (c) 17. (b) 18. (c) 19. (b) 20. (b)
21. (c) 22. (d) 23. (a) 24. (c) 25. (c) 26. (c) 27. (b) 28. (b) 29. (b) 30. (b)

ou
31. (b) 32. (d) 33. (c) 34. (d) 35. (b) 36. (b,d) 37. (a,b,d ) 38. (a,c) 39. (a,b,d) 40. (a,c)

Mathematics
1.
11.
(b)
(b)
2.
12.
(a)
(a)
3.
13.
(c)
(a)
4.
14.
(c)
(d)
5.
15.
(b)
(c)
6.
16. (Y (b)
(b)
7.
17.
(a)
(a)
8.
18.
(c)
(b)
9.
19.
(c)
(a)
10.
20.
(a)
(c)
on
21. (c) 22. (c) 23. (c) 24. (c) 25. (c) 26. (c) 27. (a) 28. (b) 29. (c) 30. (a)
31. (a) 32. (b) 33. (b) 34. (c) 35. (d) 36. (a) 37. (b) 38. (a) 39. (d) 40. (b)
41. (b) 42. (c) 43. (c) 44. (c) 45. (c) 46. (d) 47. (a) 48. (c) 49. (b) 50. (a)
pi

51. (a) 52. (a) 53. (a) 54. (c) 55. (b) 56. (b) 57. (b) 58. (c) 59. (b) 60. (c)
m

61. (d) 62. (b) 63. (a) 64. (a) 65. (d) 66. (b,d) 67. (a,b) 68. (a,c) 69. (a,c) 70. (a,b)
71. (a,c) 72. (b,d) 73. (b,d) 74. (a,b) 75. (b,c)
ha

(*) No option is correct.


C
u dy
St
Solutions
Physics
1 1 1 1 1 2
1. = + = + =
C C1 C 2 4 4 4
V=constant
C = 2µF
2 µF will be parallel to 4µF, so the resultant capacitance R

)
be
4mF 4mF
A B E=0

Potential on the surface of the hollow sphere

Tu
KQ 1 Q Q
V = = ⋅ =
6mF R 4 πε0 R 4 πε0 R
This is the potential (V) inside the charged hollow metal
C ′ = C1 + C 2 = 2µF + 4µF

ou
sphere.
= 6µ F
4. The de-Broglie wavelength associated with electron is
A 2mF B given by,

6mF
(Y λ=
h
2 meV
where, h = Planck’s constant
on
m = mass of electron
1
=
1
+
1 2
= µF e = charge on electron
C ″ 4 µF 4 µF 4 V = potential difference
pi

C ″ = 2 µF °
λ = 1 A = 10 − 10 m (given)
∴ C AB = 2 µF + 6 µ F
m

h2 h2
= 8µ F λ =
2
⇒ V =
2 meV 2 meλ2
ρl
R =
ha

2. ( 6.6 × 10 − 34 )2
A
= − 31
where, ρ = resistivity 2 × 9.1 × 10 × 1.6 × 10 − 19 × (1 × 10 − 10 )2
l = length of the resistance wire 6.6 × 10 − 34
× 6.6 × 10 − 34
C

=
A = area of cross-section 2 × 9.1 × 10 − 31
× 1.6 × 10 −19 × 10 − 20
When the wires are connected in series, then
6.6 × 6.6 × 10 2 4356 × 10 2
dy

R = R1 + R 2 = = = 150 V
2 × 9.1 × 1.6 2912
ρl ρl
R = 11 + 1 2
A1 A2 5. Work function of cesium = 2.27 eV
u

ρ1l1 ρ2l 2 The wavelength of incident light


= + (QA1 = A 2)
St

A A λ = 600 nm = 600 × 10 − 9 m
ρeq = resistivity of combination
The energy of incident photon = hν
l = l1 + l 2 hc
A= A E=
λ
l + l 2 ρ1l1 + ρ2l 2 6.6 × 10 − 34 × 3 × 10 8
Thus, ρeq ⋅ 1 = = eV
A A
600 × 10 − 9 × 1.6 × 10 − 19
ρ l + ρ 2l 2
ρeq = 1 1
l1 + l 2 = 2.06 eV
hc
<φ (work function)
3. Inside a hollow charged sphere, the electric field λ
intensity is always zero. 2.06 eV < 2.27 eV
∴ E=0 So, no emission of electrons will take place.
but potential is constant. Hence, none of the given options is correct.
WB JEE (Engg.) Solved Paper 2016 17
6. The number of de-Broglie wavelength contained in the 9. Thickness of glass plate = t
second Bohr orbit of Hydrogen atom is 2.
Refractive index = µ
1  1 1 
7. = R  2 + 2 Velocity of light = c
λ n1 n 2 distance t
Speed of light = =
For Balmer series, time T
n1 = 2, n2 = 3, 4, 5 .... T =
t
speed of light
= R  2 − 2
1 1 1

λB  2 4  Speed of light in glass plate

)
c Qµ = c 
=R  −
1 1 v =

be
 4 16  µ  v
1 3 t µt
=R⋅ ...(i) T = =
λB 16 c /µ c

Tu
Lyman series is obtained when n1 = 1, n2 = 2 , 3, 4, ... 10. Given, x = at + bt 2
For third line in Lyman series,
[ x ] = [bt 2 ]

ou
= R  2 − 2  = R 1 −
1 1 1 1
λL  1 4   16  Unit of b =
x metre m
= = 2
1 15 t2 (h)2 h
=R⋅ ... (ii)
λL

(Y
16
11. Given, A + B = A −B
3
1 / λ B R ⋅ 16 ∴ ( A + B)2 = ( A − B)2
=
1 15
R⋅ A 2 + B2 + 2 AB cos θ = A 2 + B2 − 2 AB cos θ
on
λL 16
λL 3 2 AB cos θ + 2 AB cos θ = 0
= ... (iii)
λ B 15 4 AB cos θ = 0
pi

3 A ≠ 0, B ≠ 0
λL = × λB Here,
15
m

∴ cos θ = 0
λ B = 600nm = 600 × 10 − 9 m
or θ = 90°
3
λL = × 600 × 10 − 9
ha

15 12. Given that velocity of ascending body = u


= 120 × 10 − 9 m = 120 nm.
C

Hence, none of the given options is correct. +u


8. Here, angle of incidence is 60°
dy

–u
i = 60° i
u

air 90°
glass Let there is no air resistance when body falls freely in
St

downward direction, the direction of falling body will


90°–i opposite to the ascending body.
∴ The velocity at the same height while the body falls
sin i freely = − u
µ =
sin r
13. The bodies are separated by a distance R, so the
sin i = µ sin r coordinates of m1 and m2 will be ( 0, 0) and ( R, 0)
here, r = 90 − i Xcm
sin i = µ sin( 90° − i ) = µ cos i
sin i m2
µ = = tan i = tan 60° m1
cos i
(0, 0)
µ = 3 (R, 0)
R
18 WB JEE (Engg.) Solved Paper 2016

From the formula of centre of mass, 17. In SHM, the acceleration of vibrating particle is
m x + m2 x2 proportional to displacement
Xcm = 1 1
m1 + m2 |a| = ω 2 x.
m1 × 0 + m2 × R
= Here a = 16 cm / s 2
m1 + m2
m2 ⋅ R = 16 × 10 −2 m / s 2
Xcm =
m1 + m2 Displacement, x = 4cm = 4 × 10 −2 m
∴ 16 × 10 − 2 = ω 2 ( 4 × 10 −2 )
14. T1 = 273 + 20 = 293 K

)
ω2 = 4
T2 = 273 + 40 = 313 K

be
The velocity of sound wave in gases or air is given by ω = ± 2 rad/s
γ R T1 2π
ω=
v = T

Tu
M
2π 2π
where, γ = Ratio of C p / C V T = = = π = 3142
. s
ω 2
R = gas constant
1 2
18. The work done in stretching the spring, W =

ou
γ R T1 Kx
∴ v1 = 2
M K = force constant of spring
γ R T2 x = extension in length of the spring
v2 =

v1
v2
=
T1
T2
M

=
293
313 (Y 1
W1 = Kx12
2
1
W 2 = Kx22
on
Given, v1 = 344.2 m/s 2
313 x1 = 1
∴ v 2 = v1 x2 = 1 + 1 = 2
293
pi

1
= 344. 2 × 1068
. W1 K (1)2 1
= 344. 2 × 103
. m/s = 2 =
W2 1 4
m

2
K (2 )
= 35575
. m/s 2
−− 356 m/s
~ W 2 = 4W1
ha

15. Mass of gas = 4g (given) W1 = 10 J


W 2 = 40 J
From perfect gas equation
∴ More work required = 40 J − 10 J = 30 J
C

pV = nRT
where n = number of moles 3RT
19. vrms =
M
here,  n = = 2 
4
dy

 2  R = gas constant
∴ pV = 2 RT T = absolute temperature
M = mass of gas
u

It will be the perfect gas equation for 4g of hydrogen.


3RT
vrms of H 2 =
St

16. Suppose temperature of Sun = T MH 2


When it is doubled, T ′ = 2T 3RT
From law of radiation (Stefan’s law) vrms of O 2 =
MO 2
E ∝ T4 = σ T4
vrms .H 2 MO 2
σ = Stefan’s constant. =
vrms .O 2 MH1
E1 ∝ T 4
Given vrms H 2 = c
E 2 ∝ (2T )4 MO 2
c 32
⇒ = = = 16 = 4
E1 T4 vrms O 2 MH1 2
=
E 2 2 4 .T 4
c
∴ vrms = .
E 2 = 16 E1. 4
WB JEE (Engg.) Solved Paper 2016 19
4 3
20. The tube is closed at one end. So, it will act like a 6 πηrvt = πr ρe ⋅ g
closed organ pipe. 3
24 π ⋅ r 3ρe ⋅ g 2 r 2 ρe ⋅ g
The lowest frequency produced in the tube η= = ⋅
3 × 6 πrvT 9 vT
here, D = 2 cm, r = 1 cm = 1 × 10 − 2 m
λ
l 4 ρe = 1.75 g / cm3 = 1.75 × 10 +3 kg / m3
g = 10 m / s 2
vT = terminal velocity

)
λ = 0.35 × 10 − 2 m / s
l= ,λ = 4

be
4
−2 2 +3
l = 1m 2 (10 ) × 1.75 × 10 × 10
η= × −2
f = 84 Hz
9 0.35 × 10

Tu
−4
v = nλ = 84 × 4 2 10 × 1.75 × 10 3 × 10
= ×
When wave propagate in air, the velocity 9 0.35 × 10 − 2
γp 2 × 175 × 10 × 10

ou
v = = × 10 poise
ρ 9 × 35
Here, p = 1.0 × 10 5 N / m2 − 1089 poise
~
ρ = 1.2 kg/m

(Y
3
22. Temperature of water = 0° C
γp Temperature of surrounding atmosphere = − 20° C
v2 =
ρ
Density of ice = ρ
on
v 2. ρ
γ = air –20°C
p
( 84 × 4)2 × 1.2 ice x
pi

=
1.0 × 10 5
135472.2
m

= = 1.354 0°C dx
10 5 water
≈ 1.4
ha

Coefficient of thermal conductivity = k


21. The gas bubble is rising through the liquid of density latent heat of melting = L
. g / cm3 with constant speed. The acting forces on
175
dQ kA(T1 − T2 )
C

bubble are The rate of heat flow H = =


dt x
kA[( 0 − ( − 20)]
=
dy

x
Fb kA × 20
=
x
u

dQ dm
= ⋅L (Q mL = Q )
dt dt
St

dm = ρA ⋅ dx
Fv
dQ dx kA ⋅ 20
= ρA ⋅ L=
dt dt x
Buoyancy force of liquid = Fb (in upward direction) Z 20k t 40k
∫0 ⋅ =
ρ L ∫0
⇒ =
2
x dx dt Z t
Viscous force due to liquid = FV (in downward direction) ρL
∴ FV = Fb So, thickness Z increases as a function of time t
For free falling in a viscous medium according to the above equation.
FV = 6 πηrvT
23. Let the radius of single drop = r
η = viscosity coefficient of the liquid
r = radius of bubble 2mm
Radius of small drop =
Fb = mg = v × ρ. g 2
4 = 1 mm
= π r 3ρe . g
3 = 1 × 10 − 3 m
20 WB JEE (Engg.) Solved Paper 2016

Surface tension of water = 0.072 N / m Potential drop across 1 k Ω or 1 × 10 3 Ω


The volume of large drop must be equal volume of all Collector resistor is 5 V.
small drops. V = IC R
πR 3 = 1000 πr 3 
4 4 V 5
3 3  IC = =
R 1 × 10 3
∴ R = (1000)1/ 3 ⋅ r = 10. r
= 5 × 10 − 3 A … (ii)
The initial potential energy of drops system
From Eqs. (i) and (ii),
U i = s × n × 4 πr 2
∆IC

)
= s × 1000 × 4 πr 2 ∆ IB =

be
45
= 1000 s ⋅ 4 πr 2 5 × 10 − 3 1
= = × 10 − 3
45 9
When the droplets coalesce, the final potential energy

Tu
= 0.111 × 10 − 3 A
of the system U f = s ⋅ 4 πR 2
= 111 × 10 − 6 A = 111 µA
= s ⋅ 4 π × 100 r 2

ou
Energy loss in the formation of large drop 26. Given, E = 3$i + 6$j + 2 k$ V /m
∆E = U f − U i B = 2 $i + 3$j T
= s ⋅ 4 π100r 2 − 1000s ⋅ 4 πr 2 v = 2 $i + 3$j m/s
= s ⋅ πr 2( 400 − 4000)
= − 3600s ⋅ πr 2

= − 3600 × 0.72 × 3.14 × (1 × 10 − 3 )2


(Y e = − 1.6 × 10 − 19C
The magnitude of electric field
on
| E | = ( 3)2 + ( 6)2 + (2 )2
From catenation
= 9 + 36 + 4
= 813.888 × 10 − 6
= 49 = 7
pi

= 81388
. × 10 − 4 J
The force acting on electron due to electric field
= 8146 × 10 − 4 J
m

. F = qE
| F | = | q || E |
24. 100 W 5.6 V
ha

= 1.6 × 10 − 19 × 7 N

10 V = 11.2 × 10 − 19 N
= 1.12 × 10 − 18 N
C

Breakdown voltage of Zener diode = 5.6 V Alternate Method


Voltage of source = 10 V
dy

B || v Magnetic field is parallel to velocity of electron.


Potential difference ∆V = (10 − 5.6) V = 4.4 V v || B Magnetic force
Resistance of circuit = 100 Ω Fm = q ( v × B )
u

The current passing through the Zener diode = qvBsinθ (Qθ = 0)


St

∆V = qvBsin 0
I=
R =0
4.4 ∴ Net force F = Fe + Fm
=
100
= (1.12 × 10 − 18 + 0) N
= 4.4 × 10 − 2 A
= 1.12 × 10 − 18 N
= 44 × 10 − 3 A
Hence, none of the given options is correct.
= 44 mA
27. Given, L1 = 6 mH = 6 × 10 −3 H
25. Current gain β = 45
∆Ic L2 = 8 mH = 8 × 10 −3 H
We know that, β=
∆ Ib
If two coils L1 and L2 are connected in series, the
∆Ic equivalent self-inductance L for assembly
∴ = 45 … (i)
∆ Ib L = L1 + L2 + 2 L1 L2
WB JEE (Engg.) Solved Paper 2016 21
= 6 × 10 −3 + 8 × 10 −3 + 2 6 × 8 × 10 − 6 31. The current I = I0 e − λt is of exponential nature.
−3 −3
= 14 × 10 +2 48 × 10 I=
dQ
= I0 e − λ t
dt
= 14 × 10 −3 + 2 × 7 × 10 − 3
dQ = I0e − λt dt
(Q 48 = 6.92 ~
− 7) Q t =∞
= 28 × 10 − 3 H = 28mH ∫ dQ = I0 ∫e
− λt
. dt
0 t =0
28. C=1mF R=1W
1 I0
Q = I0 × =
λ λ

)
be
32. For Fraunhoffer diffraction to occur, both source and
+ – screen should be at infinity, it is essential condition for
10 V diffraction pattern.

Tu
The time constant of the circuit 33. Information in the statement of the questions is
τ = C. R insufficient to calculate heat required to raise the
temperature from 100°C to 1000°C.
C = 1 µF = 1 × 10 −6

ou
Here,
R = 1 MΩ = 1 × 10 6 Ω 34. (i) When the body of mass m lifted up the forces acting
−6 on the body
τ = 1 × 10 × 1 × 10 = 1 s
6

We know that in one (1) time constant 63% charging is


done.

63
× q max (q = CV )
(Y T1
on
100 a=4.9 m/s2
63
= × 1 × 10 −6 × 10
100
pi

63
= × 1 µF × 10 = 6.3 µC
100 mg
6.3 × 10 −6C
m

q
⇒ V = = = 6.3 V mg
C 1 × 10 −6 F T1 − mg =
2
ha

mg 3mg
29. In series combination, T1 = mg + = ...(i)
2 2
R = R1 + R 2
(ii) When the body lowered the acting forces
C

Same current will pass in R1 and R 2. According to


question, R1 is increased by 0.5%. T2
∴ Increment in the resistance
dy

0.5
= × 400
100 a=4.9 m/s2
= 2.0 Ω
u

So to keep the potential unchanged in second


St

resistance the change required will be2.0 Ω increment. mg


30. The electric lines of force are always perpendicular to
mg
the equpotential surface ( mg − T2 ) =
2
θ = 90°
mg mg
T2 = mg − = ...(i)
2 2
On dividing Eq. (i) from Eq. (ii) we get
T1 3 mg / 2 3
Electric lines ∴ = =
T2 mg / 2 1
of force
= 3:1
Equipotential
surface
22 WB JEE (Engg.) Solved Paper 2016

35. B Both the charges are positive or both negative. (Since,


answers have the product q 1q 2 inside square root.
A Therefore, circular motion is not possible.
The statement of question is wrong. Either q 1 or q 2
should be negative.
37. The intensity of emitted electrons is
Let when link AB is removed, x is an equivalent of
1
remaining part of cube without link. I∝ 2
r

)
AB and rest part will be in parallel, so according to 1
I1 ∝ 2

be
question d
7 RR 1( x ) x
= 1 2 = = I2 ∝
1 1
= 4. 2
12 R1 + R 1 + x 1 + x
(d / 2 )2 d

Tu
7 x
= and I ∝ Number of photons per second ∝ N
12 1 + x
1
∴ N∝ 2
12 x = 7 + 7 x r

ou
12 x − 7 x = 7 ∴ Number of emitted electrons will become 4 times.
5x = 7 From Einstein's photo-electric equation
7
x= Ω KEmax = hν − φ

(Y
5
where, φ = work function of surface
36. ν = frequency of incident photon
v Since, ν remains unchanged, thus KE max as well as
on
q2 stopping potential remains unchanged.
q1 Q KEmax = eV s ∝ ν
m2 r
m1 where, V s = stopping potential.
pi

38. v
P v0
m

The necessary centripetal force, to move in a circular


a b
ha

path is provided by the Coulomb’s force. X Y


m1v 2 1 q q
= . 12 2
r 4 πε0 r t1 t2 t
O
C

1 q q
v = . 1 2 …(i)
4 πε0 r m1 Shape of the graph OP shows the acceleration in train
We know, v = rω ∴ tan θ = acceleration.
dy

2π α = 0
v
ω=
T t1

u

T = Shape of the graph pt shows the deceleration in train.


ω v
St

2π . r ∴ Similarly, β= 0
= …(ii) t2
v
β v0 / t 2 t1
∴ = =
Substitution the value of v from, Eq. (i) we get α v0 / t1 t 2
2 πr
T = Displacement = area of graph obtained between v-t
q 1q 2 1
4 πε0 r . m1 x = t1 . v0
2
1
4 π 2r 2 × 4 πε0 . r . m1 y = t 2 . v0
T = 2
q 1q 2 x t1
∴ =
16 π 3ε0r 3. m1 y t2
T =
q 1q 2 x t1 β
⇒ = =
y t2 α
None of the given options is correct.
WB JEE (Engg.) Solved Paper 2016 23
1 /3
39. Let mass of the drop = m é3 s . Rù
r>ê ú
Weight of the drop = mg ë 2 rg û
It will act downward. Hence, none of the given options is correct.
The force due to surface tension on the drop = s 2 pR
40. Current-carrying rectangular coil is placed in a
Where, R = radius of tap
non-uniform magnetic field. So, the force on each arm
It will act in upward direction. of the coil will different giving the resultant force as
The drop of water will detach when non-zero.
mg > s2 pR

)
Thus, option (a) is correct.
4
m = v ´ r = pr 3rg

be
But, Also, the torque acting on current-carrying coil in
3
4 3 non-uniform magnetic field will be non-zero.
\ pr rg > s ´ 2 pR
3 Thus, option (d) is also correct.

Tu
Chemistry

ou
1. For a carbonyl group to give Cannizaro reaction, it
should not have an a-hydrogen atom. Among the given –
O OH O OH – O
compound Ne 3 CCHO, C 6H 5C HO and HCHO respond –

(Y
4. OH –OH
to Cannizaro reaction but Cl 3CCHO do not due to the –H+
following reason.
– H
O O O Cl
on
OH
Cl Cl +– Cl
H H OH H OH
– Cl Cl Cl In alkaline medium the acidic proton present in the
HO
pi

Cl Cl molecule is abstracted to give enolate ion. In the next


step hydroxide ion if present at b-position leaves giving
O Cl O Cl
m

H a -b-unsaturated carbonyl compound. In (a), (c) and



Cl –
+ Cl (d), dehydration cannot take place due to improper
H O H O placement of groups.
ha

Cl Cl
5. The correct order is 2<1<3<4. Since, more the
When hydroxyl group attacks the carbonyl group a number of 2° — NH — group, more is the basic nature.
C

tetrahedral intermediate forms. This tetrahedral (i) Thus, 3 and 4 are more basic than 1 and 2.
intermediate will revert to a carbonyl compound by
! (ii) Between 3 and 4, 4 has one two ¾ NH 2 group
expelling the best leaving group. The C Cl 3 carbonium
dy

while 3 has only one ¾ NH 2 group.


is resonance stabilised therefore it is better leaving \ 4 is more basic than 3.
group than OH. These initial products exchange a
(iii) Between 1 and 2, 1 has one ¾ NH 2 group with no
u

proton to give ion of carboxylic acid and


trichloromethane (CHCl 3 ). resonance, thus is more basic than 2.
St

2. Primary (1°)-alcohols do not give Lucas reagent test at 6. C ¾ C bond is not formed in Cannizaro reaction while
room temperature. CH 3CH 2CH 2OH does not undergo other reactions result in the formation of C ¾ C bond.
SN 1 or SN 2 at room temperature. Cannizaro reaction
3. Iodoform test is used to detect the presence of
CH 3 ù O O
é ù é
ê ú ê ½ ú
ê ¾ C ¾ CH 3 ú or ê ¾ C ¾ H ú in a molecule.

2 H OH OH + OH
ê ½½ ú ê ½ ú
êë O úû ê OH ú
ë û
In the given compounds only CH 3 COOH does not fulfill Wurtz reaction
the structural requirement of iodoform test hence, it Å !
does not respond to iodoform test. 2 R - X + 2Na ¾® R - R + 2 Na X
24 WB JEE (Engg.) Solved Paper 2016
Reimer-Tiemann reaction 12. PCl 5 exists as [PCl 4 ]+ and [PCl 6 ]- in solid state.

OH OH O 2PCl 5 ¾® [PCl 4 ]+ [PCl 6 ]-

CHCl3 H
13. (i) Ortho and para hydrogens have different
3KOH nuclear spin.
In H 2 molecule, two protons in two H atoms with parallel
spin are called ortho-hydrogen
Friedel-Crafts acylation

)
O p p

be
Nucleus of hydrogen atom with same spin
RCOCl R
AlCl3 (ortho)

Tu
and with opposite spin are called para-hydrogen.

7. Option (a) is false as colloidal sols are not p p

ou
homogeneous.
Nucleus of hydrogen atom with different spin
Colloidal sols are heterogeneous mixture of dispersed (para)
phase and dispersion medium.
8. Paracetamol is 4-acetamidophenol, i.e.
N-(4-hydroxy phenyl) ethanamide (IUPAC Name). (Y (ii) Ortho-hydrogen is more stable than para form at
and above room temperature only, therefore para
form always tends to revert in ortho-form. Thus,
on
OH not correct.
(iii) The per cent composition of ortho and para
pi

changes with the temperature. Thus, their ratio


also change.
m

NHCOCH3 (iv) They have slightly different boiling point.


Hence, (b) is the answer.
ha

9. The option (d) is false as lonic radii of trivalent 14. H 2S 2O 8 shows ¾O ¾O ¾ (peroxy) bonding as follows:
lanthanides decreases with increase in atomic number O
O
C

due to lanthanoid contraction.


10. Only KNO 3 on heating do not giveNO 2 but all other give HO S O O S OH
(Peroxide
dy

D
(a) 2KNO 3 ¾¾ ® 2KNO 2 + O 2 O bond) O
D Peroxodisulphuric acid
(b) 2Pb(NO 3 )2 ¾¾ ® 2PbO + 4NO 2 + O 2
D
or
(c) 2Cu(NO 3 )2 ¾¾ ® 2CuO + 4NO 2 + O 2
u

Marshall’s acid
D
(d) 2 AgNO 3 ¾ ¾¾ ® 2 Ag + O 2 + 2NO 2
St

Red hot
Heavy metal nitrates liberate nitrogen dioxide on heating. 15. Iron (Fe) is used to obtain Cu-metal from CuSO 4 ( aq )
11. Due to hydrogen bonding, HF shows highest boiling because Fe is more reactive than Cu (and also easily
point. available and is cheaper) thus, replace the copper from
CuSO 4 to give copper (Cu).
Thereafter van der Waals’ force decide the boiling
point. CuSO 4 ( aq ) + Fe( s ) ¾® FeSO 4 ( aq ) + Cu( s )

Larger the size or molecules mass, greater is the van der 16. Radium in isolated form and in its chloride (i.e. Ra + )
Waals’ forces. Hence, higher is the boiling point. only differ in the number of orbital electron.
Q van der Waals’ forces is more for HI than HBr, which is But, radioactivity is independent of chemical environment
in turn is more than HCl. of an ion or atom, i.e. radioactivity is the phenomenon
Hence, correct order is HF > HI > HBr > HCl. which does not depend on the orbital electrons but
depends only on the composition of nucleus.
WB JEE (Engg.) Solved Paper 2016 25
17. The correct order of solubility of sulphates in water is 22. The element belonging to 4th period and 15th group is
BeSO 4 > MgSO 4 > CaSO 4 > SrSO 4 > BaSO 4 arsenic (As). The outer electronic configuration of 15th
group element is ns 2np3.
Solubility of 2nd group sulphates decreases as we
Since arsenic belongs to 4th period, therefore it has
move down the group due to less release of hydration
filled ‘d’ orbital too. Thus, the electronic configuration
energy. Be 2+ < Mg 2+ < Ca 2+ < Sr 2+ < Ba 2+ (Ionic
of As is [Ar ]4 s 2 3 d 10 4 p3.
Size) As hydration energy decreases more rapidly than
latice energy, the solubility decreases down the group. So, it has filled ‘s’ and ‘d’ orbital and half-filled p-orbital.

Hydration Energy µ
Charge 23. For exothermic reaction,

)
Size 1
Q Kp µ

be
While lattice energy almost remains constant. Hence, T
solubility decreases. 1
Thus, as increases (i.e. T decreases), K p increases.
T

Tu
18. Given, energy of one mole of H 2 bonds = 436 kJ
n × hc Q DH for exothermic reactions is negative.
Q E=
l 24. Given,

ou
(energy for one H ¾H bond) p° = vapour pressure of pure solvent
(n =number of H ¾H bonds) p = vapour pressure of solution
n × hc
\ l= n1 = moles of solute

l=
E
6.634 ´ 10 - 34 Js ´ 3 ´ 10 8 ms -1 ´ 6.023 ´ 10 23
436 ´ 10 3 J
m (Y n2 = moles of solvent
Thus, according to Raoult’s law.
p° - p n1
on
= x1 =
= 0.2736 ´ 10 - 34 + 8 + 23 - 3
p° n1 + n2
-6
= 0.2736 ´ 10 m (x1 = mole fraction of solute)
pi

= 273.6 nm » 274 nm p n1
or, 1- =
p° n1 + n2
19. Correct order of O ¾ O bond length is
m

p n1
= 1-
H 2O 2 > O 3 > O 2 p° n1 + n2
ha

(i) Between O 3 and O 2 n1 + n2 - n1


Þ
QBond order of O 2 > O 3 n1 + n2
Thus, bond length of O 3 > O 2. p n2
=
C

æ 1 ö p° n1 + n2
çQ Bond order µ ÷
è Bond length ø æ n2 ö
Þ p = p° çç ÷
÷
dy

(ii) Between O 3 and H 2O 2 è n1 + n 2 ø


Q O 3 has p-bonds in resonance so, has shorter
bond length than O ¾O in H 2O 2.
25. In Schottky defect, equal number of cations and anions
u

are missing creating equal number of cation and anion


\ Hence, (b) is the answer. vacancies.
St

20. Calcium carbide is calcium acetylide, having structure 26. For a spontaneous reaction, DG = negative and
È È D S = positive
Ca 2 + × C ºº C
Q DG = D H - TDS = - ve
Thus, it has one sigma ( s ) and two pi (p) bonds.
Hence, DH - TDS = - ve only when D S = + ve and
21. Let Z = atomic number DH = -ve.
M = atomic mass 27. QKCl is more ionic than NaCl while NaCl is more ionic
-a - 2b M- 4
than LiCl.
M
E ¾¾¾® EM - 4 ¾¾¾® E Also, more the ionic nature, more is the conductivity
Z Z -2 Z
thus, order of equivalent conductance at infinite dilution
M
Hence, ZE and 2 E M - 4 has same atomic number with is KCl > NaCl > LiCl.
different atomic mass thus, they are isotopes.
26 WB JEE (Engg.) Solved Paper 2016
28. QFor a compound A x By 32. (i) It is a test for ammonia.
D
x y x+ y
Ksp = x × y × S . (where,S = solubility) NH 4Cl + NaOH ¾¾® NH 3 + NaCl + H 2O
For MX 4 (Basic)
Ksp = 1 ´ ( 4)4 × S 1 + 4 ½
½
Ksp = 256 ´ S 5 ¯
1/ 5 Turns red litmus blue
æK ö
\ S = ç sp ÷
è 256 ø (ii) It is a test for SO 2-
4 ions (Orange)

)
K 2Cr2O 7 + SO 2 + SO 24- + HCl ¾®

be
29. Only gives one compound on ozonolysis.
K 2Cr2 O 4 + SO 2 + H 2SO 4
Orange
(i) O3

Tu
O C C C C O ½D
(ii) Hydrology (C2) (C3) (C4) (C5) ¯
Cr (SO ) + K SO + H O
2 4 3 2 4 2

All other options give more than one aldehyde/ketone (Green)

ou
on ozonolysis.
33. Let the distance travelled in T time = 2 pr
30. For a compound to be aromatic, (Circumference of orbit)
(i) It should be planar
(ii) It should have ( 4n + 2 )p electrons
(iii) It should be able to delocalise its p electron
(Y \ Velocity( v ) =

Also, v =
2 pr
T
n× h
…(i)

…(ii)
on
2 pm r
density.
1 1
Among the given compounds, \ = frequency n = v ´
1, 3, 5, 7–cyclooctatetraene or cyclooctatetraene is not T (Time period ) 2 pr
pi

aromatic as its structure is non-planar. From Eq. (ii),


1 nh
m

n= =
T 2 pr ´ 2 pmr
nh
ha

n=
4 p 2r 2m
Br Br T (time taken in one revolution)
+ –
(i) NaNH2 4 p 2r 2m
C

31. CH3 C C CH2 CH2 CH3 =


nh
H H 1
dy

+ – 34. Q rms velocity (vrms ) µ Thus, smaller the mass,


NaNH2 M
+ –
(i) NaNH2 more will be the rms speed. H 2 has minimum molar
CH3 C C CH2CH2CH3
mass, So it has highest value of rms speed.
u

Br
St

4 3 2 1
CH3 C C CH2 CH2 CH3
Na in liquid

35. H 3C HC C CH2 (i) ozonolysis


H (ii) Zn/H2O
NH3

CH3 CH3
+ –
NaNH2
4 3 2 H
H 3C H CH3 CH C O+ O C
H
C C 1
CH3 CH3 (HCHO)
H CH2CH2CH3 methanal
3-methyl 2-butanone
trans-alkene

Na in liquid NH 3 is a Birch reagent. It is used to alkyne 36. (a) These are cis and trans isomers not enantiomers
into trans alkene only. (Thus, false).
WB JEE (Engg.) Solved Paper 2016 27
(b) CH 3CHO on reaction with HCN gives racemic 39. pH of buffer solutions is given by
mixture of cyanohydrin (Thus, true). [salt ]
pH = pK a + log
OH H H OH [acid]
H
C O + HCN C + C (1) CH 3 COOH and CH 3 COONa
Cl2
H 3C CN H3C CN 100 ´ 0 .1 100 ´ 0 .1

(Thus, true) (cyno-hydrogen) No. of milimoles = 10 = 10


10
C 2H 5 C 2H 5 Q [salt] = [acid] = 10, pH = pK a + log

)
10
½ ½

be
(C) CH 3 ¾C ¾H and H ¾C ¾OH \ pH = pK a
½ ½
OH (2) CH 3 COOH + NaOH ¾® CH 3COONa + H 2O
CH 3 100´ 01
. 50´ 0.1

Tu
(i) (ii)
Initial = 10 5 0 0
(i) and (ii) both have R-configuration thus, are not
Final =5 0 5 5
enantiomers.
5

ou
(D) CH 3 ¾CH == NOH shows geometrical isomers. Q [salt] = [acid] = 5, pH = pK a + log
5
OH \ pH = pK a
H H

(Y
C =N C =N (3) CH 3 COOH + NaOH ¾® CH 3COONa + H 2O
H 3C OH 100 ´ 01
. 100 ´ 0.1
H3 C
(trans) (cis)
on
Initial = 10 10 0 0

37. (A) When NaCl and K 2Cr2O 7 warmed with H 2SO 4 Final =0 0 10 10
(i.e. in acidic medium), they produce deep red
pi

[salt] = 10 and [acid] = 0,


vapours of chromyl chloride (CrO 2Cl 2).
10
(B) When NaOH is passed, the product CrO 3 formed pH = pK a + log
0
m

in step (a) will react with NaOH and gives yellow


colour solution. \ pH ¹ pK a
ha

(C) Chlorine gas is not evolved, thus false. (4) CH 3COOH + NH 3 ¾® CH 3COONH 4
(D) In the given reaction, chromyl chloride (CrO 2Cl 2) is Initial 10 10 ¾® No buffer action
C

formed. All reactions are as follows: Final Due to weak CH 3COONH 4 salt
(i) K 2Cr2O 7 + 2H 2SO 4 ¾® 2KHSO 4 + 2CrO 3 + H 2O \ pH ¹ pK a
(ii) NaCl + H 2SO 4 ¾® NaHSO 4 + HCl
dy

40. (a) Ethyl chloride gives ethyl cyanide, when it reacts


(iii) CrO 3 + 2HCl ¾® CrO 2Cl 2 with KCN (ethanolic),
from (i) from (ii) Red vapours
Ethanolic
u

C 2H 5Cl + KCN ¾¾¾¾® C 2H 5 ¾ CN


(iv) CrO 2Cl 2 + 2NaOH ¾® Na 2CrO 4 + 2HCl
St

Yellow solution
(b) It does not react with KCN.
38. HgCl 2 and C 2H 2 both show linear shape as of CO 2.
(c) Benzaldehyde : KCN gives HCN in alcoholic
(Qboth have zero lone pair of electrons) medium and HCN gives cyanohydrin with
(a) HgCl 2 ¾® Cl ¾Hg ¾Cl (Linear) benzaldehyde.
OH
ƒ
(b) SnCl2 Sn (Bent) C 6H 5CHO + HCN ¾® C 6H 5 ¾CH
‚
CN
Cl Cl
(c) C2H2 OH
H C C H (Linear)
(d) Salicylic acid does not react.
COOH
(d) NO2 N (Bent)
O O
Mathematics
dy
1. Given, x + y = xe x Y
dx
dy y y=–x+1
Þ + = ex …(i) y=(x–1)
dx x
dy (–1, 2)
On comparing Eq. (i) by + Py = Q, we get
dx
1 (2, 1)

)
P = and Q = e x
x

be
1
X¢ X
ò x
dx O (1, 0) y = √5–x2
\ IF = e
= elog x = x

Tu
Hence, solution of differential equation,
y × x = ò xe x dx + C

ou
Þ xy = xe x - ò e x dx + C

Þ xy = xe x - e x + C

(Y
Let A be the area bounded by the given curves. Then,
Þ xy = e x ( x - 1) + C … (ii) 1 2
A= ò- 1 ( 5 - x 2 + x - 1) dx + ò1 ( 5 - x 2 - x + 1) dx
Q xy = e x f ( x ) + C [given]…(iii)
1 2
5 - x 2 dx + 5 - x 2 dx
on
On comparing Eqs. (ii) and (iii), we get = ò- 1 ò1
f( x ) = ( x - 1) 1 2
+ ò-1 ( x - 1) dx + ò1 (- x + 1) dx
2. The general equation of the parabola is x = ay 2 + b,
pi

1 2
2 é x2 ù é x2 ù
where a and b are arbitrary constants. = ò- 1 5 - x 2 dx + ê - xú + ê - + xú
ë2 û- 1 ë 2 û1
m

So, the differential equation is of order 2. 2


é1 5 x ù 5
3. The equation of the line is y = x + l …(i) = x 5 - x 2 + sin- 1 -
ëê 2 5 ûú - 1 2
ha

2
On comparing it with y = mx + c, we get 5 2 5 1 5
= 1+ sin- 1 + 1 + sin- 1 -
m = 1 and c = l. 2 5 2 5 2
C

The equation of the ellipse is 1 5 æ 2 1 1 4ö


2 x 2 + 3 y2 = 1 =- + sin- 1 ç ´ 1- + 1- ÷
2 2 è 5 5 5 5ø
dy

x2 y2 1 5 5p 1
or 2
+ 2
=1 =- + sin- 1(1) = - sq units
æ 1 ö æ 1 ö 2 2 4 2
ç ÷ ç ÷
è 2ø è 3ø
u

5. Sum of distances of point P from point (8, 0) and (0, 12)


x2 y2 will be minimum, if points are collinear.
St

On comparing it with + = 1, we get


a2 b2 \ Equation of line at point (8, 0) and (0, 12).
1 1
a 2 = and b 2 = . x
+
y
=1
2 3 8 12
If the line touches the ellipse, then y x
1 1 5 Þ = 1-
c 2 = a 2m2 + b 2 Þ l2 = ×1 + = 12 8
2 3 6 x
5 5 Þ y = 12 - ´ 12
Þ l2 = Þl= 8
6 6 3
Þ y = 12 - x
4. The graphs of y = | x - 1| and y = 5 - x 2 are shown in 2
\ Points ( x, y) º (2, 9), ( 4, 6) and (6, 3).
the figure and the shaded region is the required region
bounded by the two curves. Hence, the number of such points P in S is 3.
WB JEE (Engg.) Solved Paper 2016 29
l 1 1 a
6. Given, T = 2 p = × -
g (1 - a ) (1 - x ) (1 - a ) (1 - ax )
On differentiating w.r.t. l, we get 1
=
dT 2p 1 (1 - ax ) (1 - x )
= ×
dl g 2 l
10. Given, log 0. 3( x - 1) < log 0. 09( x - 1)
dT p æ 2l ö
\ DT = Dl = ×ç ÷ Þ log 0. 3( x - 1) < log( 0. 3)2 ( x - 1)
dl gl è 100 ø
Þ log 0. 3( x - 1)2 < log 0. 3( x - 1)
l 1 T

)
= 2p × = Þ ( x - 1)2 > x - 1 [Q 0.3 < 1]
g 100 100

be
2
DT 1 Þ x + 1- 2x - x + 1> 0
Þ = = 1%
T 100 Þ x2 - 3 x + 2 > 0

Tu
Hence, approximate change in the time period is 1%. Þ ( x - 1) ( x - 2 ) > 0
Þ x < 1, x > 2 Þ x > 2 [Q x <| 1]
7. Let the direction ratio’s of two diagonals of a cube are
\ (2, ¥)
(1, 1, 1) and ( - 1, 1, 1).

ou
a1a2 + b1b 2 + c1c 2 Hence, x lies in the interval (2, ¥).
\ cos q =
a12 + b12 + c12 a22 + b 22 + c 22 11. We have,
13

(Y
(1) ´ ( - 1) + (1) ´ (1) + (1) ´ (1) S (i n + i n + 1)
= n=1
2 2 2 2 2 2
(1) + (1) + (1) ( - 1) + (1) + (1)
13 13
- 1+ 1+ 1 1 = S in + S in+ 1
= =
on
n=1 n=1
3 3 3
æ 1 - i 13 ö 13
2 æ1 - i ö
= iç ÷+ i ç ÷
8. Given, log a x, log b x and logc x are in AP. è 1- i ø è 1- i ø
pi

\ 2 log b x = log a x + logc x æ1 - i ö æ1 - i ö


1 1 = iç ÷-ç ÷
Þ 2 log b x = + è1 - i ø è1 - i ø
m

log x a log x c
= i -1
2 log x ac
Þ =
ha

log x b log x a log x c 12. We have,


log x b | z1 | = | z2 | = | z3 | = 1
Þ 2 log x c = (log x ac )
log x a Þ | z1 |2 = | z2 |2 = | z3 |2 = 1
C

Þ 2 log x c = log a b × log x ac Þ z1 z1 = z2 z2 = z3 z3 = 1


Þ log x c 2 = log x ( ac )log a b 1 1 1
Þ = z1, = z2, = z3
dy

z1 z2 z3
\ c 2 = ( ac )log a b
1 1 1
9. We have, Now, + + =1
z1 z2 z3
u

1 + (1 + a ) x + (1 + a + a 2 ) x 2 + K ¥
Þ | z1 + z2 + z3 | = 1
St

¥
= S (1 + a + a2 + K + an - 1 ) xn - 1 Þ | z1 + z2 + z3 | = 1
n=1
¥ nö
\ | z1 + z2 + z3 | = 1
æ1 - a n-1
= S ç ÷ x 13. Given, p, q are roots of equation x 2 + px + q = 0.
n=1 è 1- a ø
¥
xn - 1 an xn - 1
¥ \ p + q = - p Þ2p + q = 0 … (i)
= S - S and pq = q
n = 11 - a n=1 1- a

¥ ¥ Þ p=1
1 a
= S xn - 1 - S ( ax)n - 1 Put p = 1 in Eq. (i), we get
1- a n=1 1- a n=1
2 ´ 1+ q = 0
1
= [(1 + x + x 2 + K ¥)] Þ q +2 =0
1- a
Þ q = -2
a
- [{1 + ax + ( ax )2 + K ¥}] Hence, p = 1, q = - 2
1- a
30 WB JEE (Engg.) Solved Paper 2016
n n
14. Given equation is q q q
= æç2 cos ö÷ Re æçcos + i sin ö÷
x2 - 3 x + k = 0 è 2ø è 2 2ø
n
b 3 q nq q
As, - = Ï ( 0, 1) = æç2 cos ö÷ Re æçcos + i sin ö÷
2a 2 è 2ø è 2 2ø
Since, both roots cannot lie between 0 and 1. [by De moivre’s theorem]
So, no value of k is possible. n
q nq
= æç2 cos ö÷ × cos
15. Given word is ‘ARRANGE’. è 2ø 2

)
If R’s occur together, number of letters in word = 6 1 log x y log x z

be
To arrange R’s together number of ways =
2!
=1
20. Let D = log y x 1 log y z
2! log z x log z y 1
6! log x log y log z

Tu
\ Number of ways to permuted ‘ARRANGE’ =
2! log x log x log x
1 1 1 log x log y log z
16. Given, 5
+ 6 = 4 =
Cr Cr Cr log y log y log y

ou
log x log y log z
r ! ( 5 - r )! r ! ( 6 - r )! r ! ( 4 - r )!
Þ + = log z log z log z
5! 6! 4!

(Y
( 5 - r )! ( 6 - r )! 1 1 1
Þ + = ( 4 - r )! On taking , , common from R1, R 2 and
5 30 log x log y log z
(5 - r ) (6 - r ) (5 - r ) R 3, we get
Þ + =1
5 30 log x log y log z
on
2 1
Þ 30 - 6r + 30 - 11r + r = 30 = log x log y log z
log x × log y × log z
Þ r 2 - 17 r + 30 = 0 log x log y log z
pi

2
Þ r - 15r - 2 r + 30 = 0 \ D =0 [Q all rows are identical]
Þ r( r - 15) - 2( r - 15) = 0 21.
m

We have, | B| = 64
Þ ( r - 2 )( r - 15) = 0
Þ | adj A | = 64
\ r = 15, 2
ha

Þ | A |2 = 64
Hence, the value of r is 2.
\ | A| = ± 8
17. For n = 1, 2, 3,K, we find that n3 + 2 n takes values 3,
C

22. Given,
12, 33 , K , which are divisible by 3.
écos p pù é 1 ù
- sin
18. Coefficient of x17 in ( x - 1) ( x - 2 ) K ( x - 18) ê
Q=ê 4 4 ú and x = ê 2 ú.
dy

p p ú ê 1 ú
= - (1 + 2 + 3 + ... + 18) ê sin cos ú ê ú
ë 4 4 û ë 2û
= - (171) = - 171
écos q - sin qù
u

19. Given, Let Q ( q) = ê ú


ë sin q cos q û
St

1 + nC1 cos q + nC 2 cos 2 q + K + nC n cos nq


Q Q 3 ( q) = Q( 3 q)
which is real part of complex number.
écos 3p 3p ù
- sin
( nC 0 + nC1 e iq + K ), æ pö ê
3
\ Q ç ÷ =ê 4 4 ú = é- 1 / 2 - 1 / 2 ù
i.e. Re ( nC 0 + nC1 e iq + K ) è 4ø 3p 3p ú ê 1 / 2 ú
- 1/ 2 û
ê sin cos ú ë
ë 4 4 û
= Re (1 + e iq )n = Re (1 + cos q + i sin q)n
é- 1 1 ù é 1 ù é 1 1ù
q q q - - -
= Re æç2 cos 2 n + i 2 sin × cos ö÷ p ê
Now, Q 3 æç ö÷ x = ê 2 2 ú ê 2 ú = ê 2 2ú
è 2 2 2ø è 4ø 1 1 ú ê 1 ú ê 1 1ú
ê - ú ê ú ê - ú
éQ1 + cos q = 2 cos 2 q and ù ë 2 2û ë 2û ë 2 2 û
ê 2 ú
p é - 1ù
ê q q ú \ Q 3 æç ö÷ x = ê ú
ê sin q = 2 sin × cos ú è 4ø ë 0û
ë 2 2 û
WB JEE (Engg.) Solved Paper 2016 31
23. Reflexivity For reflexive ( x, x ) Î R. S xi
=l
n
x + 2 x = 3x
=lx
which is divisible by 3.
S( yi - y )2
Þ xRx Î R \ sy =
Hence, xRy is reflexive. N

Symmetric Let x + 2 y = 3l S( lxi - lx )2


=
Þ x = 3l - 2 y n
Now, y + 2 x = y + 2 ( 3l - 2 y) l2 S( xi - x )2

)
=

be
= y + 6l - 4 y n
= 6l - 3 y 2
= l2 s x = | l| s x = | l| s
Þ y + 2 x = 3 (2 l - y)
1

Tu
which is divisible by 3. 27. Given, P( A Ç B) =
6
\ ( x, y) Î R Þ ( y, x ) Î R 31
Hence, xRy is symmetric. P( A È B) =
45

ou
Transitive x + 2 y = 3l …(i) 7
and P( B ) =
and y + 2 z = 3m …(ii) 10
7 3

(Y
On adding Eqs. (i) and (ii), we get \ P( B) = 1 - =
x + 3 y + 2 z = 3l + 3 m 10 10
Þ x + 2 z = 3( l + m - y) Now, P( A È B) = P( A ) + P( B) - P( A Ç B)
which is divisible by 3. 31 3 1
Þ = P( A ) + -
on
45 10 6
Q ( x, y) Î R and ( y, z) Î R
31 1 3
Þ ( x, z) Î R \ P( A ) = + -
45 6 10
pi

Hence, xRy is transitive.


5
Q R is reflexive, symmetric and transitive. \ =
9
m

\ R is an equivalence relation. 1
æ B ö P( A Ç B) 6 3 1
24. We have, Then, Pç ÷ = = = >
ha

è Aø P( A ) 5 10 6
A = 5 n - 4n - 1 = (1 + 4)n - 4n - 1
9
= ( nC 0 + nC1 ´ 4 + nC 2 ´ 4 2 + K + nC n ´ 4 n ) 1
C

- ( 4n + 1) A P( A Ç B) 5 1
and P æç ö÷ = = 6 = >
= 4 2 ( nC 2 + nC 3 ´ 4 + K + nC n ´ 4 n - 2 ) è Bø P( B) 3 9 6
\ A contains some multiples of 16. 10
dy

Clearly, B contains all multiples of 16 including 0. 5 3 1


and, P( A ) ´ P( B) = ´ = = P( A Ç B)
\ A Í B. 9 10 6
u

Hence, A and B are independent.


25. Given, f( x ) = ( x 2 + 1)35, " x Î R.
1° 1°
St

Since, f( x ) is even function. 28. We have, cos 15° × cos 7 × sin7


2 2
Hence, the function is not one-one.
1 1° 1°
And f( x ) > 0, " x Î R.
æ
= cos 15° ç2 cos 7 × sin7 ö÷
2 è 2 2ø
Hence, the function is not onto.
1 é æ 1° ö ù éQ2 sin q × cos qù
\ Given, the function is neither one-one nor onto. = cos 15° êsin 2 ç7 ÷ ú ê = sin2 q ú
2 ë è 2 øû ë û
26. Let xi = a1, a2, a3, K , an;
1
= cos 15° sin15°
and yi = la1, la2, la3, K , lan. 2
\ yi = lxi 1 1
Sy = ´ (2 sin15° cos 15° )
Q y= i 2 2
N 1 1 1 1
Slxi = sin 30° = ´ =
= [Q yi = lxi, N = n] 4 4 2 8
n
32 WB JEE (Engg.) Solved Paper 2016
29. Given equation is Þ a( 0 - ab ) - b( 0 - a 2 ) + 1( 0) = 0
tan x - x = 0 Þ - a 2b + a 2b = 0
Þ tan x = x
Solutions are abscissae of points of intersection of the Hence, B, C and D are collinear.
curves y = tan x and y = x. From the figure, it is clearly So, the points A, B ,C and D are always collinear.
3p ö
visible that solution lies in æç p, ÷.
è 2 ø 32. Let the equation of line AB is
x y
Y + =1
2a 2a

)
Þ x + y = 2a …(i)

be
y=x
Y
(0, 2a)

Tu
B
X¢ p X
O
(0, 2k) S P (2h, 2k)

ou
(h , k ) (2a, 0)
X¢ X
R A

(Y
–p p 3p
x= Y¢ x= x= (2h, 0)
2 2 2
30. Let circumradius of DDEF be R ¢. We know,
on
ÐFDE = 180 ° - 2 A and FE = R sin2 A.
A

pi

Let the coordinates of the mid-point of RS be ( h, k ).


E
So, R and S are (2 h, 0) and ( 0, 2 k ).
m

F
Hence, the mid-point of RS is ( h, k ).
Q P lies on line AB.
ha

B D C Then, from Eq. (i), we have


2h + 2k = 2a
Now, by sine rule in DDEF
Þ h+ k=a
C

EF R sin2 A
2R¢ = = So, the locus of ( h, k ) is x + y = a.
sin ÐFDE sin(180 ° - 2 A )
33. Given three sides of a triangle are
dy

R
\ R¢ =
2 x + 8 y - 22 = 0 …(i)
31. Let the four points be A( - a, - b ), B( a , b ), C ( 0, 0) and 5 x + 2 y - 34 = 0 …(ii)
u

D( a 2, ab ). 2 x - 3 y + 13 = 0 …(iii)
St

If A, B and C are collinear. On solving Eqs. (i) and (ii), we get


½ - a - b 1½ x1 = 6, y1 = 2 (say)
Then, ½ a b 1½ = 0 On solving Eqs. (ii) and (iii), we get
½ ½
½ 0 0 1½ x2 = 4, y2 = 7 (say)
Þ - a( b - 0) + b( a - 0) + 1( 0) = 0 On solving Eqs. (i) and (iii), we get
Þ - ab + ab = 0 x3 = - 2, y3 = 3 (say)
Hence, A, B and C ae collinear. If the points of the vertices of the triangle are
Again, if B, C and D are collinear. ( x1, y1 ), ( x2, y2 ) and ( x3, y3 ), then the area of triangle
½ a b 1½ ½ 6 2 1½

Then, ½ 0 0 1½ = 0 = 4 7 1½
½ 2 ½ 2½ ½
½ a ab 1½ ½ - 2 3 1½
WB JEE (Engg.) Solved Paper 2016 33
1 Q Circle touches the line (iii), so by the condition of
= [6(7 - 3) - 2( 4 + 2 ) + 112
( + 14)]
2 tangency,
1 1 c 2 = a 2 (1 + m2 )
= (24 - 12 + 26) = (12 + 26)
2 2 2 2
1 Þ æ k ö = 9 é1 + æ - 3 ö ù
= ´ 38 ç ÷ ê ç ÷ ú
è 4ø è 4 ø ú
2 ëê û
= 19 sq units éQc = k , a = 3, m = - 3 ù
êë 4 4 úû
34. Let the given points be A( a , b ) and B( - a, - b ).

)
Þ k = ± 15
The equation of line passing through the points A and B

be
is Y
y - y1
y - y1 = 2 ( x - x1 )
x2 - x1

Tu
-b-b
Þ y-b= ( x - a) X′
-a-a 3x+ X
4 y=
2b 3x+ 15

ou
Þ y-b= ( x - a) 4 y
2a 3x+ =0
4 y=
b –1
Þ y - b = ( x - a) Y′ 5

(Y
a
Þ ay - ab = bx - ab Hence, 3 x + 4 y = 15 touches the circle in the first
Þ bx = ay …(i) quadrant.

Since, from the given points ( a 2, ab ) and ( a, b ) are


on
37. Given equations are
satisfy the Eq. (i), but ( a 2, ab ) is the required answer x+ y=4 …(i)
because (a, b) is already given in question. and x- y=2 …(ii)
pi

From Eqs. (i) and (ii), we get


35. Given equations of straight lines
x = 3 and y = 1
x y
m

+ =K …(i) 3
a b The line through this point making an angle tan- 1 with
x y 1 4
ha

and - = …(ii) the X-axis is


a b K
3 éQ m = 3 ù
Let the point of intersection be ( a, b ). ( y - 1) = ( x - 3)
4 êë 4 úû
So, from Eqs. (i) and (ii), we get
C

a b 3x 5 3x - 5
+ =K Þ y= - = …(iii)
a b 4 4 4
dy

a b 1 Since, this line intersects the parabola


and - =
a b K y 2 = 4( x - 3)at points( x1, y1 ) and( x2, y2 ), respectively.
2 2 2 2
æ aö - æb ö = 1 Þ a - b = 1 3x - 5
u

Þ ç ÷ ç ÷ \ Putting y = in equation of parabola, we get


èaø èbø a2 b2 4
St

2
x2 y2 æ 3x - 5ö
\ Locus - = 1, which is the equation of a ç ÷ = 4( x - 3)
a 2
b 2 è 4 ø
hyperbola. Þ 9 x 2 - 94 x + 217 = 0
36. Equation of circle is x 2 + y 2 = 9 …(i) 94 217
Þ x1 + x2 = and x1 x2 =
9 9
And equation of line is 3 x + 4 y = 0 …(ii)
\ | x1 - x2| = ( x1 + x2 )2 - 4 x1 x2
Equation of line parallel to the line (ii),
2
3x + 4y = k = æ 94 ö - 4 ´ 217
ç ÷
- 3x k è 9ø 9
Þ y= + …(iii)
4 4 32
=
9
34 WB JEE (Engg.) Solved Paper 2016
38. Given equation of ellipse is \ Vertex = ( a - 1, 2 )
16 x 2 + 25 y 2 + 32 x - 100 y = 284

Simplifying the given equation, we have ellipse as 3


( x + 1)2 ( y - 2 )2 2 (1, 2)
+ =1
25 16
1
So, the auxiliary equation of circle is
2 3
( x + 1)2 + ( y - 2 )2 = 25 3 1

)
Þ x 2 + 2 x + 1 + y 2 - 4 y + 4 = 25 2 2 x+y=3

be
2x+2y–1=0
\ x 2 + y 2 + 2 x - 4 y - 20 = 0

x2 y2 From figure,
39. Given equation of hyperbola is
=1 -

Tu
a b2 2 3
clearly, - < a-1< 1
and DOPQ is an equilateral triangle. PQ is double 2
ordinate of the hyperbola. 1
\ - < a<2

ou
2
Y P (a secq, b tanq)
41. Equation of line joining the points (1, 1, 1) and
(0, 0, 0) is

X′
(0, 0) O
X (Y Þ
x-0 y-0 z-0
=
1- 0 1- 0 1- 0
x= y= z=l
= =l (say)
on
So, the point is ( l, l, l ).
The point intersects the plane 2 x + 2 y + z = 10.
Q
pi

\ 2( l ) + 2( l ) + l = 10
Y′ (a secq, – b tanq) Þ 5l = 10
m

Þ l =2
Let the coordinates of P and Q be ( a sec q, b tan q) and Hence, the point is (2, 2, 2).
( a sec q, - b tan q), respectively.
ha

In DOPQ ,
42. Equations of planes are
x + y + 2z - 6 = 0 …(i)
OP = PQ
and 2x - y + z - 9 = 0 …(ii)
C

Þ a 2sec 2q + b 2 tan2 q = (2 b tan q)2


a1a2 + b1b 2 + c1c 2
Þ a 2sec 2 q = 3b 2 tan2 q \ cos q =
a1 + b12 + c12 a22 + b 22 + c 22
2

a2
dy

Þ sin2 q = 1 ´ 2 + 1 ´ ( - 1) + 2 ´ 1
3b 2 =
1+ 1+ 4 4 + 1+ 1
Now, sin2 q < 1
u

2 - 1+ 2 3 1
a2 = = =
Þ <1 6´ 6 6 2
St

2
3b
p
b2 1 Þ cos q = cos æç ö÷
Þ > è 3ø
a2 3
p
b2 4 \ q=
Þ 1+ > 3
a2 3
2 4 2 43. Given,
\ e > Þe >
3 3 y = (1 + x )(1 + x 2 ) (1 + x 4 ) + K + (1 + x 2n )
Þ log y = log(1 + x ) + log(1 + x 2 )
40. Equation of the vertex of conic
y 2 - 4 y = 4 x - 4a + log(1 + x 4 ) + K + log(1 + x 2n )

Þ y 2 - 4 y + 4 = 4 x - 4a + 4 1 dy 1 2x 3 x2 2 nx 2n - 1
Þ = + + + ... +
y dx 1 + x 1 + x 2 1 + x 3 1 + x 2n
Þ ( y - 2 )2 = 4[ x - ( a - 1)]
WB JEE (Engg.) Solved Paper 2016 35
dy é 1 2x 3 x2 2 nx 2n - 1 ù 48. Let I = ò 2 x [f ¢ ( x ) + f( x ) log 2 ] dx
Þ = yê + + +K+ ú
dx ë1 + x 1 + x
2
1+ x 3
1 + x 2n û
Consider g ( x ) = 2 x f( x )
æ dy ö æ 1 ö Þ g ¢ ( x ) = 2 x f ¢ ( x ) + 2 x f( x ) log 2
\ ç ÷ = yç ÷=1
è dx ø x = 0 è1 + 0 ø Þ g ¢ ( x ) = 2 x [f ¢ ( x ) + f( x ) log 2 ]
x
44. Given that f( x ) is an odd differentiable function. \ I= ò g ¢( x) dx = g ( x) + C = 2 f( x ) + C
Then, f( - x ) = - f( x ) 1 æ1 ö
Þ - f ¢( - x) = - f ¢( x)
49. Let I= ò0 log çè x - 1÷ø dx

)
Þ f ¢( - x) = f ¢( x) …(i) æ1 - xö

be
1
Put x = 3 in Eq. (i), we get = ò0 log çè ÷ dx
x ø
f ¢ ( - 3) = f ¢ ( 3) 1 æ x ö
f ¢ ( - 3) = 2 [Q f ¢ ( 3) = 2] Þ I= ò0 log çè 1 - ÷ dx = - I

Tu
\

1- x b b
æ1 + x ö 1- x [Q òa f( x) dx = òa f(a + b - x ) dx]
45. We have, lim ç ÷

ou
x ®1 è 2 + x ø
\ 2I = 0 Þ I = 0
1- x
é ù
æ 1 + x ö (1 + x )(1 - x) n+ 1+ n+2 +K+ 2 n - 1ú
= lim ç ÷ 50. Q lim ê

(Y
x ®1 è 2 + x ø n®¥ ê 3 ú
ë n2 û
1
æ 1 + x ö (1 + x) é 1 2 n - 1ù 1
= lim ç ÷ = lim ê 1 + + 1+ +K+ 1+ ú
x ®1 è 2 + x ø n®¥
ë n n n û n
on
1 1 n-1
1 r
æ 1 + 1 ö (1 + 1) æ 2 ö 2
=ç ÷ =ç ÷ =
2 = lim
n®¥
å n
1+
n
r =1
è 2 + 1ø è 3ø 3
pi

1
1 2 2
é æ e öù = ò0 1 + x dx = é (1 + x )3/ 2 ù = (2 2 - 1)
êë 3 úû
ê log çè x 2 ÷ø ú 3
m

0
é 3 + 2 log x ù
46. Given, f( x ) = tan- 1 ê 2 ú
+ tan-1 ê ú
ê log(ex ) ú ë 1 - 6 log x û 51. We have,
ha

êë úû 13 + 3 3 + 5 3 + 7 3 + K
é log e - 2 log x ù - 1 é 3 + 2 log x ù Now, Tn = (2 n - 1)3
= tan- 1 ê ú + tan ê 1 - 6 log x ú
ë log e + 2 log x û = 8n3 - 3 (2 n)2 (1) + 3 (2 n) (1)2 - (1)3
C

ë û
Let 2 log x = tan q, we get = 8n3 - 12 n2 + 6n - 1
dy

é 1 - tan q ù é 3 + tan q ù \ S n = S Tn
= tan-1 ê ú + tan
-1
ê 1 - 3 tan q ú 8n2 ( n + 1)2 12 n( n + 1)(2 n + 1)
ë 1 + tan q û ë û = -
4 6
u

é p ù
= tan- 1 ê tan æç - qö÷ ú + tan- 1 [tan tan-1( 3 + q)] +
6n ( n + 1)
-n
ë è 4 ø û 2
St

p p = 2 n2 ( n + 1)2 - 2 n ( n + 1) (2 n + 1) + 3n ( n + 1) - n
= - q + tan- 1 3 + q = + tan- 1 3
4 4 = n [2 n ( n + 1)2 - 2 ( n + 1) (2 n + 1) + 3 ( n + 1) - 1]
Q f( x ) = constant = n [2 n ( n2 + 1 + 2 n) - 2 (2 n2 + 3n + 1)
\ f ¢ ¢( x) = 0 + 3n + 3 - 1]
47. Let I = ò
log x
dx = n [2 n3 + 2 n + 4n2 - 4n2 - 6n - 2 + 3n + 2 ]
3x
1 = n [2 n3 - n] = n2 (2 n2 - 1)
Again, let log x = z Þ dx = dz
2x 52. Given, a and b are roots of ax 2 + bx + c = 0.
2z 2
\ I=ò dz = ò z dz -b
3 3 \ a+b=
a
2 z2 1
= × + C = (log x )2 + C c
3 2 3 and ab =
a
36 WB JEE (Engg.) Solved Paper 2016
Now, if the roots are a2 and b 2, then 56. Given equation is
a2 + b 2 = ( a + b )2 - 2 ab x 3 - yx 2 + x - y = 0
2
- bö 2c b 2 2c Þ x 2 ( x - y) + ( x - y) = 0
= æç ÷ - = 2 - …(i)
è a ø a a a Þ ( x 2 + 1) ( x - y) = 0
2 2
And
c c
a2b 2 = ( ab )2 = æç ö÷ = 2 Now, x2 + 1 ¹ 0
è aø a and x- y=0
The equation whose roots are a2 and b 2, is \ x= y

)
æ b 2 2c ö c2 So, the equation represents a straight line.
x2 - ç 2 - ÷ x+ 2 =0

be
èa aø a
57. Let ( h, k ) be the coordinates of the mid-point of a chord,
\ a 2 x 2 - ( b 2 - 2 ac ) x + c 2 = 0. which subtends a right angle at the origin.

Tu
Then, equation of the chord is
53. Given,
hx + ky - 1 = h2 + k 2 - 1 [using T = S ¢]
(2 - w ) (2 - w 2 ) + 2 ( 3 - w ) ( 3 - w 2 ) + K +
Þ hx + ky = h2 + k 2
( n - 1)( n - w ) ( n - w 2 )

ou
The combined equation of the pair of lines joining the
Now, Tn = ( n - 1) ( n - w ) ( n - w 2 )
origin to the points of intersection of x 2 + y 2 = 1 and
= ( n - 1) ( n2 - nw - nw 2 + w 3 ) hx + ky = h2 + k 2 is

\
= ( n - 1) ( n2 + n + 1) = n3 - 1
S n = STn
n2 ( n + 1)2
(Y æ hx + ky ö
x 2 + y2 - 1ç 2
èh + k ø
2
2

÷ =0
on
= Sn3 - S1 = -n Lines given by the above equation are at right angle.
4
Therefore, coefficient of x 2 + coefficient of y 2 = 0
54. Given,
pi

1
n
C r - 1 = 36 …(i) i.e. h2 + k 2 =
2
n
m

C r = 84 …(ii) 1
\ x 2 + y2 =
n 2
C r + 1 = 126 …(iii)
ha

On dividing Eq. (i) by Eq. (ii), we get 58. Let P ( at 2, 2 at ) be a moving point on the parabola
r
=
36 y 2 = 4 ax and S( a, 0) be its focus. Let Q ( h, k ) be the
n - r + 1 84 mid-point of SP.
C

Þ 84r = 36n - 36r + 36 at 2 + a


Then, h=
Þ 120r = 36n + 36 …(iv) 2
dy

Also, on dividing Eq. (ii) by Eq. (iii), we get 2 at + 0


and k=
r+1 84 2
=
n - r 126 2h
= t2 + 1
u

Þ
Þ 126r + 126 = 84n - 84r a
St

Þ 210r = 84n - 126 …(v) k


and t =
a
On solving Eqs. (iv) and (v), we get
n = 9and r = 3 2h k2
Þ = 2 +1
\ n
C 8 = 9C 8 = 9 a a
Þ k 2 = 2 ah - a 2 [on eliminatingt ]
55. Number of males = 14
Thus, the locus of ( h, k ) is y = 2 ax - a 2
2
and females = 6
Total = 14 + 6 = 20 Now, y 2 = 2 ax - a 2
a
Males above 40 yr = 8 Þ ( y - 0)2 = 2 a æç x - ö÷
è 2ø
and females above 40 yr = 3
Now, P (selected person is female and above 40 yr) The equation of the directrix of this parabola is
3 1 a a
= = x - = - , i.e. x = 0.
6 2 2 2
WB JEE (Engg.) Solved Paper 2016 37
59. Let, 63. Given that, the ordinate decreases at the same rate at
2 2 which the abscissa increases, therefore
I= ò0 x [ x ] dx
dy dx
1 2 2
=- …(i)
x 2 ´ 1 dx dt dt
= ò0 x × 0 dx + ò1 Also, equation of ellipse
2
é x3 ù 8 1 7 16 x 2 + 9 y 2 = 400 …(ii)
=ê ú =é - ù=
ê ú On differentiating w.r.t. t , we get
ë 3 û1 ë 3 3 û 3
dx dy
16 ´ 2 x + 9 ´ 2y =0
60. Graph of f( x ), dt dt

)
dx dy

be
Y Þ 16 x + 9y =0
b dt dt
dy dx
Þ 9y = - 16 x
dt dt

Tu
dy dy
–a a X Þ 9y = - 16 x æç - ö÷ [using Eq. (i)]
dt è dt ø
16

ou
Þ 9 y = 16 x Þ y= x ..(iii)
9
Using Eq. (iii) in Eq. (ii), we get
There are two sharp turns. Hence, f( x ) cannot be 2
16
16 x 2 + 9 æç x ö÷ = 400

61.
differentiable at two points.
Given, |a + b| < |a - b| (Y Þ 16 x 2 +
è9 ø
(16)2 2
9
x = 400
on
Now,
16 ù
|a + b|2 < |a - b|2 Þ 16 x é x + x = 400
êë 9 úû
Þ (a + b ) × (a + b ) < (a - b ) (a - b ) 25 400 ´ 9
16 x é x ù = 400 Þ x 2 =
pi

Þ
Þ |a |2 + |b|2 + 2 a × b < |a |2 + |b|2 - 2 a × b êë 9 úû 25 ´ 16
Þ 4a ×b < 0 x2 = 9 Þ
m

Þ x=± 3
Þ a ×b < 0 16 ö 16 ö
\ Required points are ç 3, ÷ and æç - 3, -
æ
÷.
Þ |a ||b| cos q < 0 è 3ø è 3ø
ha

Þ cos q < 0
Þ q Îobtuse angle
64. In a dictionary, the words at each stage are arranged in
alphabetical order.
C

\ a and b are inclined at an obtuse angle.


In the given problem, we must consider the words
62. Given, beginning with C, C, H, I, N, O in order. So,
dy
dy

y + by 2 = a cos x, 0 < x < 1 …(i) Number of words starting with CC = 4!


dx Number of words starting with CH = 4!
Let y2 = z Number of words starting with CI = 4!
u

dy dz Number of words starting with CN = 4!


Þ 2y =
St

dx dx Next word starting with CO i.e. COCHIN = 1


dy 1 dz \ Required number of words = 4 ´ 4! = 96
Þ y = …(ii)
dx 2 dx 65. We have,
1 dz
\ + by 2 = a cos x [using Eq. (ii)] é2 0 0ù
2 dx A = ê 0 2 0ú
dz ê ú
Þ + 2 by 2 = 2 a cos x êë2 0 2 úû
dx
Now, IF = e 2b ò dx = e 2bx é2 0 0ù é2 0 0ù é 4 0 0ù
\ A 2 = ê 0 2 0ú ê 0 2 0ú = ê 0 4 0ú
\ z × e 2bx = ò 2 a cos x × e 2bx dx ê ú ê ú ê ú
êë2 0 2 úû êë2 0 2 úû êë 8 0 4úû
2a
Þ y 2 e 2bx = (sin x + 2 b cos x ) e 2bx + C é 22 0 0ù
4b 2 + 1 ê ú
=ê 0 22 0 ú
Þ ( 4b 2 + 1) y 2 = 2 a (sin x + 2 b cos x ) + Ce - 2bx ê2 × 2 2 2
0 2 ú
ë û
38 WB JEE (Engg.) Solved Paper 2016
é 2n 0 0ù 68. Given,
n ê ú
Now, A =ê 0 2n 0ú x 2 + y 2 - 10 x + 21 = 0
ê n ×2 n 0 n
2 ú Þ x 2 - 10 x + y 2 + 21 = 0
ë û
éa 0 0ù Now, for real roots of x, D ³ 0
But An = ê 0 a 0ú [given] 100 - 4 ( y 2 + 21) ³ 0
ê ú
êë b 0 a úû Þ y2 £ 4 Þ - 2 £ y £ 2
n
é a 0 0ù é 2 0 0ù Also,
ê 0 a 0ú = ê 0 ú

)
n
\ 2 0ú y 2 = - x 2 + 10 x - 21
ê ú ê

be
n n
êë b 0 a úû êë n × 2 0 2 ú
û Now, for real roots of y,
Hence, a = 2n -4( x 2 - 10 x + 21) ³ 0

Tu
and b = n 2 n. - x 2 + 10 x - 21 ³ 0
Þ ( x - 7 ) ( x - 3) £ 0 \ 3 £ x £ 7
66. Given equation of ellipse is
4 x 2 + 9 y2 = 1 ...(i) 69. Given, z = sin q - i cos q

ou
æ pö
On differentiating, we get p p i çq - ÷
Þ 8 x + 18 yy¢ = 0 = cos æç q - ö÷ + i sin æç q - ö÷ = e è 2ø
è 2 ø è 2 ø

(Y
-8 x
Þ y¢ = = m, …(ii) Now,
18 y æ np ö
i ç nq - ÷
np ö np ö
Also, 8 x = 9 y [equation of line] zn = e è 2ø
= cos æç nq - æ
÷ + i sin ç nq - ÷
è 2 ø è 2 ø
on
On differentiating, we get
æ np ö
Þ 8 = 9 y¢ 1 i ç - nq÷
np ö np ö
8 Þ =e = cos æç nq -
è 2 ø æ
÷ - i sin ç nq - ÷
Þ y¢ = = m, …(iii) zn è 2 ø è 2 ø
pi

9 np ö
1
zn + n = 2 cos æç nq - æ np - nqö
[Qtangents are parallel to this line] \ ÷ = 2 cos ç ÷
z è 2 ø è2 ø
m

So, by Eqs. (ii) and (iii), we get 1 np


- 8x 8 and zn - n = 2 i sin æç nq - ö.
÷
= z è 2 ø
ha

18 y 9
Þ - x = 2y 70. Given, f[f( x )] = x
Þ x = - 2y Now, f -1( x ) = f( x )
C

On substituting x = - 2 y in Eq. (i), we get i.e. f( x ) is bijective.


4 ( - 2 y)2 + 9 y 2 = 1 Hence, f( x ) has to be one-one and onto.
dy

Þ 16 y 2 + 9 y 2 = 1 Þ 25 y 2 = 1
71. We know,
1 2
Þ \ x=m y=± P ( A È B) = P( A ) + P( B) - P ( A Ç B)
u

5 5
Þ P ( A ) + P ( B) = P ( A È B) + P ( A Ç B) …(i)
æ 2 1ö æ 2 1ö 3
So, required points are ç , - ÷ and ç - , ÷.
St

è5 5ø è 5 5ø Given, £ P( A È B) £ 1
4
3000 æ 2016 ö 1 3
67. ò ç å f(t - r ¢ )f(t - 2016) ÷ dt and £ P( A Ç B) £
-3000 è ø 8 8
r ¢= 2014
7 11
3000 So, £ P( A È B) + P( A Ç B) £
= ò-3000 [f(t - 2014)f(t - 2016) + f(t - 2015)f(t - 2016) 8 8
7 11
+ f(t - 2016)f(t - 2016)]dt \ P ( A ) + P( B) ³ and P( A ) + P( B) £
3000 8 8
= ò-3000 f(t - 2016)[f(t - 2014) + f(t - 2015)
72. Q a, b and c are respectively the AM, GM and HM of first
+ f(t - 2016)]dt and (2 n - 1)th terms.
and also, we know
2016 2017 3000
= ò-3000 0 dt + ò2016 1× (0 + 0 + 1)dt + ò2017 0 dt AM ³ GM ³ HM and AM × HM = GM2

= 0 + 1 + 0 = 1, which is a real number. \ a ³ b ³ c and a × c = b 2


WB JEE (Engg.) Solved Paper 2016 39
73. Let ( h, k ) be the point on line x + y + 1 = 0. From Eqs. (i) and (ii), we get
So, h + k + 1= 0 x2
Þ = 2x + 1
h = - 1- k …(i) a
Þ x 2 = 2 ax + a
Given, distance between ( h, k ) and line 3 x + 4 y + 2 = 0
1
is . Þ x 2 - 2 ax - a = 0 …(iii)
5 2
2a ± 4a + 4a
\ x=
1 ( 3 h) + ( 4 k ) + 2 2
\ =
5 32 + 42 a2 + a )

)
2( a ±
= = a ± a2 + a

be
2
1 3 ( -1 - k ) + 4k + 2
Þ = Points of intersection of the parabola and the line are
5 25
(a + a 2 + a, 2( a + a 2 + a ) + 1)

Tu
1 -3 - 3k + 4k + 2 ö
Þ = ± æç ÷
5 è 5 ø and (a - a + a, 2( a - a 2 + a + 1)
2

1 k - 1ö Q Distance between the points = 40


= ± æç

ou
Þ ÷
5 è 5 ø \ 40 = (2 a + a ) + ( 4 a 2 + a )2
2 2

Þ ± ( k - 1) = 1 Þ 40 = 4( a 2 + a ) + 16 ( a 2 + a )

(Y 2 = a 2 + a Þ a = 1, - 2
Þ k = 0 and 2 Þ
\ h = - 1, -. 3
75. Given,
Hence, the required points are ( -1, 0) and (-3, 2 ).
f ¢ ( x ) = ( x - 1)2 ( 4 - x )
on
74. Given, equation of the parabola The sign scheme of f ¢( x )
x 2 = ay
+ + –
pi

x2 1 4
i.e. y= …(i)
a Clearly, f( x ) is increasing, for x Î ( -¥, 4) and
m

and the equation of line decreasing, for n Î ( 4, ¥).


y - 2x = 1 Since, f ¢ ( x ) = 0 at x = 4.
ha

i.e. y = 2x + 1 …(ii) So, x = 4 is a critical point.


C
u dy
St
Solved Paper 2015
W B JEE
(Engineering Entrance Exam)

)
be
Tu
Physics
1. Two particles of mass m 1 and m 2 ; approach 5. A simple pendulum of length L swings in a

ou
each other due to their mutual gravitational vertical plane. The tension of the string
attraction only. Then, when it makes an angle q with the vertical

(Y
(a) accelerations of both the particles are equal and the bob of mass m moves with a speed v
(b) acceleration of the particle of mass m1 is is (g is the gravitational acceleration)
proportional to m1 (a) mv 2 / L (b) mg cos q + mv 2 / L
on
(c) acceleration of the particle of mass m1 is (c) mg cos q - mv 2 / L (d) mg cos q
proportional to m2
(d) acceleration of the particle of mass m1 is inversely 6. The length of a metal wire is L 1 when the
pi

proportional to m1 tension is T1 and L2 when the tension is T2 .


2. Three bodies of the same material and The unstretched length of wire is
m

having masses m , m and 3m are at L1 + L2


(a) (b) L1L2
temperatures 40° C, 50° C and 60° C, 2
ha

respectively. If the bodies are brought in T L - TL T2 L1 + T1L2


(c) 2 1 1 2 (d)
thermal contact, the final temperature T2 - T1 T2 + T1
will be
7. The line AA¢ is on charged infinite
C

A
(a) 45°C (b) 54°C conducting plane which is
(c) 52°C (d) 48°C
perpendicular to the plane of the θ B
dy

3. A satellite has kinetic energy K, potential paper. The plane has a surface
energy V and total energy E. Which of the density of charge s and B is ball of A′
following statements is true? mass m with a like charge of
u

(a) K = - V /2 (b) K = V /2 magnitude q. B is connected by string from a


St

(c) E = K /2 (d) E = - K /2 point on the line AA¢. The tangent of angle (q )


formed between the line AA¢ and the string is
4. An object is located 4 m from the first of two qs qs qs qs
thin converging lenses of focal lengths 2 m (a) (b) (c) (d)
2 Î0 mg 4pe0 mg 2 pe0 mg e0 mg
and 1 m, respectively. The lenses are
separated by 3 m. The final image formed by 8. The current I is in the circuit shown is
the second lens is located from the source at –
a distance of 2V +

Source 2Ω 2Ω 2Ω
+ +
4m 3m 2V – I 2V –

(a) 8.0 m (b) 5.5 m (a) 1.33 A (b) zero (c) 2.00 A (d) 1.00 A
(c) 6.0 m (d) 6.5 m
2 WB JEE (Engineering) Solved Paper 2015

9. A hollow sphere of external radius R and 15. A straight conductor 0.1 m long moves in a
thickness t ( << R) is made of a metal of uniform magnetic field 0.1T. The velocity of
density r. The sphere will float in water, if the conductor is 15 m/s and is directed
(a) t £
R
(b) t £
R
(c) t £
R
(d) t ³
R perpendicular to the field. The emf induced
r 3r 2r 3r between the two ends of the conductor is
(a) 0.10 V (b) 0.15 V (c) 1.50 V (d) 15.00 V
10. A metal wire of circular cross-section has a
resistance R1. The wire is now stretched 16. A ray of light is incident at an angle i on a
without breaking, so that its length is doubled glass slab of refractive index m. The angle

)
and the density is assumed to remain the between reflected and refracted light is 90°.

be
same. If the resistance of the wire now Then, the relationship between i and m is
becomes R2 , then R2 : R1 is æ 1ö
(a) i = tan- 1 ç ÷ (b) tan i = m

Tu
(a) 1 : 1 (b) 1 : 2 (c) 4 : 1 (d) 1 : 4 èm ø
(c) sin i = m (d) cos i = m
11. Assume that each diode as shown in the
figure has a forward bias resistance of 50 W 17. Two particles A and B are moving as shown

ou
and an infinite reverse bias resistance. The in the figure.
current through the resistance 150 W is 6.5 kg 2.2 m/s
50 Ω A

(Y
3.6 m/s
1.5 m
B
100 Ω O 2.8 m 3.1 kg
on
10 V Their total angular momentum about the
+ – 150 Ω
point O is
(a) 9.8 kg m2 / s
pi

(b) zero
(a) 0.66 A (b) 0.05 A (c) zero (d) 0.04 A
(c) 52.7 kg m2 /s (d) 37.9 kg m2 /s
m

12. The rms speed of oxygen is v at a particular


temperature. If the temperature is doubled 18. A 20 cm long capillary tube is dipped
and oxygen molecules dissociate into vertically in water and the liquid rises upto
ha

oxygen atoms, the rms speed becomes 10 cm. If the entire system is kept is a freely
(a) v (b) 2v falling platform, the length of the water
column in the tube will be
C

(c) 2v (d) 4v
(a) 5 cm (b) 10 cm (c) 15 cm (d) 20 cm
13. Two particles, A and B, having equal
19. A train is moving with a uniform speed of
dy

charges, after being accelerated through the


same potential difference enter into a region 33 m/s and an observer is approaching the
of uniform magnetic field and the particles train with the same speed. If the train blows
u

describe circular paths of radii R1 and R2 , a whistle of frequency 1000 Hz and the
respectively. The ratio of the masses of A and velocity of sound is 333 m/s, then the
St

B is apparent frequency of the sound that the


(a) R1 / R 2 (b) R1 / R 2 observer hears is
(c) (R1 / R 2 )2 (d) (R 2 / R1 )2 (a) 1220 Hz (b) 1099 Hz (c) 1110 Hz (d) 1200 Hz

14. A large number of particles are placed 20. A photon of wavelength 300 nm interacts
around the origin, each at a distance R from with a stationary hydrogen atom in ground
the origin. The distance of the center of mass state. During the interaction, whole energy of
of the system from the origin is the photon is transferred to the electron of
the atom. State which possibility is correct.
(a) equal to R
(b) less than equal to R
(Consider, Plank constant = 4 ´ 10 -15 eVs ,
(c) greater then R velocity of light = 3 ´ 108 m /s, ionisation
(d) greater than equal to R energy of hydrogen = 13.6 eV)
WB JEE (Engineering) Solved Paper 2015 3

(a) Electron will be knocked out of the atom


(b) Electron will go to any excited state of the atom Knot
(c) Electron will go only to first excited state of the θ
B
atom
(d) Electron will keep orbiting in the ground state of
the atom
A
21. Particle A moves along X-axis with a uniform W tan q
velocity of magnitude 10 m/s. Particle B (a) (b) mW tan q
m
moves with uniform velocity 20 m/s along a

)
(c) mW 1 + tan2 q

be
direction making an angle of 60° with the (d) mW sin q
positive direction of X-axis as shown in the
25. The inputs to the digital circuit are as shown
figure. The relative velocity of B with respect
below. The output Y is

Tu
to that of A is
A
s
m/

B
20

ou
Y
B
C

60°
A 10 m/s
X (Y (a) A + B + C
(b) ( A + B) C
(c) A + B + C
on
(d) A + B + C
(a) 10 m/s along X-axis
(b) 10 3 m/s along Y-axis (perpendicular to X-axis) 26. Two particles A and B having different
(c) 10 5 m/s along the bisection of the velocities of masses are projected from a tower with same
pi

A and B speed. A is projected vertically upward and


(d) 30 m/s along negative X-axis B vertically downward. On reaching the
m

ground
22. When light is refracted from a surface, which (a) velocity of A is greater than that of B
of its following physical parameters does not
ha

(b) velocity of B is greater than that of A


change? (c) both A and B attain the same velocity
(a) Velocity (d) the particle with the larger mass attains higher
C

(b) Amplitude velocity


(c) Frequency
(d) Wavelength 27. The work function of metals is in the range of
dy

2 eV to 5 eV. Find which of the following


23. A solid maintained at t°1 C is kept in an wavelength of light cannot be used for
evacuated chamber at temperature t°2 C photoelectric effect? (Consider, Plank
u

( t2 >> t 1 ). The rate of heat absorbed by the constant = 4 ´ 10 - 15 eVs, velocity of light
body is proportional to
St

= 3 ´ 108 m /s)
(a) t 24 - t 14 (a) 510 nm (b) 650 nm
(b) (t 24 + 273) - (t 14 + 273) (c) 400 nm (d) 570 nm
(c) t 2 - t 1 28. A thin plastic sheet of refractive index 1.6 is
(d) t 22 - t 12 used to cover one of the slits of a double slit
arrangement. The central point on the screen
24. Block B lying on a table weighs W. The
is now occupied by what would have been the
coefficient of static friction between the
7th bright fringe before the plastic was used. If
block and the table is m. Assume that the cord
the wavelength of light is 600 nm, what is the
between B and the knot is horizontal. The
thickness (in mm) of the plastic sheet?
maximum weight of the block A for which
(a) 7 (b) 4
the system will be stationary is
(c) 8 (d) 6
4 WB JEE (Engineering) Solved Paper 2015

29. The length of an open organ pipe is twice the 33. In the circuit shown below, the switch is
length of another closed organ pipe. The kept in position a for a long time and is then
fundamental frequency of the open pipe is thrown to position b. The amplitude of the
100 Hz. The frequency of the third harmonic resulting oscillating current is given by
of the closed pipe is R E
(a) 100 Hz (b) 200 Hz
(c) 300 Hz (d) 150 Hz a
C Switch
30. A 5 mF capacitor is connected in series with a b

)
10 mF capacitor. When a 300 V potential

be
L
difference is applied across this
combination, the total energy stored in the (a) E L /C (b) E / R (c) infinity (d) E C / L
capacitors is

Tu
(a) 15 J (b) 1.5 J 34. A charge q is placed at one corner of a cube.
(c) 0.15 J (d) 0.10 J The electric flux through any of the three
faces adjacent to the charge is zero. The flux

ou
31. A cylinder of height h is filled with water through any one of the other three faces is
and is kept on a block of height h /2. The (a) q /3 Î0 (b) q /6 Î0 (c) q /12 Î0 (d) q /24 Î0
level of water in the cylinder is kept
constant. Four holes numbered 1, 2, 3 and 4
are at the side of the cylinder and at heights
0, h / 4, h /2 and 3 h / 4, respectively. When all
(Y
35. Two cells A and B of emf 2V and 1.5 V
respectively, are connected as shown in
figure through an external resistance 10 W.
The internal resistance of each cell is 5W.
on
four holes are opened together, the hole from
which water will reach farthest distance on The potential difference EA and EB across
the plane PQ is the hole number. the terminals of the cells A and B
pi

respectively are
4
A 2V, 5Ω
m

h 3
2
1
ha

10 Ω
h/2
P Q
B 1.5V
C

(a) 1 (b) 2 5Ω
(c) 3 (d) 4 (a) EA = 2.0 V, EB = 1.5 V
(b) EA = 2.125 V, EB = 1.375 V
dy

32. The pressure p, volume V and temperature T


(c) EA = 1.875 V, EB = 1.625V
AT - BT2 (d) EA = 1.875 V, EB = 1.375 V
for a certain gas are related by p = ,
V
u

36. Two charges + q and - q are placed at a


where A and B are constants. The work done distance a in a uniform electric field. The
St

by the gas when the temperature changes dipole moment of the combination is
from T1 to T2 while the pressure remains 2qa (cos q i$ + sin q $j ), where, q is the angle
constant, is given by between the direction of the field and the
(a) A (T2 - T1 ) + B (T22 - T12 ) line joining the two charges.

A (T2 - T1 ) B (T22 - T12 ) Which of the following statement(s) is/are


(b) - correct?
V2 - V1 V2 - V1
(a) The torque exerted by the field on the dipole
B 2 vanishes
(c) A (T2 - T1 ) - (T2 - T12 )
2 (b) The net force on the dipole vanishes
A (T2 - T12 ) (c) The torque is independent of the choice of
(d) coordinates
V2 - V1
(d) The net force is independent of a
WB JEE (Engineering) Solved Paper 2015 5

37. Find the right condition(s) for Fraunhoffer 39. A circular disc rolls on a horizontal floor
diffraction due to a single slit. without slipping and the centre of the disc
(a) Source is at infinite distance and the incident moves with a uniform velocity v. Which of
beam has converged at the slit the following values of the velocity at a point
(b) Source is near to the slit and the incident beam is
on the rim of the disc can have?
parallel
(c) Source is at infinity and the incident beam is parallel (a) v
(d) Source is near to the slit and the incident beam (b) -v
has converged at the slit (c) 3v

)
(d) Zero

be
38. The conducting loop in the form of a circle is
placed in a uniform magnetic field with its 40. Consider two particles of different masses. In
plane perpendicular to the direction of the which of the following situations the heavier

Tu
field. An emf will be induced in the loop, if of the two particles will have smaller
(a) it is translated parallel to itself de-Broglie wavelength?
(b) it is rotated about one of its diameters (a) Both have a free fall through the same height

ou
(c) it is rotated about its own axis which is parallel to (b) Both move with the same kinetic energy
the field (c) Both move with the same linear momentum
(d) the loop is deformed from the original shape (d) Both move with the same speed

(Y
Chemistry
on
41. Match the flame colours of the alkaline earth 44. CH3
HBr (1 equiv.)
metal salts in the Bunsen burner. CH2
pi

H2C
A. Calcium p. Brick red
The major product of the above reaction is
m

B. Strontium q. Apple green


C. Barium r. Crimson CH3
ha

CH3
A B C (a)
(a) p r q
C

(b) r p q Br
(c) q r p CH3
(d) p q r
dy

(b) H C
42. Extraction of gold (Au) involves the 3 Br
formation of complex ions X and Y. H 3C
u

Roasting Zn
Gold ore ¾¾¾¾® HO - + X ¾® Y + Au CH2
St

-
CN , H2O, O2 (c)
H 3C
Br
X and Y respectively are
H3 C
(a) Au(CN)-2 and Zn(CN)2-
4
(b) Au(CN)3-
4 and Zn(CN)4
2-
(d)
(c) Au(CN)-3 and Zn(CN)4- H 3C Br
6
(d) Au(CN)-4 and Zn(CN)-3 45. Cl
43. The atomic number of cerium (Ce) is 58. The
NH3
correct electronic configuration of Ce 3 + ion is →
1 1 EtOH
(a) [Xe] 4f (b) [Kr] 4f Br
(c) [Xe] 4f13 (d) [Kr] 4d 1
The product of the above reaction is
6 WB JEE (Engineering) Solved Paper 2015

NH2 Cl 52. The rms velocity of CO gas molecules at 27°C


is approximately 1000 m/s. For N2 molecules
(a) (b) at 600 K, the rms velocity is approximately
NH2 (a) 2000 m/s (b) 1414 m/s
(c) 1000 m/s (d) 1500 m/s
NH2
53. A gas can be liquefied at temperature T and
NH2 NH2 pressure p provided
(a) T = Tc and p < pc (b) T < Tc and p > pc

)
(c) T > Tc and p > pc (d) T > Tc and p < pc

be
(c) (d)
Br OEt 54. The dispersed phase and dispersion medium
of fog respectively are
46. Sulphuryl chloride (SO2Cl2 ) reacts with

Tu
(a) solid, liquid (b) liquid, liquid
white phosphorus ( P4 ) to give (c) liquid, gas (d) gas, liquid
(a) PCl 5 , SO 2 (b) OPCl 3 , SOCl 2
55. The decreasing order of basic character of

ou
(c) PCl 5 , SO 2 , S2Cl 2 (d) OPCl 3 , SO 2 , S2Cl 2
K2O, BaO, CaO and MgO is
47. The number of lone pair of electrons on the (a) K 2O > BaO > CaO > MgO
central atoms of H2O, SnCl2 , PCl3 an XeF2
respectively, are
(a) 2, 1, 1, 3
(c) 3, 1, 1, 2
(b) 2, 2, 1, 3
(d) 2, 1, 2, 3
(Y (b) K 2O > CaO > BaO > MgO
(c) MgO > BaO > CaO > K 2O
(d) MgO > CaO > BaO > K 2O
on
56. In aqueous alkaline solution, two electrons
48. Consider the following salts : NaCl, HgCl2 , reduction of HO2- gives
Hg 2Cl2 , CuCl2 , CuCl and AgCl. Identify the
(a) HO -
pi

(b) H2O
correct set of insoluble salts in water. (c) O 2 (d) O -2
(a) Hg 2Cl 2 , CuCl, AgCl (b) HgCl 2 , CuCl, AgCl
m

(c) Hg 2Cl 2 , CuCl 2 , AgCl (d) Hg 2Cl 2 , CuCl, NaCl 57. Cold ferrous sulphate solution on absorption
of NO develops brown colour due to the
49. In the following compound, the number of
ha

formation of
sp-hybridised carbon is
(a) paramagnetic [Fe(H2O)5 (NO)] SO 4
CH2 == C == CH ¾ CH ¾ C ºº CH (b) diamagnetic [Fe(H2O)5 (N3 )] SO 4
C

½ (c) paramagnetic [Fe(H2O)5 (NO 3 )][SO 4 ]2


CN (d) diamagnetic [Fe(H2O)4 (SO 4 )]NO 3
dy

(a) 2 (b) 3 (c) 4 (d) 5


58. Amongst Be, B, Mg and Al the second
50. For the reaction, A + 2 B ¾® C ; the reaction ionisation potential is maximum for
rate is doubled, if the concentration of A is
u

(a) B (b) Be
doubled. The rate is increased by four times
St

(c) Mg (d) Al
when concentrations of both A and B are
increased by four times. The order of the 59. In a mixture, two enantiomers are found to
reaction is be present in 85% and 15% respectively. The
enantiomeric excess (ee ) is
(a) 3 (b) 0 (c) 1 (d) 2
(a) 85% (b) 15%
51. At a certain temperature, the value of the (c) 70% (d) 60%
slope of the plot of osmotic pressure ( p )
60. 1,4-dimethylbenzene on heating with
against concentration (C in mol L-1 ) of a anhydrous AlCl3 and HCl produces
certain polymer solution is 291R. The
(a) 1, 2-dimethylbenzene
temperature at which osmotic pressure is (b) 1, 3-dimethylbenzene
measured is (R is gas constant) (c) 1, 2, 3-trimethylbenzene
(a) 271°C (b) 18°C (c) 564 K (d) 18 K (d) ethylbenzene
WB JEE (Engineering) Solved Paper 2015 7

61. CHO 68. Best reagent for nuclear iodination of


OH–
→ ? aromatic compounds is
CHO (a) KI / CH3COCH3 (b) I2 / CH3CN
(c) KI / CH3COOH (d) I2 / HNO 3
The product of the above reaction is
69. In the Lassaigne’s test for the detection of
CH2OH CH2O–
nitrogen in an organic compound, the
(a) (b) appearance of blue coloured compound is
COOH COOH due to

)
(a) ferric ferricyanide

be
(b) ferrous ferricyanide
CH2OH CH2O–
(c) ferric ferrocyanide
(c) (d) (d) ferrous ferrocyanide

Tu
COO– COO–
70. In the following reaction,
62. Suppose the mass of a single Ag atom is ‘m’. Ether, H3O +

ou
Ag metal crystallises in fcc lattice with unit R MgBr + HC(OEt)3 ¾¾¾® P
cell of length ‘a’. The density of Ag metal in
terms of ‘a’ and ‘m’ is The product P is
(a)
4m
a3
(b)

63. For the reactions,


2m
a3
(c)
m
a3
(d)
m
4a3
(Y (a) RCHO
(b) R 2CHOEt
(c) R 3CH
on
(d) RCH(OEt)2
2SO2 (g ) + O2 (g ) 2SO3 (g ) at 300 K, the
r
value of DG° is - 690.9 R. The equilibrium 71. Addition of sodium thiosulphate solution to
constant value for the reaction at that a solution of silver nitrate gives ‘X’ as white
pi

temperature is (R is gas constant) precipitate, insoluble in water but soluble in


excess thiosulphate solution to give ‘Y’. On
m

(a) 10 atm-1 (b) 10 atm


(c) 10 (d) 1
boiling in water, 'Y’ gives ‘Z’. ‘X’, ‘Y’ and
‘Z’are respectively
ha

64. At a particular temperature, the ratio of (a) Ag 2S2O 3 , Na 3 [Ag(S2O 3 )2 ], Ag 2S


equivalent conductance to specific
(b) Ag 2SO 4 , Na[Ag(S2O 3 )2 ], Ag 2S2
conductance of a 0.01 N NaCl solution is
C

(c) Ag 2S2O 3 , Na 5 [Ag(S2O 3 )3 ], AgS


(a) 105 cm3 (b) 103 cm3
(c) 10 cm3 (d) 105 cm2 (d) Ag 2SO 3 , Na 3 [Ag(S2O 3 )2 ], Ag 2O
dy

65. The units of surface tension and viscosity of 72. At temperature of 298 K, the emf of the
liquids respectively are following electrochemical cell
u

(a) kg m-1s -1, Nm-1 (b) kg s -2 , kg m-1s -1 Ag (s )|Ag + (0.1 M)||Zn2 + (0.1 M)|Zn (s )
(c) Nm-1, kg m-1s -2 (d) kg s -1,kg m-2 s -1
St

will be (given E °cell = - 1.562 V)


66. The ratio of volumes of CH3COOH 0.1 (N) to (a) -1.532 V (b) -1.503 V
CH3COONa 0.1 (N) required to prepare a (c) 1.532 V (d) – 3.06 V
buffer solution of pH 5.74 is
73. For the reaction, X2 Y4 ( l ) ¾® 2 XY2 (g ) at
(given pKa of CH3COOH is 4.74)
300 K, the values of DU and DS are 2 kcal and
(a) 10 : 1 (b) 5 : 1 (c) 1 : 5 (d) 1 : 10
20 cal K -1 respectively. The value of DG for
67. The reaction of methyltrichloroacetate the reaction is
(Cl3CCO2 Me) with sodium methoxide (a) - 3400 cal
(NaOMe) generates (b) 3400 cal
(a) carbocation (b) carbene (c) -2800 cal
(c) carbanion (d) carbon radical (d) 2000 cal
8 WB JEE (Engineering) Solved Paper 2015

74. The total number of aromatic species 78. The increase in rate constant of a chemical
generated in the following reaction is reaction with increasing temperature is (are)
due to the fact(s) that
(i) Cl + SbCl5 (a) the number of collisions among the reactant
molecules increases with increasing temperature
(b) the activation energy of the reaction decreases
(ii) THF
+ Sodium metal with increasing temperature
(c) the concentration of the reactant molecules
increases with increasing temperature

)
Br H
(d) the number of reactant molecules acquiring the

be
(iii) + H2O activation energy increases with increasing
temperature

Tu
79. Within the list shown below, the correct pair
H2N
of structures of alanine in pH ranges 2-4 and
9-11 is
(iv)

ou
HNO2
I. H3 N+ ¾ CH(CH3 )CO2 H
II. H2 N ¾ CH(CH3 )CO2-

(Y
(a) zero (b) 2
(c) 3 (d) 4
III. H3 N+ ¾ CH(CH3 )CO2-
IV. H2 N ¾ CH(CH3 )CO2 H
75. Roasted copper pyrite on smelting with sand (a) I and II
on
produces (b) I and III
(a) FeSiO 3 as fusible slag and Cu2S as matte (c) II and III
(b) CaSiO 3 as infusible slag and Cu2O as matte (c) III and IV
pi

(c) Ca 3 (PO 4 )2 as fusible slag and Cu2S as matte


(d) Fe 3 (PO 4 )2 as infusible slag and Cu2S as matte 80. Identify the correct method for the synthesis
m

of the compound shown below from the


76. Ionisation potential values of noble gases following alternatives.
decrease down the group with increase in
ha

atomic size. Xenon forms binary fluorides by


CH3
the direct reaction of elements. Identify the
correct statement(s) from below.
C

O2N
(a) Only the heavier noble gases form such
compounds
dy

CH3CH2CH2CH2Cl HNO3
(b) It happens because the noble gases have higher (a)
ionisation energies AlCl3 H2SO4

(c) It happens because the compounds are formed


u

with electronegative ligands CH3CH2CH2COCl Zn/Hg HNO3


(b)
St

(d) Octet of electrons provide the stable AlCl3 HCl/heat H2SO4


arrangements
77. Optical isomerism is exhibited by (c)
CH3CH2CH2COCl HNO3 Zn/Hg

(ox = oxalate anion; en = ethylenediamine). AlCl3 H2SO4 HCl/heat

(a) cis-[CrCl 2 (ox)2 ]3-


CH3CH2CH2COCl KMnO4 HNO3
(b) [Co(en)3 ]3+ (d)
AlCl3 OH– H2SO4
(c) trans-[CrCl 2 (ox)2 ]3-
(d) [Co(ox) (en)2 ]+
Mathematics
81. Let a, b, c and d be any four real numbers. 89. The least value of
Then, a n + b n = c n + d n holds for any 2 x2 + y2 + 2 xy + 2 x - 3 y + 8 for real
natural number n, if numbers x and y, is
(a) a + b = c + d (a) 2 (b) 8 (c) 3 (d) -1 / 2
(b) a - b = c - d
(c) a + b = c + d , a2 + b 2 = c 2 + d 2
90. Let f : [ -2, 2 ] ® R be a continuous function

)
such that f ( x ) assumes only irrational values.

be
(d) a - b = c - d , a2 - b 2 = c 2 - d 2
If f( 2 ) = 2 , then
82. If a and b are the roots of x2 - px + 1 = 0 and
g is a root of x2 + px + 1 = 0, then (a) f(0) = 0 (b) f( 2 - 1) = 2 - 1

Tu
(a + g )(b + g ) is (c) f( 2 - 1) = 2 + 1 (d) f( 2 - 1) = 2
(a) 0 (b) 1 (c) -1 (d) p 2
91. The minimum value of cos q + sin q +

ou
83. The number of irrational terms in the sin 2 q
binomial expansion of (3 1/ 5 + 7 1/ 3 )100 is for q Î(0, p / 2 ), is
(a) 90 (b) 88 (c) 93 (d) 95 (a) 2 + 2 (b) 2 (c) 1 + 2 (d) 2 2

84. The quadratic expression


2
(2 x + 1 ) - px + q =/ 0 for any real x, if (Y
92. The value of lim
x ®2
x
ò2
3t 2

(x - 2)
dt is
on
(a) p2 - 16 p - 8 q < 0 (b) p2 - 8 p + 16 q < 0 (a) 10 (b) 12 (c) 8 (d) 16
2 2
(c) p - 8 p - 16 q < 0 (d) p - 16 p + 8 q < 0 2x x kx
93. If cot + tan = cosec , then the value
pi

3 3 3
85. In a certain town, 60% of the families own a
of k is
car, 30% own a house and 20% own both car
m

and house. If a family is randomly chosen, (a) 1 (b) 2 (c) 3 (d) - 1


then what is the probability that this family æ p 3p ö
94. If q Î ç , ÷, then the value of
ha

owns a car or a house but not both? è2 2 ø


(a) 0.5 (b) 0.7 (c) 0.1 (d) 0.9
C

86. The letters of the word ‘COCHIN’ are æp qö


4 cos 4 q + sin2 2q + 4 cot q cos 2 ç - ÷ is
permuted and all the permutations are è4 2ø
arranged in alphabetical order as in English
dy

(a) -2cot q (b) 2cot q


dictionary. The number of words that appear (c) 2cos q (d) 2sinq
before the word ‘COCHIN’, is
95. The number of real solutions of the equation
u

(a) 360 (b) 192 (c) 96 (d) 48


(sin x - x ) (cos x - x2 ) = 0 is
St

87. Let f : R ® R be a continuous function which


x
(a) 1 (b) 2 (c) 3 (d) 4
satisfies f ( x ) = ò0 f ( t ) dt. Then, the value of 64 64
æ1 + 3i ö æ1 - 3i ö
f (log e 5 ) is 96. The value of ç ÷ +ç ÷ is
è1 - 3i ø è1 + 3i ø
(a) 0 (b) 2 (c) 5 (d) 3
(a) 0 (b) -1
x2 - x + 4 (c) 1 (d) i
88. Let f : R ® R be defined as f ( x ) = .
x2 + x + 4 97. Find the maximum value of | z| when
Then, range of the function f ( x ) is 3
3 5 3 5 z- = 2, where z being a complex number.
(a) éê , ùú (b) æç , ö÷ z
ë 5 3û è 5 3ø
3 5 5 3 (a) 1 + 3 (b) 3
(c) æç -¥, ö÷ È æç , ¥ö÷ (d) éê - , - ùú
è 5ø è 3 ø ë 3 5û (c) 1 + 2 (d) 1
10 WB JEE (Engineering) Solved Paper 2015

98. Given that, x is a real number satisfying 105. Which of the following is not always true?
5 x2 - 26 x + 5 (a)|a + b|2 = |a|2 + |b|2 , if a and b are
< 0, then
3 x2 - 10 x + 3 perpendicular to each other
1 1 (b)|a + lb| ³ |a| for all l ÎR, if a and b are
(a) x < (b) < x<3 perpendicular to each other
5 5
1 1 (c)|a + b|2 + |a - b|2 = 2(|a|2 + |b|2 )
(c) x > 5 (d) < x < or 3 < x < 5
5 3 (d)|a + lb| ³ |a| for all l ÎR, if a is parallel to b

99. The value of l such that the following system 106. If the four points with position vectors

)
of equations has no solution, is -2 i + j + k , i + j + k , j - k and lj + k are

be
2 x - y - 2 z = 2; x - 2 y + z = - 4; coplanar, then l is equal to
x + y + lz = 4 (a) 1 (b) 2 (c) -1 (d) 0

Tu
(a) 3 (b) 1 (c) 0 (d) -3 107. The least positive value of t, so that the lines
1 x x = t + a, y + 16 = 0 and y = ax are
concurrent, is

ou
100. If f(x) = 2x x( x - 1 )
(a) 2 (b) 4 (c) 16 (d) 8
3 x( x - 1 ) x ( x - 1 )( x - 2 )
x +1 108. If in a DABC , a2 cos 2 A - b2 - c2 = 0, then
( x + 1 )x
( x + 1 )x( x - 1 )
.
(Y (a)
p
4
< A<

(c) A =
p
p
2
(b)
p
2
< A<p

(d) A <
p
on
2 4
Then, f(100 ) is equal to
é 3p ù
(a) 0 (b) 1 (c) 100 (d) 10 109. {x Î R : cos x ³ sin x} I ê0, is equal to
2 úû
pi

2 2 2 ë
æ 1ö æ 1ö æ 1 ö
101. Let x n = ç1 - ÷ ç1 - ÷ ç1 - ÷ ... p 3p 3p ù
(a) éê 0, ùú È éê ,
p p 3p
(b) éê 0, ùú È éê , ùú
m

è 3ø è 6ø è 10 ø ë 4 û ë 4 2 úû ë 4 û ë2 2 û
2 p 5p 3p ù 3p ù
æ ö (c) ê 0, ùú È éê
é , (d) éê 0,
ha

ç 1 ÷ ë 4 û ë 4 2 úû ë 2 úû
ç1 - ÷ , n ³ 2. Then, the value of
ç n( n + 1 ) ÷ æ x2 x3 x4 ö p
è 2 ø 110. If sin -1 ç x - + - + . . . ÷ = , where
C

è 2 4 8 ø 6
lim x n is
n ®¥ | x| < 2, then the value of x is
dy

(a) 1/3 (b) 1/9 (c) 1/81 (d) 0 2 3 2 3


(a) (b) (c) - (d) -
102. The variance of first 20 natural numbers is 3 2 3 2
111. The area of the region bounded by the curve
u

(a) 133/4 (b) 279/12


(c) 133/2 (d) 399/4 y = x3 , its tangent at (1,1) and X-axis is
St

103. A fair coin is tossed a fixed number of times. (a)


1
sq unit (b)
1
sq unit
If the probability of getting exactly 3 heads 12 6
equals the probability of getting exactly 2 2
(c) sq unit (d) sq unit
5 heads, then the probability of getting 17 15
exactly one head is 112. If log 0.2 ( x - 1 ) > log 0.04 ( x + 5 ), then
(a) 1/64 (b) 1/32
(c) 1/16 (d) 1/8 (a) - 1 < x < 4 (b) 2 < x < 3
(c) 1 < x < 4 (d) 1 < x < 3
104. If the letters of the word ‘PROBABILITY’ are
written down at random in a row, then 113. The number of real roots of equation
probability that two B’s are together, is log e x + ex = 0 is
2 10 3 6 (a) 0 (b) 1
(a) (b) (c) (d) (c) 2 (d) 3
11 11 11 11
WB JEE (Engineering) Solved Paper 2015 11

114. The number of distinct real roots of 1 + 2 + ... + n - 1


sin x cos x cos x 120. lim is equal to
n ®¥ n n
cos x sin x cos x = 0 in the interval 1 1 2
(a) (b) (c) (d) 0
cos x cos x sin x 2 3 3
p p
- £ x £ is axe x - b log (1 + x )
4 4 121. If lim = 3, then the
x ®0 x2
(a) 0 (b) 2 (c) 1 (d) > 2
values of a and b are, respectively

)
115. Let x 1, x2 , . . . , x 15 be 15 distinct numbers (a) 2, 2 (b) 1, 2 (c) 2, 1 (d) 2, 0

be
chosen from 1,2,3, …,15. 122. If the vertex of the conic y2 - 4 y = 4 x - 4a
Then, the value of ( x 1 - 1 )( x2 - 1 )( x3 - 1 ) …
always lies between the straight lines
( x 15 - 1 ) is

Tu
x + y = 3 and 2 x + 2 y - 1 = 0, then
(a) always £ 0 (b) 0 1
(c) always even (d) always odd (a) 2 < a < 4 (b) - < a<2
2

ou
116. Let [ x ] denotes the greatest integer less than (c) 0 < a < 2
1
(d) - < a <
3
or equal to x. Then, the value of a for which 2 2
ì sin[ - x2 ] 123. Number of intersecting points of the conics

(Y
ï ,x ¹0
the function f ( x ) = í [ - x2 ] is 4 x2 + 9 y2 = 1 and 4 x2 + y2 = 4 is
ï a, x = 0
î (a) 1 (b) 2
continuous at x = 0, is
on
(c) 3 (d) 0
(a) a = 0 (b) a = sin(-1)
(c) a = sin(1) (d) a = 1
124. The value of l for which the straight line
x - l y -1 z-3
may lie on the plane
pi

= =
117. For all real values of a0 , a 1, a2 , a3 satisfying 3 2+l -1
a 1 a2 a3 x - 2 y = 0, is
m

a0 + + + = 0, the equation
2 3 4 (a) 2 (b) 0
a0 + a 1x + a2 x2 + a3 x3 = 0 has a real root in 1
ha

(c) - (d) there is no such l


the interval 2
(a) [0, 1] (b) [-1, 0] (c) [1, 2 ] (d) [-2, - 1] 125. Area of the region bounded by y =| x| and
C

118. Let f : R ® R be defined as y = -| x| + 2 is


ì0, x is irrational (a) 4 sq units
dy

f(x) = í (b) 3 sq units


îsin| x|, x is rational (c) 2 sq units
Then, which of the following is true? (d) 1 sq unit
u

(a) f is discontinuous for all x


126. Let d ( n ) denotes the number of divisors of n
St

(b) f is continuous for all x


(c) f is discontinuous at x = kp, where k is an integer including 1 and itself. Then, d (225 ),
(d) f is continuous at x = kp, where k is an integer d (1125 ) and d(640 ) are
(a) in AP
119. A particle starts moving from rest from a fixed (b) in HP
point in a fixed direction. The distance s from (c) in GP
the fixed point at a time t is given by (d) consecutive integers
s = t2 + at - b + 17, where a and b are real
127. The trigonometric equation
numbers. If the particle comes to rest after 5 s
sin -1 x = 2 sin -1 2a has a real solution, if
at a distance of s = 25 units from the fixed
point, then values of a and b are, respectively 1 1 1
(a)|a| > (b) < |a|<
(a) 10, - 33 (b) -10, - 33 2 2 2 2
1 1
(c)|a| > (d)|a| £
(c) -8, 33 (d) -10, 33 2 2 2 2
12 WB JEE (Engineering) Solved Paper 2015

128. If (2 + i ) and ( 5 - 2 i ) are the roots of the 136. If the point (2 cos q, 2 sin q ) for 0 Î(0, 2 p ) lies
equation ( x2 + ax + b ) ( x2 + cx + d ) = 0, in the region between the lines x + y = 2 and
where a, b, c and d are real constants, then x - y = 2 containing the origin, then q lies in
product of all the roots of the equation is p 3p
(a) æç q, ö÷ È æç , 2 p ö÷ (b) [0, p ]
è 2ø è 2 ø
(a) 40 (b) 9 5 (c) 45 (d) 35
æ p 3p ö p p
(c) ç , ÷ (d) é , ù
129. If f : [0, p / 2 ) ® R is defined as è2 2 ø êë 4 2 úû

1 tan q 1 137. Number of points having distance 5 from

)
be
f(q ) = - tan q 1 tan q . Then, the the straight line x - 2 y + 1 = 0 and a distance
-1 - tan q 1 13 from the line 2 x + 3 y - 1 = 0 is
range of f is (a) 1 (b) 2

Tu
(c) 4 (d) 5
(a) (2, ¥) (b) (-¥, - 2 ] (c) [2, ¥) (d) (-¥, 2 ]
130. If A and B are two matrices such that AB = B 138. Let P( x ) be a polynomial, which when

ou
divided by ( x - 3 ) and ( x - 5 ) leaves
and BA = A, then A2 + B2 equals
remainders 10 and 6, respectively. If the
(a) 2AB (b) 2BA (c) A + B (d) AB polynomial is divided by ( x - 3 ) ( x - 5 ), then
131. If w is an imaginary cube root of unity, then
the value
1 + w w2 - w
of the determinant (Y the remainder is
(a) - 2 x + 16
(c) 2 x - 16
(b) 16
(d) 60
on
1 + w2 w - w2 139. The integrating factor of the differential
dy
w + w2 w - w2 equation + (3 x2 tan -1 y - x 3 )(1 + y2 ) = 0
pi

dx
(a) -2w (b) -3w2 (c) -1 (d) 0 is
m

2 3
132. Let f ( x ) denotes the fractional part of a real (a) e x (b) e x
2 3
3 (c) e 3 x (d) e 3 x
number x. Then, the value of ò f ( x2 ) dx is
ha

0
140. If y = e - x cos 2 x , then which of the
(a) 2 3 - 2 - 1 (b) 0
following differential equation is satisfied?
(c) 2 - 3 + 1 (d) 3 - 2 + 1
C

d2ydy d2y dy
(a) + 5y = 0
+2 (b) + 5 + 2y = 0
133. Let S = {(a, b, c ) Î N ´ N ´ N : a + b + c = 21, dx 2
dx dx2 dx
a £ b £ c} and d2y d2y
dy

dy dy
(c) 2 - 5 - 2 y = 0 (d) 2 + 2 - 5y = 0
T = {(a, b, c ) Î N ´ N ´ N : a, b, c, are in AP}, dx dx dx dx
where N is the set of all natural numbers.
141. Let f : R ® R be differentiable at x = 0. If
u

Then, the number of elements in the set


S Ç T is f(0 ) = 0 and f ¢ (0 ) = 2, then the value of
St

(a) 6 (b) 7 (c) 13 (d) 14 1


lim [ f ( x ) + f (2 x ) + f (3 x ) + . . . + f (2015 x )]
2 2
x ®0 x
134. Let y = e x and y = e x sin x be two given is
curves. Then, angle between the tangents to (a) 2015
the curves at any point of their intersection is (b) 0
p p (c) 2015 ´ 2016
(a) 0 (b) p (c) (d)
2 4 (d) 2015 ´ 2014
1 4 142. If x and y are digits such that
135. The value of 2 cot -1 - cot - 1 is
2 3 17 ! = 3556 xy428096000, then x + y equals
p 3p p p (a) 15 (b) 6
(a) - (b) (c) (d)
8 2 4 2 (c) 12 (d) 13
WB JEE (Engineering) Solved Paper 2015 13

143. A person goes to office by car or scooter or 149. Let a and b be two distinct roots of
bus or train, probability of which are 1/7, 3/7, a cos q + b sin q = c, where a, b, c are three
2/7 and 1/7, respectively. Probability that he real constants and q Î[0, 2 p ]. Then, a + b is
reaches office late, if he takes car, scooter, also a root of the same equation, if
bus or train is 2/9, 1/9, 4/9, and 1/9, (a) a + b = c (b) b + c = a
respectively. Given that he reached (c) c + a = b (d) c = a
office in time, the probability that he æ1 0 0 ö
travelled by a car, is ç ÷
150. For a matrix A = ç2 1 0 ÷, if U 1, U2 and U3

)
(a) 1/7 (b) 2/7 ç3 2 1 ÷

be
(c) 3/7 (d) 4/7 è ø
( x - 2 ) dx are 3 ´ 1 column matrices satisfying
144. The value of ò is
{( x - 2 )2 ( x + 3 )7 }1 / 3 æ1 ö æ2 ö æ2 ö

Tu
ç ÷ ç ÷ ç ÷
3 æ x -2ö
4/ 3
3 æ x -2ö
3/ 4 AU 1 = ç0 ÷, AU2 = ç3 ÷, AU3 = ç3 ÷ and U is
(a) ç ÷ +C (b) ç ÷ +C ç0 ÷ ç0 ÷ ç1 ÷
20 è x + 3 ø 20 è x + 3 ø è ø è ø è ø

ou
5 æ x -2ö
4/ 3
3 æ x -2ö
5/ 3
3 ´ 3 matrix whose columns are U 1, U2 and
(c) ç ÷ +C (d) ç ÷ +C U3 . Then, sum of the elements of U -1 is
12 è x + 3 ø 20 è x + 3 ø

145. Let f : N ® R be such that f(1 ) = 1


and f (1 ) + 2 f (2 ) + 3 f (3 ) +. . . + nf ( n ) =
n( n + 1 ) f ( n ), for all n Î N, n ³ 2, where N is
(Y (a) 6 (b) 0 (c) 1
151. Let f be any continuously differentiable
(d) 2/3

function on [a, b ] and twice differentiable on


on
(a, b ) such that f (a ) = f ¢ (a ) = 0 and f ( b ) = 0.
the set of natural numbers and R is the set of Then,
real numbers. Then, the value of f(500 ) is (a) f ¢ ¢ (a) = 0
pi

(a) 1000 (b) 500 (b) f ¢( x) = 0 for some x Î(a, b )


(c) 1/500 (d) 1/1000 (c) f ¢ ¢( x) = 0 for some x Î(a, b )
m

(d) f ¢ ¢ ¢( x) = 0 for some x Î(a, b )


146. If 5 distinct balls are placed at random into 5
cells, then the probability that exactly one 152. A relation r on the set of real number R is
ha

cell remains empty, is defined as follows: x ry if and only if xy > 0.


(a) 48/125 (b) 12/125 Then, which of the following is/are true?
(a) r is reflexive and symmetric
C

(c) 8/125 (d) 1/125


(b) r is symmetric but not reflexive
147. A survey of people in a given region showed (c) r is symmetric and transitive
that 20% were smokers. The probability of (d) r is an equivalence relation
dy

death due to lung cancer, given that a person 153. If cos x and sin x are solutions of the
smoked, was 10 times the probability of differential equation
u

death due to lung cancer, given that a person


d2 y dy
did not smoke. If the probability of death due a0 + a1 + a2 y = 0 ,
St

2
to lung cancer in the region is 0.006. What is dx dx
the probability of death due to lung cancer where a0 , a 1 and a2 are real constants, then
given that a person is a smoker? which of the following is/are always true?
(a) A cos x + B sin x is a solution, where A and B are
(a) 1/140 (b) 1/70
real constants
(c) 3/140 (d) 1/10
p
(b) A cos æç x + ö÷ is a solution, where A is a real
148. In a DABC, if ÐC = 90°, r and R are the è 4ø
inradius and circumradius of the DABC constant
respectively, then 2 ( r + R) is equal to (c) A cos x sin x is a solution, where A is a real
constant
(a) b + c
p p
(b) c + a (d) A cos æç x + ö÷ + B sin æç x - ö÷ is a solution, where
è 4 ø è 4ø
(c) a + b
(d) a + b+c A and B are real constants
14 WB JEE (Engineering) Solved Paper 2015

154. Which of the following statements is /are (c) A quadratic equation with irrational coefficients
p has zero or two rational roots
correct for 0 < q < ? (d) A quadratic equation with integer coefficients has
2
zero or two irrational roots
q 3q
(a) (cos q)1/ 2 £ cos (b) (cos q)3 / 4 ³ cos
2 4 158. If the straight line (a - 1 )x - by + 4 = 0 is
5q 7q
(c) cos ³ (cos q)5 / 6 (d) cos £ (cos q)7 / 8 normal to the hyperbola xy = 1, then which
6 8 of the following does not hold?
155. Let 16 x2 - 3 y2 - 32 x - 12 y = 44 represents (a) a > 1, b > 0

)
a hyperbola. Then, (b) a > 1, b < 0

be
(c) a < 1, b < 0
(a) length of the transverse axis is 2 3
(d) a < 1, b > 0
(b) length of each latusrectum is 32 / 3

Tu
(c) eccentricity is 19 / 3 159. Suppose a machine produces metal parts
(d) equation of a directrix is x =
19 that contain some defective parts with
3 probability 0.05. How many parts should be

ou
é 1 ù produced in order that the probability of
156. For the function f ( x ) = ê ú, where [ x ] atleast one part being defective is 1/2 or
ë [x]û more? (Given that, log10 95 = 1.977 and

(Y
denotes the greatest integer less than or log10 2 = 0.3)
equal to x, which of the following statements (a) 11
are true? (b) 12
on
(a) The domain is (-¥, ¥) (c) 15
(b) The range is {0} È {-1} È {1} (d) 14
(c) The domain is (-¥, 0) È [1, ¥)
160. Let f : R ® R be such that f (2 x - 1 ) = f ( x ) for
pi

(d) The range is {0} È {1}


all x Î R. If f is continuous at x = 1 and
157. Which of the following is / are always false? f(1 ) = 1, then
m

(a) A quadratic equation with rational coefficients has (a) f(2 ) = 1


zero or two irrational roots (b) f(2 ) = 2
ha

(b) A quadratic equation with real coefficients has (c) f is continuous only at x = 1
zero or two non-real roots (d) f is continuous at all points
C
u dy
St
Answers
Physics
1. (c) 2. (b) 3. (a) 4. (b) 5. (b) 6. (c) 7. (a) 8. (a) 9. (b) 10. (c)
11. (b) 12. (c) 13. (b) 14. (b) 15. (b) 16. (b) 17. (a) 18. (d) 19. (a) 20. (d)
21. (b) 22. (c) 23. (c) 24. (b) 25. (c) 26. (c) 27. (b) 28. (a) 29. (c) 30. (c)
31. (a) 32. (c) 33. (c) 34. (d) 35. (d) 36. (a) 37. (c) 38. (b) 39. (d) 40. (d)

)
be
Chemistry
41. (a) 42. (a) 43. (a) 44. (b) 45. (c) 46. (a) 47. (a) 48. (a) 49. (c) 50. (c)

Tu
51. (b) 52. (b) 53. (b) 54. (c) 55. (a) 56. (a) 57. (a) 58. (a) 59. (c) 60. (b)
61. (c) 62. (a) 63. (a) 64. (a) 65. (b) 66. (d) 67. (b) 68. (d) 69. (c) 70. (a)
71. (a) 72. (a) 73. (c) 74. (c) 75. (a) 76. (a,c) 77. (a,b,d) 78. (a,d) 79. (a) 80. (b)

ou
Mathematics

(Y
81. (d) 82. (a) 83. (*) 84. (c) 85. (a) 86. (c) 87. (a) 88. (a) 89. (d) 90. (d)
91. (a) 92. (b) 93. (b) 94. (b) 95. (c) 96. (b) 97. (b) 98. (d) 99. (d) 100. (a)
101. (b) 102. (a) 103. (b) 104. (a) 105. (d) 106. (a) 107. (d) 108. (b) 109. (a) 110. (a)
on
111. (a) 112. (c) 113. (b) 114. (c) 115. (b) 116. (c) 117. (a) 118. (d) 119. (b) 120. (c)
121. (a) 122. (b) 123. (d) 124. (d) 125. (c) 126. (c) 127. (d) 128. (c) 129. (c) 130. (c)
131. (b) 132. (c) 133. (d) 134. (a) 135. (d) 136. (c) 137. (c) 138. (a) 139. (b) 140. (a)
pi

141. (c) 142. (a) 143. (a) 144. (a) 145. (d) 146. (a) 147. (c) 148. (c) 149. (d) 150. (b)
151. (b,c) 152. (b,c) 153. (a,b,d) 154. (a,c) 155. (a,b,c) 156. (b, c) 157. (c) 158. (a, c) 159. (c,d) 160. (c)
m

Solutions
ha

Physics
C

1. The gravitational force acting between the two Þ 3 ms (60 - q) = ms ( q - 50 ) + ms ( q - 40 )


masses m1 and m2 is given by
dy

Þ 3 (60 - q) = q - 50 + q - 40
Gm1m2 Þ 180 - 3 q = 2 q - 90
FG =
r2 270
q= = 54° C
u

Force on mass m1, 5


St

Gm1m2 3. As we know that for the earth and satellite


F1 = = m1a1
r2 system,
where, a1 = acceleration GMm
Kinetic energy, K =
Gm 2a
Þ a1 = 2 2
r GMm
Potential energy, V = -
Þ a1 µ m2 a
GMm
and similarly, a2 µ m1 and total energy, E = -
2a
2. Let the final temperature after the masses in
thermal contact is q, then from the principle of where, a = radius of the orbit of the satellite and
calorimetry. m = mass of the satellite

Heat lost = Heat gained On the basis of above three expressions for the
energies, we have
16 WB JEE (Engineering) Solved Paper 2015

æ GMm ö T1 T2
GMm ç ÷ V A = A
K= = - ç- a ÷ = -
2a 2 2 DL1 DL2
ç ÷
è ø L L
4. From the lens formula (for first lens) where, A is an cross-section of the wire. assume
1 1 1 1 1 1 to be same at all the situations.
= - Þ = - T L T L
f1 v1 u1 2 v1 ( - 4 ) Þ 1´ = 2 ´
A DL1 A DL2
1 1 1 3

)
Þ + = = …(i) T1 T2

be
2 4 v1 4 Þ =
( L1 - L ) ( L2 - L )
4 4
Þ v1 = , u2 = 3 - = 5 / 3 T L -T L
3 3 T1( L2 - L ) = T2 ( L1 - L ); L = 2 1 1 2

Tu
T2 - T1
This image will be treated as the source for
second lens, then again from lens formula, we 7. The diagram is as follows
have

ou
A
1 1 1
= -
f2 v2 u2
θ qσ

(Y
1 1 B
F= —
Þ = +5 /3 [By equation (i)] 2∈0
1 v2
1
Þ 1 -5 / 3 = Þ -3 / 2
on
A′ mg
v2
This is the final image distance from 2nd lens. So, The electric field due to charged infinite
s
pi

the overall distance of image from the primary conducting sheet is E = .


source (or object) 2 Î0
m

Let d = 4 + 3 - 1.5 = 5 .5 Now, force (electric force) on the charged ball is


qs
5. The situation is given below F = qE =
ha

2Î0
The resultant of electric force and mg balance the
θ L v tension produced in the string.
C

L T
m F qs qs
So, tan q = e = =
mg 2 Î0 2 Î0 mg
θ
dy

m mg cos θ mg
mg
8. The circuit diagram can be redrawn as The
u

For motion along a vertical circular track, the potential between A and B is
required centripetal force is along the radius and A
St

D E
towards the centre of the circle is given by
– 2V
mv2 2Ω + 2Ω
T - mg cos q =
L 2V +
+ 2Ω –2V
mv2 –
Þ T= + mg cos q I
L C F
I′ B I–I′
6. Let the initial length of the metal wire is L. For the loop ABCDA,
The strain at tension T1 is DL1 = L1 - L +2 - 2 I ¢+2 - 2 I = 0 …(i)
The strain at tension T2 is DL2 = L2 - L For the loop ABFEA,
Suppose, the young’s modulus of the wire is Y, 2 - 2 I + 2 - 2( I - I ¢ ) = 0
then 4 - 2 I - 2 I + 2 I ¢= 0
WB JEE (Engineering) Solved Paper 2015 17

4 - 4 I + 2 I ¢= 0 11. The circuit is


2 = 2I - I ¢ …(ii) 50 Ω
From Eqs. (i) and (ii), we get
2 = 2I - I ¢ , 2 = I + I ¢
100 Ω
4 = 3 I , I = 4 / 3 = 1.33 A
9. The density of material is r 150 Ω
+ –
10 V

)
be
As the lower diode attached to 100 W resistance is
R in reversed biased so, it is non-conducting.

Tu
Now, the circuit can be redrawn as,
50 Ω
t

ou
The hollow sphere will float if its weight is less
than the weight of the water displaced by the

(Y
volume of the sphere This implies mass of the + – 150 Ω
sphere is less than that for the same volume of
water. Now, mass of spherical cell 10 V
m1 = 4 pR2 ´ t ´ r
on
V
\Current through the circuit, I =
While the mass of water having same volume R
4 4 10
pi

m2 = pR3 ´ r g = pR3 = = 0.05 A


3 3 200
m

1 kg 12. As the rms speed is given by


where, r g = density of water =
m3 3 RT
ha

Vrms =
For the floatation of sphere, M
m1 £ m2 T
4 Þ Vrms µ
C

4 pR ´ t ´ r £ pR3
2 M
3
R When temperature is doubled and molecules
dissociates into atoms, then
dy

Þ tr £
3
T T
R Vrms1
Þ t£ M M
= =
u

3r
Vrms 2 2T 4T
St

10. As we know that, M /2 M


l l2 T 1
R1 = r =r = =
a V 4T 2
where, l = length of wire If Vrms1 is V , then Vrms 2 will be 2V
a = area of cross-section of the wire
13. Let the masses of two particles are m1 and m2 .
and V = volume of the wire R1 µ l2
2
As the changes are of the same magnitude and
2
R1 æ l1 ö æ 1ö being accelerated through same potential, so
Þ =ç ÷ =ç ÷
R2 è l2 ø è2 ø these charges enter into the magnetic field with
the same speeds (let V ).
Þ R2 : R1 = 4 : 1
18 WB JEE (Engineering) Solved Paper 2015

Now, radii of the circular paths followed by two 17. Total angular momentum about O is given as,
changes is given by L = L1 + L2 = m1v1r1 + m2 v2 r2
mV = -6.5 ´ 2.2 ´ 15
. + 3.1 ´ 3.6 ´ 2.8
R1 = 1
qB . + 31248
= -2145 . = 9.8 kgm 2 /s
m2V R1 m1
and R2 = Þ = 18. The height raised by liquid in capillary tube
qB R2 m2
2 l cos q
h=
14. As large number of particles is situated at a rgh
distance R from the origin. If particles are

)
As in freely falling platform a body experience

be
uniformly distributed and make a circular
boundary around the origin, then centre of mass weight lessness.
will be at the origin. So, the liquid will rise upto to length of the

Tu
While if the particles are not uniformly capillary.
distributed, then centre of mass will lie between i.e. height raised by the liquid will be 20 cm.
particle and origin. This implies the distance
19. We have,

ou
between centre of mass and origin is always less
than equal to R. Apparent frequency,
15. Given, length of conductor, l = 0.1 m æ v + u0 ö æ 333 + 33 ö

(Y
n=ç ÷ n0 = ç ÷ ´ 1000
Mangetic field, B = 0.1 T è v - us ø è 333 - 33 ø
366
Velocity of conductor, V = 15 m/s = ´ 1000 = 1220 Hz
300
on
The angle between V and B is 90°
20. The energy of the photon,
When V and B are mutually perpendicular, then hc
pi

emf (induced) is given by E=


l
e = VBl
4 ´ 10 - 15 eVs ´ 3 ´ 108 m /s
m

15 =
= 15 ´ 0.1 ´ 0.1 = = 0.15 V 300 ´ 10 - 9 m
100
2
ha

4 ´ 10
16. As situation can be diagrammatically as below = eV
300
4
= eV = 1.33 eV
C

i θ 3
The ionisation energy is 13.6 eV which is greater
dy

90° than energy of photon, so atom can not come into


r excited state and will remain in ground state.
21. The component of velocity of B along x-direction
u

VBx = 20 cos 60 °
St

From law of reflection, 1


i=q = 20 ´ = 10 m/s
2
Now, q + r + 90° = 180° VA = 10 $i
Þ i + r + 90 ° = 180 °
V = 10 $i + 10 3 $j
B
r = 90° - i
VBA = VB - VA = 10 $i + 10 3 $j - 10 $i = 10 3 $j
Also, from Snell’s law
sin i 22. When light is refracted its frequency will remain
=m
sin r unchanged.
sin i sin i 23. As we know that, the rate of cooling is
Þ = =m
sin (90° - i ) cos i dq
= bA ( q - q0 )
Þ tan i = m dt
WB JEE (Engineering) Solved Paper 2015 19

where, bA = constant = 6 ´ 10 - 7 m = 600 nm


dq
Þ µ q - q0 hc 4 ´ 10 - 15 ´ 3 ´ 108
dt and l min = =
E 5
dT
Also - µ T - T0 -7
= 2.4 ´ 10 m = 240 nm
dt
where, T = temperature Clearly, wavelength range would be
and t = time 240 £ l £ 600
So, the rate of heat absorbed by cooled body is Thus, wavelength 650 nm is not suitable for

)
proportional to t2 - t1. photoelectric effect.

be
24. Let weight of A isW ¢. From the free body diagram, 28. The change in path length = ( m - 1) t.
For equilibrium of the system, This path length change the position of bright
fringe upto seventh bright fringe. The thickness of

Tu
N
plastic substance inserted in the path of rays will
must be 7 (on assuming fringe width to be units).
f = µN

ou
W 29. Let the length of closed organ pipe is l, then,
v
n n = (2 n - 1)
T cos q = mN = mW …(i) 4l
T sin q = W ¢ …(ii)
where, T = tension in the thread lying between
knot and the support.
(Y Its third harmonics (put n = 2) in above question,
n3 =
3v
4 l
…(i)
on
On divindg Eq. (ii) by Eq. (i), we get Now, the length of open organ pipe is 2l, then
T sin q W ¢ v nv
= nn = n =
pi

T cos q mW 2 (2 l ) 4 l

Þ tan q = Its fundamental frequency,
m

mW v
n0 = = 100 …(ii)
Þ W ¢ = mW tan q 2l
ha

25. The output of digital circuit can be given as, From Eq. (i) and (ii), we get
Y = AB + C 3v
n3 = = 3 ´ 100 = 300 Hz
C

It will be same as, Y = A + B + C 4l

26. The situation is diagrammatically as below 30. According to question the figure can be drawn as
dy

A below
5 µF 10 µF
X
u
St

B
Tower
– +
When A is projected with a vertical speed the after 300 V
sometime it comes on the same level with same
speed as it was projected. Now, the downward The equivalent capacitance,
speeds of A and B at the level-X is same. So, on 1 1 1 1 1
reaching the ground, velocity of A and B are same. = + = +
C eq C1 C2 5 10
27. Given that, 2 eV £ f £ 5 eV 2+1 3
Wavelength corresponding to minimum and = =
10 10
maximum values of work function are 10
hc 4 ´ 10 - 15 ´ 3 ´ 108 Þ C eq = mF
l max = = 3
E 2
20 WB JEE (Engineering) Solved Paper 2015

Now, the energy stored in the capacitor is 1 q q


f= ´ =
1 4 6 Î0 24 Î0
U = CV 2
2
1 10 35. The figure can be redrawn as,
= ´ ´ 10 - 6 ´ 300 ´ 300
2 3
3 3 2V 5Ω
= = = 0.15 J
10 ´ 2 20 10 Ω
31. As we know that the speed of efflux is given by

)
5Ω

be
v = 2 gh, where, h = height above the hole upto
which liquid is contained. 1.5 V
Þ vµ h
The current through the circuit

Tu
As the water coming out from hole-1 has net emf
i=
maximum speed (horizontal) speed. So, it will effective resistance
reach farthest distance.

ou
2 - 15
. 0.5
= =
AT - BT 2 5 + 5 + 10 20
32. Given, p =
V 1
= = 0.025 A
Þ
Þ
pV = AT - BT 2
pDV = ADT - BTDT (Y 40
The terminal potential difference of the batteries
VA = e A - irA = 2 - 0.025 ´ 5
on
On integrating, we get
T2 T2
= 2 - 0.0125 = 1.875 V
Work = ò PdV = A òT dT - B òT TdT and VB = e B - irB
1 1
pi

B = 15
. - 0.025 ´ 5
= A (T2 - T1 ) - [(T2 )2 - (T1 )2 ] = 15
. - 0.0125 = 1.375 V
2
m

33. When switch is in position a, then capacitor will 36. The situation can be diagrammatically as,
be changed.
ha

E
+q F = qE
When switch is in position b, then circuit
becomes an L C oscillator with frequency, a θ
E
C

1 1
n= . F = –qE –q
2 p LC
E
dy

In two situation the net reactance or impedance


of the circuit is zero. The dipole moment,
E P = 2qa (cos q $i + sin q $j)
u

This implies the current, i = i.e. infinite


zero
St

current through the circuit even without any The net force on the dipole is always zero while
applied emf. net torque on the dipole is not zero.

34. Consider the diagram, 37. The source is at infinity and incident beam is
parallel.
38. Whenever, there is a change in the magnetic flux
q crossing through the loop, an emf induced in it
and hence there is an amount of induced current
through the loop. If loop is rotated about one of its
diameter, the flux through it varies and causing
the emf induced in it.
In above the flux coverage of three face is shown 39. In this case (pure rolling) the velocity of point of
in figure. Which is like a quadrant in any plane. contact is zero.
So, flux will be
WB JEE (Engineering) Solved Paper 2015 21

40. The de-Broglie equation is h


Again, will be constant only when v is constant.
h h v
l= Þ m=
mv lv Thus, in present case the de-Broglie wavelength
1 h of heavier particle will be smaller. When the
Þ m µ , if = constant
l v velocities of two particles will be same.

Chemistry

)
41. Flame colours are produced from the movement 45. It is a substitution reaction, so the product

be
of the electrons in the metal ions present in the formed is
compounds. Cl NH2
Calcium gives brick red colour. Strontium gives

Tu
crimson colour. →
NH3

Barium gives apple green colour. EtOH


Br Br

ou
42. Extraction of gold (Au) involves the following
sequence of reaction, 46. When sulphuryl chloride (SO2Cl2 ) reacts with
Roasting -
white phosphorus ( P4 ), it give phosphorus
Gold ore ¾¾¾® HO + Au(CN)2-

(Y
pentachloride (PCl5 ) and sulphur dioxide (SO2 ).
CN - , H2 O, O2 X
The reaction is as follows:
Zn P4 + 10 SO2Cl2 ¾® 4PCl5 + 10SO2
¾® Zn(CN)24 - + Au
on
Y
47. The number of lone pair of electrons on the centra
Hence, X = Au(CN)2- , Y = Zn (CN)24 - atoms of H2O, SnCl2 , PCl3 and XeF2 are 2, 1, 1 and
3 respectively.
43. Atomic number of cerium = 58
pi

··
Electronic configuration of O
··
m

H H Number of lone pairs = 2


Ce = [Xe] 4 f 2 ,5 d0 ,6 s 2
··
Electronic configuration of
ha

Cl ¾ Sn ¾ Cl Number of lone pair = 1


Ce3+ = [Xe] 4 f 1 ··
+ P
44. HBr rH + Br -
C

Cl ½ Cl Number of lone pair = 1


For, 1,3-butadiene at room temperature, there is
formation of 1,4-addition product. Cl
dy

CH3 F
1
CH2 + H Br– →
3 +
u

4
H2C 2
Xe
St

H H

+ +
CH2 ==C—CH—CH2 ! CH2—C == CH—CH2 F

Number of lone pairs = 3


CH3 CH3
→

48. Salts such as NaCl, HgCl2 , CuCl2 are soluble in


→

Br H Br water while salts such as Hg 2Cl2 , CuCl and AgCl


are insoluble in water.


CH2 ==C—CH—CH3 CH2—C == CH—CH2 sp2 sp sp2 sp3 sp sp



49. CH2 == C == CH ¾C H ¾C ºº CH
CH3 CH3
½
1,2-addition 1,4-addition C ºº N
(Minor product) (Major product) sp

Thus, number of sp -hybridised C = 4


22 WB JEE (Engineering) Solved Paper 2015

50. Rate of reaction is doubled when concentration of 57. Cold FeSO4 solution on absorption of NO
A is doubled. Again, rate of reaction becomes four develops brown colour due to the formation of
times when concentration of A is increased by [Fe(H2O)5 (NO)] SO4 .
four times. It is clearly shown that there is no FeSO4 + NO + 5H2O ¾®
effect on rate of reaction on increasing the
[Fe(H2O)5 (NO)]SO4
concentration of B. Brown ring complex
Thus, order with respect to A is 1 and order with
This complex has 3.89 BM magnetic moment
respect to B is 0. Total order of reaction = 1
shows that it has 3 unpaired electrons.

)
51.
58. Group II A Group III A
Osmotic pressure

be
Be B
Mg Al
RT The electronic configuration of boron is 2 s 2 2 p1.

Tu
And electronic configuration of B+ is 2 s 2 . Hence,
O Concentration it is difficult to remove second electron from

ou
2 s 2 shell because half-filled and full-filled orbitals
Slope, RT = 291 R or T = 291 K are more stable than others.
\Temperature = 291 - 273 = 18 ° C 59. 15% will form racemic mixture with another 15%.
52. As from the formula, vrms =

Given that (v rms )CO = 1000 m/ s


3 RT
M (Y
60.
Hence, the
= (85 - 15 ) = 70%
CH3
enantiomeric

CH3
excess is
on
(Temp.)CO = 27° C = 300 K

(Temp.) N2 = 600 K + AlCl3 + HCl →
pi

Now putting the values, we get CH3


( v rms )CO 3 R TCO M N2
m

= ´ ´ CH3
( v rms ) N2 3 R MCO TN2 1, 4-dimethyl benzene 1, 3-dimethyl benzene

CHO CH2OH
ha

1000 300 28 1 61. –


OH
= ´ = →
(v rms ) N2 28 600 2
CHO COO–
C

or, (v rms ) N2 = 1000 ´ 1.414


= 1414 m/s It is an example of intramolecular Cannizaro
reaction.
dy

53. A gas can be liquefied at temperature T when it is


lower than critical temperature, Tc and pressure p 62. Given that,
is greater than critical pressure. Mass of single Ag atom = m
u

Hence, T < Tc and p > pc. Unit cell length = a


St

Total atoms present per unit cell of fcc lattice = 4.


54. The dispersed phase is liquid and dispersion
medium is gas in the fog. Therefore, mass of unit cell becomes 4 m. Volume
of unit cell = a3
55. On the basis of electropositivity of metal, the Mass of unit cell 4m
order of basic character is Density( r ) = = 3
Volume of unit cell a
K2O > BaO > CaO > MgO
63. For the reaction,
56. In aqueous alkaline solution,
-
two electron
2 SO2 (g ) + O2 (g ) 2SO3 (g )
r
reduction of HO2- gives O H. The reaction is as
Given that, DG° = -690.9 R, T = 300 K
follows:
+ -
From the formula, DG° = - RT ln K
H2O2
e H + HO 2 -690.9 R = - R ´ 300 ´ ln K
- 690.9
HO2- + H O + 2 e e3O H
2
- or ln k = = 2.303
300
WB JEE (Engineering) Solved Paper 2015 23

-
or = 2.303 log K = 2.303 67. NaOMe r Na +
+ OMe
or K = 101 = 10
O–
Unit of K = (atm)Dn = (atm)2 - 3 = (atm)-1 O
– →
\ K = 10 atm-1 Cl3C — C + OMe Cl3C — C — OMe
OMe
64. As we know that, OMe

→
Equivalent conductance, (l)
specific conductance (K ) ´ 1000

)
= O

be
concentration –
CCl2 + Cl–
→
l 1000 CCl3 + C
or, = Carbene
K conc. MeO OMe

Tu
l 1000 W -1 cm2eq -1
or, =
K 0.01 W -1 cm-1 68. Best reagent for nuclear iodination of aromatic
compounds is I2 / HNO3 .
(Given, conc. = 0.01 N)

ou
l →
or, = 105 cm3 eq -1 HNO3 Nitronium ion
K I2

I+ +I

(Y
I
65. From the formula, Aromatic compounds
→
Surface tension, Electrophilic substitution
reaction
DW J kg m2s -2
(g ) = = 2= = kg s -2
on
DA m m2
69. To a part of sodium extract, FeSO4 solution is
dV
Form the formula, F = h × A × added and the contents are warmed. A few drops
pi

dx of FeCl3 solution are then added and resulting


or, h (coefficient of viscosity) solution is acidified with conc. HCl. The
F N
m

= = = Nm-2 × s appearance of bluish green or Prussian blue


dV -1
ms colouration confirms the presence of nitrogen.
A× m2 × The reactions that occur during this test are as
ha

dx m
or kg ms -2 × m-2 × s = kg m-1 s -1 2NaCN + FeSO4 ¾® Na 2SO4 + Fe(CN)2
Fe(CN)2 + 4NaCN ¾® Na 4 [Fe(CN)6 ]
66. Given, that pH = 5.74, pK a =4.74
C

Sodium ferrocyanide
Suppose that volume of acid solution = x L 3 Na 4 [Fe(CN)6 ] + 4 FeCl3 ¾®
Volume of salt solution = y L Fe4 [Fe(CN)6 ]3 + 12NaCl
dy

From Henderson equation. Ferric ferrocyanide


(Prussian blue)
[Salt]
pH = pK a + log 70.
u

[Acid] OC2H5 H
[CH3COONa]
St

or, pH - pK a = log RMgBr+H — C — OC2H5 R—C— OC2H5


[CH3COOH]
[CH3COONa]
or, 5.74 - 4.74 = 1 = log OC2H5 OC2H5
[CH3COOH] (Acetal)
CH3COONa]
→

or = 10 H3O+
[CH3COOH]
0.1 x
O
[CH3COOH] 1 x+ y
or = =
[CH3COONa] 10 0.1 y R—C—H
Aldehyde
x+ y (P)
x 1
Thus, =
y 10
24 WB JEE (Engineering) Solved Paper 2015

71. The sequence of reactions are 74. Criteria for aromaticity (Huckel rule)
2AgNO3 + Na2S2O3
→ Ag S O +2NaNO The cyclic p molecular orbital (electron cloud)
2 2 3 3
Silver Sodium (X) formed by overlap of p-orbitals must contain
nitrate thiosulphate (White ppt.) (4 n + 2 ) p-electrons, where n = integer 0, 1, 2, 3,

es 3
etc. This is known as Huckel rule.

xc 2 O
(e 2 S
s)
Na
Na3[Ag(S2O3)2] (Y) Not soluble
Soluble (i) Cl + SbCl5
in water
H2O/∆ r –
SbCl6

)
2πe– system

be
Ag2S (Z)
Silver sulphide (aromatic)
Hence, X = Ag 2S2O3

Tu
(ii) Na / THF
Y = Na 3 [Ag(S2O3 )2 ] Z = Ag 2S s
72. From the given cell, the cell reaction is 6πe– system

ou
2+ (aromatic)
2Ag (s ) + Zn (0.1 M ) ¾®
2Ag + (0.1 M) + Zn(s ) H Br
H2O
The Nernst equation is

(Y
(iii) r
0.0591 [Ag + ]2
Ecell = E °cell - log Tropylium cation
n [Zn2 + ]
6πe– system
on
2
(0.0591) (0.1) (aromatic)
or, Ecell = ( - 1.562) - log
2 (0.1) H2N
(where, E°cell = -1.562 V)
pi

0.0591 (iv)
or, Ecell = ( - 1.562) - log 10 -1 HNO2
2
m

0.0591 4πe– system


= -1562
. +
2
ha

(non-aromatic)
= - 1562
. + 0.02955
75. Roasted copper pyrite on smelting with sand
= -1532
. V produces FeSiO3 as fusible slag and Cu2S as
C

73. For the reaction, X 2Y4 (l ) ¾® 2 X Y2 (g ) matte.


Dng = number of gaseous products For removing the gangue, FeS, silica present in
dy

- number of gaseous reactants the lining of the Bessemer converter, acts as a flux
and forms slag (iron silicate) on reaction with
Dng = 2 - 0 = 2 FeO.
u

Given that, DU = 2 kcal


FeS + 3O2 ¾® 2FeO + 2SO2
DS = 20 cal K -1
St

Gangue

From the formula, FeO + SiO2 ¾® FeSiO3


Gangue Slag
DH = DU + Dng RT
Copper matte mainly contains Cu2S and FeS.
Putting the values, we get
æ 2 ´2 ´ 300 ö 76. Ionisation potential values of noble gases
DH = 2 + ç ÷ decrease down the group with increase in atomic
è 1000 ø
3
size. Xenon forms binary fluorides by the direct
= 3.2 kcal = 3.2 ´ 10 cal reaction of elements.
Also, DG = DH - T DS The reason is that only the heavier noble gases
= 3.2 ´103 - 300 ´ 20 form such compounds. Octet of electrons provide
= 3.2 ´103 - 6 ´ 103 the stable arrangements but this statement is not
very much valid in cases of higher element which
= -2800 cal can be ionised easily due to large distance
between nucleus and valence electrons.
WB JEE (Engineering) Solved Paper 2015 25

77. The number of reactant molecules acquiring the


Cl 3–
activation energy increases with increasing
temperature.
79. Isoelectric point for neutral amino acids ranges
ox Cr ox from pH 5.5 to 6.3. Acidic amino acids have
isoelectric point around 3 while basic amino acids
have ranges from pH 7.6 to 10.8.
Cl At isoelectric point, amino acids exist as Zwitter
ions. At low pH, it exists as cation and at high pH,

)
be
it exists as anion. Hence, alanine (pH range 2-4)
Cl 3– will exist as cation while alanine (pH range 9-11)
will exist as anion.

Tu
ox Cr ox Acidic medium:
H3 N+ ¾CH (CH3 ) CO2 H
Basic medium:

ou
Cl H2 N ¾CH (CH3 ) CO2-
80. The sequence of reaction is as follows:
O CH2 CH3

(Y
3-
trans [CrCl2 (ox )2 ] isomer- optically inactive
C CH2
(superimposable mirror images and plane of
symmetry).
on
Zn/Hg
cis-[CrCl2 (ox)2 ]3 - , [Co(en)3 ]3 + and [Co(ox) (en)2 ] + C3H7COCl
AlCl3 HCl/∆
exhibited optical isomerism.
pi

78. The increase in rate constant of a chemical


reaction with increasing temperature are due to
m

the following facts. HNO3/H2SO4

The number of collisions among the reactant


ha

molecules increases with increasing


temperature. NO2
(o, p-director)
C

Mathematics
dy

81. From option (d), we have Also, g is the root of x2 + px + 1 = 0.


a-b =c -d …(i) \ g 2 + pg + 1 = 0 Þ g 2 = - pg - 1
u

and a - b2 = c2 - d2
2
…(ii)
Now, ( a + g )(b + g ) = ab + ag + bg + g2
St

Consider, a2 - b2 = c2 - d2 = 1 + g(a + b)- pg - 1 = g ( a + b - p)


Þ ( a + b )( a - b ) = ( c - d)( c + d) = g ´0 = 0 [Q a + b = p]
Þ a+ b =c + d …(iii) a
[using Eq. (i)] Since, g = - a or -b
On adding Eqs. (i) and (iii), we get \ ( a + g )(b + g ) = 0
2a = 2c Þ a = c
83. General term of (31/5 + 71/3 )100 is given by
Þ b=d [using Eq. (iii)]
Tr + 1 = 100C r (31/5 )100 - r (71/3 )r
Thus, an + b n = c n + dn for all n ÎN.
100 - r r
82. Since, a and b are the roots of x2 - px + 1 = 0. = 100C r × 3 5
× 73
\ a + b = p and ab =1 100 - r r
For a rational term, and must be integer.
5 3
26 WB JEE (Engineering) Solved Paper 2015

Hence, r =0, 15, 30, 45, 60, 75, 90 Þ ( y - 1) x2 + ( y + 1) x + 4 y - 4 = 0


\ There are seven rational terms. For x to be real, discriminant of the above
Hence, number of irrational terms quadratic equation should be greater than or
= 101 - 7 = 94 equal to 0.
84. Given equation is (2 x + 1)2 - px + q ¹ 0 Þ ( y + 1)2 - 4( y - 1)(4 y - 4 ) ³ 0
Þ 4 x2 + 4 x + 1 - px + q ¹ 0 Þ ( y + 1)2 - 16( y - 1)2 ³ 0
Þ 4 x2 + (4 - p) x + (1 + q ) ¹ 0 Þ( y + 1 + 4 y - 4 )( y + 1 - 4 y + 4 ) ³ 0

)
Þ (5 y - 3 )(5 - 3 y ) ³ 0
Now, D <0

be
2 é3 5 ù
Þ (4 - p) - 4 (4 )(1 + q ) < 0 \ y Îê , ú
ë5 3 û
Þ 16 - 8 p + p2 - 16 - 16 q < 0

Tu
89. 2 x2 + y2 + 2 xy + 2 x - 3 y + 8
Þ p2 - 8 p - 16 q < 0
1
= [4 x2 + 2 y2 + 4 xy + 4 x - 6 y + 16]
85. Let A be the set of families who own a car and B 2

ou
be the set of families who own a house. 1
= [( y2 - 8 y ) + (4 x2 + y2 + 4 xy + 4 x
Then, P ( A ) = 60%, P ( B ) = 30% 2
+ 2 y ) + 16]
and P ( A Ç B ) = 20%
Now, P ( A È B ) = P ( A ) + P ( B ) - P ( A Ç B )
= 60 + 30 - 20 = 70%
(Y 1
= [( y - 4 )2 + (2 x + y + 1)2 - 1]
2
So, least value will be - at
1
on
Now, families who own a car or a house but not 2
both = A D B -5
y =4 and x = .
Þ P ( AD B ) = P ( A È B ) - P ( A Ç B ) 2
pi

= 70 - 20 = 50% = 0.5 90. If a function f ( x ) assumes only irrational values


m

86. The number of words starting with CC = 4! and which is also continuous, then f ( x ) must be a
constant function.
The number of words starting with CH = 4!
ha

Þ f ( x ) = 2 as f ( 2 ) = 2
The number of words starting with CI = 4!
The number of words starting with CN = 4! \ f ( 2 - 1) = 2
COCHIN is the first word in the list of words
C

2 æ pö
beginning with CO. 91. Here, cos q + sin q + , q Î ç0, ÷
sin2 q è 2ø
\ Number of words that appear before the word For minimum value, sin 2q must be maximum.
dy

COCHIN = 96 p p
\ 2q = Þ q=
x 2 4
87. Given, f ( x ) = ò f (t ) dt …(i) p p 2
u

0
Hence, cos + sin + = 2 +2
Using Leibnitz theorem, we get 4 4 sin p
St

f ¢( x ) = f ( x ) Þ f ( x ) = kex 2
x
On putting x =0 in Eq. (i), we get lim ò 3 t2 dt
x 3 t2 2
0 92. lim ò dt = x ®2
f (0 ) = ò f (t ) dt x ®2 2 ( x -2 ) lim( x - 2 )
0 x ®2

Þ 0
ke = 0 éQ af ( x ) dx = 0 ù lim 3 x2
ëê òa ûú = x ®2
[using L’ Hospital’s rule]
1
Þ k =0 [Q e° = 1]
\ f ( x ) = 0 Þ f (log e 5 ) = 0 = 3 ´ (2 )2 = 12
2x x kx
x2 - x + 4 93. Given, cot + tan = cos ec
88. Let y = 3 3 3
x2 + x + 4
x
Þ x2 y + xy + 4 y = x2 - x + 4 Let q = , then
3
WB JEE (Engineering) Solved Paper 2015 27

\ cot 2q+ tan q = cosec kq 1


= w64 + = w + w2 [Qw3 = 1]
cos 2 q sin q w64
Þ + =cosec kq
sin 2 q cos q = -1 [Q1 + w + w2 = 0]
2 cos2 q - 1 sin q 97. We have,
Þ + = cosec kq
2 sin qcos q cos q 3 3
|z |= z - +
2 cos2 q - 1 + 2 sin2 q z z
Þ = cosec kq
2 sin qcos q 3 3
Þ |z | £ z - +

)
1 z z
Þ = cosec kq

be
2sin qcos q [using triangle inequality]
Þ cosec 2q = cosec kq 3 é 3 ù
\ k =2 Þ |z | £ 2 + êQ z - z = 2 ú

Tu
z ë û
p q
94. 4 cos q+ sin 2 q + 4 cot q cos2 æç - ö÷
4 2
3
è4 2ø Þ |z | £ 2 +
|z |
= 4 cos4 q+ (2 sin qcos q)2

ou
Þ |z |2 - 2|z |- 3 £ 0
é æ p qö ù
+ 2 cot q ê2 cos2 ç - ÷ ú Þ (|z | - 3 ) (|z | + 1) £ 0
è 4 2 øû

(Y
ë Þ - 1 £ |z | £ 3
= 4 cos4 q + 4 sin2 qcos2 q Thus, the maximum value of|z| is 3.
é æ p öù
+ 2 cot q ê1 + cos ç - q÷ ú 5 x2 - 26 x + 5
98. We have, <0
on
ë è 2 øû 3 x2 - 10 x + 3
2
[Q2 cos q= 1 + cos 2 q] 5 x2 - 25 x - x + 5
Þ <0
pi

= 4 cos2 q(cos2 q+ sin2 q) + 2 cot q (1 + sin q) 3 x2 - 9 x - x + 3


= 2 cos q + 2 cot q + 2 cos q 5 x( x - 5 ) - 1( x - 5 )
m

Þ <0
= -2cos q+ 2 cot q+ 2 cos q 3 x( x - 3 ) - 1( x - 3 )
é æ p 3p ö ù ( x - 5 )(5 x - 1)
ha

Þ <0
êQ forq Î çè 2 , 2 ÷ø, cos q = - cos qú ( x - 3 )(3 x - 1)
ë û
= 2cot q æ 1 1ö
\ x Î ç , ÷ È(3,5 )
C

2 è5 3 ø
95. Given, (sin x - x )(cos x - x ) = 0
Þ sin x = x or cos x = x2 99. If the given system of equations has no solution,
dy

2 -1 -2
Now, if sin x = x, then only one solution i.e. x = 0 is
possible. Also, if cos x = x2 , then two solutions are then 1 -2 1 = 0
u

possible. Hence, there are total 3 solutions. 1 1 l


Þ 2 ( -2 l - 1) + 1( l - 1) - 2 (1 + 2 ) = 0
St

-1 + 3 i
96. We know that, w= Þ - 4 l -2 + l -1 -6 = 0
2
-1 - 3 i Þ -3 l - 9 = 0
Þ 1 - 3 i = - 2 w and w2 = Þ l = -3
2
Þ 1 + 3 i = - 2 w2 1 x
64 64 100. We have, f ( x ) = 2x x ( x - 1)
æ 1+ 3i ö æ 1- 3i ö
Now, ç ÷ +ç ÷ 3 x ( x - 1) x ( x - 1)( x - 2 )
è 1- 3i ø è 1+ 3i ø
64
x+1
64
æ -2 w2 ö æ -2 w ö ( x + 1)
=ç ÷ +ç ÷
è -2 w ø è -2 w2 ø
( x + 1) x( x - 1)
28 WB JEE (Engineering) Solved Paper 2015

2
Taking common x, ( x -1) from R2 and R3 1æ 2ö
respectively, we get Þ ç1 + ÷xn =
9 è nø
1 x x +1 1 1
f ( x ) = x( x - 1) 2 x -1 x +1 Þ lim xn = (1 + 0 )2 =
n®¥ 9 9
3x x ( x - 2 ) ( x + 1) x
102. Variance of n natural numbers
Taking common ( x + 1) from C3 , we get n2 - 1 (20 )2 - 1
1 x 1 = = [Q n = 20]
12 12

)
f ( x ) = x( x - 1)( x + 1) 2 x -1 1
400 - 1 399 133

be
3x x( x - 2 ) x = = =
12 12 4
Taking common x from R3 , we get
103. Let the coin be tossed n times.

Tu
1 x 1
Let getting head is consider to be success.
f ( x ) = x2 ( x - 1) ( x + 1) 2 x - 1 1 1 1 1
3 x -2 1 \ p = , q = 1- p = 1- =
2 2 2

ou
Applying R1 ® R1 - R2 , R2 ® R2 - R3 , It is given that,
we get P ( X =3 ) = P( X = 5 )
-1 1 0

(Y
3 n -3 5 n -5
n æ 1ö æ 1ö æ 1ö æ 1ö
2
f ( x ) = x ( x - 1)( x + 1) -1 1 0 =0 Þ C3 ç ÷ ç ÷ = nC5 ç ÷ ç ÷
è2 ø è2 ø è2 ø è2 ø
3 x -2 1 Þ n
C3 = nC5
on
[Q R1 is identical with R2 ] Þ n = 3 +5 [Q nC x = nC y Þ x + y = n]
Þ f (100 ) = 0
Þ n =8
pi

é 1 1
101. We have, xn = ê æç1 - ö÷ æç1 - ö÷
1 8 -1
æ 1ö æ 1ö
ëè 3 øè 6ø Now, P ( X = 1) = 8C1 ç ÷ ç ÷
è2 ø è2 ø
m

2
æ 1ö æ 2 öù æ 1ö 1
8
ç1 - ÷ ... ç1 - ÷ú = 8C1 ´ ç ÷ =
è 10 ø è n( n + 1)
ha

øû è2 ø 32
2
é n æ n2 + n - 2 ö ù 104. Total number of ways in which the letters of the
Þ xn = ê Õ ç ÷ú word ‘PROBABILITY’ can be arranged in a row
C

êë n=2 è n ( n + 1) ø úû 11!
2 =
é n æ ( n + 2 )( n - 1) ö ù 2! 2!
= êÕ ç ÷ú
dy

ë n=2 è n ( n + 1) ø û Number of ways in which two B’s are together


n n 2 10 !
é æn + 2ö æ n - 1ö ù =
= êÕ ç ÷ × Õç 2!
u

÷ ú
ë n=2 è n + 1 ø n =2 è n ø û \ Required probability
St

2 2
é n æ n + 2 ö ù é n æ n - 1ö ù 10 !
= êÕ ç ÷ú êÕ ç ÷ú 10 ! ´ 2 ! 2
ë n=2 è n + 1 ø û ë n=2 è n ø û = 2! = =
11! 11! 11
2
æ4 5 6 n + 2ö 2! 2!
Þ xn = ç × × × ... × ÷
è3 4 5 n + 1ø
2
105. Option (a) If a and b are perpendicular to each
æ1 2 3 n - 1ö other, then a × b = 0
ç × × × ... × ÷
è2 3 4 n ø Now consider,
2 2 |a + b|2 = (a + b ) × (a + b )
æn + 2ö æ1ö
Þ xn = ç ÷ ç ÷
è 3 ø è nø = |a|2 + |b|2
2 So, option (a) is always true.
1 æn + 2ö
Þ xn = ç ÷
9 è n ø
WB JEE (Engineering) Solved Paper 2015 29

Option (b) If a and b are perpendicular to each 1 0 - (t + a )


other, then a × b = 0 Now, 0 1 16 =0
Now consider, -a 1 0
|a + lb|2 = (a + lb ) × (a + lb ) Þ 1 (0 - 16 ) - (t + a ) (0 + a ) = 0
2 2 2 Þ -16 - a (t + a ) = 0
= |a| + l |b|
Þ a (t + a ) + 16 = 0
Þ |a + lb| = |a|2 + l2 |b|2
Þ a2 + ta + 16 = 0
³|a| for all l Î R

)
Clearly, a should be real.

be
So, option (b) is always true.
\ t2 - 4 ´ 16 ³ 0 Þ t2 - 64 ³ 0
Option (c) consider,
Hence, least positive value of t is 8.
|a + b|2 + |a - b|2 = (a + b ) × (a + b )

Tu
108. We have, a2 cos2 A - b2 - c2 = 0
+ (a - b ) × (a - b )
Þ a2 cos2 A = b2 + c2
= |a|2 + a × b + b × a + |b|2
Also in DABC, we have

ou
+ |a|2 - a × b - b × a + |b|2
b2 + c2 - a2 a2 cos2 A - a2
= 2 (|a|2 + |b|2 ) cos A = =
2 bc 2 bc
So, option (c) is always true.
Option (d) consider,
a = - b and b =/ 0
(Y =
- a2 (1 - cos2 A ) - a2 sin2 A
2 bc
=
2 bc
<0

[Q0 < A < p; a, b, c > 0]


on
Then,|a + lb| ³ |a| Þcos A < 0 Þ A lies in IInd quadrant
Þ |- b + lb| ³ |- b| p
Þ <A<p
pi

Þ |b ||l - 1| ³ | b| Þ |l - 1| ³ 1 2
1 3p
which is not true for all l, as we consider l = , 109. Given, { x Î R :|cos x|³ sin x} Ç éê0, ùú
m

2 ë 2 û
then it is not true. If we draw, the graph of|cos x| and sin x, clearly
ha

Hence, option (d) is not always true. |cos x|³ sin x when
106. If four points ( x1, y1, z1 ), ( x2 , y2 , z2 ), ( x3 , y3 , z3 ) and é p ù é 3p 3p ù
x Î ê0, ú È ê , ú
C

( x4 , y4 , z4 ) are coplanar, then ë 4û ë 4 2 û


x2 - x 1 y2 - y 1 z2 - z 1 é p ù é 3p 3p ù é 3p ù
\ ê0, ú È ê , ú Ç ê0, ú
x3 - x 1 y3 - y 1 z3 - z 1 = 0 ë 4û ë 4 2 û ë 2 û
dy

x4 - x 1 y4 - y 1 z4 - z1 é p ù é 3p 3p ù
= ê0, ú È ê , ú
3 0 0 ë 4û ë 4 2 û
u

Now, 2 0
-2 = 0 æ x2 x3 x4 ö p
110. We have, sin-1 ç x -
St

+ - + K÷ =
2 l -1 0 è 2 4 8 ø 6

Þ 3 (0 + 2 l - 2 ) = 0 æ ö
ç ÷ é
-1 ç x ÷=p a ù
Þ l =1 Þ sin êQ S¥ = ú
ç æ -xö ÷ 6 ë 1 - rû
107. Consider the given equation of lines, ç 1 - çè ÷÷
è 2 øø
x - (t + a ) = 0 …(i)
æ 2x ö p
y + 16 = 0 …(ii) Þ sin-1 ç ÷=
è2 + x ø 6
and - ax + y = 0 …(iii)
2x p 2x 1
Since, these lines are concurrent, therefore the Þ = sin Þ =
2+ x 6 2+ x 2
system of equation is consistent.
2
Þ 4x =2+ x Þ 3x =2 Þ x =
3
30 WB JEE (Engineering) Solved Paper 2015

111. We have, y = x3 and A(1,1) Þ sin x (sin x - cos x )(sin x + cos x )


dy - cos2 x (sin x - cos x ) - cos2 x
\ =3 x2 ...(i)
dx (sin x - cos x ) = 0
On putting x =1 in Eq. (i), we get Þ (sin x - cos x )
dy (sin2 x + sin x cos x - 2 cos2 x ) = 0
= 3 (1)2 = 3
dx
Þ (sin x - cos x )(sin2 x + 2 sin x
\ Equation of tangent at A(1,1) is
cos x - sin x cos x - 2 cos2 x ) = 0
y - 1 = 3 ( x - 1) Þ y = 3 x - 2

)
\Required area Þ (sin x - cos x )2 (sin x + 2 cos x ) = 0

be
1 1
= ò x3 dx - ò (3 x - 2 ) dx Þ sin x - cos x =0 or sin x + 2 cos x = 0
0 2 /3
Þ tan x =1 or tan x = -2

Tu
1 1
é x4 ù é 3 x2 ù é p pù
= ê ú -ê -2 x ú But x Î ê - , ú
4
ë û0 ë 2 û 2 /3 ë 4 4û
p

ou
1 éæ3 ö æ2 4 öù Þ Only one solution i.e. x =
= - ç -2 ÷ - ç - ÷ 4
4 êë è 2 ø è 3 3 ø úû
1 115. Since, x1, x2 ,K, x15 are among

(Y
= sq unit 1, 2, 3, …, 15. So, x1 can take any value from 1, 2,
12
3, …, 15. Among these values one of the number
112. We have, log 0.2 ( x - 1) > log 0.04 ( x + 5 ) must be 1, hence product will be 0.
on
Þ log 0.2 ( x - 1) > log (0.2)2 ( x + 5 ) ì sin [ - x2 ]
1 ï , x ¹0
Þ log 0.2 ( x - 1) > log 0.2 ( x + 5 ) 116. We have, f ( x ) = í [ - x2 ]
2 ïa ,
pi

î x =0
Þ 2 log 0.2 ( x - 1) > log 0.2 ( x + 5 )
log 0.2 ( x - 1)2 > log 0.2 ( x + 5 ) sin [ - x2 ] sin ( -1)
m

Þ Now, lim = = sin(1)


x ®0 [ - x2 ] ( -1)
Þ ( x - 1)2 < x + 5
Since, f ( x ) is continuous at x =0.
ha

[Q log a x > log a y Þ x < y, if 0 < a < 1]


Þ x2 - 2 x + 1 < x + 5 Þ lim f ( x ) = f (0 )
x ®0
Þ x2 - 3 x - 4 < 0 Þ sin(1) = a
C

2
Þ x -4 x + x -4 < 0 a3 x4 a2 x3 a1 x2
117. Let f ( x ) = + + + a0 x
Þ x ( x - 4 ) + 1( x - 4 ) < 0 4 3 2
dy

Þ ( x - 4 )( x + 1) < 0 Þ x Î ( -1,4 ) a a a
\ f (0 ) = 0, f (1) = 3 + 2 + 1 + a0 = 0
But x > 1 Þ x Î(1,4 ) 4 3 2
u

113. We have, log e x + ex =0 Þ f (0 ) = f (1)


St

Þ f ¢( x ) = 0 has atleast one real root in [0, 1].


Þ log e x = - ex
[according to Rolle’s theorem]
Since, both the graphs intersect at only one point.
\ f ¢( x ) = a3 x + a2 x2 + a1 x + a0
3
Hence, the number of real roots of equation is one.
sin x cos x cos x Hence, a3 x3 + a2 x2 + a1 x + a0 must has a real root
114. Given, cos x sin x cos x =0 in the interval [0, 1].
cos x cos x sin x ì0, x is irrational
118. We have, f ( x ) = í
Þ sin x (sin2 x - cos2 x ) îsin| x|, x is rational
- cos x (sin x cos x - cos2 x ) If f ( x ) is continuous, then sin| x|=0
Þ x = kp, where k is an integer.
+ cos x (cos2 x - sin x cos x ) = 0
WB JEE (Engineering) Solved Paper 2015 31

119. Given, s = t 2 + at - b + 17 Since, vertex lies between the lines x + y = 3 and


é ds ù 2 x + 2 y - 1 = 0, then
After 5 sec, ê ú = [2t + a] t = 5 = 0 ( a - 1 + 2 - 3 )(2 a - 2 + 4 - 1) < 0
ë dt û t = 5
æ 1 ö
Þ a= -10 Þ ( a - 2 )(2 a + 1) < 0 Þ aÎ ç - , 2 ÷
è 2 ø
At t = 5 sec, s = 25 units
\ 25 = 52 + 5 a - b + 17 123. For curve I,
[Q s = 25 and t = 5 ] x2 y2
4 x2 + 9 y2 = 1 Þ + =1

)
2 2
Þ 25 = 25 + 5( -10 ) - b + 17 æ 1ö æ 1ö

be
ç ÷ ç ÷
Þ 50 - 17 = - b è2 ø è3 ø
Þ b = -33 1
which is an equation of ellipse with a= and
2

Tu
1 + 2 + K + n -1 1
120. lim b= .
n ®¥ n n 3
æ 1 + 2 + K + n -1 + n n ö x2 y2

ou
= lim ç - ÷ For curve II, 4 x2 + y2 = 4 Þ + =1
n ®¥è n n n´nø 1 4
1/ 2 x2 y2
1 n ærö 1 n Þ + =1

(Y (1)2 (2 )2
= lim å ç ÷ - lim ´
n ®¥ n è ø
r =1 n
n ®¥ n n
which is also an equation of ellipse with a = 1 and
1 2 b = 2.
=ò x dx + 0 =
on
0 3 Hence, there is no intersecting point.
121. We have, x - x1 y - y1 z - z1
124. If a line = = lies
ax ex - b log (1 + x ) é0 ù
pi

a1 b1 c1
lim 2
= 3 ê form ú
x ®0 x ë 0 û on a plane a2 x + b2 y + c2 z = d, then
m

Using L’ Hospital’s rule, we get (i) a1a2 + b1b2 + c1 c2 = 0


b (ii) a2 x1 + b2 y1 + c2 z1 = d
aex + axex -
ha

1+ x é0 ù According to the question,


lim =3 êë 0 form úû
x ®0 2x 3 (1) + (2 + l )( -2 ) + ( -1)(0 ) = 0
Þ a- b =0 Þ a= b Þ 3 -4 - 2 l =0
C

Using L’ Hospital’s rule, we get Þ l = - 1 /2


b Also, l -2 = 0 Þ l = 2
aex + aex + axex +
dy

(1 + x )2 So, there is no such value of l.


lim =3
x ®0 2 125. Y
u

b
Þ lim 2 aex + axex + =6 B(0,2) y=|x|
x ®0 (1 + x )2
St

A
(–1,1) C(1,1)
Þ 2a+ b = 6 …(i)
X
Þ 3a = 6 Þ a = 2 O (0,0)
On putting the value of a in Eq. (i), y=–|x|+1
we get b = 2
122. We have, y2 - 4 y = 4 x - 4 a In figure, B º (0,2 ) Þ OB = 2
Þ ( y - 2 )2 - 4 = 4 x - 4 a and OABC is a square.
Þ ( y - 2 )2 = 4 x - 4 a + 4 So, side of length = 2 units
\Area of OABC = 2 sq units
Þ ( y - 2 )2 = 4 [ x - ( a - 1)]
Hence, vertex is ( a-1, 2 ). 126. Here, 225 = 32 ´ 52
Þ d(225 ) = 3 ´ 3 = 9
32 WB JEE (Engineering) Solved Paper 2015

and 1125 = 32 ´ 53 = ( -1 + w)( - w4 + w2 ) = ( w - 1)( w2 - w)


Þ d(1125 ) = 3 ´ 4 = 12 = w3 - w2 - w2 + w = - 3 w2
640 = 27 ´ 5 3 3
132. Let I = ò f ( x2 ) dx = ò { x2 } dx
Þ d(640 ) = 8 ´ 2 = 16 0 0

Hence, 9, 12 and 16 are in GP. 3


= ò ( x2 - [ x2 ]) dx
0
p p
127. We know that, - £ sin-1 x £ 3 3
2 2 =ò x2 dx - ò [ x2 ] dx

)
0 0
-p -1 p

be
Þ £ 2 sin 2 a £ 3
2 2 é x3 ù 1 2 3
= ê ú - [ ò [ x2 ] dx + ò1 [ x2 ] dx + 2
ò 2 [ x ] dx]
-p -1 p ë 3 û0 0
Þ £ sin 2 a £

Tu
4 4
é 1 2 3 ù
æ -p ö æpö = 3 - ê ò 0 dx + ò1 1 dx + ò 2 2 dx úû
Þ sinç ÷ £ 2 a £ sinç ÷ ë0
è 4 ø è4ø
= 3 - [0 + ( x )1 2 + (2 x ) 32 ]

ou
-1 1
Þ £ 2a £
2 2 = 3 - 2 + 1-2 3 + 2 2
-1 1 1

(Y
Þ £a£ Þ |a| £ = 2 - 3 +1
2 2 2 2 2 2 a+ c
133. We have, a + b + c = 21 and =b
128. If one root of a quadratic equation is of the form 2
on
a + ib, then other root will be a - ib. a+ c
Þ a+ c + = 21
So, all the roots are 2 ± i, 5 ± 2 i. 2
\Product of all the roots a+ c
Þ a + c = 14 Þ =7
pi

= (2 + i )(2 - i )( 5 + 2 i )( 5 - 2 i ) 2
Þ b =7
m

= (4 + 1)(5 + 4 ) = 5 ´ 9 = 45
So, a can take values from 1 to 6,
½ 1 tan q 1 ½ c can have values from 8 to 13
ha

129. We have, f ( q) = ½- tan q 1 tan q½ or a= b = c =7 [Q a £ b £ c]


½ ½
½ -1 - tan q 1 ½ So, there are 7 such triplets.
C

2 2 2
= 1(1 + tan q) - tan q 134. For intersecting points, ex = ex sin x
2
( - tan q + tan q) + 1 (tan q + 1) 2
Þ ex (sin x - 1) = 0
dy

2 2
= 2(1 + tan q) - 0 = 2 sec q 2
Þ ex = 0 or sin x = 1
\Range of f = (2, ¥ ) 2
ex ¹ 0 Þ sin x = 1
u

But
130. Given, AB = B and BA = A
p
St

Now, A2 + B2 = A × A + B × B Þ x=
2
= A( BA ) + B ( AB ) 2
Now, y = ex
= ( AB )A + ( BA )B
dy 2 2
= BA + AB = A + B \ = ex × 2 x = 2 xex
dx
½1 + w w2 -w ½ 2
Also, y = ex sin x
131. ½1 + w2 w - w2½
½
½ ½
½ dy 2 2 2
2
- w2½ = ex × 2 x sin x + ex cos x = 2 xex
½w + w w
dx
½ 0 w2 - w½ [Q sin x = 1, cos x = 0]
= ½ 0 w - w2½ [Q C1 ® C1 + C2 ] Since, both the curves has equal slope.
½
½ ½
½ Hence, angle between the tangents at
½- 1 + w w - w2½
intersecting point is 0.
WB JEE (Engineering) Solved Paper 2015 33

1 4 dy
135. 2 cot -1 - cot -1 139. Given, = - (3 x2 tan-1 y - x3 )(1 + y2 )
2 3 dx
3 dy
= 2 tan-1 2 - tan-1 Þ = + x3 (1 + y2 ) - 3 x2
4 dx
= p + tan-1
4
- tan-1
3 (tan-1 y )(1 + y2 )
-3 4 1 dy
Þ × = x3 - 3 x2 tan-1 y
-1 4 -1 3 (1 + y2 ) dx
= p - tan - tan
3 4
1 dy

)
ì 4 3 ü Þ × + 3 x2 tan-1 y = x3
1 + y2 dx

be
ï + ï
= p - í tan-1 3 4 ý 1 dy dt
4 3 Put tan-1 y = t Þ × =
ï 1- × ï 2
î 3 4þ 1 + y dx dx

Tu
p p dt
-1
= p - tan ¥ = p - = \ + 3 tx2 = x3
2 2 dx

ou
136. which is linear differential equation in t.
(0,2) 3 x 2 dx 3
x–y=2 Now, IF = eò = ex
(2,0)
O
(0,–2)
x+y=2
(Y
140. Given,
\
dy
dx
y = e- x cos2 x

= e- x ( - sin 2 x ) 2
…(i)
on
2 2
(2 cos q, 2 sin q) will lie on the circle x + y = 4 + cos 2 x × e- x ( -1)
(from the above figure). Since, point lies on the dy
region containing origin. Þ = - 2 sin 2 x × e- x - y
pi

dx
So, point will be on the shaded region. dy
Þ + y = - 2 sin 2 x × e- x
m

æ p 3p ö dx
\ q Îç , ÷
è2 2 ø d2 y dy
Þ + = - 2 [sin 2 x × e- x ( -1)
ha

137. Let the point be ( h, k ). dx2 dx


+ e- x 2 cos 2 x]
½h - 2k + 1 ½
= 2 sin 2 x × e- x - 4 y [from Eq. (i)]
C

Then, ½ ½= 5
2 2
½ 1 + ( - 2 ) ½ d2 y dy dy
Þ + =- - y -4y
Þ h - 2k + 1 = ± 5 …(i) dx2 dx
dy

dx
½2 h + 3 k - 1½ d2 y dy
Also, ½ ½ = 13 Þ 2
+2 + 5y =0
2 2 dx dx
u

½ 2 +3 ½
141. Given, f (0 ) = 0 and f (0 ) = 2
St

Þ 2 h + 3 k - 1 = ± 13 …(ii)
On solving Eqs. (i) and (ii), we get four points. So, 1
\ lim [ f ( x ) + f (2 x ) + f (3 x ) + K + f (2015 x )]
there are such 4 points. x ®0x
f ¢( x ) + 2 f ¢(2 x ) + 3 f ¢(3 x )
138. Q P( x ) = ( x - 3 )( x - 5 )
+ K + 2015 f ¢(2015 x )
= Q( x ) + ( ax + b ) = lim
x ®0 1
Given, P(3 ) = 10 and P(5 ) = 6 [applying L’ Hospital’s rule]
Þ 3 a + b = 10 …(i)
and 5a + b = 6 …(ii) 2 + 2 ´ 2 + 3 ´ 2 + K + 2015 ´ 2
=
On solving Eqs. (i) and (ii), we get 1
a = - 2and b = 16 = 2[1 + 2 + 3 + K + 2015]
\Remainder = - 2 x + 16
34 WB JEE (Engineering) Solved Paper 2015

2(2015 )(2015 + 1) 2 ´ 2015 ´ 2016 x+3 -5 dx


= = Put =tÞ = dt
2 2 x -2 ( x - 2 )2
= 2015 ´ 2016 1 dt 1
\ I = - ò 7 /3 = - ò t -7 /3 dt
142. Given, 17 ! = 3556 xy428096000 5 t 5
Since, 17! is divisible by 9, so sum of the digits é ù é ù
1 ê t -7 / 3 + 1 ú 1 ê t -4 / 3 ú
(48 + x + y ) must be divisible by 9. =- ê ú +C = - ê 4 ú +C
So, x + y can be 15 or 6. 5 ê -7 + 1 ú 5 ê - ú
ë 3 û ë 3 û

)
Also, 17! is divisible by 11, so|10 + x - y| must be 4 /3

be
multiple of 11 or 0. The only possibility is 3 3 æ x -2ö
= +C = ç ÷ +C
| x - y| = 1. 20 t4 /3 20 è x + 3 ø
\ x + y = 15

Tu
145. We have, f : N ® R such that f (1) = 1
143. Let the event of person goes to office by a car,
scooter, bus or train be A, B, C and D, respectively. and f (1) + 2 f (2 ) + 3 f (3 ) + ... + nf ( n)
1 3 2 = n( n + 1) f ( n), " n ³ 2

ou
We have, P ( A ) = , P ( B ) = , P (C ) =
7 7 7 Clearly, f (1) + 2 f (2 ) = 2(2 + 1) f (2 )
1 Þ f (1) = 6 f (2 ) - 2 f (2 )
and P ( D ) =

(Y
7 Þ f (1) = 4 f (2 )
Let E = He reached office in time f (1) 1
Þ f (2 ) = =
We have, 4 4
on
æE ö 2 æE ö 1 æE ö 4 Similarly,
P ç ÷ = , P ç ÷ = ,P ç ÷ =
èAø 9 è B ø 9 èC ø 9 f (1) + 2 f (2 ) + 3 f (3 ) = 3 (3 + 1) f (3 )
æE ö 1 1
Þ 1 + + 3 f (3 ) = 12 f (3 )
pi

and P ç ÷ = 2
èDø 9
3 1
m

æEö Þ 9 f (3 ) = Þ f (3 ) =
P( A ) × P ç ÷ 2 6
æ Aö è Aø and f (1) + 2 f (2 ) + 3 f (3 ) + 4 f (4 ) = 4 (5 )f (4 )
Pç ÷=
ha

èEø æEö æEö 1 1


P( A ) × P ç ÷ + P( B ) × P ç ÷ Þ 1 + + = 16 f (4 )
è Aø è Bø 2 2
2 1
C

æEö æEö
+ P (C ) × P ç ÷ + P ( D ) × P ç ÷ Þ f (4 ) = =
èC ø èDø 16 8
1 7 1
In general, f ( n) =
dy

×
7 9 2n
= 1
1 7 3 8 2 5 1 8 \ f (500 ) =
× + × + × + ×
1000
u

7 9 7 9 7 9 7 9
7 146. Clearly, 5 distinct balls can be placed into 5 cells
St

=
7 + 24 + 10 + 8 in 55 ways.
Now, the number of ways of selecting one cell is
7 1 5
= = C1.
49 7
Let the selected cell be empty. Now, we are left
( x - 2 ) dx with 5 balls and 4 cells.
144. Let I = ò
{( x - 2 )2 ( x + 3 )7 }1/3 Now, number of ways of placing 5 distinct balls
dx into 4 cells such that each cell have atleast one
=ò ball
( x - 2 )-1/3 ( x + 3 )7 /3
= 45 - 4 C1 (4 - 1)5 + 4 C2 (4 - 2 )5
dx
=ò 7 /3 - 4 C3 (4 - 3 )5

x + 3ö
( x - 2) ç ÷ 5
= 1024 - 4 ´ 3 + 6 ´ 2 - 4 5
è x -2ø
WB JEE (Engineering) Solved Paper 2015 35

= 1024 - 972 + 192 - 4 = 1216 - 976 é q ù


ê 1 - tan2 ú
= 240 2
êQ cos q = q ú
Thus, the number of ways of placing 5 distinct ê 1 + tan2 ú
balls such that exactly one cell remains empty ê 2 ú
= 5C1 ´ 240 ê q ú
ê 2 tan
\ Required probability and sin q = 2 ú
ê qú
5
C ´ 240 240 48 ê 1 + tan2 ú
= 1 5 = = ë 2û
5 625 125

)
æ 2 qö q æ 2 qö
Þ aç1 - tan ÷ + 2 b tan = c ç1 + tan ÷

be
147. Let è 2ø 2 è 2ø
S = Event that person is smoker q q q
Þ a - a tan2 + 2 b tan - c - c tan2 = 0
NS = Event that person is non-smoker 2 2 2

Tu
D = Event that death is due to lung cancer. Now, 2 q q
Þ ( c + a)tan - 2 b tan + ( c - a) = 0
probability of death due to cancer that a person is 2 2

ou
a smoker, Let a and b be the roots of the equation.
æDö æ D ö 2b c-a
P ( D ) = P ( S) × P ç ÷ + P ( NS) × P ç ÷ \a + b = and ab =
è Sø è NS ø c+a c+a

Þ 0.006 =

6
20
100
2 æDö
æDö
´ Pç ÷ +
80
´
è S ø 100 10
8 æDö
1 æDö
´ Pç ÷
è Sø
(Y Now, tan
a+b
2
=
2b
c+a
c-a
on
Þ = Pç ÷ + Pç ÷ 1-
1000 10 è S ø 100 è S ø c+ a
æDöé 2 8 ù 6 2b
Þ Pç ÷ ê + =
pi

è S ø ë 10 100 úû 1000 c+a b


= =
æ D ö é 20 + 8 ù 6 c + a-c + a a
m

Þ Pç ÷ ê =
è S ø ë 100 úû 1000 c+a
æDö 6 100 b
ha

\ Pç ÷ = ´ Since, is a root of the equation.


è S ø 1000 28 a
3 b2 æbö
= \ ( c + a) 2 - 2 b ç ÷ + c - a = 0
C

140 a è aø
Þ b2 c + b2 a - 2 b2 a + ca2 - a3 = 0
148. Tangents drawn from external points are of equal
dy

length. Þ - b2 a + b2 c + ca2 - a3 = 0
Þ BD = BE = a - r [Q CD = r ] Þ b2 c - b2 a + ca2 - a3 = 0
and AF = AE = b - r [Q CF = r ]
u

Þ b2 ( c - a) + a2 ( c - a) = 0
Q AE + BE = AB
St

Þ b - r + a - r = 2R Þ ( c - a)( b2 + a2 ) = 0
[Q AB is a diameter of circumcircle] Þ c - a = 0 or b2 + a2 = 0
Þ b + a = 2R + 2r Þ c = a or b2 + a2 = 0
Þ 2( r + R ) = a + b
æ ai ö
149. Given equation is acos q + b sin q = c ç ÷
150. Let U i = ç bi ÷, where i = 1, 2, 3
æ 2 qö q ç ÷
ç 1 - tan ÷ 2 b tan è ci ø
Þ aç 2 ÷+ 2 =c
æ 1 0 0 ö æ a1 ö
ç 1 + tan2 q ÷ 1 + tan2 q ç ÷ç ÷
è 2ø 2 \ AU1 = ç2 1 0 ÷ ç b1 ÷
ç ÷ç ÷
è3 2 1 ø è c1 ø
36 WB JEE (Engineering) Solved Paper 2015

T
æ a1 ö æ 1ö é æ1 ö ù é -1 -7 9 ù é -1 -2 0 ù
ç ÷ ç ÷ êQ AU = ç0 ÷ ú adj U = ê -2 -5 6 ú = ê -7 -5 -3 ú
Þ ç 2 a1 + b1 ÷ = ç0 ÷ ê 1 ç ÷ú ê ú ê ú
ç ÷ ç ÷ ç ÷ êë 0 -3 3 úû êë 9 6 3 úû
è3 a1 + 2 b1 + c1 ø è0 ø êë è0 ø úû
\ a1 = 1, 2 a1 + b1 = 0 é 1 2 ù
ê- 3 - 3 0 ú
Þ 2 + b1 = 0 ê ú
7 5
Þ b1 = - 2 Þ U -1 = ê - - -1ú
and 3 a1 + 2 b1 + c1 = 0 ê 3 3 ú

)
ê 3 2 1ú
Þ 3 + 2( -2 ) + c1 = 0 ê ú

be
Þ 3 - 4 + c1 = 0 ë û
Þ c1 = 1 Hence, the sum of all elements of U -1 is 0.

Tu
æ 1 0 0 ö æ a2 ö 151. We have, f is continuous and differentiable
ç ÷ç ÷ function on [a, b].
Similarly, AU2 = ç2 1 0 ÷ ç b2 ÷
ç ÷ç ÷ Also, f ( a) = f ( b ) = 0.
è3 2 1 ø è c2 ø

ou
By Rolle’s theorem $ c Î(a, b)
æ a2 ö æ2 ö é æ2 ö ù such that f ¢( c ) = 0
ç ÷ ç ÷ êQ AU = ç3 ÷ ú
Þ ç 2 a2 + b2 ÷ = ç3 ÷ ê 2 ç ÷ú Thus, $ x Î(a, b) such that f ¢( x ) = 0

(Y
ç ÷ ç ÷ ç ÷
è3 a2 + 2 b2 + c2 ø è0 ø êë è0 ø úû Let at x = c Î(a,b), f ¢( c ) = 0
\ a2 = 2 and 2 a2 + b2 = 3 Now, f is continuously differentiable on [a, b].
Þ 2 ´ 2 + b2 = 3 Þ f ¢ is continuous on [a, b]
on
Þ 4 + b2 = 3 Also, f is twice differentiable on (a, b)
Þ b2 = - 1 \ f ¢ is differentiable on (a, b)
and f ¢( a) = 0 = f ¢( c )
pi

and 3 a2 + 2 b2 + c2 = 0
Þ 3 ´ 2 + 2( -1) + c2 = 0 By Rolle’s theorem $ k Î(a, c) such that f ¢¢( k ) = 0
m

Þ 6 - 2 + c2 = 0 Thus, $ x Î(a, c) such that f ¢¢( x ) = 0


Þ c2 = - 4 So, $ x Î(a, b) such that f ¢¢( x ) = 0
ha

æ 1 0 0 ö æ a3 ö Let us consider, f ( x ) = ( x - a)2 ( x - b ),


ç ÷ç ÷ where f ( a) = f ( b ) = f ¢( a) = 0 but
Now, AU3 = ç2 1 0 ÷ ç b3 ÷
ç ÷ç ÷ f ¢¢( a) ¹ 0 and f ¢¢¢( x ) ¹ 0 for any x Î(a, b)
C

è3 2 1 ø è c3 ø
æ a3 ö æ2 ö é æ2 ö ù 152. We have, xry : xy > 0
ç ÷ ç ÷ êQ AU = ç3 ÷ ú
dy

Þ ç 2 a3 + b3 ÷ = ç3 ÷ (i) Reflexive suppose xrx Î R (Relation)


ê 3 ç ÷ú
ç ÷ ç ÷ ç ÷
è3 a3 + 2 b3 + c3 ø è 1 ø êë è1 ø úû Þ x2 > 0
u

\ a3 = 2 and 2 a3 + b3 = 3 which is not true when x = 0.


Þ 2(2 ) + b3 = 3 Hence, relation is not reflexive.
St

Þ 4 + b3 = 3 (ii) Symmetric xry Î R.


Þ b3 = - 1 Þ xy > 0
and 3 a3 + 2 b3 + c3 = 1 Þ yx > 0
Þ 3 ´ 2 + 2( -1) + c3 = 1 Þ yrx Î R
Þ 6 - 2 + c3 = 1 If ( x, y ) Î R, then hence, relation is symmetric.
Þ c3 = - 3 (iii) Transitive xry Î R
æ1 2 2ö Þ xy > 0
ç ÷ and yrz Î R
\ U = ç -2 -1 -1 ÷
ç ÷ Þ yz > 0
è 1 -4 -3 ø
Þ xy2 z > 0
|U| = 1 (3 - 4 ) - 2 (6 + 1) + 2 (8 + 1)
= - 1 - 14 + 18 = 3 Þ xz > 0
WB JEE (Engineering) Solved Paper 2015 37

Þ ( x, z ) Î R p
154. 0 < q < ; cos q is strictly decreasing function.
If ( x, y ) Î R, then ( y, z ) Î R 2
Þ ( x, z ) Î R q
Option (a) When q > , then
Hence, relation is transitive. 2
q
153. Option (a) Let f ( x ) = cos x and g( x ) = sin x cos q £ cos
2
Consider the Wronskian of f ( x ) and g( x ), q
\ cos q £ cos [correct]
f ( x ) g( x ) 2
W=

)
f ¢( x ) g ¢( x ) 3q
Option (b) When q > , then

be
cos x sin x 4
= 3q
- sin x cos x cos q £ cos
4

Tu
= cos2 x + sin2 x = 1 =/ 0 3 /4 3q
\ (cos q) £ cos [incorrect]
Thus, the functions are linearly independent. So, 4

ou
the general solution of given differential equation 5q
Option (c) When q > , then
is given by y = A cos x + B sin x, where A and B 6
are real constants. 5q
cos q £ cos

(Y
[Q if y1 and y2 are linearly independent solutions 6
of the differential equation ay ¢¢ + by ¢ + c = 0, 5 /6 5q
then the general solution is y = c1 y1 + c2 y2 , \ (cos q) £ cos [correct]
6
on
where c1 and c2 are constants]. 7q
Hence, option (a) is true. Option (d) When < q, then
8
æ pö 7q
Option (b) Let y = A cos ç x + ÷
pi

è cos q £ cos
4ø 8
æ p pö 7q
m

7 /8
= A çcos x × cos - sin x × sin ÷ But (cos q) £ cos [incorrect]
è 4 4ø 8
A
ha

= (cos x - sin x ) 155. Given equation of hyperbola is


2 16 x2 - 3 y2 - 32 x - 12 y = 44
A æ Aö
= cos x + ç - ÷ sin x Þ 16 x2 + 16 - 32 x - 3 y2 - 12 - 12 y
C

2 è 2ø
= 44 + 4
which is in the form of general solution.
Þ 16( x - 1)2 - 3( y + 2 )2 = 48
dy

Hence, option (b) is true.


Option (c) Let y = A cos x sin x, which can not be ( x - 1)2 ( y + 2 )2
Þ - =1
expressed in the form of general solution. 3 16
u

æ pö On comparing the equation with standard


Option (d) Let y = A cos ç x + ÷
St

è 4ø equation of hyperbola,
pö we get a = 3, b = 4
æ
+ B sin ç x - ÷
è 4ø Now, length of transverse axis
æ 1 1 ö = 2a = 2 3
= A çcos x × - sin x × ÷
è 2 2ø and length of latusrectum
æ 1 1 ö 2 b2 2 ´ 16 32
+ B çsin x × - cos x × ÷ = = =
è 2 2ø a 3 3
æ A B ö æ B Aö b2
= cos x ç - ÷ + sin x ç - ÷ which is in \ Eccentricity ( e) = 1 +
è 2 2ø è 2 2ø a2
the form of general solution. 16 19
= 1+ =
Hence, option (d) is true. 3 3
38 WB JEE (Engineering) Solved Paper 2015

a 159. Given probability of defective part


Equation of directrix is x = ±
e 1
= 0.05 =
3 ´ 3 20
Þ x -1= ±
19 Probability of non-defective part
3 19
Þ x =1± = 1 - 0.05 = 0.95 =
19 20
é 1 ù We know that, P ( X = r ) = nC r pr q n- r
156. We have, f ( x ) = ê ú 1 19

)
ë [ x] û where, p = , q =

be
Domain = R - { f ( x ) = 0} 20 20
Now, f ( x ) will be zero, when [ x] = 0. r ³ 1 and n = ?
1
Þ x Î[0, 1) Also, P ( X ³ 1) ³

Tu
2
\Domain of f ( x ) = R - [0, 1)
1
and range of f ( x ) = { -1, 0, 1} Þ 1 - P (X = 0 ) ³
2

ou
157. If a quadratic equation has irrational coefficients, 0 n -0
æ 1 ö æ 19 ö 1
then roots are also irrational. Þ1 - hC0 ç ÷ ç ÷ ³
è 20 ø è 20 ø 2
So, option (c) is always false.

(Y
n
1 æ 19 ö
158. Given equation straight line is Þ 1- ³ç ÷
2 è 20 ø
( a - 1) x - by + 4 = 0 n
on
1 æ 19 ö
Coefficient of x Þ ³ç ÷
Slope = - 2 è 20 ø
Coefficient of y n
1 æ 95 ö
pi

- ( a - 1) a - 1 Þ ³ç ÷
= = 2 è 100 ø
-b b
m

Equation of hyperbola is Þ log 2 -1 ³ n [log 95 - log 100]


xy = 1 Þ – log 2 ³ n [log 95 - 2]
ha

1 Þ - (0.3 ) ³ n [1977
. - 2]
Þ y=
x 0.3
dy 1 Þ n ³
0.023
C

Þ =- 2
dx x 300
Þ n ³
Slope of normal = x2 > 0 23
dy

a-1 Þ n ³ 13.04
\ >0
b \ n = 14, 15
u

Þ a - 1 > 0 and b > 0 160. Given, f : R ® R


or a - 1 < 0 and b < 0
St

and f (2 x - 1) = f ( x ), x Î R
Þ a > 1, b > 0 or a < 1, b < 0
Hence, f is continuous at x = 1 and f (1) = 1.
Solved Paper 2014

WB JEE

)
be
Engineering Entrance Exam

Tu
Physics

ou
Category I

(Y
Directions (Q. Nos. 1-45) Carry one mark each, for which only one option is correct. Any wrong answer will
lead to deduction of 1/3 mark.
on
1. Consider three vectors A = i$ + $j - 2k $, 5. The output Y of the logic circuit given below is
$ $ $ $ $ $
B = i - j + k and C = 2i - 3 j + 4 k. A vector X of
pi

A Y
the form aA + bB (a and b are numbers) is
perpendicular to C. The ratio of a and b is B
m

(a) 1 : 1 (b) 2 : 1 (a) A + B (b) A


(c) -1 : 1 (d) 3 : 1 (c) ( A + B)×A (d) ( A + B)×A
ha

2. Three capacitors 3 mF, 6 mF and 6 mF are 6. One mole of an ideal monoatomic gas is heated
connected in series to a source of 120 V. The at a constant pressure from 0°C to 100°C. Then
C

potential difference, in volt, across the 3 mF the change in the internal energy of the gas is
capacitor will be (Given, R = 8.32 J mol-1K -1 )
dy

(a) 24 (b) 30 (c) 40 (d) 60 (a) 0.83 ´ 10 3 J (b) 4.6 ´ 10 3 J


3
3. A galvanometer having internal resistance (c) 2.08 ´ 10 J (d) 1.25 ´ 10 3 J
10 W requires 0.01 A for a full scale deflection.
u

To convert this galvanometer to a voltmeter of 7. The ionization energy of hydrogen is 13.6 eV.
St

full-scale deflection at 120 V, we need to The energy of the photon released when an
connect a resistance of electron jumps from the first excited state
( n = 2) to the ground state of a hydrogen atom is
(a) 11990 W in series
(b) 11990 W in parallel (a) 3.4 eV (b) 4.53 eV
(c) 12010 W in series (c) 10.2 eV (d) 13.6 eV
(d) 12010 W in parallel 8. A parallel plate capacitor is charged and then
4. A whistle whose air column is open at both disconnected from the charging battery. If the
ends has a fundamental frequency of 5100 Hz. plates are now moved farther apart by pulling
If the speed of sound in air is 340 ms-1, the at them by means of insulating handles, then
length of the whistle, in cm, is (a) the energy stored in the capacitor decreases
(a) 5/3 (b) 10/3 (b) the capacitance of the capacitor increases
(c) the charge on the capacitor decreases
(c) 5 (d) 20/3
(d) the voltage across the capacitor increases
2 WB JEE (Engineering) · Solved Paper 2014

9. When a particle executing SHM oscillates with p


a frequency n, then the kinetic energy of the
p1
particle
(a) changes periodically with a frequency of n
(b) changes periodically with a frequency of 2 n
(c) changes periodically with a frequency of n / 2
p2
(d) remains constant

)
10. In which of the following pairs, the two
V

be
physical quantities have different dimensions? V2 V1
(a) Planck’s constant and angular momentum 1
(b) Impulse and linear momentum (a) ( p1 - p2 ) (V1 - V2 )

Tu
2
(c) Moment of inertia and moment of a force 1
(d) Energy and torque (b) ( p1 + p2 ) (V1 - V2 )
2
11. A cricket ball thrown across a field is at heights 1æ a a ö

ou
(c) çç p1 + 2 - p2 - 2 ÷÷ (V1 - V2 )
h1 and h2 from the point of projection at times t1 2 è V1 V2 ø
and t2 respectively after the throw. The ball is æ
1 a a ö

(Y
caught by a fielder at the same height as that of (d) çç p1 + + p2 + 2 ÷÷ (V1 - V2 )
2
2 è V1 V2 ø
projection. The time of flight of the ball in this
journey is 15. A wooden block is floating on water kept in a
h1 t 22 - h2 t 12 h1 t 12 + h2 t 22
on
(a) (b) beaker. 40% of the block is above the water
h1 t 2 - h2 t 1 h2 t 1 + h1 t 2 surface. Now the beaker is kept inside a lift
h1 t 22 + h2 t 12 h1t 12 - h2t 22 that starts going upward with acceleration
(c) (d)
pi

h1 t 2 + h2 t 1 h1t 1 - h2t 2 equal to g/ 2. The block will then


(a) sink
m

12. A small metal sphere of radius a is falling with (b) float with 10% above the water surface
a velocity v through a vertical column of a (c) float with 40% above the water surface
ha

viscous liquid. If the coefficient of viscosity of (d) float with 70% above the water surface
the liquid is h, then the sphere encounters an
opposing force of 16. A smooth massless string passes over a smooth
fixed pulley. Two masses m1 and m2, ( m1 > m2 )
C

6hv phv
(a) 6 pha 2 v (b) (c) 6 phav (d) 3 are tied at the two ends of the string. The
pa 6a
masses are allowed to move under gravity
dy

13. A scientist proposes a new temperature scale starting from rest. The total external force
in which the ice point is 25 X (X is the new unit acting on the two masses is
of temperature) and the steam point is 305 X. ( m1 - m2 )2
u

(a) ( m1 + m2 ) g (b) g
The specific heat capacity of water in this new m1 + m2
St

scale is (in J kg-1X-1 ) ( m1 + m2 )2


3 3 (c) ( m1 - m2 ) g (d) g
(a) 4.2 ´ 10 (b) 3.0 ´ 10 m1 - m2
(c) 1.2 ´ 10 3 (d) 1.5 ´ 10 3
17. To determine the coefficient of friction between
14. One mole of a van der Waals’ gas obeying the a rough surface and a block, the surface is kept
equation inclined at 45° and the block is released from
aö rest. The block takes a time t in moving a
æ
ç p + 2 ÷ (V - b) = RT distance d. The rough surface is then replaced
è V ø
by a smooth surface and the same experiment
undergoes the quasi-static cyclic process which is repeated. The block now takes a time t / 2 in
is shown in the p-V diagram. The net heat moving down the same distance d. The
absorbed by the gas in this process is coefficient of friction is
(a) 3/4 (b) 5/4 (c) 1/2 (d) 1 / 2
WB JEE (Engineering) · Solved Paper 2014 3

18. A metal rod is fixed rigidly at two ends so as to (a) 2.16 ´ 10 -23 (b) 3.21 ´ 10 -22
-24
prevent its thermal expansion. If L, a and Y (c) 3.21 ´ 10 (d) 1.26 ´ 10 -23
respectively denote the length of the rod,
coefficient of linear thermal expansion and 23. A very small circular loop of radius a is initially
Young’s modulus of its material, then for an (at t = 0) coplanar and concentric with a much
increase in temperature of the rod by DT, the larger fixed circular loop of radius b. A
longitudinal stress developed in the rod is constant current I flows in the larger loop. The
(a) inversely proportional to a smaller loop is rotated with a constant angular

)
(b) inversely proportional to Y speed w about the common diameter. The emf

be
(c) directly proportional to DT / Y induced in the smaller loop as a function of
(d) independent of L time t is
pa 2m 0 I pa 2m 0 I

Tu
19. A particle is moving uniformly in a circular (a) w cos ( wt ) (b) w sin ( w2 t 2 )
2b 2b
path of radius r. When it moves through an 2
pa m 0 I pa 2m 0 I
angular displacement q, then the magnitude of (c) w sin ( wt ) (d) w sin2 ( wt )

ou
the corresponding linear displacement will be 2b 2b
æq ö æq ö 24. A luminous object is separated from a screen
(a) 2 r cos ç ÷ (b) 2 r cot ç ÷
è2 ø è2 ø by distance d. A convex lens is placed between
æq ö
(c) 2 r tan ç ÷
è2 ø
æq ö
(d) 2 r sin ç ÷
è2 ø (Y the object and the screen such that it forms a
distinct image on the screen. The maximum
possible focal length of this convex lens is
on
20. A uniform rod is suspended horizontally from (a) 4d (b) 2d
its mid-point. A piece of metal whose weight is d d
(c) (d)
w is suspended at a distance l from the 2 4
pi

mid-point. Another weight W1 is suspended on


the other side at a distance l1 from the 25. An infinite sheet carrying a uniform surface
m

mid-point to bring the rod to a horizontal charge density s lies on the xy-plane. The work
position. When w is completely immersed in done to carry a charge q from the point
$ ) to the point B = a( i$ - 2$j + 6k
A = a( $i + 2$j + 3k $)
ha

water, w1 needs to be kept at a distance l2 from


the mid-point to get the rod back into (where a is a constant with the dimension of
horizontal position. The specific gravity of the length and e0 is the permittivity of free space)
C

metal piece is is
w wl1 3saq 2saq
(a) (b) (a) (b)
dy

w1 wl - w 1l2 2 e0 e0
l1 l 5saq 3saq
(c) (d) 1 (c) (d)
l1 - l2 l2 2 e0 e0
u

21. A drop of some liquid of volume 0.04 cm3 is 26. The intensity of magnetization of a bar magnet
St

placed on the surface of a glass slide. Then is 5.0 ´ 104 Am-1. The magnetic length and the
another glass slide is placed on it in such a way area of cross-section of the magnet are 12 cm
that the liquid forms a thin layer of area 20 cm2 and 1 cm2 respectively. The magnitude of
between the surfaces of the two slides. To magnetic moment of this bar magnet is
separate the slides a force of 16 ´ 105 dyne has (in SI unit)
to be applied normal to the surfaces. The (a) 0.6 (b) 1.3 (c) 1.24 (d) 2.4
surface tension of the liquid is (in dyne-cm-1 )
27. A particle moves with constant acceleration
(a) 60 (b) 70 (c) 80 (d) 90
along a straight line starting from rest. The
22. An electron in a circular orbit of radius 0.05 nm percentage increase in its displacement during
performs 1016 revolutions per second. The the 4th second compared to that in the
magnetic moment due to this rotation of 3rd second is
electron is (in Am2 ) (a) 33% (b) 40% (c) 66% (d) 77%
4 WB JEE (Engineering) · Solved Paper 2014

28. A proton of mass m and charge q is moving in a 35. If n denotes a positive integer, h the Planck’s
plane with kinetic energy E. If there exists a constant, q the charge and B the magnetic
uniform magnetic field B, perpendicular to the é nh ù
field, then the quantity ê ú has the
plane of the motion, the proton will move in a ë 2pqB û
circular path of radius dimension of
2 Em 2 Em Em 2 Eq (a) area (b) length
(a) (b) (c) (d)
qB qB 2qB mB (c) speed (d) acceleration

)
29. An artificial satellite moves in a circular orbit 36. In a transistor output characteristics

be
around the earth. Total energy of the satellite commonly used in common emitter
is given by E. The potential energy of the configuration, the base current I B ,the collector
satellite is current IC and the collector-emitter voltage

Tu
2E 2E VCE have values of the following orders of
(a) -2 E (b) 2 E (c) (d) -
3 3 magnitude in the active region

ou
(a) IB and IC both are in mA and VCE in volt
30. In which of the following phenomena, the heat (b) IB is in mA and IC is in mA and VCE in volt
waves travel along straight lines with the (c) IB is in mA and IC is in mA and VCE in mV
speed of light?

(Y
(d) IB is in mA and IC is in mA and VCE in mV
(a) Thermal conduction (b) Forced convection
(c) Natural convection (d) Thermal radiation 37. The displacement of a particle in a periodic
æ tö
motion is given by y = 4 cos 2 ç ÷ sin (1000t ).
on
31. In the bandgap between valence band and è 2ø
conduction band in a material is 5.0 eV, then This displacement may be considered as the
the material is result of superposition of n independent
pi

(a) semiconductor (b) good conductor harmonic oscillations. Here n is


(c) superconductor (d) insulator
(a) 1 (b) 2 (c) 3 (d) 4
m

32. A uniform solid spherical ball is rolling down a 38. Consider a black body radiation in a cubical
smooth inclined plane from a height h. The
ha

box at absolute temperature T. If the length of


velocity attained by the ball when it reaches
each side of the box is doubled and the
the bottom of the inclined plane is v. If the ball
temperature of the walls of the box and that of
is now thrown vertically upwards with the
C

the radiation is halved, then the total energy


same velocity v, the maximum height to which
(a) halves (b) doubles
the ball will rise is
(c) quadruples (d) remains the same
dy

5h 3h 5h 7h
(a) (b) (c) (d)
8 5 7 9 39. Consider two concentric spherical metal shells
of radii r1 and r2 (r2 > r1 ). If the outer shell has a
u

33. Two coherent monochromatic beams of charge q and the inner one is grounded, the
intensities I and 4I respectively are
St

charge on the inner shell is


superposed. The maximum and minimum -r2
intensities in the resulting pattern are (a) q (b) zero
r1
(a) 5 I and 3 I (b) 9 I and 3 I
-r
(c) 4 I and I (d) 9 I and I (c) 1 q (d) -q
r2
34. In the circuit shown assume the diode to be
ideal. When Vi increases from 2V to 6 V, the 40. Four cells, each of emf E and internal
change in the current is (in mA) resistance r, are connected in series across an
external resistance R. By mistake one of the
Vi 150 W +3V cells is connected in reverse. Then the current
in the external circuit is
2E 3E 3E 2E
(a) (b) (c) (d)
(a) zero (b) 20 4r + R 4r + R 3r + R 3r + R
(c) 80/3 (d) 40
WB JEE (Engineering) · Solved Paper 2014 5

41. Same quantity of ice is filled in each of the two 43. A car is moving with a speed of 72 km-h -1
metal containers P and Q having the same towards a roadside source that emits sound at
size, shape and wall thickness but made of a frequency of 850 Hz. The car driver listens to
different materials. The containers are kept in the sound while approaching the source and
identical surroundings. The ice in P melts again while moving away from the source after
completely in time t1 whereas in Q takes a time crossing it. If the velocity of sound is 340 ms -1,
t2. The ratio of thermal conductivities of the the difference of the two frequencies, the driver
materials of P and Q is hears is

)
(a) t 2 : t 1 (a) 50 Hz (b) 85 Hz (c) 100 Hz (d) 150 Hz

be
(b) t 1 : t 2
44. The energy of gamma (g ) ray photon is E g and
(c) t 12 : t 22
that of an X-ray photon is E X . If the visible

Tu
(d) t 22 : t 12 light photon has an energy of Ev , then we can
say that
42. For the radioactive nuclei that undergo either a or (a) EX > Eg > Ev (b) Eg > Ev > E x

ou
b decay, which one of the following cannot occur? (c) Eg > EX > Ev (d) EX > Ev > Eg
(a) Isobar of original nucleus is produced
45. The intermediate image formed by the
(b) Isotope of the original nucleus is produced

(Y
objective of a compound microscope is
(c) Nuclei with higher atomic number than that of the
(a) real, inverted and magnified
original nucleus is produced
(b) real, erect and magnified
(d) Nuclei with lower atomic number than that of the
on
(c) virtual, erect and magnified
original nucleus is produced (d) virtual, inverted and magnified
pi

Category II
m

Directions (Q. Nos. 46-55) Carry two marks each, for which only one option is correct. Any wrong answer will
lead to deduction of 2/3 mark.
ha

46. A solid uniform sphere resting on a rough consecutive layers, respectively. The integer
horizontal plane is given a horizontal impulse m = 0, 1, 2, 3, K denotes the numbers of the
C

directed through its centre so that it starts successive layers.


sliding with an initial velocity v0. When it A ray of light from the 0th layer enters the 1st
dy

finally starts rolling without slipping the speed layer at an angle of incidence of 30°. After
of its centre is undergoing the mth refraction, the ray
2 3 5 6
(a) v0 (b) v0 (c) v 0 (d) v 0 emerges parallel to the interface. If m = 1.5 and
u

7 7 7 7 Dm = 0.015, the value of m is


St

47. The de-Broglie wavelength of an electron is the m –mD m


same as that of a 50 keV X-ray photon. The
ratio of the energy of the photon to the kinetic
energy of the electron is (the energy equivalent
of electron mass is 0.5 MeV)
(a) 1 : 50 (b) 1 : 20 m –2D m
(c) 20 : 1 (d) 50 : 1 m –D m
48. A glass slab consists of thin uniform layers of m
progressively decreasing refractive indices RI
°
30

(see figure) such that the RI of any layer is


m - m Dm. Here, m and Dm denote the RI of 0th
layer and the difference in RI between any two (a) 20 (b) 30 (c) 40 (d) 50
6 WB JEE (Engineering) · Solved Paper 2014

49. A long conducting wire r1


(a) [w1(r 2 - r w ) - w2r 2 ]
carrying a current I is P r w (r 2 - r1 )
bent at 120° (see figure). r1
I d (b) [w1(r 2 - r w ) + w2r 2 ]
The magnetic field B at r w (r 2 + r1 )
120°
a point P on the right r1
(c) [w1(r 2 + r w ) - w2r1 ]
bisector of bending I r w (r 2 - r1 )
angle at a distance d r1
(d) [w1(r1 - r w ) - w2r1 ]
from the bend is (m 0 is the permeability of free r w (r 2 - r1 )

)
space)

be
3m 0 I m0 I m0 I 3 m0 I 53. A 10 W electric heater is used to heat a
(a) (b) (c) (d) container filled with 0.5 kg of water. It is found
2 pd 2 pd 3 pd 2 pd
that the temperature of water and the

Tu
50. A circuit consists of three batteries of emf container rises by 3°K in 15 min. The container
E1 = 1 V , E 2 = 2 V and E3 = 3 V and internal is then emptied, dried and filled with 2 kg of
resistances 1 W , 2 W and 1 W respectively which oil. The same heater now raises the

ou
are connected in parallel as shown in the temperature of container-oil system by 2°K in
figure. The potential difference between points 20 min. Assuming that there is no heat loss in
the process and the specific heat of water is

(Y
P and Q is
E1=1V 4200 J kg -1K -1, the specific heat of oil in the
same unit is equal to
E2=2V (a) 1.50 ´ 103
on
P E3=3V Q (b) 2.55 ´ 103
(c) 3.00 ´ 103
pi

(d) 5.10 ´ 103


(a) 1.0 V (b) 2.0 V (c) 2.2 V (d) 3.0 V
54. An object is placed 30 cm away from a convex
m

51. Sound waves are passing through two lens of focal length 10 cm and a sharp image is
routes-one in straight path and the other along formed on a screen. Now a concave lens is
ha

a semicircular path of radius r and are again placed in contact with the convex lens. The
combined into one pipe and superposed as screen now has to be moved by 45 cm to get a
shown in the figure. If the velocity of sound sharp image again. The magnitude of focal
C

waves in the pipe is v, then frequencies of length of the concave lens is (in cm)
resultant waves of maximum amplitude will be (a) 72 (b) 60 (c) 36 (d) 20
integral multiples of
dy

55. Three identical square plates rotate about the


axes shown in the figure in such a way that
their kinetic energies are equal. Each of the
u

rotation axes passes through the centre of the


St

v v square. Then the ratio of angular speeds


(a) (b)
r(p - 2 ) r(p - 1) w1 : w2 : w3 is
2v v
(c) (d)
r(p - 1) r(p + 1)

52. To determine the composition of a bimetallic


alloy, a sample is first weighed in air and then
in water. These weights are found to be w1 and
1 2 3
w2 respectively. If the densities of the two
constituents metals are r1 and r 2 respectively,
then the weight of the first metal in the sample (a) 1 : 1 : 1 (b) 2 : 2 : 1
is (where rw is the density of water) (c) 1 : 2 : 1 (d) 1 : 2 : 2
WB JEE (Engineering) · Solved Paper 2014 7

Category III
Directions (Q. Nos. 56-60) Carry two marks each, for which one or more than one options may be correct.
Marking of correct options will lead to a maximum mark of two on pro rata basis. There will be no negative
marking for these questions. However, any marking of wrong option will lead to award of zero mark against the
respective questions - irrespective of the number of correct options marked.

)
56. A heating element of resistance r is fitted (b) Einstein analysis gives a threshold frequency

be
inside an adiabatic cylinder which carries a above which no electron can be emitted
frictionless piston of mass m and cross-section (c) The maximum kinetic energy of the emitted
A as shown in diagram. The cylinder contains photoelectrons is proportional to the frequency of

Tu
one mole of an ideal incident radiation
diatomic gas. The piston (d) The maximum kinetic energy of electrons does
current flows through not depend on the intensity of radiation

ou
the element such that the 59. Half of the space between the plates of a
temperatures rises with parallel-plate capacitor is filled with a
1 r
time t as DT = at + bt 2 dielectric material of dielectric constant K. The
2
(a and b are constants), while pressure remains
constant. The atmospheric pressure above the (Y remaining half contains air as shown in the
figure. The capacitor is now given a charge Q.
Then
on
piston is P0. Then
(a) electric field in the dielectric-filled region is higher
(a) the rate of increase in internal energy is than that in the air-filled region
5
R(a + bt ) (b) on the two halves of the bottom plate the charge
pi

2 densities are unequal


5
(b) the current flowing in the element is R(a + bt ) (c) charge on the half of the top plate above the
m

2r Q
air-filled part is
(c) the piston moves upwards with constant K+1
ha

acceleration (d) capacitance of the capacitor shown above is


(d) the piston moves upwards with constant speed C
(1 + K ) 0 , where C 0 is the capacitance of the
57. A thin rod AB is held horizontally so that it can 2
C

same capacitor with the dielectric removed.


freely rotate in a vertical plane about the end A
as shown in the figure. The potential energy of 60. A stream of electrons and protons are directed
dy

the rod when it hangs vertically is taken to be towards a narrow slit in a screen (see figure).
zero. The end B of the rod is released from rest The intervening
from a horizontal position. At the instant the region has a G
u

B
rod makes an angle q with the horizontal uniform electric E
St

(a) the speed of end B is proportional to sin q field E


(b) the potential energy is proportional to (1 - cos q) (vertically downwards) and a uniform
(c) the angular acceleration is proportional to cos q magnetic field B (out of the plane of the figure)
(d) the torque about A remains the same as its initial as shown. Then
value | E|
(a) electrons and protons with speed will pass
| B|
58. Find the correct A
q
B
through the slit
statement(s) about
photoelectric effect. (b) protons with speed| E|/| B | will pass through the
slit, electrons of the same speed will not
(a) There is no significant
(c) neither electrons nor protons will go through the
time delay between the
slit irrespective of their speed
absorption of a suitable
(d) electrons will always be deflected upwards
radiation and the emission of electrons
irrespective of their speed
Chemistry
Category I
Directions (Q. Nos. 1-45) Carry one mark each, for which only one option is correct. Any wrong answer will
lead to deduction of 1/3 mark.

1. The emission spectrum of hydrogen discovered 7. The reagents to carry out the following

)
first and the region of the electromagnetic conversion are

be
spectrum in which it belongs, respectively are Me Me
(a) Lyman, ultraviolet (b) Lyman, visible Me

Tu
(c) Balmer, ultraviolet (d) Balmer, visible O
2. The electronic configuration of Cu is (a) HgSO 4 /dil ×H2SO 4
(a) [Ne] 3s 2 , 3 p6 , 3d 9 , 4s 2 (b) BH3 ; H2O 2 /NaOH

ou
(b) [Ne] 3s 2 , 3 p6 , 3d 10 , 4s1 (c) OsO 4 ; HIO 4
(d) NaNH2 / CH3I; HgSO 4 / dil ×H2SO 4
(c) [Ne] 3s 2 , 3 p6 , 3d 3 , 4s 2 , 4 p6
(d) [Ne] 3s 2 , 3 p6 , 3d 5 , 4s 2 , 4 p4

3. As per de-Broglie’s formula a macroscopic


particle of mass 100 g and moving at a velocity
(Y
8. The correct order of decreasing H¾C¾H
angle in the following molecule is
H H H H
on
of 100 cm s -1 will have a wavelength of
H H H H
(a) 6.6 ´ 10-29 cm (b) 6.6 ´ 10-30 cm
I II III
pi

(c) 6.6 ´ 10-31 cm (d) 6.6 ´ 10-32 cm


(a) I > II > III
m

4. For one mole of an ideal gas, the slope of V vs. T (b) II > I > III
curve at constant pressure of 2 atm is X L (c) III > II > I
ha

mol-1 K -1. The value of the ideal universal gas (d) I > III > II
constant ‘R ’ in terms of X is 9. During the emission of a positron from a
-1 -1 X nucleus, the mass number of the daughter
(a) X L atm mol K (b) L atm mol -1 K -1
C

2 element remains the same but the atomic


(c) 2 X L atm mol -1 K -1 (d) 2 X atm L-1mol -1 K -1 number
dy

5. At a certain temperature the time required for (a) is decreased by 1 unit


(b) is decreased by 2 units
the complete diffusion of 200 mL of H2 gas is
(c) is increased by 1 unit
u

30 min. The time required for the complete (d) remains unchanged
diffusion of 50 mL of O2 gas at the same
St

temperature will be 10. b-emission is always accompanied by


(a) 60 min (b) 30 min (a) formation of antineutrino and a-particle
(c) 45 min (d) 15 min (b) emission of a-particle and g-ray
(c) formation of antineutrino and g-ray
6. The IUPAC name of the following molecule is (d) formation of antineutrino and positron
Me Me Me 11. Four gases P, Q, R and S have almost same
values of ‘b’ but their ‘a’ values (a, b are van der
Me Waals’ constants) are in the order
Q < R < S < P. At a particular temperature,
(a) 5,6-dimethylhept-2-ene among the four gases, the most easily
(b) 2,3-dimethylhept-5-ene liquefiable one is
(c) 5,6-dimethylhept-3-ene
(a) P (b) Q (c) R (d) S
(d) 5-iso-propylhex-2-ene
WB JEE (Engineering) · Solved Paper 2014 9

246
12. Among the following structures the one which 17. 98 Cf was formed along with a neutron when
is not a resonating structure of others is an unknown radioactive substance was
O O

O O
bombarded with 6C12. The unknown substance
was
Me – Me
O O 234 234 235 238
Me Me (a) 91Pa (b) 90 Th (c) 92 U (d) 92 U
I II

18. The rate of a certain reaction is given by,
O O O O rate = k [H+ ]n.

)
Me Me The rate increases 100 times when the pH

be
O O –
Me changes from 3 to 1. The order (n ) of the
III IV reaction is

Tu
(a) I (b) II (a) 2 (b) 0 (c) 1 (d) 1.5
(c) III (d) IV
19. The values of DH and DS of a certain reaction
13. The compound that will have a permanent are -400 kJ mol-1 and -20 kJ mol-1K -1

ou
dipole moment among the following is respectively. The temperature below which the
H Cl Cl H reaction is spontaneous, is
(a) 100 K (b) 20°C (c) 20 K (d) 120°C
H
I
Cl H
II
Cl
(Y
20. The correct statement regarding the following
compounds is
on
Cl Cl OH OH OH
Br Br
Cl Cl
pi

III IV
OH OH OH
m

(a) I (b) II I II III


(c) III (d) IV (a) all three compounds are chiral
ha

14. The correct order of decreasing length of the (b) only I and II are chiral
(c) I and III are diastereomers
bond as indicated by the arrow in the following
(d) only I and III are chiral
structures is
C

21. The correct statement regarding the following


energy diagrams is
dy

+ + +
+
I II III
u

(a) I > II > III (b) II > I > III


St

(c) III > II > I (d) I > III > II


Reactant
E E
15. An atomic nucleus having low n/p ratio tries to Reactant
Product Product
find stability by
(a) the emission of an a-particle Reaction M Reaction N
(b) the emission of a positron
(c) capturing an orbital electron (K-electron capture) (a) Reaction M is faster and less exothermic than
(d) emission of a b-particle reaction N
(b) Reaction M is slower and less exothermic than
16. ( 32Ge76 , 34Se76 ) and (14Si30 , 16S32) are examples of reaction N
(a) isotopes and isobars (c) Reaction M is faster and more exothermic than
(b) isobars and isotones reaction N
(c) isotones and isotopes (d) Reaction M is slower and more exothermic than
(d) isobars and isotopes reaction N
10 WB JEE (Engineering) · Solved Paper 2014

22. In the following reaction, the product E is 28. Among the following compounds, the one(s)
CHO that gives (give) effervescence with aqueous
(i) NaOH
E NaHCO3 solution is (are)
(ii) H+
CHO (CH3CO)2O CH3COOH PhOH CH3COCHO
CH2OH CHO I II III IV
(a) (b) (a) I and II (b) I and III
CHO CO2H (c) Only II (d) I and IV

)
be
CH2OH CO2H 29. The 4th higher homologue of ethane is
(c) (d) (a) butane (b) pentane (c) hexane (d) heptane
CO2H CO2H

Tu
30. In case of heteronuclear diatomics of the type
23. If Cl2 is passed through hot aqueous NaOH, AB, where A is more electronegative than B,
the products formed have Cl in different bonding molecular orbital resembles the

ou
oxidation states. These are indicated as character of A more than that of B. The
(a) -1and +1 (b) -1and +5 statement
(c) +1 and +5 (d) -1and +3 (a) is false

24. Commercial sample of H2O2 is labeled as 10 V.


Its % strength is nearly (Y (b) is true
(c) cannot be evaluated since data is not sufficient
(d) is true only for certain systems
on
(a) 3 (b) 6 (c) 9 (d) 12
31. The hydrides of the first elements in groups
25. The enthalpy of vaporisation of a certain liquid 15-17, namely NH3 , H2O and HF respectively
at its boiling point of 35°C is 24.64 kJ mol-1. show abnormally high values for melting and
pi

The value of change in entropy for the process boiling points. This is due to
is (a) small size of N, O and F
m

(a) 704 J K -1mol -1 (b) 80 J K -1mol -1 (b) the ability to form extensive intermolecular
(c) 24.64 J K -1mol -1 (d) 7.04 J K -1mol -1 H-bonding
ha

(c) the ability to form extensive intramolecular


26. Given that H-bonding
(d) effective van der Waals’ interaction
C

C + O2 ¾® CO2; DH ° = - x kJ
32. The quantity of electricity needed to separately
2CO + O2 ¾® 2CO2; DH ° = - y kJ
electrolyze 1 M solution of ZnSO4 , AlCl3 and
dy

The heat of formation of carbon monoxide will AgNO3 completely is in the ratio of
be (a) 2 : 3 : 1 (b) 2 : 1 : 1
y - 2x 2x - y
u

(a) (b) y + 2 x (c) 2 x - y (d) (c) 2 : 1 : 3 (d) 2 : 2 : 1


2 2
St

33. The amount of electrolytes required to


27. The intermediate J in the following Wittig coagulate a given amount of AgI colloidal
reaction is solution (-ve charge) will be in the order
+
(a) NaNO 3 > Al 2 (NO 3 )3 >Ba(NO 3 )2
(i) n-BuLi
PPh3Br– [J ] (b) Al 2 (NO 3 )3 >Ba(NO 3 )2 >NaNO 3
(ii) CH2==O (c) Al 2 (NO 3 )3 >NaNO 3 >Ba(NO 3 )2
(d) NaNO 3 >Ba(NO 3 )2 > Al 2 (NO 3 )3
PPh3 PPh3
(a) O (b) 34. The value of DH for cooling 2 mole of an ideal
O monoatomic gas from 225°C to 125°C at
5
O O constant pressure will be [given C p = R]
+ 2
(c) PPh3 (b) PPh3
(a) 250 R (b) -500 R
(c) 500 R (d) -250 R
WB JEE (Engineering) · Solved Paper 2014 11

35. An amine C3 H9N reacts with benzene 41. Metal ion responsible for the Minimata disease
sulphonyl chloride to form a white precipitate is
which is insoluble in aq. NaOH. The amine is (a) Co 2+ (b) Hg 2+
Me Me Me (c) Cu2+ (d) Zn2+
(a) N (b) N Me 42. The reagent with which the following reaction
H
is best accomplished is
Me
Me Me

)
Me

be
(c) Me NH2 (b)
Me NH2

36. The number of amino acids and number of

Tu
+
peptide bonds in a linear tetrapeptide (made of N2Cl– H
different amino acids) are respectively
(a) H3PO 2 (b) H3PO 3

ou
(a) 4 and 4 (b) 5 and 5
(c) H3PO 4 (d) NaHSO 3
(c) 5 and 4 (d) 4 and 3
43. In DNA, the consecutive deoxynucleotides are
37. Among the following, the one which is not a

(Y
connected via
‘‘green house gas’’, is
(a) phosphodiester linkage
(a) N2O (b) CO 2 (c) CH4 (d) O 2
(b) phosphomonoester linkage
38. The pH of 10-4 M KOH solution will be (c) phosphotriester linkage
on
(a) 4 (a) 11 (c) 10.5 (d) 10 (d) amide linkage

39. The system that contains the maximum 44. The reaction of aniline with chloroform under
pi

number of atoms is alkaline conditions leads to the formation of


(a) phenylcyanide
m

(a) 4.25 g of NH3 (b) 8 g of O 2


(c) 2 g of H2 (d) 4 g of He (b) phenylisonitrile
(c) phenylcyanate
ha

40. Among the following observations, the correct (d) phenylisocyanate


one that differentiates between SO32- and SO2-
4
is
45. The two half-cell reactions of an
C

electrochemical cell is given as


(a) both form precipitate with BaCl 2 ,SO 2-
3 dissolves in
HCl but SO 2- Ag + + e- ¾® Ag ; Eo = - 0.3995 V
4 does not Ag + /Ag
dy

2+ 3+ -
(b) SO 2-
3 forms precipitate with BaCl 2 ,SO 2-
4 does not
Fe ¾® Fe + e ; E°Fe 3+/ Fe2+ = - 0.7120 V
(c) SO 2-
4
2-
forms precipitate with BaCl 2 ,SO 3 does not The value of cell EMF will be
u

(d) both form precipitate with BaCl 2 ,SO 2-


4 dissolves in (a) -0.3125 V (b) 0.3125 V
HCl but SO 2- does not (c) 1.114 V (d) -1.114 V
St

Category II
Directions (Q. Nos. 46-55) Carry two marks each, for which only one option is correct. Any wrong answer will
lead to deduction of 2/3 mark.

46. The compressibility factor (Z ) of one mole of a van der Waals’ gas of negligible ‘a’ value is
bp bp bp
(a) 1 (b) (c) 1 + (d) 1 -
RT RT RT
12 WB JEE (Engineering) · Solved Paper 2014

47. When phenol is treated with D2SO4 / D2O, some 50. At 25°C, the molar conductance of 0.007 M
of the hydrogens get exchanged. The final hydrofluoric acid is 150 mho cm2mol-1 and its
product in this exchange reaction is L°m = 500 mho cm2mol-1. The value of the
dissociation constant of the acid at the given
OD OD concentration at 25°C is
D D D H (a) 7 ´ 10-4 M (b) 7 ´ 10-5 M
(a) (b) (c) 9 ´ 10-3 M (d) 9 ´ 10-4 M
H H H H

)
51. To observe an elevation of boiling point of

be
D D
0.05°C, the amount of a solute (mol. wt. = 100)
OD OD to be added to 100 g of water (K b = 0.5) is

Tu
(a) 2 g (b) 0.5 g (c) 1 g (d) 0.75 g
D H H H
(c) (d) 52. The volume of ethyl alcohol (density 1.15 g/cc)
H D D D that has to be added to prepare 100 cc of 0.5 M

ou
H H ethyl alcohol solution in water is
(a) 1.15 cc (b) 2 cc (c) 2.15 cc (d) 2.30 cc
48. The most likely protonation site in the
following molecule is

Me 1
3 4
5
(Y
53. The bond angle in NF3 (102.3°) is smaller than
NH3 (107.2°). This is because of
(a) large size of F compared to H
on
2 (b) large size of N compared to F
Me 6
8 7 (c) opposite polarity of N in the two molecules
(d) small size of H compared to N
pi

(a) C-1 (b) C-2


(c) C-3 (d) C-6 54. A piece of wood from an archaeological sample
has 5.0 counts min -1 per gram of C-14, while a
m

49. The order of decreasing ease of abstraction of


fresh sample of wood has a count of
hydrogen atoms in the following molecule
15.0 min -1 g -1. If half-life of C-14 is 5770 yr,
ha

Ha the age of the archaeological sample is


Me
(a) 8,500 yr (b) 9,200 yr
Hb
C

(c) 10,000 yr (d) 11,000 yr


55. The structure of XeF6 is experimentally
dy

Hc determined to be distorted octahedron. Its


structure according to VSEPR theory is
(a) Ha > Hb > Hc
(a) octahedron
(b) Ha > Hc > Hb
u

(b) trigonal bipyramid


(c) Hb > Ha > Hc
(c) pentagonal bipyramid
St

(d) Hc > Hb > Ha


(d) tetragonal bipyramid
WB JEE (Engineering) · Solved Paper 2014 13

Category III
Directions (Q. Nos. 56-60) Carry two marks each, for which one or more than one options may be correct.
Marking of correct options will lead to a maximum mark of two on pro rate basis. There will be no negative
marking for these questions. However, any marking of wrong option will lead to award of zero mark against the
respective question - irrespective of the number of correct options marked.

)
56. Two gases X (molecular weight M X ) and Y 59. Cupric compounds are more stable than their

be
(molecular weight MY ; MY > M X ) are at the cuprous counterparts in solid state. This is
same temperature T in two different because
containers. Their root mean square velocities (a) the endothermic character of the 2nd IP of Cu is

Tu
are C X and CY respectively. If the average not so high
kinetic energies per molecule of two gases X (b) size of Cu2+ is less than Cu+
and Y are E X and EY respectively then which of (c) Cu2+ has stabler electronic configuration as

ou
the following relation(s) is(are) true? compared to Cu+
(a) EX > EY (d) the lattice energy released for cupric compounds
(b) C X > CY is much higher than Cu+
(c) EX
(d) EX
= EY = (3 / 2 ) RT
= EY = (3/2 ) kBT
57. For a spontaneous process, the correct
(Y
60. Among the following statements about the
molecules X and Y, the one(s) which correct is
on
(are)
statement(s) is (are)
CHO CHO
(a) (DGsystem ) T , p > 0
H OH HO H
pi

(b) (DSsystem ) + (DSsurroundings ) > 0


(c) (DGsystem )T , p < 0 HO H H OH
m

(d) (DUsystem ) T , V > 0


H OH HO H
58. The formal potential of Fe3 + / Fe2+ in a
ha

H OH HO H
sulphuric acid and phosphoric acid mixture
(E° = + 0.61 V ) is much lower than the
CH2OH CH2OH
standard potential (E° = + 0.77 V ). This is due
C

to X Y
+
(a) formation of the species [FeHPO 4 ] (a) X and Y are diastereomers
dy

(b) lowering of potential upon complexation (b) X and Y are enantiomers


(c) formation of the species [FeSO 4 ]+ (c) X and Y are both aldohexoses
(d) high acidity of the medium (d) X is a D-sugar and Y is an L-sugar
u
St

Mathematics
Category I
Directions (Q. Nos. 1-60) Carry one mark each, for which only one option is correct. Any wrong answer will
lead to deduction of 1/3 mark.

1. The number of solution(s) of the equation 2. The value of | z |2 + | z - 3|2 + | z - i |2 is


x + 1 - x - 1 = 4x - 1 is/are minimum when z equals
(a) 2 (b) 0 2 i i
(a) 2 - i (b) 45 + 3i (c) 1 + (d) 1 -
(c) 3 (d) 1 3 3 3
14 WB JEE (Engineering) · Solved Paper 2014

ì 2 x2 + 1 , x £ 1 2 13. Let the number of elements of the sets A and B


3. If f (x) = í 3
, then ò f (x) dx is
î 4 x - 1 , x > 1 0 be p and q, respectively. Then, the number of
relations from the set A to the set B is
(a) 47/3 (b) 50/3 (c) 1/3 (d) 47/2
(a) 2 p +q
(b) 2 pq (c) p + q (d) pq
2a sin x - sin 2x
4. If lim exists and is equal to 1, 14. In a DABC, tan A and tan B are the roots of
x®0 tan3 x
then the value of a is pq(x2 + 1) = r 2x. Then, DABC is
(a) 2 (b) 1 (c) 0 (d) -1 (a) a right angled triangle

)
(b) an acute angled triangle

be
5. The solution of the equation (c) an obtuse angled triangle
log101 log7 ( x + 7 + x ) = 0 is (d) an equilateral triangle

Tu
(a) 3 (b) 7 (c) 9 (d) 49 15. If y = 4x + 3 is parallel to a tangent to the
6. The integrating factor of the differential parabola y2 = 12x, then its distance from the
dy -1
x normal parallel to the given line is
equation (1 + x2) + y = etan

ou
is
dx 213 219
(a) (b)
(a) tan-1 x (b) 1 + x2 17 17
211 210

(Y
-1
(c) e tan x
(d) loge (1 + x2 ) (c) (d)
17 17
7. If y = cos -1 x, then it satisfies the differential x2 y2
d 2y dy 16. Let the equation of an ellipse be + = 1.
on
equation (1 - x2) 2 - x = c, where c is 144 25
dx dx Then, the radius of the circle with centre (0, 2 )
equal to and passing through the foci of the ellipse is
pi

(a) 0 (b) 3 (c) 1 (d) 2 (a) 9 (b) 7 (c) 11 (d) 5


301
8. The number of digits in 20 (given, 17. The straight lines x + y = 0, 5x + y = 4 and
m

log10 2 = 0.3010) is x + 5 y = 4 form


(a) 602 (b) 301 (c) 392 (d) 391
ha

(a) an isosceles triangle


9. The area of the region bounded by the curves (b) an equilateral triangle
y = x2 and x = y2 is (c) a scalene triangle
C

(d) a right angled triangle


(a) 1/3 (b) 1/2 (c) 1/4 (d) 3
x 13 p
10. If R be the set of all real numbers and f : R ® R 18. If sin -1 æç ö÷ + cosec-1 æç ö÷ = , then the value
dy

2 è ø 13 è ø12 2
is given by f (x) = 3x + 1. Then, the set
f -1 ([1, 6]) is of x is
ì 5ü é 5ù (a) 5 (b) 4
u

5 5
(a) í- , 0, ý (b) ê - , ú (c) 12 (d) 11
î 3 3þ ë 3 3û
St

é 1 1ù æ 5 5ö 19. The value of l for which the curve


(c) ê - , (d) ç - , ÷
3 3û
ú
è 3 3ø (7x + 5)2 + (7 y + 3)2 = l2(4x + 3 y - 24)2
ë
represents a parabola is
p 2p 4p 6 7
11. The value of tan + 2 tan + 4 cot is (a) ± (b) ±
5 5 5 5 5
p 2p 4p 3p 1 2
(a) cot (b) cot (c) cot (d) cot (c) ± (d) ±
5 5 5 5 5 5

12. Let f (x) be a differentiable function in [2, 7]. If 20. Let f (x) = x + 1 / 2. Then, the number of real
f (2) = 3 and f ¢(x) £ 5 for all x in (2, 7), then the values of x for which the three unequal terms
maximum possible value of f (x) at x = 7 is f (x), f (2x), f (4x) are in HP is
(a) 7 (b) 15 (a) 1 (b) 0
(c) 28 (d) 14 (c) 3 (d) 2
WB JEE (Engineering) · Solved Paper 2014 15

21. Let f (x) = 2x2 + 5x + 1. If we write f (x) as ì é p ùü


tan í p ê x - úý
f (x) = a (x + 1) (x - 2) + b(x - 2) (x - 1) î ë 2 ûþ
27. The function f (x) = , where [x]
2 + [x]2
+ c(x - 1) (x + 1)
denotes the greatest integer £ x, is
for real numbers a, b, c, then
(a) continuous for all values of x
(a) there are infinite number of choices for a, b, c p
(b) only one choice for a but infinite number of (b) discontinuous at x =
2
choices for b and c

)
(c) not differentiable for some values of x
(c) exactly one choice for each of a, b, c

be
(d) discontinuous at x = - 2
(d) more than one but finite number of choices for
a, b, c 28. The function f (x) = a sin| x| + be| x | is

Tu
differentiable at x = 0 when
22. If a, b are the roots of ax2 + bx + c = 0 (a ¹ 0)
(a) 3a + b = 0 (b) 3a - b = 0
and a + h , b + h are the roots of px2 + qx + r = 0 (c) a + b = 0 (d) a - b = 0
( p ¹ 0), then the ratio of the squares of their

ou
13

discriminants is
29. If the coefficient of x8 in æç ax2 + ÷ is equal to
(a) a2 : p2 (b) a : p2 è bx ø
13

(Y
(c) a2 : p (d) a : 2 p æ 1 ö
the coefficient of x-8 in ç ax - 2 ÷ , then a and
è bx ø
23. Let p, q be real numbers. If a is the root of
b will satisfy the relation
x2 + 3 p2x + 5q2 = 0, b is a root of
on
(a) ab + 1 = 0 (b) ab = 1
x2 + 9 p2x + 15q2 = 0 and 0 < a < b, then the
(c) a = 1 - b (d) a + b = - 1
equation x2 + 6 p2x + 10q2 = 0 has a root g that
2 4
always satisfies 30. If I = ò ex (x - a ) dx = 0, then a lies in the
pi

a 0
(a) g = +b (b) b < g interval
m

4
a (a) (0, 2) (b) (-1, 0)
(c) g = + b (d) a < g < b
2 (c) (2, 3) (d) (-2, - 1)
ha

24. The equation of the common tangent with 31. The solution of the differential equation
positive slope to the parabola y2 = 8 3 x and é æ y2 ö ù
ê 2 f ç 2÷ ú
C

the hyperbola 4x2 - y2 = 4 is dy y èx øú


(a) y = 6 x + 2 (b) y = 6 x- 2 y =xê 2 + is (where, c is a
dx êx æ y2 ö ú
(c) y = 3 x + 2 (d) y = 3 x- 2 ê f¢ ç 2 ÷ ú
dy

êë è x ø úû
25. The point on the parabola y2 = 64x which is
constant)
nearest to the line 4x + 3 y + 35 = 0 has
u

æ y2 ö æ y2 ö
coordinates (a) f çç 2 ÷÷ = cx (b) xf çç 2 ÷÷ = c
St

èx ø èx ø
(a) (9, - 24) (b) (1, 81)
(c) (4, - 16) (d) (-9, - 24) æ y2 ö æ y2 ö
(c) f çç 2 ÷÷ = cx2 (d) x2 f çç 2 ÷÷ = c
èx ø èx ø
26. Let z1 , z2 be two fixed complex numbers in the
argand plane and z be an arbitrary point
32. Suppose that the equation f (x) = x2 + bx + c = 0
satisfying| z - z1 | + | z - z2| = 2| z1 - z2|. Then, has two distinct real roots a and b. The angle
the locus of z will be between the tangent to the curve y = f (x) at the
(a) an ellipse æa + b æ a + böö
point ç ,f ç ÷ and the positive
(b) a straight line joining z1 and z2 è 2 è 2 ø ÷ø
(c) a parabola direction of the x-axis is
(d) a bisector of the line segment joining z1 and z2 (a) 0° (b) 30° (c) 60° (d) 90°
16 WB JEE (Engineering) · Solved Paper 2014

33. The function f (x) = x2 + bx + c, where b and c 40. Let R be the set of all real numbers and
real constants, describes f : [-1, 1] ® R be defined by
ìï 1
(a) one-to-one mapping x sin , x ¹ 0
f (x) = í x . Then,
(b) onto mapping
(c) not one-to-one but onto mapping îï0, x=0
(d) neither one-to-one nor onto mapping (a) f satisfies the conditions of Rolle’s theorem on
[-1, 1]
34. Let n ³ 2 be an integer, (b) f satisfies the conditions of Lagrange’s mean

)
é cos (2p / n ) sin (2p / n ) 0ù value theorem on [-1, 1]

be
A = ê - sin (2p / n ) cos (2p / n ) 0ú and I is the (c) f satisfies the conditions of Rolle’s theorem on [0, 1]
ê 0 0 1úû (d) f satisfies the conditions of Lagrange’s mean
ë

Tu
identity matrix of order 3. Then, value theorem on [0, 1]
(a) A n = I and A n - 1 ¹ I 41. If a , b and c are positive numbers in a GP, then
(b) A m ¹ I for any positive integer m the roots of the quadratic equation

ou
(c) A is not invertible (log e a ) x2 - (2 log e b) x + (log e c) = 0 are
(d) A m = O for a positive integer m loge c
(a) -1and
loge a

(Y
35. Ram is visiting a friend. Ram knows that his
friend has 2 children and 1 of them is a boy. loge c
(b) 1 and -
Assuming that a child is equally likely to be a loge a
boy or a girl, then the probability that the other (c) 1 and log a c
on
child is a girl, is (d) -1and logc a
(a) 1/2 (b) 1/3 (c) 2/3 (d) 7/10
42. There is a group of 265 persons who like either
pi

36. The value of the sum singing or dancing or painting. In this group
(nC1 )2 + (nC 2)2 + (nC3 )2 + K + (nC n )2 is 200 like singing, 110 like dancing and 55 like
m

(a) ( 2 n C n )2 (b) 2n
Cn painting. If 60 persons like both singing and
dancing, 30 like both singing and painting and
(c) 2 n C n + 1 (d) 2n
Cn - 1
ha

10 like all three activities, then the number of


37. The remainder obtained when persons who like only dancing and painting is
1 ! + 2 ! + 3 ! + K + 11 ! is divided by 12 is (a) 10 (b) 20 (c) 30 (d) 40
C

(a) 9 (b) 8 (c) 7 (d) 6 æ p2 ö


43. The range of the function y = 3 sin çç - x2 ÷ is
38. Out of 7 consonants and 4 vowels, the number ÷
è 16
dy

ø
of words (not necessarily meaningful) that can
be made, each consisting of 3 consonants and (a) [0, 3 / 2 ] (b) [0, 1]
(c) [0, 3 / 2 ] (d) [0, ¥)
u

2 vowels, is
(a) 24800 (b) 25100 x2
St

ò0 cos (t 2) dt
(c) 25200 (d) 25400 44. The value of lim is
x®0 x sin x
2 22 n 23 n
39. Let S = nC 0 + C1 + C2 (a) 1 (b) -1
1 2 3
(c) 2 (d) loge 2
n+1
2 n
+K+ C n. Then, S equals 45. Let f (x) be a differentiable function and
n+1
f (4) - f (x2)
2n + 1 - 1 3n + 1 - 1 f ¢(4) = 5. Then, lim equals
(a) (b) x®2 x-2
n+1 n+1
n n (a) 0 (b) 5
3 -1 2 -1
(c) (d) (c) 20 (d) -20
n n
WB JEE (Engineering) · Solved Paper 2014 17

¥
æ n! p ö 52. Suppose that f (x) is a differentiable function
46. The sum of the series å sin ç
è 720 ø
÷ is
such that f ¢(x) is continuous, f ¢(0) = 1 and f ¢¢(0)
n =1
p ö does not exist. Let g (x) = xf ¢(x). Then,
(a) sin æç æ p ö + sin æ p ö
÷ + sin ç ÷ ç ÷ (a) g ¢(0) does not exist (b) g ¢(0) = 0
è 180 ø è 360 ø è 540 ø
p p p ö æ p ö (c) g ¢(0) = 1 (d) g ¢(0) = 2
(b) sin æç ö÷ + sin æç ö÷ + sin æç ÷ + sin ç ÷
è 6ø è 30 ø è 120 ø è 360 ø 53. Let z1 be a fixed point on the circle of radius 1
p p p ö
(c) sin æç ö÷ + sin æç ö÷ + sin æç ÷ centred at the origin in the argand plane and

)
è 6ø è 30 ø è 120 ø
z1 ¹ ± 1. Consider an equilateral triangle

be
p ö æ p ö
+ sin æç ÷ + sin ç ÷ inscribed in the circle with z1 , z2, z3 as the
è 360 ø è 720 ø vertices taken in the counter clockwise
p ö æ p ö
(d) sin æç direction. Then, z1z2z3 is equal to

Tu
÷ + sin ç ÷
è 180 ø è 360 ø
(a) z12 (b) z13
47. Let I denote the 3 ´ 3 identity matrix and P be a (c) z14 (d) z1

ou
matrix obtained by rearranging the columns of
I. Then, 54. Suppose that z1 , z2, z3 are three vertices of an
(a) there are six distinct choices for P and det (P) = 1 equilateral triangle in the argand plane. Let

(Y
1
(b) there are six distinct choices for P and det(P) = ± 1 a = ( 3 + i ) and b be a non-zero complex
(c) there are more than one choices for P and some 2
of them are not invertible number. The points az1 + b , az2 + b , az3 + b will
on
(d) there are more than one choices for P and P -1 = I be
in each choice (a) the vertices of an equilateral triangle
(b) the vertices of an isosceles triangle
pi

48. The coefficient of x3 in the infinite series (c) collinear


2 (d) the vertices of a scalene triangle
expansion of , for| x| < 1, is
m

(1 - x) (2 - x)
55. The curve y = (cos x + y)1/ 2 satisfies the
1 15 1 15
(a) - (b) (c) - (d) differential equation
ha

16 8 8 16 2
d2y dy
49. For every real number x, (a) (2 y - 1) 2
+ 2 æç ö÷ + cos x = 0
dx è dx ø
C

2
x 3 2 7 3 15 4 d2y dy
let f (x) = + x + x + x + K . Then, (b) 2 - 2 y æç ö÷ + cos x = 0
1! 2! 3! 4! dx è dx ø
dy

the equation f (x) = 0 has 2 2


d y dy
(c) (2 y - 1) 2 - 2 æç ö÷ + cos x = 0
(a) no real solution dx è dx ø
(b) exactly one real solution 2 2
d y dy
(d) (2 y - 1) 2 - æç ö÷ + cos x = 0
u

(c) exactly two real solutions


dx è dx ø
(d) infinite number of real solutions
St

50. Let S denote the sum of the infinite series 56. In the argand plane, the distinct roots of
8 21 40 65 1 + z + z3 + z 4 = 0 (z is a complex number)
1+ + + + + K . Then, represent vertices of
2! 3! 4! 5!
(a) S < 8 (b) S > 12 (a) a square (b) an equilateral triangle
(c) 8 < S < 12 (d) S = 8 (c) a rhombus (d) a rectangle

51. Let [x] denote the greatest integer less than or 57. In a DABC , a , b, c are the sides of the triangle
equal to x for any real number x. Then, opposite to the angles A, B, C, respectively.
[n 2 ] Then, the value of a3 sin (B - C )
lim is equal to 3 3
+ b sin (C - A ) + c sin ( A - B) is equal to
n® ¥ n
(a) 0 (b) 1
(a) 0 (b) 2
(c) 3 (d) 2
(c) 2 (d) 1
18 WB JEE (Engineering) · Solved Paper 2014

58. Let a, b be the roots of x2 - x - 1 = 0 and 12 ! 212 12 ! 12 !


(a) 12
(b) (c) 6 12 (d)
n n
S n = a + b , for all integers n ³ 1. Then, for 6! 6! 6 2 6 612 2 6 62 612
every integer n ³ 2.
60. If a, b are the roots of the quadratic equation
(a) S n + S n - 1 = S n + 1
(b) S n - S n - 1 = S n + 1 x2 + px + q = 0, then the values of a3 + b3 and
(c) S n - 1 = S n + 1 a 4 + a 2b 2 + b 4 are respectively
(d) S n + S n - 1 = 2S n + 1 (a) 3 pq - p3 and p4 - 3 p2q + 3q 2
(b) - p(3q - p2 ) and ( p2 - q ) ( p2 + 3q )

)
59. A fair six-faced die is rolled 12 times. The

be
probability that each face turns up twice is (c) pq - 4 and p4 - q 4
equal to (d) 3 pq - p3 and ( p2 - q ) ( p2 - 3q )

Tu
Category II
Directions (Q. Nos. 61-75) Carry two marks each, for which only one option is correct. Any wrong answer will

ou
lead to deduction of 2/3 mark.

(Y
61. The solution of the differential equation 65. The equation of hyperbola whose coordinates
dy y 1 of the foci are (±8, 0) and the length of
+ =
dx x log e x x latusrectum is 24 units, is
on
(a) 3 x2 - y2 = 48 (b) 4 x2 - y2 = 48
under the condition y = 1 when x = e is 2 2
1 (c) x - 3 y = 48 (d) x2 - 4 y2 = 48
(a) 2 y = loge x +
66. A student answers a multiple choice question
pi

loge x
2 with 5 alternatives, of which exactly one is
(b) y = loge x +
correct. The probability that he knows the
m

loge x
(c) y loge x = loge x + 1
correct answer is p, 0 < p < 1. If he does not
know the correct answer, he randomly ticks
ha

(d) y = loge x + e
one answer. Given that he has answered the
62. Let f (x) = max { x + | x|, x - [x]}, where [x] question correctly, the probability that he did
denotes the greatest integer £ x. Then, the not tick the answer randomly, is
C

3 3p 5p
value of ò f (x) dx is (a) (b)
-3 4p + 3 3p + 2
5p 4p
dy

(a) 0 (b) 51/2 (c) (d)


(c) 21/2 (d) 1 4p + 1 3p + 1
1 2p 4p 6p
63. Let X n = ìí z = x + iy :| z |2 £ üý for all integers
u

67. cos + cos + cos


î nþ 7 7 7
St

¥ (a) is equal to zero (b) lies between 0 and 3


n ³ 1. Then, Ç X n is (c) is a negative number (d) lies between 3 and 6
n =1

(a) a singleton set cos x p /2


68. Suppose M = ò dx,
(b) not a finite set x+2 0

(c) an empty set p / 4 sin x cos x


N =ò dx. Then, the value of
(d) a finite set with more than one element 0 (x + 1)2
64. Applying Lagrange’s Mean Value Theorem for (M - N ) equals
a suitable function f (x) in [0, h ], we have 3 2
(a) (b)
f (h ) = f (0) + hf ¢(qh ), 0 < q < 1. Then, for p+2 p-4
f (x) = cos x, the value of lim q is 4 2
h ® 0+
(c) (d)
p -2 p+ 4
(a) 1 (b) 0 (c) 1/2 (d) 1/3
WB JEE (Engineering) · Solved Paper 2014 19

69. For any two real numbers q and f, we define 73. Let tn denotes the nth term of the infinite
qRf, if and only if sec2 q - tan 2 f = 1. The series
1 10 21 34 49
+ + + + +K . Then,
relation R is 1! 2! 3! 4! 5!
(a) reflexive but not transitive lim tn is
(b) symmetric but not reflexive n® ¥

(c) both reflexive and symmetric but not transitive (a) e (b) 0 (c) e 2 (d) 1
(d) an equivalence relation 74. A particle starting from a point A and moving

)
70. The minimum value of 2sin x + 2cos x is with a positive constant acceleration along a

be
straight line reaches another point B in time T.
1 - 1/ 2 1 + 1/ 2
(a) 2 (b) 2 Suppose that the initial velocity of the particle
2
(c) 2 (d) 2 is u > 0 and P is the mid-point of the line AB. If

Tu
the velocity of the particle at point P is v1 and if
71. We define a binary relation ~ on the set of all T
3 ´ 3 real matrices as A ~ B, if and only if there the velocity at time is v2, then
2
exist invertible matrices P and Q such that

ou
1
B = PAQ -1. The binary relation ~ is (a) v1 = v 2 (b) v1 > v 2 (c) v1 < v 2 (d) v1 = v2
2
(a) neither reflexive nor symmetric
75. A poker hand consists of 5 cards drawn at

(Y
(b) reflexive and symmetric but not transitive
(c) symmetric and transitive but not reflexive random from a well-shuffled pack of 52 cards.
(d) an equivalence relation Then, the probability that a poker hand
consists of a pair and a triple of equal face
on
72. Let a, b denote the cube roots of unity other
n
values (for example, 2 sevens and 3 kings or
302
æ aö 2 aces and 3 queens, etc.) is
than 1 and a ¹ b. Let S = å (-1)n ç ÷ . Then,
èbø 6 23
pi

n=0 (a) (b)


4165 4165
the value of S is
1797 1
m

(a) either -2w or -2 w2 (b) either -2w or 2 w2 (c) (d)


4165 4165
(c) either 2w or -2 w2 (d) either 2w or 2 w2
ha

Category III
C

Directions (Q. Nos. 76-80) Carry two marks each, for which one or more than one options may be correct.
Marking of correct options will lead to a maximum mark of two on pro rata basis. There will be no negative
dy

marking for these questions. However, any marking of wrong option will lead to award of zero mark against the
respective question-irrespective of the number of correct options marked.
u

76. If u (x) and v(x) are two independent solutions of 77. The angle of intersection between the curves
St

the differential equation y = [|sin x| + |cos x|] and x2 + y2 = 10, where


d 2y dy [x] denotes the greatest integer £ x, is
+b + cy = 0, (a) tan-1 3 (b) tan-1(-3)
dx2 dx
(c) tan-1 3 (d) tan-1(1 / 3 )
then additional solution(s) of the given
differential equation is(are) ì x |1 - t | dt , x > 0
(a) y = 5u( x) + 8v( x) ïò
78. Let f (x) = í 0 1 . Then,
(b) y = c1{u( x) - v( x)} + c 2 v( x),c1 and c 2 are arbitrary ïx - , x£1
constants î 2
(c) y = c1u( x) v( x) + c 2u( x) / v( x), c1 and c 2 are (a) f( x) is continuous at x = 1
arbitrary constants (b) f( x) is not continuous at x = 1
(d) y = u( x) v( x) (c) f( x) is differentiable at x = 1
(d) f( x) is not differentiable at x = 1
20 WB JEE (Engineering) · Solved Paper 2014

79. If the circle x2 + y2 + 2 gx + 2 fy + c = 0 cuts the 80. For two events A and B, let P ( A ) = 0.7 and
2 2
three circles x + y - 5 = 0, P (B) = 0.6. The necessarily false statement(s)
x2 + y2 - 8x - 6 y + 10 = 0 and is/are
x2 + y2 - 4x + 2 y - 2 = 0 at the extremities of (a) P( A Ç B) = 0.35
their diameters, then (b) P( A Ç B) = 0.45
(a) c = - 5 (b) fg = 147 / 25 (c) P( A Ç B) = 0.65
(c) g + 2 f = c + 2 (d) 4f = 3g (d) P( A Ç B) = 0.28

)
be
Answers
Physics

Tu
1. (a) 2. (d) 3. (a) 4. (b) 5. (b) 6. (d) 7. (c) 8. (d) 9. (b) 10. (c)
11. (a) 12. (c) 13. (d) 14. (a) 15. (c) 16. (b) 17. (a) 18. (c) 19. (d) 20. (c)
21. (c) 22. (d) 23. (c) 24. (d) 25. (a) 26. (a) 27. (b) 28. (b) 29. (a) 30. (d)

ou
31. (d) 32. (c) 33. (d) 34. (b) 35. (a) 36. (b) 37. (c) 38. (d) 39. (c) 40. (a)
41. (a) 42. (b) 43. (c) 44. (c) 45. (a) 46. (c) 47. (c) 48. (d) 49. (d) 50. (b)
51. (a)

Chemistry
52. (a) 53. (b) 54. (d) 55. (b) 56.

(Y
(a) 57. (a,c) 58. (a,d) 59. (b,c,d) 60. (c,d)
on
1. (d) 2. (b) 3. (c) 4. (c) 5. (b) 6. (a) 7. (d) 8. (b) 9. (a) 10. (c)
11. (a) 12. (d) 13. (a) 14. (c) 15. (b) 16. (b) 17. (c) 18. (c) 19. (c) 20. (d)
pi

21. (c) 22. (c) 23. (b) 24. (a) 25. (b) 26. (a) 27. (a) 28. (a) 29. (c) 30. (b)
31. (b) 32. (a) 33. (d) 34. (b) 35. (b) 36. (d) 37. (d) 38. (d) 39. (c) 40. (a)
m

41. (b) 42. (a) 43. (a) 44. (b) 45. (b) 46. (c) 47. (a) 48. (a) 49. (b) 50. (d)
51. (c) 52. (b) 53. (c) 54. (b) 55. (c) 56. (b,d) 57. (b,c) 58. (a,b,d) 59. (a,b,d) 60. (b,c,d)
ha

Mathematics
1. (b) 2. (c) 3. (a) 4. (b) 5. (c) 6. (c) 7. (d) 8. (c) 9. (a) 10. (b)
C

11. (a) 12. (c) 13. (b) 14. (a) 15. (b) 16. (c) 17. (a) 18. (a) 19. (b) 20. (a)
21. (c) 22. (a) 23. (d) 24. (a) 25. (a) 26. (a) 27. (a) 28. (c) 29. (a) 30. (a)
dy

31. (c) 32. (a) 33. (d) 34. (a) 35. (c) 36. (d) 37. (a) 38. (c) 39. (b) 40. (d)
41. (c) 42. (a) 43. (c) 44. (a) 45. (d) 46. (c) 47. (b) 48. (b) 49. (b) 50. (c)
u

51. (c) 52. (c) 53. (b) 54. (a) 55. (a) 56. (b) 57. (a) 58. (a) 59. (c) 60. (d)
61. (a) 62. (c) 63. (a) 64. (c) 65. (a) 66. (c) 67. (c) 68. (d) 69. (d) 70. (a)
St

71. (d) 72. (a) 73. (b) 74. (b) 75. (a) 76. (a,b) 77. (a) 78. (a,d) 79. (d) 80. (c,d)
Hints & Solutions
Physics
1. Vector X of the form aA + bB 5. A A+A.B
A NOT OR Y
X = aA + bB
= a ( $i + $j - 2 k$ ) + b ( $i - $j + k$ ) A

)
AND
B A .B

be
X = i$ ( a + b ) + $j ( a - b ) + k$ ( -2a + b )
By absorption laws
A vector X is perpendicular to C, i.e., X ×C = 0
Y = A + A ×B
[ $i ( a + b ) + $j ( a - b )

Tu
or Y = ( A ×B ) + A = A ×( B + 1) = A ×1 = A
+ k$ ( -2a + b )]×[2$i - 3$j + 4k$ ) = 0
or 2 ( a + b ) - 3( a - b ) + 4( -2a + b ) = 0 6. We know that DU = nCv DT

ou
or 2a + 2b - 3a + 3b - 8a + 4b = 0 where DT = (T2 - T1)
or -9a + 9b = 0 or a = b T1 = 0° C = 273 K
a 1 T2 = 100° C = 373 K

(Y
or = or a : b = 1 : 1
b 1 n = 1 (monoatomic gas)
3
2. The combination of three charges in series R = 8.32 J mol -1K -1 = 1 ´ R ´ ( 373 - 273)
2
on
1 1 1 1
= + + 3
C C1 C2 C3 = 1 ´ ´ 8.32 ´ 100 = 3 ´ 8.32 ´ 50
2
1 1 1 1 6
pi

= + + Þ C = = 1.5 mF Þ 1248 kJ
C 3 6 6 4
or = 1.25 ´ 103 J
m

The charge of this circuit


q = CV = 1.5´120 7. Energy required to removed electron in the
ha

q = 180 mC ( n = 2) stage
The potential difference across the 3 mF The energy of the photon
q = CV æ1 1ö æ 1 1ö
C

= R ç 2 - 2 ÷ Þ 13.6 eV ç - ÷
q 180 è1 n ø è1 4ø
V = = = 60 V
C 3 æ 4 - 1ö 3 40.8
= 13.6 eV ç ÷ = 13.6 eV ´ = eV
dy

V 120 è 4 ø 4 4
3. Resistance R = -G = - 10
ig 0.01 = 10.2 eV
u

= 12000 - 10 8. We know that


St

R = 11990 W e 0×A
The parallel plate capacitor C =
To convert a galvanometer into a voltmeter, high d
resistance should connect in series. When d (distance between two parallel plate)
increase, then C will be decrease
4. For an open pipe, the frequency
Q Q = CV where Q = constant
v 340
f = Þ 5100 = \ The voltage across the capacitor increases.
2l 2´l
340 2 1 9. We know that
l= Þ l= = m 1
5100 ´ 2 30 ´ 2 30 Kinetic energy K = mu2
2
100 10
l= = cm dy
30 3 where u = = wa cos wt
dt
22 WB JEE (Engineering) · Solved Paper 2014

1 The time of flight of the ball


So, K = mw2a2 cos2 wt
2 2u sin q 2
T = = ( u sin q) [from Eq. (i)]
Hence, kinetic energy varies periodically with g g
double the frequency of SHM. So, when a particle é 1 ù
executing SHM oscillates with a frequency n, h + gt 2
2 ê 1 2 1 ú 2 é h1 t 12 ù
then the kinetic energy of particle changes = ê ú= ê + ú
g ê t1 ú t1 ë g 2û
periodically with a frequency of 2n.
ë û

)
Energy [ ML2 T -2] h1 2 h æ t t 2 - t 12t 2 ö
10. (a) Planck’s constant = =

be
Frequency [ T -1] = ´ + t1 = 1 ´ ç 1 2 ÷ + t1
t1 g t 1 è h1t 2 - h2t 1 ø
= [ ML2 T -1]
h1t 1t 22 - h1t 12t 2 + h1t 12t 2 - h2t 13

Tu
Angular momentum = Moment of inertia =
t 1( h1t 2 - h2t 1)
´ Angular velocity
æ h t 2 - h2t 12 ö
= [ ML2] ´ [ T -1] = [ ML2 T -1] =ç 12 ÷

ou
è h1t 2 - h2t 1 ø
(b) Impulse = Force ´ Time
= [ MLT -2] [ T] = [ MLT -1] 12. Stokes established that if a sphere of radius a
and linear momentum = Mass ´ Velocity
= [ M] [LT -1] = [ MLT -1] (Y moves with velocity v through a fluid of viscosity
h , the viscous force opposing the motion of the
sphere is F = 6phav
on
(c) Moment of inertia = Mass ´ ( Distance )2 13. Given, 305 X - 25 X = 100° C
2
= [ ML ] (Q X is the new unit of temperature)
pi

and moment of force = Force ´ Distance ( 305 - 25) X = 100° C ⇒ 280 X = 100° C
= [ MLT -2] [L] = [ ML2 T -2] \ 1° C = 2.8 X
m

(d) Energy = [ ML2 T -2] and torque = [ ML2 T -2] The specific heat capacity of water
joule joule
ha

So, (c) option has different dimensions. = 4200 = 4200 ´


kg ° C kg ´ 2.8 X
11. For vertically moment (for h1)
= 1500 J/kg-X
C

1 2
h1 = u sin qt 1 - gt 1 = 1.5 ´ 103 J kg -1X-1
2
14. For the cyclic process
dy

1 2
h1 + gt 1
2 p
or t1 = …(i)
u sin q p1
u

1
h2 = u sin qt 2 - gt 22 (for h2 )
St

2
1 2 p2
h2 + gt 2
or t2 = 2 ...(ii)
u sin q V1
V
V2
Divide Eq. (i) by Eq. (ii) Heat absorbed = Work done
1
h1 + gt 12 / u sin q 1
= Area = ( Dp ) ´ DV
t1 2
= 2
t 2 h + 1 gt 2 / u sin q 1
2 2
2 = ( p1 - p2 ) ´ (V1 - V2 )
2
g
⇒ h1t 2 - h2t 1 = ( t 1t 22 - t 12t 2 ) 1
2 = ( p1 - p2 ) (V1 - V2 )
2
WB JEE (Engineering) · Solved Paper 2014 23

16. We know that q q


AB = AO sin Þ AB = r sin
2 2 2
æ m - m2 ö
Acceleration, a CM = ç 1 ÷ ´ g (Q m1 > m2 ) AC = AB + BC (Q AB = BC )
è m1 + m2 ø
q q
So, resultant external force, = r sin + r sin
2 2
F = ( m1 + m2 ) a CM q
2 AC = 2r sin
æ m - m2 ö 2
= ( m1 + m2 ) ´ ç 1

)
÷ ´g
è m1 + m2 ø So, the magnitude of the corresponding linear

be
q
( m1 - m2 )2 displacement will be 2r sin .
= ´g 2
( m1 + m2 ) l1

Tu
20. l
17. If the same wedge is made rough then time
taken by it to come down becomes n times more. w1 w

ou
The coefficient of friction
Wl = W1l1 ...(i)
Block Block l1 l
d d
(Y w1 w–FB
on
Rough surface Smooth surface
45° 45° æ 1ö
W - F B = Wl ç1 - ÷
è rø
pi

é 1ù é 1ù
m = ê1 - 2 ú tan q Þ m = ê1 - 2 ú tan 45° where r = specific gravity
ë n û ë 2 û
æ 1ö
m

4 -1 3 Wl ç1 - ÷ = W1l2
m= Þm= è rø
4 4
ha

æ 1 ö W1l2
18. Strain = a×DT ç1 - ÷ = [from Eq.(i)]
è r ø Wl
Stress Y ∝ DT
C

æ 1 ö W1l2
where L = length of the rod ç1 - ÷ =
è r ø W1l1
a = coefficient of linear thermal expansion 1 l2 1 l
dy

Y = Young’s modulus of its material 1- = Þ =1- 2


r l1 r l1
So, the longitudinal stress developed in the rod
1 l1 - l2 l
u

DT = or r = 1
is directly proportional to . r l1 l1 - l2
Y
St

19. 2r sin q /2 21. Let, thickness of layer be x


B So, volume V = Area ´ x
A C
V =A´x (Q x = 2r )
q /2
r x =V / A
q V V
O \ 2r = ⇒ r= ...(i)
A 2A
T
and DP =
r
We know that F = DP ´ A
q AB T
In DAOB sin = (Q AO = r ) = ´A
2 AO r
24 WB JEE (Engineering) · Solved Paper 2014

T So, the maximum possible focal length of this


F = ´A [from Eq. (i)]
æV ö d
ç ÷ convex lens is .
è 2A ø 4
F ´V 25. The given
T =
2A2 A = a( i$ + 2$j + 3k$ )
where F = 16 ´ 105 dyne , V = 0.04 cm3 ,
B = a( i$ - 2$j + 6k$ )
16 ´ 105 ´ 0.04

)
A = 20 cm2 = AB = OB - OA

be
2 ´ 202 = a( i$ - 2$j + 6k$ ) - a( i$ + 2$j + 3k$ )
8 ´ 105 ´ 4 8 ´ 105 ´ 4
= = AB = a( -4$j + 3k$ )
202 ´ 100 400 ´ 100

Tu
æ s ö $
= 8 ´ 105 ´ 10 -4 = 80 dyne/cm Work done = q ç ÷ k×AB (along to Z-axis)
è 2e 0 ø
-1
= 80 dyne cm

ou
æ s ö $ 3qsa
=q ç ÷ k×a( -4$j + 3k$ ) =
22. Given, r = 0.05 nm = 0.05 ´ 10-9 m è 2e 0 ø 2e 0
(Q1 nm = 10 -9 m) (Q i$×i$ = $j ×$j = k$ ×k$ = 1 and i$×$j = $j ×k$ = k$ ×i$ = 0)
16
n = 10 revolutions
e = 1.6 ´ 10 -19 C
(Y
26. We know that
Intensity of magnetisation
on
We know that M
I =
The magnetic moment M = Ai V
pi

M = pr 2 ´ ne [where M = magnetic moment, V = volume]


12 1
M = 3.14 ´ ( 0.05 ´ 10 -9)2 ´ 1016 ´ 1.6 ´ 10 -19 So M = IV = 5.0 ´ 10 4 ´ ´
m

100 ( 100)2
M = 0.1256 ´ 10 -18 ´ 1016 ´ 10 -19
= 60 ´ 10 4 ´ 10 -6 = 0.6 Am2
ha

or M = 1.26 ´ 10 -23 Am2


27. We know that
23. We know that t = NBAw sin wt
C

1
where N = number of loops = 1 S nth = u + a ( 2n - 1)
m I 2
B = 0 newton/amp-m 1 5
dy

2b S3rd = 0 + a( 2 ´ 3 - 1) = a (for n = 3 s)
A = pa2 metre2 2 2
m I 1 7
t = 0 ( pa2 ) w sin wt
u

\ S 4th = 0 + a( 2 ´ 4 - 1) = a (for n = 4 s)
2b 2 2
St

pa2m 0 I So, the percentage increase


= ×w sin wt
2b
S 4th - S3rd
= ´ 100
24. Focal length of a convex lens by displacement S3rd
method
Focal length of the convex lens 7 5
a- a
a2 - b 2 = 2 2 ´ 100
f = 5
4a a
where a = distance between the image and object 2
and b = distance between two positions of lens 2a
Fact The distance between the two points should be = 2 ´ 100 = 2 ´ 20 = 40%
greater than four time the focal length of the convex 5
a
lens, i.e., d > 4f . 2
WB JEE (Engineering) · Solved Paper 2014 25

28. Given, Kinetic energy = E K total = K rot + K trans


Mass = m For solid spherical ball,
2
Magnetic field = B I = mR2 (along to diameter)
5
Charge = q
and v = Rw, where R is radius of spherical ball
We know that
1 æ2 ö 1
F = qvB sin q So, K total = ç mR 2 ÷ w2 + mR 2 w2
2 è5 ø 2

)
(motion of a charged particle in a uniform 7

be
magnetic field) = mR 2 w2
10
If q = 90°
7
Then F = qvB ...(i) K = mv 2

Tu
10
We know that also (centripetal force)
Potential energy = Kinetic energy
mv 2 7
F = ...(ii) mv 2

ou
r mgh =
10
From Eqs. (i) and (ii), we get 10
v2 = gh ...(i)

(Y
mv 2 mv é 1 2 2E ù 7
qvB = ,r = êQ E = mv , v = ú
r qB ë 2 mû For vertical projection,
v 2 = u2 + 2gh ¢
2E
on
m 10 5
m 2Em gh = 0 + 2gh ¢ Þ h ¢ = h
\ r= Þ r= 7 7
qB qB
pi

33. We know that


29. We know that The maximum intensities
The potential energy of the satellite I 2 )2
m

I max = ( I 1 + ...(i)
GM e m
U =- ...(i) The minimum intensities
ha

Re I min = ( I 1 - I 2 )2 ...(ii)
The kinetic energy of the satellite
So, the ratio of the maximum and minimum of
1 GM e m
C

K = ...(ii) intensities is
2 Re 2
I max ( I 1 + I 2 )
=
The total energy I min ( I 1 - I 2 )2
dy

GM e m 1 GM e m
E =U + K = - + I max ( 4I + I )2
Re 2 Re =
I min ( 4I - I )2
u

1 GM e m 2
E =- æ3 I ö
St

2 Re =ç ÷
GM e m è I ø
2E = - Þ -2E = U 9
Re = = 9 :1
1
So, PE = - 2 ( TE )
PE = - 2E 34. I initial = 0,
Vi 150 W +3V
30. Thermal radiation is the heat waves travel along
straight lines with the speed of light.
31. The band gap of 5 eV corresponds to that of an 3
insulator. = 0.02 A
I final =
150
32. We know that total kinetic energy of a body So, change in I = 0.02 A = 20 mA
rolling without slipping
26 WB JEE (Engineering) · Solved Paper 2014

é nh ù 40. Total emf of the cell = 3E - E = 2E


35. ê ú
ë 2pqB û E E E E
where n and 2p dimensionless quantity
So, [ h] = [ mvr ]
é qvB ù é F ù
[qB] = ê =
ë v úû êë v úû Total internal resistance = 4r
é mvr ù [ mvr ] [v ]

)
\ \ Total resistance of the circuit = 4r + R
êë F / v úû = [ F ]

be
æ Vö
So, the current in the external circuit çQ i = ÷
[ mv 2 r ] è Rø
= = [ r 2] = Area
é mv 2 ù 2E

Tu
ê ú \ i =
4r + R
ë r û
41. We know that

ou
36. I B is in m amp, I C is in m amp and VCE in volt.
Relation between temperature gradient (TG)
37. Given, Y = 4 cos æç ö÷ × sin ( 1000t )
t2 and thermal conductivity ( K )
è2ø

(Y
dq dq
So, = - KA = - KA ´ ( TG )
æt ö dt dx
= 2 ´ 2 cos2 ç ÷ × sin ( 1000t )
è2ø 1 æ dq ö
i.e., TG ∝ ç = constant ÷
on
é 2 t ù K è dt ø
êëQ 2 cos 2 = ( 1 + cos t ) úû 1
or K ∝
= 2( 1 + cos t ) sin ( 1000t ) T
pi

= 2 sin ( 1000t ) + 2 cos t × sin ( 1000t ) 1


K1 ∝ ...(i)
m

= 2 sin ( 1000t ) + 2 sin ( 1000t ) × cos t t1


= 2 sin ( 1000t ) + sin ( 1000t + t ) 1
K2 ∝ ...(ii)
ha

+ sin ( 1000t - t ) t2
[Q2 sin A × cos B = sin ( A + B ) + sin ( A - B )] From the Eqs. (i) and (ii), we get
K 1 1 / t1
C

= 2 sin ( 1000t ) + sin ( 1001t ) + sin ( 999t )


=
So, n = 3 K 2 1 / t2
K 1 : K 2 = t2 : t1
dy

38. Assuming temperature of the body and cubica


box is same initially, i.e., T and finally it 42. Isotope of the original nucleus is produced.
becomes T / 2.
u

Because, temperature of body and surrounding 43. By Doppler’s Effect


St

remains same. When observer is moving with velocity v 0 ,


Hence, no net loss of radiation occur through the towards a source at rest then approach frequency
body. This total energy remains constant. æv + v 0 ö
N Approach = N ç ÷
è v ø
39. Let, the charge on the inner shell be q ¢
where N = 850 Hz, v = 340 ms -1, v 0 = 72 kmh-1
The total charge will be zero
kq ¢ kq æ 340 + 20 ö
So, + =0 (Q r2 > r1) = 20 ms -1 = 850 ç ÷
r1 r2 è 340 ø
Kq / r2 Similarly, when observer is moving away from
⇒ q¢ = - source, then
K / r1
æv - v 0 ö æ 340 - 20 ö
ær ö N Separation = N ç ÷ = 850 ç ÷
q¢ = - ç 1÷ q è v ø è 340 ø
è r2 ø
WB JEE (Engineering) · Solved Paper 2014 27

The different of the two frequency, From Eqs. (ii) and (i), we get
N Approach - N Separation E photon hc / l hc × l2 ´ 2m
= 2 =
æ 360 ö æ 320 ö K electron h / 2m × l2 h2 × l
= 850 ç ÷ - 850 ç ÷
è 340 ø è 340 ø
where m = 0.5 MeV = 5 ´ 105 eV
850 850
= ´ 40 = = 100 Hz h 2mlc
340 8.5 = 50 ´ 103 eV =
lc h
44. E g ³ 100 keV 2 ´ 5 ´ 105

)
2m æ hö
= = çQ m = ÷

be
E X = 100 eV to 100 keV h / lc 50 ´ 103 è cl ø
E v = 2.48 eV = 20 : 1

Tu
So, we can say that 48. By Snell’s law,
E g > E X > Ev m sin i = constant
45. The intermediate image formed by the objective m sin i = ( m - m Dm ) sin r

ou
of a compound microscope is real, inverted and where m = 1.5, i = 30°, r = 90°, Dm = 0.015
magnified.
1.5 sin 30° = ( 1.5 - m ´ 0.15) sin 90°
46. Let, the final velocity be v

v0
(Y 3 1
´ = ( 1.5 - m ´ 0.015) ´ 1
2 2
3 3 15m
= -
on
4 2 1000
So, angular momentum will remain conserved æ3 3ö
15 m = ç - ÷ ´ 1000
è2 4ø
pi

along point of contact


By conservation of angular momentum 6 -3 3 1000
15m = ´ 1000 Þ m = ´
m

Angular momentum will remain conserved along 4 4 15


point of contact 3000
m= Þ m = 50
ha

Iw = constant 60
2 æ vö
mv 0 r = mvr + mr 2 ´ w çQ w = ÷ 49. We know that
5 è rø
C

2 æv ö
mv 0 r = mvr + mr 2 ç ÷
5 èrø P
dy

2 90°
v0 = v + v
5 30°
d3
u

60° 2
7 5
v0 = v Þ v = v0
St

5 7 60°

h ém i ù
47. de-Broglie wavelength l = Bnet = 2 ê 0 (sin q1 + sin q2 ) ú
2mK ë 4 pr û
The kinetic energy of the electron é ù
êm ú
1 h2 =2 ê 0 ´
i
´ (sin 90° + sin 30° ) ú
K electron = × ...(i)
2m l2 ê 4p d 3 ú
where h = Planck constant êë 2 úû
l = wavelength ém 2i æ 1ö ù
=2 ê 0 ´ ´ ç1 + ÷ ú
The photon energy ë 4 p d 3 è 2 øû
hc ém 2i 3ù 3 m 0i
E photon = ...(ii) =2 ê 0 ´ ´ ú=
l ë 4p d 3 2 û 2 pd
28 WB JEE (Engineering) · Solved Paper 2014

50. 1 W, 2 W and 1 W are in parallel w 1 r1 r2 - w 2 r1 r2 = xr2rw + w 1rw r1 - xrw r1


E1=1V x ( r2 - r1) rw = r1[w 1( r2 - rw ) - w 2r2]
r1
x = [w 1( r2 - rw ) - w 2r2]
E2=2V rw ( r2 - r1)
P E3=3V Q
53. m1s1Dt + m2s2Dt = Work done
So, the required internal resistance m1s1Dt + m2s2 Dt = P1t 1

)
1 1 1 1 where m1 = 0.5 kg

be
= + +
r r1 r2 r3 Specific heat s1 = 4200 J/kg-K
1 1 1 1 Dt = Dt 1 = Dt 2 = 3 K
= + +

Tu
r 1 2 1 P1 = P2 = 10 W
1 2 + 1+ 2 2 t 1 = 15 ´ 60 = 900 s
= Þ r= W
r 2 5 s2 = Specific heat capacity of container

ou
The potential difference between points P and Q 0.5 ´ 4200 ´ ( 3 - 0) + m2s2 ´ ( 3 - 0)
E1 E2 E3 1 2 3 = 10 ´ 15 ´ 60
+ + + +

(Y
r1 r2 r3 2100 ´ 3 + m2s2 ´ 3 = 9000
E diff = =1 2 1
1/ r 5/2 9000 - 6300
m2s2 =
2+2+6 3
on
2 10 / 2 5 m2s2 = 900
= = = ´2=2 V
5/2 5/2 5 Similarly, in case of oil
51. Path difference = ( pr - 2r ) m1s 0 Dt + m2s2 Dt = P2t 2
pi

where s 0 = specific heat capacity of oil


m

P1 = P2 = 10 W
2 ´ s 0 ´ 2 + 900 ´ 2 = 10 ´ 20 ´ 60
ha

4s 0 + 1800 = 12000
= ( p - 2) r = nl (wheref = frequency) 4s 0 = 12000 - 1800
10200
C

v =f ´l s0 = = 2550
4
v é v ù
⇒ =f ⇒ f =ê ún = 2.55 ´ 103 J kg -1K -1
dy

l ë ( p - 2 ) r û
é v ù 54. For the first condition
So, multiples integral = ê ú Screen
u

ë ( p - 2) r û Object Convex lens image


St

52. By Archimedes’ Principle


F = v rw g Þ ( w 1 - w 2 ) g = v rw g
Let, the total volume be v and first metal weight 30 cm v
be x
w 1 - w 2 = ( v 1 + v 2 ) rw
æ mö 1 1 1 1 1 1
w 1 - w 2 = v 1rw + v 2rw çQ v = ÷ = - Þ = +
è rø f1 v u 10 v 30
æx w -x ö 1 1 1
w 1 - w 2 = ç rw + 1 rw ÷ = -
è r1 r2 ø v 10 30
30
w1 - w2 =
xr2rw + ( w 1 - x ) rw r1 ⇒ v = = 15 cm
r1r2 2
WB JEE (Engineering) · Solved Paper 2014 29

where f1 = 10 cm, u = - 30 cm dQ dT 7
\ = nC p ´ Þ i 2 r = R ´ [ a + bt ]
v =? dt dt 2
Screen Screen 7R
Object i = ( a + bt )
image image 2r
pV = nRT
nRT nR æ 1 2ö
or V = = ç at + bt ÷
15 cm 45 cm p p è 2 ø

)
30 cm

be
nR æ 1 2ö
X = ç at + bt ÷
pA è 2 ø
For the second condition when concave lens is
placed nR

Tu
v = ( a + bt )
v ¢ = ( 15 + 45) cm = 60 cm pA
1 1 1 nR
= - and acceleration = ´b

ou
F v¢ u pA
(where F = focal length of combination) So, the rate of increase in internal energy is
1 1 1 5
R ( a + bt ) and the piston moves upwards with

(Y
\ = + 2
F 60 30
60 constant acceleration.
F = cm = 20 cm
3 57. Loss in potential energy = gain in kinetic energy
on
The magnitude of focal length of concave lens
q
1 1 1 1 1 1
= + Þ = + Þ f2 = - 20 cm
pi

F f1 f2 20 10 f2
L sin q
(Negative sign for concave lens) 2
m

55. We know that


ha

1 2 q
K = Iw
2 L cos q
where K = kinetic energy 2
C

CM
I = moment of inertia mg
w = angular speed
dy

1 L 1
So, w∝ ⇒ mg sin q = Iw2
I 2 2
w ∝ sin q
u

1 1 1 So,
w1 : w2 : w3 = : :
St

I1 I2 I3 æ 1 2ö
and v ∝ sin q çQ K = Iw ÷
1 1 1 è 2 ø
= : : = 2 : 2 :1
1 1 2 The speed of end B is proportional to sin q
56. We know that and
nfRT 5R æ 1 2ö L
Internal energy, U = = ç at + bt ÷ U = mgh = mg ( 1 - sin q) (Q t = Ia )
2 2 è 2 ø 2
Differentiate with respect to t L mI 2
⇒ mg cos q = ´a
dU 5R 2 3
= [ a + bt ]
dt 2 a ∝ cos q
But, dQ = nC pdT The angular acceleration is proportional to cos q.
30 WB JEE (Engineering) · Solved Paper 2014

58. There is no significant time delay between the KQ Q


Q1 = and Q2 =
absorption of a suitable radiation and the K +1 K +1
emission of electrons and the maximum kinetic s E s K
energy of electrons does not depend on the So, E = Þ 1 = 1 ´ 1
e 0K E2 s2 1
intensity of radiation.
Q1 K 2 K 1
So, (a) and (d) are correct options. = ´ = ´
Q2 K 1 1 K
59. We know that
= 1:1

)
Ke 0 A e A
, C2 = 0

be
C1 = So, (b), (c) and (d) are correct options.
2d 2d
e×A 60. Neither electrons nor protons will go through
and C eq = ( K + 1)
the slit irrespective of their speed.

Tu
2d
C
C eq = 0 ( K + 1) Eq qvB
2 +
qvB

ou
Eq
Q 1 C1 K
= =
Q 2 C2 1 And electrons will always be deflected upwards
s1 K irrespective of their speed.
⇒ =
s2 1
(Y So, (c) and (d) are correct options.
on
Chemistry
1. In 1885, Balmer for the first time showed that and h = 6.6 ´ 10 -34 J - s
pi

the wave numbers of spectral lines present in = 6.6 ´ 10 -27 erg-s


the visible region in hydrogen spectrum are
On substituting values, we get
m

given by
6.6 ´ 10 -27
æ 1 1ö l= = 6.6 ´ 10 -31 cm
n ( cm-1) = 109677 ç 2 - 2 ÷ 100 ´ 100
ha

è ( 2) n ø
Here, n = 3, 4, 5, K 4. Ideal gas equation is
nRT
C

Thus, Balmer spectrum of hydrogen was pV = nRT or V = +C


discovered first and it lies in the visible region. p
RT
dy

2. Atomic number of Cu is 29. or V = +0 (for 1 mol)


p
Thus, its electronic configuration can be written as
Cu29 = 1s2, 2s2, 2p 6, 3s2, 3p 6, 3d 10, 4s1 When V vs T is plotted, a straight line is
u

obtained, slope of which is given by R / p . Thus,


(Since, fully filled orbitals are more stable, so one
St

R R
electron from 4s orbital transfers into 3d orbital.) Slope = Þ X =
p 2
The electronic configuration of neon is
or R = 2 X L atm mol -1 K -1
Ne10 = 1s2, 2s2, 2p 6
Thus, using inert gas, the configuration of Cu 5. According to Graham’s law of diffusion,
can also be represented as 1 V
Rate of diffusion, r ∝ =
Cu29 = [ Ne] 3s2, 3p 6, 3d 10, 4s1 M t

3. According to de-Broglie, [Here, M = molecular mass,


h V = volume and t = time]
wavelength, l = Thus, for H2 gas,
mv
200 1
Given, m = 100 g, v = 100 cm s -1 = ...(i)
30 2
WB JEE (Engineering) · Solved Paper 2014 31

For O2 gas, 9. When a positron is emitted, a proton is


50 1 converted into neutron and neutrino. Thus, the
= ...(ii)
t 32 number of protons and hence the atomic
From Eqs. (i) and (ii), we get number decreases by one unit.
1 1 0
200 ´ t 32 1H ¾® 0n + +1e + n
= Proton Neutron Positron Neutrino
30 ´ 50 2
10. During the b- emission, a neutron gets
16 ´ 30 ´ 50

)
t = converted into a proton along with the formation

be
200 of antineutrino and electron.
4 ´ 30 1 1 0
= = 30 min 0n ¾® 1H + -1e + n + g - ray
4 Anti
Proton

Tu
b
particle neutrino
6. Me Me 1
6 7 2 Me 11. Higher the value of a, more will be the tendency
5 to get liquefy. Since, value of a is highest for P1

ou
Me 3
4 thus, P is the most liquefiable gas among the
The longest chain contains 7C-atom, so the root given.

(Y
word is hept and suffix -ene is added for double
12. Resonating structures involve movement of
bond along with its position. Since, two methyl
electrons, but they do not involve movement of
groups are also present at 5 and 6 positions, so
atom or group of atoms.
the correct IUPAC name of the compound is
O– O
on
O O
5,6-dimethylhept-2-ene.
Me Me
O – O
7. Me + NaNH2 – NH3 Me C—Na Me Me
pi

CH3I I II
H OH HgSO4
O–
m

Me C—CH3 O
dil H2SO4
Me C C—CH3 (Addition of Me
water) O Me
ha

Tautomerisation
III
Me—CH2—C—CH3 or Me Me Thus, structure IV is not a resonating structure
C

O of others.
O
13. Symmetrical molecules and symmetrical
(Here, Me means methyl group)
dy

trans-alkene have a net dipole moment zero.


Thus, the correct set of reagents is CH2Cl2 is not a symmetrical molecule, thus it
NaNH2 / CH3I; HgSO 4 / dil H2SO 4. will have a permanent dipole.
u

m3
8. H¾ C¾H bond angle is about 120°, when C is H Cl Cl H
St

m1
bonded through a double bond but in case of
CH 4, the H¾ C¾H bond angle is only 109º 28¢. Cl
Hm H
Cyclic compounds have a H¾ C¾H bond angle 2 m 4Cl
greater than 109° 28¢ due to hindrance. Symmetrical
m 1+m 2– m 3– m 4=0 trans-alkene
Thus, the correct order of H¾ C¾H bond angle m 3 and m 4 > m 1 and m 2 m net = 0
is Thus, m net =0
II > I > III m3
Cl Cl
H m1 Br Br
H H
\ 120° 109.5° 115°
Cl m m net =0
H H H m 4Cl
2
m 1+m 2 = m 3+ m 4
\ m net=0
32 WB JEE (Engineering) · Solved Paper 2014

14. The length of carbon-carbon single bond is 18. Given, rate = k [H + ] n


always greater than that of the carbon-carbon
Initial pH = 3 so [H + ] = 1 ´ 10 -3 , initial rate = r1
double bond.
In III, positive charge is not in conjugation of Final pH = 1 so [H + ] = 1 ´ 10 -1,
double bond, so the bond does not acquire a Final rate = r2 = 100r1
double bond character. However, chances of On substituting values, we get
acquiring double bond character (of the r1 = k [1 ´ 10 -3] n ...(i)
indicated bond) is much more in I than that in

)
-1 n
r2 = 100r1 = k [10 ] ...(ii)

be
case of II.
Thus, the correct order of decreasing bond Dividing eqs (i) by (ii),
length is n
é 1 ´ 10 -3 ù

Tu
r1
+ =ê ú
> + + > + 100r1 ë 1 ´ 10 -1 û
III II I 1 é 10 -2 ù
n 1 n
é 1 ù é 1 ù

ou
(No double bond (Both the positive (Readily acquire =ê ú Þ ê =ê
character) charge have equal double bond
character)
100 ë 1 û ë 100 úû ë 100 úû
tendency to impart
double bond character) \ n =1

15. Emission of a positron results in the conversion


of a proton into neutron, along with the release (Y Thus, the reaction is of first order.
19. Given, DH = - 400 kJ mol -1
on
of neutrino. Thus, the number of protons DS = - 20 kJ mol -1 K -1
decreases and of neutron increases, i.e., n/p
ratio increases and the nuclei becomes stable. Gibbs Helmholtz equation is
pi

1 1 0 DG = DH - TDS
1H ¾® 0n + +1e +n
Proton Neutron Positron For a reaction to be spontaneous, DG must be
m

16. 76
and 34Se76 have the same mass number less than 0, i.e., negative.
32 Ge
\ 0 ³ DH - TDS
but different atomic number, so they are isobars.
ha

T DS £ DH
Number of neutrons,
30 DH -400
in 14Si = 30 - 14 = 16 T £ £ £ 20 K
C

DS -20
32
in 16S = 32 - 16 = 16
20. 1 R
Because of the presence of same number of
dy

OH
neutrons, 14Si30 and 16S32 are called isotones. S 2-R, 3-R configuration, so it
H 2
3 is a chiral compound.
y 2
17. Let, the unknown radioactive substance be x A . H
u

3
y
A + 6C 12 246 1 OH
¾® 98 Cf + 0n
St

x 1

On comparing atomic numbers, we get 1 R


OH 2-R, 3-S configuration, i.e., it
x + 6 = 98 + 0 S
2
has plane of symmetry or in
x = 98 - 6 = 92 3 other words, rotation of half
3 2
On comparing mass numbers, we get H part is cancelled by the other
OH half, so it is not a chiral
y + 12 = 246 + 1 1
y = 247 - 12 compound.
= 235 S
OH
Since, uranium has the atomic number 92, thus, S
2-S, 3-S configuration, so it
the given unknown substance was 92 U 235 . is also a chiral compound.
OH
WB JEE (Engineering) · Solved Paper 2014 33

21. In reaction M, there is only an intermediate step, 25. Entropy of vaporisation ( DS )


so it is a fast reaction. Further, in reaction M enthalpy of vaporisation ( DH )
energy of product is much lesser than that of the =
boiling point (in K)
reactant, as compared to as shown in reaction N.
Thus, reaction M is faster and more exothermic Given, DH = 24.64 kJ mol -1 and
than reaction N. boiling point = 35 + 273 = 308 K
22. Aldehydes lacking a-H atom, undergo 24.64 ´ 103 J mol -1
\ DS =

)
Cannizzaro reaction if treated with concentrated 308 K

be
alkali (NaOH), i.e., one mole of the aldehyde is = 80 JK -1mol -1
reduced to alcohol and other mole is oxidised to
salt of acid. Thus, the given aldehyde (glyoxal) 26. Given,

Tu
undergoes intramolecular Cannizzaro reaction, C + O2 ¾® CO2; DH ° = - x kJ ...(i)
i.e., its half part is oxidised and half is reduced.
2CO + O2 ¾® 2CO2; DH ° = - y kJ ...(ii)
Reduced part

ou
Required equation is
CHO 1.NaOH CH2OH H+ CH2OH 1
C + O2 ¾® CO, DH°1 = ?
CHO COONa CO2H 2
Oxidised part

23. If Cl2 is passed through hot aqueous NaOH, it


(Y On reversing eq. (ii), we get
2CO2 ¾® 2CO + O2; DH °2 = + y kJ ...(iii)
on
Dividing eq. (iii) by 2 gives
results in the formation of sodium chloride and 1 y
sodium chlorate as CO2 ¾® CO + O2; DH °3 = + kJ ...(iv)
2 2
pi

6NaOH + 3Cl2 ¾® 5NaCl + NaClO3 + 3H2O On adding eq. (i) and (iv), we get (required
Hot
In NaCl, let oxidation state of Cl be x. equation)
m

1
+1 + x = 0 or x = -1 C + O2 ¾® CO; DH °1 = DH ° + DH °3
2
ha

In NaClO3 , let oxidation state of Cl be y.


æ yö
+1 + y + ( -2 ) 3 = 0 = ç - x + ÷ kJ
è 2ø
y -5 =0
C

y - 2x
\ y = +5 = kJ
2
Thus, the obtained compounds have Cl in -1 and
dy

+
+5 oxidation states. 27. PPh3 +
Bu– –
PPh3
24. ‘10 V H2O2’ means 1 L of this solution will H
u


produce 10 L O2 at STP. CH2==O
St

2H2O2 ¾® 2H2O + O2 PPh3


68 g 22.4 L at STP O
Q 22.4 L of O2 is obtained from H2O2 =68 g
\ 10 L of O2 will be obtained from 28. Only carboxylic acids and their derivatives like
68 anhydride are capable to give effervescence with
H2O2 = ´ 10 = 30.36 g aqueous NaHCO3 solution, because they are
22.4
more acidic as compared to H2CO3 (carbonic
\ 1000 mL of the given solution contains 30.36 g acid).
H2O2 and 100 mL of the given solution contains
II CH3COOH + NaHCO3
30.36 ´ 100
= 3.03 g H2O2 ¾® CH3COONa + H2O + CO2
1000
I (CH3CO)2O + 2NaHCO3
Thus, % strength of H2O2 is 3.03 ( » 3).
¾® 2 CH3COONa + H2O + CO2
34 WB JEE (Engineering) · Solved Paper 2014

29. The molecular formula of ethane is C2H 6. To get Thus, the order of amount of electrolytes
its fourth homologue, we have to add required is in the order
4 ´ CH2 = C 4H 8 to it. NaNO3 > Ba(NO3 )2 > Al2(NO3 )3
C2H 6 + C 4H 8 = C 6H14 Note In paper, the formula of aluminium nitrate
C 6 H14 is the molecular formula of hexane. is incorrect. Correct formula of aluminium nitrate
is Al2(NO3 )3 .
Thus, hexane is the fourth homologue of ethane.
5
30. The given statement is true as electrons are 34. DH = - nC p DT = - 2 ´ R ( 225 - 125)

)
2

be
shifted more towards the more electronegative
= - 5R ( 100) = - 500R
atom.
31. Hydrides of N, O and F because of the small size 35. Since, the amine gives white precipitate with

Tu
and high electronegativity of elements, have benzene sulphonyl chloride which is insoluble in
ability to form extensive intermolecular (i.e., aq. NaOH, so it must be a secondary amine (i.e.,
between two molecules) hydrogen bonding. amine having RNHR group).

ou
Thus, a large amount of energy is required to
Among the given compound, only
break these bonds, i.e., the melting and boiling Me
points of hydrides of these elements are N Me is a secondary amine, so it

(Y
abnormally high. ½
H
32. From the Faraday’s IInd law, we know that represents the structure of given amine.
E
on
W ∝ Zit, W∝ it 36. Combination of four amino acid units gives a
96500 linear tetrapeptide, in which three peptide
1 1 bonds are present.
or it ∝ or electricity ∝
pi

E E O H CH3
Thus, the amount of electricity required for ½½ ½ ½
m

ZnSO 4, AlCl3 and AgNO3 is in the ratio of H2 N ¾ CH2 ¾ C ¾ OH + H ¾ N ¾ CH ¾COOH


2 : 3 : 1. (Here, 2, 3 and 1 are the valencies of O O
ha

metal atom in the given compounds). ½½ ½½


Alternatively +H2 N ¾ CH2 ¾ C ¾ OH + H2 N ¾ CH2 ¾C ¾ OH
In ZnSO 4, O H CH3 O H
C

Zn2 + + 2e - ¾® Zn ½½ ½ ½ ½½ ½
1 mol 2 mol H2 N ¾ CH2 1
¾4C2 N3
¾4 ¾ CH ¾ C4¾2N4
¾ CH2
or 2 Faraday 1 3
Peptide bond Peptide bond
dy

In AlCl 3 ,
Al3+
+ 3e -
¾® Al O O
1 mol 3F ½½ ½½
¾ C ¾ N ¾CH2 ¾ C ¾ OH
u

In AgNO 3 , ½
St

Ag + + e - ¾® Ag 1424 H
3
1 mol 1F
Peptide bond
Thus, the ratio of amounts of electricity required (Tetrapeptide)
is 2 : 3 : 1 37. Among the given gases, N2O, CO2 and CH 4 have
33. Since, AgI is a negatively charged sol, so the ability to trap thermal radiation but O2 does
not. That’s why it is not a green house gas.
according to Hardy Schulze law, higher the
positive charge on the metal atoms, more is the Note O2 is the gas, presence of which is
tendency of metal atom to cause coagulation of essential for life and in absence of which life is
negatively charged colloid. not possible.
The order of positive charge is 38. 10-4 M KOH = 10-4 M [OH -]
Na + < Ba2 + < Al3 + We know that
[H + ] [OH - ] = 1 ´ 10 -14
WB JEE (Engineering) · Solved Paper 2014 35

[H + ] ´ 10 -4 = 1 ´ 10 -14
1 ´ 10 -14 O
[H + ] = = 1 ´ 10 -10 M
1 ´ 10 -4 5 CH2
O Base-I
+ -10 4
\ pH = - log [H ] = - log ( 1 ´ 10 ) H H 1

= 10 H H
3 2
mass O OH
39. Number of atoms = ´ NA

)
molar mass HO—P==O

be
´ Number of atoms in 1 mole
\ Number of atoms in 4.25 g NH3 O

Tu
4.25 5CH
2 O Base-II
= ´ NA ´ 4
17 4 1
H H
= NA H H

ou
3 2
Number of atoms in 8 g
O OH
8 N
O2 = ´ NA ´ 2 = A

(Y
32 2 HO—P==O
Number of atoms in 2 g O
2
H2 = ´ N A ´ 2 = 2 N A
on
2 44. Aniline is a primary amine (contains ¾NH2
Number of atoms in 4 g group). When it is treated with chloroform under
4 alkaline conditions, it results in the formation of
pi

He = ´ N A ´ 1 = N A a bad smelling compound, known by the name


4
isonitrile or carbylamine. This reaction is known
m

Thus, 2 g H2 contains the maximum number of


as carbylamine reaction.
atoms among the given.
NH2
ha

40. SO2- 2-
3 and SO 4 when treated with BaCl2 , give
white ppt of BaSO3 and BaSO 4 respectively. + CHCl3+Alc. KOH
BaCl2 + SO23 - ¾® BaSO3 + 2Cl -
C

Aniline
NC
Barium
sulphide
BaCl2 + SO24- ¾® BaSO 4 + 2Cl - +KCl+H2O
dy

Barium sulphate
Phenyl
Out of these two, SO2- 2-
3 is soluble in HCl but SO 4 isonitrile
u

does not.
St

41. Eating animal poisoned with mercury ( Hg 2+ ) 45. Species with more negative E ° (standard
causes deformity, known as minamata reduction potential) generally acts as reducing
(minimata) disease which is characterised by agent while with less negative E ° acts as
diarrhoea, hemolysis, impairment of various oxidising agent. Thus, the overall reaction is
senses, meningitis and death. Ag + + Fe2+ ¾® Fe3+ + Ag
42. H3PO2 is the reagent that converts ¾ N+
2 Cl
-
DE ° = E ° OA - E °RA
group into H. During this process N2 and HCl
gases are released. = - 0.3995 - ( -0.7120) V
43. In DNA, the consecutive deoxynucleotides are = - 0.3995 + 0.7120 V
linked through phosphodiester linkage. = + 0.3125 V
36 WB JEE (Engineering) · Solved Paper 2014

46. van der Waals’ equation is Addition of proton at any other carbon atom,
æ a ö interupt in the delocalisation of p electrons by
ç p + 2 ÷ (V - b ) = RT (for 1 mole) disturbing planarity of molecules and hence,
è V ø
a makes it less stable. Thus, these all are less
If a is negligible, p + 2 » p reactive towards protonation.
V 3 4
p (V - b ) = RT Þ pV - pb = RT Me 1 Me
2 5 H+ +
pV pb pV pb Me 6
Me
or - =1 Þ =1+ 8 H

)
7
RT RT RT RT Aromatic

be
47. In acidic medium, phenol exists in following 49. Abstraction of ( H a ) hydrogen atom results in the
resonating structures. formation of 3° allylic carbocation whereas

Tu
OH + + abstraction of Hc gives 2° allylic carbocation
O—H O—H (which is less stable as compared to former one).
– However, abstraction of H b gives only a

ou
2° carbocation which is less stable as compared
to former two. More the stability of generated

carbocation, higher is the tendency of

(Y
+
O—H OH abstraction of hydrogen.
Thus, the order of decreasing ease of abstraction

of hydrogen atoms from the given molecule is
on
H a > Hc > H b
l°c 150
Since, o/p positions are electron rich sites, so 50. Degree of dissociation, a = = = 0.3
pi

electrophile will attack on these site, i.e., l° m 500


hydrogen of these sites get exchanged by Given, C = 0.007 M
m

D (deuterium). Hence, the final product of the Hydrofluoric acid dissociates in the following
OD manner
ha

D D HF ¾® H + + F -
reaction is which is formed as C 0 0 Initially
C

H H C -Ca Ca Ca At time t
= C(1 - a)
D
Dissociation constant,
dy

OH O O
[H + ] [F - ] C a × Ca Ca2
H H D Ka = = =
D2SO4 [HF ] C (1 - a) (1 - a)
u

6 H D
D2O On substituting values, we get
St

0.007 ´ ( 0.3)2 63 ´ 10 -3 ´ 10 -2
OD OD Ka = =
( 1 - 0.3) 0.7
D D D
Repeat at = 9 ´ 10 -4 M
6 the o and
p-position 51. Elevation of boiling point,
D w ´ K b ´ 1000
DTb =
M ´W
48. Addition of a proton at C1 results in the
(Here, w and W = weights of solute and solvent
formation of tropylium carbocation (an aromatic
respectively,
species with more stability due to delocalisation
of p electrons), thus, it is the most reactive site M = molecular weight of solute and
towards protonation. K b = constant)
WB JEE (Engineering) · Solved Paper 2014 37

On substituting values, we get 2.303 ´ 5770 æ 15 ö


\ t = log ç ÷
w ´ 0.5 ´ 1000 0.693 è5ø
0.05 =
100 ´ 100 = 19175.0 log 3 yr
0.05 ´ 100 ´ 100 = 9148 yr » 9200 yr
or w = = 1g
0.5 ´ 1000
F
55. In XeF6 , Xe, the central atom F
52. Molarity F
contains, 8 valence electrons. Out

)
mass ´ 1000
= of which 6 are utilised with fluorine F Xe

be
molecular weight ´ volume of solution in bonding (i.e., it contains six bond
F
Molecular weight of ethyl alcohol, pairs of electrons) while one pair F
remains as lone pair.

Tu
C2H5OH = 12 ´ 2 + 5 + 16 + 1
= 24 + 22 = 46 g mol -1 Thus, the total pairs = 6 + 1 = 7
Hence, its shape is pentagonal bipyramid
On substituting the values, we get

ou
according to VSEPR theory.
mass ´ 1000
0.5 M =
46 ´ 100 56. Given, molecular weight of X = M X

(Y
0.5 ´ 46 Mol. wt. of Y = M Y and M Y > M X
Mass = = 2.3 g
10 Root mean square velocities = C X and CY
\ Volume of ethyl alcohol required, respectively
on
mass of ethyl alcohol 2.3 g Average KE/molecule = E X and E Y respectively
V = = = 2 cc
density of ethyl alcohol 1.15 g / cc We know that,
Root mean square velocity,
pi

53. In NF3, N is less electronegative as compared to


3RT 1
F but in NH3 , it is more electronegative than H. C= or C ∝
m

M M
And in case of same central atom, as the
electronegativity of other atoms increases, bond CX M
\ = Y
ha

angle decreases. Thus, bond angle in NF3 is CY MX


smaller than that in NH3 because of the Since MY > M X
difference in the polarity of N in these
C

\ C X > CY
molecules.
3
KE/molecules = nRT
2
dy

3
Þ E X ¹ EY ¹ RT
N N 2
u

F F H H 3
Further, EX = E Y = k BT
St

F H 2
54. Age of archaeological sample, 57. For a process to be spontaneous,
2.303 æN ö DS system + DS surrounding > 0
t = log ç 0 ÷
k è N ø ( DG system )T , p < 0
(Here, N 0 = initial activity, ( DU system )T , V < 0
N = remaining activity) 58. Since, H2SO 4 and H3PO 4 both are strong acids so
0.693 their presence makes the medium more acidic.
and k=
t 1/ 2 The Fe3+ ions form complex, [FeHPO 4]+ with
(where t 1/ 2 = half-life) these acid and hence, the concentration of Fe3+
0.693 -1 ion reduces which results in lowering in the
= yr
5770 formal reduction potential.
38 WB JEE (Engineering) · Solved Paper 2014

59. Electronic configurations of cuprous ( Cu+ ) and cupric ( Cu2+ ) ions are as follows
Cu+ = [ Ar] 3d 104s 0
Cu2 + = [ Ar] 3d 94s 0
Thus, electronic configuration of Cu+ is more stable but it is less stable because in Cu2+ due to its small size
the nuclear charge is sufficient to hold 27 electrons but in Cu+ such a condition is not true.
Further, the IInd IP of Cu is not very high as compared to its Ist IP. Consequently a large amount of lattice

)
energy is released for cupric compounds as compared to Cu+ compounds.

be
60. Both the given molecules contain 6 carbon atoms with an aldehyde group, so these are called aldohexoses.
In X, the —OH attached to second last carbon in on right hand side while in Y it is on left hand side, so X is

Tu
D-sugar and Y is L-sugar. X and Y are the mirror images of each other, i.e., these are enantiomers.

Mathematics

ou
1. Given equation is = x 2 + y 2 + ( x - 3)2 + y 2 + x 2 + ( y - 1)2
x + 1 - x - 1 = 4x - 1 ...(i)

(Y
= x 2 + y 2 + x 2 - 6x + 9 + y 2 + x 2 + y 2
On squaring both sides, we get + 1 - 2y
( x + 1) + ( x - 1) - 2 x 2 - 1 = 4x - 1 2 2
on
= 3x + 3y - 6x - 2y + 10
2
Þ 2x - 2 x - 1 = 4x - 1 æ 2 1ö
= 3( x 2 - 2x + 1) + 3 çy 2 - y + ÷
⇒ è 3 9ø
pi

2
-2 x - 1 = 2 x - 1
1
Again, squaring both sides, we get + 10 - 3 -
m

3
4 ( x 2 - 1) = 4x 2 + 1 - 4x
2
ha

æ 1ö 20
⇒ -4 = + 1 - 4x = 3( x - 1)2 + 3 çy - ÷ +
è 3ø 3
⇒ 4x = 5
1
C

5 It is minimum, when x - 1 = 0 and y - =0


⇒ x = 3
4 1
\ x = 1 and y =
dy

5 3
But, when we put x = in Eq. (i), we get
4 1
\ z =1+ i
5 5 5 3
u

+1- -1 = 4 ´ -1
4 4 4 ì2x 2 + 1, x £ 1
St

3. Given, f ( x ) = í 3
9 1 3 1 î4x - 1, x > 1
⇒ - = 5 -1 ⇒ - =2
4 4 2 2 2 1 2
\ ò0 f ( x ) dx = ò f ( x ) dx + ò1 f ( x ) dx
⇒ 1 = 2, which is not true. 0
1 2
Hence, no value of x satisfy the given equation. = ò ( 2x 2 + 1) dx + ò1 (4x
3
- 1) dx
0

2. Let z = x + iy 1 2
é 2x 3 ù é 4x 4 ù
=ê + xú + ê - xú
\ | z |2 + | z - 3 |2 + | z - i |2 ë 3 û0 ë 4 û1
= | x + iy |2 + |( x - 3) + iy |2 2
= ( 1)3 + 1 - ( 0 + 0)
3
+ | x + i ( y - 1)|2 + [( 2) 4 - 2 - {( 1) 4 - 1}]
WB JEE (Engineering) · Solved Paper 2014 39

2 It is a linear differential equation of the form of


= + 1 + [16 - 2 - 0]
3 dy
+ Py = Q
2 2 + 45 dx
= + 15 =
3 3 1 e tan x
-1

47 where, P = 2
and Q =
= sq units 1+ x 1 + x2
3
\ Integrating factor, IF = e ò
P dx
2a sin x - sin 2x
4. lim

)
1
x ®0 tan3 x ò dx

be
1 + x2 -1

æ 3 5
ö æ 3 ö 5 =e = e tan x
x x (2x ) (2x )
2a ç x - + -K÷ - ç2x - + -K÷
= lim
è 3! 5! ø è 3! 5! ø 7. Given, y = cos -1 x ⇒ y = (cos -1 x )2

Tu
x ®0 3
æ x3 2 5 ö On differentiating both sides w.r.t. x, we get
çx + + x + K÷
è 3 15 ø dy -1
= 2 (cos -1 x ) ´

ou
æ 2a 23 ö æ 2a 25 ö dx 1 - x2
(2a - 2) x + ç - + ÷ x 3 + ç - ÷ x 5 + K
è 3 ! 3 ! ø è 5! 5! ø
= lim 3
Again, differentiating both sides w.r.t. x, we get
x ®0
æ x2 2 4 ö

(Y
3
x ç1 + + x + K÷ é 2 -1 ù
è 3 15 ø ê 1- x ´ 2 ú
ê 1- x ú
Since, it is given that given limit is exist, then ê -1 æ 1 ö ( -2 x ) ú
- cos x ´ ç ÷
on
2a - 2 = 0 ⇒ a = 1 d 2y ê è 2 ø ( 1 - x 2 )1/ 2 ú
= - 2 ê ú
5. Given, log 101 log 7 ( x + 7 + x ) = 0 dx 2 ê ( 1 - x 2 )2 ú
ê ú
pi

\ log 7 ( x + 7 + x ) = ( 101) 0 ê ú
ê ú
m

⇒ log 7 ( x + 7 + x) =1 ê
ë
ú
û
⇒ ( x +7+ x ) = 71 é x cos -1 x ù
ha

ê -1 + ú
⇒ x +7+ x =7 ( 1 - x 2 )1/ 2 ú
= -2 ê 2
ê (1 - x ) ú
On squaring both sides, we get
C

ê ú
ë û
( x + 7) + x + 2 x 2 + 7x = 49
é 2x cos -1 x ù
dy

⇒ 2x - 42 = - 2 x 2 + 7x 2 ê2 - ú
dy ê ( 1 - x 2 )1/ 2 ú
=
⇒ x - 21 = - x 2 + 7x dx 2 ê (1 - x 2 ) ú
u

ê ú
Again, squaring both sides, we get ë û
St

x 2 + 441 - 42x = x 2 + 7x d 2y dy
⇒ (1 - x 2 ) =2 + x
⇒ 49x = 441 dx 2 dx
2
d y dy
441 ⇒ (1 - x 2 ) 2 - x =2
⇒ x = ⇒ x =9 dx dx
49
But, it is given
6. Given equation is d 2y dy
(1 - x 2 ) 2 - x =c
dy -1
dx dx
(1 + x 2 )
+ y = e tan x
dx \ c =2
or it can be rewritten as 8. Let y = 20301
tan -1 x
dy 1 e
+ y = Number of digits = Integral part of
dx ( 1 + x 2 ) 1 + x2 ( 301 log 10 20) + 1
40 WB JEE (Engineering) · Solved Paper 2014

= Integral part of [301 (log 10 10 + log 10 2)] + 1 p 2p 4p


11. tan + 2 tan + 4 cot
= Integral part of [301( 1 + 0.3010)] + 1 5 5 5
= Integral part of [301 ´ 1.3010] + 1 p 2p 4p
sin sin cos
= Integral part of [391.601] + 1 = 5 +2 5 +4 5
p 2p 4p
= 391 + 1 = 392 cos cos sin
5 5 5
9. Given curves are y = x 2 and x = y 2, which is the p 2p æ 2 2p 2p ö
sin sin 4 çcos - sin2 ÷

)
form of parabola. 5 5 è 5 5 ø
= +2 +

be
y x 2 =y p 2p 2p 2p
(1, 1) cos cos 2 sin cos
5 5 5 5
y 2=x
p 2p æ 2 2p 2 2p ö

Tu
x¢ sin sin 2 çcos - sin ÷
O 1
x
= 5 +2 5 + è 5 5 ø
p 2p 2p 2p
cos cos sin cos

ou

5 5 5 5
p é 2 2p 2 2 p 2 2p ù
sin ê sin 5 + cos 5 - sin 5 ú
The point of intersection, x = ( x 2 )2 5
= +2ê

(Y
p 2p 2p ú
⇒ x = x 4 ⇒ x (1 - x 3 ) = 0 cos ê cos sin ú
5 ë 5 5 û
⇒ x = 0 and 1 = x 3
p 2p
sin 2 cos
on
⇒ x = 0 and x = 1 5 + 5
=
When x = 0, then y = 0 p 2p
cos sin
When x = 1, then y = 12 = 1 5 5
pi

p æ 2 p pö
\ The point of intersection is (0, 0) and (1, 1). sin 2 çcos - sin2 ÷
è 5 5ø
m

= 5 +
\ Area of shaded region p p p
1 cos 2 sin cos
= ò ( y 2 - y 1) dx 5 5 5
ha

0
1 p p sin2 p
1 é x 3/ 2 x 3 ù sin2 + cos2 -
=ò [ x - x 2] dx = ê - ú = 5 5 5
C

0
ë3 / 2 3 û0 p
sin cos
p
2 3 / 2 ( 1)3 5 5
= ( 1) - -0-0 2 p
dy

3 3 cos
2 1 1 = 5
= - = sq units p p
3 3 3 sin cos
u

5 5
10. Given, f ( x ) = 3x 2 + 1
St

p
= cot
Let y = 3x 2 + 1 5
y -1 12. Given, f ¢( x ) £ 5
⇒ 3x 2 = y - 1 ⇒ x 2 =
3 f ( 7) - f ( 2)
\ £5
y -1 x -2
⇒ x =±
3 [by Lagrange mean value theorem]
x -1 f ( 7) - 3
\ f -1( x ) = ± ⇒ £5
3 7 -2
When x Î[1, 6] ⇒ f ( 7) £ 5( 7 - 2) + 3
-1 é 5 5ù
Then, f (x ) Îê- , ú \ f ( 7) £ 28
ë 3 3û
WB JEE (Engineering) · Solved Paper 2014 41

13. Given, number of elements of A and B are p and 16. Given equation of ellipse is
q, respectively. x2 y2
+ =1
\ The number of relations from the set A to the 144 25
set B is 2 pq . Here, a2 = 144 and b 2 = 25
14. Given equation can be rewritten as b2 25
r2 Now, e = 1- = 1-
x2 - x + 1=0 a2 144

)
pq
119 119

be
r2 = =
\ tan A + tan B = 144 12
pq \ Foci of an ellipse = ( ± ae, 0)

Tu
and tan A tan B = 1 æ 119 ö
We know, A + B + C = 180° = ç ±12 ´ , 0÷
è 12 ø
⇒ ( A + B ) = 180° - C

ou
= ( ± 119, 0)
⇒ tan ( A + B ) = tan ( 180° - C )
Since, the circle with centre ( 0, 2 ) and passing
tan A + tan B

(Y
= - tan C through foci ( ± 119, 0) of the ellipse.
1 - tan A tan B
\ Radius of circle = ( 119 - 0)2 + ( 0 - 2 )2
r 2 / pq
⇒ = - tan C
on
1-1 = 119 + 2 = 121 = 11
⇒ ¥ = - tan C ⇒ C = 90° 17. Given lines are
pi

Hence, DABC is a right angled triangle. x + y =0 ...(i)


15. Given equation of parabola is 5x + y = 4 ...(ii)
m

and x + 5y = 4 ...(iii)
y 2 = 12x ...(i)
On solving above equations pairwise, we get
ha

On differentiating both sides w.r.t. x, we get


æ2 2ö
dy dy 6 A( 1, - 1), B ç , ÷, C ( -1, 1)
2y = 12 ⇒ = è3 3ø
dx dx y
C

2 2
Since, the normal to the curve is parallel to the æ2 ö æ2 ö
Now, AB = ç - 1÷ + ç + 1÷
line y = 4x + 3. è3 ø è3 ø
dy

\ Slope of normal curve = Slope of line 1 25 26 26


= + = =
y 9 9 9 3
⇒ - =4
u

6 2ö 2ö
2 2
æ æ
BC = ç -1 - ÷ + ç1 - ÷
St

⇒ y = - 24 è 3 ø è 3ø
From Eq. (i), we get 25 1 26 26
( -24)2 = 12x = + = =
9 9 9 3
⇒ 24 ´ 24 = 12x and CA = ( 1 + 1)2 + ( -1 - 1)2
⇒ x = 48 = 4 + 4 =2 2
\ Normal point on a curve is ( 48, - 24). Here, we see that AB = BC
\ Distance from ( 48, - 24) to the line Hence, given lines form an isosceles triangle.
4x - y + 3 = 0 is
4 ´ 48 + 24 + 3 219 18. Given,
=
42 + 12 17 æx ö æ 13 ö p
sin-1 ç ÷ + cosec -1 ç ÷ = ...(i)
è 13 ø è 12 ø 2
42 WB JEE (Engineering) · Solved Paper 2014

13 1 2x + 1
Let cosec -1 =y 20. Given, f (x ) = x + =
12 2 2
13 12 1
Then, cosec y = ⇒ sin y = \ f ( 2x ) = 2x +
12 13 2
cos y = 1 - sin2 y 4x + 1
\ ⇒ f ( 2x ) =
2
2
æ 12 ö 144 1 8x + 1
= 1- ç ÷ = 1- and f ( 4x ) = 4x + ⇒ f ( 4x ) =

)
è 13 ø 169 2 2

be
25 5 5 Since, f ( x ), f ( 2x ) and f ( 4x ) are in HP.
= = ⇒ y = cos -1 1 1 1
169 13 13 \ , and are in AP.

Tu
Eq. (i) becomes, f ( x ) f ( 2x ) f ( 4x )
æx ö æ5ö p 1 1
sin-1 ç ÷ + cos -1 ç ÷ = ...(i) +
è 13 ø è 13 ø 2 1 f ( x ) f ( 4x )
⇒ =

ou
f ( 2x ) 2
We know that
p 2 2
sin-1 q + cos -1 q = +

(Y
2 2x + 1 8x + 1
2 ⇒ =
\ Both angles of Eq. (i) should be same. 4x + 1 2
x =5 2 10x + 2
\ ⇒ =
on
4x + 1 ( 2x + 1) ( 8x + 1)
19. Given curve is
( 7x + 5)2 + ( 7y + 3)2 = l2( 4x + 3y - 24)2 ⇒ ( 2x + 1) ( 8x + 1) = ( 5x + 1) ( 4x + 1)
pi

⇒ 49x 2 + 25 + 70x + 49y 2 + 9 + 42y ⇒ 16x 2 + 10x + 1 = 20x 2 + 9x + 1


⇒ 4x 2 - x = 0 ⇒ x ( 4x - 1) = 0
m

2 2 2
= l (16x + 9y + 576 + 24xy - 144y - 192x ) 1
⇒ x = [Q x ¹ 0]
ha

Þ 4
( 49 - 16l2 ) x 2 + ( 49 - 9l2 ) y 2 + ( 70 + 192l2 ) x Hence, one real value of x for which the three
+ ( 42 + 144l2 ) y - 24l2 xy + ( 25 - 576l2 ) = 0 unequal terms are in HP.
C

On comparing with 21. Given, f ( x ) = 2x 2 + 5x + 1 ...(i)


2 2
ax + 2hxy + by + 2gx + 2fy + c = 0 , we get Also, f ( x ) = a( x + 1) ( x - 2) + b ( x - 2) ( x - 1)
dy

2 2
a = 49 - 16l , b = ( 49 - 9 l ), h = - 12l 2 + c ( x - 1) ( x + 1)
= a( x 2 - x - 2 ) + b ( x 2 - 3 x + 2 )
Condition for parabola is ab = h2 .
u

+ c ( x 2 - 1)
( 49 - 16l2 ) ( 49 - 9l2 ) = ( -12l2 )2
St

\
⇒ 2401 - 441l2 - 784l2 + 144l4 = 144l4 ⇒ f ( x ) = ( a + b + c ) x 2 + ( -a - 3b ) x
+ ( -2a + 2b - c ) ...(ii)
⇒ 2401 - 1225l2 = 0
2
On equating the coefficients of x , x and constant
2401
⇒ l2 = term in Eqs. (i) and (ii), we get
1225
a + b + c = 2, -a - 3b = 5
( 49)2
⇒ l2 = and -2a + 2b - c = 1
( 35)2
On solving above equations, we get
49
⇒ l=± 35 5 38
35 a=- , b = and c =
4 4 4
7
⇒ l=± Hence, exactly one choice for each of b and c.
5
WB JEE (Engineering) · Solved Paper 2014 43

22. Given, a, b are the roots of ax 2 + bx + c = 0 and 24. Equation of tangent in slope form of parabola
a + h, b + h are the roots of px + qx + r = 02 y 2 = 8 3 x is
b c y = mx + c ...(i)
\ a+b=- , ab =
a a a
where, c=
q r m
and a + h + b + h = - , ( a + h ) ( b + h ) =
p p 2 3
\ c= ...(ii)
m

)
Now, ( a + h ) - (b + h ) = a - b

be
⇒ [( a + h ) - ( b + h )]2 = ( a - b )2 Also, tangent to the hyperbola
4x 2 - y 2 = 4
⇒ [( a + h ) + ( b + h )]2 - 4( a + h ) ( b + h )
x2 y 2

Tu
= ( a + b )2 - 4ab or - = 1 is
1 4
q 2 4r b 2 4c
⇒ - = 2 - c 2 = a2 m2 - b 2

ou
p2 p a a
c 2 = 1m2 - 4
q 2 - 4pr b 2 - 4ac
⇒ = 2
p2 a2 æ2 3 ö

(Y
⇒ ç ÷ = m2 - 4 [from Eq. (ii)]
b 2 - 4ac a2 è m ø
\ = 12
q 2 - 4pr p 2 ⇒ = m2 - 4
m2
on
Hence, the ratio of the square of their ⇒ m 4 - 4m2 - 12 = 0
discriminants is a2 : p 2 .
⇒ m 4 - 6m2 + 2m2 - 12 = 0
pi

23. Since, a is a root of ⇒ m2( m2 - 6) + 2( m2 - 6) = 0


2 2 2
x + 3p x + 5q = 0
m

⇒ ( m2 + 2) ( m2 - 6) = 0
2 2 2
\ a + 3p a + 5q = 0 ...(i) ⇒ m - 6 = 0 and m2 + 2 ¹ 0
2
ha

and b is a root of ⇒ m2 = 6
2 2 2
x + 9p x + 15q = 0 ⇒ m=± 6
C

\ b2 + 9p 2b + 15q 2 = 0 ...(ii) i.e., m = 6 as m is positive slope.


Let f ( x ) = x 2 + 6p 2 x + 10q 2 \ From Eq. (i),
dy

Then, f ( a ) = a2 + 6p 2 a + 10q 2 2 3
y = 6x +
2 2 2
= ( a + 3p a + 5q ) + 3p a + 5q 2 2 6
u

= 0 + 3p 2 a + 5q 2 [from Eq. (i)] ⇒ y = 6x + 2


St

⇒ f ( a) > 0 25. Given equation of parabola is


and 2
f ( b ) = b + 6p b + 10q2 2 y 2 = 64x ...(i)

= ( b2 + 9p 2b + 15q 2 ) - ( 3p 2b + 5q 2 ) The point at which the tangent to the curve is


parallel to the line is the nearest point on the
= 0 - ( 3p 2b + 5q 2 ) [ from Eq. (ii)] curve.
⇒ f (b) < 0 On differentiating both sides of Eq. (i), we get
dy
Thus, f ( x ) is a polynomial such that f ( a ) > 0 and 2y = 64
f ( b ) < 0. dx
dy 32
Therefore, there exists g satisfying a < g < b such ⇒ =
that f ( g ) = 0. dx y
44 WB JEE (Engineering) · Solved Paper 2014

4 æ 1 ö
13
Also, slope of the given line is - . Now, the general term in çax - 2 ÷ is
3 è bx ø
4 32
\ - = ⇒ y = - 24 13 13 - r æ 1 ö
r
3 y T ¢r + 1 = C r ( ax ) ç- 2 ÷
è bx ø
From Eq. (i), ( -24)2 = 64x ⇒ x = 9
= 13C r a13 - r ´ b - r ´ ( x )13 - 3r ( -1) r
\ Hence, the required point is ( 9, - 24).
For coefficient of x -8, put 13 - 3r = - 8

)
26. We know that
⇒ 3r = 21 ⇒ r = 7

be
| z - z1 | + | z - z2 | = k will represent an ellipse, if
| z1 - z2 | < k. \ T ¢8 = ( -1)7 13C7a13 - 7 b -7 x -8
= ( -1)7 13C7a 6b -7 x -8

Tu
Hence, the equation | z - z1 | + | z - z2 |
= 2 | z1 - z2 | represent an ellipse. According to the given condition,
13
ì é p ùü æ 1 ö
Coefficient of x 8 in çax 2 +

ou
tan í p ê x - úý ÷
î ë 2 ûþ è bx ø
27. Given, f ( x ) =
2 + [ x ]2 æ 1 ö
13
= Coefficient of x -8 in çax - 2 ÷

é
é
ë


Since, ê x - ú is an integer for all x, therefore
2û (Y \ 13C 6 a7 b -6 = -13 C7a 6b -7
a7 a6
è bx ø
on
p ê x - ú is an integral multiple of p for all x.
ë 2û ⇒ 13
C7 = -13
C 7
b6 b7
ì é p ùü
Hence, tan í p êë x - 2 úûý = 0 for all x a7 b6 1
pi

î þ ⇒ 6
=- 7 ⇒ a=- ⇒ ab + 1 = 0
a b b
Also, 2 + [ x ]2 ¹ 0 for all x
m

2 4
30. Given, I = ò e x ( x - a) dx = 0
0
Hence, f ( x ) = 0 for all x.
ha

2 x4 2 x4
Hence, f ( x ) is continuous and derivable for all x. ⇒ ò0 e x dx = ò0 e a dx
28. Given, f ( x ) = a sin| x | + be| x | 2 4
Here, we see that ò e x x dx gives us an area
C

0
We know that sin| x | and e| x | is not 4
between two curves e x and x from x = 0 to x = 2.
differentiable at x = 0. 2 x4
dy

Similarly, ò0 e a dx gives us a same area, so a


Therefore, for f ( x ) to differentiable at x = 0, we
should lies between 0 to 2, i.e., a Î( 0, 2).
must have a = b = 0.
u

31. Given differential equation can be rewritten as


\ a + b =0
æ y2 ö
St

13 x f ç 2÷
1 ö
29. The general term in æçax 2 + ÷ is dy y
= +
èx ø
è bx ø dx x æ y2 ö
r y f¢ ç 2 ÷
13 2 13 - r æ 1 ö èx ø
Tr + 1 = C r ( ax ) ç ÷
è bx ø dy dv
Put y = vx ⇒ =v + x
= 13C r a13 - r ´ b - r ( x )26 - 3r dx dx
For coefficient of x 8, put 26 - 3r = 8 \ Given equation becomes,
æ v 2x 2 ö
⇒ 3r = 18 ⇒ r = 6 x fç 2 ÷
dv vx è x ø
\ T7 = 13C 6 a 13 - 6b -6x 8 v +x = +
dx x æ v 2x 2 ö
= 13C 6 a7 b -6x 8 vx f¢ ç 2 ÷
è x ø
WB JEE (Engineering) · Solved Paper 2014 45

dv f (v 2 ) 34. Given,
⇒ x =
dx v f¢ ( v 2 ) é æ 2p ö æ 2p ö ù
ê cos çè n ÷ø sin çè n ÷ø 0ú
v f¢ ( v 2 ) dx ê ú
⇒ dv = æ 2p ö æ 2p ö
f (v 2 ) x A = ê - sin ç ÷ cos ç ÷ 0ú
ê è n ø è n ø ú
On integrating both sides, we get ê 0 0 1ú
1 ê ú
log f ( v 2 ) = log x + log c1 ë û

)
2 é æ 2p ö æ 2p ö ù

be
⇒ log f ( v 2 ) = 2 log xc1 ê cos çè n ÷ø sin çè n ÷ø 0ú
ê ú
æ y2 ö æ 2p ö æ 2p ö
⇒ f ( v 2 ) = ( xc1)2 ⇒ f ç 2 ÷ = x 2c12 Now, A ´ A = ê - sin ç ÷ cos ç ÷ 0ú ´
è n ø è n ø

Tu
ê ú
èx ø ê 0 0 1ú
æ y2 ö ê ú
⇒ f ç 2 ÷ = x 2c [put c12 = c ] ë û

ou
èx ø é æ 2p ö æ 2p ö ù
ê cos çè n ÷ø sin çè n ÷ø 0ú
32. Since, a and b are the roots of ê ú
ê - sin æç 2p ö÷ cos æç 2p ö÷ 0ú
\
f ( x ) = x 2 + bx + c
a + b = - b and ab = c
æ a + bö æ a + bö
2
æ a + bö
(Y ê
ê
ê
ë
0
è n ø è n ø
0
ú

ú
û
on
\ f ç ÷=ç ÷ +bç ÷+c
è 2 ø è 2 ø è 2 ø
é 2 æ 2p ö 2 æ 2p ö
2 ê cos ç ÷ - sin ç ÷
æ bö æ bö è n ø è n ø
= ç- ÷ + b ç- ÷ + c ê
pi

è 2ø è 2ø æ 2 p ö æ 2
ê - sin ç ÷ cos ç ÷ - sin ç 2p ö÷ cos æç 2p ö÷
p ö æ
b2 b2 b2 ê è n ø è n ø è n ø è n ø
m

= - +c=- +c ê 0
4 2 4 ê
dy ë
ha

Now, = f ¢ ( x ) = 2x + b
dx æ 2p ö æ 2p ö æ 2p ö æ 2p ö ù
cos ç ÷ sin ç ÷ + sin ç ÷ cos ç ÷ 0ú
æa + b æ a + böö è n ø è n ø è n ø è n ø
At point ç ,f ç ÷ , ú
è 2 ø ÷ø
C

è 2 æ 2 p ö æ 2 p ö
- sin2 ç ÷ + cos2 ç ÷ 0ú
è n ø è n ø ú
æ b b2 ö
ú
i.e., ç - , - + c ÷, 0
dy

è 2 4 ø 1ú
û
dy æ bö é æ 2p ö æ 2p ö æ 2p ö ù
= 2 ç- ÷ + b = 0 cos ç2 ´ ÷ 2 sin ç ÷ cos ç ÷ 0ú
u

dx è 2ø ê è ø è ø è ø
n n n
ê ú
æ 2p ö æ 2p ö 2p ö
St

Hence, the slope of the tangent to the curve and ê æ


= -2 sin ç ÷ cos ç ÷ cos ç2 ´ ÷ 0ú
the positive direction of x-axis is 0°. ê è n ø è n ø è n ø ú
ê 0 0 1ú
33. Given function is ê ú
f ( x ) = x 2 + bx + c ë û
é æ 2p ö æ 2p ö ù
It is a quadratic equation in x. ê cos çè2 ´ n ÷ø sin çè2 ´ n ÷ø 0ú
So, we will get a parabola either downward or ê ú
æ 2p ö æ 2p ö
upward. = ê - sin ç2 ´ ÷ cos ç2 ´ ÷ 0ú
ê è n ø è n ø ú
Hence, it is a many one mapping and not onto ê 0 0 1ú
mapping. ê ú
Hence, it is neither one-to-one nor onto mapping. ë û
46 WB JEE (Engineering) · Solved Paper 2014

Similarly, 37. Let S = 1! + 2 ! + 3 ! + 4 ! + K + 11!


Here, we see that from 4! to 11!, we always get a
é æ n-1 2p ö æ n-1 2p ö ù
ê cos çè2 ´ n ÷ø sin çè2 ´ n ÷ø 0ú 12 factor, so it is always divisible by 12.
ê ú Now, 1! + 2 ! + 3 ! = 1 + 2 + 6 = 9
æ 2p ö æ n-1 2p ö
A n = ê - sin ç2 n-1 ´ ÷ cos ç2 ´ ÷ 0ú Hence, when S is divided by 12, the remainder is
ê è n ø è n ø ú
ê 0 0 1ú 9.
ê ú

)
ë û 38. 3 consonants can be selected from 7 consonants

be
é 1 0 0ù = 7C3 ways
= ê0 1 0ú = I
ê0 0 1ú 2 vowels can be selected from 4 vowels

Tu
ë û
= 4C2 ways
and
é æ n-2 2p ö æ n-2 2p ö ù \ Required number of words
ê cos çè2 ´ ÷ sin ç2 ´ ÷ 0ú

ou
n ø è n ø = 7C3 ´ 4C2 ´ 5 !
ê ú
æ 2p ö æ n-2 2p ö
A n-1 = ê - sin ç2 n-2 ´ ÷ cos ç2 ´ ÷ 0ú [selected 5 letters can be arrange in 5!, so get, a
ê è n ø è n ø ú different words]
ê
ê
ë
¹I
0 0 1ú
ú
û (Y
39. We know that
= 35 ´ 6 ´ 120 = 25200
on
( 1 + x ) n = nC 0 + x nC1 + x 2 nC2 + ...+ x n nC n
\ A m ¹ I for any positive integer m.
On integrating both sides from 0 to 2, we get
pi

35. Total sample space = {B1B2, B1G1, G1B2} 2


é (1 + x )n + 1 ù
Favourable ways = {B1G1, G1B2} ê ú
ë n +1 û0
m

2
\ Required probability = 2
3 é x2 n x3 n xn+1 n ù
= ê x nC 0 +
ha

C1 + C2 + K + Cnú
36. We know that ë 2 3 n+1 û0
( 1 + x ) n = nC 0 + nC1x + nC2 x 2 ( 3) n + 1 1 22 n 23 n
= 2 nC 0 +
C

Þ - C1 + C2
+ K + nC nx n ...(i) n+1 n+1 2 3

and n n
( x + 1) = C 0 x + C1x n n n -1 2n + 1 n
dy

+K+ Cn - 0
n+1
+ nC2 x n - 2 + K + nC n ...(ii)
2 n 22 n 23 n 2n + 1 n
On multiplying Eqs. (i) and (ii), we get ⇒ C0 + C1 + C2 + K +
u

Cn
1 2 3 n+1
St

( 1 + x )2n = ( nC 0 + nC1x + nC2 x 2 + K + nC nx n ) 3n + 1 - 1


=
´ ( nC 0 x n + nC1x n - 1 + nC2 x n - 2 + K + nC n ) n+1

Coefficient of x n in RHS ì 1
ï x sin , x ¹ 0
40. Given, f (x ) = í x
= ( nC 0 )2 + ( nC1)2 + K + ( nC n )2
îï 0, x =0
and coefficient of x n in LHS = 2nC n Continuity at x = 0,
n 2 n 2 2n !
n 2 1
\ ( C 0 ) + ( C1 ) + K + ( C n ) = LHL = lim x sin =0
n!n! x ® 0- x
( 2n ) ! 1
⇒ ( nC1)2 + K + ( nC n )2 = -1 RHL = lim x sin =0
n!n! x ®0 + x
= 2nC n - 1 and f ( 0) = 0
WB JEE (Engineering) · Solved Paper 2014 47

\ LHL = RHL = f ( 0) 42. Let D denotes dancing, P denotes painting and S


Hence, f ( x ) is continuous for all values of x. denotes singing.
Differentiability at x = 0 \ n( D È P È S ) = 265,
1 n( S ) = 200, n( D ) = 110, n( P ) = 55,
h sin
f ( h ) - f ( 0) h n( S Ç D ) = 60, n( S Ç P ) = 30
LHD = lim = lim
h®0 h h®0 h and n( D Ç P Ç S ) = 10
1
= - lim sin Qn( D È P È S ) = n( D ) + n( P ) + n( S ) - n( D Ç P )

)
h®0 h

be
- n( P Ç S ) - n( S Ç D ) + n( D Ç P Ç S )
æ 1ö
= - sin ç ÷ = some definite value \ 265 = 110 + 55 + 200 - n( D Ç P )
è0ø
- 30 - 60 + 10

Tu
Thus, f ( x ) is not differentiable at x = 0.
⇒ 265 = 285 - n( D Ç P )
Now, f ( 0) = 0 and f ( 1) = 1 sin 1
\ f ( 0) ¹ f ( 1), Rolle’s theorem is not satisfy. ⇒ n( D Ç P ) = 20

ou
f ( 1) - f ( 0) \ Only person who like dancing and painting
Now, f ¢ (c ) =
b -0 = n( D Ç P ) - n( D Ç P Ç S )

which lies between 0 to 1.


1 sin ( 1) - 0
1-0
= sin 1 = sin 1
(Y = 20 - 10 = 10

43. Given function is y = 3 sin çç


æ p2

è 16
ö
- x2 ÷
÷
on
ø
Hence, their exist a real number c Î( 0, 1).
é pù
Hence, Lagrange’s mean value theorem is satisfy Here, domain is ê0, ú
ë 4û
pi

on [0, 1].
When, x =0
m

41. Since, a, b and c are in GP. æ p2 ö


\ b 2 = ac y = 3 sin ç - 0÷
ç 16 ÷
è ø
ha

Given equation is
(log e a ) x 2 - ( 2 log e b ) x + (log e c ) = 0 æpö 1 3
= 3 sin ç ÷ = 3 ´ =
è4ø 2 2
C

Put x = 1, we get
p æ p2 p2 ö
log e a - 2 log e b + log e c = 0 When x = , y = 3 sin ç - ÷
4 ç 16 16 ÷

dy

2 log e b = log e a + log e c è ø


⇒ log e b 2 = log e ac = 3 sin ( 0) = 0
u

⇒ b 2 = ac , which is true. é 3 ù
Hence, range of given function is ê0, .
ë 2 úû
St

Hence, one of the root of given equation is 1.


x2
Let another root be a. cos ( t 2 ) dt
44. lim
ò0 é0 ù
êë 0 form úû
2
2 log e b log e b
\ Sum of roots, 1 + a = = x ®0 x sin x
log e a log e a
cos ( x 4) ´ 2x
log e ac = lim [L¢ Hospital’s rule]
⇒ a= -1 x ® 0 sin x + x cos x
log e a
(log e a + log e c ) 2 [cos x 4 - x sin ( x 4) ´ 4x 3
= -1 = lim
log e a
x ®0 cos x + cos x - x sin x
log e c [L¢ Hospital’s rule]
= = log a c 2 [cos 0 - 0] 2
log e a = = =1
cos 0 + cos 0 - 0 1 + 1
Hence, roots are 1 and log a c.
48 WB JEE (Engineering) · Solved Paper 2014

45. Given, f ¢ ( 4) = 5 é0 1 0ù é0 0 1ù
2
ê0 0 1ú , ê0 1 0ú
f ( 4) - f ( x ) é0 ù ê 1 0 0ú ê 1 0 0ú
Now, lim êë 0 form úû ë û ë û
x ®2 x -2
who will give a determinant either -1 or 1.
0 - f ¢ ( x 2 ) × 2x Hence, there are six distinct choices for P and
= lim
x ®2 1 det ( P ) = ± 1.
-f ¢ ( 4) × 2 ´ 2
= 48. Given expression can be rewritten as

)
1

be
-1
= - ( 5) ´ 4 = - 20 æ xö
2 ( 1 - x ) -1 ´ 2 -1 ç 1 - ÷
è 2ø
46. We know that
= [1 + x + x 2 + x 3 + ...]

Tu
sin np = 0, ∀ n Î N .
é 2 3 ù
¥
æn!pö æ p ö æ2! p ö æxö æxö æxö
ê1 + ç ÷ + ç ÷ + ç ÷ + Kú
\E = å sin ç ÷ = sin ç ÷ + sin ç ÷ è 2 ø è 2 ø è 2 ø

ou
è 720 ø è 720 ø è 720 ø êë úû
n =1

æ3! p ö æ4! p ö æ5! p ö \ Coefficient of x 3 in the above expression is


+ sin ç ÷ + sin ç ÷ + sin ç ÷
è 720 ø è 720 ø è 720 ø é æ 1 ö3 æ 1 ö2 æ 1 ö

(Y
ù
= ê ç ÷ + ç ÷ + ç ÷ + 1ú
æ6! p ö æ 720 ! p ö êë è 2 ø è 2 ø è 2 ø úû
+ sin ç ÷ + K + sin ç ÷
è 720 ø è 720 ø
é1 1 1 ù
on
= ê + + + 1ú
æ p ö æ p ö æ p ö æpö ë8 4 2 û
= sin ç ÷ + sin ç ÷ + sin ç ÷ + sin ç ÷
è 720 ø è 320 ø è 120 ø è 30 ø
é 1 + 2 + 4 + 8 ù 15
=ê úû = 8
pi

æpö æ 720 ! p ö ë 8
+ sin ç ÷ + sin p + K + sin ç ÷
è6ø è 720 ø
m

49. Given,
Here, we see that after five terms of the above
x 3 2 7 3 15 4
series all angles are multiple of p, i.e., sin np, so f (x ) = + x + x + x +K
ha

1! 2 ! 3! 4!
all further values are zero.
æ p ö æ p ö æ p ö ( 21 - 1) x ( 22 - 1) x 2 ( 23 - 1) x 3
\ E = sin ç ÷ + sinç ÷ + sinç ÷ = + +
è 720 ø è 320 ø è 120 ø 1! 2! 3!
C

æpö æpö ( 2 4 - 1) 4
+ sinç ÷ + sinç ÷ + x +K
è 30 ø è6ø 4!
dy

2x ( 2x )2 ( 2x )3 ( 2x ) 4
é 1 0 0ù = + + + +K
47. Given, I = ê0 1 0ú 1! 2! 3! 4!
u

ê0 0 1ú æx x2 x3 x 4 ö
ë û -ç + + + + K÷
St

Then, det ( I ) = 1 è 1! 2 ! 3 ! 4 ! ø
If we take I as 2x ( 2x )2 ( 2x )3 ( 2x ) 4
=1+ + + + +K
é0 1 0ù 1! 2! 3! 4!
A1 = ê 1 0 0ú
ê0 0 1ú æ x x2 x3 x 4 ö
ë û - ç1 + + + + + K÷
è 1! 2 ! 3 ! 4 ! ø
Then, det ( I 1) = - 1
Similarly, there are four other possibilities, ⇒ f ( x ) = e 2x - e x
é 1 0 0ù é0 0 1ù When we put x = 0, we get
ê0 0 1ú , ê 1 0 0ú , f ( 0) = e 0 - e 0 = 1 - 1 = 0
ê0 1 0ú ê0 1 0ú
ë û ë û Hence, exactly one real solution exists.
WB JEE (Engineering) · Solved Paper 2014 49

8 21 40 65 \ z2 = z1w
50. Let S = 1 + + + + +K
2! 3! 4! 5! and z3 = z1w2
Again, let S1 = 1 + 8 + 21 + 40 + 65 + K + Tn \ z1z2z3 = z1 ´ z1w ´ z1w2
and S1 = + 1 + 8 + 21 + 40 + K + Tn
= z13 w3
0 = 1 + 7 + 13 + 19 + 25 + K - Tn
Tn = 1 + 7 + 13 + 19 + 25 + K + n terms = z13 [Q w3 = 1]
n 54. Since, z1, z2 and z3 are the vertices of an

)
= [2( 1) + ( n - 1) 6]
2

be
equilateral triangle, therefore
= n [1 + 3( n - 1)] = n( 3n - 2) | z1 - z2 | = | z2 - z3 |
n( 3 n - 2 )
\ S=å = | z3 - z1 | = k (say)

Tu
n! 1
3n - 2 Also, a = ( 3 + i)
=å 2
( n - 1) !

ou
1 1
Þ | a| = 3 + 1 = ´2 =1
3n - 3 + 1 2 2

( n - 1) ! Let A = az1 + b, B = az2 + b
S=å
3
( n - 2) !
+

é
å
1
( n - 1) !
1 1 ù
(Y and
Now,
C = az3 + b
| AB | = | az2 + b - ( az1 + b )|
= | a ( z2 - z1)|
on
= 3e + e êëQ e = 1 + 1 ! + 2 ! + Kúû = | a || z2 - z1 |
= 4e = | 1|| z2 - z1 |
pi

We know 2 <e <3 = 1| z2 - z1 |


\ 8 < 4e < 12 = | z2 - z1 | = k
m

⇒ 8 < S < 12 Similarly, BC = CA = k


Hence, the points az1 + b, az2 + b and az3 + b
ha

51. We have, are the vertices of an equilateral triangle.


n 2 - 1 < [ n 2] £ n 2 [Q x - 1 £ [ x ] £ x ] 55. Given curve is
C

1 y = (cos x + y )1/ 2
⇒ 2 - < [ n 2] £ 1
n
On differentiating both sides w.r.t. x, we get
dy

\ By Sandwich theorem,
dy 1 æ dy ö
æ 1ö = ´ ç - sin x + ÷
lim ç 2 - ÷ = 2 dx 2(cos x + y )1/ 2 è dx ø
n®¥ è nø
u

æ dy ö
52. Given, f ¢ ( 0) = 1 and f ¢ ¢( 0) does not exist. ç - sin x + ÷
St

è dx ø
=
Also, given g ( x ) = xf ¢ ( x ) 2y
\ g ¢ ( x ) = xf ¢ ¢ ( x ) + f ¢ ( x ) dy dy
⇒ 2y = - sin x +
Put x = 0, we get dx dx
g ¢ ( 0) = 0 f ¢ ¢ ( 0) + f ¢ ( 0) dy
⇒ ( 2y - 1) = - sin x
= 0 + 1= 1 dx
Again, differentiating both sides w.r.t. x, we get
53. Given, z1 ¹ ± 1
d 2y dy æ dy ö
Since, z2 and z3 can be obtained by rotating ( 2y - 1) + ç2 ÷ = - cos x
dx 2 dx è dx ø
2p 4p
vector representing through and , d 2y dy æ dy ö
3 3 ⇒ ( 2y - 1) + ç2 ÷ + cos x = 0
respectively dx 2 dx è dx ø
50 WB JEE (Engineering) · Solved Paper 2014

56. Given equation is ⇒ ( a + b )2 - 2ab + ( a + b )


1 + z + z3 + z 4 = 0 = ( a + b )3 - 3ab ( a + b )
⇒ ( 1 + z ) + z 3( 1 + z ) = 0 ⇒ 12 - 2( -1) + 1 = 13 - 3( -1) ( 1)
⇒ (1 + z ) (1 + z 3 ) = 1 ⇒ 1+ 2 + 1=1+ 3
⇒ 3
z = - 1, z = - 1 ⇒ 4 = 4, which is true.
⇒ z = - 1, z = - 1, - w, - w2 Hence, option (a) is correct.

)
be
Hence, roots are in cubic roots of unity. 59. Required probability
Hence, these roots are the vertices of an 12
æ 1ö
equilateral triangle. = 12C2 ´ 10C2 ´ 8C2 ´ 6C2 ´ 4C2 ´ 2C2 ´ ç ÷

Tu
è6ø
57. å a3 sin ( B - C ) =
12 !
´
10 !
´
8!
´
6!
= å k3 sin3 A sin ( B - C ) 10 ! ´ 2 ! 8 ! ´ 2 ! 6 ! ´ 2 ! 4 ! ´ 2 !

ou
12
é ù 4! 2! æ 1ö
a b c ´ ´ ´ç ÷
êQ = = = kú 2 ! ´ 2 ! 2 ! ´ 1! è 6 ø
sin A sin B sin C

(Y
ë û
12 !
= å k3 [sin2 A sin ( B + C ) sin ( B - C )] =
2 ´ 612
6

éì 1 ü
= k3 êísin2 A ´ (cos 2C - cos 2B )ý
on
60. Q Sum of roots, a + b = - p and ab = q
ëî 2 þ
1 \ ( a3 + b3 ) = ( a + b )3 - 3ab ( a + b )
+ sin2 B ´ (cos 2A - cos 2C )
pi

2 = ( - p )3 - 3q ( - p )
2 1 ù = - p 3 + 3pq
+ sin C ´ (cos 2B - cos 2A ) ú
m

2 û
and a 4 + a2b2 + b 4 = ( a 4 + b 4) + ( ab )2
k3
ha

= [sin2 A ( 1 - 2 sin2 C - 1 + 2 sin2 B ) = ( a2 + b2 )2 - ( ab )2


2
+ sin2 B ( 1 - 2 sin2 A - 1 + 2 sin2 C ) = [( a + b )2 - 2ab]2 - ( ab )2
C

+ sin2 C ( 1 - 2 sin2 B - 1 + 2 sin2 A )] = [( - p )2 - 2q ]2 - 3( q )2

k3 = [ p 2 - 2q ]2 - 3q 2
dy

= [ -2 sin2 A sin2 C + 2 sin2 A sin2 B


2 = p 4 - 4p 2q + 4q 2 - q 2
- 2 sin2 B sin2 A + 2 sin2 B sin2 C = p 4 - 4p 2q + 3q 2
u

2 2 2 2
- 2 sin C sin B + 2 sin C sin A]
= p 4 - 3p 2q - p 2q + 3q 2
St

3
k
= [0] = 0 = p 2( p 2 - 3q ) - q ( p 2 - 3q )
2
2
= ( p 2 - q ) ( p 2 - 3q )
58. Given, a and b are the roots of x - x - 1 = 0
\ a + b =1
61. Given, differential equation is
dy y 1
and ab = - 1 + =
dx x log e x x
Now, consider option (a),
It is a linear equation of the form
Sn + Sn - 1 = Sn + 1
dy
Put n = 2, we get, S2 + S1 = S3 + Py = Q
dx
\ ( a2 + b2 ) + ( a + b ) = a3 + b3 1 1
where P= and Q =
[QS n = a n + b n, given] x log e x x
WB JEE (Engineering) · Solved Paper 2014 51

\Integrating factor, IF = e \ X 1 = {x 2 + y 2 £ 1},


1
ò dx
ì 1ü
=e x loge x
X2 = í x2 + y 2 £ ý
î 2þ
= elog (loge x )
ì 1ü
= log e x X3 = í x2 + y 2 £ ý
î 3þ
\ Solution of differential equation is
...........................
1

)
y ´ log e x = ò log e x dx ...........................
x

be
...........................
(log e x )2
Þ y ´ log e x = +C ...(i)
2 X ¥ = {x 2 + y £ 0}

Tu
When y = 1 and x = e, then ¥
(log e e )2 \ Ç X n = X1 Ç X2 Ç X3 ÇK Ç X ¥
1 ´ log e e = +C n =1
2

ou
= {x 2 + y 2 = 0}
1
Þ 1= + C ¥
2 Hence, Ç X n is a singleton set.

(Y
1 n =1
Þ C=
2 64. We know that in a Lagrange mean value
1 theorem there exist c Î( a, b ) such that
On putting C = in Eq. (i), we get
on
2 f ( b ) - f (a )
f ¢ (c ) =
(log e x )2 1 b -a
y ´ log e x = +
2 2 f ( h ) - cos 0
pi

1 \ f ¢ ( qh ) =
Þ 2y = log e x + h -0
log e x
m

cos h - cos 0
Þ - sin( qh ) =
62. Given, f ( x ) = max {x + | x |, x - [ x ]} h -0
ha

ì2 x , x ³0 [Q f ( x ) = cos x ]

î x - [ x ], x £ 0 cos h - 1
=
C

3 0 3 h
\ ò-3 f ( x ) dx = ò x - [ x ] dx + ò0 2x dx
-3 æ h2 ö
0 3 ç1 - ÷ -1
=3 ò ( 1 + x ) dx + 2 ò x dx
dy

è 2!ø
-1 0 =
[Q x - [ x ] is a periodic function at x = 1] h
0 3 [neglecting higher power of h]
u

é x2 ù é x2 ù
= 3 êx + ú +2ê ú h2
St

ë 2 û -1 ë 2 û0 -
⇒ - sin ( qh ) = 2
é æ 1ö ù h
= 3 ê0 - 0 - ç -1 + ÷ ú + 32 - 0
ë è 2 øû h
é 1ù 3 ⇒ sin ( qh ) = 2
=3 ê ú + 9 = + 9 1
ë2û 2
æhö
21 ⇒ qh = sin-1 ç ÷
= è2ø
2
h 1
1 sin-1 ´
63. Given, X n = ìíz = x + iy :| z |2 £ üý ⇒ q= 2 2
î n þ 1

ì 2 2 1ü 2
= íx + y £ ý
î nþ
52 WB JEE (Engineering) · Solved Paper 2014

h æE ö æE ö 1
sin-1 P ç ÷ = 1 and P ç ÷ =
1 2 è E2 ø è E1 ø 5
\ lim q = lim
h®q + 2 h ® 0+ h
2 \ The probability that student did not tick the
answer randomly
1 1
= ´1= = The probability that student tick the answer
2 2
correctly
65. Let equation of hyperbola be
æE ö

)
x2 y 2 P( E2 ) P ç ÷

be
- =1 ...(i) è E2 ø
a2 b 2 =
æE ö æE ö
Given, foci, ( ±8, 0) = ( ± ae, 0) P ( E 1) P ç ÷ + P ( E 2 ) P ç ÷
è E1 ø è E2 ø

Tu
⇒ ae = 8 ...(ii)
p ( 1)
2b 2 =
and length of latusrectum = 1
( 1 - p ) + p ( 1)

ou
a 5
2b 2 p 5p
\ 24 = = =
a 1 - p + 5p 1 + 4p
Þ
\ From Eq. (ii),
b 2 = 12a ...(iii)
(Y
67. Let
5
2 pr
7
=q
on
a2e2 = 64
⇒ 2 pr = 3 q + 4 q
æ a2 + b 2 ö
⇒ a2 ç ÷ = 64 ⇒ 4 q = 2 pr - 3 q
pi

è a2 ø
⇒ sin 4q = sin ( 2pr - 3q)
⇒ a2 + b 2 = 64
m

⇒ sin 4q = - sin 3q
⇒ a2 + 12a = 64
⇒ 2 sin 2q cos 2q = - [3 sin q - 4 sin3 q]
ha

⇒ a2 + 12a - 64 = 0
⇒ 2 ´ 2 sin q cos q ( 2 cos2 q - 1)
⇒ a2 + 16a - 4a - 64 = 0
C

= - 3 sin q + 4 sin3 q
⇒ a( a + 16) - 4( a + 16) = 0
Þ sin q [8 cos3 q - 4 cos q + 3 - 4( 1 - cos2 q)] = 0
⇒ ( a + 16) ( a - 4) = 0
dy

⇒ 8 cos3 q + 4 cos2 q - 4 cos q - 1 = 0 ...(i)


⇒ a =4
2p 4p 6p
[Qa cannot be negative] Thus, cos , cos and cos are the roots of
u

7 7 7
On putting a = 4 in Eq. (iii), we get the above equation.
St

b 2 = 12 ´ 4 ⇒ b 2 = 48 2p 4p 6p 1
\ cos + cos + cos =-
\ From Eq. (i), 7 7 7 2
x2 y 2 p/ 2 cos x
- =1 68. Given, M = ò dx
16 48 0 ( x + 2)
⇒ 3x 2 - y 2 = 48 p/ 4 sin x cos x
and N =ò dx
0 ( x + 1)2
66. Let, E 1 = Student does not know the answer
p/ 4 1 sin 2x
E 2 = Student knows the answer ⇒ N =ò dx
0 2 ( x + 1)2
and E = Student answer correctly.
dt
\ P ( E 1) = 1 - p , Put 2x = t ⇒ dx =
2
P( E2 ) = p
WB JEE (Engineering) · Solved Paper 2014 53

p/ 2 sin t Now, qRy


\ N =ò 2
dt
0
æt ö Then, sec2 q - tan2 y = 1
4 ç + 1÷
è2 ø
⇒ sec2 q - tan2 y + 1 = 1 + 1
p/ 2 sin t
=ò dt ⇒ sec2 q - tan2 y + sec2 f - tan2 f = 1 + 1
0 ( t + 2)2
Þ qR f and fR y
p/ 2 sin x
=ò dx Thus, it is transitive.
0 ( x + 2)2

)
Hence, it is an equivalence relation.

be
p/ 2 1
\ M -N =ò cos x ´ dx
0 II ( x + 2) 70. We know that
I AM ³ GM

Tu
p/ 2 sin x 2 sin x
+ 2 cos x
-ò dx \ ³ 2 sin x 2 cos x
0 ( x + 2)2 2
p/ 2

ou
é sin x ù p/ 2 sin x ⇒ 2 sin x + 2 cos x ³ 2 2 sin x + cos x
=ê ú -ò - dx
ë x + 2û0 0 ( x + 2)2 sin x + cos x
⇒ 2 sin x + 2 cos x ³ 2 ´ 2 2

(Y
p/ 2 sin x
-ò dx sin x + cos x
0 ( x + 2)2 1+
⇒ 2 sin x + 2 cos x ³ 2 2
p
sin æ pö
on
= 2 = 1 =
2 But sin x + cos x = 2 sin ç x + ÷ ³ - 2
p p+4 è 4ø
+2 p +4
2 2 1-
2
pi

\ 2 sin x + 2 cos x ³ 2 2
69. Given relation is defined as
1
m

1-
qRf such that sec2 q - tan2 f = 1 ⇒ 2 sin x + 2 cos x ³ 2 2, ∀ x ÎR
For Reflexive 1
ha

1-
Hence, minimum value is 2 2.
When qRq
sec2 q - tan2 q = 1 71. Let the relation defined as
C

⇒ 1 = 1, which is true. R = {( A, B ) : B = PAQ -1}


Thus, it is reflexive. For reflexive
dy

For Symmetric A = IAI -1


When qRf ⇒ ( A, A ) Î R
u

sec2 q - tan2 f = 1
⇒ R is reflexive
St

⇒ ( 1 + tan2 q) - ( sec2 f - 1) = 1 For symmetric


⇒ 2 + tan2 q - sec2 f = 1 Let ( A, B ) Î R
⇒ sec2 f - tan2 q = 1 \ B = PAQ -1

⇒ fRq ⇒ BQ = PA( Q -1Q )


Thus, it is symmetric. ⇒ BQ = PA
For Transitive ⇒ P ( BQ ) = P -1P ( A )
-1

When qRf and fRy, then ⇒ P -1BQ = A


2 2
sec q - tan f = 1 ⇒ ( B, A ) Î R
and sec2 f - tan2 y = 1 ⇒ R is symmetric.
54 WB JEE (Engineering) · Solved Paper 2014

For transitive ⇒ t ¢n = 1 + [9 + 11 + 13 + 15 + K ( n - 1) term]


Let ( A, B ) Î R, ( B, C ) Î R én -1 ù
=1+ ê {2 ´ 9 + ( n - 2) 2} ú
Then, A = PBQ -1 and B = RCS -1 ë 2 û
⇒ A = PRCS -1Q -1 = 1 + ( n - 1) [9 + n - 2]
⇒ A = ( PR ) C ( QS ) -1 = 1 + ( n - 1) ( n + 7)
t¢ 1 + ( n - 1) ( n + 7)
⇒ ( A, C ) Î R \ tn = n =
n! n!

)
⇒ R is transitive.

be
1 + ( n - 1) ( n + 7)
=
Hence, R is an equivalence relation. n!
72. Case I Let a = w and b = w2 1 + n2 + 6n - 7 n2 + 6n - 6

Tu
= =
302 n n! n!
æ wö
\ S= å ( -1)n çè w2 ÷ø =
n
+
6
-
1

ou
n=0 ( n - 1) ! ( n - 1) ! n !
302
n -1+ 1 6 1
= å ( -1) n ( w2 ) n = + -
( n - 1) ! ( n - 1) ! n !

(Y
n=0

= 1 - w2 + w4 - w6 + w8 - w10 + w12 1 7 1
= + -
( n - 2) ! ( n - 1) ! n !
+ K + w600 - w602 + w604
on
é 1 7 1ù
= 1 - w2 + w - 1 + w2 - w + 1 + K \ lim t n = lim ê + - ú =0
n®¥ n ® ¥ ë ( n - 2) ! ( n - 1) ! n !û
+ 1 - w2 + w
pi

74 . Let time taken from point A to P is t.


= 0 + K + 1 - w2 + w
(v1, t)
m

u
= - w2 - w2 = - 2w2 [Q1 + w + w2 = 0] A P B
v2 , T ö
2
Case II Let a = w and b = w 2ö
ha

302 2 n When a particle moving from point A to P, using


n æw ö
\ S= å ( - 1) ç
è wø
÷ first equation of motion
C

n=0 v = u + at
302 n When final velocity is v 1 and time t 2 , then
æ w4 ö
= å ( -1)n çè w3 ÷ø v 1 = u + at
dy

n=0
Þ v 1 - u = at …(i)
302 When final velocity is v 2 and time T / 2, then
= å ( -1) n ( w)
u

aT
n=0 v2 = u + …(ii)
2
St

= 1 - w + w2 - w3 + w4 - w5 + w6 - K When a particle moving from point P to B using


+ w300 - w301 + w302 first equation of motion
= 1 - w + w2 - 1 + w - w2 + 1 v = u + at
When intial velocity v 1 and time T - t , then
- K + 1 - w + w2
0 = v 1 - a (T - t )
= 0 + K + 1 + w2 - w Þ v 1 = aT - at …(iii)
T
= - w - w = - 2w When initial velocity v 2 and time , then
2
73. Let S = 1 + 10 + 21 + 34 + 49 + K + t ¢n aT
0 = v2 -
and S = 1 + 10 + 21 + 34 + K + t ¢n 2
aT
0 = 1 + 9 + 11 + 13 + 15 + K - t ¢n Þ v2 = Þ aT = 2v 2 …(iv)
2
WB JEE (Engineering) · Solved Paper 2014 55

\ From Eq. (iii), Slope of line y = 1 is m2 = 0


v 1 = 2v 2 - ( v 1 - u ) \ Angle between two curves is
[from Eqs. (i) and (iv)] m - m2
Þ 2v 1 = 2v 2 + u tan q = 1 =3
1 + m1m2
u
Þ v1 = v2 +
2 ⇒ q = tan-1( 3)
\ v1 > v2
78. Given,

)
75. \ Required probability ì x | 1 - t | dt , x > 1
ïò

be
13
C1 ´ 4C2 ´ 12C1 ´ 4C3 f (x ) = í 0 ...(i)
= 52 1
C5 ï x - , x £1
î 2

Tu
13 ´ 6 ´ 12 ´ 4 x
= Now, for x > 1, ò | 1 - t | dt
52 ´ 51 ´ 50 ´ 49 ´ 48 0
5 ´4 ´3 ´2 ´1 1 x

ou
= ò ( 1 - t ) dt + ò1 (t - 1) dt
13 ´ 6 ´ 12 ´ 4 0
= 1 x
52 ´ 51 ´ 10 ´ 49 ´ 2 é t2 ù é t2 ù
= êt - ú + ê - t ú

(Y
6 2 û0 ë 2
= ë û1
4165 2
é 1 ù éx 1 ù
76. We know that u( x ) and v ( x ) are two = ê1 - - 0ú + ê - x - + 1ú
on
ë 2 û ë 2 2 û
independent solutions of the given differential
equation, then their linear combination is also 1 x2 1
= + -x - +1
the solution of the given equation. 2 2 2
pi

Here, we see that y = 5u( x ) + 8v ( x ) is a linear x2


⇒ -x +1
m

combination and y = c1{u( x ) - v ( x )}+ c2v ( x ) is 2


also a linear combination of two independent \ Eq. (i) becomes,
ha

solutions.
ì x2
ï - x + 1, x > 1
77. Given, y = [|sin x | + |cos x |] and x 2 + y 2 = 10 f (x ) = í 2
ï x - 1,
C

We know that (|sin x | + |cos x |) Î [1, 2] x £1


î 2
\ y =1 Continuity at x = 1,
dy

The point of intersection of given curve is LHL = lim f ( x )


x ® 1-
x 2 + 12 = 10 æ 1ö
= lim ç x - ÷ = 1 -
1
u

x ®1 è 2ø 2
⇒ x2 = 9
St

RHL = lim f ( x )
⇒ x = ±3 x ® 1+ 2
æx ö
\ Point of intersection is ( ±3, 1) = lim ç - x + 1÷
x ®1 è 2 ø
Now, x 2 + y 2 = 10
1 1
dy = - 1 + 1=
⇒ 2x + 2y =0 2 2
dx 1 1
dy x and f ( 1) = 1 - =
⇒ =- 2 2
dx y
\ f ( 1) = lim f ( x ) = lim f ( x )
At point ( -3, 1) x ® 1- x ® 1+
dy 3 Hence, f ( x ) is continuous at x = 1.
= = 3 ⇒ m1 = 3
dx 1 Differentiability at x = 1,
56 WB JEE (Engineering) · Solved Paper 2014

f ( 1 - h ) - f ( 1) ⇒ -2( 4g + 3f ) = 5 + 10
LHD = lim
h®0 -h 15
1 æ 1ö ⇒ 4g + 3f = - ...(i)
(1 - h ) - - ç1 - ÷ 2
2 è 2ø -h and 2 [ - g ( 2) + ( - f ) ( -1)] = c - 2
= lim = lim =1
h®0 -h h ® 0 -h
⇒ 2 [ -2 g + f ] = 5 - 2
f ( 1 + h ) - f ( 1)
RHD = lim 3
h®0 h ⇒ -2 g + f = ...(ii)

)
2
( 1 + h )2 æ 1ö

be
- (1 + h ) + 1 - ç1 - ÷ On solving Eqs. (i) and (ii), we get
2 è 2ø
= lim 9 12
h®0 h f =- and g = -
10 10

Tu
1 + h2 + 2h 1 -9 12 27
-1- h + 1- \ fg = ´- =
= lim 2 2 10 10 25
h®0 h

ou
-9
and 4f = 4 ´
1 h2 1 10
+ +h-h-
= lim 2 2 2 -36
⇒ 4f =

(Y
h®0 h 10
h2 ⇒ 4f = 3g
= lim =0
h ® 0 2h
on
\ LHD ¹ RHD
80. Given, P ( A ) = 0.7 and P ( B ) = 0.6
Hence, f ( x ) is not differentiable at x = 1. We know that
pi

P( A Ç B) £ P( B)
79. Centres and constant terms in the circles
2 2 2 2 \ P ( A Ç B ) £ 0.6 ...(i)
x + y - 5 = 0, x + y - 8x - 6y + 10 = 0 and
m

From Booley’s inequality


x 2 + y 2 - 4x + 2y - 2 = 0 are C1¢( 0, 0), c1 = - 5, n
æ n ö
C2¢ ( 4, 3), c2 = 10 and C3¢ ( 2, - 1), c3 = - 2 P ç Ç A i ÷ ³ å P ( A i ) - ( n - 1)
ha

èi =1 ø i =1
Also, centre and constant of circle
x 2 + y 2 + 2gx + 2fy + c = 0 \ P ( A Ç B ) ³ [( P ( A ) + P ( B ) - ( 2 - 1)]
C

⇒ P ( A Ç B ) ³ [0.7 + 0.6 - 1]
is C 4¢ ( - g , - f ) and c 4 = c
Since, the first circle intersect all the three at the ⇒ P ( A Ç B ) ³ 0.3 ...(ii)
dy

extremities of diameter, therefore they are From Eqs. (i) and (ii), we get
orthogonal to each other. 0.3 £ P ( A Ç B ) £ 0.6
u

\ 2( g1g2 + f1f2 ) = c1 + c Hence, options (c) and are (d) always incorrect.
St

\ 2 [ - g ( 0) + ( - f ) ( 0)] = c - 5
⇒ c =5
2 [ - g ( 4) + ( - f ) ( 3)] = c + 10
Engineering Entrance Exam
WB JEE

)
Solved Paper 2013

be
Tu
Physics

ou
Category I
Directions (Q. Nos. 1 to 45) [Carry one mark each, for which only one option is correct. Any
wrong answer will lead to deduction of 1/3 mark.]
1. The r.m.s speed of the molecules of a gas at
100°C is v. The temperature at which the
(Ycase, the inputs to the gates are A = 0, B = 0,
C = 0. In the second case, the inputs are
on
r.m.s. speed will be 3v is A = 1, B = 0, C = 1. The output D in the first
(a) 546°C (b) 646°C case and second case respectively are
pi

(c) 746°C (d) 846°C


A
B
m

2. The equation of state of a gas is given by C D


æ p + a ö ( V - b2 ) = cT, where p, V , T are
ç ÷
è v3 ø
ha

(a) 0 and 0 (b) 0 and 1


pressure, volume and temperature respec- (c) 1 and 0 (d) 1 and 1
tively and a, b, c are constants. The dimens- 5. Two soap bubbles of radii r and 2r are
C

ions of a and b are respectively connected by a capillary tube-valve


(a) [ML 8 T -2 and L 3 / 2 ] arrangement shown in the diagram. The
dy

(b) [ ML 5 T -2 and L 3 ] valve is now opened. Then which one of the


(c) [ ML 5 T -2 and L 6 ] following will result.
(d) [ ML 6 T -2 and L3 / 2 ]
u

3. A frictionless piston-cylinder based enclosure


St

contains some amount of gas at a pressure of Valve


400 kPa. Then heat is transferred fo the gas
at constant pressure in a quasi-static process.
The piston moves up slowly through a height
of 10 cm. If the piston has a cross-sectional
area of 0.3 m2 , the work done by the gas in
(a) The radii of the bubbles will remain unchanged
this process is
(b) The bubbles will have equal radii
(a) 6 kJ (b) 12 kJ (c) 7.5 kJ (d) 24 kJ
(c) The radius of the smaller bubble will decrease and
4. A NOR gate and a NAND gate are connected that of the bigger bubble will decrease
as shown in the figure. Two different sets of (d) The radius of the smaller bubble will decrease and
inputs are given to this setup. In the first that of the bigger bubble will increase
2 | WB JEE (Engineering) l Solved Paper 2013

6. The velocity of a car travelling on a straight 11. Four small objects each of mass m are fixed
road is 3.6 kmh -1 at an instant of time. Now at the corners of a rectangular wire-frame of
travelling with uniform acceleration for 10 s, negligible mass and of sides a and b (a > b).
the velocity becomes exactly double. If the If the wire frame is now roated about an axis
wheel radius of the car is 25 cm, then which passing along the side of length b, then the
of the following is the closest to the number moment of inertia of the system for this axis
of revolutions that the wheel makes during of rotation is
this 10 s? (a) 2 ma2 (b) 4 ma2

)
be
(a) 84 (b) 95 (c) 2 m (a2 + b 2 ) (d) 2 m (a2 - b 2 )
(c) 126 (d) 135
12. The de-Broglie wavelength of an electron

Tu
7. The ionization energy of the hydrogen atom (mass = 1 ´ 10- 30 kg, charge = 16
. ´ 10-19 C)
is 13.6 eV. The potential energy of the with a kinetic energy of 200 eV is (Planck’s
electron in n = 2 state of hydrogen atom is constant = 6.6 ´ 10-34 Js

ou
(a) + 3.4 eV (b) - 3.4 eV
(a) 9.60 ´ 10-11 m (b) 8.25 ´ 10-11 m
(c) + 6.8 eV (d) - 6.8 eV
(c) 6.25 ´ 10-11 m (d) 5.00 ´ 10-11 m

(Y
8. In the electrical circuit shown in the figure,
13. The number of atoms of a radioactive
the current through the 4W resistor is
substance of half-life T is N 0 at t = 0. The
time necessary to decay from N 0 / 2 atoms to
on
3Ω 2Ω
9V N 0 / 10 atoms will be
5
8Ω 4Ω (a) T (b) Tlog 5
pi

2
5 T
(c) Tlog é ù (d) log 5
êë 2 úû 2
m

3Ω 2Ω

(a) 1A (b) 0.5 A 14. A mass M at rest is broken into two pieces
ha

(c) 0.25 A (d) 0.1 A having masses m and ( M - m). The two
masses are then separated by a distance r.
9. A current of 1 A is flowing along positive The gravitational force between them will be
C

x-axis through a straight wire of length the maximum when the ratio of the masses
0.5 m placed in a region of a magnetic field [ m : ( M - m)] of the two parts is
dy

given by B = (2 i + 4 j )T. The magnitude and (a) 1:1 (b) 1:2 (c) 1:3 (d) 1:4
the direction of the force experienced by the
wire respectively are 15. A bullet of mass m travelling with a speed v
u

(a) 18 N, along positive z-axis hits a block of mass M initially at rest and
St

(b) 20 N, along positive x-axis gets embedded in it. The combined system is
(c) 2 N, along positive z-axis free to move and there is no other force
(d) 4 N, along positive y-axis acting on the system. The heat generated in
the process will be
10. S1 and S2 are the coherent point sources of
light located in the xy-plane at points (0,0) (a) Zero
mv 2
and (0, 3l) respectively. Here l is the (b)
wavelength of light. At which one of the 2
Mmv 2
following points (given as coordinates), the (c)
intensity of interference will be maximum? 2 (M - m)

(a) (3l, 0) (b) (4l, 0) mMv 2


(d)
(c) (5l/4,0) (d) (2l/3,0) 2 (M + m)
WB JEE (Engineering) l Solved Paper 2013 | 3

16. A planet moves around the sun in an 21. At two different places, the angles of dip are
elliptical orbit with the sun at one of its foci. respectively 30° and 45°. At these two places
The physical quantity associated with the the ratio of horizontal component of earth’s
motion of the planet that remains constant magnetic field is
with time is (a) 3 : 2 (b) 1 : 2 (c) 1: 2 (d) 1 : 3
(a) velocity (b) centripetal force
(c) linear momentum (d) angular momentum 22. An equilateral triangle is made by uniform
wires AB, BC, CA. A current I enters at A

)
17. A particle of mass M and charge q is and leaves from the mid point of BC. If the

be
released from rest in a region of uniform lengths of each side of the triangle is L, the
electric field magnitude E. After a time t, the magnetic field B at the centroid O of the

Tu
distance travelled by the charge is S and the triangle is
kinetic energy attained by the particle is T.
Then, the ratio T / S A

ou
(a) remains constant with time t
(b) varies linearly with the mass M of the particle O
(c) is independent of the charge q

(Y
(d) is independent of the magnitude of the electric B C
field E m0 æ 4 L ö m0 æ 4 L ö
(a) ç ÷ (b) ç ÷
4p è L ø 2 pè L ø
on
18. The specific heat C of a solid at low m 2 Lö
temperature shows temperature dependence (c) 0 æç ÷ (d) zero
4pè L ø
according to the relation C = DT 3, where D is
pi

a constant and T is the temperature is 23. A particle is moving with a uniform speed v
kelvin. A piece of this solid of mass m kg is in a circular path of radius r with the centre
m

taken and its temperature is raised from at O. When the particle moves from a point P
20 K to 30 K. The amount of heat required in to Q on the circle such that Ð POQ = q, then
ha

the process in energy units is the magnitude of the change in velocity is


(a) 5x10 4 Dm (b) (33/4)x10 4 Dm
(a) 2 v sin (2 q) (b) zero
(c) (65/4)x10 4 Dm (d) (5/4)x10 4 Dm
C

q q
(c) 2 v sin æç ö÷ (d) 2 v cos æç ö÷
19. The least distance of vision of a long sighted è2 ø è2 ø
dy

person is 60 cm. By using a spectacle lens,


this distance is reduced to 12 cm. The power 24. A capacitor of capacitance C0 is charged to a
of the lens is potential V0 and is connected with another
u

capacitor of capacitance C as shown. After


(a) + 5.0 D (b) + (20/3) D
closing the switch S, the common potential
St

(c) -(10/3) D (d) +2.0 D


across the two capacitors becomes V. The
20. A particle of mass M and charge q, initially capacitance C is given by
at rest, is accelerated by a uniform electric S
field E through a distance D and is then
allowed to approach a fixed static charge Q of
the same sign. The distance of the closest
approach of the charge q will then be V0 C0 C
qQ Q C (V - V ) C (V - V0 )
(a) (b) (a) 0 0 (b) 0
4 p e0 D 4 p e0 ED V0 V0
qQ Q C 0 (V + V0 ) C 0 (V0 - V )
(c) (d) (c) (d)
2 p e0 D 2 4 p e0 E V V
4 | WB JEE (Engineering) l Solved Paper 2013

25. As shown in figure below, a charge +2 C is 30. Water is flowing in streamline motion
situated at the origin O and another charge through a horizontal tube. The pressure at a
+5 C is on the x-axis at the point A. The later point in the tube is p where the velocity of
charge from the point A is then brought to a flow is v. At another point, where the
point B on the y-axis. The work done is pressure is p / 2, the velocity of flow is
1 [density of water = r]
(given = 9 ´ 109 m/F) p p
4 p e0 (a) v 2 + (b) v 2 -

)
r r

be
2p 2p
B(0,2)m (c) v 2 + (d) v 2 -
r r

Tu
31. A wire of initial length L and raidus r is
0 (+2C)
stretched by a length l. Another wire of same
A(2,0)m

ou
material but with initial length 2L and
(a) 45 ´ 109 J (b) 90 ´ 109 J radius 2r is stretched by a length 2l. The
(c) zero (d) -45 ´ 109 J ratio of stored elastic energy per unit volume

26. An electric cell of emf E is connected across a


copper wire of diameter d and length l. The (Yin the first and second wire is
(a) 1 : 4 (b) 1 : 2 (c) 2 : 1 (d) 1 : 1
on
drift velocity of electrons in the wire is vd . If 32. Two spheres of the same material, but of
the length of the wire is changed to 2l, the radii R and 3R are allowed to fall vertically
new drift velocity of electrons in the copper downwards through a liquid of density r.
pi

wire will be The ratio of their terminal velocities is


(a) vd (b) 2vd (a) 1 : 3 (b) 1 : 6
m

(c) vd /2 (d) vd / 4 (c) 1 : 9 (d) 1 : 1


ha

27. A bar magnet has a magnetic moment of 33. An alpha particle (4 He) has a mass of
200 Am2 . The magnet is suspended in a 4.00300 amu. A proton has a mass of
magnetic field of 0.30 NA -1m -1. The torque 1.00783 amu and a neutron has a mass of
C

required to rotate the magnet from its 1.00867 amu respectively. The binding
equilibrium position through an angle of energy of alpha estimated from these data is
30°, will be the closest to
dy

(a) 30 N-m (b) 30 3 N-m (a) 27.9 MeV (b) 22.3 MeV
(c) 60 N-m (d) 60 3 N-m (c) 35.0 MeV (d) 20.4 MeV
u

28. An ideal mono-atomic gas of given mass is 34. The equivalent resistance between the
St

heated at constant pressure. In this process, points a and b of the electrical network
the fraction of supplied heat energy used for shown in the figure is
the increase of the internal energy of the gas is
(a) 3/8 (b) 3/5 (c) 3/4 (d) 2/5 r
29. The glass prisms P1 and P2 are to be r r
combined together to produce dispersion a b b
without deviation. The angles of the prisms
r r
P1 and P2 are selected as 4° and 3° r
respectively. If the refractive index of prism
P1 is 1.54, then that of P2 will be (a) 6 r (b) 4 r
(a) 1.48 (b) 1.58 (c) 1.62 (d) 1.72 (c) 2 r (d) r
WB JEE (Engineering) l Solved Paper 2013 | 5

35. An object placed at a distance of 16 cm from a 41. Four identical plates each of area a are
convex lens produces an image of separated by a distance d. The connection is
magnification m ( m > 1). If the object is shown below. what is the capacitance
moved towards the lens by 8 cm, then again between P and Q?
an image of magnification m is obtained. The
numerical value of the focal length of the P Q
lens is

)
(a) 12 cm (b) 14 cm (c) 18 cm (d) 20 cm (a) 2 ae0 / d (b) ae0 / (2 d )

be
(c) ae0 / d (d) 4ae0 / d
36. A travelling acoustic wave frequency 500 Hz
is moving along the positive x-direction with 42. A particle is acted upon by a constant
a velocity of 300 ms -1. The phase difference

Tu
power. Then, which of the following
between two points x1 and x2 is 60°. Then physical quantity remains constant ?
the minimum separation between the two (a) Speed

ou
points is (b) Rate of change of acceleration
(a) 1 mm (b) 1 cm (c) 10 cm (d) 10 mm (c) Kinetic energy
(d) Rate of change of kinetic energy

(Y
37. A shell of mass 5M, acted upon by no
external force and initially at rest, bursts 43. In an n- p -n transistor
into three fragments of masses M, 2M and (a) the emitter has higher degree of doping
2M respectively. The first two fragments
on
compared to that of the collector
move in opposite directions with velocities of (b) the collector has higher degree of doping
magnitudes 2v and v respectively. The third compared to that of the emitter
pi

fragment will (c) both the emitter and collector have same degree
of doping
(a) move with a velocity v in a direction perpendicular
(d) the base region is most heavily doped
m

to the other two


(b) move with a velocity2v in the direction of velocity of 44. The vectors are given by A$ = $i + 2 $j + 2 k$ and
ha

the first fragment $ . Another vector C has the


(c) be at rest B = 3 $i + 6 $j + 2 k
(d) move with velocity v in the direction of velocity of same magnitude as B but has the same
C

the second fragment direction as A. Then which of the following


vectors represents C?
38. A particle moves along x-axis and its
7 $
+ 2 $j + 2 k)
$
dy

displacement at any time is given by (a) (i


3
x( t) = 2t 3 - 3 t2 + 4 t in SI units. The velocity of 3
the particle when its acceleration is zero, is (b) ($i - 2 $j + 2 k)
$
7
u

(a) 2.5 ms -1 (b) 3.5 ms -1 (c) 4.54 ms -1 (d) 8.5 ms -1 7


(c) ($i - 2 $j + 2 k)
$
St

9
39. The fundamental frequency of a closed pipe 9
(d) ($i + 2 $j + 2 k)
$
is equal to the frequency of the second 7
harmonic of an open pipe. The ratio of their
lengths is 45. A car moving at a velocity of 17 ms -1
(a) 1 : 2 (b) 1 : 4 (c) 1 : 8 (d) 1 : 16
towards an approaching bus that blows a
horn at a frequency of 640 Hz on a straight
40. An alternating current in a circuit is given track. The frequency of this horn appears to
by I = 20 sin(100 p t + 005
. p) A. The r.m.s. be 680 Hz to the car driver. If the velocity of
value and the frequency of current sound in air is 340 ms -1, then the velocity of
respectively are the approaching bus is
(a) 10 A and 100 Hz (b) 10 A and 50 Hz (a) 2 ms -1 (b) 4 ms -1
(c) 10 2 A and 50 Hz (d) 10 2 A and 100 Hz (c) 8 ms -1 (d) 10 ms -1
6 | WB JEE (Engineering) l Solved Paper 2013

Category II
Directions (Q. Nos. 46 to 55) [Carry two marks each, for which only one option is correct. Any
wrong answer will lead to deduction of 2/3 mark.]
46. A small mass m, attached to one end of a 50. A magnetic field B = 2t + 4 t2 (where, t =
spring with a negligible mass and an time) is applied perpendicular to the plane
unstretched length L, executes virtual of a circular wire of radius r and resistance
oscillation with angular frequency w0 . When R. If all the units are in SI the electric charge

)
the mass is rotated with an angular speed w

be
that flows through the circular wire during
by holding the other end of the spring at a t = 0 s to t = 2 s is
fixed point, the mass moves uniformly in a 6pr 2 20p r 2
(a) (b)

Tu
circular path in a horizontal plane. Then the
R R
increase in length of the spring during the 32 pr 2 48pr 2
rotation is (c) (d)
R R

ou
w2 L w20 L
(a) (b)
w20 -w 2
w - w20
2
51. Two simple harmonic motions are given by
2
w L w20 L x1 = a sin wt + a cos wt and
(c)
w20
(d)

47. A sphere of radius R has a volume density of


w 2

(Y x2 = a sin wt +
a
3
cos wt

The ratio of the amplitudes of first and


on
charge r = kr, where r is the distance from second motion and the phase difference
the centre of the sphere and k is constant. between them are respectively
3 p 3 p
pi

The magnitude of the electric field which (a) and (b) and
exists at the surface of the sphere is given by 2 12 2 12
2 p 3 p
m

(e 0 = permittivity of free space) (c) and (d) and


4 pk R 4 kR 4 pk R k R2 3 12 2 6
(a) (b) (c) (d)
ha

3 e0 3 e0 e0 4 e0
52. A cylindrical block floats vertically in a liquid
of density r1 kept in a container such that the
48. A body is projected from the ground with a
C

velocity v = (3 i$ + 10 $j ) ms -1. The maximum fraction of volume of the cylinder inside the
height attained and the range of the body liquid is x1. Then some amount of another
respectively are (given g = 10 ms -2 ) immiscible liquid of density r2 (r2 < r1) is
dy

(a) 5 m and 6 m (b) 3 m and 10 m


added to the liquid in the container so that
(c) 6 m and 5 m (d) 3 m and 5 m the cylinder now floats just fully immersed in
u

the liquids with x2 fraction of volume of the


49. A cell of emf E is connected to a resistance R1 cylinder inside the liquid of density r1. The
St

for time t and the amount of heat generated ratio r1 / r2 will be


in it is H. If the resistance R1 is replaced by
1 - x2 1 - x1 x1 - x2 x2
another resistance R2 and is connected to (a) (b) (c) (d) -1
x1 - x2 x1 + x2 x1 + x2 x1
the cell at the same time t, the amount of
heat generated in R2 is 4H. Then internal
53. A particle of mass M and charge q is at rest
resistance of the cell is
at the mid point between two other fixed
2 R1 + R 2 2 R2 - R1
(a) (b) R1R 2 similar charges each of magnitude Q placed
2 R 2 - 2 R1
a distance 2d apart. The system is collinear
R 2 - 2 R1 R2 - R1 as shown in the figure. The particle is now
(c) R1R 2 (d) R1R 2
2 R2 - R1 R2 + R1 displaced by a small amount x ( x < < d)
along the joining the two charges and is left
WB JEE (Engineering) l Solved Paper 2013 | 7

to itself. It will now oscillate about the mean constant and e be the charge of an electron,
position with a time period (e 0 = permittivity then the frequency of light in the second
of free space) case is
e e
d d (a) n1 - (V2 + V1 ) (b) n1 + n (V2 + V1 )
h h
q e e
Q Q (c) n1 - (V2 - V1 ) (d) n1 + (V2 - V1 )
h h
p 3 Me0d p 2 Me0d 3
(a) 2 (b) 2

)
Qq Qq
55. 3 moles of mono-atomic gas ( g = 5 / 3) is mixed

be
with 1 mole of a diatomic gas ( g = 7 / 3). The
p 3 Me0d 3 p 3 Me0 value of g for the mixture will be
(c) 2 (d) 2
Qq Qqd 3 9

Tu
(a)
11
54. The stopping potential for photoelectrons 11
from a metal surface is V1 when (b)
7

ou
monochromatic light of frequency nn is 12
incident on it. The stopping potential (c)
7
becomes V2 when monochromatic light of 15
(d)

(Y
another frequency is incident on the 7
same metal surface. If h be the Planck’s
on
Category III
Directions (Q. Nos. 56 to 60) [Carry two marks each, for which only one or more than one option
may be correct. Marking of correct option will lead to a maximum mark of two on pro data basis.
pi

There will be no negative marking for these questions. However, any marking of wrong option
will lead to award of zero mark against the respective question - irrespective of the number of
m

correct options marked.]


ha

56. An electron of charge e and mass m is


moving in a circular of radius r with a
uniform angular speed w. Then which of the
C

following statements are correct?


(a) The equivalent current flowing in the circular path
dy

is proportional to r 2
(b) The magnetic moment due to circular current loop
is independent of m
u

m F
(c) The magnetic moment due to circular current loop
(a) The tension in the string is F
St

equal to2e / m times the angular momentum of the


electron (b) The tension in the string is 3 N
(d) The angular momentum of the particle is (c) The work done by the tension on the block is 20 J
proportional to the areal velocity of electron. during this 1s
(d) The work done against the force of gravity is 10 J
57. A block of mass m ( = 01
. kg) is hanging over a
frictionless light fixed pulley by an 58. If E and B are the magnitudes of electric and
inextensible string of negligible mass. The magnetic fields respectively in some region
other end of the string is pulled by a constant of space, then the possibilities for which a
force F in the vertically downward direction. charged particle may move in that space
The linear momentum of the block increases with a uniform velocity of magnitude v are
by 2 kgms -1 in 1s after the block starts from (a) E = vB (b) E ¹ 0, B = 0
rest. Then, (given g = 10 ms -2 ) (c) E = 0, B ¹ 0 (d) E ¹ 0, B ¹ 0
8 | WB JEE (Engineering) l Solved Paper 2013

59. A bar of length marrying a small mass m at (c) The total distance travelled by the mass in air is
one of its ends rotates with a uniform proportional to w2
(d) The total distance travelled by the mass in air and
angular speed win a vertical plane about the
its total time of flight are both independent on its
mid point of the bar. During the rotation, at
mass.
some instant of time when the bar is
horizontal, the mass is detached from the 60. A biconvex lens of focal length f and radii of
bar but the bar continues to rotate with curvature of both the surfaces R is made of a
some w. The mass moves vertically up, comes

)
material of refractive index n1. This lens is

be
back and reaches the bar at the same point. placed in a liquid of refractive index n2 . Now
At that place, the acceleration due to gravity this lens will behave like
is g. (a) either as a convex or as a concave lens

Tu
w2 l depending solely on R
(a) This possible if the quantity is an integer
2pg (b) a convex lens depending on n1 and n2
(c) a concave lens depending on n1 and n2
(b) The total time of flight of the mass is proportional

ou
(d) a convex lens of same focal length irrespective of
to w2
R, n1 and n2

Chemistry
Category I (Y
on
Directions (Q. Nos. 1 to 45) [Carry one mark each, for which only one option is correct. Any
wrong answer will lead to deduction of 1/3 mark.]
1. At 25°C, the solubility product of salt of MX2 4. (+)-2-chloro-2-phenylethane in toluene
pi

type is 3.2 ´ 10-8 in water. The solubility racemises slowly in the presence of small
m

(in mol/L) of MX2 in water at the same amount of SbCl2 , due to the formation of
temperature will be (a) carbanion (b) carbene
(a) 1.2 ´ 10-3
ha

(c) free-radical (d) carbocation


(b) 2 ´ 10-3
5. Acid catalysed hydrolysis of ethyl acetate
(c) 3.2 ´ 10-3
C

follows a pseudo-first order kinetics with


(d) 1.75 ´ 10-3 respect to ester. If the reaction is carried out
with large excess of ester, the order with
dy

2. The IUPAC name of the compound X is


respect to ester will be
O CN (a) 1.5 (b) 0
 
u

(X = C C ) (c) 0.5 (d) 1


CH3
St

CH3 CH2 CH3 6. The different colours of litmus in acidic,


(a) 4-cyano-4-methyl-2-oxopentane neutral and basic solutions are, respectively.
(b) 2-cyano-2-methyl-4-oxopentane (a) red, orange and blue
(c) 2, 2-dimethyl-4-oxopentanenitrile (b) blue, violet and red
(d) 4-cyano-4-methyl-2-pentanone (c) red, colourless and blue
(d) red, violet and blue
3. In SOCl2 , the Cl ¾ S ¾ Cl and Cl ¾ S ¾ O
bond angles are 7. Baeyer’s reagent is
(a) 130° and 115° (a) alkaline potassium permanganate
(b) 106° and 96° (b) acidified potassium permanganate
(c) 107° and 108° (c) neutral potassium permanganate
(d) 96° and 106° (d) alkaline potassium manganate
WB JEE (Engineering) l Solved Paper 2013 | 9

8. The correct order of equivalent conductances 14. Chlorine gas reacts with red hot calcium
at infinite dilution in water at room oxide to give
temperature for H + , K + , CH 3COO- and HO- (a) bleaching powder and dichlorine monoxide
ions is (b) bleaching powder and water
(a) HO - > H+ > K + > CH3COO - (c) calcium chloride and chlorine dioxide
(b) H+ > HO - > K + > CH3COO - (d) calcium chloride and oxygen
(c) H+ > K + > HO - > CH3COO -
(d) H+ > K + > CH3COO - > HO - 15. For a chemical reaction at 27°C, the

)
activation energy is 600 R. The ratio of the

be
9. Nitric acid can be obtained from ammonia rate constants at 327°C to that of at 27°C
via the formations of the intermediate will be
compounds (d) e 2

Tu
(a) 2 (b) 40 (c) e
(a) nitric oxide and nitrogen dioxide
(b) nitrogen and nitric oxide 16. 2-methylpropane on monochlorination
(c) nitric oxide and dinitrogen pentoxide under photochemical condition give

ou
(d) nitrogen and nitrous oxide (a) 2-chloro-2-methylpropane as major product
(b) (1 : 1) mixture of 1-chloro-2-methylpropane and
10. In the following species, the one which is

(Y
2-chloro-2-methylpropane
likely to be the intermediate during benzoin (c) 1-chloro-2-methylpropane as a major product
condensation of benzaldehyde is (d) (1 : 9) mixture of 1-chloro-2-methylpropane and
Å Å OH 2-chloro-2-methylpropane
on
(a) Ph ¾ C ºº O (b) Ph ¾ C
CN 17. The half-life for decay of 14
C by b-emission is
s OH s 5730 yr. The fraction of 14 C decays, in a
pi

(c) Ph ¾ C (d) Ph ¾ C == O
sample that is 22920 yr old, would be
CN
1 1 7 15
m

(a) (b) (c) (d)


11. In O2 and H2O2 , the O ¾ O bond lengths are 8 16 8 16
o
ha

1.21 and 1.48 A respectively. In ozone, the 18. A van der Waals’ gas may behave ideally when
average O ¾ O bond length is (a) the volume is very low
(a) 1.28 Å (b) 1.18 Å (c) 1.44 Å (d) 1.52 Å (b) the temperature is very high
C

(c) the pressure is very low


12. The change of entropy (dS) is defined as (d) the temperature, pressure and volume all are very
dq dH
dy

(a) dS = (b) dS = high


T T
dqrev dH - dG 19. The optically active molecule is
(c) dS = (d) dS =
u

T T
COOMe COOMe
St

13. Correct pair of compounds which gives blue


colouration/precipitate and white precipitate, HO H D OH
respectively, when their Lassaigne’s test is (a) (b)
HO H D OH
separately done is
(a) NH2NH2 , HCl and ClCH2COOH COOMe COOMe
(b) NH2CSNH2 and PhCH2Cl
COOMe COOH
(c) NH2NH2 , COOH and NH2CONH2
H Me H OH H OH
N COOH (c) (d)
H OH H OH
 
Cl
(d) and COOH COOH
10 | WB JEE (Engineering) l Solved Paper 2013

20. In diborane, the number of electrons that 26. When aniline is nitrated with nitrating
accounts for bonding in the bridges is mixture in ice cold condition, the major
(a) six (b) two product obtained is
(c) eight (d) four (a) p-nitroaniline (b) 2, 4-dinitroaniline
(c) o-nitroaniline (d) m-nitroaniline
21. The reaction of nitroprusside anion with
sulphide ion gives purple colouration due to 27. The measured freezing point depression for
the formation of a 0.1 m aqueous CH 3COOH solution is

)
0.19°C. The acid dissociation constant

be
(a) the tetranionic complex of iron (II) coordinating to
one NOS - ion K a at this concentration will be (Given, K f
(b) the dianoinic complex of iron (II) coordinating to the molal cryoscopic constant = 1.86 K
one NCS - ion kg mol -1)

Tu
(c) the trianoionic complex of iron (III) coordinating to
one NOS - ion (a) 4.76 ´ 10-5 (b) 4 ´ 10-5
-5
(d) the tetranionic complex of iron (III) coordinating to (c) 8 ´ 10 (d) 2 ´ 10-5

ou
one NCS - ion
28. The ore chromite is
22. At 25°C, pH of a 10-8 M aqueous KOH (a) FeCr2O 4 (b) CoCr2O 3
solution will be
(a) 6.0
(c) 8.02
(b) 7.02
(d) 9.02
(Y (c) CrFe 2O 4

29. ‘Sulphan’ is
(d) FeCr2O 3
on
(a) a mixture of SO 3 and H2SO 5
23. An optically active compound having (b) 100% conc. H2SO 4
molecular formula C2H16 on ozonolysis gives (c) a mixture of gypsum and conc. H2SO 4
pi

acetone as one of the products. The structure (d) 100% oleum (a mixture of 100% SO 3 in 100%
of the compound is H2SO 4 )
m

H3 C CH3 H3 C H
30. Pressure-volume (pV) work done by an
ideal gaseous system at constant volume is
ha

(a) C==C H (b) C==C CH3


H 3C C H 3C C (where E is internal energy of the system)
H5C2 H H5C2 H
(a) -Dp / p (b) zero (c) -VDp (d) -DE
C

H3CH2HC CH3 H3 C CH2CH3 31. Amongst [Ni(H2O) 6 ] 2 +, [Ni(PPh 3)2 Cl2 ],


(c) C==C CH3 (d) C==C CH3 [Ni(CO)4 ] and [Ni(CN4 ) ] 2 - the paramagnetic
dy

H C H 3C C species are
H3 C H H H
(a) [NiCl 4 ]2- , [Ni(H2O)6 ]2+ , [Ni(PPh3 )2 Cl 2 ]
u

(b) [Ni(CO)4 ], [Ni(PPh3 )2 Cl 2 ], [NiCl 4 ]2-


24. Mixing of two different ideal gases under (c) [Ni(CN)4 ]2- , [Ni(H2O)6 ]2+ , [NiCl 4 ]2-
St

isothermal reversible condition will lead to (d) [NI(PPh3 )2 Cl 2 ], [Ni(CO)4 ], [Ni(CN)4 ]2-
(a) increase of Gibbs free energy of the system
(b) no change of entropy of the system 32. Ribose and 2-deoxyribose can be
(c) increase of entropy of the system differentiated by
(d) increase of enthalpy of the system
(a) Fehling’s reagent (b) Tollen’s reagent
(c) Barfoed’s reagent (d) Osazone formation
25. The ground state electronic configuration of
CO molecule is 33. Number of hydrogen ions present in
(a) 1s2 2 s2 1p 4 3s2 10 milionth part of 1.33 cm 3 of pure water
(b) 1s2 2 s2 3s2 1p 2 2 p 2 at 25°C is
(c) 1s2 2 s2 1p 2 3s2 2 p 2 (a) 6.023 million (b) 60 million
(d) 1s2 1p 2 2 s2 2 s2 (c) 8.01 million (d) 80.23 million
WB JEE (Engineering) l Solved Paper 2013 | 11

34. The correct order of acid strength of the 41. The condition of spontaneity of a process is
following substituted phenols in water at (a) lowering of enthropy at constant temperature and
28°C is pressure
(a) p-nitrophenol < p-fluorophenol < p-chlorophenol (b) lowering of Gibbs free energy of system at
(b) p-chlorophenol < p-fluorophenol < p-nitrophenol constant temperature and pressure
(c) p-fluorophenol < p-chlorophenol < p-nitrophenol (c) increase of entropy of system at constant
(d) p-flurophenol < p-nitrophenol < p-chlorophenol temperature and pressure
(d) increase of Gibbs free energy of the universe at

)
35. For isothermal expansion of an ideal gas, constant temperature and pressure

be
the correct combination of the
thermodynamic parameters will be 42. The increasing order of O ¾ N ¾ O bond
(a) DU = 0, Q = 0, W ¹ 0 and DH ¹ 0 angle in the species NO2 , NO2 + and NO2 - is

Tu
(b) DU ¹ 0, Q ¹ 0, W ¹ 0 and DH = 0 (a) NO 2 + < NO 2 < NO 2 - (b) NO 2 < NO -2 < NO+2
(c) DU = 0, Q ¹ 0, W = 0 and DH ¹ 0 (c) NO+2 < NO -2 < NO 2 (d) NO 2 < NO+2 < NO -2
(d) DU = 0, Q ¹ 0, W ¹ 0 and DH = 0

ou
43. The correct structure of the dipeptide gly-ala is
36. Addition of excess potassium iodide solution
to a solution of mercuric chloride gives the CH3 O H

(Y
O
  


halide complex (a) H2N CHC NH CC
(a) tetrahedral K 2 [HgI4 ] 


OH
(b) trigonal K[HgI3 ] H
on
(c) linear Hg 2I2 CH2SH O
(d) square planar K 2 [HgCl 2I2 ]  
(b) NH2 CH C NH CH2 COH

pi

37. Amongst the following, the one which can O


exist in free state as a stable compound is
m

H O O
(a) C 7H9O (b) C 8H12O (c) C 6H12O (d) C10H17O   

(c) H2NCCNH  CH C—OH
ha

38. A conductivity cell has been calibrated with  


a 0.01 M 1 : 1 electrolyte solution (specific H CH3
conductance, k = 1.25 ´ 10-3 S cm -1) in the O
C

H O
cell and the measured resistance was 800 W   
at 25°C. The cell constant will be (d) H2N CCNHCHC—OH
 
dy

(a) 1.02 cm -1 (b) 0.102 cm -1 H CH2SH


(c) 1.00 cm -1 (d) 0.5 cm -1
u

39. The orange solid on heating gives a 44. Equivalent conductivity at infinite dilution
for sodium-potassium oxalate
St

colourless gas and a green solid which can be


reduced to metal by aluminium powder. The [(COO- )2 Na +K + ] will be [given molar
orange and the green solids are, respectively conductivities of oxalate, K + and Na + ions at
infinite dilution are 148.2, 50.1,
(a) NH4Cr2O 7 and Cr2O 3 (b) Na 2Cr2O 7 and Cr2O 3
73.5 S cm2 mol -1 respectively]
(c) K 2Cr2O 7 and CrO 3 (d) (NH4 )2 CrO 4 and CrO 3
(a) 271.8 S cm 2 eq -1 (b) 67.95 S cm 2 eq -1
40. The best method for the preparation of (c) 543.6 S cm 2 eq -1 (d) 135.9 S cm 2 eq -1
2, 2-dimethylbutane is via the reaction of
45. For BCl 3, AlCl 3 and GaCl 3 the increasing
(a) Me 3CBr and MeCH2Br in Na/ether
order of ionic character is
(b) (Me 3C)2 CuLi and MeCH2Br
(a) BCl 3 < AlCl 3 <GaCl 3 (b) GaCl 3 < AlCl 3 <BCl 3
(c) (MeCH2 )2 CuLi and Me 3CBr
(d) Me 3CMgI and MeCH2I (c) BCl 3 <GaCl 3 < AlCl 3 (d) AlCl 3 <BCl 3 <GaCl 3
12 | WB JEE (Engineering) l Solved Paper 2013

Category II
Directions (Q. Nos. 46 to 55) [Carry two marks each, for which only one option is correct. Any
wrong answer will lead to deduction of 2/3 mark.]
46. In borax the number of B ¾ O ¾ B links and O CBr3 O CH2Br
B ¾ OH bonds present are, respectively C C
(a) five and four (b) four and five  
(c) three and four (d) five and five (c) (d)

)
be
47. Reaction of benzene with Me 3CCOCl in the
presence of anhydrous AlCl 3 gives 52. The standard Gibbs free energy change
( DG o ) at 25°C for the dissociation of N2O4 ( g)

Tu
Me3C O to NO2 ( g) is (given, equilibrium constant
N CMe3 = 0.15, R = 8.314 JK/mol)
 
(a) 1.1 KJ

ou
(a) (b) (b) 4.7 KJ
(c) 8.1 KJ
CMe3 AlCl3
(d) 38.2 KJ

(Y
Me3C O

C
 53. Silicon oil is obtained from the hydrolysis
(c)  (d) and polymerisation of
on
C (a) trimetylchlorosilane and dimethyldichlorosilane
Me3C O (b) trimethylchlorosilane and methyldichlorosilane
(c) methyltrichlorosilane and dimethyldichlorosilane
pi

48. 1 ´ 10-3 mole of HCl is added to a buffer (d) triethylchlorosilane and dimethyldichlorosilane
solution made up of 0.01 M acetic acid and
m

0.01 M sodium acetate. The final pH of the F


D  D
buffer will be (given, pK a of acetic acid is
ha

54. Treatment of with NaNH2 /liq. NH 3


4.75 at 25°C)
gives
(a) 4.60 (b) 4.66 (c) 4.75 (d) 4.8
C

NH2 NH2
49. On heating, chloric acid decomposes to H  D 
H D
(a) HClO 4 , Cl 2 , O 2 and H2O (a) (b)
dy

(b) HClO 2 , Cl 2 , O 2 and H2O


(c) HClO, Cl 2O and H2O 2 F NH2 H
(d) HCl, HClO, Cl 2O, and H2O H  D  HD 
u

H NH2
50. The best method for preparation ofMe 3CCN is (c) (d) +
St

(a) to react Me 3COH with HCN


(b) to react Me 3CBr with NaCN 55. Identify the correct statement.
(c) to react Me 3CMgBr with ClCN
(a) Quantum numbers (n, l, m, s) are arbitrarily
(d) to react Me 3CLi with NH2CN
(b) All the quantum numbers (n, l, m, s) for any pair of
51. Bromination of PhCOMe in acetic medium electron in an atom can be identical under special
produces mainly circumstance
Me O Me O (c) All the quantum numbers (n, l, m, s) may not be
C C required to describe an electron of an atom
 Br  completely
(a) (b) (d) All the quantum numbers (n, l, m, s) are
required to describe an electron of an atom

Br completely
WB JEE (Engineering) l Solved Paper 2013 | 13

Category III
Directions [Q. Nos. 56 to 60] [Carry two marks each, for which only one or more than one
option may be correct. Marking of correct option will lead to a maximum mark of two on pro
data basis. There will be no negative marking for these questions. However any marking of
wrong option will lead to award of zero mark against the respective question - irrespective of
the number of correct options marked.]
56. In basic medium the amount of Ni2 + in a 58. Tautomerism is exhibited by

)
solution can be estimated with the

be
(a) (Me3CCO)3CH (b) O
dimethylglyoxime reagent. The correct
statement(s) about the reaction and the O

Tu
product is (are) (c) O (d)
O O
(a) in a ammoniacal solution Ni 2+ salts give
cherry-red precipitate of nickel(II) O

ou
dimethylglyoximate 59. The important advantage(s) of Lintz and
(b) two dimethylglyoximate units are bound to one Donawitz (L.D.) process for the manufacture
Ni 2+

(Y
of steel is (are)
(c) in the complex two dimethylglyoximate units are (a) the process is very quick
hydrogen bonded to each other (b) operating costs are low
(c) better quality steel is obtained
on
(d) each dimethylglyoximate unit forms a six
membered chelate ring with Ni 2+ (d) scrap iron can be used
60. Consider the following reaction for
pi

57. Correct statement(s) in cases of n-butanol 2NO2 ( g) + F2 ( g) ¾¾® 2NO2F( g). The
and t-butanol is (are) expression for the rate of reaction in terms of
m

(a) both are having equal solubility in water the rate of change of partial pressure of
(b) t-butanol is more soluble in water than n-butanol reactant and product is/are
ha

(c) boiling point of t-butanol is lower than n-butanol 1 é dp (NO 2 )ù 1 é dp (NO 2 )ù


(a) rate = - (b) rate =
(d) boiling point of n-butanol is lower than t-butanol 2 êë dt úû 2 êë dt úû
C

1 é dp (NO 2F )ù 1 é dp (NO 2F )ù
(c) rate = - úû (d) rate = 2 êë
2 êë dt dt úû
u dy

Mathematics
St

Category I
Directions (Q. Nos. 1 to 60) [Carry one mark each, for which only one option is correct. Any
wrong answer will lead to deduction of 1/3 mark.]
1. Each of a and b can take values 1 or 2 with 2. Cards are drawn one-by-one without
equal probability. The probability that the replacement from a well shuffled pack of 52
equation ax2 + bx + 1 = 0 has real roots, is cards. Then, the probability that a face card
equal to (jack, queen or king) will appear for the first
1 1 1 1 time on the third turn is equal to
(a) (b) (c) (d)
2 4 8 16 300 36 12 4
(a) (b) (c) (c)
2197 85 85 51
14 | WB JEE (Engineering) l Solved Paper 2013

3. There are two coins, one unbiased with 7. A point P lies on the circle x2 + y2 = 169. If
1 Q = (5, 12) and R = ( -12, 5), then the ÐQPR
probaility or getting heads and the other
2 is
3
one is biased with probability of getting (a)
p
(b)
p
(c)
p
(d)
p
4 6 4 3 2
heads. A coin is selected at random and
tossed. It shows heads up. Then, the 8. A point moves, so that the sum of squares of
probability that the unbiased coin was its distance from the points (1, 2) and ( -2, 1)

)
selected is is always 6. Then, its locus is

be
2 3 3 1
(a) (b) (a) the straight line y - = -3 æç x + ö÷
3 5 2 è 2ø
1 2

Tu
(c) (d) 1 3 1
2 5 (b) a circle with centre æç - , ö÷ and radius
è 2 2ø 2
4. Lines x + y = 1 and 3 y = x + 3 intersect the (c) a parabola with focus (1, 2) and directrix passing

ou
ellipse x2 + 9 y2 = 9 at the points P, Q and R. through (-2, 1)
The area of the DPQR is (d) an ellipse with foci (1, 2) and (-2, 1)
36 18

(Y
(a) (b) 9. A circle passing through (0, 0), (2, 6),
5 5
9 1 (6, 2) cut the x-axis at the point P ¹ (0, 0).
(c) (d)
5 5 Then, the lenght of OP, where O is the
on
origin, is
5. For the variable , the locus of the point of 5 5
(a) (b) (c) 5 (d) 10
intersection of the lines 3 tx - 2 y + 6t = 0 and 2 2
pi

3 x + 2ty - 6 = 0 is
x 2 y2 10. For the variable t, the locus of the points of
m

(a) the ellipse + =1 1


4 9 intersection of lines x - 2 y = t and x + 2 y = is
2 2 t
x y
ha

(b) the ellipse + =1 (a) the straight line x = y


9 4
(b) the circle with centre at the origin and radius 1
x 2 y2
(c) the hyperbola - =1 (c) the ellipse with centre at the origin and one focus
C

4 9
æ 2 ö
x 2 y2 ç , 0÷
(d) the hyperbola - =1 è 5 ø
9 4
dy

(d) the hyperbola with centre at the origin and one


6. The locus of the mid-points of the chords of æ 5 ö
focus ç , 0÷
an ellipse x2 + 4 y2 = 4 that are drawn from è 2 ø
u

the positive end of the minor axis, is


St

1 11. The number of onto functions from the set


(a) a circle with centre æç ,0ö÷ and radius 1
è2 ø {1, 2, ..., 11 } to the set {1, 2, ..., 10} is
1 11 !
(b) a parabola with focus æç ,0ö÷ and directrix x = -1 (a) 5 ´ 11 ! (b) 10 ! (c) (d) 10 ´ 11 !
è2 ø 2
1
(c) an ellipse with centre æç 0, ö÷ , major axis 1 and 12. Let p ( x) be a quadratic polynomial with
è 2ø
constant term 1. Suppose p ( x), when divided
1
minor axis by x - 1 leaves remainder 2 and when
2
1 divided by x + 1 leaves remainder 4. Then,
(d) a hyperbola with centre æç 0, ö÷ , transverse axis 1 the sum of the roots of p ( x) = 0 is
è 2ø
1 1 1
and conjugate axis (a) -1 (b) 1 (c) - (d)
2 2 2
WB JEE (Engineering) l Solved Paper 2013 | 15

é 1 (2013) x 1 ù 19. The number of solutions of the equation


13. The limit of ê 2
+ x - x ú as x ® 0
ëx e -1 e - 1û x + y + z = 10 where x, y and z are positive
integers
(a) approaches +¥ (b) approaches -¥
(a) 36 (b) 55
(c) is equal to loge (2013) (d) does not exist
(c) 72 (d) 45
14. Eleven apples are distributed among a girl
20. The value of the integral
and a boy, Then, which one of the following
x2013

)
statements is true ? 1 ì 1ü
ò-1 íî e|x|( x2 + cos x) + e|x|ýþ dx is equal to

be
(a) atleast one of them will receive 7 apples
(b) the girl receives atleast 4 apples or the boy
receives atleast 9 apples (a) 0 (b) 1 - e -1

Tu
(c) the girl receives atleast 5 apples or the boy (c) 2e -1 (d) 2 (1 - e -1 )
receives atleast 8 apples
(d) the girl receives atleast 4 apples or the boy p
21. For 0 £ P, Q £ , if sin P + cos Q = 2, then the

ou
receives atleast 8 apples 2
æ P + Qö
15. If z1 = 2 + 3 i and z2 = 3 + 4 i be two points on value of tan ç ÷ is equal to
è 2 ø

(Y
the complex plane. Then, the set of complex
number z satisfying | z - z1 |2 + | z - z2 |2 1
(a) 1 (b)
= | z1 - z2 |2 represents 2
1 3
on
(a) a straight line (b) a point (c) (d)
(c) a circle (d) a pair of straight line 2 2

16. Five numbers are in HP. The middle term is 22. If f ( x) = 2100 x + 1, g ( x) = 3100 x + 1, then the
pi

1 and the ratio of the second and the fourth set of real numbers x such that f { g( x)} = x is
terms is 2 : 1. Then, the sum of the first three (a) empty
m

terms is (b) a singleton


11 (c) a finite set with more than one element
ha

(a) (b) 5 (d) infinite


2
14
(c) 2 (d) ì1 1ü
23. The limit of í 1 + x - 1 + 2 ý as x ® 0
C

3
îx x þ
æcos p p æ 1 ö
ç - sin ö÷ ç ÷ (a) does not exist (b) is equal to
1
dy

17. If p = ç 4 4 and X = ç 2 ÷. Then, 2


p p ÷ 1
çsin cos ÷ ç ÷ (c) is equal to 0 (d) is equal to 1
è 4 4 ø è 2ø
u

P 3 X is equal to 24. The value of cos2 75° + cos2 45° + cos2 15°
- cos2 30° - cos2 60° is
St

æ- 1 ö
0 ç ÷ (a) 0 (b) 1
(a) æç ö÷ (b) ç 2 ÷ 1 1
è1 ø ç 1 ÷ (c) (d)
è 2 ø 2 4
æ- 1 ö
-1 ç ÷ 25. The maximum and minimum values of
(c) æç ö÷ (d) ç 2 ÷ cos 6 q + sin 6 q are respectively
è 0ø ç- 1 ÷ 1
è 2ø (a) 1and
4
(b) 1and 0
18. If a and b are roots of x2 - x + 1 = 0, then the
(c) 2 and 0
value of a 2013 + b2013 is 1
(d) 1 and
(a) 2 (b) -2 (c) -1 (d) 1 2
16 | WB JEE (Engineering) l Solved Paper 2013

26. If z = x + iy, where x and y are real numbers æ1 0 0ö æ1 0 0ö


and i = -1, then the points ( x, y) for which 33. If I = ç0 1 0÷ and P = ç0 -1 0 ÷. Then,
z-1 ç0 0 1÷ ç0 0 -2÷ø
is real, lie on è ø è
z-i 3 2
the matrix P + 2 P is equal to
(a) an ellipse (b) a circle (a) P (b) I – P
(c) a parabola (d) a straight line (c) 2I + P (d) 2I - P

27. If a, b and c are in AP, then the straight line 34. The value of determinant

)
ax + 2by + c = 0 will always pass through a

be
fixed point whose coordinates are 1 + a2 - b2 2ab -2b
(a) (1, - 1) (b) (-1, 1) 2ab 1 - a2 + b2 2a is
(c) (1, - 2 ) (d) (-2, 1) 2 2

Tu
2b -2a 1- a - b
28. The equation 2x2 + 5xy - 12 y2 = 0 represents (a) 0 (b) (1 + a2 + b 2 )
a (c) (1 + a2 + b 2 )2 (d) (1 + a2 + b 2 )3

ou
(a) circle
(b) pair of non-perpendicular intersecting straight 35. If a , b are the roots of the quadratic equation
x2 + ax + b = 0, ( b ¹ 0), then the quadratic

(Y
lines
(c) pair of perpendicular straight lines 1 1
equation whose roots are a - , b - , is
(d) hyperbola b a
(a) ax2 + a(b - 1)x + (a - 1)2 = 0
on
29. If one end of a diameter of the circle
3 x2 + 3 y2 - 9 x + 6 y + 5 = 0 is (1, 2), then the (b) bx2 + a(b - 1) x + (b - 1)2 = 0
other end is (c) x2 + ax + bv = 0
pi

(a) (2, 1) (b) (2, 4) (c) (2, - 4) (d) (-4, 2 ) (d) abx2 + bx + a = 0
m

30. The line y = x intersects the hyperbola 36. If the distance between the foci of an ellipse
x2 y2 is equal to the length of the latusrectum,
- = 1 at the points P and Q. The
ha

9 25 then its eccentricity is


eccentricity of ellipse with PQ as major axis (a)
1
( 5 - 1) (b)
1
( 5 + 1)
5
C

and minor axis of length is 4 2


2 1
(c) ( 5 - 1)
1
(d) ( 5 + 1)
5 5 5 2 2 2 4
(a) (b) (c) (d)
dy

3 3 9 9
37. The equation of the circle passing through
æ 1ö the point (1, 1) and the points of intersection
u

31. The limit of x sin çç e x ÷÷ as x ® 0 of x2 + y2 - 6x - 8 = 0 and x2 + y2 - 6 = 0 is


è ø
St

(a) x2 + y2 + 3 x - 5 = 0 (b) x2 + y2 - 4 x + 2 = 0
(a) is equal to 0 (b) is equal to 1 2 2
(c) x + y + 6 x - 4 = 0 (d) x2 + y2 - 4 y - 2 = 0
e
(c) is equal to (d) does not exist
2 38. The number of lines which pass through the
point (2, - 3) and are at a distance 8 from the
32. The value of
point ( -1, 2) is
é 1 1 1 1 ù (a) infinite (b) 4 (c) 2 (d) 0
1000 ê + + +...+ ú
ë1 ´ 2 2 ´ 3 3 ´ 4 999 ´ 1000 û
39. Six positive numbers are in GP, such that
is
their product is 1000. If the fourth term is 1,
(a) 1000 (b) 999 then the last term is
1
(c) 1001 (d) 1 1
999 (a) 1000 (b) 100 (c) (d)
100 1000
WB JEE (Engineering) l Solved Paper 2013 | 17
1000
40. If a and b are the roots of the quadratic 46. The limit of å ( -1) n x n as x ® ¥
equation ax2 + bx + c = 0 and 3 b2 = 16ac, n=1
then
(a) does not exist
(a) a = 4 b or b = 4 a (b) a = - 4 b or b = - 4 a (b) exists and equals to 0
(c) a = 3 b or b = 3 a (d) a = - 3 b or b = - 3 a (c) exists and approaches to + µ
(d) exists and approaches -¥
41. In the set of all 3 ´ 3 real matrices a relation
is defined as follows. A matrix A is related to 47. Let f (q ) = (1 + sin2 q ) (2 - sin2 q). Then, for

)
a matrix B, if and only if there is a

be
all values of q
non-singular 3 ´ 3 matrix P, such that 9
(a) f(q) > (b) f(q) < 2
B = P -1 AP. This relation is 4

Tu
(a) reflexive, symmetric but not transitive 11 9
(c) f(q) > (d) 2 £ f(q) £
(b) reflexive, transitive but not symmetric 4 4
(c) symmetric, transitive but not reflexive
48. If f ( x) = ex ( x - 2)2 , then

ou
(d) an equivalence relation
(a) f is increasing in (- µ, 0) and (2, µ) and decreasing
42. For any two real numbers a and b, we define in (0, 2 ).
a R b if and only if sin2 a + cos2 b = 1. The

(Y
(b) f is increasing in (- µ, 0) and decreasing in (0, µ).
relation R is (c) f is increasing in (2, µ) and decreasing in (- µ, 0).
(a) reflexive but not symmetric (d) f is increasing in (0, 2 ) and decreasing in (- µ, 0)
on
(b) symmetric but not transitive and (2, µ).
(c) transitive but not reflexive
(d) an equivalence relation 49. Let n be a positive even integer. If the ratio
pi

of the largest coefficient and the 2nd largest


43. For the curve x2 + 4 xy + 8 y2 = 64 the coefficient in the expansion of (1 + x) n is
tangents are parallel to the x-axis only at the
m

11 : 10. Then, the number of terms in the


points expansion of (1 + x) n is
ha

(a) (0, 2 2 ) and (0, - 2 2 ) (a) 20 (b) 21


(b) (8, - 4) and (-8, 4) (c) 10 (d) 11
(c) (8 2 , - 2 2 ) and (-8 2 , 2 2 )
C

(d) (9, 0) and (-8, 0) 50. Five numbers are in AP with common
difference ¹ 0. If the 1st, 3rd and 4th terms
ì x3 - 3 x + 2 , x < 2,
44. If f ( x) = í are in GP, then
dy

3 2
î x - 6x + 9 x + 2, x³2
(a) the 5th term is always 0.
then (b) the 1st term is always 0.
u

(a) lim f( x) does not exist (c) the middle term is always 0.
x® 2 (d) the middle term is always -2.
St

(b) f is not continuous at x = 2


(c) f is continuous but not differentiable at x = 2 51. Let exp (x) denote the exponential function
(d) f is continuous and differentiable at x = 2 æ 1ö
p ex . If f ( x) = exp çç x x ÷÷, x > 0, then the
45. The (tan n + 1 x) dx è ø
value of I= ò
4
0
1
p minimum value of f in the interval [2, 5] is
+ ò 2 tan n + 1 æ x ö dx is
ç ÷ æ 1ö æ 1ö
2 0 è2ø
(a) exp çe e ÷ (b) exp ç2 2 ÷
1 n+2 ç ÷ ç ÷
(a) (b) è ø è ø
n 2n + 1 æ 1ö æ 1ö
2n - 1 2n - 3 (c) exp ç 5 5 ÷ (d) exp ç 3 3 ÷
(c) (d) ç ÷ ç ÷
n 3n - 2 è ø è ø
18 | WB JEE (Engineering) l Solved Paper 2013

52. The minimum value of the function 56. The value of the integral
f ( x) = 2|x - 1|+ |x - 2|is x + 1ö
2 xæ
(a) 0 (b) 1 (c) 2 (d) 3 ò1 e çlog e x +
è x ø
÷ dx is

53. The sum of the series (a) e 2 (1 + loge 2 ) (b) e 2 - e


2
1 25 1 25 1 25 (c) e (1 + loge 2 ) - e (d) e 2 - e (1 + loge 2 )
C0 + C1 + C2 + ...
1´2 2´3 3´4
57. The number of solutions of the equation

)
1 25

be
+ C25 is 1 æ x + 1ö
26 ´ 27 log 2
÷ + log 9 ( x + 5) = 1 is

2 è x + 5ø
2 27 - 1 2 27 - 28
(a) (b)

Tu
26 ´ 27 26 ´ 27 (a) 0 (b) 1 (c) 2 (d) infinite
1 æ 2 26 + 1 ö æ 2 26 - 1ö 1 1
(c) çç ÷ (d) çç ÷÷ 58. If P = 1 + + +K
2 è 26 ´ 27 ÷ø è 52 ø 2 ´ 2 3 ´ 22

ou
54. If P, Q and R are angles of an isosceles 1 1 1
p and Q= + + + ..., then

(Y
triangle and Ð P = , then the value of 1´2 3 ´ 4 5´6
2
3 (a) P = Q (b) 2P = Q
æ P Pö
çcos - i sin ÷ + (cos Q + i sin Q) (c) P = 2Q (d) P = 4Q
è 3 3ø
on
(cos R - i sin R) + (cos P - i sin P) 59. The area of the region bounded by the
(cos Q - i sin Q) (cos R - i sin R) is parabola y = x2 - 4 x + 5 and the straight
pi

(a) i (b) -i (c) 1 (d) -1 line y = x + 1 is


1 9
(a) (b) 2 (c) 3 (d)
m

55. Let f : R ® R be such that f is injective and 2 2


f ( x) f ( y) = f ( x + y) for all x, y Î R, if p 3p
60. If f ( x) = sin x + 2 cos2 x, £x£ . Then, f
ha

f ( x), f ( y) and f ( z) are in G P, then x, y and z 4 4


are in attains its
(a) AP always p p
C

(a) minimum at x = (b) maximum at x =


(b) GP always 4 2
p 1
(c) AP depending on the values of x, y and z (c) minimum x = (d) maximum at x = sin-1 æç ö÷
è 4ø
dy

(d) GP depending on the values of x, y and z 2

Category II
u

Directions [(Q. Nos. 61 to 75) Carry two marks each, for which only one option is correct. Any
St

wrong answer will lead to deduction of 2/3 mark.]


61. An objective type test paper has 5 questions. 5 3 3 3
(a) (b) (c) (d)
Out of these 5 questions, 3 questions have 32 128 256 64
four options each (a, b, c, d) with one option 62. The solution of the differential equation
being the correct answer. The other dy
2 questions have two options each, namely ( y2 + 2x) = y satisfies x = 1, y = 1. Then,
dx
true and false. A candidate randomly ticks the solution is
the options. Then, the probability that (a) x = y2 (1 + loge y) (b) y = x2 (1 + loge x)
he/she will tick the correct option in atleast 2
(c) x = y (1 - loge y) (d) y = x2 (1 - loge x)
four questions, is
WB JEE (Engineering) l Solved Paper 2013 | 19

63. A family of curves is such that the length 1 1 1


68. If x=1+ + + + ... and
intercepted on the y-axis between the origin 2 ´ 1! 4 ´ 2! 8 ´ 3 !
and the tangent at a point is three times the x2 x4 x 6
y=1+ + + + ...
ordinate of the point of contact. The family of 1! 2! 3 !
curves is Then, the value of log e y is
(a) xy = C, C is a constant 1
(a) e (b) e 2 (c) 1 (d)
(b) xy2, = C, C is a constant e
(c) x2 y = C, C is a constant

)
æ 2 -2 -4 ö

be
(d) x2 y2 = C, C is a constant 69. If P = ç -1 3 4 ÷, then P5 is equal to
ç 1 -2 -3 ÷
64. The solution of the differential equation è ø

Tu
æxö ì æxö ü (a) P (b) 2P (c) -P (d) -2P
y sin ç ÷ dx = í x sin ç ÷ - yý dy satisfying
è yø î è y ø þ 12 + 22
70. The value of the infinite series

ou
æ
y ç ÷ = 1 is 3!
è4ø 12 + 22 + 32 12 + 22 + 32 + 42
x 1
+ + + ...is
4! 5!

(Y
(a) cos = - loge y +
y 2
5e 1 5e
x 1 (a) e (b) 5 e (c) - (d)
(b) sin = loge y + 6 2 6
y 2
on
x 1 p
(sin x - x cos x)
(c) sin = loge x -
y 2 71. The value of integral òp3 dx is
x 1
x ( x + sin x)
(d) cos = - loge x - 6
pi

y 2 ì 2 (p + 3) ü ì p+ 3 ü
(a) loge í ý (b) loge í ý
î(2 p + 3 3 )þ î2 (2 p + 3 3 )þ
m

65. A line passing through the point of ì2 p + 3 3 ü ì2 (2 p + 3 3 )ü


intersection of x + y = 4 and x - y = 2 makes (c) loge í ý (d) loge í ý
î 2 (p + 3) þ p+ 3
ha

3 î þ
an angle tan -1 æç ö÷ with the x-axis. It
è4ø 72. If f ( x) = x2 / 3, x ³ 0. Then, the area of the
2
intersects the parabola y = 4 ( x - 3) at region enclosed by the curve y = f ( x) and the
C

points ( x1, y1) and ( x2 , y2 ) respectively. Then, three lines y = x, x = 1 and x = 8 is


|x1 - x2 |is equal to 63 93 105 129
dy

(a) (b) (c) (d)


16 32 2 5 7 10
(a) (b)
9 9
40 80 æ 1 1 1 ö
u

(c) (d) 73. If f ( x) = x ç + + ÷, x > 1. Then,


9 9 è x - 1 x x + 1ø
St

66. If sin2 q + 3 cos q = 2, then cos 3 q + sec3 q is (a) f( x) £ 1 (b) 1 < f( x) £ 2


equal out to (c) 2 < f( x) £ 3 (d) f( x) > 3
(a) 1 (b) 4 (c) 9 (d) 18 74. If P be a point on the parabola y2 = 4 ax with
67. If [a] denote the greatest integer which is focus F. Let Q denote the foot of the
less than or equal to a. Then, the value of the perpendicular from P onto the directrix.
p tan Ð PQF
integral ò 2p [sin x cos x ] dx is Then, is
- tan Ð PFQ
2
p 1 1
(a) (b) p (a) 1 (b) (c) 2 (d)
2 2 4
p
(c) -p (d) -
2
20 | WB JEE (Engineering) l Solved Paper 2013

x cos t (b) F is increasing in (0, p ) and decreasing in (p, 2 p ).


75. If F ( x) = ò0 dt, 0 £ x £ 2p. Then,
(1 + t2 ) (c) F is increasing (p, 2 p ) and decreasing in (0, p ).
p 3p
p 3p ö
(a) F is increasing in æç , æ pö
÷ and decreasing in ç 0, ÷ (d) F is increasing in æç 0, ö÷ and æç , 2 p ö÷
è2 2 ø è 2ø è 2ø è2 ø
æ 3p ö p 3 pö
and ç , 2 p ÷. and decreasing in æç , ÷.
è2 ø è2 2 ø
Category III

)
Directions (Q. Nos. 76 to 80) [Carry two marks each, for which one or more than one option is

be
correct. Marking of correct option will lead to a maximum mark of two on pro data basis. There
will be no negative marking for these questions. However any marking of wrong option will
lead to award of zero mark against the respective question - irrespective of the number of

Tu
correct options marked.]
76. The equations of the circles, which touch 79. Consider the system of equations

ou
both the axes and the line 4 x + 3 y = 12 and x + y + z = 0 ax + by + g z = 0
have centres in the first quadrant, are a 2 x + b2 y + g 2 z = 0
(a) x2 + y2 + x - y + 1 = 0 Then, the system of equations has
(b) x2 + y2 - 2 x - 2 y + 1 = 0
(c) x2 + y2 - 12 x - 12 y + 36 = 0 (Y(a) a unique solution for all values of a, b and g .
(b) infinite number of solutions, if any two of a, b, g are
equal.
on
(d) x2 + y2 - 6 x - 6 y + 36 = 0
(c) a unique solution, if a, b and g are distinct.
(d) more than one, but finite number of solutions
77. The area of the region enclosed between depending on values of a, b and g .
1
pi

parabola y2 = x and the line y = mx is .


48 80. Which of the following real valued functions
m

Then, the value of m is is/are not even functions?


(a) -2 (b) -1 (c) 1 (d) 2 (a) f( x) = x3 sin x
ha

78. If sin a, cos a be the roots of the equation (b) f( x) = x2 cos x


2
x - bx + c = 0. Then, which of the following (c) f( x) = e x x3 sin x
C

statements is/are correct ? (d) f( x) = x - [ x], where [ x] denotes the greatest


1 1 integer less than or equal to x.
(a) c £ (b) b £ 2 (c) c > (d) b > 2
2 2
u dy
St
Answers
Physics
1. (d) 2. (a) 3. (b) 4. (d) 5. (d) 6. (b) 7. (d) 8. (b) 9. (c) 10. (b)
11. (a) 12. (b) 13. (c) 14. (a) 15. (d) 16. (d) 17. (a) 18. (c) 19. (b) 20. (b)
21. (a) 22. (d) 23. (c) 24. (d) 25. (c) 26. (c) 27. (a) 28. (b) 29. (d) 30. (a)
31. (d) 32. (c) 33. (a) 34. (d) 35. (a) 36. (c) 37. (c) 38. (a) 39. (b) 40. (c)
41. (a) 42. (d) 43. (a) 44. (a) 45. (b) 46. (a) 47. (d) 48. (a) 49. (b) 50. (b)

)
51. (a) 52. (a) 53. (c) 54. (d) 55. (b) 56.(b, d) 57.(a, b, d) 58.(a, c, d) 59.(a, c, d) 60. (b, c)

be
Chemistry

Tu
1. (b) 2. (c) 3. (d) 4. (d) 5. (b) 6. (d) 7. (a) 8. (b) 9. (a) 10. (c)
11. (a) 12. (c) 13. (d) 14. (d) 15. (c) 16. (c) 17. (d) 18. (c) 19. (c) 20. (d)
21. (a) 22. (b) 23. (b) 24. (c) 25. (a) 26. (a) 27. (b) 28. (a) 29. (d) 30. (b)

ou
31. (a) 32. (d) 33. (c) 34. (c) 35. (d) 36. (a) 37. (b) 38. (c) 39. (b) 40. (b)
41. (b) 42. (*) 43. (c) 44. (d) 45. (c) 46. (a) 47. (b) 48. (b) 49. (a) 50. (c)
51. (d) 52. (b) 53. (a) 54. (d) 55. (d) 56. (a, b, c,d) 57. (b,c) 58.(a, b, d) 59.(a, c, d) 60. (a, d)

(*) No option is correct.


Mathematics
(Y
on
1. (b) 2. (c) 3. (d) 4. (b) 5. (a) 6. (c) 7. (b) 8. (b) 9. (c) 10. (d)
11. (d) 12. (d) 13. (a) 14. (d) 15. (c) 16. (a) 17. (c) 18. (b) 19. (a) 20. (d)
pi

21. (d) 22. (b) 23. (b) 24. (c) 25. (a) 26. (d) 27. (a) 28. (b) 29. (c) 30. (d)
31. (a) 32. (b) 33. (c) 34. (d) 35. (b) 36. (c) 37. (a) 38. (d) 39. (c) 40. (c)
m

41. (d) 42. (d) 43. (b) 44. (c) 45. (a) 46. (c) 47. (d) 48. (a) 49. (b) 50. (a)
51. (c) 52. (b) 53. (b) 54. (b) 55. (a) 56. (c) 57. (c) 58. (c) 59. (d) 60. (c)
61. (d) 62. (a) 63. (c) 64. (*) 65. (b) 66. (d) 67. (d) 68. (a) 69. (a) 70. (c)
ha

71. (a) 72. (d) 73. (d) 74. (a) 75. (d) 76. (b, c) 77. (a, d) 78. (a, b) 79. (b, c) 80. (c, d)
C
u dy
St
Hints & Solutions
Physics
3 RT 4T
1. vrms = 5. Pressure difference =
M r

)
4T

be
Given, T = 100° C = 373 K For smaller soap, patm - pi1 =
r
3 R ´ 373
\ v= 4T
M For bigger soap, patm - pi2 =

Tu
2r
3 RT
For 3v= As the pressure inside smaller bubble is greater
M than pressure inside bigger bubble, so air flows

ou
3v 3 RT / M from smaller to bigger and thus radius of the
\ =
v 3 R ´ 373 / M smaller bubble will decrease and that of the
bigger bubble will increase.
Þ 3=
T
373
T
(Y
6. We have, q = 2 pn =
æ vf2 vi2 ö
ç 2 - 2÷
èr r ø
on
Þ 3= æ aö
373 ç2 ÷
è rø
Þ T = 1119 K
pi

vf2 - vi2
or T = 846° C Þ n= » 95
(2 ar ) (2 p )
aù a
m

2. [P ] = éê Þ [ML-1T -2 ] = 3 3
ë V 3 úû [L ] 7. Given, En = 13.6 eV
ha

8 -2
Þ a = [ML T ] Energy of an electron in nth state
[V ] = [b2 ] Þ [ L3 ] = b2 Þ b = [ L3 / 2 ] -13.6 z 2eV
En =
n2
C

3. Wconstant pressure = p ´ DV
\ Energy of an electron in n = 2 state
= 400 ´ 10 3 ´ 0.3 ´ 10 ´ 10 -2 18 - 6 z 2
dy

E2 = = -3.4 eV
= 400 ´ 10 ´ 3 (2 )2
= 12000 = 12 kJ So, PE = 2 En = 2
u

4. In first case A = 0, B = 0 = 2 ´ (-3.4)


St

\ Output of NOR gate, Y = A + B = 1 » - 6.8 eV


This output is the input for NAND gate, i.e., Y = 1 8. The current through the various branches of the
and C = 0 circuit will be shown as
\ D = Y ×C = 1 I B I1
A C
In second case A = 1, B = 0 3Ω 2Ω
\Output of NOR gate, Y = A + B = 0 9V

This output is the input for NAND gate i . e ., Y = 0 8Ω 4Ω


(I–I1)
and C = 1
I1
\ D = Y ×C = 1 2Ω 2Ω
F D
I E
WB JEE (Engineering) l Solved Paper 2013 | 23

According to Kirchhoff’s second law in closed Moment of inertia of the system about side of
circuit BCDEB length b say CD is
2 I1 + 4I1 + 2 I1 - 8 ( I - I1 ) = 0 = M.I. of mass at A about CD + M.I. of mass at B
Þ 16 I1 - 8 I = 0 about CD + M . I. of mass at C about CD + M.I. of
8I 1 mass ot D about CD
Þ I1 = Þ I1 = I …(i) = m(a)2 + m(a)2 + m(0 )2 + m(0 )2
16 2
In closed circuit ABEFA = 2 ma2

)
- 9 + 3 I + 8 (I - I1 ) + 2 I = 0 h

be
12. The de-Broglie wavelength, l =
æ1 ö 2 mk
Þ 13 I - 8 I1 = 9 Þ 13 I - 8 ç I ÷ = 9
è2 ø
Given, h = 6.6 ´ 10 -34 J -s

Tu
9
Þ I = = 1A …(ii) m = 1 ´ 10 -30 kg
9
1 K = 200 eV = 200 ´ 1.6 ´ 10 -19 J

ou
So current through 4 W resistor I1 = ´ 1 = 0 .5 A
2 Substituting all these values
9. The force experienced by the wire placed in 6.6 ´ 10 -34

(Y
l=
magnetic field, F = Bil 2 ´ 1 ´ 10 -30 ´ 200 ´ 1.6 ´ 10 -19
^ ^ æ ^ 1ö
= (2 i + 4 j ) ç1 i ´ ÷ = 0.825 ´ 10 -10 = 8.25 ´ 10 -11 m
è 2ø
on
1^ ^ N(t )
^ ^ ^
= i ´ i + 4 ´ (i ´ j ) = 2 k N 13. We have, = e - lt or N(t ) = N0 e - lt
2 N0
pi

Direction of this force can be find out by \For the given condition,
Fleming’s left hand rule which is along pasitive N0 N
= N0 e - lt 1 and 0 = N0 e - lt 2
m

z-axis. 2 10
10. The given situation can be show as 1 1
= e - lt 1 and = e - lt 2
ha

Þ
2 10
S2 or e lt 1 = 2 and e lt 2 = 10
(0, 3λ)
C

Taking log on both sides,



log 2
lt1 = log 2 Þ t1 =
dy

l
S1 log 2 ´ T
(0, 0) (4λ, 0) or t1 =
log2
u

The intensity will be maximum at those given


and lt 2 = log 10
St

points where the path difference between the


two interfering waves is an integral multiple of log 10
Þ t2 =
wavelength i . e ., Dx = nl l
For the the given points, the intensity will be log 10 ´ T
or t2 =
maximum for (4l, 0 ). log 2
11. The given situation can be shown as é log 10 ù é log 10 - log 2 ù
\ (t 2 - t1 ) = T ê - 1ú = T ê ú
A a Dm ë log 2 û ë log 2 û
m
é log 5 ù
b = Tê ú
ë log 2 û
m C é 5ù
B m Þ ( t 2 - t1 ) = T log ê ú
ë2û
24 | WB JEE (Engineering) l Solved Paper 2013

14. The gravitation force between two masses, 16. A planet revolves around the sun, is an elliptical
Gm1m2 orbit under the effect of gravitational pull on the
F= planet.
r
Planet
here, m1 = m
and m2 = (M - m)
Gm (M - m) Fg
\ F=
r2

)
dF

be
For maximum gravitational force, =0 Sun
dm
d
\ [m(M - m)] = 0

Tu
dm
M So, torque, C = r ´ F = r F sin 180° n = 0
By solving, we get m = dL
2 As C = ; so L = a constant

ou
m 1 dt
So, =
( M - m) 1 Þ Angular momentum is constant.

(Y
17. Given, mass of the particle = M,
15. Mass of bullet = m1 = m
Charge on the particle = q
Initial speed of bullet = u1 = v
Electric field = E
on
Mass of block = m2 = M
Initial velocity, u = 0
Initial speed of block = u 2 = 0
F qE
\ Acceleration, a = =
pi

Let the common velocity of the bodies after M M


collision = V
\Distance travelled in electric field,
m

According to conservation of linear momentum 1


S = ut + at 2
m ´ v + M ´ 0 = (m + M ) V 2
ha

mv 1 æ qE ö
V= S = ç ÷ t2
(m + M ) 2 èMø
C

2
\Heat generated = loss in KE 1 æ qEt ö
Also, kinetic energy T = Mç ÷
= Initial KE - Final KE 2 è M ø
dy

2
1 1 1 æ qEt ö
= mv 2 - (m + M ) v 2 Mç ÷
2 2 T 2 è M ø
So, = = qE
u

2 S 1 æ qE ö 2
1 1 æ mv ö ç ÷t
St

= mv 2 - (m + M ) ç ÷ 2 èMø
2 2 è m + Mø
T
Þ Ratio of remains constant with time t.
1 1 m2 v 2 S
= mv 2 -
2 2 (m + M ) 18. Amount of heat required, Q = ò dQ
1 æ m ö T2 = 30
= mv 2 ç1 - ÷ =ò mcdT …(i)
2 è m + Mø T1 = 20

1 æ m + M - mö Given, C = DT 3
= mv 2 ç ÷ 30 30
2 è m+ M ø Q = ò m DT 3 dT = mD 3
\
20 ò20 T dT
1 mM 1 65
= v2 = mD [(30 )2 - (20 )2 ] = ´ 10 4 mD
2 (m + M ) 4 4
WB JEE (Engineering) l Solved Paper 2013 | 25

19. Here, v = - 60 cm, u = - 12 cm 23. v


Q
1 1 1
\ By using the relation, = -
f v u v
1 1 1 θ P
We have, = - O r
f -60 -12
1 1 100
Þ = cm or m
f 15 15

)
100 20 \Change in magnitude of velocity

be
So, the power of lens, P = =+ D
15 3 | Dv| = v 2 + v 2 - 2 v 2 cos q
20. At the distance of closest approach, the entire q

Tu
K E of a particle is converted into electric = 2 v sin
2
potential energy.
24. When the switch S is closed, the common
i . e .,KE =PE

ou
potential across the two capacitors becomes V,
Here, KE of the particle = work done in moving so charge on isolated plates remains same.
the particle in uniform electric field (E ) through a
\ C0V0 = (C0 + C )V

(Y
distance (D )
Þ CV = C0V0 - C0V
= qE ´ D
C ( V - V)
PE of the particle = potential ´ charge Þ C= 0 0
V
on
q
= ´Q 25. Work done = U f - U i
4pe 0 r0
1 æ 1 1ö
pi

qR Q = ´2 ´ 5´ ç - ÷ = 0
\ qED = Þ r0 = è2 2ø
4pe 0 r0 4pe 0 ED 4pe 0
m

21. The horizontal component of earth’s magnetic 26. The drift velocity is given as
field is given by, H = R cos d i E
ha

vd = =
where, d is the angle of dip so H1 = R cos 30° neA R ´ neA
and H2 = R cos 45° E´A E
= =
C

H1 R cos 30° 3/2 3 rl ´ neA Ine


\ = = =
H2 R cos 45° 1 / 2 2 When length of wire changed to 2l, the new drift
dy

velocity,
22. The given situation can be shown as
E
vd ¢ =
r ´ 2 l ´ ne
u

I
A vd ¢ E / r2 lne 1
St

\ = =
vd E / r lne 2
vd
Þ vd ¢ =
O 2

B C
27. Given, M = 200 A -m 2 B = 0.30 NA -1M-1
and q = 30°
I
We know that the Torque,
The magnetic field at centroid O = Magnetic field t =M´B
due to left part + Magnetic field due to right part
1
\ B = B1 + B2 = 0 Þ | t | = MB sin q = 200 ´ 0.3 ´
2
(Qdirection ofB due to both parts is different of O) = 100 ´ 0.3 = 30 N-m
26 | WB JEE (Engineering) l Solved Paper 2013

DU CV 34. The equivalent circuit for the given electrical


28. Fraction = =
DQ Cp network is
Cp 5 r
For monoatomic gas =g=
CV 3 r r
CV 1 3 a b
So, = = r r
Cp g 5 r

)
29. Given for prism P1 r r

be
m = 1. 54, A = 4°
a 2r b
For prism P2

Tu
m¢ = ?, A¢ = 3° r r

For no deviation = d + d¢ = 0 This is a balanced Wheatstone bridge, so the

ou
Þ (m - 1)A = (m ¢ - 1)A¢ arm containing 2 r not in use. So, we have
Þ (1.54 - 1)4 = (m ¢ - 1)3 r r
On solving we get m¢ = 1.72
30. As the water is flowing through the horizontal
tube, so in the streamline flow of water, the sun of
(Y a

r r
b
on
static pressure and dynamic pressure is
constant i . e ., 2r
1 P 1 a b
pi

P + rv 2 = + rv12
2 2 2 2r
m

P
Þ v1 = + v2 So, the equivalent resistance between aand b is
r 1 1 1 2r
ha

= + Þ r¢ = =r
31. Elastic energy per unit volume of wire. r ¢ 2r 2r 2
1 f
m= ´ Young’s modulus ´ (Strain) 2 35. Linear magnification, m =
C

2 f +u
2
u1 ì (Strain)1 ü l 2 4 L2 As given that magnification is same for both
dy

\ =í ý = 2 ´ 2 = 1: 1 cases this is possible if the two different values of


u 2 î (Strain)2 þ L 4l
u are of opposite sign.
2 r 2 (r - s ) g f -f
u

32. The terminal velocity, v = i . e ., =


9h f - 16 f - 8
St

As, all other parameters are constant, therefore Þ 16 - f = f - 8


v µ r2 Þ 2 f = 24
vR (R )2 1 or f = 12 cm
Þ = =
v3 R (3R )2 9 36. By using the relation, v = nl
33. The mass defect, Dm = 2(mp + mn ) - m He v 300 3
We have, l=
= = m
n 500 5
= 2(1.00783 + 1.00867 ) - 4.00300
2p
= 0.0300 amu The phase difference, f = (Dx )
l
So, the binding energy, E = Dmc 2 p 2p ´ 5 1
Þ = (Dx ) Þ Dx = m = 10 cm
= 0.03 ´ 931 MeV » 27.9 MeV 3 3 10
WB JEE (Engineering) l Solved Paper 2013 | 27

37. Let the velocity of third fragment is v¢. Then by the From figure, it is clear that we have two
conservation of linear momentum capacitors C1 and C2 . Positive plates of C1 are
5 (0 ) = M ´ 2 v - 2 M ´ v + 2 M ´ v ¢ connected to positive plate of C2 and negative
plate to negative. Therefore C1 and C2 are in
Þ v¢ = 0 parallel.
i . e ., the third fragment will be at rest. 2e a
So, Cp = C1 + C2 = 2C = 0
38. Given, x (t ) = 2 t 3 - 3 t 2 + 4 t d
dx

)
So, velocity, v = = (6 t 2 - 6 t + 4)
42. We have

be
dt dW
Power = Rate of doing work =
dv dt
and acceleration a = = (12 t - 6)
dt dW

Tu
or, P= = rate of change in KE
when acceleration is zero, i . e .,(12 t - 6) = 0 dt
6 1 dW d (KE )
or t = = s i . e ., P= = = constant

ou
12 2 dt dt
\Velocity of the particle at zero acceleration is 43. In an n-p-n or p-n-p transistor, the left hand side
2
thick layer of the transistor is heavily doped

(Y
æ 1ö æ 1ö
v = 6 ç ÷ - 6 ç ÷ 4 = 2.5 m/s known as emitter and right hand side thick layer
è2ø è2ø
of the transistor is moderately doped known as
39. Let the length of closed pipe is L1 and that of pipe collector.
on
is L2 . 44. (a) Given, A = $i + 2 $j + 2 k$ and B = 3 $i + 6$j + 2k$
v
Fundamental frequency of closed pipe, n1 = $i + 2 $j + 2k$
pi

4L1 So, C= ´ 32 + 62 + 2 2
1+ 4 + 4
and frequency of second harmonic of open pipe,
m

v $i + 2 $j + 2k$ 7
n2 = 2 ´ = ´ 49 = ( $i + 2 $j + 2k$ )
2 L2 3 3
ha

Given, n1 = n 2 45. Given, velocity of sound, v = 340 m/s


v 2v L 1
i . e ., = Þ 1 = Velocity of listner, vL = 17 m/s
C

4 L1 2 L2 L2 4
Velocity of source = vS
40. Given, I = 20 sin (100pt + 0.05 p ) Frequency of horn emitted
dy

The root mean square value of alternating v = 640 Hz


I 20
current = Irms = 0 = = 10 2 A
u

2 2 Car Bus
17 m/s
St

Also, w = 100 p
Þ 2 pf = 100 p or f = 50 Hz
41. Suppose the pair of plates is connected to The apparent frequency
positive terminal of the battery and the pair of ( v + vL )
plates Q is connected to the negative terminal of n¢ = n
v - vs
the battery.
æ 340 + 17 ö
+ + + + + + + + d 680 = 640 ç ÷
è 340 - vS ø
P Q
+ + + + + + + + d On solving we get vS = 4 m/s
28 | WB JEE (Engineering) l Solved Paper 2013

46. The given situation can be shown as 49. We have, H = I 2 R


Kx cos θ
According to given condition, I12 R1 = H
and I22 R2 = 4 H
E2 E2
θ Kx Þ R = H and
2 1
R2 = 4 H
(R1 + r ) (R2 + r )2

Kx sin θ R2 4 R1
\ =

)
(R2 + r )2 (R1 + r )2

be
mg Þ R2 (R1 + r ) = 2 R1 (R2 + r )
2
From figure, Kx sin q = mw (L + x ) sin q R1R2 [ R1 - 2 R2 ]

Tu
On solving, r=
2 2 R1 - R2
Þ Kx = mw (L + x ) …(i)
K 50. Given, B = 2 t + 4 t 2

ou
Also, = w0
m
at t = 0, B1 = 0
Þ K = mw20 and at t = 2, B2 = 2 ´ 2 + 4(2 )2
Substituting the value in Eq. (i)
mw20 x 2
= mw (L + x ) (Y We have, DQ =
= 4 + 16 = 20Wb/m 2
Df pr 2 (B2 - B1 )
on
wL 2 =
Þ x= R R
w20 - w2 pr 2 [20 - 0 ] 20 pr 2
= =
pi

47. Given, r = K × r R R
By Gauss’s theorem 51. The given situation can be shown as
m

2 a
E (4pr 2 ) =
ò r ´ 4pr dr a/ 3
ha

e0
2 2a and 2a 3
=
ò Kr ´ 4pr dr
C

e0 π/4 π/6
a a
2
Kr
Þ E= Fon second SHM
dy

4e 0
a1 3
Here r =R Ratio of amplitude = =
a2 2
u

KR 2 p p p
So, E=
St

4eo and phase difference, - =


4 6 12
48. (a) Given, v = (3 $i + 10 $j ) m/s 52. The given situation is shown as
Þ v x = 3 and vy = 10
(1- x1 ) (1- x2 ) l
vy2 2
\Maximum height attained, H = l x2
2g x1 1 l
1

10 ´ 10
= = 5m As the Bouyant force in both the cases are same
2 ´ 10
\ r1 x1g = r1 x2 g + r 2 (1 - x2 ) g
2 vy 3 ´ 2 ´ 10
Range = v x ´ T = v x ´ = = 6m r1 (1 - x2 )
g 10 On solving =
r 2 ( x1 - x2 )
WB JEE (Engineering) l Solved Paper 2013 | 29

53. Restoring force on displacement of x 57. The free body diagram


é q Qq ù F - mg = 2
F=K ê 2
- ú
ë (d - x ) (d + x )2 û
é 1 1 ù
= KQq ê 2
- ú
ë (d - x ) (d + x )2 û T=F
é 4 dx ù
= KQq ê 2 F

)
2 2ú
ë (d - x ) û

be
mg
é 4dx ù
= KQq ê 4 ú If (d >> x )
ëd û Þ F = 2 + mg = 3 N

Tu
é 4x ù unbalanced force 2
Þ F = KQq ê 3 ú also, a = = = 20 m /s 2
ëd û mass 0 .1

ou
F 4KQqx 1 2 1
Acceleration, a = = \ S = at = ´ 20 ´ 1 = 10 m
m Md 3 2 2
4KQq Hence, work done by tension

(Y
or w2 =
Md 3 = F ´ 10 = 3 ´ 10 = 30 J
2p Md 3 So, the work done against gravity
\ T= =2p
on
w 4KQq = mg ´ S = 1 ´ 10 = 10 J
3 3
p Md e 0 58. The charged particle will move with uniform
=2
pi

Qq velocity in that space if E = 0 and B ¹ 0, since a


moving charge in a magnetic field experience a
54. Here, hn1 = f 0 = eV1 …(i)
m

force. Also if E ¹ 0, B ¹ 0, the charged particle


and hn 2 = f 0 + eV2 …(ii) will move with uniform velocity in that space.
ha

From Eqs. (i) and (ii), we have If E = vB, then particle again moves with uniform
velocity.
h(n 2 - n1 ) = e (V2 - V1 )
1
e 59. We have, v = wl
C

Þ n 2 = (V2 - V1 ) + n1 2
h
v
55. The number of degrees of freedom for the
dy

mixture is
m
n1f1 + n2 f2
fmix =
u

n1 + n2
St

3 ´ 3 + 1´ 5 7
= = 2 v wl 2p wl
4 2 T= = or n =
2 g g w g
\ g = 1+
f (as completes n rotations with in T)
Þ g = 1+ =
4 11 lw2
\ n=
7 7 2 pg
56. Magnetic moment, m = IA 2 v 2 l 2 w2
Distance travelled = 2 h = =
ev evr 2g 4g
´ pr 2 =
=
2 pr 2
60. The lens will behave like a convex lens or a
dA
Angular momentum = 2m concave lens depending upon the value of n1
dt and n2 .
30 | WB JEE (Engineering) l Solved Paper 2013

Chemistry
1. For MX2 type salt, 7. Baeyer’s reagent is 1% cold dilute alkaline
3 -8 3 potassium permanganate. It is used to identify
Solubility product, Ksp = 4s ; 3 . 2 ´ 10 = 4s
unsaturation. All unsaturated compounds lose
3.2 ´ 10 -8 its purple colour.
or s=3 = 2 ´ 10 -3 mol/L
4 8. Equivalent conductance (Leq ) is defined as the

)
1 conducting power of all the ions produced by
2. O CN

be

one gram equivalent of an electrolyte in a given


4 2
5 C 3 C CH3 solution.In case of weak electrolytes (as
CH3 CH2 CH3 CH 3COOH, NH 4OH, AgCl etc.), ionisation is very

Tu
2, 2-dimethyl-4-oxopentanenitrile small compared to strong electrolytes (as KCl,
NaOH etc.), hence Leq of weak electrolyte
3. In SOCl 2 , the Cl—S—Cl and Cl—S—O bond is low. Moreover, ionisation of H + is

ou
angles are respectively 96° and 106°. maximum among given ions. Thus, correct of
4. A planar carbocation is generated when SbCl 5 order of Leq is
remove Cl - from the substrate. This carbocation H+ > OH - > K+ > CH 3COO -
is subsequently attacked by Cl - (nucleophile)
from both the sides (i.e. from top and bottom) to
produce a racemic mixture.
(Y
9. Ostwald process of for manufacture of nitric
acid,
on
Pt gauge
5. CH 3COOC 2H 5 + H 2O r CH COOH3
4NH 3 + SO 2 ¾¾¾¾® 4NO + 6H 2O
ammonia 800 - 900 °C nitric oxide
+ C 2H 5OH
pi

If above reaction is carried out with large excess DH = - 21.5 kcal


of ester, the rate of reaction does not depend 50 °C
2NO + O 2 ¾¾® 2NO 2
m

upon cencentration of both the reactants. Due to


nitrogen dioxide
which, the reaction becomes of zero order.
4NO 2 + 2H 2O + O 2 ¾® 4HNO 3
ha

6. Medium Colour of Litmus Paper nitric acid


Acidic Red
Neutral Violet
C

Basic Blue
dy

10. When benzaldehyde is heated with aqueous ethanolic NaCN or KCN, it dimerises to form an a-hydroxy
ketone called benzoin, and this reaction is formed as benzoin condensation.
It involves self condensation of an aromatic aldehyde in the presence of CN - as catalyst.
u
St

O
O H O H HO CN H
C C C N C C
Proton
exchange Slow
C N + +
Intermediate

CN O H O
C C C C
+ CN–
OH H OH
benzoin
WB JEE (Engineering) l Solved Paper 2013 | 31

11. Ozone shows following resonance structure H Cl


| |
O O O 16. CH 3 —C —CH 3 ¾¾® CH 3 —C — CH 3
O O O O O O | |
CH 3 CH 3
It contains bond angle 116.8° and O—O bond (minor)
length 1.28 Å. + CH 3 —CH —CH 2 —Cl
12. The entropy is the measure of disorder or |
CH 3

)
randomness in a system. When a system
(major)

be
changes from one state to another, the change
of entropy, dS is given by There are nine primary hydrogens and one
dq tertiary hydrogen in 2-methyl propane. Tertiary
dS = rev

Tu
T hydrogen atoms react with Cl about 5.5 times as
fast as primary. Even though the primary
H Me hydrogens are less reactive, there are so many of
N

ou
them that the primary product is the major
D product.
13. + Na ¾¾® sodium extract (NaCN)
total time (t ) 22920

(Y
FeSO 4 17. n = = =4
¾¾¾® Fe 4 [Fe(CN)6 ]3 half -life (t / 2 ) 5730
FeCl 3 Prussion blue n
æ 1ö
Left amount, N = N0 ç ÷
on
COOH è2ø
Cl 4
D æ 1ö N
+ Na ¾¾® sodium extract (NaCl) = N0 ç ÷ = 0
è2ø 16
pi

AgNO 3
¾¾¾® AgCl ¯ + NaNO 3 N0
\Decayed fraction = N0 -
m

White Precipetate
16
Hydrozine, NH 2 × NH 2 does not respond 16 N0 - N0 15 N0
= =
ha

Lassaigne's test because it does not contain 16 16


any carbon and hence, NaCN is not formed.
Compound, NH 2 — C — NH 2 contains both
18. At very low pressure, compressibility factor, Z is
approximatedly one.
C

||
S \van der Waals’ gas may behave ideally.
N and S, hence, it will give red colour in
19.
dy

Lassaigne test. COOMe COOMe


hot
14. 2CaO + Cl 2 ¾¾®
Red
CaCl 2 + O 2 ­ HO H D OH
(a) (b)
u

15. From Arrhenius equation HO H D OH


St

k E é1 1ù COOMe COOMe
ln 2 = a ê - ú
k1 R ë T1 T2 û
COOMe COOH
k2 600 R æ 1 1 ö H OH H OH
or ln = ç - ÷
k1 R è 300 600 ø (c) (d)
H OH H OH
k2 æ 2 - 1ö
or ln = 600 ç ÷ COOH COOH
k1 è 600 ø
Only molecule (c) is optically active. Other
k2 k2 moelcules are meso, due to presence of plane of
or ln = 1 or =e
k1 k1 symmetry.
32 | WB JEE (Engineering) l Solved Paper 2013

20. H H H H H H 25. 2p s4 –
2p
B B or B B π*
H H H H H H (3s2) s3

Structure of B2H6
Boron electron (1π4)
Hydrogen electron
π
In diborone, odd-electron bonds are found in the (2s2) s2
bridges. From figure, it is clear that 4 electrons 2s

)
2s

be
are present for bonding in the bridges. C-atom
O-atom
21. Na 2S + Na 2 [Fe(CN)5 NO] ¾® (1s2) s1
sodium nitropruside'

Tu
CO
Na 4 [Fe(CN )5 [NOS ] (Carbon Monoxide)
+ purple colour
Na 4 [Fe(CN)5 NOS] r 4Na 26. Direct nitration of aniline with nitric acid gives a

ou
tetra anionic complex of iron (II) co-ordinating
complex mixture of mono, di- and tri-nitro
+ [Fe(CN)5 NOS]4- compounds and oxidation products. If —NH 2
to one NOS - ions. group is protected by acetylation and then
22. NaOH r Na + OH
H O r H + OH
2
+
+

-
-
[OH - ]=10 -8 M
[OH - ]=10 -7M
(Y
NH2
nitrated with nitrating mixture, p-isomer is the
main product.
NHCOCH3 NHCOCH3 NH2
on
\[OH - ]total =(10 -8 + 10 -7 )M CH3COCl HNO3 NaOH/
-7
= 10 (1.1)
pi

NO2 NO2
= 1.1 ´ 10 -7
p-nitroaniline
m

\ pOH = log 1.1 ´ 10 -7 ~


- 6.98 27. Q DTf = i ´ K f ´ m
ha

\ pH = 14 - 6.98 DTf 0.19


\ i= = = 1.02
= 7.02 K f ´ m 1.86 ´ 0.1
i - 1 1. 02 - 1
C

23. H3C H O3 Again from, a = =


n -1 2 -1
C C « CH3 Zn/H2O
H 3C C = 0.02 = 2.0 ´ 10 -2
dy

- +
H5C2 H for CH 3COOH q CH COO + H 3
optically active Ka = Ca 2
u

compound
H3C H = 0.1 ´ (2 ´ 10 -3 )2
St

C O+ O C CH3 = 4 ´ 10 -5
H3C C
28. Ore chomite is FeCr2O 4 .
acetone H5C2 H
29. H 2SO 4 saturated with SO 3 is called oleum or
24. On mixing of two different ideal gases under
isothermal reversible conditions, DSmix is always sulphan.
positive i . e ., increasing. H 2SO 4 + SO 3 ¾® H 2S 2O 7
WB JEE (Engineering) l Solved Paper 2013 | 33

30. From first law of thermodynamics 34. The order of electron withdrawing tendency from
DE = q + W benzene ring i.e.,
where, work do net (W) = p DV — F < — Cl < — NO 2
= p´V (Q DV = 0) \ Correct order of acidic strength of substituted
phenols will be
=0
OH OH OH
31. Species having unpaired electrons are
paramagnetic

)
< <

be
Ni 2 + = [Ar ] 3 d 8
F Cl NO2

Tu
(i) In [NiCl 4 ]2- and [Ni (H 2O )6 ]2+ ligands Cl - and 35. For isothermal expansion of an ideal gas
H 2O are weak ligands, therefore no pairing will be DT = 0
possible. Thus, there are two unpained electrons. \From DU = nC v DT ,

ou
(ii) In [Ni (Ph 3 )2 Cl 2 ], although PPh 3 has DU = 0 and, from
d-acceptance nature but presence of Cl, makes DH = nC p Dt = 0

(Y
electrons unpaired.
From first law of thermodynamics,
32. Ribose forms osazone with Ph—NH—NH 2
DU = Q + W
whereas in deoxyribose, one — OH group is
on
missing, due to which, it cannot form osazone. as DU = 0 Þ Q ¹ 0
CHO CH N—NH—Ph and W¹0
pi

\ Parameters are
H—C—OH C N—NH—Ph
DU = 0, Q ¹ 0 ; W ¹ 0
m

H—C—OH Ph—NH—NH2 H—C—OH


and DH = 0
ha

H—C—OH H—C—OH 36. HgCl 2 + 4 KI ¾® K 2 [Hg I4 ] + 2KCl


CH2OH CH2OH Hg = [Xe ] 4f 14 , 5 d 10 6s2
C

ribose osazone
Hg 2 + = [Xe] 4f 14 , 5 d 10 , 6s°
-7
33. 10 million = 10
5d 6s 6p
dy

\10 millionth part of 1.33 cm 3 = 1.33 ´ 10 -7 cm 3


= 1.33 ´ 10 -7 mL
HgI4 = × × × ×
u

For pure water, [H+ ] = 10 -7 mol/L


sp3-hybridisation tetrahedral
St

Þ 1 L water contains [H + ] = 10 -7 mol


37. Molecules having whole number for degree of
10 -7
or 1 mL water contains [H + ] = = 10 -10 mol unsaturation can exist in free state as stable
1000 compounds.
or 10 millionth part of 1× 33 cm 3 water contains Sn ( v - 2 )
Degree of unsaturation = +1
[H + ] = 1× 33 ´ 10 -7 ´ 10 -10 mol 2
where, n = number of atoms of a particular type
= 1× 33 ´ 10 -17 mol
v = valency of the atom.
\Number of H + ions = 1.33 ´ 10 -17 ´ NA
(a) For C 7H 9O,
= 1.33 ´ 10 -17 ´ 6.022 ´ 10 23 7 (4 - 2 ) + 9 (1 - 2 ) + 1 (2 - 2 )
DU = + 1 = 3× 5
= 8.009 ´ 10 6 = 8.01million 2
34 | WB JEE (Engineering) l Solved Paper 2013

(b) For C 8H12O, NO +2 ion is isoelectronic with CO 2 molecule. It is


DU =
8 (4 - 2 ) + 9 (1 - 2 ) + 1 (2 - 2 )
+ 1 = 3× 0 a linear ion and its central atom (N + ) undergoes
2 sp-hybridisation. Hence, its bond angle is 180°.
(c) For C 6H11O, In NO -2 ion, N-atom undergoes sp2 hybridisation.
6 (4 - 2 ) + 11 (1 - 2 ) + 1 (2 - 2 ) The angle between hybrid orbital should be 120°
DU = + 1 = 1× 5
2 but one lone pair of electrons is lying on N-atom,
(d) For C10H17O 2 , hence bond angle decreases to 115°.

)
10 (4 - 2 ) + 17 (1 - 2 ) + 2 (2 - 2 ) In NO 2 molecule, N-atom has one unpaired

be
DU = + 1 = 2×5 electron in sp2 -hybrid orbital. The bond angle
2
\ C 8H12O will exist in free state or a stable should be 120° but actually, it is 132°. It may be
due to one unpaired electron in sp2 -hybrid orbital.

Tu
compound.
38. Given, k = 1× 25 ´ 10 -3 S cm -1 Therefore, the increasing order of bond angle is
NO -2 < NO 2 < NO +2

ou
1 1
r= = S -1 cm 115° 132° 180°
k 1.25 ´ 10 -3
l 43. The structure of dipeptide gly-ala is

(Y
From R =r O
O
A
H
1 l H2N—CH2—C —N—CH—C—OH
800 = ´
1.25 ´ 10 -3 A (Gly)
on
CH3
l (Ala)
(where, = cell constant)
A
pi

l 44. l¥m = l¥m (oxalate) + l¥m (Na + ) + l¥m (K + )


\ = 800 ´ 1.25 ´ 10 -3
A = (148 × 2 + 73 × 5 + 50 × 1)
m

= 1 cm -1 = 271.85 cm 2 mol -1
ha

39. Orange solid is ammonium dichromate. l¥m 271.8


Q l¥eq = =
D
(NH 4 )2 Cr2O 7 ¾¾ ® N2 ­ + Cr2O 3 + 4 H 2O n × factor 2
colourless = 135.9 Scm 2 eq -1
C

orange solid green solid


gas

Cr2O 3 + 2 Al ¾¾® 2Cr + Al 2O 3 45. Since, ionic character is inversely proportional to


polarising power of cation therefore correct order is
dy

40. 2, 2 - dimethyl butane is prepared from Corey - AlCl 3 > GaCl 3 > BCl 3
House alkane synthesis.
46. OH
u

SN2
(Me 3C )2 CuLi + MeCH 2Br ¾ ¾¾®
St

O—B—O
Br
Me 3C—CH 2CH 3 + CuLi HO—B O B—OH
Me 3C
O—B—O
41. From Gibbs Helmholtz equation,
OH
DG = DH - TDS
5 B — O — B bonds
A process will be spontaneous, if
4 B — OH bonds
DG = – ve
47. H COCMe3 CMe3
42. No option is correct.
As the number of lone pair of electrons + ClCOCMe3Anhy . AlCl3
– HCl – CO
increases, bond angle decreases.
WB JEE (Engineering) l Solved Paper 2013 | 35

48. CH 3COO - + H + ¾® CH 3COOH subshell and the shape of the orbital occupied
Initially 0 × 01 0 × 001 0 × 01 by the electron.
at equi. 0 × 01 - 0 × 001 0 × 01 + 0 × 001 m describes the preferred orientation of orbitals
= 0 × 009 = 0 × 011 in space.
(salt) 0.009 s describes the spining of an electron on its axis.
pH = pK a + log = 4.75 + log
(acid) 0.011
56. Estimation of Ni 2+ is carried out as.
= 4.66 D
Filtrate of group III + NH 4OH + NH 4Cl ¾¾ ®

)
D
49. 3HClO 3 ¾¾® HClO 4 + Cl 2 + 2O 2 + H 2O

be
passing H 2S gas ® black ppt of NiS.
Me 3C This black ppt of NiS is soluble in conc. HCl in
50. Mg + Cl — CN ¾® presence of oxidising agent like KClO 3

Tu
Br Cl NiS + 2HCl + O ¾® NiCl 2 + H 2 . O+S
Me 3C—CN + Mg Conc
Br Now this NiCl 2 , in basic medium, treated

ou
51. O CH3 O CH2Br with dimethyl glyoxime, cherry red ppt of nickel (II)
C C dimethyl glyoximate is obtained, in which two
Bromination dimethyl glyoximate units are hydrogen bonded
CH3COOH

52. DG° = - 2.303 RT log K (Y to each other and each unit forms a
six-membered chelate ring with Ni 2+ .
NiCl2 H3C—C NOH
on
= - 2.303 ´ 8.314 ´ 298 ´ log 0.15 in +2
NH4OH H3C—C NOH
= - 2.303 ´ 8.314 ´ 298 ´ (- 0.82 ) dimethyl glyoxime
pi

= 4678.7 J = 4.67 kJ
O H—O
53. Silicon oil in a polymer of trimethylchloro
m

silane and dimethyldichloro silane. These H3C—C N N C—CH3


Ni
are useful as broad spectrum antifoaming
ha

H3C—C N N C—CH3
agents.
Polymerisation O—H O
Si (Mel 3 ) Cl + Si (Me )2 Cl 2 + H 2O – ¾¾¾¾¾®
C

nickel (II) dimethyl glyoximate


Me Me Me Me cherry red ppt
CH 3
dy

—Si—O—Si—O—Si—O—Si— |
57. CH 3CH 2CH 2CH 2OH CH 3 —C —CH 3
Me Me Me Me |
n-butanol
u

54. F OH
D D
t - butanol
St

LiN. NH3
+ NaNH2
More branching results high solubility and low
NH2 H
D H D boiling point.
NH2
+ 58. Availability of acidic a - H- atoms at (*) marked
positions, enable the compounds to show keto-
This reaction proceeds via benzyne mechanism
enol tautomerism.
D
with intermediate as . O
MeCCO O
55. All the quantum numbers (n, l , m, s) are required to MeCCO— CH*
describe an electron of an atom completely. MeCCO O O
e.g., n describes the position and energy of the (a) (b) (d)
electron in an orbit or shell. l is used to describe
36 | WB JEE (Engineering) l Solved Paper 2013

59. Lintz and Donawitz (L.D.) process is an iron quickly remaining pure iron is mixed with
important process for manufacturing of steel. In spiegeleisen (alloy Fe, Mn, C) to produce better
India, Rourkela steel plant is based upon this quality steel.
process. 60. 2NO 2 (g ) + F2 (g ) ¾® 2 NO 2F(g )
This process is carried out in a converter similar
to bessemer converter having lining (usually 1 é dp (NO 2 )ù
Rate of reaction = -
refractory made of MgO and (CaO ). 2 êë dt úû

In this method scrap iron can be used along with é dp (F2 )ù

)
=-ê
ë dt ûú

be
flux in converter : On passing oxygen gas at
4 - 12 atm pressure into converter, temperature
1 é dp (NO 2F)ù
of 2000 - 2500° C range produced which =+
2 êë dt úû

Tu
separates C, Si, Mn etc., impurities as slag from

ou
Mathematics
1. The given equation 3. Let E ® Event of head showing up
2
ax + bx + 1 = 0
has real roots.
…(i)
(Y E1 ® Event of biased coin chosen
E2 ® Event of unbiased coin chosen
on
1 1
\Discriminant (D ) ³ 0 Now, P(E2 ) = and P(E1 ) =
2 2
Þ b2 - 4a ³ 0 …(ii)
pi

æ Eö 1 æEö 3
Also, P ç ÷ = and P ç ÷ =
From Eq. (ii), we observe that è E2 ø 2 è E1 ø 4
m

a has to be 1 and b has to be 2. (by conditional probability)


1 1 1
So, the required probability = ´ = By Baye’s theorem
ha

2 2 4
æ Eö
2. Ist turn Total number of face card = 12 P(E2 )× P ç ÷
æE ö è E2 ø
Pç 2 ÷ =
C

Total number of elements in sample space, è Eø æ Eö æEö


P(E2 )× P ç ÷ + P(E1 )× P ç ÷
n(s) = 52 è E2 ø è E1 ø
dy

52 - 12 40 1 1
\P1 (no face card in first turn) = = ´
52 52 2 2 2
= =
u

IInd turn P2 (no face card in second turn) 1 1 1 3 5


´ + ´
(52 - 13) 39 2 2 2 4
St

= =
(52 - 1) 51 4. Given equation of ellipse
IIIrd turn P3 (face card in third turn) x 2 + 9y 2 = 9

=
(52 - 40 ) 12
= x2 y2
Þ + =1 …(i)
(51 - 1) 50 9 1
\Required P (face card on third turn) and equation of lines
= P1 ´ P2 ´ P3 x + y = 1and 3 y = x + 3
40 39 12 12 x y
= ´ ´ = or + =1
52 51 50 85 (-3) 1
WB JEE (Engineering) l Solved Paper 2013 | 37
y
Þ 9 (4 - x 2 ) = 4 y 2

3y = x + 3
Þ 36 - 9 x 2 = 4 y 2
(0, 1)P
(–3, 0)
Q Þ 9 x 2 + 4 y 2 = 36
x′ x
O A x2 y2
R Þ + = 1 which represents an ellipse.
x+y = 1 4 9
9, – 4
y
5 5 6.
y′

)
be
0 1 1 B (0, 1)
1
\Area of DPQR = -3 0 1 (h, k)
2 9 / 5 -4 / 5 1 x′ x
M O A (2, 0)

Tu
1 é 24 12 ù P (x, y)
= +
2 êë 5 5 úû

ou
y′
1 36 18
= ´ = Given equation of an ellipse is
2 5 5
x 2 + 4y 2 = 4
5. Given equation of lines are

and
3t x - 2 y + 6t = 0
3 x + 2 ty - 6 = 0
…(i)
…(ii)
(Y Þ
x2
4
+
y2
1
=1 …(i)
on
\ Coordinate of positive end of minor axis is
On multiplying Eq. (i) by t and then adding in
B(0, 1.) Let mid-point of the chord BP is M(h, k )
Eq. (ii), we get
æ 0 + x 1+ yö
pi

(3 t 2 + 3)x + 6 t 2 - 6 = 0 Then, (h, k ) = ç


è 2
, ÷
2 ø
2(1 - t 2 )
m

Þ x= x
(1 + t 2 ) Þ h= Þ x = 2h
2
ha

Þ x + xt 2 = 2 - 2t 2 and k=
1+ y
Þ y = 2k - 1
2
Þ ( x + 2 ) t 2 = (2 - x )
\ P( x, y ) º {2 h, (2 k - 1}
)
C

2-x
Þ t2 = …(iii) Since, the point ‘P’ lies cllipes so form Eq. (i), we
2+ x
get
dy

On multiplying Eq. (ii) by t and then subtract (2 h )12 + 4(2 k - 1)2 = 4


from Eq. (i), we get
Þ 4h 2 + 4(2 k - 1)2 = 4
(-2 - 2 t 2 )y + 6 t + 6 t = 0
u

Þ h 2 + 4k 2 + 1 - 4k = 1
12 t = 2(1 + t 2 )y
St

Þ h 2 + 4k 2 - 4k = 0
On squaring both sides, we get
144 t 2 = 4 y 2 (1 + t 2 )2 Thus, required locus is an ellipse whose
2
equation is
æ2 - x ö 2 æ 2- xö
Þ 144 ç ÷ = 4 y ç1 + ÷ [form Eq. (iii)] x 2 + 4y 2 - 4y = 0
è2 + x ø è 2 + xø
2
2 æ 1ö
æ2 - x ö 2 æ 4 ö 2 çy - ÷
Þ 36ç ÷=y ç ÷ ( x - 0) è 2ø
è2 + x ø è2 + x ø Þ + =1
1 æ 1ö
ç ÷
(2 - x ) 16 y 2 è 4ø
Þ 36 =
(2 + x ) (2 + x )2 æ 1ö 1
whose centre ç 0, ÷ and major and minor axis
Þ 2
36 (4 - x ) = 16 y 2 è 2ø 2
38 | WB JEE (Engineering) l Solved Paper 2013

7. Given equation of circle is 9. Let the equation of circle is


x + y = 169
2 2
x 2 + y 2 + 2 gx + 2 fy + c = 0 …(i)
Its centre = ( 0, 0 ) and radius = 13 When, circle (i) passes through the origin
y Then, c =0 …(ii)
B (0, 13) When, circle (i) passes through the point (2, 6).
R(–12, 5) Then, 4 + 36 + 4g + 12 f + 0 = 0
Q(5, 12)

)
x' A ⇒ 4g + 12 f + 40 = 0

be
x
O (13, 0)
⇒ g + 3 f = − 10 …(iii)
When, circle (i) passes through the point (6, 2)
P

Tu
y' Then, 36 + 4 + 12 g + 4 f + 0 = 0 [from Eq. (i)]
⇒ 12 g + 4 f + 40 = 0
−5  y − y ⇒ 3 g + f + 10 = 0

ou
…(iv)
Now, slope of OR = = m1 Q slope = 2 
12  x2 − x  On solving Eqs. (iii) and (iv), we get
12 −5 −5
g= and f =

(Y
and slope of OQ = = m2
5 2 2
π ∴Equation of circle becomes
Q m1 ⋅ m2 = − 1 ⇒ ∠ROQ =
2 x 2 + y 2 − 5x − 5y = 0
on
…(v)
We know that, angle made by the chord of circle Circle cut the x-axis.
at circumference is equal to the half of the angle
So, put y = 0 in Eq. (v), we get
pi

made by the same chord at the centre of circle.


1 1 π π x 2 − 5 x = 0 ⇒ x( x − 5) = 0
∴ ∠QPR = ⋅ ∠ROQ = × =
m

2 2 2 4 ⇒ x=5
8. Let P be any point, whose coordinate is (h, k ). So, the circle cut the x-axis at point P(5, 0 ).
ha

Given, ∴The length of OP = 5


P moves, so that the sum of squares of its 10. Given equation of lines are
C

distances from the points A(1, 2 ) and B(− 2, 1) is 6.


x − 2y = t …(i)
i . e ., (PA)2 + (PB)2 = 6 1
and x + 2y = …(ii)
dy

⇒ (h − 1)2 + (k − 2 )2 + (h + 2 )2 + (k − 1)2 = 6 t
⇒ h 2 + 1 − 2 h + k 2 + 4 − 4k + h 2 + 4 + 4h On multiplying Eqs. (i) and (ii), we get
u

1
+ k 2 + 1 − 2k = 6 ( x − 2 y) ( x + 2 y) = t ×
St

t
⇒ 2 h 2 + 2 k 2 + 2 h − 6k + 4 = 0 ⇒ x 2 − 4y 2 = 1
⇒ h + k + h − 3k + 2 = 0
2 2
( x − 0 )2 ( y − 0 )2
⇒ − =1
∴Required locus is 1 (1 / 4)
x 2 + y 2 + x − 3y + 2 = 0 which represent a hyperbola.
Which represent a circle. 1
Here, a2 = 1 and b2 =
 −1 3 4
Whose centre is  , 
 2 2 1
1+
a2 + b2 4 = 5
1 9 5 1 ∴Eccentricity (e ) = =
and radius = + −2 = −2 = a2 1 2
4 4 2 2
WB JEE (Engineering) l Solved Paper 2013 | 39

and focus 14. When eleven apples are distributed among a girl
 5   5  and a boy, then the girl receives atleast 14
= (± ae , 0 ) =  ± 1 × , 0 =  ± , 0 apples or the boys receives atleast 8 apples.
 2   2 
(by hypothesis)
and centre = (0, 0 ) 15. Given, z1 = 2 + 3 i and z2 = 3 + 4i
11. Let A = {1, 2, ..., 11}
Now, we have
∴ n( A) = 11 and B = {1, 2, ..., 10} | z − z1|2 + | z − z2|2 = | z1 − z2|2 (let z = x + iy )

)
∴ n(B) = 10

be
⇒ |( x + iy ) − (2 + 3i )|2 + |( x + iy ) − (3 + 4 i )|2
∴Hence number of onto function = |(2 + 3 i ) − (3 + 4 i )|2
= n( A )
Cn( B ) × n(B)! × n(B) ⇒ |( x − 2 ) + i ( y − 3)|2 + |( x − 3) + i ( y − 4)|2

Tu
= C10 × 10 ! × 10
11
= |−1 − i|2
= (11 × 10 !) × 10 = 11! × 10 ⇒ ( x − 2 ) + ( y − 3)2 + ( x − 3)2 + ( y − 4)2 = 1 + 1
2

ou
12. Let p( x ) = ax 2 + bx + c …(i) ⇒ x 2 + 4 − 4x + y 2 + 9 − 6y
Given, constant term ‘c’ = 1 + x 2 + 9 − 6 x + y 2 + 16 − 8 y = 2
∴ p( x ) = ax 2 + bx + 1
Now, by given condition, p(1) = 2
…(ii)
(remainder) (Y ⇒

2 x 2 + 2 y 2 − 10 x − 14 y + 36 = 0
x 2 + y 2 − 5 x − 7 y + 18 = 0
on
⇒ a + b + 1= 2  5 7
which represent a circle with centre   and
⇒ a+ b=1 …(iii)  2, 2 
p(−1) = 4 25 49 1
pi

and (remainder) radius + − 18 =


4 4 2
⇒ a − b + 1= 4
m

⇒ a−b=3 …(iv) 16. Let a − 2d , a − d , a, a + d , a + 2d are in AP,


On adding Eqs. (iii) and (iv), we get 1 1 1 1 1
ha

then , , , , are in HP.


a − 2d a − d a a, + d a + 2d
2 a = 4 ⇒ a = 2 form eys (iii) b = − 1
On putting the values of a and b in Eq. (ii), we get Given, middle term = 1
C

1
p( x ) = 2 x − x + 1 = 0
2 ⇒ = 1⇒ a = 1
a
(Coefficient of x ) − (−1) Second term 2
∴Sum of the roots = − =
dy

and =
(Coefficient of x 2 ) 2 Fourth term 1
1 1 2
= ⇒ ×a+d =
u

2 a−d 1
St

1 1
(2013)x 1  ⇒ a = 3d ⇒ d = (Qa = 1)
13. lim  + x − x  3
x→ 0
x
2
e − 1 e − 1
∴Sum of first three terms
1 (2013)x − 1 1 1 3 11
= lim  2 +  = + + 1= 3 + + 1=
x→ 0 x
 ex −1  1−
2
1−
1 2 2
3 3
1 (2013)x − 1 x 
= lim  2 + ⋅ x   π π  1 1
x→ 0 x
 x e − 1 cos − sin   − 
17. Given, P =  4 4 = 2 2
1 (2013)x − 1 x π π  1 1 
= lim + lim ⋅ lim x  sin cos 
x→ 0 x 2 x→ 0 x x→ 0 e − 1  4 4  2 2 
= + ∞ + log(2013)⋅ 1 1 1 −1
⇒ P=  
2 1 1 
=+∞
40 | WB JEE (Engineering) l Solved Paper 2013

1 1 −1 1 −1 1  x 2013 1 


Now, P 2 = P ⋅ P =     20. Let I = ∫  | x| 2 + | x|  dx
2 1 1  1 1  −1 e ( x + cos x )
 e 
1 1 − 1 −1 − 1
=   1 x 2013 1 1
2 1 + 1 −1 + 1 ⇒ I=∫ dx + ∫−1 e| x| dx
−1 e| x|( x 2 + cos x )
1  0 −2  0 −1
=   = 
2 2 0   1 0  Here,
x 2013
is an odd function
1 1 −1  0 −1 e ( x 2 + cos x )
| x|

)
P3 = P ⋅ P2 =   ⋅ 
2 1 1   1 0 

be
1
and is an even function.
1  0 − 1 − 1 − 0 e| x|
=  
2  0 + 1 −1 + 0  2 af ( x ) dx; f ( x ) is even 
 ∫0

Tu
a
1  −1 −1  ∫− a
Q f ( x ) dx = 
=     0, f ( x ) is odd 
2  1 −1
1

ou
1 / 2 
1 1 ∴ I = 0 + 2 ∫ e − xdx = − 2(e − x )10 = −2(e −11)
Also, given X =  1  = 
0

 2 
 2 1 = 2(1 − e −1 )

∴ P3 X =
1  −1 −1 1 1
 ⋅
2  1 −1 2 1


1  −1 −1 1  −2  −1


(Y
21. Given, 0 ≤ P, Q ≤
π
2
sin P + cos Q = 2
on
and …(i)
=   =   = 
2  1 −1 2  0   0  This equation hold only
18. Given equation is x 2 − x + 1 = 0 π
pi

when, P=
  2
1± 1− 4
Q x = b ± b − 4ac 
2
x=
m

  and Q=0
2  2a 
π
LHS = sin P + cos Q = sin + cos 0
ha

1± i 3 1+ i 3 1− i 3 2
= = ,
2 2 2 = 1 + 1 = 2 = RHS
−1 + i 3 −1 − i 3
C

⇒ −x= , π 
2 2  P + Q  + 0 π
∴ tan   = tan  2  = tan = 1
⇒ + x = ω, − ω 2  2   2  4
dy

 
Since, (α , β ) are the roots of given equation.
Then, α = − ω and β = − ω 2 22. Given, f ( x ) = 2100 ⋅ x + 1
u

∴ α 2013 + β 2013 = − (ω )2013 + (− ω 2 )2013 and g( x ) = 3100 ⋅ x + 1


St

= − ω 2013 − ω 4026 Now, f { g( x )} = x


= − (ω )
3 671
− (ω )
3 1342 ⇒ f (3100 . x + 1) = x

= − (1)671 − (1)1342 (Q ω 3 = 1) ⇒ 2100 { 3100 ⋅ x + 1} + 1 = x


= −1− 1= −2 ⇒ 6100 ⋅ x + 2100 + 1 = x
19. Given equation, is x + y + z = 10 ⇒ x(1 − 6100 ) = (1 + 2100 )
where, x, y and z are positive integers. 1 + 2100
( 10 − 1) ⇒ x=
∴Required number of solutions = C( 3 − 1) 1 − 6100
9× 8 Hence, fog( x ) = x represent a singleton set.
= 9C2 = = 36
2
WB JEE (Engineering) l Solved Paper 2013 | 41

 1+ x 1  −3 3 3
23. lim  − 1+ 2  ⇒ ≤ cos 4 θ ≤
x→ 0 8 8 8
 x x 
5 3 5 3 5 3
 1 + x − 1 + x 2  0 ⇒ − ≤ + cos 4θ ≤ +
  8 8 8 8 8 8
= lim    form
x→ 0 x 0 1
  ⇒ ≤ f(θ ) ≤ 1 [from Eq. (i)]
4
1 x
− So, the maximum value is 1and minimum value is
2 1+ x 1 + x2

)
= lim (Use L-Hospital rule) 1
.

be
x→ 0 1 4
1 1 26. Given, z = x + iy
= −0=
2 1+ 0 2

Tu
z − 1 ( x + iy ) − 1
Now, =
24. Given z − i ( x + iy ) − i
cos 2 75° + cos 2 45° + cos 2 15° − cos 2 30 ° ( x − 1) + iy x − i ( y − 1)

ou
= ×
− cos 60 °
2 x + i ( y − 1) x − i ( y − 1)
x( x − 1) + ixy − i ( x − 1) ( y − 1) + y( y − 1)
 3 1 
2
 3 1 
2 2
=

(Y
 1 x 2 + ( y − 1)2
= −  +  + + 
2 2 2 2  2  2 2 2 2
( x 2 + y 2 − x − y ) + i ( xy − xy + y + x − 1)
 3
2
 1
2 =
( x 2 + y 2 + 1−2 y )
on
−  − 
 2   2
 x2 + y2 − x − y   x + y −1 
 3 + 1 − 2 3 1  3 + 1 + 2 3 3 1 = 2  +i 2  ...(i )
 x + y − 2 y + 1
2
 x + y − 2 y + 1
2
pi

=  + +  − −
 8  2  8  4 4
z −1
Given, is real.
m

1 3 1 1 3 3 1 3 1 z−i
= − + + + − − = − 1=
2 4 2 2 4 4 4 2 2 So, its imaginary part should be zero.
ha

25. Let f(θ ) = sin 6 θ + cos 6 θ x + y −1


i . e ., =0
⇒ f(θ ) = (sin 2 θ )3 + (cos 2 θ )3 x2 + y2 − 2 y + 1
C

= (sin 2 θ + cos 2 θ ) ⇒ x + y =1
(Q x 2 + y 2 − 2 y + 1 ≠ 0 )
(sin 4 θ + cos 4 θ − sin 2 θ ⋅ cos 2 θ )
dy

which represent a straight line.


[Q a3 + b3 = (a + b) (a2 + b2 − ab)]
27. Given that, a, b and c are in AP
u

= 1⋅ {(sin 2 θ + cos 2 θ )2 − 3 sin 2 θ ⋅ cos 2 θ}


∴ 2b = a + c
St

 3 
= 1⋅ 1 − ⋅ 4 sin 2 θ ⋅ cos 2 θ ⇒ c = 2b − a
 4  …(i)
3 and equation of straight line is
= 1− (sin 2θ)2 (Qsin 2 A = 2 sin A cos A)
4 ax + 2 by + c = 0
3 ⇒ ax + 2 by + (2 b − a) = 0
= 1 − (1 − cos 4 θ )
8 ⇒ a( x − 1) + b(2 y + 2 ) = a ⋅ 0 + b ⋅ 0
3 3
= 1 − + cos 4 θ On comparing, we get
8 8 x − 1= 0 ⇒ x = 1
5 3
f(θ ) = + ⋅ cos 4 θ …(i) and 2y + 2 = 0
8 8
⇒ y = −1
Q −1 ≤ cos 4 θ ≤ 1
Hence, the required fixed point is (1, − 1).
42 | WB JEE (Engineering) l Solved Paper 2013

28. Given equation is For intersection point of both curve put y = x, we


2 x + 5 xy − 12 y = 0
2 2
…(i) get
x2 x2
⇒ 2 x 2 + 8 xy − 3 xy − 12 y 2 = 0 − =1
9 25
⇒ 2 x( x + 4 y ) − 3 y( x + 4 y ) = 0 9 × 25  15
2
⇒ x2 = = 
⇒ ( x + 4 y ) (2 x − 3 y ) = 0 16  4
∴ x + 4 y = 0 and 2 x − 3 y = 0 15 15
⇒ x=± and y = ±

)
which represent a pair of straight lines. 4 4

be
Compair Eq. (i) with ax 2 + 2 hxy + by 2 = 0  15 15
∴Intersetion points P  , 
 4 4
we get a = 2 and b = − 12

Tu
Q a+ b≠0  −15 −15
and Q , 
 4 4 
So, lines are not perpendicular to each other.
Since, PQ is major axis, then its length

ou
Hence, it is a pair of non-perpendicular
intersecting straight lines. 15 15
=2 2⋅ =
29. Given equaitson of circle is 4 2


3x 2 + 3y 2 − 9x + 6y + 5 = 0

x 2 + y 2 − 3x + 2 y +
5
=0
(Y and length of minor axis is

i . e ., Major axis, 2 a =
15
⇒a=
5
2
(given)

15
on
3
2 2 2
3 
Centre =  , − 1 5 5
2  and minor axis, 2 b = ⇒ b=
pi

2 2 2
9 5
and radius = + 1− ∴Eccentricity of an ellipse
m

4 3 2
a2 − b2  b
19 1 19 = = 1−  
= = a2  a
ha

12 2 3
2
We know that, centre of the circle is the mid-point  1 8 2 2
= 1−   = =
of the diameter.  3 9 9
C

Lives one and of point of dianetev in (1, 2) Let the 31. lim x sin e (1/ x )
other end point of diameter is (h, k ). x→ 0
dy

3  1 + h 2 + k LHL = f (0 − 0 ) = lim (− h )sin e ( −1/ h )


Then,  , − 1 =  ,  h→ 0
2   2 2 
= − 0 × sin(e −∞ ) = − 0 × sin(0 ) = 0
u

1+ h 3
⇒ =
St

2 2 = 0 × (a finite number between −1to +1)


⇒ 1+ h = 3 =0 (Q − 1 ≤ sin x ≤ 1)
⇒ h =2 Q LHL = RHL
2+k ∴ lim x sin(e 1/ x ) exist and equal to 0
and = −1 x→ 0
2
 1 1 1 1 
⇒ 2 + k = −2 32. 1000  × + +K+
1 × 2 2 × 3 3 × 4 999 × 1000 
⇒ k = −4
So, the other end point is (2, − 4).  1  1 1  1 1
= 1000 1 −  +  −  +  − 
  2   2 3  3 4
30. Given equation of hyperbola and line are
x2 y2  1 1 
− = 1 and y = x respectively. +K+  − 
 999 1000  
9 25
WB JEE (Engineering) l Solved Paper 2013 | 43
 1  35. Given equation is, x 2 + ax + b = 0, (b ≠ 0 )
= 1000 1 − 
 1000 
its roots are α and β.
999
= 1000 × Then, sum of roots = α + β = − a …(i)
1000
= 999 Product of roots = α ⋅ β = b …(ii)
 1 0 0 Now,
33. Given, I =  0 1 0  1  1  α + β
  α −  + β −  = (α + β ) −  
 0 0 1

)
 β α  αβ 

be
1 0 0  (− a)
and P =  0 −1 0  = −a− [from Eqs.(i) and (ii)]
  b
 0 0 −2

Tu
a a
The characteristic equation of P is = − a + = (1 − b) …(iii)
b b
| P − λI| = 0  1  1 1
α −  β −  = αβ − 1 − 1 +

ou
1− λ 0 0 and
 β α αβ
⇒ 0 −1 − λ 0 =0
−2 − λ 1
0 0 =b+ − 2 [from Eq. (ii)] …(iv)

(Y
b
⇒ (1 − λ ) {(1 + λ ) (2 + λ )} = 0
1 1
⇒ (1 − λ2 ) (2 + λ ) = 0 = (b2 − 2 b + 1) = (b − 1)2
b b
on
⇒ 2 − 2 λ2 + λ − λ3 = 0 ∴ Required of quadratic equation whose roots
⇒ λ3 + 2 λ2 − λ − 2 = 0  1  1
are α −  andβ −  is
 β  α
pi

We know that, Caylay Hamilton theorem states


that ‘Every square matrix satisfy its characteristic  1  1 
x 2 −  α −  +  β −   x
m

equation’.
  β   α 
∴ P 3 + 2P 2 − P − 2I = 0
 1  1 
ha

⇒ P 3 + 2P 2 = P + 2I +  α −   β −   = 0
  β   α 
34. Let On putting the values from Eqs. (i) and (ii), we get
C

1 + a2 − b2 2 ab −2 b a 1
x 2 − (1 − b) x + (b − 1)2 = 0
∆= 2 ab 1 − a2 + b2 2a b b
dy

2b −2 a 1 − a2 − b2 ⇒ bx 2 + a(b − 1)x + (b − 1)2 = 0 , b ≠ 0


Apply C1 → C1 − bC3 and C2 → aC3 + C2 36. Given,
u

In ellipse the distance between the foci


1 + a2 − b2 + 2 b2 2 ab − 2 ab −2 b
St

∆= 2 ab − 2 ab 1 − a2 + b2 + 2 a2 2a = Length of the latusrectum


2 b − b + a2 b + b3 −2 a + a −`a3 − ab2 1 − a2 − b2 2 b2
⇒ 2 ae =
a
(1 + a + b )
2 2
0 −2 b b2
= 0 (1 + a2 + b2 ) 2a ⇒ a2e = b2 ⇒ e = 2 …(i)
a
b(1 + a2 + b2 ) − a(1 + a2 + b2 ) (1 − a2 − b2 ) b2
Q e2 = 1− 2
1 0 −2 b a
= (1 + a2 + b2 )2 0 1 2a ⇒ e2 = 1− e [from Eq. (i)]
b − a (1 − a2 − b2 )
⇒ e2 + e − 1= 0
= (1 + a + b ) {(1 − a − b + 2 a ) + 2 b }
2 2 2 2 2 2 2
−1 ± 1 + 4
= (1 + a2 + b2 )2 (1 + a2 + b2 ) = (1 + a2 + b2 )3 ⇒ e=
2
44 | WB JEE (Engineering) l Solved Paper 2013

(by quadratic formula) ⇒ a2 r 2 = 1


−1 ± 5 1
⇒ e= ⇒ r2 = …(ii) [from Eq. (i)]
2 10
5 −1 ∴ Last term = ar 5 = [a2 (r 2 )5 ]1/ 2
⇒ e= (Q 1 > e > 0 )
2 1/ 2
 1 
= 10 × 5 [from Eqs. (i) and (ii)]
37. Let S1 ≡ x + y − 6 x − 8 = 0
2 2
 10 

)
and S2 ≡ x2 + y2 − 6 = 0  1 
1/ 2
1

be
=  4 =
Now, the equation of the circle passing through  10  100
the point (1, 1) and the point of intersection of S1
40. Given that, (α, β) are roots of the quadratic

Tu
and S 2 is
equation
S1 + λS 2 = 0
ax 2 + bx + c = 0
⇒ ( x 2 + y 2 − 6 x − 8) + λ ( x 2 + y 2 − 6) = 0

ou
and 3 b2 = 16 ac …(i)
⇒ (1 + λ )x 2 + (1 + λ )y 2 − 6 x
−b c
+ (−8 − 6λ ) = 0 …(i) ∴ α+β= and αβ = …(ii)

(Y
a a
Since, Eq. (i) passes through the point (1, 1.)
From Eq. (i), we get
Q Put x = 1, y = 1 in Eq. (ii), we get
3b ⋅ b = 16 ⋅ a ⋅ c
on
(1 + λ ) + (1 + λ ) − 6 + (− 8 − 6λ ) = 0 2
 b c
⇒ − 4λ − 12 = 0 ⇒ 3   = 16   (divide by a2 )
 a  a
⇒ λ = −3
pi

⇒ 3 (α + β )2 = 16 αβ [from Eq. (ii)]


On putting the value of ‘λ’ in Eq. (i), we get
⇒ 3 α 2 + 3 β 2 + 6 αβ = 16 αβ
m

− 2 x 2 − 2 y 2 − 6 x + 10 = 0
⇒ 3 α 2 + 3 β 2 = 10 αβ
⇒ x 2 + y 2 + 3x − 5 = 0
ha

α  β
38. The distance from the point (−1, 2 ) of the line ⇒ 3   + 3   = 10
 β α
which passes through (2, − 3) α
C

Let =x
= (2 + 1)2 + (− 3 − 2 )2 β
= 9 + 25 = 34 < 8 3
dy

Then, 3x += 10
x
Given that, the distance between the point (− 1, 2 )
⇒ 3 x 2 − 10 x + 3 = 0
and the line is 8.
u

⇒ 3x 2 − 9x − x + 3 = 0
But the maximum distance of the line passing
St

through (2, − 3) from (− 1, 2 ) is 34. So, there is no ⇒ 3 x( x − 3) − 1( x − 3) = 0


such line possible. ⇒ ( x − 3) (3 x − 1) = 0
1 α 1
39. Let the six terms of GP are ⇒ x = ,3 ⇒ = ,3
a a a 3 β 3
, , , ar , ar 3 , ar 5
r5 r3 r ∴ β = 3 α or α = 3 β
Now, according to the question 41. Let the relation defined as
a a a R = {( A, B)| B = P −1 AP} …(i)
⋅ ⋅ ⋅ ar ⋅ ar 3 ⋅ ar 5 = 1000
r5 r3 r For reflexive, −1
A = I AI
⇒ a6 = (10 )3 ⇒ a2 = 10 …(i)
⇒ ( A, A) ∈ R
Also, given fourth term = ar = 1 ⇒ R is reflexive
WB JEE (Engineering) l Solved Paper 2013 | 45

For symmetric 43. Given curve is, x 2 + 4 xy + 8 y 2 = 64 …(i)


Let ( A, B) ∈ R On differentiating w.r.t x, we get
Q B = P −1 AP  dy  dy
2 x + 4  y + x  + 16 y =0
⇒ PB = AP ⇒ PBP −1 = A  dx  dx
⇒ A = (P −1 )−1 B(P −1 ) ⇒ 2 x + 4 y + (4 x + 16 y )
dy
=0
dx
⇒ (B, A) ∈ R ⇒ R is symmetric.

)
dy ( x + 2 y)
⇒ =−

be
For transitive
dx 2( x + 4 y )
Let ( A, B) ∈ R, (B, C ) ∈ R
Since, tangent are parallel to x-axis only.
Q A = P −1BP and B = Q −1CQ

Tu
dy
⇒ A = P −1Q −1CQP = (QP )−1C(QP ) i . e ., =0
dx
⇒ ( A, C ) ∈ R ( x + 2 y)

ou
⇒ − = 0 ⇒ x + 2y = 0
⇒ R is transitive. 2( x + 4 y )
Since, R is reflexive, symmetric and transitive. …(ii)
So, R is an equivalence relation.
42. Let the given relation defined as
R = {(a, b)|sin 2 a + cos 2 b = 1}
(Y Now, on putting the valus of x from Eqs. (i) in
(ii), we get
4 y 2 − 8 y 2 + 8 y 2 = 64
on
For reflexive, sin 2 a + cos 2 a = 1 ⇒ y 2 = 16
(Q sin 2 θ + cos 2 θ = 1, ∀ θ ∈ R )
pi

⇒ y=±4
⇒ a Ra ⇒ (a, a) ∈ R From Eq. (ii)
m

⇒ R is reflexive. When y = 4, x = − 8
For symmetric, sin 2 a + cos 2 b = 1 y = − 4, x = 8
ha

and when
⇒ 1 − cos 2 a + 1 − sin 2 b = 1 Hence required points are (− 8, 4) and (8, − 4)
⇒ sin 2 b + cos 2 a = 1 44. Given,
C

⇒ bRa  x 3 − 3 x + 2, x <2
f( x) =  3
 x − 6 x + 9 x + 2, x ≥ 2
2
dy

Hence, R is symmetric.
For transitive LHL
= f (2 − 0 ) = lim (2 − h )3 − 3 (2 − h ) + 2
u

Let aRb , bRc h→ 0


St

⇒ sin a + cos b = 1
2 2
…(i) = (2 )3 − 6 + 2 = 8 − 6 + 2 = 4
and sin 2 b + cos 2 c = 1 …(ii) RHL = f (2 + 0 ) = lim (2 + h )3 − 6 (2 + h )2
h→ 0
On adding Eqs. (i) and (ii), we get + 9 (2 + h ) + 2
sin 2 a + (sin 2 b + cos 2 b) + cos 2 c = 2 = (2 )3 − 6 (2 )2 + 9 (2 ) + 2
⇒ sin 2 a + cos 2 c + 1 = 2 = 8 − 24 + 18 + 2 = 4
⇒ sin a + cos c = 1
2 2 Q LHL = RHL
∴ lim f ( x ) exist
Hence, R is transitive also. x→ 2

Therefore, relation R is an equivalence relation. and f(2 ) = (2 )3 − 6 (2 )2 + 9 (2 ) + 2


= 8 − 24 + 18 + 2 = 4
46 | WB JEE (Engineering) l Solved Paper 2013

∴ LHL = RHL = f(2 ) (− x )1000 − 1 x1001 − x


= lim (− x )⋅   = lim
So, f ( x ) is continuous at x = 2 x→ ∞
 (− x − 1)  x → ∞ x + 1
 3 x 2 − 3, x <2 x1000 − 1
Now, f′ ( x) =  2 = lim =+∞
 3 x − 12 x + 9, x ≥2 x→ ∞  1
1+  
∴ Lf′ (2 ) = 3 (2 ) − 3 = 12 − 3 = 9
2  x

and Rf′ (2 ) = 3(2 )2 − 12(2 ) + 9 47. Given

)
= 12 − 24 + 9 = − 3 f(θ ) = (1 + sin 2 θ ) (2 − sin 2 θ )

be
Q Lf ′ (2 ) ≠ Rf ′ (2 ) = 2 + 2 sin 2 θ − sin 2 θ − sin 4 θ
∴f ( x ) is not differentiable at x = 2 = − sin 4 θ + sin 2 θ + 2

Tu
Hence, f is continuous but not differentiable at = − (sin 4 θ − sin 2 θ − 2 )
x = 2.
 1 9
= − sin 4 θ − sin 2 θ + − 

ou
45. Given,  4 4
π/4 1 π/2  x
I=∫ (tan n + 1 x ) dx + tan n − 1   dx
2 ∫0
2
9  2 1
0  2 =+ − sin θ −  …(i)

In second integral, put t =


x
2
⇒ dx = 2 dt
(Y Q
4  2
− 1 ≤ sin θ ≤ 1 ⇒ 0 ≤ sin 2 θ ≤ 1
1 1 1
on
⇒ Also, when x = 0 then t = 0, ⇒ − ≤ sin 2 θ − ≤
2 2 2
When x = π / 2, then t = π / 4 2
π/4 π/4  1 1
Then, I = ∫ (tan n + 1 x ) dx + tan n − 1 t dt ⇒ 0 ≤ sin 2 θ −  ≤
∫0
pi

0
 2 4
π/4 π/4 2
I=∫ tan n + 1 x ⋅ dx + tan n − 1 x dx  1
∫0
m

1
⇒ 0 ≥ − sin 2 θ −  ≥ −
0
 2 4
b b
{Q ∫ f ( x ) dx = ∫ f ( y ) dy}
ha

2
9 9  2 1 9 1
a a
⇒ ≥ − sin θ −  ≥ −
π/4 4 4  2  4 4
⇒ I=∫ (tan n + 1 x + tan n − 1 x ) dx
C

0 9
π/4 ⇒ 2 ≤ f(θ ) ≤ [ from Eq. (i)].
⇒ I=∫ tan n − 1 x ⋅ (tan 2 x + 1) dx 4
0
48. Given function is, f ( x ) = e x ( x − 2 )2
dy

π/4
⇒ I=∫ tan n − 1 x (sec 2 x ) dx
0 ⇒ f ′ ( x ) = e x ( x − 2 )2 + 2( x − 2 ) e x
Put t = tan x
u

= e x ( x − 2 ) ( x − 2 + 2 ) = x( x − 2 ) e x
⇒ dt = sec x dx 2
St

Now, sign scheme of f ′ ( x ) is


Also, when x = 0, then t = 0
+ – +
when x = π / 4, then t = 1 –∞ +∞
1
0 2
1 n −1 t n  1
I =∫t dt =   = So, f is increasing in (− ∞, 0 ) and (2, ∞ ) and
0 n
 0 n
decreasing in (0, 2 ).
1000
49. Let the number of terms, n = 2 m
46. lim
x→ ∞
∑ (− 1)n x n
n =1 Now, by condition
= lim { − x + x − x + x + ... + x
2 3 4 1000
} Largest coefficient in (1 + x )n 11
x→ ∞ =
Second largest coefficient in (1 + x )n 10
(given)
WB JEE (Engineering) l Solved Paper 2013 | 47
2m
Cm 11 1 
 − 2
⇒ 2m
= ⇒ g′ ( x ) = x  x 
(1 − log x )
Cm − 1 10
(m − 1)! (m + 1)! 11 For maximum or minimum of g( x ) put
⇒ = g′ ( x ) = 0
m! m! 10
1 
1 m(m − 1)! (m + 1) m ! 11  − 2
 x 
⇒ = ⇒ x (1 − log x ) = 0
m m! m! 10
⇒ log x = 1 = log e
m + 1 m !⋅ m ! 11

)
⇒ = ⇒ x =e

be
m m !⋅ m ! 10
⇒ 10 m + 10 = 11 m ⇒ m = 10 and g′ ′ ( x )|x = e > 0
So, g( x ) is minimum at x = e

Tu
∴ n = 20
Hence, total number of term ∴g( x )increases in (0, e ) and decreases in (e , ∞ ), it
will be minimum at either 2 or 5
= n + 1 = 20 + 1 = 21

ou
1 1 1
5
50. Let the tine number is in AP are ∴ 2 2 > 55 ⇒ Minimum value of f ( x ) = e ( 5 )
(a − 2d ), (a − d ), a, (a + d ), (a + 2d ) 52. Given,
where, d ≠ 0
Given, 1st, 3rd and 4th terms are in GP.
⇒ a2 = (a − 2d ) (a + d )
(Y ⇒
f ( x ) = 2| x − 1| + | x − 2|
 −2( x − 1) − ( x − 2 ), x < 1

f ( x ) = 2( x − 1) − ( x − 2 ), 1 ≤ x < 2
on
⇒ a = a − 2 ad + ad − 2 d
2 2 2  2( x − 1) + ( x − 2 ), x ≥ 2
−3 x + 4, x <1

pi

⇒ 2d + ad = 0 ⇒ d (2d + a) = 0
2
⇒ f( x) =  x, 1≤ x < 2
Q d ≠0  3 x − 4, x ≥2
m

∴ a + 2d = 0 ⇒ a = − 2d −3, x <1

f ′ ( x ) =  1, 1≤ x < 2
ha

Hence, terms are


 3, x ≥2
− 4d , − 3d , − 2 d , − d , 0
∴The fifth term is always 0. So, f ( x ) will be minimum at x = 1
C

51. Given that, and the minimum value is 1.


1 53. Given series is,
dy

x , x>0
f ( x ) = e ( x) 1 25 1 25 1 25
C0 + C1 + C2
Taking log on both sides, we get 1× 2 2×3 3× 4
u

1 1 25
+K+ C25
log + ( x ) = ( x ) x = g( x ) (say) 26 × 27
St

…(i)
1 x x
∫0 (1 + x )25 dx = ∫ [25 C0 + 25
Here, g( x ) = x x Q C1 x
0

1 + 25
C2 x 2 + K + 25
C25 x 25 ]dx
⇒ log g( x ) = log x
x On integrating w.r.t. x , taking limits 0 to x, we get
On differentiating w.r.t. x, we get x
1 (1 + x )26 
x ⋅ − log x  26 
1
⋅ g′ ( x ) = x  0
g( x ) x2 x
 x2 x3 x 26 
 1 − log x  =  25 C0 x + 25
C1 ⋅ + 25
C2 +K+ C25 ⋅
25
=
 x2 
  2 3 26  0
48 | WB JEE (Engineering) l Solved Paper 2013

1 1 55. Let the function, f ( x ) = akx


⇒  (1 + x ) − 
26
26 26 
Which define in f : R → R and injective also.
x2 x 26
= C0 x +
25 25
C1 ⋅ +K+ 25
C25 ⋅ Now, we have
2 26
f ( x )f ( y ) = f ( x + y )
Again, integrating w.r.t. x, taking limits 0 to 1, we ⇒ ak x ⋅ ak y = a k( x + y )
get
1 1 ⇒ ak( x + y ) = ak( x + y )
26 ∫0
[1 + x )26 − 1]dx

)
Q f ( x ), f ( y ) and f ( z ) are in GP

be
1 x2 x 26  ∴ f ( y )2 = f ( x )⋅ f ( z )
= ∫  25 C0 x + 25C1 ⋅ + K + 25 C25 dx
0
 2 26  ⇒ a2 ky = akx ⋅ akz

Tu
e 2 ky = e k( x + z )
1
1 (1 + x )27  ⇒
⇒ − x
26  27 0 On comparing, we get

ou
1 2ky = k( x + z ) ⇒ 2 y = x + z
 x 2 25 x3 x 27 
=  25 C0 + C1 ⋅ + K + 25C25 ⇒ x, y and z are in AP.
 2 2×3 26 × 27  0

(Y
56. Let I = ∫ e x loge x +
x + 1
2
1 2 27 1  1 25 1 25  dx
⇒ − 1−  = C0 + ⋅ C1 1  x 

26  27 27  2 2×3
2 e x
I = ∫ e x ⋅ loge x + e x +
on
1 ⇒  dx
+K+ ⋅25 C25 1  x
26 × 27
2 2 2ex
I = ∫ e x loge x dx + ∫ e xdx + ∫
pi

1 25 1 25 1 25 ⇒ dx
∴ ⋅ C0 + C1 + C2 1 1 1 x
1× 2 2×3 3× 4 2
m

⇒ I = ∫ e x loge x dx + [e x ]12 + [e x loge x ]12


1 2 27 − 28 1
+K+ ⋅25 C25 =
26 × 27 26 × 27 2
ha

− ∫ e x loge x dx
1
54. Given that, P, Q and R are angles of an isosceles
⇒ I = (e 2 − e 1 ) + (e 2 loge 2 − 0 )
π
C

triangle and ∠P =
2 = e 2 (1 + loge 2 ) − e
π 57. Given equation is
∴ Q=R= (Q P + Q + R = 180 ° )
dy

4 1  x + 1
 + log 9 ( x + 5) = 1
2
3
3 log
 P P 2  x + 5
Now, cos − i sin  + (cos Q + i sin Q )
u

 3 3
1  1   x + 1 1
(cos R − i sin R ) ⇒ ⋅  log 3   + log 3 ( x + 5) = 1
2
St

2  1/ 2  x + 5 2
+ (cos P − i sin P ) (cos Q − i sin Q )
 1 
(cos R − i sin R ) Q log a n b = log a b
 n 
3
 −i P 
= e 3  + e iQ ⋅ e − i R + e − ipe − iQ ⋅ e −iR 2  x + 1 1
  ⇒ log 3   + ⋅ 2 log 3 ( x + 5) = log 3 3
  2  x + 5 2

(Q cos θ + i sin θ = e iθ )  x + 1 
⇒ log 3  ⋅ ( x + 5)  = log 3 3
  x + 5 
= e − iP + e i(Q − R ) + e − i ( P + Q + R )
π π (Q log m + log n = log mn and log n n = 1)
= e −iπ / 2 + e i( 0 ) + e −iπ (Q = R = P= )
2 2 ⇒ ( x + 1) = 3 ⇒ x = 2
= − i + 1 + (−1) = − i So, only one solution is possible.
WB JEE (Engineering) l Solved Paper 2013 | 49

58. Given, ⇒ x( x − 4) − 1 ( x − 4) = 0

P = 1+
1
+
1
+K ⇒ ( x − 1)( x − 4) = 0 or x = 1, 4
2 × 2 3 × 22 then from is (ii) y = 2, 5
1 1 1 4
and Q= + + +K ∴ Required area = ∫ {( x + 1) − ( x 2 − 4 x + 5)} dx
1× 2 3 × 4 5 × 6 1
4
P (1 / 2 )1 (1 / 2 )2 (1 / 2 )3 4  − x 3 5x 2 
Now, = + + +K = ∫ (− x 2 + 5 x − 4)dx =  + − 4x 
1
 3 2 1

)
2 1 2 3

be
1 2 3
 1 / 2  (1 / 2 ) (1 / 2 )  64 1 5 
⇒ − p / 2 = −  − − ... = − + 40 − 16 + − + 4
 1  3  3 3 2 

Tu
 1 5
= (−21 − + 28) =
9
⇒ − P / 2 = loge 1 − 
 2 2 2
P 1 60. Given,

ou
⇒ − = loge
2 2  π 3π 
f ( x ) = sin x + 2 cos 2 x, x ∈ ,
⇒ P = 2 loge 2 …(i)  4 4 


 1  1 1  1 1
Now, Q = 1 −  +  −  +  −  + K
 2   3 4  5 6
Q = loge 2 …(ii)
(Y ∴
and
f ′ ( x ) = cos x − 4 cos x ⋅ sin x
f ′ ′ ( x ) = − sin x − 4 cos 2 x
For maximum or minimum of f ( x )
on
Now, from Eqs. (i) and (ii), we get Put f′ ( x) = 0
P = 2Q ⇒ cos x − 4 cos x ⋅ sin x = 0
pi

59. Given equation of parabola is ⇒ cos x(1 − 4 sin x ) = 0


π 1
y = x 2 − 4x + 5 ⇒ cos x = 0 = cos and sin x ≠
m

2 4
⇒ y = ( x − 2 )2 + 1  π 3π  π
ha

Q x∈ , ⇒ x=
⇒ ( x − 2 ) = ( y − 1)
2
…(i)  4 4  2
and equation of line is  π π
Now, f′′   = − sin − 4 cos π
C

 2 2
y=x+1
⇒ x − y = −1 …(ii) = − 1 + 4 = 3 > 0 (min)
dy

π
y
R So, f ( x ) is minimum at x =
(4, 5) 2
u

and its minimum value is


 π π π
St

(0, 5) f   = sin + 2 cos 2 = 1 − 2 × 0 = 1


P  2 2 2
(1, 2)
(0, 1) (2, 1) 61. Total sample space, n(S ) = 43 ⋅ 2 2 and total
x′ x
(1, 0) (2, 0) number of favourable cases
n(E ) = (3 C1 ⋅ 3 + 2C1 ⋅ 1) + 1
y′ n(E )
∴ Required probability =
On putting the value of ( y − 1) from Eqs. (ii) in (i), n(S )
we get
C1 ⋅ 3 + 2 C1 ⋅ 1 + 1 3 ⋅ 3 + 2 + 1
3

( x − 2 )2 = x ⇒ x 2 + 4 − 4x = x = =
43 ⋅ 2 2 43 ⋅ 4
⇒ x 2 − 5x + 4 = 0 12 3
= =
⇒ x 2 − 4x − x + 4 = 0 64 ⋅ 4 64
50 | WB JEE (Engineering) l Solved Paper 2013

62. Given differential equation is ⇒ log y = − 2 log x + log C


dy dx  y + 2 x  2 ⇒ log y + log x 2 = log C
( y2 + 2 x) = y⇒ = 
dx dy  y  ⇒ yx 2 = C, where C is a constant.
dx 2x
⇒ =y+ 64. Given differential equation is
dy y
 x   x 
dx 2 y sin   dx =  x sin   − y  dy
⇒ − ⋅x = y  y   y 

)
dy y
 x

be
2
∫ − y dy x sin   − y
IF = e =e −2 log y
=y −2 1
= 2 dx  y x 1
⇒ = = − …(i)
y dy  x y  x
y sin   sin  

Tu
∴ Complete solution is  y  y
1 1 x
x ⋅ 2 = ∫ y ⋅ 2 dy + C On putting v= ⇒ x = vy

ou
y y y
x dy dx dv
y2 ∫ y
⇒ = + C = loge y + C ⇒ = v ⋅1 + y in Eq. (i), we get
dy dy
⇒ x = y 2 loge y + Cy 2
At + x = 1, y = 1 then from Eq. (i), we get
…(i)
(Y v+ y
dv
dy
=v−
1
sin v
on
dv 1
1= 0 + C ⇒ C = 1 ⇒ y =−
dy sin v
∴ From Eq. (i), we get
dy
⇒ − ∫ sin v dv = ∫
pi

x = y 2 (loge y + 1) (on integrating)


y
Which is the required solution.
⇒ cos v = log y + C
m

63. Let the general equation of tangent.  x


⇒ cos   = log y + C …(i)
ha

Which passes through the point ( x, y ) is  y


dy π
(Y − y ) = ( X − x) Given at x = , y = 1 then from Eq. (i)
dx
C

4
dy dy  π
⇒ Y−y=X −x …(i) ⇒ cos   = log(1) + C
dx dx  4
dy

for length of Y-intercept, put X = 0 in Eq. (i), we


 1
get ⇒ C = 
 2
u

dy
Y−y=−x On putting the value of C in Eq. (i), we get
dx
St

dy  x 1
⇒ Y=y−x …(ii) cos   = loge y +
dx  y 2
Now,according to the guestion Which is the required solution. So no option is
Y-intercept = 3 × ordinate of the point of contact correct.
dy 65. Given lines are
⇒ y−x = 3y
dx x + y = 4 and x − y = 2
dy
⇒ −x = 2y On solving these lines, we get
dx
x = 3 and y = 1
dy 2 dx
⇒ ∫ y = − ∫ x (on integrating) Now, the equation of line which passes through
the intersection point (3, 1) having slope
WB JEE (Engineering) l Solved Paper 2013 | 51

 3 3 3− 5 + 3+ 5
θ = tan −1   is ( y − 1) = ( x − 3) = 
 4 4  2 
⇒ 4y − 4 = 3x − 9 {(cos θ + secθ )2 − 3 cos θ ⋅ secθ}
⇒ 3x − 4y = 5 …(i)
= 3 ⋅ {(3)2 − 3} = 3 (9 − 3) = 3 × 6 = 18
Now for the intersection point of the line (i) with
 3 x − 5 π/2 π /2 1 
parabola y 2 = 4( x − 3) . Put y =  , 67. Let I = ∫ [sin x ⋅ cos x ]dx = ∫ sin 2 x dx
 4  −π / 2 −π / 2 2 

)
y

be
(3 x − 5)2
then we get = 4( x − 3)
16
⇒ 9 x 2 + 25 − 30 x = 64 x − 192

Tu
x′ x
⇒ 9 x 2 − 94 x + 217 = 0 –π O +π

94 ± 8836 − 7812 94 ± 1024

ou
⇒ x= =
18 18 y′
94 ± 32 126 62
⇒ x= = or

⇒ x1 =
21
3
18
=7
18 18

(Y Put
Also, when
θ = 2 x ⇒ dθ = 2dx
x = − π / 2, then θ = − π
π
x = , then θ = π
on
when
31 2
and x2 =
9 1 π 1 
Then, I= ∫ sin θ dθ
2 −π 2 
pi

31 32 32
∴ | x1 − x2| = 7 − = =
9 9 9 1 0 π
= ∫ (−1) dx + ∫ (0 )dx 
m

66. Given, sin θ + 3 cos θ = 2


2 2 − π 0 
1 π
⇒ 1 − cos 2 θ + 3 cos θ = 2 = [− x ]0− π + 0 = −
ha

2 2
⇒ cos 2 θ − 3 cos θ + 1 = 0
68. Given,
3± 9− 4
C

⇒ cos θ = (by quadratic formula) x = 1+


1
+
1
+
1
+K
2 2 × 1! 4 × 2 ! 8 × 3 !
3± 5
dy

⇒ cos θ = (1 / 2 )1 (1 / 2 )2 (1 / 2 )3
2 ⇒ x = 1+ + + + ...
1! 2! 3!
3− 5
u

Here, cos θ =
2 ⇒ x = e 1/ 2 ⇒ x 2 = e …(i)
St

 3+ 5  x 2
x x4 6
Q cos θ ≠ − 1 ≤ cos θ ≤ 1 and y = 1+ + + +K
 2  1! 2! 3!
2 3+ 5 ( x 2 )1 ( x 2 )2 ( x 2 )3
and secθ = × ⇒ y = 1+ + + +K
3− 5 3− 5 1! 2! 3!
2
2(3 + 5 ) 3 + 5 ⇒ y = e x = ee [from Eq. (i)]
= =
4 2
Taking log on both sides, we get
Now, cos 3 θ + sec 3θ = (cos θ + secθ )
loge y = e
(cos 2 θ + sec 2θ − cos θ ⋅ secθ)
52 | WB JEE (Engineering) l Solved Paper 2013

2 −2 −4 π / 3 (sin x − x cos x )


71. Let I = ∫ dx
69. Given, P =  −1 3 4 π /6 x( x + sin x )
 
1 −2 −3 π/ 3 (x + sin x ) − x(1 + cos x )
⇒ I=∫ dx
 2 −2 −4  2 −2 −4 π /6 x( x + sin x )
P 2 = P ⋅ P =  −1 3 4   −1 3 4 
   π / 3 1 1 + cos x 
 1 −2 −3  1 −2 −3 ⇒ I=∫  −  dx
π/6  x x + sin x 
 4 + 2 − 4 −4 − 6 + 8 −8 − 8 + 12
=  −2 − 3 + 4 2 + 9 − 8 4 + 12 − 12

)
π/31+ cos x
  ⇒ I = [log x ]ππ // 36 − ∫ ⋅ dx

be
 2 + 2 − 3 −2 − 6 + 6 −4 − 8 + 9  π /6 x + sin x
 2 −2 −4  put t = x + sin x
=  −1 3 4  = P dt = (1 + cos x )dx in IInd term

Tu
  
 1 −2 −3
 π 3
 π π ' +  dt
∴ P4 = P2 = P ⇒ I = log − log  − ∫ π3 1 2 
 6

ou
3  +  t
6 2
⇒ P5 = P2 = P
π 3
 + 
70. Given infinite series is
I = log 2 − [log t ] π
2 

(Y
3
⇒ 1
12 + 2 2 12 + 2 2 + 32  + 
+  6 2
3! 4!
 π 3  π 1 
12 + 2 2 + 32 + 42 ⇒ I = log 2 − log  +  − log  +  
on
+ +K 3 2   6 2
5!  
12 + 2 2 + 32 + 42 + ... + (r + 1)2  2π + 3 3
pi

nth term, t n = ⇒ I = log 2 − log  


(r + 2 )!  π+3 
m

n
12 + 2 2 + 32 + K + (r + 1)2  m
Now , S n = Σt n = ∑ Q log m − log n = log 
r =1
(r + 2 )!  n
ha

n
(r + 1)(r + 2 )(2 n + 3)  2(π + 3) 
=∑ I = log  
r =1
6 (r + 2 )!  2π + 3 3
C

 n(n + 1) (2 n + 1)  2π + 6 
Q ∑ n2 = = log  
 6   2π + 3 3
dy

n
(2 r + 3) 1 n  2 3
=∑ = ∑ +  72. Given, f ( x ) = x 2 / 3 , x ≥ 0 and line y = x
r =1
6⋅ r ! 6 r = 1 (r − 1)! r !
y
u

1  2 3   2 3  2 3 
=  +  +  +  +  +  + K
St

6  1 1!  1! 2 !  2 ! 3 ! 
y=x

1  2 2 2  3 3 3 
=  + + + K +  + + + K 
6  1! 1! 2 !   1! 2 ! 3 ! 
1  1 1  1 1  y=x2/3
= 2 1 + + + K + 3  + + K 
6  1! 2 !   1! 2 !  x′
O
x
1
= {2e + 3(e − 1)}
6
x=1 x=8
1
= {2e + 3 e − 3} y′
6
1 5e 1 8
= (5 e − 3) = − ∴ Required area A = ∫ ( x − x 2 / 3 ) dx
6 6 2 x =1
WB JEE (Engineering) l Solved Paper 2013 | 53
8
 x 2 3 5/ 3   3   1 3 75. Given function is
= − x  =  32 − × 32 −  − 
 2 5 1  5   2 5 F( x ) = ∫
x cos t
dt , 0 ≤ x ≤ 2 π
0 (1 + t2)
2 (5 − 6) 64 1 On differentiation w.r.t. x., (apply Leibnitz rule)
= 32 × − = +
5 10 5 10 cos x cos x
F′ ( x ) = × 1= , where (1 + x 2 ) > 0
128 + 1 129 1 + x2 1 + x2
= =
10 10
 π  3π 

)
Here, cos x > 0 ⇒ x ∈  0,  ∪  , 2 π

be
73. Given function is  2  2 
 1 1 1   π 3π 
f( x) = x + + , x > 1 and cos x < 0 ⇒ x ∈  , 
 x − 1 x x + 1 2 2 

Tu
 2x 1 2 x2  π  3π 
= x 2 + = 2 +1 So, F is increasing in  0,  and  , 2 π  and
x − 1 x x − 1  2   2 

ou
   π 3 π 
decreasing in  , 
 2  2 2 
= + 1 > 3 (Q x > 1)
 1 − 
(Y
76. Radius (r ) = perpendicular distance on line
1 
  x2   4 x + 3 y = 12 from centre
∴ f( x) > 3 y
on
74. Equation of parabola is
y 2 = 4ax …(i)
pi

y
r)
(0, r) (r,
m

(–a, 2a) P(a, 2a)


=

Q
x′ x
O (r, 0) 4x+3y=12
ha

x′ x
O F(a, 0)
C

x=–a y′

directrix |4r + 3 r − 12|


⇒ r=
dy

16 + 9
y′
⇒ |7 r − 12| = 5 r
u

Let the parametric coordinate of point P on the ⇒ 7 r − 12 = ± 5 r


St

parabola is (a, 2 a). ∴ 2 r = 12 ⇒ r = 6


Now, QF = 2 2 a, and 12 r = 12 ⇒ r = 1
PQ = 2 a and PF = 2 a (i) When centre is (1, 1) and radius is 1, then
we observe that, QF 2 = PQ 2 + PF 2 equation of circle is
⇒ 8 a2 = 4 a2 + 4 a2 = 8 a2 ( x − 1)2 + ( y − 1)2 = 1

So, ∆QPF form a right angle isoceles triangle. ⇒ x2 + y2 − 2 x − 2 y + 1 = 0


In which,∠PQF = ∠PFQ (ii) When centre is (6, 6) and radius is 2, then
⇒ tan ∠PQF = tan ∠PFQ equation of circle is
tan ∠PQF ( x − 6)2 + ( y − 6)2 = 36
⇒ =1
tan ∠PFQ ⇒ x 2 + y 2 − 12 x − 12 y + 36 = 0
54 | WB JEE (Engineering) l Solved Paper 2013

77. Equation of parabola is y 2 = x and line y = mx and 2 sin α ⋅ cos α = x2c [from Eq. (iii)]
For intersection point of both curves put x = y , 2 ⇒ sin 2α = 2c (Q − 1 ≤ sin 2α ≤ 1)
1
we get ⇒ 2c ≤ 1 ⇒ c ≤
y = my 2 ⇒ y(my − 1) = 0 2
1 79. Given system of equations is
⇒ y = 0 or y =
m x+ y+ z=0
1 αx + βy + γ z = 0

)
Then, x = 0 or x = 2

be
m α 2 x + β2 y + γ 2 z = 0
 1 1
∴Intersection points are (0, 0 ) and P  2 ,  1 1 1
 m m
The coefficient matrix, A =  α β γ

Tu
y y = mx  2 
α β γ2 
2

P
y2 = x 1 1 1

ou
Now,| A| = α β γ = (α − β ) (β − γ ) (γ − α )
O
) 1 , 1
m2 m ) α 2 β2 γ2
x′ x
(0, 0)
(Y (i) The system of equations has a unique
solution, if α , β and γ are distinct
i . e ., | A| ≠ 0
on
y′ (ii) The system of equations has infinite number
of solutions, if any two of α , β and γ are
∴Required area
pi

equal.
1/ m
1/ m  y 2  y2 y3  | A| = 0
=∫  − y  dy =  − i . e .,
m  3  0
m

0
2 m 80. we know that,
1 1 1 1 if f (− x ) = f ( x ), then function is even and if
ha

= − = = (given) f (− x ) = − f ( x ), then function is odd.


2 m3 3 m3 6 m3 48
1 1 (a) f ( x ) = x 3 sin x
⇒ =± ⇒ m3 = ± 8
C

6 m3 48 f (− x ) = (− x )3 sin(− x )
Now, if m3 = 8 = − x 3 (− sin x )
dy

⇒ m3 = (2 )3 ⇒ m = 2 = x 3 sin x = f ( x )
If m = −8
3
So, f ( x ) is even.
u

⇒ m3 = (−2 )3 ⇒ m = − 2 (b) f ( x ) = x 2 cos x


St

78. Given equation is f (− x ) = (− x )2 cos(− x ) = x 2 cos x = f ( x )


x 2 − bx + c = 0 …(i) So, f ( x ) is even.
and roots are sin α and cos α (c) f ( x ) = e x x 3 sin x
Also sin α + cos α = b …(ii)
f (− x ) = e − x (− x )3 sin(− x )
and sin α ⋅ cos α = c …(iii)
Q sin 2 α + cos 2 α = (sin α + cos α )2 = e − x x 3 sin x ≠ f ( x )
− 2 sin α ⋅ cos α ∴ f ( x ) is not even.
⇒ 1 = b2 − 2c, [from Eqs. (ii) and (iii)] (d) f ( x ) = x − [ x ]
Q − 1 + 1 ≤ (sin α + cos α ) ≤ 1 + 1 f (− x ) = (− x ) − [− x ] ≠ f ( x )
∴ f ( x ) is not even.
⇒ − 2 ≤b≤ 2 [ from Eq. (ii)]
WB JEE (Engineering) l Solved Paper 2012 | 1

Engineering Entrance Exam


WB JEE

)
Solved Paper 2012

be
Tu
Physics
1. In a mercury thermometer, the ice point (lower qQ æa - bö qQ æb - aö
(a) ç ÷ (b) ç ÷
fixed point) is marked as 10° and the steam 4pe0 è ab ø 4pe0 è ab ø

ou
point (upper fixed point) is marked as 130°. At qQ æ b 1ö qQ æ a 1ö
(c) ç 2 - ÷ (d) ç 2 - ÷
40°C temperature, what will this thermometer 4pe0 èa bø 4pe0 èb bø

(Y
read?
(a) 78° (b) 66° (c) 62° (d) 58° 6. What current will flow through the 2 kW
resistor in the circuit shown in the figure?
2. The magnetic flux linked with a coil satisfies
on
6 kW 4 kW
the relation f = 4 t 2 + 6 t + 9 Wb, where t is the
time in seconds. The emf induced in the coil at
t = 2 s is
pi

72 V 3 kW 2 kW
(a) 22 V (b) 18 V (c) 16 V (d) 40 V
m

3. Water is flowing through a very narrow tube.


The velocity of water below which the flow (a) 3 mA (b) 6 mA (c) 12 mA (d) 36 mA
ha

remains a streamline flow is known as 7. In a region, the intensity of an electric field is


(a) relative velocity (b) terminal velocity $ in NC -1. The electric
given by E = 2$i + 3 $j + k
(c) critical velocity (d) particle velocity flux through a surface S = 10$i m2 in the region is
C

4. If the velocity of light in vacuum is 3 ´ 108 ms-1, (a) 5Nm2C -1 (b) 10Nm2C -1
the time taken (in nanosecond) to travel (c) 15Nm2C -1 (d) 20Nm2C -1
dy

through a glass plate of thickness 10 cm and


refractive index 1.5 is 8. A train approaching a railway platform with a
speed of 20 ms-1 starts blowing the whistle.
u

(a) 0.5 (b) 1.0 (c) 2.0 (d) 3.0


Speed of sound in air is 340 ms-1. If the
St

5. A charge + q is placed at the origin O of X-Y frequency of the emitted sound from the
axes as shown in the figure. The work done in whistle is 640 Hz, the frequency of sound to a
taking a charge Q from A to B along the person standing on the platform will appear t
straight line AB is o be

Y (a) 600 Hz (b) 640 Hz (c) 680 Hz (d) 720 Hz

9. A straight wire of length 2 m carries a current


B (0, b) of 10 A. If this wire is placed in a uniform
magnetic field of 0.15 T making an angle of 45°
with the magnetic field, the applied force on
X the wire will be
O A (a, 0)
3
(a) 1.5 N (b) 3 N (c) 3 2 N (d) N
2
2 | WB JEE (Engineering) l Solved Paper 2012

10. What is the phase difference between two This maximum acceleration of the car, for the
simple harmonic motions represented by box to remain stationary, is.
p (a) 8 ms -2 (b) 6 ms -2 (c) 4 ms -2 (d) 2 ms -2
x1 = A sinæç wt + ö÷ and x2 = A cos(wt ) ?
è 6ø
p p p 2p 16. The dimension of angular momentum is
(a) (b) (c) (d)
6 3 2 3 (a) [M0L1T-1 ] (b) [M1L2 T-2 ] (c) [M1L2 T-1 ] (d) [M2L1T-2 ]

11. Heat is produced at a rate given by H in a 17. If A = B + C have scalar magnitudes of 5, 4, 3

)
resistor, when it is connected across a supply of units respectively, then the angle betweenA

be
voltage V. If now the resistance of the resistor and C is
V
is doubled and the supply voltage is made ,
(a) cos -1 æç ö÷ (b) cos -1 æç ö÷
3 4
3
è 5ø è 5ø

Tu
then the rate of production of heat in the
p
(d) sin-1 æç ö÷
3
resistor will be (c)
2 è 4ø
H H

ou
(a) (b) (c) 6H (d) 18H
18 9 18. A particle is travelling along a straight line
OX. The distance x (in metre) of the particle
12. Two elements A and B with atomic numbers from O at a time t is given by x = 37 + 27t - t 3,
Z A and Z B are used to produce characteristic
X-rays with frequencies n A and n B,
respectively. If Z A : Z B = 1 : 2 , then n A : n B will (Ywhere t is time in seconds. The distance of the
particle from O when it comes to rest is
(a) 81 m (b) 91 m (c) 101 m (d) 111 m
on
be
(a) 1 : 2 (b) 1 : 8 (c) 4 : 1 (d) 1 : 4 19. A particle is projected from the ground with a
kinetic energy E at an angle of 60° with the
pi

13. The de-Broglie wavelength of an electron horizontal. Its kinetic energy at the highest
c
moving with a velocity (c = velocity of light in point of its motion will be
2
m

vacuum) is equal to the wavelength of a E E E E


(a) (b) (c) (d)
photon. The ratio of the kinetic energies of 2 2 4 8
ha

electron and photon is


20. A bullet on penetrating 30 cm into its target
(a) 1 : 4 (b) 1 : 2 (c) 1 : 1 (d) 2 : 1 loses its velocity by 50%. What additional
C

14. Two infinite parallel metal planes, contain distance will it penetrate into the target before
electric charges with charge densities +s and-s it comes to rest?
dy

respectively and they are separated by a small (a) 30 cm (b) 20 cm (c) 10 cm (d) 5 cm
distance in air. If the permittivity of air is e0,
then the magnitude of the field between the
21. When a spring is stretched by 10 cm, the
u

potential energy stored is E. When the spring


two planes with its direction will be
is stretched by 10 cm more, the potential
s
St

(a) , towards the positively charged plane energy stored in the spring becomes
e0
s (a) 2 E (b) 4 E (c) 6 E (d) 10 E
(b) , towards the negatively charged plane
e0 22. Average distance of the Earth from the Sun is
s L1. If one year of the Earth = D days, then one
(c) , towards the positively charged plane
(2 e0 ) year of another planet whose average distance
(d) 0, towards any direction from the Sun is L2 will be
1 3
15. A box of mass 2 kg is placed on the roof of a car. æL ö2 æL ö2
The box would remain stationary untill the car (a) Dçç 2 ÷ days
÷ (b) Dçç 2 ÷ days
÷
è L1 ø è L1 ø
attains a maximum acceleration. Coefficient of 2
static friction between the box and the roof of æL ö3 æL ö
the car is 0.2 and g = 10ms-2. (c) Dçç 2 ÷ days
÷ (d) Dçç 2 ÷ days
÷
è L1 ø è L1 ø
WB JEE (Engineering) l Solved Paper 2012 | 3

23. A spherical ball A of mass 4 kg, moving along a (a) 6 s, 64 m (b) 6 s, 48 m


straight line strikes another spherical ball B of (c) 4 s, 32 m (d) 4 s, 16 m
mass 1 kg at rest. After the collision, A and B
29. A Wheatstone bridge has the resistance 10W,
move with velocities v1ms-1 and v2ms-1
10W, 10W and 30W in its four arms. What
respectively, making angles of 30° and 60° with
resistance joined in parallel to the 30W
respect to the original direction of motion of A.
v resistance will bring it to the balanced
The ratio 1 will be conditions?
v2

)
3 4 1 (a) 2W (b) 5W (c) 10W (d) 15W

be
(a) (b) (c) (d) 3
4 3 3 30. An electric bulb marked as 50 W - 200 V is
connected across a 100 V supply. The present
24. The decimal number equivalent to a binary

Tu
power of the bulb is
number 1011001 is
(a) 37.5 W (b) 25 W (c) 12.5 W (d) 10 W
(a) 13 (b) 17 (c) 89 (d) 178

ou
31. A magnetic needle is placed in a uniform
25. The frequency of the first overtone of a closed
magnetic field and is aligned with the field.
pipe of length l1 is equal to that of the first
The needle is now rotated by an angle of 60°
overtone of an open pipe of length l2. The ratio

(Y
and the work done is W. The torque on the
of their lengths ( l1 : l2 ) is
magnetic needle at this position is
(a) 2 : 3 (b) 4 : 5 (c) 3 : 5 (d) 3 : 4
3 3
(a) 2 3 W (b) 3 W (c) W (d) W
on
26. The I-V characteristics of a metal wire at two 2 4
different temperatures (T1 and T2) are given in
the adjoining figure. Here, we can conclude that 32. In the adjoining figure, the potential difference
pi

between X and Y is 60 V. The potential


I
difference between the points M and N will be
T1
m

2C
X M
ha

T2
C C
C

Y N
V 2C
(a) T1 > T2 (b) T1 < T2 (c) T1 = T2 (d) T1 = 2 T2
(a) 10 V (b) 15 V (c) 20 V (d) 30 V
dy

27. In a slide caliper, ( m + 1) number of vernier 33. A body, when fully immersed in a liquid of
divisions is equal to m number of smallest specific gravity 1.2 weighs 44 gwt. The same
u

main scale divisions. If d unit is the magnitude body when fully immersed in water weighs 50
of the smallest main scale division, then the
St

gwt. The mass of the body is


magnitude of the vernier constant is
(a) 36 g (b) 48 g (c) 64 g (d) 80 g
d d
(a) unit (b) unit
(m + 1) m 34. When a certain metal surface is illuminated
md (m + 1)d with light of frequency n, the stopping potential
(c) unit (d) unit
(m + 1) m for photoelectric current is V0. When the same
n
surface is illuminated by light of frequency ,
28. From the top of a tower, 80 m high from the 2
ground, a stone is thrown in the horizontal the stopping potential
direction with a velocity of 8 ms-1. The stone V
is 0 . The threshold frequency for
reaches the ground after a time t and falls at a 4
distance of d from the foot of the tower. photoelectric, emission is
Assuming, g = 10 m/s2, the time t and distance n n 2n 4n
(a) (b) (c) (d)
d are given respectively by 6 3 3 3
4 | WB JEE (Engineering) l Solved Paper 2012

35. Three blocks of masses 4 kg, 2 kg, 1 kg 38. 22320 cal of heat is supplied to 100 g of ice at
respectively, are in contact on a frictionless 0° C. If the latent heat of fusion of ice is
table as shown in the figure. If a force of 14 N is 80 cal g -1 and latent heat of vaporisation of
applied on the 4 kg block, the contact force water is 540 cal g -1, the final amount of water
between the 4 kg and the 2 kg block will be thus obtained and its temperature
respectively are
14 N (a) 8 g, 100° C (b) 100 g, 90°C
4 kg 2 kg 1 kg
(c) 92 g, 100° C (d) 82 g, 100° C

)
be
(a) 2 N (b) 6 N
39. A progressive wave moving along x-axis is
(c) 8 N (d) 14 N
é 2p ù

Tu
36. Let L be the length and d be the diameter of represented by y = A sin ê (vt - x )ú . The
cross-section of a wire. Wires of the same ë l û
material with different L and d are subjected to wavelength ( l ) at, which the maximum

ou
the same tension along the length of the wire. In
which of the following cases, the extension of particle velocity is 3 times the wave velocity is

(c) æç ö÷ pA (d) æç ö÷ pA
wire will be the maximum? A 2A 3 2
(a) (b)

(Y
(a) L = 200 cm, d = 0.5 mm 3 3p è 4ø è 3ø
(b) L = 300 cm, d = 10
. mm
40. Two radioactive substances A and B have
(c) L = 50 cm, d = 0.05 mm
decay constants 5l and l, respectively. At
(d) L = 100 cm, d = 02
on
. mm
t = 0, they have the same number of nuclei.
37. An object placed in front of a concave mirror at a The ratio of number of nuclei of A to that of B
2
distance of x cm from the pole gives a 3 times æ1ö
pi

will be ç ÷ after a time interval of


magnified real image. If it is moved to a distance è eø
of ( x + 5) cm, the magnification of the image
m

1 1 1 1
becomes 2. The focal length of the mirror is (a) (b) (c) (d)
l 2l 3l 4l
(a) 15 cm (b) 20 cm (c) 25 cm (d) 30 cm
ha
C

Chemistry
1. Li occupies higher position in the
dy

*
(b) two unpaired electrons is p MO
electrochemical series of metals as compared to *
Cu since (c) two unpaired electrons is s MO
(d) two unpaired electrons is sb MO
u

(a) the standard reduction potential of Li+ / Li is lower


than that of Cu2+ /Cu 4. A 100 mL 0.1 = (M) solution of ammonium
St

(b) the standard reduction potential of Cu2+ /Cu is acetate is diluted by adding 100 mL of water.
lower, than that of Li/Li The pH of the resulting solution will be (pK a of
(c) the standard oxidation potential of Li/Li+ is lower, acetic acid is nearly equal to pK b of NH4OH)
than that of Cu/Cu2+
(d) Li is smaller in size as compared to Cu (a) 4.9 (b) 5
(c) 7 (d) 10
24
2. 11 Na is radioactive and it decays to
5. In 2-butene, which one of the following
(a) 9 F and a-particles
20
(b) 13 Al 24 and positron statements is true?
(c) 11 Na23 and neutron (d) 12 Mg 24 and b-particles
(a) C1 ¾ C2 bond is a sp3 - sp3 - s-bond
3. The paramagnetic behaviour of B2 is due to the (b) C2 ¾ C3 bond is a sp3 - sp2 - s-bond
presence of
(c) C1 ¾ C2 bond is a sp3 - sp2 - s-bond
(a) two unpaired electrons is p b MO (d) C1 ¾ C2 bond is a sp2 - sp2 - s-bond
WB JEE (Engineering) l Solved Paper 2012 | 5

6. The well known compounds, ( + )-lactic acid and 12. Which of the following will show a negative
( - )-lactic acid, have the same molecular deviation from Raoult's law?
formula, C 3H6O 3. The correct relationship (a) Acetone-benzene
between them is (b) Acetone-ethanol
(a) constitutional isomerism (c) Benzene-methanol
(b) geometrical isomerism (d) Acetone-chloroform
(c) identicalness
(d) optical isomerism
13. In a reversible chemical reaction at

)
equilibrium, if the concentration of any one of

be
7. The stability of Me2C==CH2 is more than that the reactants is doubled, then the equilibrium
of MeCH2CH == CH2 due to constant will
(a) inductive effect of the Me groups (a) also be doubled

Tu
(b) resonance effect of the Me groups (b) be halved
(c) hyperconjugative effect of the Me groups (c) remains the same
(d) resonance as well as inductive effect of the Me (d) becomes one-fourth

ou
groups
14. Identify the correct statement from the
8. Which one of the following characteristics following in a chemical reaction.

(Y
belongs to an electrophile?
(a) The entropy always increases.
(a) It is any species having electron deficiency which (b) The change in entropy alongwith suitable change
reacts at an electron rich C-centre. in enthalpy decides the rate of reaction.
on
(b) It is any species having electron enrichment, that (c) The enthalpy always decreases.
reacts at an electron deficient C-centre. (d) Both the enthalpy and the entropy remain constant.
(c) It is cationic in nature.
(d) It is anionic in nature. 15. Which one of the following is wrong about
pi

molecularity of a reaction?
9. Which one of the following methods is used to
m

prepare Me3COEt with a good yield? (a) It may be whole number or fractional.
(b) It is calculated from reaction mechanism
(a) Mixing EtONa with Me 3 CCl (c) It is the number of molecules of the reactants
ha

(b) Mixing Me 3 CONa with EtCl taking part in a single step chemical reaction.
(c) Heating a mixture of (1 : 1) EtOH and Me 3 COH in (d) It is always equal to the order of elementary
the presence of conc. H 2 SO4 . reaction.
C

(d) Treatment of Me 3 COH with EtMgI


16. Which of the following does not represent the
10. 58.4g of NaCl and 180g of glucose were mathematical expression for the Heisenberg
dy

separately dissolved in 1000 mL of water. uncertainty principle?


Identify the correct statement regarding the
(a) Dx × DP ³ h / (4p ) (b) Dx × Dv ³ h / (4pm)
elevation of boiling point (bp) of the resulting
(c) DE × Dt ³ h / (4p ) (d) DE × Dx ³ h / (4p )
u

solutions.
St

(a) NaCl solution will show higher elevation of boiling 17. The stable bivalency of Pb and trivalency of
point. Bi is
(b) Glucose solution will show higher elevation of (a) due to d contraction in Pb and Bi
boiling point. (b) due to relativistic contraction of the 6s-orbitals of
(c) Both the solutions will show equal elevation of Pb and Bi, leading to inert pair effect
boiling point. (c) due to screening effect
(d) The boiling point elevation will be shown by neither (d) due to attainment of noble liquid configuration
of the solutions.
18. The equivalent weight of K 2Cr2O7 in acidic
11. Equal weights of CH4 and H2 are mixed in an medium is expressed in terms of its molecular
empty container at 25°C. The fraction of the weight (M) as
total pressure exerted by H2 is
(a) M/3 (b) M/4
(a) 1/9 (b) 1/2 (c) 8/9 (d) 16/17 (c) M/6 (d) M/7
6 | WB JEE (Engineering) l Solved Paper 2012

19. Which of the following is correct? 27. Which one of the following properties is
(a) Radius of Ca 2+
< Cl < S - 2- exhibited by phenol?
(b) Radius of Cl - < S2 - < Ca2 + (a) It is soluble in aq. NaOH and evolves CO2 with
(c) Radius of S2 - = Cl - = Ca2 + aq. NaHCO3 .
(d) Radius of S2 - < Cl - < Ca2 + (b) It is soluble in aq. NaOH and does not evolve CO2
with aq. NaHCO3 .
20. CO is practically non-polar since (c) It is not soluble in aq. NaOH but evolves CO2 with
(a) the s-electron drift from C to O is almost nullified aq. NaHCO3 .

)
by the p-electron drift from O to C (d) It is insoluble in aq. NaOH and does not evolve

be
(b) the s-electron drift from O to C is almost nullified CO2 with aq. NaHCO3 .
by the p-electron drift from C to O
(c) the bond moment is low
28. The basicity of aniline is weaker in comparison

Tu
(d) there is a triple bond between C and 0 to that of methyl amine due to
(a) hyperconjugative effect of Me-group in MeNH 2
21. The number of acidic protons in H3PO 3 are (b) resonance effect of phenyl group in aniline

ou
(a) 0 (b) 1 (c) 2 (d) 3 (c) lower molecular weight of methyl amine as
compared to that of aniline
22. When H2O 2 is shaken with an acidified (d) resonance effect of ¾ NH 2 group in MeNH 2

(Y
solution of K 2Cr2O7 in the presence of ether, the
ethereal layer turns blue due to the formation 29. Under identical conditions, the SN 1 reaction
of will occur most efficiently with

(b) CrO2- (a) tert-butyl chloride


on
(a) Cr2 O3 4
(b) 1-chlorobutane
(c) Cr2 (SO4 )3 (d) CrO5 (c) 2-methyl-1-chloropropane
23. The state of hybridisation of the central atom (d) 2-chlorobutane
pi

and the number of lone pairs over the central 30. Identify the method by which Me3CCO 2H can
atom in POCl 3 are
m

be prepared.
(a) sp, 0 (b) sp2 , 0 (c) sp3 , 0 (d) dsp2 , 1 (a) Treating 1 mole of MeCOMe with 2 moles of
ha

MeMgI.
24. Upon treatment with I2 and aqueous NaOH, (b) Treating 1 mole of MeCO 2Me with 3 moles of
which of the following compounds will from MeMgI.
iodoform?
C

(c) Treating 1 mole of MeCHO with 3 moles of MeMgI.


(a) CH 3 CH 2 CH 2 CH 2 CHO (d) Treating 1 mole of dry ice with 1 mol of Me 3CMgI
(b) CH 3 CH 2 COCH 2 CH 3
dy

(c) CH 3 CH 2 CH 2 CH 2 CH 2 OH 31. 20 mL 0.1 (N) acetic acid is mixed with


(d) CH 3 CH 2 CH 2 CH(OH)CH 3 10 mL 0.1 (N) solution of NaOH. The pH of the
resulting solution is (pK a of acetic acid is 4.74)
u

25. Upon treatment with Al(OEt)3 followed by


(a) 3.74 (b) 4.74
usual reactions (work up), CH3CHO will
St

(c) 5.74 (d) 6.74


produce
(a) only CH 3 COOCH 2 CH 3 32. In the brown ring complex[Fe(H2O)5(NO)]SO 4,
(b) a mixutre of CH 3 COOH and EtOH nitric oxide behaves as
(c) only CH 3 COOH (a) NO+ (b) netural NO molecule
(d) only EtOH (c) NO- (d) NO2-
26. Friedel-Craft’s reaction using MeCl and 33. The most contributing tautomeric enol from of
anhydrous AlCl 3 will take place most MeCOCH2CO 2Et is
efficiently with
(a) CH 2 == C(OH)CH 2 CO2 Et
(a) benzene (b) MeC(OH) == CHCO2 Et
(b) nitrobenzene (c) MeCOCH == C(OH)OEt
(c) acetophenone (d) CH 2 == C(OH)CH == C(OH)OEt
(d) toluene
WB JEE (Engineering) l Solved Paper 2012 | 7

34. By passing excess Cl 2( g ) in boiling toluene, 37. Which of the following is correct?
which one of the following compounds is (a) Evaporation of water causes an increase in
exclusively formed? disorder of the system.
Me Me (b) Melting of ice causes a decrease in randomness
Cl Cl of the system.
Cl (c) Condensation of steam causes an increase in
(a) (b)
disorder of the system.
Cl (d) There is practically no change in the randomness

)
Cl Cl of the system, when water is evaporated.

be
CCl3 CCl3
Cl 38. On passing C ampere of current for time t sec
Cl through 1 L of 2 (M) CuSO 4 solution
(c) (d) (Atomic weight of Cu = 63.5), the amount m of

Tu
Cu (in gram) deposited on cathode will be
Cl (a) m = Ct / (63.5 ´ 96500)

ou
(b) m = Ct / (3125
. ´ 96500)
35. An equimolar mixture of toluene and
(c) m = (C ´ 96500) / (3125
. ´t )
chlorobenzene is treated with a mixture of
(d) m = (3125
. ´ C ´ t ) / 96500
conc. H2SO 4 and conc. HNO 3. Indicate the
correct statement from the following.
(a) p-nitrotoluene is formed in excess.
(b) Equimolar amounts of p-nitrotoluene and
(Y
39. If the first ionisation energy of H atom is 13.6
eV, then the second ionisation energy of He
on
p-nitrochlorobenzene are formed. atom is
(c) p-nitrochlorobenzene is formed in excess. (a) 27.2 eV (b) 40.8 eV
(d) m-nitrochlorobenzene is formed in excess.
pi

(c) 54.4 eV (d) 108.8 eV


36. Among the following carbocations 40. The weight of oxalic acid that will be required
Ph 2C +CH2Me (I), PhCH 2CH 2CH + Ph (II),
m

to prepare a 1000 mL (N/20) solution is


Ph 2 CHCH + Me (III) and Ph 2C(Me)CH2 (IV),
(a) 126/100 g (b) 63/40 g
ha

the order of stability is


(c) 63/20 g (d) 126/20 g
(a) IV > II > I > III (b) I > II > III > IV
(c) II > I > IV > III (d) I > IV > III > II
C

Mathematics
dy

1. If ( a + b ) and ( a - b ) are the roots of the (a) 0 (b) 1


equation x 2 + px + q = 0, where a, b, p and q (c) 2 (d) 3
u

are real, then the roots of the equation


( p2 - 4q)( p2x 2 + 4 px ) - 16q = 0 are
3. The sum of the series
St

1n 1 1
æ1 1 ö æ1 1 ö 1+ C1 + nC2 + nCn is equal to
(a) çç + ÷÷ and çç - ÷ n+1
b ÷ø
a a
2 3
è b ø è
æ 1 1ö æ 1 1ö 2n + 1 - 1 3(2 n - 1)
(b) ç + ÷ and ç - ÷ (a) (b)
è a bø è a bø n+1 2n
æ 1 1 ö æ 1 1 ö 2n + 1 2n + 1
(c) çç + ÷ and çç - ÷÷ (c) (d)
è a b ÷ø è a b ø n+1 2n
(d) ( a + b ) and ( a - b ) ¥ 1 + 2 + K + ( r - 1)
4. The value of S
2. The number of solutions of the equation r = `2 r!
log 2( x 2 + 2x - 1) = 1 is e 3e
(a) e (b) 2e (c) (d)
2 2
8 | WB JEE (Engineering) l Solved Paper 2012

5. If P = éê
1 2 1ù 13. Two coins are available, one fair and the other
, Q = PP T , then the value of the
ë1 3 1úû two-headed. Choose a coin and toss it once;
determinant of Q is assume that the unbiased coin is chosen with
3
(a) 2 (b) -2 (c) 1 (d) 0 probability . Given that the outcome is head,
4
6. The remainder obtained when 1 ! + 2 ! K + 95 ! the probability that the two-headed coin was
is divided by 15 is chosen, is
(a) 14 (b) 3 (c) 1 (d) 0 3 2

)
(a) (b)
5 5

be
7. If P, Q and R are angles of DPQR, then the (c)
1
(d)
2
-1 cos R cos Q 5 7
-1 cos P is equal to

Tu
value of cos R 14. Let R be the set of real numbers and the
cos Q cos P -1 functions f : R ® R and g : R ® R be defined
1 by f ( x ) = x 2 + 2x - 3 and g( x ) = x + 1. Then, the
(a) -1

ou
(b) 0 (c) (d) 1 value of x for which f ( g( x )) = g( f ( x )) is
2
(a) -1 (b) 0
8. The number of real values of a for which the (c) 1 (d) 2

(Y
system of equations
x + 3 y + 5 z = ax 15. If a, b and c are in arithmetic progression, then
the roots of the equation ax 2 - 2bx + c = 0 are
5x + y + 3 z = ay
on
c 1
3 x + 5 y + z = az (a) 1 and (b) - and -c
a a
has infinite number of solutions is c c
(a) 1 (b) 2 (c) 4 (d) 6 (c) -1and - (d) -2 and -
pi

a 2a
9. The total number of injections (one one into 16. The equation y2 + 4x + 4 y + k = 0 represents a
m

mappings) from { a1, a 2, a 3, a 4 } to parabola whose latusrectum is


{ b1, b2, b3, b4, b5, b6, b7 } is
ha

(a) 1 (b) 2
(a) 400 (b) 420 (c) 800 (d) 840 (c) 3 (d) 4
10
10. Let (1 + x )10 = S cr xr and 17. If the circles x 2 + y2 + 2x + 2ky + 6 = 0 and
C

r =0
7 5 x 2 + y2 + 2ky + k = 0 intersect orthogonally,
(1 + x )7 = S dr xr . If P = S c2r and then k is equal to
r =0 r =0
dy

3 P 3 3
Q = S d2r + 1, then is equal to (a) 2 or - (b) -2 or -
r =0 Q 2 2
3 3
(c) 2 or (d) -2 or
u

(a) 4 (b) 8 (c) 16 (d) 32 2 2


St

11. Two decks of playing cards are well shuffled 18. If four distinct points (2k, 3 k), (2, 0), (0, 3),
and 26 cards are randomly distributed to a (0, 0) lie on a circle, then
player. Then, the probability that the player
(a) k < 0 (b) 0 < k < 1
gets all distinct cards is
(c) k = 1 (d) k >1
(a) 52
C 26 / 104
C 26 (b) 2 ´ 52C 26 / 104
C 26
(c) 2 13
´ C 26 /
52 104
C 26 (d) 2 26
´ C 26 /
52 104
C 26 19. The line joining a( b cos a , b sin a ) and
B ( a cos b, a sin b ), where a ¹ b, is produced to
12. An urn contains 8 red and 5 white balls. Three the point M ( x, y) so that AM : MB = b : a.
balls are drawn at random. Then, the a +b a +b
then, x cos + y sin is equal to
probability that balls of both colours are drawn 2 2
is
(a) 0 (b) 1
40 70 3 10
(a) (b) (c) (d) (c) -1 (d) a2 + b 2
143 143 13 13
WB JEE (Engineering) l Solved Paper 2012 | 9
x2 28. Maximum value of the function f ( x ) =
x 2
+ on
20. Let the foci of the ellipse + y2 = 1 subtend a
9 8 x
right angle at a point P. Then, the locus of P is the interval [1, 6] is
(a) x + y = 1
2 2
(b) x + y = 2
2 2 9 13 17
(a) 1 (b) (c) (d)
8 12 8
(c) x + y = 4
2 2
(d) x + y = 8
2 2

p 3p
29. For - < x < , the value of
21. The general solution of the differential 2 2
x + y+1 d ì cos x ü

)
dy -1
equation = is ítan ý is equal to

be
dx 2x + 2 y + 1 dx î 1 + sin x þ

(a) loge|3 x + 3 y + 2| + 3 x + 6 y = C (a)


1
(b) -
1

Tu
(b) loge|3 x + 3 y + 2| - 3 x + 6 y = C 2 2
(c) loge|3 x + 3 y + 2| - 3 x - 6 y = C sin x
(c) 1 (d)
(d) loge|3 x + 3 y + 2| + 3 x - 6 y = C (1 + sin x)2

ou
2
22. The value of the integral 30. The value of the integral ò (1 + 2 sin x )e|x|dx is
p / 2æ 1 + sin 2x + cos 2x ö -2
òp / 6 çç sin x + cos x ÷÷ dx is equal to equal to

(Y
è ø
(a) 0 (b) e 2 - 1 (c) 2(e 2 - 1) (d) 1
(a) 16 (b) 8
(c) 4 (d) 1 31. The maximum value of | z|, when the complex
on
number z satisfies the condition½z + ½ = 2 is
p/2
2
1
23. The value of the integral ò dx is ½ z½
0 1 + (tan x )101
(a) 3 (b) 3 + 2
pi

equal to
(c) 3 + 1 (d) 3 - 1
p p p
(a) 1 (b) (c) (d)
m

50
6 8 4 æ3 3ö
32. If çç + i ÷÷ = 3 25( x + iy), where x and y are
24. The integrating factor of the differential è2 2 ø
ha

dy
equation 3 x log e x + y = 2 log e x is given by real, then the ordered pair ( x, y) is
dx
(a) (-3, 0) (b) (0, 3)
(a) (loge x)3
C

(b) loge (loge x) æ1 3ö


(c) (0, - 3) (d) çç , ÷÷
(c) loge x (d) (loge x)1/ 3 è2 2 ø
dy

25. Number of solutions of the equation z -1


33. If is purely imaginary, then
tan x + sec x = 2 cos x, x Î [0, p ] is z +1
u

(a) 0 (b) 1 (c) 2 (d) 3 1


(a)| z| = (b)| z| = 1 (c)| z| = 2 (d)| z| = 3
p / 4 sin x + cos x 2
26. The value of the integral ò
St

dx is
0 3 + sin 2x
equal to
34. There are 100 students in a class. In an
examination, 50 of them failed in
(a) loge 2 (b) loge 3 Mathematics, 45 failed in Physics, 40 failed in
1 1
(c) loge 2 (d) loge 3 Biology and 32 failed in exactly two of the three
4 4 subjects. Only one student passed in all the
æ 3x - 1 ö subjects. Then, the number of students failing
27. Let y = çç x ÷ sin x + log e(1 + x ),
÷ x > -1. in all the three subjects
è3 + 1ø
dy (a) is 12
Then , at x = 0, equals (b) is 4
dx
(c) is 2
(a) 1 (b) 0 (d) cannot be determined from the given information
(c) -1 (d) -2
10 | WB JEE (Engineering) l Solved Paper 2012

35. A vehicle registration number consists of 2 44. The equations x2 + x + a = 0 and


letters of English alphabet followed by 4 digits, x + ax + 1 = 0 have a common real root
2

where the first digit is not zero. Then, the total (a) for no value of a
number of vehicles with distinct registration (b) for exactly one value of a
numbers is (c) for exactly two values of a
(a) 262 ´ 104 (b) 26
P2 ´ 10 P4 (d) for exactly three values of a
(c) 26
P2 ´ 9 ´ 10
P3 (d) 262 ´ 9 ´ 103 45. If 64, 27, 36 are the Pth Qth and Rth terms of a

)
GP, then P + 2Q is equal to
36. The number of words that can be written using

be
all the letters of the word ‘IRRATIONAL’ is (a) R (b) 2R
10! 10! 10! (c) 3R (d) 4R
(a) (b) (c) (d) 10! 3p

Tu
(2 !)3 (2 !)2 2! 46. If sin -1 x + sin -1 y + sin -1 z = , then the
2
37. Four speakers will address a meeting where 1
value of x 9 + y9 + z 9 - is equal to
x 9 y9 z 9

ou
speaker Q will always speak P. Then, the
number of ways in which the order of speakers
(a) 0 (b) 1 (c) 2 (d) 3
can be prepared is

(Y
(a) 256 (b) 128 (c) 24 (d) 12 47. Let p, q and r be the sides opposite to the
angles P, Q and R, respectively in a DPQR. If
38. The number of diagonals in a regular polygon r 2 sin P sin Q = pq, then the triangle is
of 100 sides is
on
(a) equilateral
(a) 4950 (b) 4850 (c) 4750 (d) 4650 (b) acute angled but not equilateral
(c) obtuse angled
39. Let the coefficients of powers of x in the 2nd,
pi

(d) right angled


3rd and 4th terms in the expansion of (1 + x )n ,
where n is a positive integer, be in arithmetic
m

48. Letp, q andr be the sides opposite to the


progression. Then, the sum of the coefficients
angles P, Q and R respectively in a DPQR
of odd powers of x in the expansion is
ha

æ P - Q + Rö
(a) 32 (b) 64 (c) 128 (d) 256 Then,2 pr sin ç ÷ equals
è 2 ø
40. Let f ( x ) = ax + bx + c, g( x ) = px 2 + qx + r
2
C

(a) p2 + q 2 + r 2 (b) p2 + r 2 - q 2
such that f (1) = g(1), f (2) = g(2) and
f (3 ) - g(3 ) = 2. Then f (4) - g(4) is (c) q 2 + r 2 - p2 (d) p2 + q 2 - r 2
dy

(a) 4 (b) 5 (c) 6 (d) 7 49. Let P(2, - 3 ), Q ( -2, 1) be the vertices of the
DPQR. If the centroid of DPQR lies on the line
41. The sum 1 ´ 1 ! + 2 ´ 2 ! + K + 50 ´ 50 ! equals
2x + 3 y = 1, then the locus of R is
u

(a) 51! (b) 51! - 1


(a) 2 x + 3 y = 9 (b) 2 x - 3 y = 7
(c) 51! + 1 (d) 2 ´ 51!
St

(c) 3 x + 2 y = 5 (d) 3 x - 2 y = 5
42. Six numbers are in AP such that their sum is 3. px - 1
The first term is 4 times the third term. Then, 50. lim
the fifth term is
x ®0 1 + x -1

(a) -15 (b) -3 (c) 9 (d) -4 (a) does not exist (b) equals loge (p 2 )
(c) equals 1 (d) lies between 10 and 11
43. The sum of the infinite series
1 1 ×3 1 ×3 ×5 1 ×3 ×5 × 7 51. If f is a real-valued differentiable function
1+ + + + +K such that f ( x )f ¢ ( x ) < 0 for all real x, then
3 3 × 6 3 × 6 × 9 3 × 6 × 9 × 12
is equal to (a) f( x) must be an increasing function
(b) f( x) must be a decreasing function
3 1
(a) 2 (b) 3 (c) (d) (c)|f( x)| must be an increasing function
2 3 (d)|f( x)| must be a decreasing function
WB JEE (Engineering) l Solved Paper 2012 | 11

52. Rolle’s theorem is applicable in the interval 59. The number of integer values of m , for which
[ -2, 2] for the function the x-coordinate of the point of intersection of
(a) f( x) = x3 (b) f( x) = 4 x4 the lines 3 x + 4 y = 9 and y = mx + 1 is also an
integer, is
(c) f( x) = 2 x3 + 3 (d) f( x) = p| x|
(a) 0 (b) 2
2
d y dy (c) 4 (d) 1
53. The solution of 25 - 10 + y = 0, y(0) = 1,
dx 2 dx
60. If a straight line passes through the point ( a, b)
y(1) = 2e1/ 5 is

)
and the portion of the line intercepted between

be
(a) y = e 5 x + e -5 x (b) y = (1 + x)e 5 x the axes is divided equally at that point, then
(c) y = (1 + x)e x / 5 (d) y = (1 + x)e - x / 5 x y
+ is
a b

Tu
54. Let P be the mid point of a chord joining the (a) 0 (b) 1 (c) 2 (d) 4
vertex of the parabola y2 = 8x to another point
10
on it. Then, the locus of P is 61. The coefficient of x in the expansion of

ou
(a) y = 2 x
2
(b) y = 4 x
2 1 + (1 + x ) + K + (1 + x )20 is
x2 y2 (a) 19 C 9 (b) 20
C10
(c) + y2 = 1 (d) x2 + =1 21 22

(Y
4 4 (c) C11 (d) C12

55. The line x = 2 y intersects the ellipse 62. The system of linear equations :
x2
on
+ y = 1 at the points P and Q. The equation
2 lx + y + z = 3
4
x - y - 2z = 6
of the circle with PQ as diameter is
pi

1 - x + y + z = m has
(a) x2 + y2 = (b) x2 + y2 = 1
2 (a) infinite number of solutions for l ¹ -1and all m
m

5
(c) x2 + y2 = 2 (d) x2 + y2 = (b) infinite number of solutions for l = - 1 and m = 3
2 (c) no solution for l ¹ -1
ha

56. The eccentric angle in the first quadrant of a (d) unique solution for l = - 1and m = 3
x 2 y2 63. Let A and B be two events with P( A C ) = 0.3,
point on the ellipse + = 1 at a distance 3
C

10 8 P( B) = 0.4 and P( A Ç BC ) = 0.5. Then,


units from the centre of the ellipse is P( B| A È BC ) is equal to
p p p p 1 1 1 2
dy

(a) (b) (c) (d) (a) (b) (c) (d)


6 4 3 2 4 3 2 3
57. The transverse axis of a hyperbola is along the 64. Let p, q and r be the altitudes of a triangle with
u

X-axis and its length is 2a. The vertex of the area S and perimeter 2 t. Then, the
St

hyperbola bisects the line segment joining the


1 1 1
centre and the focus. The equation of the value of + + is
hyperbola is p q r
(a) 6 x2 - y2 = 3a2 (b) x2 - 3 y2 = 3a2 S t S 2S
(a) (b) (c) (d)
(c) x - 6 y = 3a
2 2 2
(d) 3 x - y = 3a
2 2 2 t S 2t t

58. A point moves in such a way that the difference 65. Let C1 and C2 denote the centres of the circles
of its distance from two points x 2 + y2 = 4 and ( x - 2)2 + y2 = 1 respectively
(8, 0) and (-8, 0) always remains 4. Then, the and let P and Q be their points of
locus of the point is intersection.Then, the areas of DC1PQ and
D C2PQ are in the ratio
(a) a circle (b) a parabola
(c) an ellipse (d) a hyperbola (a) 3 : 1 (b) 5 : 1
(c) 7 : 1 (d) 9 : 1
12 | WB JEE (Engineering) l Solved Paper 2012

66. A straight line through the point of 73. The value of the integral
intersection of the lines x + 2 y = 4 and 5
2x + y = 4 meets the coordinate axes at A and B. ò1 [|x - 3| + |1 - x|] dx is equal to
The locus of the mid-point of AB is
(a) 4 (b) 8
(a) 3( x + y) = 2 xy (b) 2( x + y) = 3 xy (c) 12 (d) 16
(c) 2( x + y) = xy (d) x + y = 3 xy
74. If f ( x ) and g( x ) are twice differentiable
67. Let P and Q be the points on the parabola functions on (0, 3) satisfying

)
y2 = 4x, so that the line segment PQ subtends f ¢ ¢ ( x ) = g ¢ ¢ ( x ), f ¢ (1) = 4, g¢ (1) = 6 , f (2) = 3,

be
right angle at the vertex. If PQ intersects the g(2) = 9, then f (1) - g(1) is
axis of the parabola at R, then the distance of
(a) 4 (b) -4
the vertex from R is
(d) -2

Tu
(c) 0
(a) 1 (b) 2 (c) 4 (d) 6
75. Let [ x ] denote the greatest integer less than or
68. The incentre of an equilateral triangle is (1, 1) equal to x, then the value of the integral

ou
and the equation of one side is 3 x + 4 y + 3 = 0. 1
Then, the equation of the circumcircle of the ò-1(| x| - 2[ x])dx is equal to
triangle is (a) 3 (b) 2
(a) x2 + y2 - 2 x - 2 y - 2 = 0
(b) x2 + y2 - 2 x - 2 y - 14 = 0
(c) x2 + y2 - 2 x - 2 y + 2 = 0
(Y (c) -2 (d) -3

76. The points representing the complex number z


æ z -2ö p
on
for which arg ç ÷
ç z + 2÷ = 3
lie on
(d) x + y - 2 x - 2 y + 14 = 0
2 2
è ø
1 (a) a circle (b) a straight line
pi

( n !) n (c) an ellipse (d) a parabola


69. The value of lim is
n ®¥ n
m

77. Let a, b, c, p, q and r be positive real numbers


1 1 1
(a) 1 (b) (c) (d) such that a, b and c are in GP andq p = bq = cr .
e2 2e e
ha

Then,
70. The area of the region bounded by the curves (a) p, q , r are in GP (b) p, q , r are in AP
1 (d) p2 , q 2 , r 2 are in AP
y = x 3, y = , x = 2 is (c) p, q , r are in HP
C

x
1
(a) 4 - loge 2 (b) + loge 2 78. Let Sk be the sum of an infinite GP series
dy

4
15 whose first term is k and common ratio is
(c) 3 - loge 2 (d) - loge 2 k ¥ ( - 1 )k
4 ( k > 0). Then, the value of S is
k+1
u

k =1 S
k
71. Let y be the solution of the differential
equal to
St

dy y2
equation x = satisfying y(1) = 1.
dx 1 - y log x (a) loge 4 (b) loge 2 - 1
(c) 1 - loge 2 (d) 1 - loge 4
Then, y satisfies
-1 79. The quadratic equation
(a) y = xy (b) y = xy
(c) y = x y +1
(d) y = x y + 2 2x 2 - ( a 3 + 8a - 1)x + a 2 - 4a = 0 possesses
roots of opposite sign. Then,
72. The area of the region, bounded by the curves (a) a £ 0 (b) 0 < a < 4
y = sin -1 x + x(1 - x ) and (c) 4 £ a < 8 (d) a ³ 8
y = sin-1 x - x(1 - x) in the first quadrant, is 80. If log e( x 2 - 16) £ log e(4x -`11), then
1 1 1
(a) 1 (b) (c) (d) (a) 4 < x £ 5 (b) x < -4 or x > 4
2 3 4
(c) -1 £ x £ 5 (d) x < -1or x > 5
WB JEE (Engineering) l Solved Paper 2012 | 13
Answers
Physics
1. (d) 2. (a) 3. (c) 4. (a) 5. (a) 6. (a) 7. (d) 8. (c) 9. (d) 10. (b)
11. (a) 12. (d) 13. (b) 14. (b) 15. (d) 16. (c) 17. (a) 18. (b) 19. (c) 20. (c)
21. (b) 22. (b) 23. (a) 24. (c) 25. (d) 26. (b) 27. (a) 28. (c) 29. (d) 30. (c)
31. (b) 32. (d) 33. (d) 34. (b) 35. (b) 36. (c) 37. (d) 38. (a) 39. (d) 40. (b)

)
Chemistry

be
1. (a) 2. (d) 3. (a) 4. (c) 5. (c) 6. (d) 7. (c) 8. (a) 9. (b) 10. (a)
11. (c) 12. (d) 13. (c) 14. (b) 15. (a) 16. (d) 17. (b) 18. (c) 19. (a) 20. (a)
21. 22. 23. 24. 25. 26. 27. 28. 29. 30.

Tu
(c) (d) (c) (d) (a) (d) (b) (b) (a) (d)
31. (b) 32. (a) 33. (b) 34. (d) 35. (a) 36. (b) 37. (a) 38. (d) 39. (c) 40. (c)

Mathematics

ou
1. (a) 2. (c) 3. (a) 4. (c) 5. (a) 6. (b) 7. (b) 8. (a) 9. (d) 10. (b)
11. (d) 12. (d) 13. (b) 14. (a) 15. (a) 16. (d) 17. (a) 18. (c) 19. (a) 20. (d)

(Y
21. (d) 22. (d) 23. (d) 24. (d) 25. (c) 26. (d) 27. (a) 28. (d) 29. (b) 30. (c)
31. (c) 32. (d) 33. (b) 34. (c) 35. (d) 36. (a) 37. (d) 38. (b) 39. (b) 40. (c)
41. (b) 42. (d) 43. (b) 44. (b) 45. (c) 46. (c) 47. (d) 48. (b) 49. (a) 50. (b)
on
51. (d) 52. (b) 53. (c) 54. (b) 55. (d) 56. (b) 57. (d) 58. (d) 59. (b) 60. (c)
61. (c) 62. (b) 63. (a) 64. (b) 65. (c) 66. (b) 67. (c) 68. (b) 69. (d) 70. (b)
71. (b) 72. (c) 73. (c) 74. (b) 75. (a) 76. (a) 77. (c) 78. (d) 79. (b) 80. (c)
pi

Hints & Solutions


m
ha

Physics
x - 10 40 æ c 3 ´ 10 8 ö
1. = ç\ = = 2 ´ 10 8 ms -1 ÷÷
130 - 10 100 ç m
C

è .
15 ø
100 x - 1000 = 4800 t = 0.5 ´ 10 -9 s = 0. 5 ns
100 x = 5800 Þ x = 58°
dy

5. We know that,
2. Given, f = 4 t 2 + 6 t + 9 Wb and t = 2 s Y
df
u

We know that, e =
dt B (0, b)
St

df æ df ö
Here, = 8t + 6 Þ ç ÷ = 8 ´2 + 6
dt è dt ø t =2

e = 22 V X
O A (a, 0)
3. The velocity of water below, which the flow
remains streamline flow is known as critical Q
W = qDV and V =
velocity. 4pe0 r
4. Given, v = 3 ´ 10 8 ms -1, d = 10 m and m = 15
. æ Q Q ö
Hence, W = q çç - ÷÷
d 10 ´ 10 -2 è 4pe0 b 4pe0 a ø
Here, t = =
c 2 ´ 10 8 Qq æ a - b ö
m = ç ÷
4pe0 è ab ø
14 | WB JEE (Engineering) l Solved Paper 2012

2
6. From KCL, we have æV ö
(i) ç ÷ 2
6 kW 4 kW è 3ø = V
So, H¢ =
2R 18R
H
72 V 3 kW 2 kW H¢ =
18
12. Given, ZA : ZB = 1 : 2
72
i= = 9 ´ 10 -3 A We know that,

)
8 ´ 10 3

be
n µ Z2
i ´ 6 = (9 - i ) ´ 3
Hence,
i = 3 mA
n A (1)2

Tu
7. Given, E = (2 $i + 3$j + k$ )NC -1 =
n B (2 )2
and S = 10 $i m2 nA : nB = 1: 4

ou
We know that,
13. de-Broglie wavelength,
f = E×S h
l=
f = (2 $i + 3$j + k$ )× (10 $i )

(Y
mv
f = 20Nm 2C -1 Here,
-1 -1 h h
8. Given, v = 340ms , u s = 20ms and le = and l p =
on
c mpc
v0 = 640 Hz me
2
From Doppler’s law, Given, le = l p
pi

æ 340 ö h h
n = çç ÷÷ 640 So, =
è 340 - 20 ø c mpc
m

me
n = 680 Hz 2
me
=2
ha

9. Given, i = 10 A, B = 0.15 T, q = 45° and l = 2 m


mp
Here,
Ratio of KE,
F = ilB sin q = 10 ´ 2 ´ 0.15 sin 45°
C

1
3 me ve2
= N Ke 2 1
2 = =
1
dy

Kp mp vp2 2
10. Given, 2
æ pö s s
x1 = A sin ç wt + ÷ 14. E = +
u

è 6ø 2 e0 2 e0
St

æ pö s
x2 = A cos(wt ) = A cos t ç wt + ÷ E = , towards the negatively charged plane.
è 2ø e0
Phase difference, 15. Given, m = 2 kg, m = 0.2 and g = 10ms -2
Df = f2 - f1
Here, ma = m mg
p p 3p - p p
Df = - = = Þ a=m g
2 6 6 3
Þ a = 0. 2 ´ 10
V2
11. We know that, H = a = 2 m /s2
R
V 16. Dimension of angular momentum,
Here, in second condition V¢ becomes and R¢
3 L = r ´p
becomes 2R. = [L][MLT -1 ] = [ML2 T -1 ]
WB JEE (Engineering) l Solved Paper 2012 | 15
v2
17. Here, triangle is 23.
30º
q 30º
4 kg 1 kg
A v1
C
Along, Y-axis momentum remains zero.
Here,
B
4v1 sin 30 ° = v2 sin 60 °
æ| C| ö
÷÷ = cos -1 æç ö÷
3
q = cos -1 çç v1 3

)
è 5ø =
è| A| ø

be
v2 4
18. Given, x = 37 + 27t - t 3 24. Given, binary number = 1011001
dx

Tu
v= = 27 - 3 t 2 Its equivalent decimal number
dt
= 2° + 0 + 0 + 23 + 24 + 26
According to problem,
= 1 + 8 + 16 + 64 = 89

ou
v = 0 Þ 27 - 3 t 2 = 0
Here, t = 3s (2 n - 1)
25. Here, v1 = v
x = 37 + 27 ´ 3 - (3)2 = 91 m 4l1
1
19. At ground, E = mu 2
3
At highest point, E ¢ =
1
m (u cos 60 ° )2
(Y Hence,
v2 =
n
2l2
v1 = v2
v
on
2 l1 3
E =
E¢ = l2 4
4
pi

26. R increases with temperature and slope of V-I


20. Given, s = 30 cm = 30 ´ 10 -2 m and loss in graph gives resistance, so T1 < T2 .
m

velocity is 50%
27. (m + 1) vernier division = m main scale division
Equation of motion
ha

v 2 = u 2 + 2 as æ m ö
One division on vernier scale = çç ÷÷ division
u2 è m + 1ø
Here, = u 2 + 2 a ´ 30 ´ 10 -2 …(i)
on main scale
C

4
u2 æ m ö d
0= + 3a ´ x …(ii) Vernier constant = çç1 - ÷÷d = unit
4 è m + 1 ø m +1
dy

On solving Eqs. (i) and (ii), we get


28. Given, h = 80 m, v = 8ms -1 and g = 10 ms -2
x = 10 cm
1 2 1
u

21. For 1st situation, Here, h= gt Þ 80 = ´ 10 ´t 2


2 2
St

1
E= k(10 ´ 10 -2 )2 …(i) t = 4s
2
\ x = vt = 8 ´ 4 = 32 m
For 2nd situation
1 29. In balanced Wheatstone bridge,
E ¢ = K (20 ´ 10 -2 )2 …(ii) 1 1 1
2 = +
On comparing Eqs. i and ii, we get 10 x 30
E ¢ = 4E x = 15W
22. By Kepler’s law, 30. We know that ,
T 2 µ R3 V2
R=
3/ 2 P
æL ö
Hence, T = D çç 2 ÷÷ Given, V = 200 V, P = 50 W
è L1 ø
and V ¢ = 100V
16 | WB JEE (Engineering) l Solved Paper 2012

V 2 200 ´ 200 14 - N = 8
Hence, R= =
P 50 N = 6N
V ¢2 100 ´ 100 ´ 50
P¢ = = = 12.5 W 36. We know that,
R 200 ´ 200 F ´L 4L
DL = =
31. Given, work done = W and q = 60 ° A ´ Y pd 2
We know that, 4
Here, is a constant.
W = MB(1 - cos q) p

)
MB L

be
W = MB(1 - cos 60 ° ) = Hence, DL µ 2
2 d
Hence,| t| = MB sin 60 ° = 3W 37. Given, v1 = 3 x and v2 = 2( x + 5)

Tu
32. Capacitors in series, In Ist case,
2C1 ´ V1 = C ´ V2 1 1 1
+ = …(i)
- x -3 x f

ou
2V1 = V2 …(i)
1 1 1
and V1 + V2 + V1 = 60 …(ii) + = …(ii)
-( x + 5) -2( x + 5) f
Solving Eqs. (i) and (ii), we get
V2 = 30 V
33. Balancing equation, (Y On solving Eqs. (i) and (ii), we get f = 30 cm
38. Heat required to convert ice to water at 100°C,
Q = m ´ L + msDT = 18000 cal
on
W = mg - Vrg
Hence, 44 = m - 12.V Amount of heat left = 4320 cal
50 = m - V m ´ L = 4320
pi

On solving Eqs. (i) and (ii), we get m = 8 g steam


m = 80 g é 2p ù
m

39. y = A sin ê (vt - x )ú


34. In Ist case, ë l û
hn = hn 0 + eV0
ha

…(i) 2p
v= VA
hn eV l
= hn 0 + 0 …(ii)
2 4 (vp )max = 3v
C

Solving Eqs. (i) and (ii), we get 2p


l= A
n 3
V0 =
dy

3 40. We know that,


35. We know that, F = ma N = N0e -lt
u

F 14
a= = = 2ms -2 For A,
m 7 NA = N0e -5 lt
St

For B,
NB = N0e - lt
N 14 N N 1
Given, A = 2
NB e
N 1
=
NB e 4 lt
4 kg
1
Hence, from the figure t =
2l
14 - N = 4a
WB JEE (Engineering) l Solved Paper 2012 | 17

Chemistry
1. In the electrochemical series, metals are 8. Electrophiles are electron deficient species
arranged in increasing order of their standard (neutral or cationic). They attack on electron rich
reduction potential. The standard reduction C-atom.
potentials of Li + /Li and Cu 2+ /Cu are -3.05 V and
EtCl
+ 0.34 V respectively. Therefore, Li occupies 9. Me3CONa ¾¾¾® Me3COEt
higher position in series as compared to Cu. (1 °alkyl halide)
- NaCl

)
2. 11 Na
24
¾® 12 Mg
24
+ -1b
0

be
(Stable)
This reaction is known as Williamson’s synthesis.

3. B2 = 5 + 5 = 10e - 10. Elevation in boiling point,DTb = i ´ K b ´ m


n

Tu
* *
= s1s2 s 1s2 , s2 s2 s 2 s2 , p2 p1x , p2 p1y Molality of NaCl solution = ´ 1000
W
Due to the presence of 2 unpaired electrons, in 58.5
p-bonding orbitals, B2 shows paramagnetic 1000

ou
= 58.5 ´ 1000 =
behaviour. WH 2O WH 2O
4. Ammonium acetate is a salt of weak acid and Molality of C 6H12O 6 solution
weak base. When solutions of such salts diluted
resulting pH of solution is calculated from
following relationship
(Y 180
= 180
´ 1000
WH 2O
=
1000
WH 2O
on
1 1
pH = 7 + pK a - pK b Both the solutions have same molarity but the
2 2
values for NaCl and glucose are 2 and 1
\ pK a = pK b
pi

respectively.
pH of diluted solution = 7 \DTb (NaCl) = 2 ´ DTb(C 6 H12 O 6 )
m

5. The structure of 2-butene is as 11. Ratio of number of moles of CH4 and H2 is


H H x x
ha

s CH4 : H2 = : = 1: 8
s
Hs s s
s Cs C p C C sH
16 2
1 s 2 s 3 s 4 Hence, partial pressure exerted by
C

H H H H 8 8
(4 s-bonds H2 = ´ ptotal = ´ ptotal
sp3 hybridised)
(3s-bonds 1+ 8 9
sp2 hybridised)
dy

12. O H
6. COOH HOOC CH3 C CH3 CCl3
u

Acetone Chloroform
H C OH HO C H
St

Acetone and chloroform will show a negative


CH3 H3 C deviation due to their association after mixing.
(+) lactic acid (–) lactic acid 13. Equilibrium constant is independent of the
Both are optical isomers because they rotate the concentration of reactants.
plane of polarised light in opposite directions. 14. DG(T , p ) = DH - TDS determines the existence of
7. Me H a reaction.
C C has 6-hyper conjugate forms while
Me H 15. Molecularity of a reaction can never be fractional.
MeCH2 H It is always a whole number.
C C has 2-hyperconjugate forms.
H H 16. From Heisenberg uncertainty principle,
h
Therefore, Me2C == CH2 is more stable, than Dx × Dp ³ …(i)
4p
MeCH2CH == CH2.
18 | WB JEE (Engineering) l Solved Paper 2012

h 22. K 2Cr2O7 + H2SO 4 + 4H2O 2 ¾®


or Dx × mDv ³
4p
K 2SO 4 + 5H2O + 2CrO 5
h
or Dx × Dv ³ …(ii) 23. O Dark blue
4pm
This principle is also applicable for pairs like P
Cl
energy-time (DE × Dt ) and angular moment-angle Cl
Cl
(Dw × Dq) along with position-moment (Dx × Dp).
h 4 s-bonds Þ sp3 hybridisation without lone pair

)
Thus, DE × Dt ³ …(iii) of electrons.

be
4p
[O]
17. 82 Pb = 6s2 6 p2 24. CH3CH2CH2CH(OH)CH3 ¾®
= 6s2 6 p3

Tu
83 Bi CH3CH2CH2COCH3
2 O Keto methyl group
Due to inertness of 6s electrons in Pb and Bi,
they show bivalency and trivalency respectively ½½
CH3CH2CH2 C ¾ CH3 + 3I2 + 4NaOH ¾®

ou
instead of tetra and pentavalency.
O
18. In acidic medium,
½½

(Y
+6
K 2 C r2O7 ¾® Cr 3+ CH3CH2CH2 C ¾ ONa + CHI3
Iodoform
Equivalent weight of +3H2O + 3NaI
Molecular weight
on
K 2Cr2O7 = Al(OC2H 5 )3
Change in oxidation state of Cr 25. 2CH3CHO ¾¾¾¾¾® CH3COOCH2CH3
Acetaldehyde Tischenko reaction Ethyl acetate
M M
= =
pi

2(6 - 3) 6 26. CH3 CH3 CH3


CH3
Anhy. AlCl3
m

19. + CH3Cl +
Atomic Number of
Species Toluene o-xylene
ha

Number electrons CH3


Ca2 + 20 20 - 2 = 18
p-xylene
Cl - 17 17 + 1 = 18 Friedel-Craft’s reaction is an electrophilic
C

S2 - 16 16 + 2 = 18 substitution reaction and presence of an


electron releasing substituent like Me, (makes
For isoelectronic species, an increase in atomic
the benzene nucleus more reactive towards
dy

number, result in decreased size. Thus, order of


such reactions)
radius will be
Thus, toluene is most reactive among the given.
Ca 2+ < Cl - < S2-
u

p 27. Phenol is a weak acid. It reacts with NaOH to


¬¾
St

p produce salt.
20. C === O C 6H5OH + NaOH ¾® C 6H5ONa + H2O
s
Sodium
In CO molecule,C ¾ O s moment and phenoxide
O ¾ C p-moment cancels each other. So, that it But it is not sufficient acidic to evolve CO 2 from
is non-polar. NaHCO 3 solution.
21. O 28. NH2

P ··
H OH Acidic ; CH3NH2
Acidic H proton
Methyl amine
proton Aniline

There are two acidic protons in H3PO 3.


WB JEE (Engineering) l Solved Paper 2012 | 19

The lone pair of electrons, present in N-atom in 35. CH3 CH3 CH3
aniline involve in ring resonance. So, it is less HNO3 NO2
/H2SO4
basic than methyl amine. +
29. SN 1reaction is most effective in 3° carbon. Toluene o-nitro toluene
NO2
p-nitrotoluene
(CH3 )3 ¾ C ¾ Cl CH3CH2CH2CH2Cl (excess)
tert -butyl chloride(3 °) 1-chlorobutane(1 °)
Cl Cl

)
Cl

be
Cl ¾ CH2 ¾ CH(CH3 )¾ CH3 HNO3 NO2
2-methyl -1-chloropropane(1 °) /H2SO4
+
CH3CH2CH(Cl)CH3

Tu
o-nitro chloro
2-chloro butane (2 °) NO2
benzene
p-nitro chloro
30. O C O + (CH3 )3 ––C––MgI benzene
(Very low yield)

ou
Dry ice
C(CH3 )3
As toluene and chlorobenzene, both can react
HOH
O C––OMgI with mixture of conc. H2SO 4 and conc. HNO 3 ,

(CH3 )3 C––COOH + Mg
OH
I
(Y but being more reactive, toluene will yield
p-nitrotoluene in excess.
36. The order of stability of carbocation is 3° benzylic
on
31. CH3COOH + NaOH
> 2° benzylic > 2° > 1°.
Initial 20 ´ 0.1 = 2 mmol 10 ´ 0.1 = 1 mmol Thus, the correct order of stability will be
At time t( 2 - 1) = 1 mmol ( 1 - 1) = 0 mmol Ph H
pi

¾® CH3COONa + H2O ½ ½
From Henderson’s equation 1m mol Ph ¾ C ¾ CH2CH3 > PhCH2CH2 ¾ C ¾ Ph
m

½ ½
[CH3COONa] + +
pH = pK a + log
ha

(I) (II)
[CH3COOH]
1 H Me
= 474
. + log = 474 . ½ ½
1
> Ph 2CH ¾ C + > Ph ¾ C ¾ CH2
C

32. In Fe complex, NO behaves as NO + ½ ½


O H O Me Ph
33.
dy

(III) (IV)

CH3 C C C OC2H5
37. Water evaporates into steam. In this physical
u

H state its disorder or randomness increases.


O H O
St

63.5
CH3 C C C OC2H5 ´C ´t
ECt
38. m = = 2
H F 96500
34. CH3 CH2Cl . ´C ´t
3175
=
96500
Cl2 Cl2
(excess) (excess) 22
39. Second ionisation energy = 13.6 ´ = 54.4 eV
12
CHCl2 CCl3
NE V 1 63 ´ 1000 63
40. w = = ´ = g
Cl2 1000 20 1000 20
(excess)
20 | WB JEE (Engineering) l Solved Paper 2012

Mathematics
1. Since, (a + b ) and (a - b ) are the roots of (n + 1) n(n - 1) ù
+ + ....+ 1ú
the equation x + px + q = 0
2 3! û
1 n+1
\ Sum of roots = - p = [ C1 + n + 1C2 + K n + 1Cn + 1 ]
n+1
Þ (a + b ) + (a - b ) = - p 1
p = [ n + 1C0 + n + 1C1 + n + 1C2 + ...+ n + 1Cn + 1 - 1]
Þ 2a = - p Þ a = - n+1

)
2 [n + 1C0 = 1]

be
and product of roots = q 1
(a + b )(a - b ) = q = (2 n + 1 - 1)
n+1
a2 - b = q

Tu
¥ (r - 1)r ¥ 1
Þ 4. S = S
r = 2 2r ! r = 2 2(r - 2 )!
2
æ pö p2
b = a2 - q = ç- ÷ - q = -q 1é 1 1 1 ù 1

ou
è 2ø 4 = ê + + + K ¥ú = e
2 ë 0 ! 1! 2 ! û 2
Þ p2 - 4q = 4b
5. Given, P = éê ù
\The given equation 1 2 1
( p2 - 4 q ) ( p2 x 2 + 4 px ) - 16q = 0
4 b( p2 x 2 + 4 px ) - 16(a 2 - b ) = 0 (Y
\
ë1 3 1úû

Q = PPT = ê
é1 2 1ù ê
ú
é 1 1ù
ë1 3 1û ê 1 1ú
2 3ú
on
[a 2 - b = q ] ë û
b(4 a 2 x 2 - 8 ax ) - 4(a 2 - b ) = 0 é1 ´ 1 + 2 ´ 2 + 1 ´ 1 1 ´ 1 + 2 ´ 3 + 1 ´ 1ù

ë1 ´ 1 + 3 ´ 2 + 1 ´ 1 1 ´ 1 + 3 ´ 3 + 1 ´ 1úû
pi

a 2bx 2 - 2 abx + b = a 2
é1 + 4 + 1 1 + 6 + 1ù é 6 8 ù
Þ (ax b - b )2 = a 2 =ê =
ë1 + 6 + 1 1 + 9 + 1ûú ëê 8 11ûú
m

Þ ax b - b = ± a é6 8 ù
\The determinant of Q = ê = 66 - 64 = 2
ë 8 11úû
ha

1 1
\ x= ±
a b 6. Since, 1! = 1, 2 ! = 2, 3 ! = 6, 4 ! = 24, 5 ! = 120 Since, all
æ1 1 ö æ1 1 ö
C

terms from 5! onwards are divisible by 15 and


Þ x = çç + ÷ and ç -
÷ ç
÷
èa bø èa b ÷ø 1! + 2 ! + 3 ! + 4 ! = 33
\The required remainder after dividing by 15 will be 3.
dy

2. Given, log 2 ( x 2 + 2 x - 1) = 1
-1 cos R cos Q
Þ log 2 ( x 2 + 2 x - 1) = log 2 2 7. Let D = cos R -1 cos P
-1
u

Þ x2 + 2 x - 1 = 2 cos Q cos P
St

Þ x2 + 2 x - 3 = 0 Multiplying C1 by p and then doing


Þ x + 3x - x - 3 = 0
2 C1 ® C1 + q C2 + rC3 , we get
-p cos R cos Q
Þ x( x + 3) - 1( x + 3) = 0 D=
1
p cos R -1 cos P
Þ ( x + 3)( x - 1) = 0 p p cos Q cos P -1
Þ x = 1, - 3 - p + q cos R + r cos Q cos R cos Q
Since, x = 1 and x = - 3 are satisfy the given 1
= p cos R - q + r cos P -1 cos P
equation therefore the number of solutions of p p cos Q + q cos P - r cos P -1
the equation are two.
0 cos R cos Q
1 1 1 n 1
3. 1 + nC1 + nC2 + K Cn = 0 -1 cos P = 0
2 3 n+1 p 0 cos P -1
1 é (n + 1) n Using the projection formula
= ê (n + 1) +
n + 1ë 2! p = q cos R + r cos Q
WB JEE (Engineering) l Solved Paper 2012 | 21

8. The system of equations are 11. Since, these are 52 distinct cards in decks and
x + 3 y + 5 z = ax each distinct card is 2 in number. Therefore, 2
decks will also contain only 52 distinct cards two
5 x + y + 3 z = ay each.
3 x + 5 y + z = az \Probability that the player gets all distinct cards
Þ (1 - a) x + 3 y + 5 z = 0 …(i) 52
C26 ´ 2 26
= 104
5 x + (1 - a )y + 3 z = 0 …(ii) C26

)
3 x + 5 y + (1 - a )z = 0 …(iii)
12. Total number of selections of three balls in which

be
For infinite number of solutions, we must have balls of both colours are drawn
1- a 3 5
= 2 red balls and 1 white ball
5 1- a 3 =0

Tu
3 5 1- a +1red ball and 2 white balls
= 8C2 ´5 C1 + 8C1 ´5 C2 = 140 + 80 = 220
9- a 3 5
Þ 9 - a 1- a 3 = 0, \Required probability

ou
9- a 5 1- a 220 220 ´ 6 10
= 13 = =
C1 ® C1 + C2 + C3 C3 13 ´ 12 ´ 11 13
Taking (9 - a) common from the first column,
we get
1 3 5
(Y
13. Let F denotes fair coin, T denotes two headed, H
denotes head occurs
3 3 1
on
Þ (9 - a ) 1 1 - a 3 =0 \ P(F ) = , P(T ) = 1 - =
1 5 1- a 4 4 4
æ Hö
1 3 5 P ç ÷. P(T )
pi

Þ (9 - a ) 0 - a - 2 -2 =0 æT ö èT ø
\ Pç ÷ =
0 2 - a -4 è Hø æ Hö æ Hö
P ç ÷. P(T ) + P ç ÷. P(F )
m

a+2 2 èT ø èFø
Þ (9 - a ) =0
-2 a+4 (By Baye’s theorem)
ha

Þ (9 - a)(a 2 + 6 a + 8 + 4) = 0 1

Þ (a - 9)(a 2 + 6 a + 12 ) = 0 4 2
= =
1 1 3 5
C

Þ a = 9 is the only real value of a. 1× + ×


4 2 4
9. Let A = { a1, a2 , a3 , a4 } 14. According to question,
dy

B = { b1, b2 , b3 , b4 , b5 , b6 , b7} f (g ( x )) = g(f ( x ))


\n( A) = 4, n(B) = 7 Þ f ( x + 1) = g( x 2 + 2 x - 3)
\The total number of infections = 7P4
u

Þ ( x + 1)2 + 2( x + 1) - 3 = x 2 + 2 x - 3 + 1
7!
St

= = 7 × 6 × 5 × 4 = 840 Þ x + 1 + 2 x + 2 x + 2 - 3 = x2 + 2 x - 2
2
3!
5
Þ x 2 + 4x = x 2 + 2 x - 2
10. P = S C2 r Þ x 2 + 4x - x 2 - 2 x + 2 = 0
r =0
Þ 2x + 2 = 0Þ 2x = -2
210 Þ x = -1
= 10C0 + 10
C2 + ...... + C10 =
10
= 29
2
3 15. Since, a, b and c are in AP.
Q = S d 2 r + 1 = d1 + d 3 + d 5 + d 7 \ 2b = a + c
r =0

27 Given, quadratic equation,


= 7C1 + 7C3 + 7C5 + 7C7 = = 26 ax 2 - 2 bx + c = 0
2
P 29 Þ ax 2 - (a + c )x + c = 0 (2b = a + c )
\ = = 23 = 8 Þ ax 2 - ax - cx + c = 0
Q 26
22 | WB JEE (Engineering) l Solved Paper 2012

Þ ax( x - 1) - c ( x - 1) = 0 ba cos b - ab cos a


\ x= …(i)
Þ ( x - 1)(ax - c ) = 0 b-a
c ba sin b - ab sin a
Þ x = 1, y= …(ii)
a b-a
16. y 2 + 4 x + 4 y + k = 0 Divide Eq. (i) by Eq. (ii), we get
y + 2 ´2 y + 4 - 4 + 4 x + k = 0
2 x cos b - cos a
=
y sin b - sin a
( y + 2 )2 = - 4 x - k + 4

)
a+b a -b

be
æ 4+ kö 2 sin sin
( y + 2 )2 = - 4ç x - ÷ = 2 2
è 4 ø a+b a -b
-2 cos sin
\Latusrectum = 4 units 2 2

Tu
a+b a+b
17. Two circles are orthogonally if and only if \x cos + y sin =0
2 2
2(g1g 2 + f1f2 ) = c1 + c 2

ou
x2 y2 1 2 2
Þ 2 [(1 ´ 0 + k ) k ] = 6 + k 20. + = 1 Þ e = 1- =
9 1 9 3
Þ 2k 2 = 6 + k
Two foci are (± ae , 0 ) i.e., (± 2 2 , 0 )
Þ
Þ
Þ
2k 2 - k - 6 = 0
2 k 2 - 4k + 3k - 6 = 0
2 k(k - 2 ) + 3(k - 2 ) = 0
(Y Let P(h, k ) be any point on the ellipse
\
k-0
´
h -2 2 h + 2 2
k-0
= -1
on
Þ (k - 2 )(2 k + 3) = 0 [From given condition]
3 Þ h 2 - 8 = -k 2
Þ k = 2, -
pi

2 Þ x2 + y2 = 8
18. Since, join of (2, 0) and (0, 3) subtends 90° at (0, 0).
m

21. Given differential equation,


Þ It is a diameter, dy x + y +1
= …(i)
ha

dx 2 x + 2 y + 1
(0, 3)
Put x + y = v
dy dv dy dv
C

Þ 1+ = Þ = -1
dx dx dx dx
\Eq. (i) becomes,
dy

(0, 0) (2, 0) dy v+1 dv v +1


= Þ - 1=
dx 2 v + 1 dx 2v + 1
dv v +1
u

Equation is Þ = +1
dx 2 v + 1
St

( x - 2 )( x - 0 ) + ( y - 0 )( y - 3) = 0 dv v + 1 + 2 v + 1
x 2 + y 2 - 2 x - 3y = 0 Þ =
dx 2v + 1
(2 k, 3k ) lies on it dv 3v + 2 2v + 1
Þ = Þ dv = dx
Þ 4k 2 + 9k 2 - 4k - 9k = 0 dx 2 v + 1 3v + 2
Þ 13k 2 = 13k é2 1ù
êë 3 (3v + 2 ) - 3 úû
Þ k =1 Þ dv = dx
Since, k ¹ 0 otherwise (2 k, 3k ) will be (0, 0). 3v + 2
é2 1 1 ù
19. Since, AM : BM = b : a Þ ê - ´ ú dv = dx
ë 3 3 ( 3v + 2 )û
\M divides AB externally in the ratio b : a
WB JEE (Engineering) l Solved Paper 2012 | 23
1
Integrating both sides, we get ò 3 x log dx
æ2 1 1 ö \ IF = e e x

Þ ò ççè 3 - 3 ´ (3v + 2 )÷÷ødv = ò dx + C ¢ Put loge x = t Þ


1
dx = dt
x
Where, c is a constant of integration.
dt
2 1 log(3v + 2 ) = e 1/ 3 ò = e 1/ 3 log t
Þ v- = x + C¢ t
3 3 3 1/ 3
2 1 = e log t = t 1/ 3 = (loge x )1/ 3
Þ ( x + y ) - log(3 x + 3 y + 2 ) = x + C ¢

)
3 9 25. tan x + sec x = 2 cos x

be
Þ 6 x + 6 y - log(3 x + 3 y + 2 ) = 9 x + C sin x 1
Þ 3 x - 6 y + log(3 x + 3 y + 2 ) = C Þ + = 2 cos x
cos x cos x

Tu
p / 2æ 1 +sin 2 x + cos 2 x ö Þ sin x + 1 = 2 cos 2 x = 2(1 - sin 2 x ) = 2 - 2 sin 2 x
22. Let I = ò çç ÷÷ dx
p/6
è sin x + cos x ø Þ 2 sin 2 x + sin x - 1 = 0

ou
p / 2 æ 1 + 2 sin x cos x + 2 cos x - 1ö
2
= ò çç ÷ dx Þ 2 sin x + 2 sin x - sin x - 1 = 0
2

p/6 (sin x + cos x ) ÷


è ø Þ 2 sin x(sin x + 2 ) - 1(sin x + 1) = 0
p / 2 2 cos x(sin x + cos x )

(Y
=ò dx Þ (sin x + 2 )(2 sin x - 1) = 0
p/6 (sin x + cos x ) Þ sin x = - 2, which is not possible.

p/2
2 cos xdx = 2 [sin x ]pp//26 1 p 5p
or sin x = Þ x= ,
on
p/6
2 6 6
æ p pö æ 1ö 1 x Î[0, p ]
= 2 çsin - sin ÷ = 2 ç1 - ÷ = 2 ´ = 1
è 2 6ø è 2ø 2 \The number of solution = 2
pi

p
1 p / 4 sin x + cos x
23. Let I = ò 2 dx 26. Let I = ò dx
3 + sin 2 x
m

0 1 + (tan x )101 0

p p/ 4 sin x + cos x
= ò2
dx

p/2 dx =ò dx
ha

101
1 + (cot x )101
0 3 + 2 sin x cos x
0
ì æp öü
0
1 + ítan ç - x ÷ ý p/4 sin x + cos x
î è2 øþ =ò - dx …(i)
0 (sin x - cos x )2 - 4
C

p/ 2 tan x101
=ò dx Put sin x - cos x = t
0 tan x101 + 1
Þ (cos x + sin x )dx = dt
dy

1 + tan x101 - 1
p/2 p
=ò = [ x ]p0/ 2 - I Þ I = - I When x = 0 Þ t = - 1
0 1 + tan x101 2 p
and x = Þ t = 0
u

p p 4
Þ 2I = Þ I=
St

2 4 \Eq. (i) becomes,


dy 0
24. Given, 3 x loge x + y = 2 loge x 0 dt 1 é ½t - 2½ù
dx I = -ò 2 = - êlog½ ½ú
-1 t - 4 4 ë ½t + 2½û -1
Dividing both sides by 3x loge x, we get
dy 1 2 loge x 1 1 1
+ y= = - (log 1 - log 3) = - (0 - log 3) = log 3
dx 3 x loge x 3 x loge x 4 4 4
dy 1 2 æ 3 x - 1ö
Þ + y= 27. Given,y = çç ÷ sin x + loge (1 + x ), x > - 1
÷
è 3 + 1ø
x
dx 3 x loge x 3x
dy Differentiating w.r.t. x, we get
Which is linear form + Py = Q, where P and Q
dx dy d é æç 3 x - 1 ö÷ ù d
are function of x and the integrating factor is = êç x ÷ sin x ú + loge (1 + x )
dx dx êë è 3 + 1ø úû dx
given by the following formula e ò
Pdx
24 | WB JEE (Engineering) l Solved Paper 2012

æ 3 x - 1ö d æ 3 x - 1ö 2 2 2
ò-2 (1 + 2 sin x )e dx = ò e xdx + 2 ò sin xe| x|dx
d | x|
= çç x ÷ sin x + sin x ç ÷ 30.
÷ ç 3 x + 1÷ -2 -2
è 3 + 1ø dx dx è ø
(Since, first function is even and second
1 d
+ (1 + x ) function is odd)
1 + x dx 2
æ 3x - 1ö = 2 ò e xdx + 2, 0 = 2[e x ]20 = 2(e 2 - 1)
= çç x ÷ cos x + sin x 0
÷
è 3 + 1ø 31. We have the identity,
d d
(3 x + 1) (3 x - 1) - (3 x - 1) (3 x + 1) | z| = ½z + - ½ £ ½z + ½ + ½ ½
2 2 2 2

)
dx dx 1
+ ½ z z½ ½ z½ ½z½

be
(3 x + 1)2 1+ x
2
æ 3 x - 1ö Þ | z| £ 2 + Þ| z|2 £ 2| z| + 2
= çç x ÷ cos x + sin x(3 x + 1)(3 x loge 3)
÷ | z|

Tu
è 3 + 1ø
Þ | z|2 - 2| z| + 1 £ 3 Þ (| z| - 1)2 £ 3
-(3 x - 1)(3 x loge 3) 1
+ Þ - 3 £| z| - 1 £ 3
(3 x + 1)2 1+ x

ou
æ dy ö 1 Þ - 3 + 1 £| z| £ 3 + 1
\ ç ÷ =0+0+ =1
è dx ø at x =0 1+ 0 Hence, the maximum value = 3 + 1

28. f ( x ) =
x 2
+
8 x
1 2 x 2 - 16
(Y
32. Let z =
3
2
+i
2
3

æ 3ö
and r =
9 3
+ =
4 4
12
4
= 3
on
\ f ¢( x) = - 2 = ç ÷
8 x 8x 2 æ 1 ö p
q = tan -1 ç 2 ÷ = tan -1 ç ÷=
For maximum or minimum f ¢( x )must be vanished ç 3 ÷ è 3ø 6
ç 2 ÷
pi

x 2 - 16 è ø
\ f ¢( x) = 0 Þ =0
8x 2 3 i 3 ip
m

\ + = 3e
Þ x = 4, - 4 and x Î[1, 6] ¹ - 4 2 2 6
Also, in [1, 4], f ¢ ( x ) < 0 50
æ3 ö 50
ha

ç + i 3÷ æ ip ö
Þ f ( x ) is decreasing \ ç2 = ç 3e ÷
è 2 ÷ø è 6ø
In [4, 6], f ¢ ( x ) > 0 Þ f ( x ) is increasing
æ ip ö 50 i 50 p
C

1 2 17
f(1) = + = = ( 3) 50 ç
e6÷ = 325 e 6
8 1 8 ç ÷
6 2 3 1 13 è ø
f(8) = + = + =
dy

i 25 p
50
8 6 4 3 12 æ 3 i 3ö
Þ ç + ÷ = 325 e 3
17 ç2 2 ÷ø
Hence, maximum value of f ( x ) in [1, 6] is . è
u

8
æ 25p 25p ö
d ì -1 cos x ü = 325 çcos + i sin ÷
St

1 d cos x
29. ítan ý= è 3 3 ø
dx î 1 + sin x þ æ cos x ö dx 1 + sin x
2
1 + çç ÷÷ = 325 (cos 1500 + i sin 1500 )
è 1 + sin x ø = 325 [cos(360 ´ 4 + 60 ) + i sin(360 ´ 4 + 60 )]
(1 +`sin x )2
= = 325 (cos 60 + i sin 60 )
1 + sin x + 2 sin x + cos 2 x
2
50
(1 + sin x )(- sin x ) - cos(cos x ) æ3 3 ö÷ æ1 3 ö÷
Þ çç + i = 325 çç + i
(1 + sin x )2 è2 2 ÷ø è2 2 ÷ø
1 According to question,
= ´ [-(sin x + sin 2 x + cos 2 x )]
2 + 2 sin x æ3 ö
50
ç + i 3÷ = 325 ( x + iy )
=
1
´ -(1 + sin x ) = -
1 ç2 ÷
è 2 ø
2(1 + sin x ) 2
WB JEE (Engineering) l Solved Paper 2012 | 25

æ1 3 ö÷ = 9 ´ 10 ´ 10 ´ 10
325 çç + i ÷ = 2 ( x + iy )
25

è 2 2 ø \ The total number of vehicles with distinct


registration number
1 3
Which is true only when x = , y = = 26 ´ 26 ´ 9 ´ 10 ´ 10 ´ 10 = 262 ´ 9 ´ 10 3
2 2
z -1 36. These are 10 letters in the word IRRATIONAL in
33. Let w = which these are 2 I, 2 R and 2 A.
z +1
10 ! 10 !
\The total number of words = =

)
Then, using componendo and dividendo, we get
2 ! 2 ! 2 ! (2 !)3

be
1+ w ½w + 1½
z= Þ | z| = ½ ½
37. Four speakers will address the meeting in 4!
1- w ½w - 1½
ways = 24 different ways in which half number of

Tu
Put | z| = 1 cases will be such that P Speaks before Q and half
Þ | w + 1| = | w - 1| …(i) number of case will be such that P speaks afterQ
Let w = u + iv 24
\Required number of ways= = 12

ou
Then,| w + 1| = |u + iv + 1| = |(u + 1) + iv| 2
= (u + 1)2 + v 2 and| w - 1| = (u - 1)2 + v 2 38. The number of diagonals in a regular polygon of
From Eq. (i), we get
(u + 1)2 + v 2 = (u - 1)2 + v 2 (Y n sides is given by the following formula n C2 - n
\ The number of diagonals in a regular polygon
of 100 sides
on
Þ (u + 1)2 + v 2 = (u - 1)2 + v 2 100 × 99
= 100C2 - 100 = - 100
Þ u =0 2
z -1 9900 - 200 9700
pi

\w = is a pure imaginary number. = = = 4850


z +1 2 2
m

34. According to question, 39. According to question,


n
a + b + c + d + e + f + g + h = 100 …(i) C1, nC2 and n C3 are in AP.
ha

h =1 …(ii) 2 n(n - 1) n(n - 1)(n - 2 )


Þ =n+
a + b + d + e = 50 …(iii) 2! 3!
Þ n 2 - 9 n + 14 = 0
C

M P
Þ (n - 7 )(n - 2 ) = 0
a b c
Þ n = 7, since n ¹ 2
dy

d
e f \The sum of the coefficients of odd powers of x
in the expansion of (1 +`x )n is
u

g
2n 27
= = 2 6 = 64
St

B h 2 2

Venn diagram shows failed students 40. Given,f ( x ) = ax 2 + bx + c ,


b + c + d + f = 45 …(iv) g( x ) = px 2 + qx + r
d + e + f + g = 40 …(v) Since, f (1) = g(1)
b + e + f = 32 …(vi) Þ a+ b+c = p+q +r …(i)
On solving, the above equations, we get d = 2 f (2 ) = g(2 )
35. The total number of arrangements of 2 letters of Þ 4a + 2 b + c = 4 p + 2q + r …(ii)
English alphabet Subtracting Eq. (ii) from Eq. (i), we get
= 26 ´ 26 3a + b = 3 p + q …(iii)
The total number of arrangements of 4 digit f (3) - g(3) = 2
numbers in which first digit is not zero Þ (9 a + 3 b + c ) - (9 p + 3 q + r ) = 2
26 | WB JEE (Engineering) l Solved Paper 2012

-1/ 2
Þ 3(3 a + b) + c - 3(3 p + q ) - r = 2 æ2ö
3
æ 2ö
ç ÷ .... ¥ = ç1 - ÷ = 31/ 2 = 3
Þ c - r =2 …(iv) è ø
3 è 3ø
(3a + b = 3 p + q)
From Eq. (i) we get 44. Let a be the common roots
(a - p) + (b - q ) + (c - r ) = 0 \ a2 + a + a = 0 …(i)
Þ ( a - p) + ( b - q ) + 2 = 0 …(v) and a + aa + 1 = 0
2
…(ii)
From Eq. (ii), we get a2 a 1
= = [a ¹ 1]

)
4(a - p) + 2 )(b - q ) + c - r = 0 1 - a2 a - 1 a - 1

be
Þ 2(a - p) + (b - q ) + 1 = 0 …(vi) Eliminating a, we get
Subtracting Eq. (v) from Eq. (vi), we get (a - 1)2 = (1 - a2 )(a - 1)
( a - p) - 1 = 0

Tu
Þ (a - 1) = 1 - a2
a- p=1
\From Eq. (v), b - q = - 3 Þ a2 + a - 2 = 0
Þ a2 + 2 a - a - 2 = 0

ou
Now,
f (4) - g(4) = (16 a + 4b + c ) - (16 p + 4q + r ) Þ a(a + 2 ) - 1(a + 2 ) = 0
= 16 (a - p) + 4(b - q ) + (c - r ) …(vii)
Þ (a + 2 )(a - 1) = 0
Substituting the values of (a - p), (b - q ), (c - r )
from above in Eq. (vii), we get
f (4) - g(4) = 16 ´ 1 + 4(-3) + 2 = 16 - 12 + 2 = 6
(Y Þ a = -2
45. Let a be the first term and r be the common ratio
[Qa ¹ 1]
on
of a GP.
41. 1 ´ 1! + 2 ´ 2 ! + K + 50 ´ 50 ! \Pth, Qth and Rth terms of a GP are
= (2 - 11) ! + (3 - 1)2 ! + (4 - 1)3 ! + K + (51 - 1)50 ! respectively ar P - 1, ar Q - 1 and ar R - 1
pi

= (2 ! - 1!) + (3 ! - 2 !) + (4 ! - 3 !) + .. + (51! - 50 !) According to question,


= 51! - 1! = 51! - 1
ar P - 1 = 64
m

…(i)
42. Let the numbers be ar Q -1
= 27 …(ii)
a - 5d , a - 3d , a - d , a + d , a + 3d , a + 5d
ha

R -1
ar = 36 …(iii)
\ a - 5d + a - 3d + a - d + a + d
+ a + 3d + a + 5d = 3 (\Sum = 3) Dividing Eq, (i) by Eq (ii) we get
C

3
1 æ 4ö
Þ 6 a = 3Þ a = r P -Q = ç ÷ …(iv)
2 è 3ø
Also, given T1 = 4T3 , where T1, T3 are respectively,
dy

Dividing Eq. (ii) by Eq. (iii), we get


first and third terms of AP. 3
3 rQ - R =
Þ a -`5d = 4(a -`d ) Þ d = - 3 a = - 4
u

2 3
æ 3ö
\The fifth term r 3Q - 3 R = ç ÷
St

Þ …(v)
1 æ 3ö 1 9 è 4ø
a + 3d = + 3ç - ÷ = - = - 4
2 è 2ø 2 2 Multiplying Eq. (iv) and Eq. (v), we get
r P - Q ´ r 3Q - 3 R = 1
43. Given series,
1 1× 3 1× 3 × 5 1× 3 × 5 × 7 Þ r P - Q + 3Q - 3 R = 1
1+ + + + + K¥
3 3 × 6 3 × 6 × 9 3 × 6 × 9 × 12 Þ r P + 2Q - 3 R = r 0
1 3 1 3 5 Þ P + 2Q - 3R = 0 Þ P + 2Q = 3R
. 2 . . 3
1 2 2 æ2ö æ2ö
= 1+ + ç ÷ + 2 2 2 ç ÷ .... ¥ p
3 1× 2 è 3 ø 1× 2 × 3 è 3 ø 46. We know that|sin -1 x| £
2
1æ1 ö 1æ1 öæ 1 ö
ç + 1÷ 2 ç + 1÷ ç + 2 ÷ Hence, from the given relation we observe that
1 2 2 è2 ø æ2ö 2 è2 ø è2 ø p
= 1+ . + ç ÷ + each of sin -1 x, sin -1 y and sin -1 z will be
2 3 2! è 3ø 3! 2
WB JEE (Engineering) l Solved Paper 2012 | 27
p æ h -2 + k ö
So that x = y = z = sin =1 2 x + 3 y = 1, therefore the point ç , ÷
2 è3 3 ø
1
\ x 9 + y9 + z9 - 9 9 9
Must be satisfy the equation of the given line
x y z æ hö æ -2 + k ö 2 h - 6 + 3k
1 i.e, 2 ç ÷ + 3 ç ÷ =1Þ =1
= (1)9 + (1)9 + (1)9 - è 3ø è 3 ø 3
(1)9 (1)9 (1)9 Þ 2 h - 6 + 3k = 3 Þ 2 h + 3k = 9
1 Replace h = x and k = y, we get the required
= 1+ 1+ 1- = 3 -1= 2

)
1 ´1 ´1 locus of R .

be
47. We know that in D ABC i.e, 2 x + 3y = 9
a
=
b
=
c
= 2R px - 1 é0 ù
50. lim

Tu
sin A sin B sin C x ®0 1+ x - 1 êë 0 from úû
\ r 2 sin P sin Q = pq p x loge p
p q = lim = lim 2 1 + x (p x loge p )

ou
Þ r 2. . = pq, where R1 is circumradius of x ®0 1 x ®0
2 R1 2 R1 2 1+ x
DPQR.
= 2 1(p 0 loge p ) = 2 loge p =`loge p 2
Þ
Þ
Þ
r 2 = 4R12 Þ r = 2 R1
2 R1 sin R = 2 R1
R1 = 90 °
(Y
51. Given, f ( x )f ¢ ( x ) < 0
Þ f ( x ) and f ¢( x ) must be of opposite sign.
on
\DPQR is right angled. (i) Let f ( x ) = e - x
48. In DPQR, P + Q + R = 180 ° \ f ¢( x) = - e - x
pi

æ P - Q + Rö 180 ° - Q - Q Þ f ( x ) > 0 and f ¢ ( x ) < 0, " x Î R


\2 pr sin ç ÷ = 2 pr sin
è ø (ii) Let f ( x ) = - e - x
m

2 2
= 2 pr sin(90 ° - Q ) = 2 pr cos Q \ f ¢( x) = e - x
ha

æ 2ö
ç In DPQR cos Q = P + r -`q ÷
2 2
Þ f ( x ) < 0 and f ¢ ( x ) > 0 " x Î R
ç 2 pr ÷ But| f ( x )| = |± e - x| = e - x in both cases
è ø
æ p2 + r 2 - q 2 ö
C

d
= 2 pr çç ÷ = p2 + r 2 - q 2 \ | f ( x )| = |-e - x| < 0 in both case "x Î R
2 pr ÷ dx
è ø
Þ | f ( x )| must be a decreasing function.
dy

49. Let the vertices of R be (h, k )


52. If we take f ( x ) = 4 x 4 , then
P (2, –3)
(i) f ( x ) is continuous in (-2, 2 )
u

(ii) f ( x ) is differentiable in (-2, 2 )


St

(iii) f (-2 ) = f (2 )
So, f ( x ) = 4 x 4 satisfies all the conditions of
Rolle’s theorem therefore $ a point c such that
(–2, 1) Q R (h, k) f ¢ (c ) = 0
\Centroid of triangle is given by Þ 16c 3 = 0 Þ c = 0 Î (-2, 2 )
æ x1 + x2 + x3 y1 + y2 + y3 ö 53. Let y = e mx be the solution of given differential
ç , ÷
è 3 3 ø equation,
æ 2 + h - 2 -3 + 1 + k ö æ h -2 + k ö d 2y
i.e ç , ÷ Þç , ÷ Þ
dy
= me mx Þ 2 = m2e mx
è 3 3 ø è3 3 ø dx dx
æ h -2 + k ö
Since, the point ç , ÷ lies on the line
è3 3 ø
28 | WB JEE (Engineering) l Solved Paper 2012

d 2y dy \From Eq. (i), x = ± 2


\ 25 - 10 + y=0
dx 2 dx æ 1 ö æ 1 ö
\P ç 2 , ÷ and Q ç - 2 , - ÷
Þ 25m2e mx - 10 me mx + e mx = 0 è 2ø è 2ø
Þ e mx (25m2 - 10 m + 1) = 0 \Equation of circle with PQ as diameter is
Þ Auxiliary equation æ 1 öæ 1 ö
( x - 2 )( x + 2 ) + ç y - ÷ç y + ÷
Þ 25m2 - 10 m + 1 = 0 è 2 øè 2ø
1
Þ x2 - 2 + y2 - = 0

)
e mx ¹ 0
2

be
Þ (5m) - 2(5m) ´ 1 + 1 = 0
2
5
Þ x2 + y2 =
Þ (5m - 1)2 = 0 2

Tu
Þ m= ,
1 1 56. Let P( 10 cos q, 8 sin q) be the required point
5 5 x2 y2
on + =1
Since, roots are real and equal, . 10 8

ou
\General solution is y = (c1 + c 2 x )e x / 5 …(i) Whose distance from centre (0, 0) is 3 units.
y(0 ) = 1Þ c1 = 1 \10 cos 2 q + 8 sin 2 q = 9 = 9(sin 2 q + cos 2 q)

Þ
Þ
y(1) = 2e 1/ 5 Þ 2e 1/ 5 = (c1 + c 2 )e 1/ 5
c1 + c 2 = 2
c1 = 1
(Y Þ tan 2 q = 1 Þ q =
p
4
57. Let e be the eccentricity of hyperbola and length
on
Putting the value of c1 and c 2 in Eq. (i), we get of conjugate axis be 2b. Since, vertex (a, 0 )
particular solution bisects the join of centre (0, 0 ) and focus (ae , 0 )
ae + 0
pi

y = (1 + x )e x / 5 a= Þ e =2
2
54. Let the chord be MN.
m

b2 = a2 (e 2 - 1) = 3a2
Let other end of chord joining from vertex to it
x2 y2
lying on y 2 = 8 x be y2=4ax \Required hyperbola is 2 - 2 = 1
ha

2 N a b
N(2 t , 4t ) 2 (2 t , 4t) x2 y2
\Mid-point of Þ - = 1 Þ 3 x 2 - y 2 = 3 a2
P(t2, 2 t) a2 3 a2
C

(0, 0)
MN = (t 2 , 2 t )
M
\ x =`t 2 , y = 2 t
58. We know by the definition of hyperbola, ‘‘A
dy

hyperbola is the locus of a point which moves in


2
æ yö such a way that the difference between two fixed
Þ x = ç ÷ Þ y 2 = 4x
è2ø points remains constant,
u

is the required locus of P. i.e| PS - PS ¢| = 2 a


St

55. Solving x =`2 y 59. Given lines are, 3 x + 4 y = 9 …(i)


2y

y = mx + 1
=

and …(ii)
px

x2 Solving Eqs. (i) and (ii), we get


+ y2 = 1
4 5
x2 + y2=1
x=
3 + 4m
Q 4
Put x =`2 y in Eq. (ii), we If m = -1then x = - 5and m = - 2, then x = -1are
get the only possibilities.
(2 y )2 4y 2 x y
+ y2 = 1 Þ + y2 = 1 60. Let the line be AB = + = 1 …(i)
4 4 a b
1 \ The coordinates of A and B are respectively
Þ 2 y2 = 1 Þ y = ±
2 (a, 0 ) and (0, b)
WB JEE (Engineering) l Solved Paper 2012 | 29

64. According to question, A


1 1 1
(0, b) S = a. p = b. q = c . r
A 2 2 2
1 a 1 b
( , ) \ = , = , c
(a, 0) p 2S q 2S p b
O B 1 c
=
r 2S

)
1 1 1
\ + + B a C

be
Since (a , b ) is the mid-point ofAB p q r
a b 1 2t t
\ a = ,b = = (a + b + c ) = =
2 2 2S 2S S

Tu
\ a = 2 a and b = 2b 65. x 2 + y 2 = 4 …(i)
x y
From Eq. (i), + =1 ( x - 2) + y = 1
2 2
…(ii)

ou
2 a 2b
x y C1 = (0, 0 ), C2 = (2, 0 )
Þ + =2 On solving Eqs. (i) and (ii), we get
a b
61. The given series is in GP. Hence, its sum
S =
1{(1 + x )20 + 1 - 1} (1 + x )21 - 1
=
(Y P
on
C1 C2
(1 + x ) - 1 x
Therefore, the required coefficient of x10 in the Q
N
(1 + x )21 - 1
pi

expansion of
x
m

= Coefficient of x11 in the expansion of 7 15


(1 + x )21 - 1 = 21C11 x= ,y=±
4 4
ha

é1 -1 -2 6 ù 7 2 15 15
62. Augmented matrix [ A : B] = ê -1 1 1 mú C1N = , PQ = =
4 4 2
êl 1 1 3 úû
ë 1
C

. PQ. C1N
é 1 -1 -2 6 ù Area(DC1PQ ) 2
=
~ê 0 0 -1 m + 6ú Area(DC2 PQ ) 1 . PQ. C N
ê l + 1 0 -1 9 úû
dy

2
ë 2
Applying R2 ® R2 + R1, R3 ® R3 + R1 7 7
-1 -2 C1N
é 1 6 ù
u

= = 4 = 4 = 7 :1
~ê 0 0 -1 m + 6ú C2 N 2 - 7 1
St

êl + 1 0 0 3-m ú 4 4
ë û
Infinite number of solutions for l = - 1 and m = 3 Hence, the required option is (c).

æ B ö P(B Ç ( A È B ))
c 66. Given lines are, x + 2 y = 4 …(i)
63. P ç c ÷
=
è AÈB ø P( A È B )
C and 2x + y = 4 …(ii)
P( A Ç B) Solving Eqs. (i) and (ii), we get
= 4 4
P( A) + P(Bc ) - P( A Ç Bc ) x =` , y =
3 3
P( A) - P( A Ç BC )
= x y
P( A) + P(Bc ) - P( A Ç Bc ) Let AB : + = 1 …(iii)
a b
0.7 - 0.5 0.2 1
= = = Meets X-axis at A and Y - axis at B.
0.7 + 0.6 - 0.5 0.8 4 If the straight line (iii) pases through the
30 | WB JEE (Engineering) l Solved Paper 2012

æ 4 4ö 4 4 3(1) + 4(1) + 3
Point ç , ÷ , then + =1 = =2
è 3 3ø 3a 3b 32 + 42
1 1 3
Þ + = …(iv) AG = 2 GD = 4
a b 4
\Equation of circumcircle with centre at (1, 1) and
Let the mid-point ofAB be (h, k ) radius = 4 units
a+0 0+b
\ h= ,k = Þ a = 2 h, b = 2 k ( x - 1)2 + ( y - 1)2 = 42
2 2
Þ x 2 + y 2 - 2 x - 2 y - 14 = 0

)
1 1 3
\From Eq. (iv), + = Þ 2 h + 2 k = 3hk

be
1/ n
2h 2k 4 (n !)1/ n æ n !ö
69. lim = lim ç n ÷
Replace x, y from h, k respectively, we get x ®¥ n n ®¥ è n ø
required locus

Tu
n ! 1× 2 × 3 .... n
i.e 2( x + y ) = 3 xy We have, n =
n n × n × n .... n
67. According to question, it is given that slope of PX ì n !ü
1/ n
ì1 2 3 r nü
1/ n

ou
and QX are perpendicular to each other, i.e, \í ný = í . . .... K ý
în þ în n n n nþ
Slope of PX ´ slope of XQ = - 1 1/ n 1/ n
2 t 2m ì n !ü æ1 2 3 r nö
Þ lim í n ý = lim ç . . K K ÷

(Y
Þ ´ = - 1 Þ tm = - 4 n ®¥î n þ n ®¥ è n n n
t 2 m2 n nø
1/ n
x -t 2 y - 2t ì n !ü
PQ : 2 = Let A = lim í n ý
t - m2 2 t - 2 m n ®¥î n þ
on
1/ n
ì1 2 3 r nü
y2=4x Then, A = lim í . . . K K ý
n ®¥î n n n n nþ
pi

P(t2, 2t)
1 ærö 1
(0, 0) Þ log A = lim S log ç ÷ = ò log x dx
m

n ®¥ n è nø 0
X 90º R
1
é 1 ù
= ê x log x -`ò . x dx ú
ha

ë x û0
Q (m 2, 2m)
Integrating by parts
C

Let PQ meets axis of parabola i.e. X-axis at = [ x log x - x ]10 = - 1


R(a , 0 ), then 1
Þ A = e -1 =
a -t2 0 -2t
dy

e
= Þ a - t 2 = t 2 - tm
(t + m)(t - m) 2 (t - m) 70. First of all we draw the graph,
= - t 2 + 4 Þ a = 4Þ AR = 4 1
u

y = x 3, y = , x =2
x
St

68. Since, triangle is equilateral therefore incentre (1,


1) lies on the centroid of the DABC. y=1/x y=x3
\GD = Length of perpendicular from the point
G(1, 1) to the line 3 x + 4 y + 3 = 0
(1, 1)
A M
O
(0, 0) N P
x=2

G (1, 1)

D
B C

3x + 4y + 3 = 0 \Required area i.e (OMPNO )


WB JEE (Engineering) l Solved Paper 2012 | 31
1 3 5 5
1
= ò x 3 dx + ò
2
dx é x2 ù é x2 ù é x2 ù
0 1 x = ê 3x - ú +ê - 3x ú + ê - xú
1 ë 2 û1 ë 2 û3 ë 2 û1
é x4 ù 1
= ê ú + [loge x ]12 = + loge 2 æ 9ö æ 1ö æ 5 ´ 5 ö
4 4 = ç 3 ´ 3 -` ÷ - ç 3 ´ 1 - ÷ + ç - 3 ´ 5÷
ë û0 è 2ø è 2ø è 2 ø
71. Let y = x y æ 3 ´ 3 ö æ 5 ´ 5 ö æ 1 ö
-ç - 3 ´ 3÷ + ç - 5÷ - ç - 1÷
Þ log y = y log x è 2 ø è 2 ø è2 ø

)
æ 9ö æ 1 ö æ 25 ö æ9 ö
Differentiating both sides w.r.t x, we get = ç 9 - ÷ - ç 3 - ÷ + ç - 15÷ - ç - 9÷

be
1 dy 1 dy è 2ø è 2ø è 2 ø è2 ø
= y ´ + log x
y dx x dx æ 25 ö æ 1ö
+ ç - 5÷ - ç - ÷
è2 ø è 2ø

Tu
1 dy dy y
Þ - log x =
y dx dx x 9 5 5 9 15 1
= - - + + +
dy æ 1 ö y 2 2 2 2 2 2
Þ ç - log x ÷ =

ou
dx è y 9 - 5 - 5 + 9 + 15 + 1 24
ø x = = = 12
2 2
dy æ 1 - y log x ö y
Þ ç ÷=

(Y
dx è y ø x 74. According to question
f ¢ ¢( x) = g¢ ¢( x)
xdy y2
Þ = Integrating w.r.t. x, we get
dx (1 - y log x )
on
f ¢ ( x ) = g ¢ ( x ) + C1
72. sin -1 x is defined, if -1 £ x £ 1 Put x = 1Þ f ¢ (1) = g ¢ (1) + C1
In first quadrant 0 £ x £ 1 and x(1 - x ) ³ 0 Þ 4 = 6 + C1
pi

\ y = sin -1 x + x(1 - x ) …(i) \ C1 = - 2


Lies above y = sin -1 x - x(1 - x ) \ f ¢( x) = g¢( x) - 2
m

…(ii)
On solving, we get Again, integrating w.r.t. x, we get
ha

2 x(1 - x ) = 0 f ( x ) = g( x ) - 2 x + C2
Þ x = 0, 1 Þ 3 = 9 - 4 + C2
\Required area = ò (y1 - y2 )dx
1 Þ C2 = - 2
C

0
\ f ( x ) = g( x ) - 2 x - 2
1
= ò {sin -1 x + x(1 - x )} -{ sin -1 x - x(1 -`x )}]dx Put x = 1, we get
dy

0
1 f (1) - g(1) = - 2(1) - 2 = - 4
= 2 ò ( x - x 2 )dx 1
75. Let I = ò (| x| - 2[ x ])dx
0
u

3 ù1 -1
éx x 2
æ1 1ö 1
= 2ê - ú = 2ç - ÷= 0
= ò (| x| - 2[ x ])dx +
1
ò0 (| x| - 2[ x ])dx
St

ë 2 3 û0 è2 3ø 3 -1
0 1
5 = ò ( - x - 2 (-1))dx +`ò (x - 2(0 ))dx
73. ò1 [| x - 3| + |1 - x|]dx -1 0
0 1
5
= ò |x - 3|dx +`ò |x - 3|dx +
5 5 = ò ( - x + 2 )dx + ò0x dx
1 1 ò1 |1 - x|dx -1
0 1
3 5 5 æ x2 ö é x2 ù
= ò |x - 3|dx + ò3|x -`3|dx + ò1 |1 -`x|dx = çç - +`2 x ÷÷ + ê ú
1
3 5 5 è 2 ø -1 ë 2 û 0
=ò - ( x - 3)dx +`ò ( x - 3)dx + ò1 - (1 - x )dx
1 3 æ 1 ö 1 1 1
= - ç - + 2(-1)÷ + = + 2 + = 1 + 2 = 3
3 5
= ò (3 - x )dx + ò ( x - 3)dx + ò ( x - 1)dx
5
è 2 ø 2 2 2
1 3 1
32 | WB JEE (Engineering) l Solved Paper 2012

76. Let z = x + iy 1 1 1
78. S1 = 1 + + + K¥ = =2
z -2 x + iy - 2 2 22 1 -`
1
\ = 2
z + 2 x + iy + 2
2
( x - 2 ) + iy ( x + 2 ) - iy 2 æ2ö 2
= ´ S 2 = 2 + 2. + 2 ç ÷ + .... ¥ = =6
( x + 2 ) + iy ( x + 2 ) - iy 3 è 3ø 1-
2
3
( x - 2 )( x + 2 ) + iy( x + 2 ) - iy( x - 2 ) - i 2 y 2
= æ 3ö æ 3ö
2
3
( x + 2 )2 - (iy )2 S 3 = 3 + 3ç ÷ + 3ç ÷ + K ¥ = = 12

)
è 4ø è 4ø 1-
3

be
x 2 - 4 + ixy + 2 iy - ixy + 2 iy + y 2 4
=
( x + 2 )2 + y 2 æ 4ö æ 4ö
2
4
S4 = 4 + 4 ç ÷ + 4 ç ÷ + K ¥ = = 20

Tu
( x + y - 4) + 4iy
2 2
è ø
5 è ø
5 4
= 1-
( x + 2 )2 + y 2 5
¥ (-1)k 1 1 1 1
x2 + y2 - 4 4iy \ S =- + - + - .... ¥

ou
= + 2 k =`1 S k S1 S 2 S 3 S 4
( x + 4 + 4x + y ) x + 4 + 4x + y 2
2 2

1 1 1 1
æ z -2 ö 4y =- + - + -K ¥
\ arg çç ÷÷ = tan -1 2

(Y
2 6 12 20
è z + 2ø x + 4 + 4x + y 2 1 1 1 1
=- + - + - .... ¥
x 2 + 4 + 4x + y 2 p 1× 2 2 × 3 3 × 4 4 × 5
´ = (given)
x2 + y2 - 4 æ 1ö æ 1 1ö æ 1 1ö æ 1 1ö
on
3
= - ç1 - ÷ + ç - ÷ - ç - ÷ + ç - ÷ - K ¥
p 4y è 2 ø è 2 3ø è 3 4ø è 4 5ø
\ tan = 2
3 x + y2 - 4 æ1 1 1 1 ö
= - 1 + 2ç - + - + K ¥ ÷
pi

4y è 2 3 4 5 ø
3= 2
x + y2 - 4 æ 1 1 1 1 ö
m

= - 1 - 2 ç1 - + - + - .... ¥ ÷ + 2 ¥
4 è 2 3 4 5 ø
x2 + y2 - 4 - y = 0, which represents
ha

3 = - 1 - 2 loge 2 + 2 = 1 - loge 4
a circle. 79. Since, roots are opposite sign.
77. Let ap = bq = c r = k \Product of the roots < 0
C

\ a=k 1/ p
,b=k 1/ q
,c = k 1/ r Þ a2 - 4 a < 0
Þ a(a - 4) < 0
dy

Since, a, b, c are in GP.


b c Þ 0<a<4
=
a b 80. Given, loge ( x 2 - 16) £ loge (4 x - 11) if and only if
u

k1/ q k1/ r x 2 - 16 £ 4 x - 11
= 1/ q
St

1/ p
k k
Þ x 2 - 4x - 5 £ 0
k1/ q - 1/ p = k1/ r - 1/ q
Þ x 2 - 5x + x - 5 £ 0
1 1 1 1
- = - Þ x( x - 5) + 1( x - 5) £ 0
q p r q
1 1 1 Þ ( x - 5)( x + 1) £ 0
\ , , are in GP.
p q r Sign scheme of x 2 - 4 x - 5 £ 0
+ – +
Þ p, q, r are in HP.
–1 5
Þ -1 £ x £ 5
St
udy
C
ha
m
pi
on
(Y
ou
Tu
be
)
St
udy
C
ha
m
pi
on
(Y
ou
Tu
be
)
WB JEE
Engineering Entrance Exam

Practice Set 1

)
be
Tu
Physics

ou
Category I (Q. Nos. 1 to 30) 3. When the momentum of a proton is changed

(Y
Carry 1 marks each and only one option is by an amount p 0 , the corresponding change
correct. In case of incorrect answer or any in the de-Broglie wavelength is found to be
combination of more than one answer, 1/4 mark 0.25%. Then, the original momentum of the
proton was
on
will be deducted.
(a) p0 (b) 100 p0
1. A ray of light is incident on a transparent (c) 400 p0 (d) 4 p0
pi

glass slab of refractive index 1.62. If the 4. Difference between nth and (n + 1)th Bohr’s
reflected and refracted rays are mutually radius H-atom is equal to its (n − 1)th Bohr’s
m

perpendicular, then what is the angle of radius. The value of n is


incidence?
(a) 1 (b) 2 (c) 3 (d) 4
ha

5. The circuit has two oppositely connected


Medium 1 ideal diodes in parallel. What is the current
r
C

flowing in the circuit?


4Ω
90°
dy

D1 D2
r′
Medium 2 12V
u

2Ω 2Ω
St

(µ1 > µ2)


(a) 58.3° (b) 85.3°
(c) 60° (d) 65° (a) 1.71 A (b) 2 A
(c) 2.31 A (d) 1.33 A
2. For a substance, the average life for
α-emission is 1620 yr and for β-emission is 6. A truth table is given below. Which of the
1 following has this type of truth table?
405 yr. After how much time, the th of the
4 A B Y
material remains after α and β-emission?
0 0 1
(a) 1500 yr
1 0 0
(b) 300 yr
(c) 449 yr 0 1 0
(d) 810 yr 1 1 0
4 WB JEE (Engineering) Practice Set 1
(a) NOR gate (b) OR gate 12. The planet neptune travels around the sun
(c) AND gate (d) NAND gate with a period of 165 yr. What is the radius of
7. The correct dimensional formula for pressure orbit approximately, if the orbit is considered
is given by as circular?
(a) [ML−1T−1 ] (b) [ML−1T−2 ] (a) 20 R1 (b) 30 R1 (c) 25 R1 (d) 35 R1
(c) [ML2 T−2 ] (d) [MLT−2 ] 13. A steel wire of length 4m and diameter 5 mm
is stretched by 5 kg-wt. The increase in its
8. The velocity of transverse wave in a string is length, if the Young’s modulus of steel wire

)
T is 2.4 × 10 12 dyne cm −2 is

be
v= , where T is the tension in the string
M (a) 0.003 m (b) 0.0041 cm
and M is mass per unit length. If T = 3.0 kgf, (c) 0.00041 cm (d) 0.005 cm

Tu
mass of string is 2.5 g and length of string is
14. The terminal velocity of a copper ball of radius
1.00 m, then the percentage error in the 2 mm falling through a tank of oil at 20°C is
measurement of velocity is
6.5 cms −1 . The viscosity of the oil at 20°C is

ou
(a) 0.5 (b) 0.7 (c) 2.3 (d) 3.6
[Take, density of oil = 15. × 10 3 kgm −3 ,
9. Three equal weight A, B and C of mass 2 kg density of copper = 8 .9 × 10 3 kgm −3 ]

(Y
each are hanging on a string passing over a (a) 3.3 × 10−1 kg m −1s −1 (b) 6.3 × 10−2 kg m −1s −1
fixed frictionless pulley as shown in the −3 −1 −1
. × 10
(c) 92 kg m s (d) 9.9 × 10−1 kg m −1s −1
figure. The tension in the string connecting
on
weight B and C is
15. A black body at 227°C radiates heat at a rate
of 7 cal/cm 2 s. At a temperature of 727°C, the
rate of heat radiated in the same units will be
pi

(a) 112 (b) 105 (c) 101 (d) 89


m

16. A flask contains argon and chlorine in the


A ratio of 2 : 1 by mass. The temperature of the
B
ha

mixture is 27°C. Root mean square speed vrms


of the molecules of the two gases is
C [Take, atomic mass of argon = 39.9 u,
C

(a) zero (b) 13 N (c) 3.3 N (d) 19.6 N


molecular mass of chlorine = 70 .9 u]
(a) 1.33 (b) 3.3
dy

10. The displacement of a body along X -axis (c) 2.2 (d) 0.22
depends on time as x = t + 1, then the
17. Two moles of an ideal gas is contained in a
velocity of a body
u

cylinder fitted with a frictionless movable


(a) increase with time (b) decrease with time piston. Exposed to the atmosphere, at an
St

(c) independent of time (d) None of these initial temperature T0 . The gas is slowly
11. A heavy particle is projected from a point at heated, so that its volume becomes four
the foot of a flying plane, inclined at an angle times the initial value.
45° to the horizontal, in the vertical plane (a) 2 RT0 (b) 4RT0 (c) 6RT0 (d) zero
containing the line of greatest slope through 18. The diameter of each plate of an air capacitor
the point. If θ(> 45 °) is the inclination with is 4 cm. To make the capacity of this parallel
the horizontal of the initial direction of plate capacitor equal to that of a sphere of
projection, for what value of tan φ will the diameter 20 cm. The distance between the
particle strike the plane? plates will be
(a) tanθ = 1 (b) tanθ = 2 (a) 1 × 10−3 cm (b) 1 cm
(c) sinθ =
1
(d) sinθ =
1 (c) 4 × 10−3 m (d) 1 × 10−3 m
2 2
WB JEE (Engineering) Practice Set 1 5

19. A copper ball 1 cm in diameter is immersed 23. A proton, a deuteron and an α-particle
in oil with a density 800 kg m −3 . What is the moving with same kinetic energy enter a
charge of the ball, if in a homogeneous region of uniform magnetic field moving at
electric field, it is suspended in oil? The right angles to the direction of field. Ratio of
electric field is directed vertically upwards radii of circular paths travelled by these
and its intensity E = 3600 V cm −1 . The particle is
density of copper is 8600 kg m −3 (a) 2 : 2 : 1 (b) 1 : 2 : 1
. × 10−7 C
(a) 110 . × 10−8 C
(b) 110 (c) 2 : 1 : 1 (d) 1 : 2 : 2
. × 10−7 C . × 10−8 C

)
(c) 111 (d) 111

be
24. A vertical wire carries a current in upward
20. Two charges of magnitude ‘Q’ are located at direction. An electron beam sent horizontally
distance ‘r’ as shown in figure. A third charge towards the wire will be deflected

Tu
q is placed on the line joining the above two (a) upwards (b) downwards
charges such that all the three charges are in (c) towards left (d) towards right
equilibrium.
25. The supply voltage to room is 120 V. The

ou
Q Q
A r B resistance of the lead wires is 6 Ω. A 60 W
bulb is already switched ON. What is the
What is the magnitude, sign and position of

(Y
decrease of voltage across the bulb, when a
the charge?
240 W heater is switched ON in parallel to
(a) Q / 4 and it is located exactly midway A and B the bulb?
(b) − Q / 4 and it is located exactly midway A and B
on
(a) 2.9 V (b) 13.3 V (c) zero (d) 10.04 V
(c) Q / 2 and it is located exactly midway A and B
(d) None of the above 26. In the Wheatstone’s network figure P = 10 Ω,
Q = 20 Ω, R = 15Ω, S = 30 Ω. The current
pi

21. The magnetic moment of an electron orbiting passing through battery of negligible
in a circular orbits of radius r with a speed v resistance is
m

is equal to B
(a) evr / 2 (b) evr R
ha

(c) er / 2 v (d) None of these P


A C
22. The magnetic field at the centre of the G
circular loop as shown in figure, when a
C

single wire is bent to form a circular loop and Q


S
also extends to form straight section is
dy

D
u

R (a) 0.36 A (b) 0 A (c) 0.18 A (d) 0.72 A


St

27. A coil of inductance 300 mH and resistance


2Ω is connected to a source of voltage 2 V.
B The current reaches half of its steady state
90° C
I value in
A (a) 0.3 s (b) 0.1 s (c) 0.15 s (d) 0.05 s
28. In Young’s double slit experiment, intensity
µ 0I µ 0I  1 
(a) (b) 1 +  at a point is (1/4) of the maximum intensity,
2R 2R  π 2 angular position of this point is
µ 0I  1  µ 0I  1  (a) sin−1(λ / 4d ) (b) sin−1(λ / 2d )
(c) 1 −  (d) 1 − 
2R  π 2 R  π 2
 λ  λ
(c) sin−1   (d) sin−1  
 3d  d 
6 WB JEE (Engineering) Practice Set 1

29. An object is placed at 10 cm in front of a 34. A plane electromagnetic wave of frequency


concave mirror of radius of curvature 15 cm. 25 MHz travels in free space along the
Which one of the following statement x-direction. At a particular point in space and
regarding the nature of image is true? time, E = 6.3 $j Vm −1 , the value of magnetic
(a) Image is small, imaginary and inverted field B at that point is (in tesla)
$
(a) 6.3 kT $
(b) 6.3 × 10–8 kT
(b) Image is magnified, real and inverted
(c) Image is magnified, false and straight $
(c) 2.1 × 10–8 kT $
(d) 2.1 kT
(d) Image is small, imaginary and straight
35. A sinusoidal voltage of peak value 283 V and

)
30. In a single slit diffraction experiment, first

be
frequency 50 Hz is applied to a series L-C-R
minima for λ 1 = 660 nm coincides with first circuit in which R = 3 Ω , L = 25.48 mH and
maxima for wavelength λ 2 , then λ 2 will be C = 796 µF. Then, the phase difference between
equal to

Tu
the voltage across the source and the current is
(a) 240 nm (b) 345 nm (a) 53.1° (b) 42.4° (c) 28.3° (d) 0°
(c) 440 nm (d) 330 nm

ou
Category III (Q. Nos. 36 to 40)
Category II (Q. Nos. 31 to 35)
Carry 2 marks each and one or more option(s)
Carry 2 marks each and only one option is is/are correct. If all correct answers are not marked
correct. In case of incorrect answer or any
combination of more than one answer, 1/2 mark
will be deducted.
(Y
and also no incorrect answer is marked then score
= 2 × number of correct answers marked ÷ actual
number of correct answers. If any wrong option is
on
31. The equivalent inductance of two inductors is marked or if any combination including a wrong
2.4 H when connected in parallel and 10 H option is marked, the answer will be considered
pi

when connected in series. What is the value wrong, but there is no negative marking for the
of inductances of the individual inductors? same and zero marks will be awarded.
m

(a) 2 H, 8 H (b) 4 H, 6 H
36. The maximum KE (E k) of the emitted
(c) 3 H, 7 H (d) 5 H, 5 H
photoelectrons against frequency of the
ha

32. Two syringes of different cross-sections incident radiation is plotted as shown in


(without needles) filled with water are figure. This graph help in determining the
connected with a tightly fitted rubber tube following quantities.
C

filled with water. Diameters of the smaller Ek


piston and longer piston are 1 cm and 3 cm
dy

respectively. If the smaller piston is pushed


in through 6 cm, how much does the longer ν
piston move out?
u

(a) 0.37 cm (b) 0.67 cm


St

(c) 37 cm (d) 67 cm
(a) Planck’s constant
33. In an experiment on the specific heat of a (b) Charge on an electron
metal, a 0.20 kg block of the metal at 150°C is (c) Threshold frequency
dropped in a copper calorimeter (of water (d) Work function of cathode metal
equivalent 0.025 kg) containing 150 cc of
water at 27°C. The final temperature is 40ºC. 37. Consider a cycle followed by an engine as
Calculate the specific heat of the metal. If shown in figure
heat losses to the surroundings are not 1 to 2 is isothermal
negligible, is our answer greater or smaller 2 to 3 is adiabatic
than the actual value of specific heat of the 3 to 1 is adiabatic
metal?
Such a process does not exist because
(a) 0.02 (b) 0.2
(c) 0.01 (d) 0.1
WB JEE (Engineering) Practice Set 1 7

p (a) In order to charge the battery, the output


1 voltage V of the generator must be greater than E
2 (b) In order to charge the battery, the output
voltage V of the generator must be at least twice
of E
3 (c) The charging current i through the circuit is
V−E
V given by i =
(R + r )
(a) heat is completely converted to mechanical
(d) The charging current i through the circuit is

)
energy in such a process, which is not possible.
V

be
(b) mechanical energy is completely converted to given by i =
(R + r )
heat in this process, which is not possible.
(c) curves representing two adiabatic processes
40. Magnetic field intensity B at the centre of the

Tu
don’t intersect.
circular loop is
(d) curves representing an adiabatic process and
an isothermal process don’t intersect. i2

ou
38. A projectile has the same range R for two
angles of projections. If T1 and T2 be the times
of flight in the two cases, then (using θ as the
angle of projection corresponding to T1 )
(a) T1 T2 ∝ R
(c) T1 /T2 = tanθ
(b) T1 T2 ∝ R
(d) T1 /T2 = 1
2
(Y i
O
R

i
on
i1
39. A battery of emf E and internal resistance r is
connected with an external voltage source (a) zero
pi

(generator) through a resistance R as shown µ (2 π − θ )i


(b) 0
in figure. Choose the correct statements. 4 πR
m

µ iθ
(c) 0
+ – 4 πR
ha

E r µ 0i 2( π − θ )
(d)
4 πR
C

V
R + –
dy

Chemistry
u

Category I (Q. Nos. 41 to 70) 42. Among the following, the least stable
St

Carry 1 marks each and only one option is resonance structure is


correct. In case of incorrect answer or any r
r O s
r O
combination of more than one answer, 1/4 mark (a) s N (b) r N
will be deducted.
O O
s s
41. In the following structure, which is the better s
site of protonation? s
r O r O
(c) r N (d) N
O N–H
O O
s s
(a) Nitrogen (b) Oxygen
(c) Double bond (d) All of the above
8 WB JEE (Engineering) Practice Set 1

43. Correct order of nucleophilicity is 49. The equilibrium constant for the reaction,
(a) CH−3 < NH−2 < OH− < F − H 2 (g) + S(s)
3 H 2S(g) is 18.5 at 925 K and
(b) F − < OH− < CH−3 < NH−2 9.25 at 1000 K respectively. The enthalpy of
(c) OH− < NH−2 < F − < CH−3 the reaction is
(d) F − < OH− < NH−2 < CH−3 (a) 2 kJ mol −1 (b) + 71kJ mol −1
−1
h (c) − 71kJ mol (d) 75 kJ mol −1
44. If is written as h direc-h, the orbital

angular momentum of a d-subshell will be 50. NaOH is a strong base. What will be pH of

)
5.0 × 10 −2 M NaOH solution? (log 2 = 0 .3)

be
h h
(a) (b) (a) 13.70 (b) 12.70 (c) 14.00 (d) 13.00
6 2
(c) 6 h (d) 2 6 h 51.

Tu
O
45. In which case racemic mixture is obtained on
mixing its mirror image is 1 : 1 molar ratio? H2/Ni
X=?

ou
(a) [Ni(dmg)2 ] (b) [Cr(en)3 ]3+
(c) cis − [Cu(gly)2 ] (d) All of these OH O

(Y
46. In the chemical reactions
NH2 (a) (b)

NaNO2 CuCN OH
on
A B
HCl, 278K ∆
(c) (d) None of these
Compound A and B respectively are
pi

(a) fluorobenzene and phenol


(b) benzene diazonium chloride and benzonitrile 52. For the process,
m

(c) phenol and bromobenzene l → H 2O ( g)


H 2O()
(d) nitrobenzene and chlorobenzene (1 bar, 273 K) (1 bar, 373 K),
ha

The correct set of thermodynamic


47. The term anomers of glucose refers to parameters is
(a) isomers of glucose that differ in configurations (a) ∆G = 0, ∆S = + ve
C

at carbons one and four (C − 1 and C − 4) (b) ∆G = 0, ∆S = − ve


(b) a mixture of (D) - glucose and (L) - glucose (c) ∆G = + ve, ∆S = 0
(c) enantiomers of glucose (d) ∆G = − ve , ∆S = + ve
dy

(d) isomers of glucose that differ in configuration at


carbon one (C − 1) 53. Which has the smallest bond angle
(X —S—X ) in the given molecules?
u

48. A compound liberates CO 2 with NaHCO 3 and (a) OSBr2 (b) OSCl 2
St

also gives colour with neutral FeCl 3 solution. (c) OSF2 (d) OSl 2
The compound can be
54. For an isomerisation reaction A 3B, the
OH OH
temperature dependence of equilibrium
COOCH3
constant is given by
(a) (b)
2000
log e K = 4 .0 −
OH T
CH—COOH CH2—COOH The value of ∆S° at 300 K is therefore,
(c) OH (d) (a) 4R (b) 5 R
(c) 400 R (d) 2000 R
WB JEE (Engineering) Practice Set 1 9

55. In van der Waals’ equation of state for a 63. Which of the following will produce
non-ideal gas, the term that accounts for isopropyl amine?
NH 2OH LiAlH 4
intermolecular force is I. (CH 3)2 CO   → X    → ?
(a) (V − b ) (b) (RT )−1 NH 3 LiAlH 4
II. CH 3  CH 2  CHO  → X    → ?
(c)  p + 2 
a Heat
(d) RT
 V  NH 3
III. (CH 3)2 CH OH + PCl 5 → X  → ?
56. Which one of the following electronic IV. CH 3  CH 2  CH 2 NH 2   → ?
Heat

)
arrangements is absurd?
(a) I, II (b) I, III (c) II, III (d) III only

be
(a) n = 3, l = 2, m = − 3 (b) n = 4, l = 3, m = 3
(c) n = 4, l = 3, m = 2 (d) n = 2, l = 1, m = 0 64. How many coulombs of charge is required to
convert 12.3 gm of nitrobenzene to aniline?

Tu
57. Maximum freezing point will be for 1 molal (a) 57900 C (b) 59700 C
solution of assuming equal ionisation in each (c) 75900 C (d) 95700 C
care

ou
(a) [Fe(H2O)6 ]Cl 3 65. If excess of AgNO 3 solution is added to 100
(b) [Fe(H2O)5 Cl]Cl 2 ⋅ H2O ml of a 0.024 M solution of dichloro bis
(c) [Fe(H2O)4 Cl 2 ]Cl ⋅ 2H2O (ethylenediamine) cobalt (III) chloride, how

(Y
(d) [Fe(H2O)3 Cl 3 ]⋅ 3H2O many moles of AgCl be precipitated?
(a) 0.024 mol (b) 0.012 mol
58. The correct order of increasing ionic (c) 0.016 mol (d) 0.0048 mol
character is
on
(a) BaCl 2 < CaCl 2 < MgCl 2 < BeCl 2 66. FeCr2O 4 
I→ Na 2CrO 4 
II
→ Cr2O 3 
III
→ Cr
(b) BeCl 2 < MgCl 2 < CaCl 2 < BaCl 2 I, II and III are
(c) BeCl 2 < BaCl 2 < MgCl 2 < CaCl 2
pi

I II III
(d) BaCl 2 < CaCl 2 < MgCl 2 < BeCl 2
(a) Na 2CO 3 / air H+ / NH4Cl Al
m

59. The correct order of electron affinity of the (b) NaOH / air C C
elements of oxygen family in the periodic (c) Na 2CO 3 / air C C
ha

table is
(d) NaOH / air Al C
(a) O > S > Se (b) S > Se > O
(c) S > O > Se (d) Se > O > S 67. An inorganic compound on strong heating
C

gave a blackish brown powder and two


60. The second ionisation energy of the following
oxides of sulphur. The powder was dissolved
elements follows the order.
dy

in HCl and a yellow solution was obtained


(a) C > N > O > F (b) O > N > F > C which gave a blood red coloured solution
(c) O > F > N > C (d) F > O > N > C with thiocyanate ions. The inorganic
u

61. Under S.T.P. 1 mol of N 2 and 3 mol of H 2 will compound may be


St

form on complete reaction (a) CuSO 4 (b) FeSO 4 (c) ZnSO 4 (d) NiSO 4
(a) 22.4 L of NH3 (b) 4 moles of NH3 68. Which is/are correct order of ionic mobility?
(c) 44.8 L of NH3 (d) 8 moles of NH3 (a) Li + < Na + < K + (b) Al 3+ < Mg 2 + < Na +

62. 4AgNO 3 + 2H 2O + H 3PO 2 → 4Ag + X + Y (c) Both (a) and (b) (d) None of these

If X is oxyacid of nitrogen and Y is oxyacid of 69. CO 2+ (aq) + SCN − (aq) → complex (X )


phosphorous then correct statement is/are Ni(2aq+ ) + Dimethyl glyoxime → complex (Y)
(a) X is HNO 3 and Y is H3PO 4
(b) Oxidation number of Ag changed from 0 to +2 The coordination number of cobalt and nickel
(c) Oxidation number of ‘phosphorous changed in complexes (X ) and (Y) are four. The IUPAC
from +1to +5. names of the complexes (X ) and (Y) are
(d) Both (a) and (c) respectively
10 WB JEE (Engineering) Practice Set 1
(a) tetrathiocyanato-S-cobalt (II) and bis(dimethyl (a) M and N are non-mirror image stereoisomers
glyoximate) nickel (II) (b) M and O are identical
(b) tetrathiocyanato-S-cobaltate (II) and bis (c) M and P are enantiomers
(dimethyl glyoxime) nickel (II) (d) M and Q are identical
(c) tetrathiocyanato-S-cobaltate (III) and bis
(dimethyl glyoximato) nickel (II) 74. The equilibrium constant, K c for the
1
(d) tetrathiocyanato-S-cobaltate (II) and bis reaction SO 3 (g) - SO 2 (g) + O 2 (g)
(dimethyl glyoximato) nickel (II) 2
is 4 .9 × 10 −2 , The value of K C for the
70. In the following first order competing reaction.

)
reactions where t1 = t2 2 SO 2 (g) + O 2 (g) -

be
2 SO 3 (g) will be
(a) 2.40 × 10−3 (b) 416 (c) 9.8 × 10−2 (d) 4.9 × 10−2
k
A + Reagent 
1
→ Product
k
B + Reagent 2→ Product 75. Read the following reactions

Tu
k O
The ratio of 1 if only 50% of B and 94% of 
k2 (i) CH MgBr (excess)
I. H 3C  C OEt   3     → ?

ou
A have been reacted is (ii) H +
O
(a) 0.06 (b) 0.246 (c) 2.06 (d) 4.06

(i) Ph MgBr (excess)
II. H  C OEt      → ?
Category II (Q. Nos. 71 to 75)
Carry 2 marks each and only one option is correct.
In case of incorrect answer or any combination of
(Y O

(ii) H +
on
(i) CH MgBr (excess)
III. H  C OEt   3     → ?
more than one answer, 1/2 mark will be deducted. (ii) H +
+
71. In aqueous solution of Cu disproportionates as IV. ClCH2COOEt   3    
(i) CH MgBr (excess)
→ ?
pi

2Cu (aq) → Cu (aq) + Cu(s)


+ 2+ (ii) H +

In which set of reactions the product will be


m

What is the E 0 value for this reaction if


3° alcohol
0
E Cu 2+
/ Cu +
= 0 .15 V and E Cu
0
2+ = 0 .34 V
ha

/ Cu (a) I, II (b) I, II, III (c) I, IV (d) II, IV


(a) 0.48 V (b) 0.28 V (c) 0.38 V (d) 0.18 V
72. How many optical isomers are possible on
C

monochlorination of 2-methyl butane ?


Category III (Q. Nos. 76 to 80)
(a) 2 (b) 4 (c) 6 (d) 8 Carry 2 marks each and one or more option(s)
is/are correct. If all correct answers are not marked
dy

73. Which of the given statements about N , O, P


and Q with respect to M is incorrect? and also no incorrect answer is marked then score
Cl = 2 × number of correct answers marked ÷ actual
u

HO HO H number of correct answers. If any wrong option is


H H
marked or if any combination including a wrong
St

HO H HO
option is marked, the answer will be considered
Cl CH3 OH
H OH wrong, but there is no negative marking for the
H same and zero marks will be awarded.
CH3 Cl CH3
(M) (N)
76. The following statement(s) is/are correct
(O)
(a) A plot of log( X ) versus time is linear for a first
CH3 CH3
order reaction X → P
H OH HO H 1
(b) A plot of log K p versus is linear
T
1
HO H HO H (c) A plot of P versus is linear at constant temperature
V
Cl Cl
1
(d) A plot of log P versus is linear at constant
(P) (Q) T
WB JEE (Engineering) Practice Set 1 11

77. The product of the reaction D-glyceraldehyde (a) (NH2 )2 C == 0 (b) (NH2 )2 C == S
Me CO H C ==CHMgCl O (c) p NH2C 6H4SO 3H (d) C 6H5SO 3H
→
2
→
2
→
3
is/are?
HCl 79. Which of the following options are correct for
CHO CHO [Fe(CN)6 ]3− complex ?
H OH H H (a) d 2 sp3 hybridisation (b) sp3d 2 hybridisation
(a) H OH (b) H OH (c) Diamagnetic (d) Paramagnetic
CH2OH CH2OH
80. The reaction of CH 3 C == CHCH 3

)
be
CH2OH CHO 
CH 3
H OH HO H
(c) HO (d) H with NaIO 4 or boiling KMnO 4 produces

Tu
H OH

CH2OH CH2OH (a) CH3  C  CH3 (b) CH3  CHO



78. Which of the following compounds may give

ou
O
blood red colouration while performing
(c) CH3  C  OH (d) CH3  CH2  OH
Lassaigne’s test for nitrogen? 

(Y
O

Mathematics
on
pi

Category I (Q. Nos. 1 to 50) 4. Mean of 9 observations is 100 and mean of


Only one answer is correct. Correct answer will 6 observations is 80, then the mean of 15
m

fetch full marks 1. Incorrect answer or any observation is


combination of more than one answer will (a) 29 (b) 92 (c) 184 (d) 90
ha

fetch−1/4 marks. 5. If [ x ] denotes the greatest integer less than or


equal to x, then [log 10 87213
. ] is equal to
1. If A = { x : 2cos 2 x + sin x ≤ 2} and (a) 3 (b) 4 (c) 5 (d) 6
C

 π 3π 
B = x : ≤ x ≤  , then A ∩ B is equal to 6. If an error of 1° is made in measuring the
 2 2
angle of a sector of radius 60 cm, then the
dy

π 5π  3π 
(a)  x : ≤ x ≤ (b)  x : π ≤ x ≤
approximate error in its area is
 
 2 6   2  (a) 2.5 π cm 2 (b) 10 π cm 2 (c) 50 π cm 2 (d) 25 π cm 2
π 5π 3π 
u

(c)  x : ≤ x ≤ or π ≤ x ≤  [ x] + [ x 2] + [ x 3] + … + [ x 2n + 1] + n + 1
 2 6 2  7. lim ,
St

x → 0− 1 + [ x 2 ] + | x| + 2 x
(d) None of the above
n∈N
2. For any integer n, the argument of (a) 0 (b) 1 (c) 2 n + 1 (d) n
( 3 + i)4 n + 1
z= is 8. ∫ sec x − 1 dx is equal to
(1 − i 3)4 n  x x 1
(a) − 2 log  cos + cos 2 −  + C
π π 2π  2 2 2
(a) (b) (c) (d) All of these
6 3 3  x 2 x 1
(b) 2 log  cos + cos −  +C
3. Ten coins are tossed. The probability of  2 2 2
getting atleast 8 tails is  x x 1
(c) log  cos + cos 2 −  + C
(a)
3
(b)
7
(c)
5
(d)
1  2 2 2
256 128 256 64 (d) None of the above
12 WB JEE (Engineering) Practice Set 1

3x − 4 16. If AB = A and BA = B, then A2 , B2 is equal to


9. If f   = x + 2, then ∫ f (x) dx is equal to (b) − A, − B
3x + 4 (a) A, B
(c) 2 A, B (d) None of these
3x − 4 2 2
(a) e x + 2 ln + C (b) ln| x − 1| + x + C
3x + 4 3 3 17. The function f : (−∞ , − 1] → (0 , e 5] defined by
3
8 2 − 3x + 2
(c) − ln| x − 1| + x + C (d) None of these f (x) = e x is
3 3
(a) Many one and onto (b) Many one and into
10. The number of four-letter words that can be (c) One-one and onto (d) One-one and into

)
formed using the letters of the word 1

be
‘MEDITERRANEAN’ such that E is the first 18. If f (x) = 64 x 3 + and a , b are the roots of
letter and R the last is x3
1
13! 13! 4x + = 3, then

Tu
(a) (b) 61 (c) 59 (d) x
3! 2 ! 3! 2 ! 2 !
(a) f(a) = f(b ) (b) f(a) = 11
11. The general solution of the differential (c) f(b ) = 8 (d) None of these

ou
dy
equation = y tan x − y 2 sec x is 19. A line segment of length 63 having direction
dx ratios as 3 , − 2, 6. If it makes an obtuse angle

(Y
(a) sec y = (c + tan y) x (b) sec x = (c + tan x)y with X -axis, components of line vector are
(c) tan x = (c + sec x)y (d) None of these
(a) 27, 18, − 54 (b) −27, − 18, − 54
12. The term independent of x in the expansion (c) 27, − 18, − 54 (d) −27, 18, − 54
on
9
3x 2 1 20. If sin α , sin β and cosα are in GP, then roots of
of (1 + x + 2 x )  4
−  is
 2 3x x 2 + 2 x cotβ + 1 = 0 are always
pi

−1 11 19 7 (a) imaginary
(a) (b) (c) (d)
27 27 54 18 (b) imaginary and negative
m

(c) real
13. Locus of mid-point of the portion between (d) greater than one
the axes of x cos α + y sin α = p, where p is
ha

constant, is 21. If f (x) = 3 x 10 − 7 x 8 + 5 x 6 − 21x 3 + 3 x 2 − 7,


(a) x2 + y2 =
4
(b) x2 + y2 =
2 f (1 − α) − f ()
1
then lim is
C

p2 p2 α→ 0 α 3 + 3α
1 1 4
(c) x2 + y2 = 2 p2 (d) + = 53 53 53 55
x2 y2 p2 (a) − (b) (c) − (d) −
dy

5 3 3 3
14. The line 2 x + y = 1 touches a hyperbola and 22. A tangent PT is drawn to the circle
u

passes through the point of intersection of a x 2 + y 2 = 4 at the point P( 3 , 1). A straight


directrix and the X -axis. The equation of the
St

line L, perpendicular to PT is a tangent to the


hyperbola is
circle (x − 3)2 + y 2 = 1. A possible equation of
x2 y2 x2 y2
(a) − =3 (b) − =1 L is
1 2 1 3
x 2
y 2 (a) x − 3 y = 1 (b) x + 3y = 1
(c) − =1 (d) None of these (c) x − 3 y = − 1 (d) x + 3y = 5
3 2
23. There are five different green dyes, four
10 ! 11! 12 !
different blue dyes and three different red
15. 11! 12! 13! equals dyes. The total number of combination of
12 ! 13 ! 14 ! dyes that can be chosen taking atleast one
green and one blue dye is
(a) 2 (11! 12 ! 13!) (b) 2 (11! 12 !)
(c) 2 (10! 11! 12 !) (d) 2 (10! 11!) (a) 3720 (b) 2 (c) 3256 (d) 3255
WB JEE (Engineering) Practice Set 1 13

24. The locus of the points of intersection of 1 cos x 1 − cos x


tangents at the extremities of the chords of 32. If ∆(x) = 1 + sin x cos x 1 + sin x − cos x ,
the ellipse having the equation x 2 + 2 y 2 = 6 sin x sin x 1
which touch the ellipse x 2 + 4 y 2 = 4 is π
(a) x2 + y2 = 6 (b) x2 + y2 = 4 then ∫ 2 ∆(x) dx equals
0
(c) x2 + y2 = 9 (d) x2 + 2 y2 = 10 1 −1
(a) − (b)
4 2
25. The solution of the differential equation

)
(c) 0 (d) None of these
dy

be
+ y
+ 1 = ex is
dx 33. If the coordinates of vertices of a triangle are
(a) xe x + y + y = c (b) x + e x + y = c (4 , 7) and (6,1) and the third vertex moves on

Tu
(c) − x e x + y + y = c (d) x + e −( x + y ) = c the line 9 x + 7 y = 28 , then the locus of the
centroid is
26. 49 n + 16 n − 1 is divisible by (a) 9 x + 7 y + 42 = 0 (b) 9 x + 7 y − 58 = 0

ou
(a) 29 (b) 64 (c) 17 (d) 5 (c) 7 x + 9 y − 42 = 0 (d) 7 x − 9 y + 58 = 0
x n− x
 4  1 2  m  m
 x sin  x  + x 
34. lim nC x   1 −  is equal to

(Y  n  n
n→ ∞
27. The value of lim   is
x → −∞
 1 + | x|3  mx ⋅ e m mx ⋅ e −m
(a) 1 (b) (c) 0 (d)
  x! x!
on
(a) 1 (b) ∞ (c) 0 (d) −1 35. The domain of f (x) is (0, 1), therefore
28. The inclination of the straight line through domain of f (e x ) + f (ln| x|) is
pi

the point (−3 , 6) and mid-point of line joining (a) (−e, 1) (b) (e, 1)
points (4 , − 5) and (−2 , 9) is (c) (−1, e ) (d) (− e, − 1)
m

3π π π π
(a) (b) (c) (d)
4 3 8 4 36. The last term in the binomial expansion of
ha

n log 3 8
3 1   1 
29. The function  2−  is  3  .
 2 3⋅ 9 
 π x  πx 
C

f (x) = sin   − cos   is Then the 5th term from the beginning is
 n!   (n + 1)!
1 10
(a) ⋅ C4 (b) 10 C 5 (c) 2 ⋅10 C 4 (d) 10 C 6
(a) 2(n + 1)!
dy

2
(b) periodic, with period 2(n!)
(c) non-periodic 37. The sum of two non-integral roots of
u

(d) periodic, with (n + 1) x 2 5


3 = 0 is
St

30. If the intercept made on line y = mx by lines 3 x


y = 2 and y = 6 is less than 5, then range of m 5 4 x
is (a) −5 (b) 5
(a)  −∞, −  ∪  , + ∞  (b)  − ,  (c) − 18
4 4 3 3 (d) 18
 3  3   4 4
x −2 y−3 z− 4
(c)  , − 
4 4
(d) None of these 38. The lines = = and
 3 3 1 1 −k
x −1 y − 4 z − 5
= = are coplanar, if
(n!)1 / n k 2 1
31. lim is equal to
n→ ∞ n (a) k = 0, − 3 (b) k = 0, 3
2 3 −2 1 (c) k = − 1, 1 (d) k = − 1, 3
(a) e (b) e (c) e (d)
e
14 WB JEE (Engineering) Practice Set 1

39. The normal at an end of a latus rectum of the 46. Radius of the circle passing through the foci
x2 y2 x2 4 2
ellipse 2
+ 2
= 1 passes through an end of of the ellipse + y = 1 having centre at
a b 4 7
the minor axis if 1 
 , 2 is
(a) e 2 + e = 1 (b) e 4 + e 2 = 1 2 
(c) e 3 + e 2 = 1 (d) e 3 + e = 1 7
(a) 5 (b) 2 2 (c) 4 (d)
2
40. Consider the family of lines

)
(x + y − 1) + λ (2 x + 3 y − 5) = 0 and xe x
47. If ∫

be
dx = f (x) 1 + e x − 2 log| g(x)| + C ,
(3 x + 2 y − 4) + µ (x + 2 y − 6) = 0 , equation of a 1 + ex
straight line that belongs to both the families
is then

Tu
(a) x − 2 y + 8 = 0 (b) 2 x − y − 8 = 0 (a) f( x) = 3( x − 2 ) (b) f( x) = x − 1
(c) x − y + 8 (d) 2 x + y + 8 = 0 1+ e − 1
x
1+ ex + 1
(c) g ( x) = (d) g ( x) =

ou
41. If u, v , w , z are positive real numbers such 1+ e + 1
x
1+ ex − 1
that u + v + w + z = 2, then m = (u + v)(w + z) 1 − cos(ax 2 + bx + c)
48. The value of lim where

(Y
satisfies the relation
x→α (x − α)2
(a) 3 < m ≤ 4 (b) 1 ≤ m ≤ 2
(c) 2 ≤ m ≤ 3 (d) 0 < m ≤ 1 α and β are the roots of ax 2 + bx + c = 0 is
(a − b )2
on
cosec 2 x − 2005 (b) (α − β )2

(a)
42. 2005
dx 2
cos x 1
− cot x tan x (c) a2 (α − β )2 (d) None of these
pi

(a) +C (b) +C 2
(cos x)2005 (cos x)2005
− tan x 49. If the graph of y = ax 3 + bx 2 + cx + d is
m

cot x
(c) +C (d) +C
(cos x)2005 (cos x)2005 symmetric about the line x = k, then the
value of a + k is
ha

43. Domain of f (x) = sin −1[2 − 4 x 2] is; where [⋅] −c −c


(a) (b) (c) c − 2 b (d) c + 2d
denotes the greatest integer function is a 2b
C

(a) (−2, 2 )
50. The equation of circle having centre as its
(b) [−1, 1]
origin and passing through the vertices of an
 3   3
dy

(c)  − , 0 ∪  0,  equilateral triangle whose median is of 6 a is


 2   2 
(a) x2 + y2 = 4a2 (b) x2 + y2 = 9a2
(d) None of the above
(c) x + y = 16 a
2 2 2
(d) None of these
u

44. If x1 , x 2 , x 3 are roots of


St

x−a x−b b a Category II (Q. No. 51 to 65)


+ = + , given (a , b > 0),
b a x−a x−b Carry 2 marks each and only one option is
x1 > x 2 > x 3 and x1 − x 2 − x 3 = c, then a , b, c correct. In case of incorrect answer or any
are in combination of more than one answer, 1/2 mark
(a) GP (b) HP will be deducted.
(c) AP (d) None of these
51. If a , b, c and d are unit vectors, then
45. On the parabola y = x 2, the point at least |a − b|2 + |b − c |2 + |c − d|2 + |d − a|2
distance from the straight line y = 2 x − 4 is
+ |c − a |2 + |b − d|2
(a) (0, 0) (b) (0, 1)
(c) (1, 1) (d) (−1, 0) does not exceed
(a) 2 (b) 8 (c) 12 (d) 6
WB JEE (Engineering) Practice Set 1 15

52. If z is a point on the argand plane such that 58. The lengths of the intercepts made by any
z−2 circle on the coordinate axes are equal if the
|z − 1| = 1, then equals
z centre lies on line represented by
(a) cot(arg z) (b) i tan(arg z) (a) x + y = 1 (b) x − y = 1
(c) tan(arg z) (d) i cot(arg z) (c) x + y + 1 = 0 (d) x2 − y2 = 0

53. The largest term common to the sequences 1, 59. The number of solutions of| x| = cos x is
11, 21, 31 …… to 100 terms and 31, 36, 41, (a) 3 (b) 2 (c) 1 (d) 0

)
46 …… to 100 terms is

be
(a) 471 (b) 521 60. The largest term in the sequence
(c) 421 (d) 371 n2
an = is given by
 3 1  n + 200
3

Tu
 
54. If P =  2 2  , A = 1 1 and Q = PAP T , (a)
8
(b)
49
0 1
 −1 3   49 529

ou
8
 2 2  (c) (d) None of these
89
then P T (Q2005)P equals
61. If a , b, c are three non coplanar, non zero

(Y
 3  1 2005
(a)  1 2  (b)  vectors then
0 0 1 
 1  (a ⋅ a)b × c + (a ⋅ b)c × a + (a ⋅ c)a × b is
 1 2005 equal to
on
1 2005
(d)  3
1 
(c) 
1 0   2
(a) [c a b] b (b) [a b c ] c
 (c) [b c a ] a (d) [a b c ] b
pi

55. If the function f :[1, ∞) → [1, ∞) is defined by 62. The range of


f (x) = 2 x (x − 1)
, then f −1 (x) is  1  1
f (x) = sin −1  x 2 +  + cos −1  x 2 − , (where
m

 2  2
(a)   (1 − 1 + 4log 2 x)
1
2 [⋅] denotes the greatest integer function) is
ha

π π
1 x( x − 1) (b)   (c) − , 0 (d)  0, 
1
(b) (a) { π}
2 4  2   2
C

1
(c) (1 + 1 + 4log 2 x ) 63. A rod of length l slides with its end on two
2
x( x + 1) perpendicular lines. The locus of point which
(d)  
1
dy

divides it in the ratio 1 : 2 is


2
(a) 9 x2 − 36 y2 = 4l 2 (b) 36 x2 + 9 y2 = 4l 2
56. Without changing the direction of coordinate (c) 9 x + 36 y = 4l
2 2 2
(d) 36 x2 + 9 y2 = l 2
u

axes, origin is transferred to (α , β) so that the


St

nx
linear terms in x 2 + y 2 + 2 x − 4 y + 6 = 0 are  1 1 1 1
1x + 2 x + 3 x + …… n x 
eliminated. The point (α , β) is 64. The value of lim  is
x→∞ n 
(a) (−1, 2 ) (b) (2, − 1)  
(c) (1, − 2 ) (d) (−1, − 2 )  
(a) 0 (b) 1
r
57. If tr = be the rth term of a series, (c) n! (d) (n + 1)!
r + r +1
4 2

n 65. The digit at units place in the number


then lim
n→ ∞
∑ tr is 17 1995 + 111995 − 7 1995 is
r =1
1 (a) 1 (b) 0
(a) (b) 2 (c) 0 (d) 1 (c) 2 (d) None of these
2
16 WB JEE (Engineering) Practice Set 1

Category III (Q. Nos. 66 to 75) (b) locus to z is a circle



(c) arg ( z) = ±
Carry 2 marks each and one or more option(s) 3
is/are correct. If all correct answers are not (d)| z| = |a|
marked and also no incorrect answer is marked
then score = 2 × number of correct answers 71. If sinβ is the geometric mean between sinα
marked ÷ actual number of correct answer. If any and cosα, then cos2β is equal to
π π
wrong option is marked or if, any combination (a) 2 cos 2  − α  (b) 2 sin2  − α 
including a wrong option is marked, the answer 4  4 

)
2 π  2 π
will be considered wrong, but there is no negative (d) 2 cos  + α 

be
(c) 2 sin  + α 
marking for the same and zero marks will be 4  4 
awarded.
72. A line which makes an acute angle θ with the

Tu
sin(x + a) positive direction of X -axis is drawn through
66. The function f (x) = has no maxima
sin(x + b) the point P(3 , 4) to meet the line x = 6 at R
and y = 8 at S, then

ou
or minima if
(a) a + b = nπ, n ∈ I (a) PR = 3sec θ
(b) b − a = 2 nπ, n ∈ I (b) PS = 4 cosec θ
2 (3sinθ + 4cos θ)

(Y
(c) b − a = (2 n + 1)π, n ∈ I (c) PR + PS =
(d) b − a = nπ, n ∈ I sin2θ
9 16
(d) + = 1
67. The function f is such that (PR )2 (PS )2
on
f (xy) = f (x) + f (y), x , y > 0 and f ′ ()
1 = 2 and
A sq. Unit be the area bounded by the curves 73. If the circle
y = f (x), x = 2 and the X -axis, then x2 + y2 + 2 gx + 2 fy + c = 0, cuts each of the
pi

circle x2 + y2 − 4 = 0
(b) A = 4 log 
2 
(a) f( x) = log| x| 
 e x2 + y2 − 6 x − 8 y + 10 = 0
m

(c) f( x) = 2 log| x| (d) A = 2(2 log 2 + 5) and x 2 + y2 + 2 x − 4 y − 2 = 0


ha

1 2 2 at the extremities of a diameter, then


(a) g f = 6 (b) g + f = c − 1
68. If A = 2 1 2, then
  (c) c = − 4 (d) g 2 + f 2 − c = 17
C

2 2 1
1 74. The domain of function
(a) A 2 − 4 A − 5I3 = 0 (b) A −1 = ( A − 4I3 )
 x − 1
dy

5 1
(c) A 3 is not invertible (d) A 2 is invertible f (x) = log 0. 4   + 2 is
 x + 5 x − 36
69. If a , b, c , d are four unequal positive numbers (a) (−∞, 0) − {− 6} (b) (0, ∞ ) − {1, 6}
u

which are in AP, then (c) (1, ∞ ) − {6, − 6} (d) [1, ∞ ) − {6}
St

1 1 1 1 1 1 1 1
(a) + = + (b) + < +   1 
a d b c a d b c 75. The function f (x) =  x 2  2  , x ≠ 0 is ([x]
1 1 1 1 1 1 4   x 
(c) + > + (d) + >
a d b c b c a+d represents the greatest integer ≤ x)
(a) continuous at x = 1
70. Let ‘z’ be a complex number and ‘a’ be a real (b) discontinuous at x = − 1
parameter such that z 2 + az + a 2 = 0 , then (c) discontinuous at infinitely many points
(d) continuous everywhere
(a) locus of z is a pair of straight lines
Answers
Physics
1. (a) 2. (c) 3. (c) 4. (d) 5. (b) 6. (c) 7. (b) 8. (d) 9. (b) 10. (a)
11. (b) 12. (b) 13. (b) 14. (d) 15. (a) 16. (a) 17. (c) 18. (d) 19. (d) 20. (b)
21. (a) 22. (b) 23. (b) 24. (a) 25. (d) 26. (a) 27. (b) 28. (c) 29. (b) 30. (c)

)
31. (b) 32. (b) 33. (d) 34. (c) 35. (a) 36. (a, c, d) 37. (a, d) 38. (a, c) 39. (a, c) 40. (a)

be
Chemistry
41. (b) 42. (a) 43. (d) 44. (c) 45. (b) 46. (b) 47. (d) 48. (d) 49. (c) 50. (b)

Tu
51. (a) 52. (d) 53. (c) 54. (a) 55. (c) 56. (a) 57. (d) 58. (b) 59. (b) 60. (c)
61. (c) 62. (d) 63. (b) 64. (a) 65. (a) 66. (a) 67. (b) 68. (c) 69. (d) 70. (d)
71. (c) 72. (b) 73. (d) 74. (b) 75. (c) 76. (b, d) 77. (a, d) 78. (b, c) 79. (a, d) 80. (a, c)

ou
Mathematics

(Y
1.(c) 2.(a) 3.(b) 4.(b) 5.(a) 6.(b) 7.(a) 8.(a) 9.(c) 10.(c)
11.(b) 12.(d) 13.(d) 14.(b) 15.(c) 16.(a) 17.(d) 18.(a) 19.(d) 20.(c)
21.(b) 22.(a) 23.(a) 24.(c) 25.(d) 26.(b) 27.(d) 28.(a) 29.(a) 30.(c)
on
31.(d) 32.(b) 33.(b) 34.(d) 35.(d) 36.(d) 37.(a) 38.(a) 39.(b) 40.(a)
41.(d) 42.(a) 43.(c) 44.(b) 45.(c) 46.(a) 47.(c) 48.(c) 49.(b) 50.(c)
51.(c) 52.(b) 53.(b) 54.(b) 55.(c) 56.(a) 57.(a) 58.(d) 59.(b) 60.(d)
pi

61.(c) 62.(a) 63.(b) 64.(c) 65.(a) 66.(b, c, d) 67.(b,c) 68.(a,b,d) 69.(c,d) 70.(a,c,d)
71.(b,d) 72.(a,b,c,d) 73.(a,b,c,d) 74.(c) 75.(b,c)
m

* For detailed solutions visit http://tinyurl.com/y3z3nwxa or scan


ha
C
u dy
St
WB JEE
Engineering Entrance Exam

Practice Set 2

)
be
Tu
Physics

ou
Category I (Q. Nos. 1 to 30) (Take, atomic mass of nitrogen = 14 .0076 u)

(Y
Carry 1 marks each and only one option is (a) 0.01 nm (b) 0.09 nm
(c) 0.03 nm (d) 0.2 nm
correct. In case of incorrect answer or any
combination of more than one answer, 1/4 mark 4. Taking the Bohr radius as a 0 = 53 pm, the
on
will be deducted. radius of Li + + ion in its ground state, on the
1. A ray of light from a denser medium strikes a basis of Bohr’s model, will be about
pi

rarer medium at angle of incidence i. The (a) 53 pm (b) 27 pm


reflected and refracted rays make an angle of (c)18 pm (d) 13 pm
m

reflection and refraction are r and r′, 5. In a Zener regulated power supply, a Zener
respectively. The critical angle is diode with Vz = 6 .0 V is used for regulation.
ha

The load current is to be 4.0 mA and the


Denser unregulated input 10.0 V. The value of series
r
resistor RS must be
C

I
dy

r′
RS
Rarer

Unregulated IL
u

(a) sin−1(tan r ) (b) sin−1(cot i ) voltage IZ


St

(c) tan−1(sin r ) (d) tan−1(sin i ) (Regulated


(Load) VZ
RL voltage)
2. The half-life of At is 100 µs. The time
215

taken for the radioactivity of a sample of


215 1
At to decay to th of its initial value is (a) 167 Ω (b) 120 Ω
16 (c) 250 Ω (d) 20 Ω
(a) 400 µs (b) 6.3 µs (c) 40 µs (d) 300 µs
6. Each of the two inputs A and B can assume
3. What is the de-Broglie wavelength of a values either 0 or 1. Then, which of the
nitrogen molecule in air at 300 K? Assume following will be equal to A + B ?
that the molecule is moving with the root (a) A ⋅ B (b) A + B
mean square speed of molecules at this (c) A + B (d) A ⋅ B
temperature.
WB JEE (Engineering) Practice Set 2 19

7. The correct dimensional formula for linear 13. A uniform rod of length (L) and area of
momentum is given by cross-section (A) is subjected to tensile load
(a) ML2 T−2 (b) MLT−1 (F). If σ be the poisson’s ratio and Y be the
(c) ML2 T−1 (d) MLT−2 Young’s modulus of the material of the rod,
then find the volumetric strain produced in
8. A wire has a mass (0.3 ± 0.003) g, radius the rod.
(0 .5 + 0 .005) mm and length (6 ± 0 .06) cm. The 5 F
(a) (1 + 2 σ ) (b) (1 − 2 σ )
maximum percentage error in the AY AY

)
measurement of its density is (c) Zero (d) None of these

be
(a) 1 (b) 2 (c) 3 (d) 4
14. Suppose a fluid like oil enclosed between two
9. If the surface is smooth, the acceleration of glass plates as shown in figure. The bottom

Tu
the block m2 will be plate is fixed, while the top plate is moved
with a constant velocity v relative to fixed
m1 plate.

ou
∆x=v∆t
F
B E C F v

(Y l
on
m2

m2 g 2 m2 g
(a) (b)
4m1 + m2 4m1 + m2
pi

2 m1g 2 m1g A D
(c) (d)
m

m1 + 4m2 m1 + m2
If oil is replaced by honey, then which one
has greater viscous force?
10. The work done in time t on a body of mass m
ha

which is accelerated from rest to a speed v in (a) Honey > Oil (b) Honey < Oil
(c) Honey = Oil (d) Honey ≥ Oil
time t1 as a function of time t is given by
C

(a)
1 mv 2
t (b)
mv 2
t 15. A black body radiate energy at rate of X W/m 2
2 t1 t1 at a high temperature of T kelvin. When
1  mv  2 1 mv 2 2 T 
dy

(c)  t (d) t temperature is reduced to   kelvin, the


2  t1  2 t 12  2
radiant energy is
u

11. The speed of a projectile u reduces by 50% on X X X


reaching maximum height. What is the range (a) (b) (c) (d) 2 X
St

16 4 2
on the horizontal plane?
u2 3 u2 u2 4 u2 16. Certain amount of heat is given to 100 g of
(a) × (b) (c) ×3 (d) copper to increase its temperature by 21°C. If
g 2 g 2g g
the same amount of heat is given to 50 g
12. The time period of a satellite of the earth is water, then the rise in its temperature is
5h. If the separation between the earth and (Take, specific heat capacity of
the satellite is increased to 4 times the copper = 400 Jkg −1 K −1 and that of
previous value, then the new time period will water = 4200 Jkg −1 K −1 )
become
(a) 4°C (b) 5.25°C
(a) 10 h (b) 80 h (c) 8°C (d) 6°C
(c) 40 h (d) 20 h
20 WB JEE (Engineering) Practice Set 2

17. The top surface of an incompressible liquid is 22. A long straight solid metal wire of radius R
open to the atmosphere. The pressure at a carries a current i uniformly distributed over
depth h1 below the surface is p1 . How does its circular cross-section. Find the magnetic
the pressure p 2 at depth h2 = 2 h1 compare field inside the wire at a distance r from the
with p1 ? axis of wire.
µ 0µ r ir
(a) p2 > 2 p1 (b) p2 = 2 p1 (a) B =
2 πR 2
(c) p2 < 2 p1 (d) p2 = p1 µ 0µ r ir
(b)B =
18. A network of four capacitors each of 12 µF 4 πR 2

)
µ 0µ r ir

be
capacitance, if connected to a 500 V supply as (c) B =
shown in figure. What is the total amount of 8 πR 2
µ 0µ r ir
charge stored? (d) B =
πR 2

Tu
C2

23. A current i ampere flows along an infinitely


long straight thin walled tube, then the

ou
magnetic induction at any point inside tube is
C1 C3
(a) infinite
(b) zero

C4 (Y µ 2i
(c) 0 ⋅ tesla
4π r
2i
(d) tesla
on
r
500V 24. A wire has a non-uniform cross-sectional
(a) 6000 µC (b) 5000 µC (c) 3000 µC (d) 8000 µC
pi

area as shown in figure. A steady current i


flows through it. Which one of the following
19. Four particles, each having a charge q are
m

statement is correct?
placed on the four corners A, B, C and D of a
regular pentagon ABCDE. The distance of A B
ha

each corner from the centre is a. Find the


electric field at the centre of the pentagon.
q q (a) The drift speed of electron is constant
(a) along OE (b) along OC
C

4 π ε0 a 2
4 π ε0 a 2 (b) The drift speed increases on moving from A to B
q q (c) The drift speed decreases on moving from A to B
(c) along OD (d) along OA
dy

4 π ε0 a 2 4 π ε0 a 2 (d) The drift speed varies randomly

20. Two point charges placed at certain distance r 25. A battery of internal resistance 4 Ω is
connected to the network of resistance as
u

in air exert a force F on each other. Then, the


distance r at which, these charges will exert shown in the figure. In order to give the
St

the same force in a medium of dielectric maximum power to the network, the value of
constant K is given by R (in Ω) should be
R R
(a) r (b) r / K (c) r / K (d) r K
21. If magnetic field B is present at a place E R 6R R
alongwith an electric field of intensity E,
then force experienced by a moving charge q
in that region is given by R 4R
(a) F = (qE + v × B), where v is velocity of charge
(b) F = q (E + B) × v, where v is velocity of charge
(c) F = E + q (v × B), where v is velocity of charge
(d) F = q (E + v × B), where v is velocity of charge
(a) 4 / 9 (b) 8 / 9 (c) 2 (d) 18
WB JEE (Engineering) Practice Set 2 21

26. In the circuit, if no current flows through the 29. In a Young’s double slit experiment, the
galvanometer when the key K is closed, the fringe width obtained is 0.4 cm, when light
bridge is balanced. The balancing condition of wavelength 5400 Å is used. If the distance
for bridge is between the screen and the slit is reduced to
B half, then what should be the wavelength of
R1 R2 light used to obtain fringes 0.0048 m wide?
K (a) 12960 Å (b) 1300 Å
A C (c) 1400 Å (d) 1500 Å
G

)
30. Find the angular spread between central

be
C1 C2
maximum and first order maximum of the
I
D diffraction pattern due to a single slit of

Tu
width 0.20 nm, when light of wavelength
5460 Å is incident on it normally.
E
S (a) 9 × 10−4 rad . × 10−3 rad
(b) 819

ou
−5
(a)
C1 R
= 1 (b)
C1 R 2
= (c) 6 × 10 rad (d) 8.65 × 10−4 rad
C 2 R2 C2 R1
C12 R12 C12 R2 Category II (Q. Nos. 31 to 35)

(Y
(c) = (d) =
C 22 R 22 C 22 R1 Carry 2 marks each and only one option is
correct. In case of incorrect answer or any
27. An inductor (L = 100 mH), a resistor
on
combination of more than one answer, 1/2 mark
(R = 100 Ω) and a battery (E = 100 V) are will be deducted.
initially connected in series as shown in the
figure. After a long time, the battery is
pi

31. A gaseous mixture consists of 16 g of helium


disconnected after short circuiting the points Cp
A and B. The current in the circuit, 1 ms after and 16 g of oxygen. The ratio of the
m

CV
the short circuit is
mixture is
L
ha

(a) 1.59 (b) 1.62


(c) 1.4 (d) 1.54
C

R 32. Air is filled in a motor car tube at 27°C


temperature and 2 atmospheric pressure. If
the tube suddenly bursts, then the final
dy

A B
 1  2 / 7 
E temperature will be   = 0 .82
 2  
u

1
(a) A (b) e A
e (a) 246 K (b) 210 K
St

(c) 0.1 A (d) 1 A (c) 320 K (d) 180 K

28. A car is fitted with a convex side-view mirror 33. A block of copper having mass 2 kg is heated
of focal length 20 cm. A second car 2.8 m to a temperature of 500°C and then placed
behind the first car is overtaking the first car in a large block of ice at 0°C. What is the
at relative speed 15 m/s. The speed of the maximum amount of ice that can melt?
image of the second car as seen in the mirror The specific heat of copper is 400 J kg −1 C −1
of the first one is and latent heat of fusion of water is
(a)
1
m/s (b) 10 m/s 3.5 × 10 5 J kg −1 .
15 4 6
1 (a) kg (b) kg
(c) 15 m/s (d) m/s 3 5
10 8 10
(c) kg (d) kg
7 9
22 WB JEE (Engineering) Practice Set 2
p
34. An iron rod of length L and magnetic moment
m is bent in the form of a semi-circle. Now, its
magnetic moment will be I
2m
(a) m (b) IV
π
m A II
(c) (d) mπ
π
B
35. In the given circuit, calculate the Q-factor of III

)
the circuit.

be
200mH C V
10W
(a) Change in internal energy is same in IV and III

Tu
cases, but not in I and II
(b) Change in internal energy is same in all the four
cases

ou
(c) Work done is maximum in case I
(d) Work done is minimum in case II
50Hz

(a) 6.32
(b) 5.32
(c) 4.00
(Y
38. A particle is hurled into air from a point on
the horizontal ground at an angle with the
vertical. If the air exerts a constant resistive
on
(d) 3.32 force,
(a) the path of projectile will be parabolic path
Category III (Q. Nos. 36 to 40) (b) the time of ascent will be equal to time of
pi

descent
Carry 2 marks each and one or more option(s) (c) the total energy of the projectile is not conserved
m

is/are correct. If all correct answers are not marked (d) at the highest point, the velocity of projectile is
and also no incorrect answer is marked then score horizontal
= 2 × number of correct answers marked ÷ actual
ha

number of correct answers. If any wrong option is 39. Temperature dependence of resistivity ρ(T) of
marked or if any combination including a wrong semiconductors, insulators and metals is
significantly based on the following factors:
C

option is marked, the answer will be considered


wrong, but there is no negative marking for the (a) Number of charge carriers can change with
same and zero marks will be awarded. temperature T
dy

(b) Time interval between two successive collisions


36. In a photoelectric experiment, the can depend on T
wavelength of the incident light is decreased (c) Length of material can be function of T
u

from 6000 Å to 4000 Å, while the intensity of (d) Mass of carriers is a function of T
St

radiation remains the same, 40. Magnetic field due to a current carrying wire
(a) the cut-off potential will increase
loop along its axis can be calculated by
(b) the cut-off potential will decrease
(a) finding electric field of coil and then by using
(c) the kinetic energy of the emitted photoelectron
E/B=c
will increase
(b) finding electric field of a small element of
(d) the photoelectric current will increase
current carrying coil and then by using E / B = c
37. Figure shows the p-V diagram of an ideal gas (c) suming up the magnetic field of small elements
of the coil
undergoing a change of state from A to B.
(d) finding magnetic field of a wire of length l and
Four different paths I, II, III and IV as shown then by substituting l = 2 πr, r being radius of
in the figure may lead to the same changes of loop
state.
Chemistry
Category I (Q. Nos. 41-70) 48. The uncertainty in position of a particle of
Carry 1 mark each and only one option is correct. 25 g in space is 10 −5m. Hence, its uncertainty
In case of incorrect answer or any combination of in velocity (in ms −1 ) is
more than one answer, 1/ 4 mark will be deducted. (Given : Planck’s constant h = 6 .6 × 10 −34 J-s)
(a) 2.1 × 10−28 (b) 2.1 × 10−20

)
41. The angular momentum for an electron
−34

be
revolving in s-subshell will be (c) 2.1 × 10 (d) 2.1 × 1012
1 h h h N
(a) . (b) zero (c) (d) 2 . 49. 5 mL of N.HCl, 20 mL of H 2SO 4 and 30 mL
2 2π 2π π
2

Tu
42. Atomic number of vanadium (V), chromium N
of HNO 3 are mixed and volume made to 1 L.
(Cr), maganese (Mn) and Iron (Fe) are 3

ou
respectively 23, 24, 25 and 26. Which of these The normality of the resulting solution will be
way be expected to have highest II nd (a)
N
(b)
N
(c)
N
(d)
N
ionisation enthalpy? 5 10 20 40
(a) V (b) Cr (c) Mn (d) Fe

43. The first ionisation potential of Na is 5.1 eV,


the value of electron gain enthalpy of Na +
(Y
50. Which of the following formula does not
correctly represent the bonding capacity of
the atoms involved?
on
+
will be H
(a) − 2.55 eV (b) − 5.1 eV (a) H—P—H (b) F F
pi

(c) − 10.2 eV (d) + 2.55 eV O


H
44. Normality of ‘30 volume’ H 2O 2 solution is
m

O O
(c) O N O (d) H—C—C
(a) 1.6 (b) 91.07 (c) 10.72 (d) 5.36 O
ha

—H
45. Among the following complex ions, the
species whose central atom does not have ‘d’ 51. Hydrogen can not be produced by the action
electrons is of dil.H 2SO 4 on
C

(a) [MnO 4 ]− (b) [Co(NH3 )6 ]3+ (a) Cu (b) Zn (c) Fe (d) Al


(c) [Fe(CN)6 ] 3−
(d) [Cr(H2O)6 ] 3+
52. The oxidation state of Cr in [Cr(NH 3)4 Cl 2 ]+ is
dy

(a) 0 (b) + 1 (c) + 2 (d) + 3


46. The molecular shape of SF4 , CF4 and XeF4 are:
53. The pK a of a weak acid [HA] is 4.5 the pOH
u

(a) different with 1, 0 and 2 lone-pair of electrons


on the central atom, respectively of an aqueous buffered solution of HA in
St

(b) different with 0, 1 and 2 lone-pair of electrons


which 50% of it is ionised is?
on the central atom, respectively (a) 4.5 (b) 2.5 (c) 9.5 (d) 7.0
(c) same with 1, 1 and 1 lone-pair of electrons on 54. For a reaction
the central atom, respectively
CO(g) + Cl 2 (g) ] COCl 2(g), the K p / K c is
(d) same with 2, 0 and 1 lone-pair of electrons on
equal to
the central atom, respectively
1
(a) (b) RT (c) RT (d) (RT )2
47. A gas is initially at 1 atm pressure, to RT
1 55. A spontaneous change is one, in which the
compress it to th of its initial volume.
4 system suffers
Pressure to be applied is (a) an increase in internal energy
1 (b) lowering in entropy (c) lowering in free energy
(a) 1 atm (b) 2 atm (c) 4 atm (d) atm
4 (d) no change in energy
24 WB JEE (Engineering) Practice Set 2

56. When a mole of sodium chloride is dissolved in 63. Which of the following nomenclature is not
water at 298 K, the free energy change will be according to IUPAC system?
(Given, lattice energy of NaCl (a) Br  CH2  CH == CH2
(1-bromo prop 2-ene)
= − 777.8 kJmol −1 , Hydration energy of
CH3
NaCl = − 774.1 kJ mol −1 and ∆S at 298K 
= 0 .043 kJmol −1 ) (b) CH3  CH2  C  CH2  CH CH3
 
(a) − 37
. kJ (b) − 12.814 kJ Br3 CH3

)
(c) − 9114
. kJ (d) − 16.54 kJ

be
(4-bromo 2,4, dimethyl hexane)
57. Graphite is a good conductor of electricity. (c) CH3  CH  CH  CH2 CH3
because it contain  

Tu
CH3 Ph
(a) bonded electrons (b) mobile electrons
(2-methyl 3-phenyl pentane)
(c) strong C—C bonds (d) strong C==C bonds
(d) CH3  C  CH2  CH2  CH2  COOH
58. Which of the following does not form M 3+ 

ou
ion? O
(a) B (b) Al (c) Ga (d) In (5-oxohexanoic acid)

59. Sodium is made by the electrolysis of molten


mixture of about 40% NaCl and 60% CaCl 2 ,
because
(Y
64. Which of the following energy level diagram
for [FeF6 ]3− is correct on the basis of crystal
field theory?
on
(a) Ca 2+ can reduce NaCl to Na eg
(b) CaCl 2 helps in conduction of electricity
(c) the mixture has lower melting point than NaCl (a)
pi

(d) Ca 2+ ion can displace Na from NaCl 5


3d
t2g
m

60. The compound which does not exhibit optical


isomerism is
ha

(a) CH3CHBrCOOH (b) CH2ClCH2COOH t2g


(c) CH3CH.OH.COOC 2H5 (d) CH3CHOHCOOH

61. On ozonolysis of organic compound (A) we (b)


C

get acetone and propionaldehyde as 5


3d
eg
equimolar mixture.
dy

Identify (A) from the following compounds.


(a) 2-methyl pent 1-ene (b) pent 1-ene eg
(c) pent -2-ene (d) 2-methyl pent-2-ene
u

(c)
62. The most stable conformation of n-butane is
St

5
3d
CH3 CH3 t2g

H CH3 H H
(a) (b) t2g
H H H H
(d)
H CH3 5
3d
CH3 CH3 eg
CH3 H
65. In XeF2 , XeF4 and XeF6 , the number of
(c) (d)
lone-pairs on the Xe-atom are respectively.
H H H H (a) 2, 3, 1 (b) 1, 2, 3
H H H CH3 (c) 3, 1, 2 (d) 3, 2, 1
WB JEE (Engineering) Practice Set 2 25

66. The salts of Cu in + 1 oxidation are unstable (b) X = path of reaction with catalyst,
because Y = path of reaction without catalyst
(a) Cu+ and 3d 10 configuration (c) X = energy of activation with catalyst,
Y = energy of activation without catalyst
(b) Cu+ disproportionates easily to Cu(0) and Cu2+
(d) X = energy of endothermic reaction,
(c) Cu+ disproportionates easily to Cu2+ and Cu3+
Y = energy of exothermic reaction
(d) Cu+ is easily reduced to Cu2+
67. Number of P—O—P bonds present in cyclic 72. The amount of metal deposited, when a
metaphosphoric acid are current of 12 ampere with 75% efficiency is

)
passed through the cell for 3 H .

be
(a) four (b) three (c) two (d) one
(Given, Z = 4 × 10 −4 ).
68. Which of the following changes take place (a) 32.4 g (b) 38.8 g
during roasting? (c) 36.0 g (d) 22.4 g

Tu
(i) Impurities are removed as their volatile
73. Molar conductivity of NH 4OH can be
oxides.
calculated by the equation
(ii) Ore is converted in to oxide.

ou
(a) Λ°NH 4 OH = Λ°Ba(OH) 2 + Λ°NH 4 Cl − Λ°BaCl 2
(iii) Changes like oxidation, chlorination, etc.
can take place. (b) Λ°NH 4 OH = Λ°BaCl 2 + Λ°NH 4 Cl − Λ°Ba(OH) 2
(a) (i) and (ii) only
(c) (i) and (iii) only
(b) (ii) and (iii) only
(d) (i), (ii) and (iii) (Y (c) Λ°NH 4 OH =
Λ°Ba(OH) 2 + 2 Λ°NH 4 Cl − Λ°BaCl 2
2
Λ°NH 4 Cl + Λ°Ba(OH) 2
on
69. The movement of dispersion medium under (d) Λ°NH 4 Cl =
the influence of electric field is known as 2
(a) electrodialysis (b) electrophoresis
74. Halogen acids react with alcohols to form
pi

(c) electroosmosis (d) cataphoresis


alkyl halides. The reaction follows a
70. The half-life period of a first order reaction is nucleophilic substitution mechanism. What
m

10 min, what percentage of the reaction will will be the major product of the following
be completed in 100 min? reaction
ha

(a) 25% (b) 50% (c) 75% (d) 99.9% CH 3  CH  CH CH 3 + HCl →
 
C

Category II (Q. Nos. 71-75) CH 3 OH


Carry 2 marks each and only one option is correct. (a) CH3  CH  CH  CH3
 
dy

In case of incorrect answer or any combination of


more than one answer, 1/2 mark will be deducted. CH3 Cl
(b) CH3  CH  CH  CH3
 
u

71. The graph of the effect of catalyst on


activation energy is given below. Choose the Cl CH3
St

blanks X and Y with appropriate statements. CH3



(c) CH3  C  CH2  CH3

Cl
Potential energy

R Y (d) CH3  CH2  CH2  CH2  CH2  Cl


X
75. 3 moles of P and 2 moles of Q are mixed.
P What will be the total vapour-pressure in the
solution. If their partial vapour pressures are
Reaction coordinate 80 and 60 torr respectively.
(a) X = energy of activation without catalyst, (a) 80 torr (b) 140 torr
Y = energy of activation with catalyst (c) 72 torr (d) 70 torr
26 WB JEE (Engineering) Practice Set 2

Category III (Q. Nos. 76-80) 78. Which of the following has same
hybridisation of the central atom?
Carry 2 marks each and one or more option(s)
is/are correct. If all correct answers are not (a) H2SO 4 (b) NH3 (c) H2O (d) PCl 5
marked and also no incorrect answer is marked 79. Which of the following solution show same
then score = 2 × number of correct answers number of moles of solute in the given
marked ÷ actual number of correct answers. If any solution?
wrong option is marked or if any combination (a) 100 mL solution of 0.1 M HCl
(b) 50 mL solution of 0.2 M NaOH

)
including a wrong option is marked, the answer

be
will considered wrong, but there is no negative (c) 200 mL solution of 0.01 M HCl
marking for the same and zero mark will be (d) 75 mL solution of 0.025 M NaOH
awarded. NH2

Tu
76. Which of the following has same oxidation 80. on reaction with HNO 2 forms
number for the central atom?

ou
(a) KMnO 4 (b) KClO 3 NH2 OH
(c) K 2Cr2O 7 (d) CrO 5 (a) (b)
NO2

(Y
77. Which of the following are used to convert
R CHO into RCH 2OH ? OH
(a) H2 /Pd
(b) LiAlH4
on
(c) NaBH4 (c) (d)
(d) Reaction with RMgX followed by hydrolysis HO
pi

Mathematics
m
ha

Category I (Q. Nos. 1 to 50) π


3. If sin −1 x − cos −1 x = , then x is
Only one answer is correct. Correct answer will 6
C

fetch full marks 1. Incorrect answer or any 1 3 1


(a) (b) (c) (d) 2
combination of more than one answer will fetch 2 2 2
−1/4 marks.
dy

π /4

∫ (tan x + tan n − 2 x)
n
4. The value of
| x|
3 3
 x 
1. lim  −    , a > 0 where [ x ] denotes 0

 a  
u

x→a  a
  [ x ] [ x ]2 [ x ]3 
d x − + − + …
St

the greatest integer less than or equal to x, is  1! 2! 3! 


(a) a2 − 3 (b) a2 − 1
(c) a 2
(d) None of these where [ x ] is greatest integer function, is
1 1 1 1
x2 (a) (b) (c) (d)
2. Let f (x) = ∫ (1 + x 2){1 + 1 + x2}
dx and n n+2 n−1 n−2
x
1 − x 
f (0) = 0 . Then, f ()
1 is equal to 5. The function f (x) = ∫ log   dx is
π 1 + x 
(a) loge (1 + 2) (b) loge (1 + 2) − 0
4 (a) an even function (b) an odd function
π
(c) loge (1 + 2) + (d) None of these (c) a periodic function (d) None of these
4
WB JEE (Engineering) Practice Set 2 27

π π 12. If an error of 1° is made is measuring the


6. If f (x) is an integrable function on  ,  and
6 3  angle of a sector of radius 30 cm, then the
π /3 approximate error in its area is
I1 = ∫ sec θ f (2sin2θ)dθ and
2
(a) 450 cm 2 (b) 25π cm 2
π /6 (c) 2.5 π cm 2 (d) None of these
π /3
I2 = ∫ cosec θf (2sin2θ)dθ, then 13. If the sides of a triangle are in GP and its
2

π /6
larger angle is twice the smallest, then the
common ratio r satisfies the inequality

)
(a) I1 = 2 I2 (b) I1 = 3I2

be
(a) 0 < r < 2 (b) 1 < r < 2
(c) 2 I1 = I2 (d) None of these (c) 1 < r < 2 (d) None of these

14. If 9 log 3(log 2 x ) = log 2 x − (log 2 x)2 + 1, then x is

Tu
7. The value of ∫|cos x − sin x|dx is
0 equal to
1
1 (a) 1 (b)

ou
(a) (b) 2 2 2
2
2 (c) 3 (d) None of these
(c) (d) 4 2

(Y
2 15. Let Z1 and Z 2 be nth roots of unity which
subtend a right angle at the origin. Then, n
8. The value must be of the form
2 −2 
 1 + x  1 − x  (a) 4K + 1 (b) 4K + 2
on
∫  p ln1 − x  + q ln1 + x  + r dx depends

(c) 4K + 3 (d) 4K
−2
16. If z is a complex number satisfying
pi

on the value of
(a) p (b) q |z|2 − |z| − 2 < 0 , then the value of|z 2 + z sin θ|,
for all value of θ is
m

(c) r (d) p and q


(a) equal to 4 (b) equal to 6
9. Solution of the differential equation
ha

(c) more than 6 (d) less than 6


2
 dy  dy 17. If the equation ax 2 + bx + 6 = 0 has real roots,
x   + 2 xy + y = 0 is
 dx  dx where a ∈ R, b ∈ R, then the greatest value of
C

(a) x + y = a (b) x− y=a 3a + b, is


(c) x2 + y2 = a2 (d) x+ y= a (a) 4 (b) −1
dy

(c) −2 (d) 1
10. The general solution of 18. There are 3 sections in a question paper each
3
ydx − xdy − 3 x 2 y 2 e x dx = 0 is equal to
u

containing 5 questions. A candidate has to


x 3 y 3 solve only 5 questions, choosing at least one
St

(a) = e x + c (b) = e x + c
y x question from each section. In how many
3 ways can be make his choice?
(c) xy = e x + c (d) xy = e x + c
(a) 15 C 5 (b) 3 C1 × 12
C4
11. If a hyperbola passing through the origin has (c) 2250 (d) 2253
3 x − 4 y − 1 = 0 and 4 x − 3 y − 6 = 0 as its
asymptotes then the equations of its 19. The number of ways in which 7 plus (+) signs
transverse and conjugate axes are and 5 minus (−) sign can be arranged in a row
so that no two minus signs are together, is
(a) x − y − 5 = 0 and x + y + 1 = 0
(a) 8 C 5 × 5! (b) 7C 5 × 5!
(b) x − y = 0 and x + y + 5 = 0
8
(c) x + y − 5 = 0 and x − y − 1 = 0 (c) C 5 (d) 7!
(d) x + y − 1 = 0 and x − y − 5 = 0
28 WB JEE (Engineering) Practice Set 2

20. If 49 n + 16 n + λ is divisible by 64 for all (c) onto but not one-to-one


n ∈ N , then the least negative integral value (d) neither one-to-one nor onto
of λ is 28. Let the relation R in the set N of natural
(a) −2 (b) −1 (c) −3 (d) −4 numbers be defined as (x , y) ∈ R if
21. The number of terms which are free from x 2 − 4 xy + 3 y 2 = 0 for all x , y ∈ N . The
radical signs in the expansion of relation R is
(a) reflexive and transitive
(y1 / 5 + x 1 / 10)55 is
(b) reflexive symmetric

)
(a) 5 (b) 6 (c) symmetric and transitive

be
(c) 7 (d) None of these (d) on equivalence relation
 i 1 − 2 i 29. Two numbers are selected randomly from the
22. The matrix A =  is which of
0  set S = {1, 2, 3, 4, 5, 6} without replacement

Tu
−1 − 2 i

one by one. The probability that the minimum
the following?
of the two numbers is less than 4 is
(a) Symmetric (b) Skew-symmetric

ou
1 14 1 4
(c) Hermitian (d) Skew-hermitian (a) (b) (c) (d)
15 15 5 5
23. If A and B are square matrices of size n × n 30. If x follows a binomial distribution with
such that A 2 − B2 = (A − B)(A + B), then
which of the following is true?
(a) A = B
(Y equals
1
parameters n = 8 and p = , then p(| x − 4| ≤ 2)
2
on
(b) AB = BA 118 119
(c) Either A as B is a zero matrix (a) (b)
128 128
(d) Either A or B is an identity matrix 117
pi

(c) (d) None of these


 0 0 −1 128
24. Let A =  0 −1 0 . The only correct
m

  31. The number of real solutions of


−1 0 0  1 + |e x − 1| = e x (e x − 2), is
ha

statement about the matrix A is (a) 0 (b) 1 (c) 2 (d) 4


(a) A −1 does not exist
(b) A = (−1) I is a unit matrix
C

32. The area of a triangle ABC is 3 sq units and


(c) A is a zero matrix ∠B = 60 °. If a 2 , b2 , c 2 are in AP, the length of
(d) A 2 = I
dy

the side AC is
25. In a 4 × 4 matrix the sum of each raw, (a) 2 3 units (b) 2 units
column and both the main diagonal is α. (c) 3 units (d) 3 3 units
u

Then the sum of the four corner elements


33. The lines x + y =|a|and ax − y = 1 intersect
St

(a) is also α (b) may not be α


(c) is never equal to α (d) None of these each other in the first quadrant. Then the set
of all possible values of a is the interval
26. Let P = {θ : sin θ − cos θ = 2 cos θ} and (a) [1, ∞ ) (b) (−1, ∞ )
Q = {θ : sin θ + cos θ = 2 sin θ} be two sets. (c) (−1, 1) (d) (0, ∞ )
Then,
(a) p ⊂ Q and Q − P = φ (b) θ ⊄ P 34. Let A(2, − 3) and B(−2, 1) be vertices of a
(c) P ⊄ Q (d) P = Q triangle ABC. If the centroid of this triangle
moves on the line 2 x + 3 y = 1, then the locus
27. Let the function f : R → R be defined by
of the vertex c is the line
f (x) = 2 x + sin x . Then, f is
(a) 3 x − 2 y = 3 (b) 2 x − 3 y = 7
(a) one-to-one and onto
(c) 3 x + 2 y = 5 (d) 2 x + 3 y = 9
(b) one-to-one but not onto
WB JEE (Engineering) Practice Set 2 29

35. The Locus of the orthocentre of the triangle (a) x + ey + e 3 a = 0 (b) x − ey − e 3 a = 0


formed by the lines (1 + p) x − py + p(1 + p) = 0 , (c) x − ey − e a = 0
2
(d) None of these
(1 + q) x − qy + q(1 + q) = 0 and y = 0, where
41. The equations of the tangents to the
p ≠ q, is
hyperbola 3 x 2 − y 2 = 3 which are
(a) hyperbola (b) a parabola
perpendicular to the line x + 3 y = 2 are
(c) an ellipse (d) a straight line
(a) y = 3 x ± 6 (b) y = 3 x ± 2 3
36. If the angle between the pair of straight lines (c) y = 3 x ± 3 (d) None of these

)
represented by the equation
x2 y2

be
x 2 − 3 xy + λy 2 + 3 x − 5 y + 2 = 0 , is tan −1 3, 42. Tangent at a point on the ellipse =1 2
+
a b2
where ‘λ’ is a non-negative real number.
is drawn which cuts the coordinate axes at A
Then, λ =

Tu
and B. The minimum area of the triangle OAB
(a) 2 (b) 0 (c) 3 (d) 1 is (O being origin)
a3 + b 3 + ab

ou
37. If a variable line drawn through the point of (a) ab (b)
x y 3
intersection of straight lines + = 1 and a2 + b 2
α β (c) a2 + b 2 (d)
4
x y
β α
+ = 1 meets the coordinate axes in A and

B, then the locus of the mid-point of AB is


(Y
43. The acute angle between the lines whose
direction ratios are given by l + m − n = 0 and
on
l 2 + m 2 − n2 = 0 , is
(a) αβ( x + y) = xy(α + β ) π π π
(a) 0 (b) (c) (d)
(b) αβ( x + y) = 2 xy(α + β ) 6 4 3
pi

(c) (α + β )( x + y) = 2αβxy
(d) None of the above 44. Equation of the line passing through (1, 1, 1)
m

and parallel to the plane 2 x + 3 y + z + 5 = 0 ,


38. The limiting points of the coaxial system is
ha

containing the two circles x−1 y−1 z−1 x−1 y−1 z−1
(a) = = (b) = =
x 2 + y 2 + 2 x − 2 y + 2 = 0 and 1 2 1 −1 1 −1
x−1 y−1 z−1 x−1 y−1 z−1
25(x 2 + y 2) − 10 x − 80 y + 65 = 0 are (c) = = (d) = =
C

3 2 1 2 3 1
(b) (1, − 1),  − , − 
1 8
(a) (1, − 1), (−5, 40) {(a − n)nx − tan x }sin nx
 5 5 45. If lim = 0, where n is
dy

x→0 x2
(c) (−1, 1),  ,  (d) (−1, 1),  − , − 
1 8 1 8
 5 5  5 5 a non-zero real number, then a is equal to
n+1 1
u

(a) 0 (b) (c) n (d) n +


39. If H (x , y) = 0 represents the equation of a n n
St

hyperbola and A(x , y) = 0,C (x , y) = 0 the joint


e t f (x) − e x f ()
t 1
equation of its asymptotes and the conjugate 46. If lim = 2 and f (0) = , then
t→ x (t − x)( f (x)) 2
2
hyperbola respectively, then for any point
(α , β) in the plane, H(α , β), A(α , β) and C(α , β) f ′ (0) =
are in (a) 4 (b) 2 (c) 0 (d) 1
(a) AP (b) GP
(c) HP (d) None of these 47. Consider the function f (x) = sin x + { x }, then
(a) f is of period 2 π
40. The equation of the normal to the ellipse
2 2 (b) f is of period π
x y
2
+= 1 at the positive end of the (c) f is not periodic
a b2 π
(d) f is of period
latusrectum is 2
30 WB JEE (Engineering) Practice Set 2
 sin x 
  55. If a and b are vectors in space given by
 sin x   x − sin x 
48. lim   is equal to $i − 2 $j 2 $i + $j + 3 k$
x → 0 x  a= and b = , then the value
1
5 14
(a) e (b) (c) 1 (d) 0 of (2a + b) ⋅[(a × b) × (a − 2 b)] is
e
(a) 4 (b) 5 (c) 6 (d) 7
49. If f (x) = x + tan x and f is the inverse of g,
  1 
then g ′(x) equals 56. The minimum value ofarg    for|z| = 1,
1 1  1 − z  

)
(a) (b)
1 + [g ( x) − x]2 2 − [g ( x) − x]2 z ≠ 1 is

be
1 π π π
(c) (d) None of these (a) (b) (c) (d) π
2 + [ g ( x) − x] 2 6 3 2

Tu
50. If f (x) and g(x) are differentiable function for 57. Consider the system of linear equations
0 ≤ x ≤ 1 such that f (0) = 10 , g(0) = 2, f ()
1 = 2, x1 + 2 x 2 + x 3 = 3; 2 x1 + 3 x 2 + x 3 = 3
1 = 4 , then in the interval (0, 1)
g() 3 x1 + 5 x 2 + 2 x 3 = 1

ou
(a) f ′( x) = 0 for all x
The system has
(b) f ′( x) + 4g ′( x) = 0 for atleast one x
(c) f ′( x) = 2 g ′( x) for atmost one x (a) a unique solution
(d) None of the above

Category II (Q. Nos. 51 to 65)


(Y (b) no solution
(c) infinite number of solutions
(d) exactly three solutions
on
Carry 2 marks each and only one option is correct. 58. The value of parameter α, for which the
In case of incorrect answer or any combination of function f : R → R given by f (x) = 1 + αx ,
α ≠ 0 is the inverse of itself is
pi

more than one answer, 1/2 mark will be deducted.


(a) −2 (b) −1 (c) 1 (d) 2
n r
m

51. The value of ∑ ∑ n


C r ⋅ C p is equal to
r
59. The function f :[0 , 3] → [1, 29], defined by
r=0 p=0
f (x) = 2 x 3 − 15 x 2 + 36 x + 1, is
ha

(a) 3n − 2 n (b) 3n − 2 n − 2
(a) one-one and onto
(c) 3n − 2 n + 2 (d) None of these
(b) onto but not one-one
C

52. The coefficient of x 20 in the expansion of (c) one-one but not onto
−5 (d) neither one-one nor onto
 1
(1 + x 2)40  x 2 + 2 + 2  is
dy

 x  60. If the line passing through the points (5, 1, a)


(a) 30
C10 (b) 30
C 25 and (3 , b, 1) crosses the yz-plane at the point
 17 13 
u

(c) 1 (d) None of these 0 , , −  , then


 2 2
St

53. If a = i$ + $j + k$ , a ⋅ b = 1 and a × b = $j − k$ ,
(a) a = 6, b = 4 (b) a = 8,b = 2
then b (c) a = 2, b = 8 (d) a = 4,b = 6
(a) $i − $j + k$ (b) 2 $j − k$
(c) $i (d) 2 $i 61. The coordinates of the point of intersection
of tangents drawn to the hyperbola
54. If θ is the angle between the line x 2 y2
− = 1 at the points where it is
r = (i$ + 2 $j − k$) + λ ($i − $j + k$) a 2 b2
and the plane r ⋅ (2 $i − $j + k$) = 4, then cosθ = intersected by the line lx + my + n = 0 are
 a2 l b2 m  a2 l b 2 m 
1 2 2 (a)  ,−  (b)  − , 
(a) (b)  n n   n n 
2 2 3
1  a2 l b 2 m   a2 l b2 m
(c) (d) None of these (c)  ,  (d)  − ,− 
3  n n   n n 
WB JEE (Engineering) Practice Set 2 31

1 67. If S is the area of the region enclosed by


(1 − cos 2 x) 2
62. The value of lim 2 is y = e − x , y = 0, x = 0 and x = 1. Then
x→0 x 1
(a) S ≥
(a) 1 (b) −1 e
(c) 0 (d) None of these 1
(b) S ≥ 1 −
e
63. Let f (x) = sin x , g(x) = [ x + 1] and h(x) = gof (x) 1 1 
(c) S ≤ 1 + 
where [⋅] is the greatest integer function. 4 e

)
 π 1 1  1 
Then, h′   is (d) S ≤ + 1 −

be
 2 
2 e 2
(a) 1 (b) −1
68. The vector(s) which is/are coplanar with

Tu
(c) non-existent (d) None of these
vectors $i + $j + 2 k$ and $i + 2 $j + k$ are
x
64. The function f (x) = cos + { x }, where { x } = perpendicular to the vector i$ + $j + k$ is/are
2

ou
the fractional part of x is a (a) $j − k$ (b) − $i + $j
(a) periodic function with period 4π (c) $i − $j (d) − $j + k$
(b) periodic function with period 1
69. Let h(x) = f (x) − { f (x)} 2 + { f (x)} 3 for every
(c) periodic function with interminate period
(d) None of the above

65. A particle’s velocity V at time t is given by


(Y real number x. Then,
(a) h is increasing whenever f is increasing
on
(b) h is increasing whenever f is decreasing
πt
v = 2 e 2 t cos
. The least value of t at which (c) h is increasing whenever f is decreasing
3 (d) nothing can be said in general
pi

the acceleration becomes zero is


3 70. Let z1 and z 2be complex numbers such that
(a) 0 (b)
z1 ≠ z 2 and|z1| = |z 2| . If z1 has positive real
m

2
tan−1   (d) cot −1  
(c)
3 6 3 6 part and z 2 has negative imaginary part, then
π  π π  π z1 + z 2
ha

may be
z1 − z 2
Category III (Q. Nos. 66 to 75) (a) zero
C

Carry 2 marks each and one or more option(s) (b) real and positive
is/are correct. If all correct answers are not (c) real and negative
dy

marked and also no incorrect answer is marked (d) purely imaginary


then score = 2 × number of correct answers 1 a b
marked ÷ actual number of correct answer. If any 71. In a ∆ABC, if1 c a = 0, then
u

wrong option is marked or if, any combination  


1 b c
St

including a wrong option is marked, the answer


will be considered wrong, but there is no negative sin 2 A + sin 2 B + sin 2 C is
marking for the same and zero marks will be 3 3 9 5
(a) (b) (c) (d) 2
awarded. 2 4 4

66. For a ∈ R (the set of all real numbers),a ≠ −1 , 72. The locus of the point of intersection of the
(1 + 2 + … + n )
a a a
1 lines x cos α + y sin α = a and
lim a −1
= x sin α − y cos α = b, where α is a variable is
n→ ∞ (n + 1) [(na + 1) + (na + 2) + 60
(a) x2 + y2 = a2 − b 2
… (na + n)] (b) x2 − y2 = a2 − b 2
Then, a is equal to (c) x2 + y2 = a2 + b 2
15 17
(a) 5 (b) 7 (c) − (d) − (d) None of the above
2 2
32 WB JEE (Engineering) Practice Set 2
2
73. The locus of a point p(α , β) having under the (a) 2 (b)
3
condition that the line y = α x + β is a tangent 1
(c) (d) None of these
x 2 y2 2
to the hyperbola 2 − 2 = 1 is
a b
75. The equation sin x + x cos x = 0 has at least
(a) a hyperbola (b) a parabola
(c) a circle (d) an ellipse one root in the interval
π
(a)  − , 0 (b) (0, π )
74. Let f and g be differentiable functions  2 

)
satisfying g ′ (a) = 2, g(a) = b and fog = I π π
(c)  − , 

be
(d) None of these
(Identity function). The f ′(b) is equal to  2 2

Answers

Tu
ou
Physics
1. (a) 2. (a) 3. (c) 4. (c) 5. (a) 6. (a) 7. (b) 8. (d) 9. (a) 10. (d)

(Y
11. (a) 12. (c) 13. (b) 14. (a) 15. (a) 16. (a) 17. (c) 18. (d) 19. (a) 20. (c)
21. (d) 22. (a) 23. (b) 24. (c) 25. (c) 26. (b) 27. (a) 28. (a) 29. (a) 30. (b)
31. (b) 32. (a) 33. (c) 34. (b) 35. (a) 36. (a, c) 37. (b, c) 38. (a, c, d) 39. (a, b) 40. (c)
on
Chemistry
41. (b) 42. (b) 43. (b) 44. (d) 45. (a) 46. (a) 47. (c) 48. (a) 49. (d) 50. (d)
pi

51. (a) 52. (d) 53. (c) 54. (a) 55. (c) 56. (c) 57. (b) 58. (a) 59. (c) 60. (b)
61. (d) 62. (b) 63. (a) 64. (c) 65. (d) 66. (b) 67. (b) 68. (d) 69. (c) 70. (d)
m

71. (c) 72. (b) 73. (c) 74. (c) 75. (c) 76. (c, d) 77. (a, b, c) 78. (a, b, c) 79. (a, b) 80. (b, c, d)
ha

Mathematics
1. (c) 2. (b) 3. (b) 4. (c) 5. (a) 6. (d) 7. (d) 8. (c) 9. (d) 10. (a)
C

11. (c) 12. (c) 13. (b) 14. (d) 15. (d) 16. (d) 17. (c) 18. (c) 19. (c) 20. (b)
21. (b) 22. (d) 23. (b) 24. (d) 25. (a) 26. (d) 27. (a) 28. (a) 29. (d) 30. (b)
31. (b) 32. (b) 33. (a) 34. (d) 35. (d) 36. (b) 37. (b) 38. (c) 39. (a) 40. (b)
dy

41. (a) 42. (a) 43. (d) 44. (b) 45. (d) 46. (c) 47. (c) 48. (b) 49. (c) 50. (b)
51. (d) 52. (b) 53. (c) 54. (c) 55. (b) 56. (c) 57. (b) 58. (b) 59. (b) 60. (a)
u

61. (b) 62. (d) 63. (c) 64. (d) 65. (c) 66. (b, d) 67. (b, d) 68. (a, d) 69. (b,c) 70. (a, d)
St

71. (b) 72. (c) 73. (a) 74. (c) 75. (b)

* For detailed solutions visit http://tinyurl.com/y5h46qkb or scan


WB JEE
Engineering Entrance Exam

Practice Set 3

)
be
Tu
Physics

ou
Category I (Q. Nos. 1 to 30) 4. An electron of a stationary hydrogen atom
Carry 1 marks each and only one option is passes from the fifth energy-level of the
correct. In case of incorrect answer or any
combination of more than one answer, 1/4 mark
will be deducted.
(Y ground level. The velocity, that the atom
acquired as a result of photon emission will
be (m is the mass of the electron, R = Rydberg
on
constant and h = Planck’s constant)
1. The angle of minimum deviation for a glass 25 hR 24 m 24 hR 25 hR
(a) ⋅ (b) ⋅ (c) ⋅ (d) ⋅
prism with µ = 3 equals the refracting angle 24 m 25 hR 25 m 24 m
pi

of the prism. What is the angle of the prism? 5. A Zener diode, having breakdown voltage equal
m

(a) 60° (b) 30° to 15 V is used in a voltage regulator circuit


(c) 45° (d) 90° shown in figure. The current through the
ha

diode is
2. In the chemical analysis of a rock, the mass
ratio of two radioactive isotopes is found to +
250 Ω
be 100 : 1. The mean times of the two
C

isotopes are 4 × 10 9 yr and 2 × 10 9 yr


20 V Z =15 V 1k Ω
respectively. If it is assumed that at the time
dy

of formation, the atoms of both the isotopes


were is equal proportion. Calculate the age of –
the rock. Ratio of the atomic weight of the
u

two isotopes is 1.02 : 1. (a) 15 mA (b) 10 mA


St

(c) 5 mA (d) 20 mA
(a) 1834
. × 10 yr
10

. × 1010 yr
(b) 184 6. The circuit
. × 1010 yr
(c) 181 NOR NAND NOT
. × 1010 yr
(d) 18

3. The wavelength associated with an electron


accelerated through a potential difference of is equivalent to
100 V, is of the order of (a) AND gate (b) NAND gate
(a) 1.2 Å (b) 100 Å (c) NOR gate (d) OR gate
(c) 10.5 Å (d) 1000 Å
34 WB JEE (Engineering) Practice Set 3

7. The dimensional formula of torque is 13. A 5 cm length of the cube has its upper face
(a) [ML−2 T−2 ] (b) [ML2 T−2 ] displaced by 0.2 cm by a tangential force of
(c) [MLT−2 ] (d) [ML−1T−2 ] 8 N. The modulus of rigidity of the material
of cube is
8. The length of a simple pendulum is about (a) 5 × 104 Nm−2 (b) 6 × 104 Nm−2
100 cm known to an accuracy of 1 mm. Its
(c) 7 × 104 Nm−2 (d) 8 × 104 Nm−2
period of oscillation is 2 s determined by
measuring the time for 100 oscillations using 14. A square plate of 10 cm side moves parallel to
a clock of 0.1 s resolution. What is the

)
another plate with a velocity of 10 cm s −1 ;

be
accuracy in the determined of g?
both the plates immersed in water. If the
(a) 2% (b) 0.5% (c) 0.1% (d) 0.2%
viscous force is 200 dyne and viscosity of
9. The pulleys and strings shown in figure are water is 0.01 poise, what is their separation

Tu
smooth and of negligible mass. For the distance?
system to the under equilibrium, the angle (a) 0.05 cm (b)1cm (c) 0.07 cm (d) 7 cm
θ should be

ou
15. In the above problem, if the sun radiates as
an ideal black body, what is the temperature
of its surface?

(Y (a) 6803 K
(c) 5803 K
(b) 5603 K
(d) 5503 K
on
m2 16. At what temperature does the average
translation kinetic energy of a molecule in a
gas becomes equal to the kinetic energy of a
pi

electron accelerated from rest through a


m m potential difference of one volt?
m

(k = 1.38 × 10 –23 JK −1)


(a) 0° (b) 60° (c) 45° (d) 30°
(a) 7330 K (b) 7730 K (c) 7530 K (d) 7430 K
ha

10. A ball is thrown upwards with a speed u


from a height h above the ground. The time 17. Two containers of equal volume contained
taken by the ball to hit the ground is the same gas at the pressures p1 and p 2 and
C

2h 8h absolute temperatures T1 and T2 , respectively.


(a) (b) On joining the vessels, the gas reaches a
g g
common pressure p and a common
dy

u 2 + 2 gh u 2h temperature T. The ratio p/T is equal to


(c) (d) +
g g g
(a) 1 1 + p2T2
pT
(b) 1 2 + p2T1
pT
u

T1 × T2 T1 + T2
11. Two stones are projected with the same
1  pT + p2T1  1 2 − p2T1
St

pT
velocity but making different angles with the (c)  1 2  (d)
horizontal. Their ranges are equal. If angle of 2  TT
1 2  T1 × T2
projection of one is 30° and its maximum
height is Y , then the maximum height of 18. A 10 µF capacitor and a 20 µF capacitor are
other stone will be connected in series across 200 V supply line.
(a) 3Y (b) 2Y (c) Y / 2 (d) Y / 3 The charged capacitors are then disconnected
from the line and reconnected with their
12. The time period of moon’s revolution is positive plates together and negative plates
27.3 days and radius of the earth is together and no external voltage is applied.
6.37 × 10 6 m, distance to the moon is What is the potential difference across each
3.84 × 10 8 m, then the mass of the earth is capacitor?
(approximately) 800 800
(a) V (b) V (c) 400 V (d) 200 V
(a) 10 24
kg 16
(b) 10 kg 16
(c) 10 kg 5
(d) 10 kg 9 3
WB JEE (Engineering) Practice Set 3 35

19. Equal charges q each are placed at the magnitude of magnetic field induction at the
vertices A and B of an equilateral triangle centre of the loop is
ABC of side a. The magnitude of the electric i
field intensity at the point C is A C B

(a)
1

q
(b)
1 q 2
⋅ r θ
4 πε0 a2 4 πε0 a2
O
1 q 3 1 2q
(c) ⋅ (d) ⋅
4 πε0 a2 4 πε0 a2

)
D

be
20. Two point charges exerted on each other a µ 0 iθ µ0 i
(a) ⋅ (b) ⋅ ( π − θ)
force F when they are placed r distance apart 3π r 2π r
µ i

Tu
in air. If they are placed R distance apart in a (c) 0 ⋅ (2 π − θ) (d) zero
medium of dielectric constant K, they exert 2π r
the same force. The distance R equals
25. If each of the resistances in the network in

ou
r r
(a) (b) rK (c) r K (d) figure R, the equivalent resistance between
K K terminals A and B (in ohm) is

(Y
21. A current i flowing through i Q
the loop as shown in figure. O 2r
The magnetic field at the r
RΩ RΩ RΩ
on
centre ‘O’ is
7µ 0i O
(a) acting downwards i
12 r
A
pi

5µ 0 i RΩ B
(b) acting upwards
12 r P S
RΩ
m

7µ 0i
(c) acting upwards
12 r (a) R (b) 2R (c) 3R (d) 5R
5µ 0 i
ha

(d) acting downwards 26. Two cells with the same emf E and different
12 r
internal resistances r1 and r2 are connected in
series to an external resistance R. Can a value
C

22. An electron is revolving around a proton in a


circular path of diameter 0.1 nm. It produces for R be selected such that the potential
a magnetic field 14 T at a proton. Then the difference (PD) at the first cell should be
dy

angular speed of the electron is zero?


(a) 8.8 × 106 rad s −1 (b) 4.4 × 1016 rad s −1 E E
– + – +
(c) 2.2 × 1016 rad s −1 (d) 1.1 × 1016 rad s −1
u

r1 r2
St

23. A straight wire of length 30 cm and mass i


60 mg lies in a direction 30° East to North.
The earth’s magnetic field applied in R
horizontal direction and has a magnitude of
0.8 G. What current must be passed through r + r
(a) r1r2 (b) 1 2 (c) r1 + r2 (d) r1 − r2
the wire, so that it may float in air? 2
(a) 50 A (b) 40 A
27. In a series resonant L-C-R circuit, the voltage
(c) 20 A (d) 10 A
across R is 100 V and resistance R = 1 kΩ with
24. Equal current i flows in two segments of a capacitance C = 2 µF. The resonant frequency
circular loop in the direction shown in figure. ω is 200 rad/s. At resonance, the voltage
The radius of the circular loop is r. The across L is
36 WB JEE (Engineering) Practice Set 3
(a) 2.5 × 10−2 V (b) 40 V 32. The thickness of ice on a lake is 10 cm and
(c) 250 V (d) 4 × 10−3 V the temperature of air is − 10 °C. If rate of
cooling of water inside lake is 20000 cal min −1
28. The focal length of a lens in air is 20 cm through each square metre surface, then K
when we put it in a liquid, it becomes for ice (in Wm −1 °C −1 ) is
disappear. What is the refractive index of the
liquid? (R1 = R2 = 10 cm) (a) 14 (b) 10 (c) 3 (d) 4
4 4 5 3 33. A lead bullet strikes against a steel armour
(a) (b) (c) (d)

)
3 5 4 4 plate with a velocity of 480 m/s. If the bullet

be
falls dead after impact, find the rise in it’s
29. Two slits in Young’s experiment are 0.003 cm
temperature, assuming that the heat produced
apart. The interference fringes for light of
is equally shared between it and the target.

Tu
wavelength 7000 Å are formed on a screen
Specific heat of lead is 0.03 cal/g°C.
90 cm away. Calculate the distance of the
fifth bright fringe. (a) 457K (b) 400K (c) 500K (d) 600K

ou
(a) 12 cm (b) 10.5 cm 34. OABC is a current carrying square loop, an
(c) 11.5 cm (d) 20 cm electron is projected from the centre of loop
along its diagonal AC as shown in the figure.
30. Red light of wavelength 5400 Å from a
distant source falls on a slit 0.80 mm wide.
Calculate the distance between first two dark
bands on each side of the central bright band
(Y Unit vector in the direction of initial
acceleration will be
Y
on
in the diffraction pattern observed on a
screen place 1.4 m from the slit. e–
A B
pi

(a) 1.89 mm (b) 4 mm (c) 1 mm (d) 3 mm V


m

Category II (Q. Nos. 31 to 35)


Carry 2 marks each and only one option is X
ha

O C
correct. In case of incorrect answer or any
combination of more than one answer, 1/2 mark  $i + $j  $i + $j
(a) k$ (b) −  
 (c) − k$ (d)
will be deducted.  2  2
C

31. In the circuit shown here, the point C is kept 35. In a series L-C circuit, L = 4 H and C = 25 µF.
dy

connected to a point A till the current


If the frequency is twice of resonant
flowing through the circuit becomes
frequency, the net reactance of circuit is
constant. Afterward, suddenly point C is
(b) 100 Ω (c) 200 Ω (d) 600 Ω
u

disconnected from point A and connected to (a) zero


point B at time t = 0. Ratio of the voltage
St

across resistance R and the inductor L at Category III (Q. Nos. 36 to 40)
t = L / R will be equal to Carry 2 marks each and one or more option(s)
A C R is/are correct. If all correct answers are not marked
and also no incorrect answer is marked then score
= 2 × number of correct answers marked ÷ actual
L
number of correct answers. If any wrong option is
B marked or if any combination including a wrong
option is marked, the answer will be considered
wrong, but there is no negative marking for the
e 1− e same and zero marks will be awarded.
(a) (b) 1 (c) − 1 (d)
1− e e
WB JEE (Engineering) Practice Set 3 37

36. Relativistic corrections become necessary


when the expression for the kinetic energy
1 mv 2 , becomes comparable with mc 2 , where R′
A B
2 R
m is the mass of the particle. At what
de-Broglie wavelength will relativistic
corrections become important for an V r
electron?

)
(a) λ = 10nm (b) λ = 10−1 nm (a) Potential drop across AB is nearly constant as

be
−4 −6
R′ is varied.
(c) λ = 10 nm (d) λ = 10 nm (b) Current through R′ is nearly a constant as R′ is
varied.
37. Figure shows the p-V diagram of an ideal

Tu
(c) Current i depends sensitively on R′.
gas undergoing a change of state from A to V
B. Four different parts I, II, III and IV as (d) I ≥ always
r+ R
shown in the figure may lead to the same

ou
change of the state. 40. A particle of charge + q and mass m moving
under the influence of a uniform electric field

(Y
A ^ ^
E i and a uniform magnetic field B k follows
p a trajectory from P to Q as shown in figure.
^ ^
The velocities at P and Q are v i and −2 v i,
on
respectively. Which of the following
B statement(s) is/are correct?
C
pi

y
E
m

V
P
(a) ∆QA → B = negative v r
B
ha

(b) ∆UC → A = negative


a
(c) ∆WCAB = negative
(d) ∆U A → B = negative Q
x
C

2a
38. Two projectiles A and B are projected with 2v
the same speed at angles 15° and 75°,
dy

respectively to the maximum and have same 3  µv 2 


(a) E =  
horizontal range. If h be the maximum 4  qa 
height and T total time of the flight of a
u

(b) Rate of work done by the electric field at P is


projectile, then
3  mv 3 
St

(a) hA > hB (b) hA < hB  .


4  a 
(c) TA < TB (d) TA > TB
(c) Rate of work done by the electric field at P is
39. Consider a simple circuit shown in figure zero.
stands for a variable resistance R ′ . R ′ can vary (d) Rate of work done by both the fields at Q is
from R0 to infinity. r is internal resistance of zero.
the battery (r << R << R0).
Chemistry
Category I (Q. Nos. 41 to 70)
(b)CH2 CH2  CH2
Carry 1 marks each and only one option is
 
correct. In case of incorrect answer or any NO Cl
combination of more than one answer, 1/4 mark
(c) CH3  CH  CH2 (d) CH3  CH  CH2
will be deducted.
   

)
NO Cl Cl NO

be
41. Phenol on heating with CHCl 3 and NaOH
gives salicylaldehyde. The reaction is called 45. Which of the following processes is used in
(a) Aldol reaction extractive metallurgy of magnesium?

Tu
(b) Claisen reaction (a) Self reduction
(c) Reimer-Tiemann reaction (b) Aqueous solution electrolysis
(d) Hell Volhard - Zelinsky reaction (c) Thermite reduction

ou
42. One of the product of the following reaction (d) Fused salt electrolysis
is X. 46. Which one of the following benefaction
CH2—CH==CH2

+ HCI X (Y processes is used for the mineral


Al 2O 3 ⋅ 2H 2O?
(a) Liquation (b) Magnetic separation
on
Phenol
(c) Leaching (d) Froth floatation

structure of X is 47. Amongst the following alcohols which would


react fastest with conc. HCl and ZnCl 2 ?
pi

Cl
(a) 2-methyl butanol (b) Pentanol
m

CH—CH2—CH3 CH—CH2—CH3 (c) 2-methyl butan-2-ol (d) 2-pentanol


Cl 48. Which of the following sets of quantum
ha

(a) (b) numbers represents the highest energy of an


atom?
CH—CH—CH3 CH2—CH2—CH2—Cl 1
(a) n = 3, l = 1, m = 1, s = +
C

2
Cl 1
(c) (d) (b) n = 3, l = 0, m = 0, s = +
dy

2
1
(c) n = 3, l = 2, m = 1, s = +
43. The product of which of the following 2
u

1
CH2—CO (d) n = 4, l = 0, m = 0, s = +
St

reaction on dehydration gives O? 2


CH2—CO
49. For the reaction below, the product A and B
KCN H O+
(a) 1, 1-dibromo ethane  →   3
→ respectively are
(b) 1, 1, 1-trichloro ethane  →
Alkaline hydrolysis
NH2
H O+
→  
(c) 1, 2-dibromo ethane KCN 3
→
NaNO2 HBF4
(d) None of the above A B
HCl, 278K
44. CH 3 C == CH 2 + NOCl → P.
(a) nitrobenzene and chlorobenzene
Identify the product P. (b) phenol and benzene
(a) CH3 CH2  CH NO (c) nitrobenzene and fluorobenzene
 (d) benzene diazonium chloride and fluorobenzene
Cl
WB JEE (Engineering) Practice Set 3 39

50. The compressibility factor for a real gas at 57. 1 L of 1.0 M CuSO 4 solution is electrolysed by
high pressure is passing 1.5 F charge. Final molarity of the
(a) 1 +
RT
(b) 1 +
pb
(c) 1 −
pb
(d) 1 solution will be [atomic weight of Cu = 63.5 ]
pb RT RT (a) 0.10 M (b) 0.20 M
(c) 0.25 M (d) 0.30 M
51. 29.5 mg of an organic compound containing
nitrogen was digested according to Kjeldahl’s 58. Potassium permaganate acts as an oxidant in
method and the evolved ammonia was neutral, alkaline as well as acidic medium.
absorbed in 20 mL of 0.1 M HCl solution. The The final products obtained from it in the

)
be
excess of the acid required 15 mL of 0.1 M three conditions are, respectively.
NaOH solution for complete neutralisation. (a) MnO 22 − , Mn3+,Mn2+ (b) MnO, MnO 2− 3+
4 , Mn
The percentage of nitrogen in the compound
(c) MnO 2 , MnO 2 , Mn2+ (d) MnO 2 , MnO 2−
4 , Mn
3+

Tu
is
(a) 59.0 (b) 47.4 (c) 23.7 (d) 29.5 59. The absolute configuration of
H O 2C C O 2H

ou
52. The correct order of increasing basicity of the
given conjugate bases (R == CH 3) is
– – – – HO H H OH
(a) RCOO < HC ≡≡ C < R < NH2

(b) R < HC ≡≡ C < RCOO <NH2
– –
(c) RCOO <N H2 < HC ≡≡ C < R
– – –

– – (Y (a) S, S
(c) R, S
(b) R, R
(d) S, R

60. A gaseous hydrocarbon gives 0.72 g of water


on
– – – –
(d) RCOO <HC ≡≡ C <NH2 < R and 3.08 g of CO 2 upon combustion. The
empirical formula of the hydrocarbon is
53. The two form of d-glucopyranose obtained
pi

(a) C 3 H4 (b) C 6H5


from the solution of d-glucose are called
(c) C 7H8 (d) C 2H4
(a) epimer (b) anomer
m

(c) enantiomer (d) isomer 61. A blue colouration is not obtained when
(a) ferric chloride reacts with potassium
ha

54. The correct stability order for the following ferrocyanide


species is (b) anhydrous CuSO 4 is dissolved in water
+ (c) ammonium hydroxide dissolves in copper
C

+
O sulphate
(I) (II) (d) copper sulphate solution reacts withK 4 [Fe(CN)6 ]
dy

+ 62. On heating benzylamine with chloroform


O and ethanoic KOH, product obtained is
+
u

(III) (IV) (a) benzyl alcohol (b) benzonitrile


(c) benzyl isocyanide (d) benzaldehyde
St

(a) II > IV > I > III (b) I > II > III > IV
(c) II > I > IV > III (d) I > III > II > IV 63. Which of the following complex species is
not expected to exhibit optical isomerism?
55. Increasing value of magnetic moment of
(a) [Co(NH3 )Cl 3 ] (b) [Co(en)(NH3 )Cl 2 ]+
I. [Fe(CN)6 ]4 − II. [Fe(CN)6 ]3 − (c) [Co(en)3 ]3+ (d) [Co(en)2 )Cl 2 ]+
III. [Cr(NH3 )6 ]3 + IV. [Ni(H2O)4 ]2 + is
64. The treatment of CH 3 MgX with
(a) I < II < IV < III (b) I < II < III < IV CH 3C ≡≡ C H, produces
(c) IV < III < II < I (d) II < III < I < IV
(a) CH3 C ≡≡ CH
56. If pH of acid in hydrogen electrode is 10, its (b) CH3 C ≡≡ C CH3
potential will be (c) CH4
(a) 0.59 V (b) 0.059 V (d) CH3 C == C CH3
 
(c) 0.0059 V (d) 0.00059 V H H
40 WB JEE (Engineering) Practice Set 3

65. Rusting follows the following reactions ketone with molecular formula C8H 8O.
1 Which shows positive iodoform test. The
(i) 2H + + O 2 + 2 e − → H 2O; E° = 123
. V structure of (A) is
2
(a) C 2H5COOC 6H5 (b) C 6H5COOC 2H5
(ii) Fe2+ + 2 e − → Fe ; E° = − 0 .44 V (c) H3COCH2COC 6H5 (d) CH3 COOC 2H5
The net work done is 72. The energy required to break one mole of
(a) 152 kJ (b) 322 kJ Cl—Cl bonds in Cl 2 is 242 kJ mol −1 . The
(c) 132 kJ (d) 233 kJ

)
longest wavelength of light capable of

be
66. At pH of 0.1 M solution of following increases in breaking a single Cl—Cl bond is
the order (a) 594 nm (b) 694 nm
(a) NaCN < NH4Cl < NaCl < HCl (c) 600 nm (d) 494 nm

Tu
(b) HCl < NaCl < NaCN < NH4Cl
(c) NaCl < NH4Cl < NaCN < HCl 73. Arrange the following compounds in the
(d) HCl < NH4Cl < NaCl < NaCN order of decreasing acidity.

ou
OH OH OH OH
67. In which of the following molecules, the
van der Waals’ forces is likely to be the most

(Y
important in determining the melting and
boiling points?
(a) H2S (b) Br2 Cl CH3 NO2 OCH3
on
(c) HCl (d) CO (I) (II) (III) (IV)

68. The quantum numbers for the outer electrons of (a) III > I > II > IV (b) IV > III > I > II
an atom are given by
pi

(c) II > IV > I > III (d) I > II > III > IV
1
n = 2, l = 0 , m = 0 , s = + 74. Which one of the following has an optical
m

2 isomer?
(a) hydrogen(b) lithium (c) beryllium (d) boron (a) [Co(en)3 ]3+ (b) [Zn(en)2 ]2+
ha

3+
69. X litres of carbon monoxide is present at S.T.P. It (c) [Co(H2O)4 (en)] (d) [Zn(en)(NH3 )2 ]2+
is completely oxidised to CO 2 . The volume of CO 2
75. The oxidation numbers of phosphorus is
C

formed is 11.207 L at S.T.P. What is the value of


X in litres? Ba(H 2PO 2)2 and xenon in Na 4 XeO 6 are
respectively
(a) 12.414 L (b) 44.828 L
dy

(c) 11.207 L (d) 8.6035 L (a) + 2 and + 6


(b) + 3 and + 4
70. Which of the following salts undergoes anionic (c) + 1 and + 8
u

hydrolysis? (d) − 1 and − 6


St

(a) CuSO 4 (b) FeCl 3


(c) NH4Cl (d) Na 2CO 3 Category III (Q. Nos. 76 to 80)
Carry 2 marks each and one or more option(s)
Category II (Q. Nos. 71 to 75 ) is/are correct. If all correct answers are not
marked and also no incorrect answer is
Carry 2 marks each and only one option is correct. In
case of incorrect answer or any combination of more marked then score = 2 × number of correct
than one answer, 1/2 mark will be deducted. answers marked ÷ actual number of correct
answers. If any wrong option is marked or if
71. An ester (A) with molecular formula, C9H10O 2 any combination including a wrong option is
was treated with excess of CH 3 MgBr and the marked, the answer will considered wrong,
complex so formed was treated with H 2SO 4 to but there is no negative marking for the same
give an olefin (B). Ozonolysis of (B) gave a and zero mark will be awarded.
WB JEE (Engineering) Practice Set 3 41

76. Which of the following statements is/are 78. Which statement among the following is/are
correct about half-life period for a incorrect?
reaction? (a) Value of ∆G ° f cannot be determined
1 (b) Absolute value of heat content of the system
(a) For a zero order reaction t 1/ 2 ∝
a can be easily determined by calorimetery
1 (c) Absolute value of entropy cannot be known
(b) For a second order reaction t 1/ 2 ∝ (d) Absolute value of internal energy cannot be known
a
1 79. Which of the following, the hybrid orbitals of
(c) For third order reaction t 1/ 2 ∝

)
a the central atom have the same s-character?

be
(d) Time taken for 75% completion of a first order (a) XeO 3 (b) CH4 (c) [Ni(CN)4 ]2− (d) Ni(CO)4
reaction is twice to t 1/ 2
80. NaNO 3 is heated in a closed vessel, O 2 is liberated

Tu
77. Which among the following have regular and NaNO 2 is left behind. At equilibrium,
geometry? (a) increasing pressure favours reverse reaction
(a) BF4−

ou
(b) NF3 (b) addition of NaNO 2 favours reverse reaction
(c) BF3 (d) PF3 (c) increasing temperature favours forward reaction
(d) addition of NaNO 2 favours forward reaction

(Y
on
Mathematics
pi

Category I (Q. Nos. 1 to 50) sin2 x


(b) x cos x + sin x − +C
4
m

Only one answer is correct. Correct answer will sin2 x


fetch full marks 1. Incorrect answer or any (c) sin x + x cos x − +C
2
ha

combination of more than one answer will fetch sin2 x


−1/4 marks. (d) cos x + x sin x − +C
2
x2
C

π
a − a2 − x 2 −
1. Let L = lim 4 , a > 0. If L is 2
x 2 cos x
x→0 x 4 4. The value of ∫ 1 + ex
dx is equal to
dy

π
finite, then −
2
1 1
(a) a = 2 (b) a = 1 (c) L = (d) L = π2 π2
−2 +2
u

64 32 (a) (b)
4 4
2. If ∫ f (x) dx = ψ(x), then ∫ x 5 f (x 3) dx is equal to (c) π 2 − e − π / 2 (d) π 2 + e π / 2
St

1 x + y
(a) [ x3 ψ( x3 ) − ∫ x2 ψ( x3 ) dx] + C 2
3 ( ∫ e t dt)2
1 0
(b) x3 ψ( x3 ) − 3∫ x3 ψ ( x3 )dx + C 5. The value of lim is equal to
x→∞ x + y
3 2t2
1
(c) x3 ψ( x3 ) − ∫ x2 ψ( x3 ) dx + C
∫ e dt
0
3
1 (a) 0 (b) 1
(d) [ x3 ψ( x3 ) − ∫ x3 ψ ( x3 ) dx] + C  xcos t 2 
3 ∫ 
(c) lim  0  −1 (d) − 1
3. The value of ∫ (e log x + sin x) cos x dx x→ 0


x 

cos 2 x
(a) x sin x + cos x − +C
4
42 WB JEE (Engineering) Practice Set 3

x4 abtuse and if r denotes the radius of the


6. If ∫ dx = g(x) + c, then g(x) is equal to incircle of the triangle, then
a +x
6 6
r 2 is equal to
1
(a) log x3 − a6 + x 6 (a) 2 (b) 4
3 (c) 3 (d) 6
(b) x3 + a6 + x 6
14. If log a x × log 5 a = log x 5, a ≠ 1, a > 0, then x =
1
(c) log x3 + a6 + x6 (a) a (b) 5,
1
3 5

)
(d) None of the above

be
(c) 1 (d) None of these
2
7. ∫| x 2 + 2 x − 3| dx is equal to 15. If z and ω are two non-zero complex numbers
π
such that|zω| = 1 and arg(z) − arg(ω) =

Tu
0 , then
(a) 4 (b) 6 (c) 3 (d) 2 2
z ω is equal to
 1 2
3 5 7  2
(a) − i (c) − 1

ou
(b) 1 (d) i
8. lim  + + + +…
x → ∞ 1− x 3
1+ x 2
1− x 3
1+ x 2
  16. If { x } denotes the fractional part of x, then
equals  3 2n 
(a) −
5
6
(b) −
10
3
(c)
5
6
(d)
10
3 (Y   , n ∈ N is
 8 
(a)
3
(b)
7
on
9. The solution of the differential equation 8 8
−1 dy 1
(1 + y 2) + (x − e tan y
) = 0 , is (c) (d) None of these
dx 8
pi

−1
(a) xe 2 tan−1 y = e tan y
+ k 17. If the roots of the equation
−1
(b) ( x − 2 ) = ke tan x 3 + bx 2 + cx − 1 = 0 form an increasing GP,
m

tan −1 y 2 tan −1 y then b belongs to the interval.


(c) 2 x e =e + k
ha

tan −1 y −1 (a) (−3, ∞ ) (b) (−∞, − 3)


(d) xe = tan y + k
(c) (−1, ∞ ) (d) (−∞, − 1)
10. The solution of x dx + ydy = a(x 2 + y 2)dy, is 18. The letters of the ward COCHIN are
C

(a) x2 + y2 = ce ay (b) x2 + y2 = ce 2 ay permuted and all the permutations are


(c) x + y = e
2 2 2cay
(d) None of these arranged in an alphabetical order as in an
dy

English dictionary. The number of words that


11. If e and e′ be the eccentricities of two conics appear before the word COCH1N, is
s = 0 and s′ = 0 and if e 2 + e ′ 2 = 3, then both S (a) 360 (b) 192
u

and S′ can be (c) 96 (d) 48


St

(a) hyperbolas (b) ellipses 19. Suppose that six students, including Madhu
(c) parabolas (d) None of these and Pooja are having six beds arranged in a
12. A point on the parabola y = 18 x at which 2 row. Further, suppose that Madhu does not
want a bed adjacent to Pooja. Then the
the ordinate increases at twice the rate of the number of ways, the beds can be alloted to
abscissa is students is
(a)  ,  (c)  − ,  (d) (2, 4)
9 9 9 9
(b) (2, − 4) (a) 384 (b) 264
 8 2  8 2
(c) 480 (d) 600
13. Consider a ∆ABC and let a , b and c denote the 20. For all n ∈ N , 33n − 26 n − 1 is divisible by
lengths of the sides opposite to vertices A , B
and C, respectively. Suppose a = 6, b = 10 and (a) 24 (b) 64
the area of the triangle is 15 3. If ∠ ACB is (c) 17 (d) 676
WB JEE (Engineering) Practice Set 3 43

21. The number of rational terms in the 28. Let W denote the words in the English
expansion of (1 + 2 + 3 3)6 is dictionary, define the relation R by
R = {(x , y) ∈ N × W : the words x and y have
(a) 6 (b) 7 (c) 5 (d) 8 at least one letter is common}. Then R is
(a) not reflexive, symmetric and transitive
 cos 2 θcos θ sin θ
22. If E(θ) =   and θ and φ (b) reflexive, symmetric and not transitive
cos θ sin θ sin 2 θ  (c) reflexive, not symmetric and transitive
π (d) reflexive, symmetric and transitive
differ by an odd multiple of , then E(θ) ⋅ E(φ)

)
2 29. Two numbers are selected randomly from the

be
is a set S = {1, 2, 3, 4, 5, 6} without replacement
(a) null matrix (b) unit matrix one by one. The probability that the

Tu
(c) diagonal matrix (d) None of these minimum of two numbers is less than 4 is
1 14
(a) (b)
23. Let M be a 3 × 3 non-singular matrix with 15 15
det (M) = α. If M −1 adj (adj M) = kI , then the 1 4

ou
(c) (d)
value of k is 5 5
(a) 1 (b) α (c) α 2 (d) α 3 30. A random variable X takes the values 0, 1, 2, 3
24. If α is a non-real cube of −2, then the value of
1 2α 1 (Y and its mean is 1.3. If P(X = 3) = 2 P(X = 1)
and P(X = 2) = 0 .3, then P(X = 0) is
(a) 0.1 (b) 0.2
on
α 2
1 3α 2 , is (c) 0.3 (d) 0.4
2 2α 1 31. If a and b are distinct positive real numbers
such that a , a1 , a 2 , a 3 , a 4 , a 5 , b are in AP,
pi

(a) − 11 (b) −12 (c) −13 (d) 0


a , b1 , b2 , b3 , b4 , b5 , b are in GP and
1 0  1 0  a , c1 , c2 , c3 , c4 , c5 , b are in HP, then the roots of
m

25. If A =   and I = 0 1, then which one


1 1   a 3 x 2 + b3 x + c3 = 0 are
of the following holds for all n ≥ 1, by the
ha

(a) real and distinct (b) real and equal


principle of mathematical induction (c) imaginary (d) None of these
(a) A n = 2 n − 1 A + (n − 1)I (b) A n = nA + (n − 1)I 32. If the angles A , B and C of a triangle are in
C

(c) A n = 2 n − 1 A − (n − 1)I (d) A n = nA − (n − 1)I arithmetic progression and if a , b and c


denote the lengths of the sides opposite to
4
dy

26. If A = {(x , y) : y = , x ≠ 0 } and A , B and C respectively, then the value of the


x a c
expression sin 2C + sin 2 A is
B = {(x , y) : x 2 + y 2 = 8 , x , y ∈ R}, then c a
u

(a) A ∩ B = φ (a)
1
(b)
3
St

(b) A ∩ B contains one point only 2 2


(c) A ∩ B contains two points only (c) 1 (d) 3
(d) A ∩ B contains 4 points only
33. Let A be the fixed point (0, 4) and B(2t, 0) be a
27. Let the function f : R − { − b} → R − {1} be moving point. Let M be the mid-point of AB
x+a and let the perpendicular bisector of AB meet
defined by f (x) = , a ≠ b, then the Y -axis at R. The locus of the mid-point P
x+b
of MR is
(a) f is one-one but not onto 1
(b) f is onto but not one-one (a) x2 + y2 = (b) ( y − 2 )2 − x2 = 4
4
(c) f is both one-one and onto (c) y + x2 = 2 (d) 3 x2 + y2 = 8
(d) None of the above
44 WB JEE (Engineering) Practice Set 3

34. Let a , b, c and d be non-zero numbers. If the 41. The equation of a tangent to the hyperbola
point of intersection of the lines 16 x 2 − 25 y 2 − 96 x + 100 y − 356 = 0 , which
4 ax + 2 ay + c = 0 and 5 bx + 2 by + d = 0 lies in π
the fourth quadrant and is equidistant from makes an angle with the transverse axis, is
the two axes, then 4
(a) 3bc − 2 ad = 0 (b) 3bc + 2 ad = 0 (a) y = x + 2 (b) y = x − 5 (c) y = x + 3 (d) x = y + 2
(c) 2 bc − 3ad = 0 (d) 2 bc + 3ad = 0
42. The slope of a common tangent to the ellipse
35. If a variable line passes through the point of x2 y2

)
intersection of the lines x + 2 y − 1 = 0 and 2
+ = 1 and a concentric circle of radius r is
b2

be
2 x − y − 1 = 0 and meets the coordinate axes a
in A and B, then the locus of the mid-point of r 2 − b2 r 2 − b2
(a) tan−1 (b)
AB is a −r
2 2
a2 − r 2

Tu
(a) x + 3 y = 0 (b) x + 3 y = 10 r 2 − b2 a2 − r 2
(c) x + 3 y = 10 xy (d) None of these (c) (d)
a −r
2 2
r 2 − b2

ou
36. Let the perpendiculars from any point on the
line 2 x + 11 y = 5 upon the lines 43. The equation of the plane containing the line
24 x + 7 y − 20 = 0 and 4 x − 3 y − 2 = 0 have x +1 y − 3 z + 2
= = and the point (0, 7, −7), is
the lengths P1 and P2 , respectively. Then,
(a) 2 P1 = P2
(c) P1 = 2 P2
(b) P1 = P2
(d) None of these (Y −3 2 1
(a) x + y + z = 1
(c) x + y + z = 0
(b) x + y + z = 2
(d) None of these
on
37. The locus of the centre of the circles which 44. Let a , b and c be three real numbers satisfying
touch both the circles x 2 + y 2 = a 2 and 1 9 7 
[ a b c] 8 2 7  = [0 0 0 ].
pi

x 2 + y 2 = 4 ax externally has the equation


 
(a) 12( x − a)2 − 4 y2 = 3a2 (b) 9( x − a)2 − 5 y2 = 2 a2 7 3 7 
m

(c) 8 x2 − 3( y − a)2 = 9a2 (d) None of these


If the point P(a , b, c) lies on the plane
ha

38. The range of values of a for which the point 2 x + y + z = 1, then the value of 7 a + b + c is
(a , 4) is outside the circles x 2 + y 2 + 10 x = 0 (a) 0 (b) 12 (c) 7 (d) 6
and x 2 + y 2 − 12 x + 20 = 0 , is
C

45. If [⋅] denotes the greatest integer function,


(a) (−∞, − 8) ∪ (−2, 6) ∪ (6, ∞ ) tan([ −2 π 2 ] x 2) − x 2 tan[ −2 π 2 ]
(b) (−∞, − 2 ) then lim is
dy

(c) (−∞, − 8) ∪ (−2, ∞ )


x→0 sin 2 x
(d) None of the above equal to
(a) − 20 + tan20° (b) 20 + tan20°
u

39. A hyperbola, having the transverse axis of (c) 20 (d) None of these
length 2sinθ, is confocal with the ellipse
St

1
3 x 2 + 4 y 2 = 12. Then, its equation is 46. If f (x) = , then the derivation of the
1− x
(a) x2cosec 2θ − y2sec 2θ = 1
composite function f [ f { f (x)}] is equal to
(b) x2sec 2θ − y2cosec 2θ = 1 1
(a) 0 (b) (c) 1 (d) 2
(c) x2 sin2 θ − y2 cos 2 θ = 1 2
(d) x2 cos 2 θ − y2 sin2 θ = 1
47. If the period of the function
sin(sin(nx))
40. The number of values of c such that the f (x) = , n ∈ N is 6 π, then n is
straight line y = 4 x + c touches the curve  x
tan  
x2  n
+ y 2 = 1, is
4 equal to
(a) 0 (b) 1 (c) 2 (d) infinite (a) 3 (b) 2
(c) 1 (d) None of these
WB JEE (Engineering) Practice Set 3 45

48. Let the sequence < bn > of real numbers 54. Let a = 2$i + $j − 2k$ and b = $i + $j. If c is a vector
satisfy the recurrence relation such that a ⋅ c =|c|,|c − a| = 2 2 and the angle
1 125  between a × b and c is 30°. Then,|(a × b) × c|
bn + 1 = 2 bn + 2  , bn ≠ 0, then lim bn is
3 bn  n→ ∞ is equal to
2 3
equal to (a) (b) (c) 2 (d) 3
3 2
(a) 0 (b) ∞
2 55. Let S1 , S2 ,… be squares such that for each
(c) 5 (d)

)
3
n ≥ 1, the length of a side of Sn equals the

be
49. The set of all points where the function length of a diagonal of Sn + 1 . If the length of
f (x) = 3
x 2 | x| is differentiable is a side of S1 is 10 cm, then for which of the
following values of n is the area of Sn less

Tu
(a) [0, ∞ ) (b) (0, ∞ )
than 1 sq cm?
(c) (−∞, ∞ ) (d) (−∞, 0) ∪ (0, ∞ )
(a) 7 (b) 8 (c) 5 (d) 6
50. Let f be twice differentiable function

ou
56. The value of k for which the inequality
1 = 1, f (2) = 4 , f (3) = 9 , then
satisfying f ()
|Re(z)| + |1m (z)| ≤ λ |z| is true for all z ∈ C , is
(a) f ′( x) = 2 for all x ∈ R
(a) 2 (b) 2
(b) f ′( x) = 5 = f ′′( x), for some x ∈[1, 3]
(c) there exists at least one x ∈(1, 3) such that
f ′′( x) = 2 (Y (c) 1 (d) None of these

57. Given, 2 x − y + 2z = 2; x − 2 y + z = − 4;
on
(d) None of the above
x + y + λz = 4, then the value of λ such that
the given system of equations has no
Category II (Q. No. 51 to 65)
solution, is
pi

Carry 2 marks each and only one option is (a) 3 (b) 1 (c) 0 (d) −3
correct. In case of incorrect answer or any
m

combination of more than one answer, 1/2 mark 58. Let X and Y be two non-empty sets and
will be deducted. f : X → Y be a function such that
ha

f (C) = { f (x) : x ∈ C } for C ⊆ X


51. If C 0 , C1 , C 2 ,…, C n denote the binomial and f −1 (D) = { x : f (x) ∈ D} for D ⊆ Y
coefficient in the expansion of (1 + x)n, then
C

If A ⊆ X and B ⊆ Y , then
C1 C 3 C 5
+ + + … is equal to (a) f −1(f( A)) = A
2 4 6
(b) f −1(f( A)) = A only if f( X ) = Y
dy

2n − 1 2n
(a) (b) (c) f(f −1(B)) = B only if B ⊆ f( X )
n+1 n+2
(d) f(f −1(B)) = B
u

2n − 1 2n
(c) (d)
n+1 59. The function f : (−∞ , − 1] → (0 , e 5] defined by
St

n
3
− 3x + 2
52. If I n = ∫ (log x)n dx , then I n + n ⋅ I n − 1 is equal to f (x) = e x is
(a) ( x log x)n (b) x (log x)n (a) one-one and onto (b) one-one and into
(c) n(log x)n (d) (log x)n − 1 (c) many one and onto (d) many one and onto

53. Let a = $i + 2$j + k$ , b = $i − $j + k$ , c = $i + $j − k$ . A 60. The incentre of the triangle with vertices
(1, 3), (0 , 0) and (2, 0) is
vector caplanar to a and b has a projection
 3
(b)  ,
1 2 1 
along c of magnitude , then the vector is (a)  1,  
3  2   3 3
2 3
(d)  1,
(a) 4$i − $j + 4k$ (b) 4$i + $j − 4k$ 1 
(c)  ,  
3 2   3
(c) 2 $i + $j + k$ (d) None of these
46 WB JEE (Engineering) Practice Set 3

x2 y2 66. Let f (x) = 7 tan 8 x + 7 tan 6 x − 3 tan 2 x − 3 tan 4 x


61. If radii of director circles of + = 1 and
 π π
2 2
a b
for all x ∈  − ,  . Then the correct
x2 y2  2 2
− = 1 are 2r and r, respectively and ee
a12 b12 expression(s) is/are
π
and eh be the eccentricities of the ellipse and 1
(a) ∫ 4 x f( x)dx =
hyperbola respectively, then 0 12
π
(a) 2e h2 − ee2 = 6 (b) ee2 − 4e h2 = 6 (b) ∫ 4 f( x)dx = 0

)
0
(c) 4e h2 − ee2 =6 (d) None of these π

be
1
(c) ∫ x f( x)dx =
4
log[ x ] 0 6
62. lim , where [ x ] denotes the greatest π
x→∞ x (d) ∫ f( x)dx = 1

Tu
4
0
integer less than or equal to x, is
(a) 0 (b) 1 67. In a ∆PQR, let ∠PQR = 30 ° and the sides PQ
(c) −1 (d) non-existent and QR have lengths 10 3 and 10 respectively,

ou
then which of the following statement(s)
63. If f (x) = min{1, x , x }, then
2 3
is/are correct?

(Y
(a) f( x) is everywhere continuous (a) ∠QPR = 45°
(b) f( x) is continuous and differentiable everywhere (b) The area of the ∆PQR is 25 3 and ∠QRP = 120°
(c) f( x) is not differentiable at two points (c) The radius of the incircle of the ∆PQR is10 3 − 15
(d) f( x) is not differentiable at one point (d) The area of the circumcircle of the ∆PQR is 100π
on
64. Let f (x) = sec −1[1 + cos 2 x ], where [⋅] denotes 68. If the line x = α divides the area of region
the greatest integer function. Then, the R = {(x , y) ∈ R 2 : x 3 ≤ y ≤ x , 0 ≤ x ≤ 1} into two
pi

range of f (x) is equal parts, then


(a) [1, 2 ] (b) [0, 2 ] (a) 2α 4 − 4α 2 + 1 = 0 (b) α 4 + 4α 2 − 1 = 0
m

(c) {sec −11, sec −1 2} (d) None of these 1 1


(c) < α < 1 (d) 0 < α ≤
2 2
ha

65. A spherical iron ball 10 cm in radius is


coated with a layer of ice of uniform 69. Let S be the set of all non-zero real numbers
thickness that melts at a rate of 50 cm 3 /min. α such that the quadratic equation
C

When the thickness of ice is 5 cm, then the αx 2 − x + α = 0 has two distinct real roots
rate at which the thickness of ice decreases, x1 and x 2 satisfying the inequality| x1 − x 2| < 1.
is Which of the following interval(s) is/are a
dy

5 1 subset of S?
(a) cm/min (b) cm/min
(a)  − , −
1 
(b)  − , 0
6π 54π 1 1

 2 5  5 
u

1 1
(c) cm/min (d) cm/min
(c)  0,
1 
(d) 
18π 36π 1 1
St

 , 
 5  5 2
Category III (Q. Nos. 66 to 75) 70. Let z1 and z 2 be two distinct complex numbers
Carry 2 marks each and one or more option(s) and let z = (1 − t)z1 + tz 2 for some real numbers
is/are correct. If all correct answers are not t with 0 < t < 1. If arg(ω) denotes the principal
marked and also no incorrect answer is marked argument of a non-zero complex number ω,
then score = 2 × number of correct answers then
marked ÷ actual number of correct answer. If any (a)| z − z1| + | z − z2| = | z1 − z2|
wrong option is marked or if, any combination (b) arg ( z − z1 ) = arg ( z − z2 )
including a wrong option is marked, the answer z − z1 z − z1
(c) =0
will considered wrong, but there is no negative z2 − z1 z2 − z1
marking for the same and zero marks will be (d) arg ( z − z1 ) = arg ( z2 − z1 )
awarded.
WB JEE (Engineering) Practice Set 3 47

71. If α , β , γ are the roots of x 3 + ax 2 + b = 0, The equation of a common tangent with


positive slope to the circle as well as to the
α β γ
hyperbola is
then the value of β γ α is (a) 2 x − 5 y − 20 = 0
γ α β (b) 2 x − 5 y + 4 = 0
(c) 3 x − 4 y + 8 = 0
(a) − a3 (b) a3 − 3b
(d) 4 x − 3 y + 4 = 0
(c) a3 (d) a2 − 3b
74. If y is a function of x and log(x + y) − 2 xy = 0,

)
72. If A(2, 3) and B(−2, 1) are two vertices of a then the value of y′(0) is equal to

be
triangle and third vertex moves on the line (a) 1 (b) −1 (c) 2 (d) 0
2 x + 3 y = 9 , then the locus of the centroid of  π
the triangle, is 75. In the interval 0 ,  the function
 2

Tu
(a) 2 x + 3 y = 1 (b) 2 x + y = 3
(c) 2 x − 3 y = 1 (d) x − y = 1
f (x) = tan n x + cot n x attains
(a) the minimum value which is independent of n
73. The circle x 2 + y 2 − 8 x = 0 and hyperbola

ou
(b) a minimum value which is a function of n
x 2 y2 (c) the minimum value of 1
− = 1 intersect at the points A and B.
9 4 (d) None of the above

Answers
(Y
on
Physics
pi

1.(a) 2.(a) 3.(a) 4.(c) 5.(c) 6.(c) 7.(b) 8.(d) 9.(c) 10.(c)
m

11.(a) 12.(a) 13.(d) 14.(a) 15.(c) 16.(b) 17.(c) 18.(a) 19.(c) 20.(d)
21.(d) 22.(b) 23.(a) 24.(b) 25.(a) 26.(d) 27.(c) 28.(c) 29.(b) 30.(a)
ha

31.(c) 32.(a) 33.(a) 34.(b) 35.(d) 36.(c, d) 37.(a, c, d) 38.(b, c) 39.(a, d) 40.(a, b, d)

Chemistry
C

41. (c) 42. (a) 43. (c) 44. (d) 45. (d) 46. (c) 47. (c) 48. (c) 49. (d) 50. (b)
51. (c) 52. (d) 53. (b) 54. (d) 55. (a) 56. (a) 57. (c) 58. (c) 59. (b) 60. (c)
dy

61. (d) 62. (c) 63. (a) 64. (c) 65. (b) 66. (d) 67. (b) 68. (b) 69. (c) 70. (d)
71. (b) 73. (d) 73. (a) 74. (a) 75. (c) 76. (b, d) 77. (a, c) 78. (a, b) 79. (b, c, d) 80. (a, c)
u

Mathematics
St

1. (c) 2. (c) 3. (a) 4. (a) 5. (a) 6. (c) 7. (a) 8. (b) 9. (c) 10. (b)
11. (a) 12. (a) 13. (c) 14. (b) 15. (a) 16. (c) 17. (b) 18. (c) 19. (c) 20. (d)
21. (b) 22. (a) 23. (b) 24. (c) 25. (d) 26. (c) 27. (c) 28. (b) 29. (d) 30. (d)
31. (c) 32. (d) 33. (c) 34. (a) 35. (c) 36. (b) 37. (a) 38. (a) 39. (a) 40. (c)
41. (a) 42. (b) 43. (c) 44. (d) 45. (a) 46. (c) 47. (a) 48. (c) 49. (d) 50. (c)
51. (a) 52. (b) 53. (a) 54. (b) 55. (b) 56. (b) 57. (b) 58. (c) 59. (b) 60. (d)
61. (c) 62. (a) 63. (a,d) 64. (c) 65. (c) 66. (a,b) 67. (b,c,d) 68. (a,c) 69. (a,d) 70. (a,c,d)
71. (c) 72. (a) 73. (b) 74. (a) 75. (a)

* For detailed solutions visit http://tinyurl.com/y3e5w2dk or scan


WB JEE
Engineering Entrance Exam

Practice Set 4

)
be
Tu
Physics

ou
Category I (Q. Nos. 1 to 30) 4. Ionisation potential of hydrogen atom is

(Y
Carry 1 mark each and onlyone option is correct. 13.6 eV. The least energy of photon of Balmer
In case of incorrect answer or any combination of series is
more than one answer, 1/ 4 mark will be deducted. (a) 3.4 eV (b) 1.89 eV (c) 10.2 eV (d) 8.5 eV
on
1. A particle is moving at a constant velocity v 5. The current gain of a transistor in a common
from a large distance towards a concave configuration is 40. If the emitter current is
pi

mirror of radius R along the principal axis. 82 mA, then base current is
The speed of the image as a function of the (a) 0.02 mA (b) 0.20 mA (c) 2.0 mA (d) 0.4 mA
m

distance x of the particle from the mirror is


6. The combination of gate shown below yields
R2v R2
(a) (b)
ha

(R − 2 x)2 ( R − 2 x) A
R2v 2 v2
(c) (d)
(R − 2 x)2 (R − 2 x)2
C

2. The activity of a radioactive substance is R1 at B


dy

time t1 and R2 at time t2 (> t1) . Its decay


constant is λ 0 . Then, (a) OR gate (b) NOT gate
(c) XOR gate (d) NAND gate
R1e λ( t 1 − t 2 )
u

(a) R1t 1 = R 2t 2 (b) R 2 =


R − R2 7. The dimensions of solar constants are
St

(c) 1 = constant (d) R 2 = R1e λ( t 2 − t 1 )


t 2 − t1 (a) [M0L0 T0 ] (b) [MLT−2 ] (c) [ML2 T−2 ] (d) [MT−3 ]

3. In a photoemissive cell with exciting 8. The period of oscillation of a simple


wavelength λ, the fastest electron has speed l
pendulum is T = 2π . Measured value of l is
v. If the exciting wavelength is changed to g
3 λ / 4 , then the speed of the fastest emitted 20.0 cm known to 1 mm accuracy and time
electron will be for 100 oscillation of the pendulum is found
1/ 2 1/ 2 to be 90 s using a wrist watch of 1 s
(a) v   (b) v  
3 4
 4  3 resolution. The accuracy in the determination
of g is
(c) less than v(4/ 3)1/ 2 (d) greater than v(4/ 3)1/ 2
(a) 2% (b) 3% (c) 1% (d) 5%
WB JEE (Engineering) Practice Set 4 49

9. A particle of mass m collides with another 16. If C s is the velocity of sound in air and C is
stationary particle of mass m. If the particle the rms velocity, then
m stops just after collision. The coefficient of (a) C s < C (b) C s = C
restitution is equal to γ
1/ 2
m m M−m (c) C s = C   (d) None of these
(a) (b) (c) 1 (d)  3
M+ m M M+ m
17. At constant temperature, the volume of gas is
10. A machine gun fire 360 bullets per minute to be decreased by 4%. The pressure must be
with a velocity of 600 ms −1 . If the power of

)
increased by

be
gun is 5.4 kW. Mass of each bullet is (a) 4% (b) 4.16% (c) 8% (d) 3.86%
(a) 5 kg (b) 0.5 kg (c) 5 g (d) 0.5 g
18. Find the equivalent capacitance of the system

Tu
11. A range of projectile fixed at an angle of 15° across the terminals A and B. All the
is 50 m. If it is fired with the same speed at capacitors have equal capacitances.
an angle of 45°. Its range will be
C

ou
(a) 25 m (b) 50 m (c) 100 m (d) 77.6 m A

12. A geostationary satellite is orbiting. The earth C C

(Y
at a height 6 R above the earth surface, where
R is radius of earth. The time period of
C C C
another satellite at a height 2.5 R from
earth’s surface would be
on
B
6 2.5 (a) 2C (b) 3C (c) 4C (d) 5C
(a) 24 h (b) h (c) h (d) 6 2 h
2.5 6
19. Three point charges q, 2q and 4 q are be placed
pi

13. A rubber cube of each side 7 cm has one side


on a 9 cm long straight line. Find the
fixed, while a tangential force equal to the
m

position of the charge q such that the


weight of 300 kgf is applied to the opposite
potential energy of this system is minimum
face. What is the shearing strain produced
ha

(a) 1 cm from 2q (b) 2 cm from 2q


and the distance through which the strained
(c) 3 cm from 2q (d) 4 cm from 2q
side moves? (The modulus of rigidity for
rubber is 2 × 10 7 dyne cm −2 , g = 10 ms −2 ) 20. The electric strength of air is 2 × 10 7 NC−1.
C

(a) 0.3 rad, 2.1 cm (b) 0.2 rad. 2.1 cm The maximum charge that a metallic sphere
(c) 0.4 rad, 2.1 cm (d) 0.3 rad, 2.5 cm of diameter 6 mm can hold is
dy

(a) 3 nC (b) 20 nC (c) 1.5 nC (d) 2 nC


14. At what speed will the velocity of a stream of
water be equal to 20 cm of mercury column? 21. Magnetic field at the centre of a circular loop
u

(Take, g = 10 ms −2 ) of area A is B. The magnetic moment of the


St

(a) 6.4 ms −1
(b) 7.3756 ms −1 loop will be
(c) 6.4756 ms −1 (d) None of these BA 2 BA 3 / 2 BA 3 / 2 2 BA 3 / 2
(a) (b) (c) (d)
15. Two metallic spheres s1 and s 2 are made of µ0π µ0π µ 0 π1/ 2 µ 0 π1/ 2
the same material and have identical surface
finish. The mass of s1 is three times that of s 2 . 22. Two circular coil mounted parallel to each
Both the spheres are heated to the same high other on the same axis carry steady currents.
temperature and placed in the same room If an observer between the coil reports that
having lower temperature but are thermally one coil is carrying a clockwise current i1 ,
insulated from each other. The ratio of initial while the other is carrying a counter
rates of colling of s1 to that s 2 is clockwise current i2 between the coils, then
1/ 3
there is
(b)  
1 1 1 3
(a) (c) (d) (a) steady repulsive force(b) zero force
3  3 3 1 (c) a repulsive force (d) a steady attractive force
50 WB JEE (Engineering) Practice Set 4

23. The magnetic susceptibility of a 29. A beam of light of wavelength 600 nm from a
paramagnetic material at − 73 °C is 0.0075. Its distance source falls on a single slit 1.0 mm
value at − 173 °C will be wide and the resulting diffraction pattern is
(a) 0.0045 (b) 0.0030 (c) 0.015 (d) 0.0075 observed on a screen 2 m away. What is the
distance between first dark fringe on either
24. The magnetic moment µ of a revolving side of the central bright fringe.
electron around the nucleus varies with (a) 1.5 mm (b) 1 mm (c) 1.2 mm (d) 2.4 mm
principal quantum numbers n as
30. In young’s double slit experiment, the length

)
1 1
(a) µ ∝ n (b) µ ∝ (c) µ ∝ n2 (d) µ ∝ of band is 1 mm. The ring width is 1.021 mm.

be
2
n n
The number of fringe is
25. The value of i1 is the circuit diagram will be (a) 45 (b) 46 (c) 47 (d) 48

Tu
E K Category II (Q. Nos. 31 to 35)
Carry 2 mark each and onlyone option is correct.
8V

ou
B
In case of incorrect answer or any combination of
more than one answer, 1/ 2 mark will be deducted.
2Ω 2Ω 31. An inductor of inductance L = 440 mH and

(Y
5Ω
4Ω 4Ω resistors of resistances R1 = 2 Ω and R2 = 2 Ω
are connected to battery of emf 12 V as
A i1 C
shown in the figure. The internal resistance
on
5Ω of battery is negligible.The switch S is closed
2Ω 2Ω
at t = 0. The potential drop across L as a
D function of time is
pi

1 3 3
(a) 1 A (b) A (c) A (d) A
m

2 4 2 E L

R1
26. Under what condition will the strength of
ha

S R
current in a wire of resistance R be the same
for connection is n-series or in parallel of n
C

identical cells each of the internal resistance


12 −3t
r1 when (a) e V (b) 6(1 − e −t / 0. 2 ) V
t
(a) R = nr (b) R = r / n (c) 12e −5t V (d) 6e −5t V
dy

(c) R = r (d) R → ∞, r → 0
32. Water flows through a horizontal pipe of
27. In an L-R circuit, the value of L is 0.4 / π and
u

variable cross-section at rate of 20 L per min.


the value of R is 30 Ω. If in the circuit, an What will be velocity of water at a point
St

alternating emf of 200 V at 50 cycle per where diameter is 4 cm?


second is connected, the impedance of the (a) 0.2651 ms −1 (b) 0.5639 ms −1
circuit and current will be (c) 0.4639 ms −1 (d) 0.3639 ms −1
(a) 11.4 Ω, 17.5 A (b) 30.7 Ω, 6.5 Ω
33. Water falls from a height of 500 m. What is
(c) 40.4 Ω, 5A (d) 50 Ω, 4A
the rise in temperature of water at the bottom,
28. Two identical glass (µ g = 3 / 2) equiconvex if whole energy is used up in heating water?
lenses of focal length f are kept in contact. (a) 0.96°C (b) 1.02°C (c) 1.16°C (d) 0.23°C
The space between the two lenses is filled 34. Instantaneous displacement current of 1.0 A
with water (µ w = 4 / 3). The focal length of in the space between the parallel plates of
the combination is 1 µF capacitor can be established by changing
f 4f 3f potential difference of
(a) f (b) (c) (d)
2 3 4 (a) 10−6 Vs −1 (b) 106 Vs −1 (c) 10−8 Vs −1 (d) 108 Vs −1
WB JEE (Engineering) Practice Set 4 51

35. The self inductance of the motor of an (a) Heat is absorbed by the gas in all the three
electric fan is 10 H. In order to impart paths
(b) Heat absorbed/released by the gas is maximum
maximum power at 50 Hz. It should be
in path 2
connected to a capacitor of
(c) Temperature of the gas increases first and then
(a) 3 × 10−6 F (b) 2 × 10−6 F decreased continuously in path 2
−6
(c) 10 F (d) 10−6 F (d) Change in internal energy of the gas is same
along all the three paths
Category III (Q. Nos. 36 to 40)

)
38. A particle is projected from a point A with a
Carry 2 mark each and onlyone option is/are

be
velocity v at an angle of elevation θ. At a
correct. If all correct answers are not marked and certain point B, the particle moves at right
also no incorrect answer is marked then score angle to its initial direction. Then,
= 2 × number of correct answers marked ÷ actual

Tu
(a) velocity of particle at B is v sin θ
number of correct answers. If any wrong option is (b) velocity of particle at B is v cot θ
marked or if any combination including a wrong (c) velocity of particle at B is v tan θ

ou
option is marked, the answer will be considered v
(d) velocity of flight from A to B is
wrong, but there is not negative marking for the g sin θ
same and zero mark will be awarded.

(Y
39. The potential difference between the points A
36. When photons of energy 4.25 eV strike the and B in the circuit shown in figure is 16 V.
surface of a metal, the ejected photoelectrons Which is/are the correct statements out of
have a maximum kinetic energy E A eV and
on
the following?
de-Broglie wavelength λ A . The maximum
kinetic energy of a photoelectrons diberated 9V 1Ω 3V 1Ω
4Ω + – 3Ω
from another metal B by photons of energy
pi

A + – B
4.70 eV is E B = (E A − 150
. ) eV. If the 1Ω
de-Broglie wavelength of the these
m

2Ω
photoelectron is λ B = 2 λ A , then
(a) The current through the 2Ω resistor is 3.5 A
(a) the work function of A is 2.25 eV
ha

(b) The current through the 4Ω resistor is 2.5 A


(b) the work function of B is 4.20 eV
(c) The current through the 3Ω resistor is 1.5 A
(c) EA = 2.5 eV (d) EB = 2.75 eV
(d) The potential difference between the terminals
C

37. A gas undergoes a change in its state from of the 9 V battery is 7 V


position A to position B via three different
40. A charged particle P leaves the origin with
paths as shown in the figure. Select the
dy

speed v = v0 , at some inclination with the


correct statement.
X -axis. There is uniform magnetic field B
Y
along the X -axis. P strikes a fixed target T on
u

the X -axis for a minimum value of B = B0 . P


St

1 will also strikes T, if


p
2 (a) B = 2 B0 , v = 2 v 0
A
3
B (b) B = 2 B0 , v = v 0
(c) B = B0 , v = 2 v 0
(d) B = B0 / 2, v = v 0 / 2
X
O V
Chemistry
Category I (Q. Nos. 41 to 70) 47. KF combines with HF to form KHF2 . The
compound contains the species.
Carry 1 mark each and only one option is correct.
(a) K + , F − and H+ (b) K + , F − and HF
In case of incorrect answer or any combination of
(c) K + and [HF2 ]− (d) [KHF]+ and F −
more than one answer, 1/4 mark will be deducted.

)
48. Which of the following factors is most

be
41. How many moles of magnesium phosphate important in making the fluoride as strongest
[Mg 3 (PO 4)2 ] will contain 0.25 moles of oxidising agent?
oxygen atoms? (a) Electron affinity

Tu
(a) 0.02 (b) 3125
. × 10−2 (b) Ionisation enthalpy
(c) 1.25 × 10−2 (d) 2.5 × 10−2 (c) Hydration enthalpy
(d) Bond dissociation energy

ou
42. 250 mL of sodium carbonate solution 49. The electrons can be identified by quantum
contains 2.65 g of Na 2CO 3 . If 10 ml of this numbers n and l, the correct order of
solution is diluted to 500 mL, the

(Y
increasing energy for the following orbitals is
concentration of the diluted solution will be 1. n = 4, l = 1 2. n = 4 , l = 0
(a) 0.01 M (b) 0.001 M 2. n = 3 , l = 2 4. n = 3 , l = 1
(c) 0.05 M (d) 0.002 M
on
(a) (3) < (4) < (2) < (1) (b) (4) < (2) < (3) < (1)
43. As temperature is raised from 20°C to 40°C, (c) (2) < (4) < (1) < (3) (d) (1) < (3) < (2) < (4)
the average kinetic energy of neon atoms 50. Hydrolysis of trichloromethane with aqueous
pi

changes by a factor of which of the following KOH gives


m

1 313 (a) methanol (b) acetic acid


(a) (b) (c) ethanol (d) formic acid
2 293
ha

313
(c) (d) 2 51. For the reaction,
293
∆/Cu
—N2Cl —Cl + N2
44. A blue liquid formed by equimolar mixture of
C

two gases at − 30 °C is (A)


(a) N2O (b) N2O 3
the half life is independent of concentration
dy

(c) N2O 4 (d) N2O 5


of (A). After 10 min. volume of N 2 gas is 10 L
45. Which of the following oxide is neutral in and after complete reaction is 50 L. Hence,
u

nature? rate constant is


2.303 2.303
St

(a) CO (b) SnO 2 (a) log 5 min−1 (b) . min−1


log 125
(c) ZnO (d) SO 2 10 10
2.303 2.303
(c) log 2 min−1 (d) log 4 min−1
46. Which of the following is the correct 10 10
statement about cryolite?
(a) Cryolite is NaAl 3F6 and is used in electrolysis of
52. Which of the following pair of compounds
alumina for decreasing the electrical conductivity. cannot exist together in a solution?
(b) Cryolite is Na 3 AlF6 and is used in electrolysis of (a) NaHCO 3 and NaOH (b) NaHCO 3 and H2O
alumina for lowering the melting point of alumina. (c) NaHCO 3 and NaCl (d) Na 2CO 3 and NaOH
(c) Cryolite is Na 3 AlF6 and is used in electrolysis of 53. An acid, that cannot suitable for preparation
alumina for freezing the alumina.
of H 2 by the action of metals is
(d) Cryolite is NaAl 3F6 and is used in electrolysis of
alumina to produce Na, Al and fluorine. (a) HCl (b) CH3COOH
(c) HNO 3 (d) H2SO 4
WB JEE (Engineering) Practice Set 4 53

54. During electrolysis of fused calcium hydride, (a) III > IV > II > I (b) III > II > IV > I
hydrogen is produced at (c) III > II > I > IV (d) II > III > IV > I
(a) cathode 62. The decreasing order of rate of bromination
(b) anode of the following compounds is
(c) hydrogen will not liberate ⊕ ⊕
(d) H2 produced will react with oxygen and give H2O (I) ph NMe3 (II) ph CH 2 N Me3
55. Assuming that each salt is 90% dissociated. (III) ph  Me (IV) ph N.Me2
Which of the following will have highest (a) (I) >(II) >(III) > (IV)

)
osmotic pressure? (b) (IV) >(III) >(II) > (I)

be
(a) Decinormal Al 2 (SO 4 )3 (b) Decinormal BaCl 2 (c) (III) >(IV) >(I) > (II)
(c) Decinormal Na 2SO 4 (d) Decinormal Urea. (d) (III) >(IV) >(II) > (I)

Tu
56. For a reaction, 63. Which one of the following arrangement
represents the correct order of electron gain
A(s) + 2 B⊕ → A 2 + + 2 B enthalpy (with negative sign) of the given

ou
° atomic species?
If, K c = 10 12 , then E(cell) is
(a) Cl < F < S < O (b) O < S < F < Cl
(a) 0.354 V (b) 0.708 V (c) 0.0098 V (d) 1.36 V (c) S < O < Cl < F (d) F < Cl < O < S
57. Radon is produced by radioactive decay of
(a) Ra (b) U (Y
64. Which of the following is correct option for
free expansion of an ideal gas under
on
(c) Th (d) None of these adiabatic condition?
(a) q ≠ 0, ∆T = 0, W = 0 (b) q = 0, ∆T = 0, W = 0
58. The equivalent weight of MnSO 4 is half of
(c) q = 0, ∆T < 0, W ≠ 0 (d) q = 0, ∆T ≠ 0, W = 0
its molecular weight, when it is converted to
pi

65. A reaction is A + B → C + D, initially start


(a) Mn2O 3 (b) MnO 2
with equal concentrations of (A) and (B), we
m

(c) MnO −4 (d) MNO 2−


4 find the moles of C is two times of (A). What
is the equilibrium constant of the reaction?
ha

59. Geometry of Ni(CO)4 and Ni(P.Ph 3)2 Cl 2 are


(a) 2 (b) 4
(a) both are square planar 1 1
(c) (d)
(b) square planar and tetrahedral respectively 2 4
C

(c) both are tetrahedral


(d) tetrahedral and square planar respectively 66. A solid compound ‘X ’ on heating gives CO 2
gas and a residue. The residue mixed with
dy

60. Arsenic (III) sulphide forms with negative water forms ‘Y ’. On passing an excess of CO 2
charge over it. Which of the following ionic through ‘Y ’ in water, a clear solution of ‘Z’ is
substance should be most effective in obtained. On boiling the ‘Z’, ‘X ’ is reformed.
u

coagulation of solution? The compound ‘X ’ is


St

(a) KCl (b) MgCl 2 (a) Ca(HCO 3 )2 (b) CaCO 3


(c) Al 2 (SO 4 )3 (d) Na 3PO 4 (c) Na 2CO 3 (d) K 2CO 3

61. The decreasing order of reactivity of the 67. Which of the following undergoes
following compounds with HBr is nucleophilic substitution exclusively by
SN1 mechanism?
OH H3C—
I. II. (a) Benzyl chloride (b) Ethylchloride
OH (c) Chloro propane (d) Isopropyl chloride
OH
III. MeO— IV. 68. Hydrolysis of sucrose is called
(a) inversion (b) esterification
OH OMe (c) hydration (d) saponification
54 WB JEE (Engineering) Practice Set 4

69. Which of the following structure represent . × 10 −6 at 184°C and R = 0.000831


(K c = 18
neoprene polymer? kJ/mol K) when K p and K c are compared at
(a) —
( CH2—C )n
CH—CH2— 184°C, it is found that
Cl (a) K p ≤ Kc (b) K p = Kc (c) K p < Kc (d) K p > Kc

(b) — )n
( CH2—CH— 74. Which of the following is a polar molecule
(a) SF4 (b) SiF4 (c) BaCl 2 (d) BF3
(c) — )n
( CH2—CH—
75. Among the given compounds, which one can

)
Cl
be distinguished by AgNO 3

be
(d) — )n
( CH—CH2—
(a) ethene (b) ethylene
C6H5 (c) acetylene (d) diethyl ether

Tu
70. Which one of the following is employed as a Category III (Q Nos. 76 to 80)
tranquilizer drug?
Carry 2 marks each and one or more options(s)

ou
(a) Promethazine (b) Valium
is/are correct. If all correct answers are not
(c) Naproxen (d) Mifepristone
marked and also no incorrect answer is marked
then score = 2 × number of correct answers

(Y
Category II (Q Nos. 71 to 75)
marked ÷ actual number of correct answers. If any
Carry 2 marks each and only one option is wrong option is marked or if any combination
correct. In case of incorrect answer or any including a wrong option is marked, the answer
on
combination of more than one answer, 1/2 mark will considered wrong, but there is no negative
will be deducted. marking for the same and zero mark will be
pi

71. In the following reaction, identify product (C) awarded.


ph NH 2 HNO
2→ (A) HF
→ B 
D
→ (C) 76. Which of the following graphs is correct for a
m

0°C BF3
+
zero order reaction?
(a) pH N BF4
ha

Concentration
Reaction rate

(b) F
of reactant
(a) (b)
C

(c) F F

(d) ph—F Time


dy

Time

72. The major organic compound formed from


the following reaction
u

Reaction rate

Concentration
of reactant

O slope = – k
(c)
St

(i) CH3NH2 (d)


Product is
(ii)LiAlH4 (iii)H2O
O—NHCH3 Time Time
(a) (b)
77. Which of the following are cyclic compounds ?
NHCH3
(a) Borazole
NHCH3 NHCH3 (b) Pyrrole
(c) (d) (c) Anthracene
(d) Isobutylene
OH O
78. Which of the following species are
73. For the reaction, isoelectronic as well as isostructural?
2NO 2(g) - 2NO(g) + O 2 (g). (a) CH4 (b) NH+4 (c) HF (d) NO −3
WB JEE (Engineering) Practice Set 4 55

79. Which of the following options are not in 80. Which of the following reactions would give
accordance with the property mentioned propanal?
against them? (a) Ethyl propanoate + DBAH (diisobutyl aluminium
(a) F2 >Cl 2 >Br2 >I2 Oxidising power hydride) [(i - C 4H9 )2 AlH] at –70°C followed by
(b) MI > MBr > MCl > MF Ionic character of metal halide hydrolysis
(c) F2 >Cl 2 >Br2 >I2 Bond dissociation enthalpy (b) Propyl propanoate + DIBAL-H/H2O
(d) HI <HBr <HCl <HF Hydrogen-halogen bond (c) Ethyl cyanide + DBAH
strength (d) Propanoyl chloride + LBAH

)
be
Mathematics

Tu
Category I (Q. Nos. 1 to 50) 6. If ∫ f (x)dx = 2{ f (x)} 3 + c, then f (x) is

ou
Only one answer is correct. Correct answer will fetch x 1 x
full marks 1. Incorrect answer or any combination of (a) (b) x3 (c) (d)
2 x 3
more than one answer will fetch−1/4 marks.

1. lim
x→∞
log[ x ]
x
, where [ x ] denotes the greatest (Y
7. The value of the integral
400 π


0
1 − cos 2 x dx is
on
integer less than or equal to x, is (a) 200 2 (b) 400 2
(a) 0 (b) 1 (c) 800 2 (d) None of these
(c) −1 (d) Non-existent 1 1 2 4 1 
pi

8. lim  sec 2 + sec 2 + … sec 2 1°


2. If u = ∫ e sin bx dx and v = ∫ e cos bx dx , then n→ ∞  n2 2 2 2
ax ax n n n n 
m

equals
(u + v )(a + b ) is equal to
2 2 2 2
1
(a) tan1° (b) tan1°
(a) 2e ax (b) e 2 ax (c) 2e 2 ax (d) bxe ax
ha

2
1 1
3. The equation of a curve passing through the (c) cosec1° (d) sec 1°
3
2 2
point (0, 1) be given by y = ∫x ⋅ e x dx . If the
2
C

9. The general solution of the differential


equation of the curve be written in the form equation (x + y)(dx − dy) = dx + dy is
x = f (y), then f (y) is equal to
dy

(a) ( x − y) = ke x − y (b) x + y = ke x + y
(a) loge (3 y − 2 ) (b) 3 loge (3 y − 2 ) (c) ( x + y) = k( x − y) (d) x + y = ke x − y
(c) 3 loge (2 − 3 y) (d) None of these
u

10. Solution of the differential equation


1
 π π
St

4. If ∫ t f ()
t dt = 1 − sin x for all x ∈ 0 , , then
2
cos x dy = y(sin x − y)dx , 0 < x < is
sin x  2 2
 1  (a) y tan x = sec x + c (b) tan x = (sec x + c )y
f   is equal to (c) sec x = (tan x + c )y (d) ysec x = tan x + c
 3
(a) 3 (b) 3 11. If the chords of tangents from two points
(c)
1
(d) None of these
(x1 , y1) and (x 2 , y 2) to the hyperbola
3 x 2 y2 x x
2
− 2 = 1 are at right angles, then 1 2
2 a b y1 y 2
5. The value of ∫ xd([ x ] − x) is equal to
0
a2 b2 b4 a4
1 (a) − (b) − (c) − (d) −
(a) (b) 1 (c) −1 (d) 0 b 2
a 2
a 4
b4
2
56 WB JEE (Engineering) Practice Set 4

12. If errors of 1% each are made in the base 20. For n ∈ N , 32n + 2 − 23 n − 9 is divisible by
radius and height of a cylinder, then the (a) 3 (b) 9 (c) 64 (d) 81
percentage error in its volume is
(a) 1% (b) 2% 21. The sum of the rational terms in the
(c) 3% (d) None of these expansion of ( 2 + 5 3)10 is
(a) 32 (b) 9
13. A GP consists of 2n terms. It the sum of the
(c) 41 (d) None of these
terms occupying the add places is S1 , and that
S

)
of the terms in the even places is S2 , then 2 is 22. If A satisfies the equation
x 3 − 5 x 2 + 4 x + λ = 0 , then A −1 exists if

be
S1
(a) independent of a (a) λ ≠ 1 (b) λ ≠ 2 (c) λ ≠ − 1 (d) λ ≠ 0
(b) independent of r

Tu
(c) independent of a and r 23. If B is a non-singular matrix and A is a
(d) dependent on r square matrix, then det (B−1 AB) is equal to
(sin x + 2 2 cos x) ≥ 2, −2 π ≤ x ≤ 2 π, (a) det ( A −1 ) (b) det (B−1 )

ou
14. If log 3
(c) det ( A) (d) det (B)
then the number of values of x is
(a) 0 (b) 3 x + λ x x 
(c) infinite (d) None of these

15. If arg (z) < 0, then arg (− z) − arg(z) is equal to (Y


24. If f (x) = x

 x
x+λ
x x
then f (3 x) − f (x) is equal to
x
+
,

λ
on
(a) π (b) − π
π π (a) 3 xλ2 (b) 6 xλ2
(c) − (d)
2 2 (c) xλ2 (d) None of these
pi

16. Let z = cos θ + i sin θ. Then, the value of 0 −1


0
25. Let A =  0 −1 0 . The only correct
m

15
 
∑Im(z 2m − 1) at θ = 2° is −1 0 0 
m=1
ha

statement about the matrix A is


1 1
(a) (b) (a) A −1 does not exist
sin2 ° 3sin2 °
(b) A = (−1)I is a unit matrix
C

1 1
(c) (d) (c) A is a zero matrix
2 sin2 ° 4sin2 ° (d) A 2 = I
17. If a , b, c ∈ R and the quadratic equation
dy

26. Let A , B, C be three sets such that


x 2 + (a + b) x + c = 0 has no real roots, then
A ∪ B ∪ C = U, where U is the universal set.
(a) c(a + b + c ) > 0 (b) c + (a + b + c )c > 0
u

Then,
(c) c − c(a + b + c ) > 0 (d) c(a + b − c ) > 0
{(A − B) ∪ (B − C) ∪ (C − A)} ′ is equal to
St

18. The number of triangles whose vertices are at (a) A ∪ B ∪ C


the vertices of an octagon but name of whose (b) A ∪ (B ∩ C )
sides happen to came from the sides of the (c) A ∩ B ∩ C
octagon is (d) A ∩ (B ∪ C )
(a) 24 (b) 52 (c) 48 (d) 16
27. The function f :[0 , 3] → [1, 29], defined by
19. If n objects are arranged in a row, then the f (x) = 2 x 3 − 15 x 2 + 36 x + 1 is
number of ways of selecting three of these
objects so that no two of then are next to (a) one-one and onto
(b) onto but not one-one
each other is
n−3 n−3 (c) one-one but not onto
(a) C3 (b) C2 (d) neither one-one nor onto
n−2
(c) C3 (d) None of these
WB JEE (Engineering) Practice Set 4 57

28. Let R be the relation over the set of all 35. Let A(2, − 3) and B(−2, 1) be vertices of a
straight lines in a plane such that triangle ABC. If the centroid of this triangle
l1 Rl2 ⇔ l1 ⊥ l2 . Then, R is moves on the line 2 x + 3 y = 1, then the locus
(a) symmetric (b) reflexive of the vertex C is the line
(c) transitive (d) an equivalence relation (a) 3 x − 2 y = 3 (b) 2 x − 3 y = 7
(c) 3 x + 2 y = 5 (d) 2 x + 3 y = 9
29. A biased coin with probability P, 0 < P < 1, 36. The value of λ, for which the lines joining the
of heads is tossed with a head appears for point intersection of curves C1 and C 2 to the

)
the first time. If the probability that the origin are equally inclined to the axis of X .

be
2 C1 : λx 2 + 3 y 2 − 2 λxy + 9 x = 0
number of tosses required is even is , then P
5 C 2 :3 x 2 − 4 y 2 + 8 xy − 3 x = 0
is equal to

Tu
4
2 1 1 1 (a) λ = (b) λ = 12
(a) (b) (c) (d) 3
3 2 3 4
(c) λ = 1 (d) None of these
30. A random variable X takes values − 1, 0 , 1, 2

ou
37. The locus of the centre of a circle which cuts
1 + 3P 1 − P 1 + 2P 1 − 4 P off an intercept of constant length on the
with probabilities , , ,
4 4 4 4 X -axis and which passes through a fixed
respectively, where P varies over R. Then the
minimum and maximum values of the mean
of X are respectively
(Y point on the Y -axis is
(a) a circle
(c) an ellipse
(b) a parabola
(d) a hyperbola
on
7 1 1 5
(a) − and (b) − and 38. If one of the diameter of the circle
4 2 16 16
7 5 1 5 x 2 + y 2 − 2 x − 6 y + 6 = 0 is a chord to the circle
(c) − and (d) − and
pi

4 16 16 4 with centre (2 , 1,) then the radius of circle is


(a) 3 (b) 2 (c) 3 (d) 2
m

31. The solution of the equation


5log a x
+ 5 ×log a 5 = 3, a > 0 is 39. The product of the lengths of perpendiculars
drawn from any point on the hyperbola
ha

log a 5 −log 5 a
(a) 2 (b) 2
(c) 2 −log a 5 (d) 2log 5 a
x 2 − 2 y 2 − 2 = 0 to its asymptotes is
1 2 3
(a) (b) (c) (d) 2
32. The sides of a ∆ABC are in AP such that a 2 3 2
C

minimum { b, c}. Then cos A may be equal to


40. The radius of the circle passing through the
3c − 4 b 3c − 4 b foci of the ellipse 9 x 2 + 16 y 2 = 144 and
dy

(a) (b)
2b 2c having its centre at (0, 3) is
4c − 3 b 4c − 3 b 7
(c) (d) (a) 4 (b) 3 (c) 12 (d)
u

2b 2c 2
St

6 6 41. The equation of the tangent to the curve


33. A variable line through the point  ,  cuts
 5 5 4 x 2 − 9 y 2 = 1 which is parallel to 4 y = 5 x + 7 is
the coordinate axes in the points A and B. If (a) 24 y − 30 x = 17 (b) 30 y − 24 x = ± 161
the point P divides AB internally in the ratio (c) 24 y − 30 x = ± 161 (d) None of these
2 : 1, then the locus of P is 2
πx + sin 4 πx + x − [ x ]
(a) xy = 2 x + y (b) 5 xy = 2 x + y 42. The function f (x) = 3sin ,
(c) 5 xy = 2(2 x + y) (d) 5 xy = 2( x + 2 y) where [ x ] denotes the greatest integer less
than or equal to x, is
34. The bisector of the acute angle formed
(a) a periodic function with period 1
between the lines 4 x − 3 y + 7 = 0 and
(b) a periodic function with period 2
3 x − 4 y + 14 = 0 has the equation 1
(c) a periodic function with period
(a) x + y + 3 = 0 (b) x − y − 3 = 0 2
(c) x − y + 3 = 0 (d) 3 x + y − 7 = 0 (d) not a periodic function
58 WB JEE (Engineering) Practice Set 4

a2 50. If f (x) is continuous and differentiable


43. If tanθ1tanθ2 = − , then the chord joining
b2  1
function such that f   = 0 for all n ∈ N , then
x2 y2  n
two points θ1 and θ 2 on the ellipse + =1
a2 b2 (a) f( x) = 0 for all x ∈ N ∪ (0, 1]
will subtend a right angle at (b) f(0) = 0, f′(0) = 0
(a) focus (b) centre (c) f ′(0) = (0), f ′′(0) = 0
(c) end of the major axis (d) end of minor axis (d) f(0) and f′(0) may or may not be zero

)
44. The equation of the plane through the line Category II (Q. Nos. 51 to 65)

be
x + y + z − 3 = 0 = 2 x − y + 3 z + 1 and parallel
x y z Carry 2 marks each and only one option is
to the line = = , is correct. In case of incorrect answer or any
1 2 3

Tu
combination of more than one answer, 1/2 mark
(a) x − 5 y + 3 z = 7 (b) x − 5 y + 3 z = − 7
(c) x + 5 y + 3 z = 7 (d) x + 5 y + 3 z = − 7
will be deducted.
∞ ∞

ou
1 π2 1
45. If α and β are the roots of the equation 51. If ∑ (2r − 1)2 = 8
, then ∑ 2 is equal to
r
ax 2 + bx + c = 0 , then r =1 r =1

(Y
1 π2 π2
(a) (b)
x −α
lim (1 + ax 2 + bx + c) is 24 3
x→α π2
(c) (d) None of these
(a) e a( α − β ) (b) e a( β − α )
on
6
(c) 1 (d) None of these
52. If I n = ∫ (log x)n dx , then I n + n ⋅ I n − 1 is
46. Let f and g be differentiable functions
pi

equal to
satisfying g ′ (a) = 2, g(a) = b and fog = 1
(identity function). Then, f ′(b) is equal to (a) ( x log x)n (b) x (log x)n (c) n(log x)n (d) (log x)n − 1
m

2
(a) 2 (b) 53. Given,|a| = |b| = 1 and|a + b| = 3. If c be
3
ha

1 vector such that c − a − 2 b = 3(a × b), then


(c) (d) None of these
2 c ⋅ b is equal to
1 1 3 5
(a) − (b) (c) (d)
C

47. A line makes the same angle θ, with each of 2 2 2 2


the X and Z-axes. If the angle β, which it $ β = b$i + c$j + ak$ and
makes with Y -axis, is such that 54. Let α = a$i + b$j + ck,
dy

sin 2 β = 3 sin 2 θ, then cos 2 θ equals γ = c$i + a$j + bk$ be three coplanar vectors with
2 1 3 2 a ≠ b, and v = $i + $j + k$ . Then v is
u

(a) (b) (c) (d)


5 5 5 3 perpendicular to
St

 x (a) α (b) β
 2  (c) γ (d) All of these
48. lim where [⋅] denotes the greatest
π log (sin x)
x→ e 55. A particle P starts from the point Z 0 = 1 + 2i,
2
integer function) where i = − 1 . It moves first horizontally
(a) does not exist (b) equals 1 away from origin by 5 units and then
(c) equals 0 (d) equals −1 vertically away from origin by 3 units to
reach a pont Z1 . From Z1 , the particle moves
49. Let f be a function satisfying f ′ (x) = f (x). 2 units in the direction of the vector $i + $j
Then, ( f −1)′′ (x) is equal to π
1 1
and then it moves through an angle in
(a) − (b) − (c) f( x) (d) f −1( x) 2
x3 x2 anticlockwise direction on a circle with
WB JEE (Engineering) Practice Set 4 59

centre at origin, to reach a point Z 2 . The point local minimum at x = 3. If P()


1 = 6 and
Z 2 is given by : P(3) = 2, then P′(0) is
(a) 6 + 7 i (b) − 7 + 6i (c) 7 + 6i (d) −6 + 7i (a) 8 (b) 9 (c) 3 (d) 6

56. The set of points Z in the complex plane 64. If the graph of the function y = f (x) is
satisfying|Z − i|Z|| = |Z + i|Z|| is contained or symmetrical about the line x = 2, then
equal to the set of points Z satisfying (a) f( x) = − f(− x) (b) f(2 + x) = f(2 − x)
(a) Im (Z ) = 0 (b) Im (Z ) ≤ 1 (c) f( x) = f(− x) (d) f( x + 2 ) = f( x − 2 )
(c)|Re(Z )| ≤ 2 (d)|Z| ≤ 3

)
65. The altitude of a cone is 20 cm and its semi-

be
57. If the system of lines equations vertical angle is 30°. If the semi-vertical angle is
x + 4 ay + az = 0; x + 3b + bz = 0; x + 2cy + cz = 0 increasing at the rate of 2° per second, then the
radius of the base is increasing at the rate of

Tu
have a non-trivial solution, then a , b, c are in
160
(a) HP (b) GP (a) 30 cm/sec (b) cm/sec
(c) AP (d) None of these 3
(c) 10 cm/sec (d) 160 cm/sec

ou
58. Let the function f : R − { − b} → R − {1} be
x+a Category III (Q. Nos. 66 to 75)
defined by f (x) = , a ≠ b, then

(Y
x+b Carry 2 marks each and one or more option(s)
(a) f is one-one but not onto is/are correct. If all correct answers are not marked
(b) f is onto but not one-one and also no incorrect answer is marked then score
= 2 × number of correct answers marked ÷ actual
on
(c) f is both one-one and onto
(d) None of the above number of correct answer. If any wrong option is
marked or if, any combination including a wrong
59. Let f : R → R be given by
pi

option is marked, the answer will be considered


f (x) = [ x 2 ] + [ x + 1] − 3 wrong, but there is no negative marking for the
m

Where [ x ] denotes the greatest integer less same and zero marks will be awarded.
than or equal to x. Then, f (x) is 98 k +1
k +1
ha

(a) many-one and onto (b) many-one and into 66. If ∑ ∫ x (x + 1)


dx , then
(c) one-one and onto (d) one-one and onto k=1 k
(a) I > loge 99 (b) I < loge 99
C

60. The area of the triangle formed by the origin, 49 49


the point P(x , y) and its reflection in X -axis is (c) I < (d) I >
50 50
1
dy

(a) xy (b) 2| x ⋅ y| (c) | xy| (d)| xy|


2 67. Let f : R → R, g : R → R and h : R → R be
differentiable functions such that
61. The foci of a hyperbola are (−5, 18) and
u

f (x) = x 3 + 3 x + 2, g( f (x)) = x and


(10, 20) and it touches the Y -axis. The length h(g(g(x)) = x for all x ∈ R. Then,
St

of its transverse axis is 1


89 (a) g ′(2 ) = (b) h′(1) = 666
(a) 100 (b) (c) 89 (d) 50 15
2 (c) h(0) = 16 (d) h(g (3)) = 36
πx

tan
a  2a 68. Area of the region bounded by the curve
62. The value of lim 2 −  is y = e x and lines x = 0 and y = e is
x→a  x
e

1
2/ π −2/ π 1/ π
(a) e − 1 (b) ∫ ln(e + 1 − y) dy
(a) e π (b) e (c) e (d) e 1
1 e
63. Let P(x) be a real polynomial of least degree (c) e − ∫ e dyx
(d) ∫ ln y dy
which has a local maximum at x = 1 and a 0 1
60 WB JEE (Engineering) Practice Set 4

69. The function f (x) =


sin x
is decreasing in the and (a 2 + 1) x − 6 by + 2(a 3 + a) = 0 ,
x then P is equal to
interval
π π (a) 3a (b) −2a (c) −3a (d) 2a
(a)  − , 0 (b)  0, 
 2   2 73. If a hyperbola passing through the origin has
(c) (0, π ) (d) None of these 3 x − 4 y − 1 = 0 and 4 x − 3 y − 6 = 0 as its
asymptotes, then the equations of its
70. If Z1 = a + ib and Z 2 = c + id are complex
transverse and conjugate axes are
numbers such that|Z1| = |Z 2| = 1 and

)
(a) x − y − 5 = 0 and x + y + 1 = 0
Re(Z1 Z 2) = 0 , then the pair of complex

be
(b) x − y = 0 and x + y + 5 = 0
number ω1 = a + ic and ω 2 = b + id satisfies (c) x + y − 5 = 0 and x − y − 1 = 0
(a)|ω1| = 1 (b)|ω2| = 1 (d) x + y − 1 = 0 and x − y − 5 = 0
(c) Re(ω1ω2 ) = 0

Tu
(d) None of these
74. Let f be a differentiable function satisfying
71. Let P = [ a ij ] be a 3 × 3 matrix and let Q = [ bij ], f (x) + f (y) + f (z) + f (x) f (y) f (z) = 14 for all
where bij = 2 i + j a ij for 1 ≤ i , j ≤ 3. If the x , y , z ∈ R, then

ou
determinant of P is 2, then the determinant (a) f ′( x) < 0 for all x ∈ R (b) f ′( x) = 0 for all x ∈ R
of the matrix Q is (c) f ′( x) > 0 for all x ∈ R (d) None of these
(a) 210 (b) 211 (c) 212

72. If the point (P , 5) lies on the line parallel to


(d) 213
(Y
75. Let f : R → R be defined as f (x) = | x| + | x 2 − 1|.
The total number of points at which f attains
on
Y -axis and passing through the intersection either a local maximum or a local minimum is
of the lines 2(a 2 + 1) x + by + 4 (a 3 + a) = 0 (a) 2 (b) 4 (c) 5 (d) 6
pi

Answers
m
ha

Physics
1. (a) 2. (b) 3. (d) 4. (b) 5. (b) 6. (a) 7. (d) 8. (b) 9. (b) 10. (c)
11. (c) 12. (d) 13. (a) 14. (b) 15. (b) 16. (c) 17. (b) 18. (a) 19. (c) 20. (d)
C

21. (d) 22. (d) 23. (c) 24. (a) 25. (a) 26. (c) 27. (d) 28. (d) 29. (c) 30. (c)
31. (c) 32. (a) 33. (c) 34. (b) 35. (c) 36. (a, b) 37. (a, d) 38. (b, d) 39. (a, d) 40. (a, d)
dy

Chemistry
41. (b) 42. (d) 43. (c) 44. (b) 45. (a) 46. (b) 47. (c) 48. (c) 49. (b) 50. (d)
u

51. (b) 52. (a) 53. (c) 54. (b) 55. (a) 56. (a) 57. (a) 58. (b) 59. (c) 60. (c)
61. (c) 62. (b) 63. (b) 64. (b) 65. (b) 66. (b) 67. (a) 68. (a) 69. (a) 70. (b)
St

71. (d) 72. (b) 73. (d) 74. (a) 75. (c) 76. (a, d) 77. (a, b, c) 78. (a, b) 79. (b, c) 80. (a,b,c,d)

Mathematics
1. (a) 2. (b) 3. (b) 4. (a) 5. (b) 6. (d) 7. (c) 8. (a) 9. (d) 10. (c)
11. (d) 12. (c) 13. (d) 14. (d) 15. (a) 16. (d) 17. (b) 18. (d) 19. (c) 20. (c)
21. (c) 22. (d) 23. (c) 24. (b) 25. (d) 26. (c) 27. (b) 28. (a) 29. (c) 30. (d)
31. (b) 32. (d) 33. (c) 34. (c) 35. (d) 36. (b) 37. (b) 38. (c) 39. (b) 40. (a)
41. (c) 42. (a) 43. (b) 44. (a) 45. (a) 46. (c) 47. (c) 48. (c) 49. (b) 50. (b)
51. (c) 52. (b) 53. (d) 54. (d) 55. (d) 56. (a) 57. (a) 58. (c) 59. (b) 60. (d)
61. (c) 62. (c) 63. (b) 64. (b) 65. (b) 66. (b,d) 67. (b,c) 68. (b,c,d) 69. (b,c) 70. (a,b,c)
71. (d) 72. (b) 73. (c) 74. (b) 75. (c)

* For detailed solutions visit http://tinyurl.com/y2y9bsfn or scan


WB JEE
Engineering Entrance Exam

Practice Set 5

)
be
Tu
Physics

ou
Category I (Q. Nos. 1 to 30) 4. An element with atomic number Z = 11 emits

(Y
Carry 1 marks each and only one option is kα X-ray of wavelength λ. The atomic number of
correct. In case of incorrect answer or any element which emits kα X-ray of wavelength
combination of more than one answer, 1/4 mark 4λ, then
on
(a) 6 (b) 4 (c) 11 (d) 44
will be deducted.
5. A Zener diode of power rating 1 W is to be used
1. Consider a ray of light incident from air into a as a voltage regulator. If Zener has a breakdown
pi

slap of glass (refractive index) of width d, at an of 5 V and it has to regulate voltage which
angle θ. The phase difference between the ray fluctuated between 3 V and 7 V, what should be
m

reflected by the top surface of the glass and the the value of RS for safe operation?
bottom surface is
− 1/ 2
2 πnd 
ha

1 2 
(a)  1 − 2 sin θ + π
λ  n 
Unregulated Regulated
1/ 2
2 πd  1 2  voltage voltage
(b)  1 − 2 sin θ
C

λ  n 
1/ 2
2 πd  1 − 1 sin2 θ + π / 2
(c)   (a) 5 Ω (b) 6 Ω (c) 8 Ω (d) 10 Ω
λ  n2 
dy

1/ 2
2 πd  1 − 1 sin2 θ + 2 π 6. The combination of NAND gates shown here
(d)  
λ  n2  under figure, are equivalent to
u

2. A source contains two phosphorous radio A


St

32
nuclides 15 P (T1 / 2 = 14.3 days) and 15
33
P
(T1 / 2 = 25.3 days). C
33
Initially, 10% of the decay come from 15 P. B
How long one must wait until 90% do so?
(a) 250 days (b) 295 days
(c) 305 days (d) 208 days A
C
3. The wavelength of characteristic X-rays λ kα B

line emitted by hydrogen like atom is 0.32 Å.


The wavelength of λ kβ line emitted by the same (a) an OR gate and an AND gate respectively
(b) an AND gate and a NOT gate respectively
element is
(c) an AND gate and an OR gate respectively
(a) 0.21 Å (b) 0.27 Å (d) an OR gate and a NOT gate respectively
(c) 0.33 Å (d) 0.40 Å
62 WB JEE (Engineering) Practice Set 5

7. The correct dimensional formula for power is same speed as on Earth and the density of
given by planet is same as that of Earth. (Take, escape
(a) ML2T −2 (b) MLT −1 (c) ML2T -3 (d) MLT −2 speed on the surface of the Earth to be 11.2
km/s and radius of Earth to be 6400 km.)
8. The relative density of material of a body is the (a) 2 km (b) 2.2 km
ratio of its weight in air and the loss of its (c) 2.5 km (d) 3 km
weight in water. By using a spring balance, the
weight of the body in air is measured to be 13. The energy stored per unit volume in copper
5.00 ± 0.05 N. The weight of the body in water is wire, which produces longitudinal strain of
measured to be 4.00 ± 0.05 N. Then, the 0.1% is (Young’s modulus = 1.1 × 1011 Nm−2)

)
(a) 11 × 10 4 Jm−3 (b) 11 × 10 3 Jm−3

be
maximum possible percentage error in relative
density is (c) 5.5 × 10 4 Jm−3 (d) 5.5 × 10 3 Jm−3
(a) 11% (b) 10% (c) 9% (d) 7%
14. A 50 kg girl wearing high heel shoes balance on

Tu
9. The two bodies of mass m1 and m2(m1 > m2) a single heel. The heel is circular with the
respectively are tied to the ends of a massless diameter 1.0 cm. What is the pressure exerted
string, which passes over a light and frictionless on the horizontal floor?

ou
pulley. The masses are initially at rest and then (a) 6.24 × 10 6 Pa (b) 9 × 10 3 Pa
released. Then, acceleration of the centre of (c) 3 × 10 6 Pa (d) 2 × 10 4 Pa
mass of the system is

(Y
15. An ideal gas heat engine operates in Carnot
cycle between 227°C and 127°C. It absorbs
6 × 104 cal of heat of higher temperature.
on
Amount of heat converted to work is
(a) 1.2 × 10 4 cal (b) 6 × 10 4 cal
T
(c) 4.8 × 10 4 cal (d) 2.4 × 10 4 cal
a2
pi

m2
16. In an adiabatic process, when pressure is
m

T Cp 3
2
increased by %. If = , then the volume
a1 m 1 3 CV 2
ha

2 2 decreases by about
 m − m2   m − m2 
(a)  1  g (b)  1  9 2 4
 m1 + m2   m1 + m2  (a) % (b) % (c) % (d) 4%
4 3 9
C

(c) g (d) zero


10. If r1 and r2 are the position vectors of two 17. An ideal refrigerator has a freezer at a
temperature of − 13°C. The coefficient of
dy

positive charges q1 and q2 with respect to same


origin, then force on q2 by q1 is directed in the performance of the engine is 5. The
direction given by unit vector temperature of the air (to which heat is
r2 − r1 r1 − r2 rejected) will be
u

(a) $r21 = (b) $r21 = (a) 320°C (b) 39°C (c) 325°K (d) 325°C
| r2 − r1| | r2 − r1|
St

(c) $r21 = r2 = r1 (d) r$21 = r1 − r2 18. In the circuits shown in the figure, the potential
difference across the 4.5 µF capacitor is
11. A boat crosses a river of width 1 km by shortest
path in 15 min. If the speed of boat in still 3µF
water is 5 kmh −1 , then what is the speed of the 4.5µF
river?
(a) 4 kmh−1 (b) 12 kmh−1
(c) 5 kmh−1 (d) 3 kmh−1
6µF
12. A man can jump upto a height of h0 = 1 m on
the surface of the Earth. What should be the 12V
radius of a spherical planet, so that the man
makes a jump on its surface and escape out of 8
its gravity? Assume that the man jumps with (a) 8V (b) 4V (c) 6V (d) V
3
WB JEE (Engineering) Practice Set 5 63

19. In a region, the potential is represented by 24. Three infinite straight wires A, B and C carry
V (x , y , z) = 6 x − 8 xy − 8 y + 6 yz, where V is in currents as shown in the figure. The net force
volts and x , y , z are in metres. The electric force on the wire B is directed
experienced by a charge of 2 C situated at point
(1, 1, 1) is
(a) 4 35 N (b) 30 N
(c) 24 N (d) 6 5 N 1A 2A 3A

20. The electrostatic force of repulsion between two

)
positively charged ions carrying equal charges is

be
3.7 × 10−9 N. When they are separated by a (a) towards A
distance of 5 Å. How many electrons are (b) towards C
missing from each ion? (c) normal to plane of paper

Tu
(a) 5 (b) 2 (c) 3 (d) 1 (d) zero

21. A current I = 5.0 A flows along a thin wire 25. The current i drawn from the 5 V source will be

ou
shaped as shown in figure. The radius of 10Ω
the curved part of the wire is equal to
R = 120 mm. The angle 2φ = 90°, find the
5Ω 10Ω 20Ω

(Y
magnetic induction of the field at the point O.
10Ω
5V
+ –
O
on
I
90° R (a) 0.17 A (b) 0.33 A (c) 0.5 A (d) 0.67 A
26. An ideal battery of emf 2V and a series
pi

A B resistance R, are connected in the primary


−5 circuit of a potentiometer of length 1 m and
(a) 2 × 10 T (b) 2.8 × 10 T
5
m

resistance 5Ω. The value of R, to give a potential


(c) 2.8 × 10 −5 T (d) 2 × 10 5 T
difference of 5 mV across 10 cm of
22. An electron after being accelerated through a potentiometer, where is
ha

potential difference of 104 V enters a uniform (a) 180 Ω (b) 190 Ω (c) 195 Ω (d) 200 Ω
magnetic field of 0.04 T perpendicular to its
27. A coil of inductance 300 mH and resistance 2Ω
C

direction of motion. Calculate the radius of


curvature of its trajectory. is connected to source of voltage 2V. The current
(a) 8.4 mm (b) 8.43 mm reaches half of its steady state value in
dy

(c) 8 mm (d) 8.2 mm (a) 0.05 s (b) 0.1 s (c) 0.15 s (d) 0.3 s

23. A current path shaped as shown in figure 28. A concave lens forms the image of an object
produces a magnetic field at P, the centre of the such that distance between the object and
u

arc. If the arc subtends an angle of 30° and the image is 10 cm; and the magnification
1
St

radius of the arc is 0.6 m. What is the magnitude produced is . The focal length of the lens will
of the field at P, if the current is 3.0 A? 4
A be
(a) 8.6 cm (b) 6.2 cm (c) 10 cm (d) 4.4 cm
C
29. A parallel beam of monochromatic light of
wavelength 450 nm passes through a slit of
P 30° width 0.2 mm. Find the angular divergence in
which most of light is diffracted.
D (a) 4.5 × 10 −3rad (b) 5.5 × 10 −3 rad
(c) 3.2 × 10 −2 rad (d) 3.5 × 10 −3 rad
E
(a) 2.62 × 10 −6 T (b) 2.62 × 10 −7 T 30. White light is used to illuminate the two slits in
(c)3.62 × 10 −9 T (d) 2.62 × 10 −8 T
a Young’s double slit experiment. The
separation between the slits is b and the screen
64 WB JEE (Engineering) Practice Set 5
is at a distance D > > > b from the slit. At a point Category III (Q. Nos. 36 to 40)
on the screen directly in front of one of the
slits. Find the missing wavelength. Carry 2 marks each and one or more option(s)
is/are correct. If all correct answers are not marked
b2 b2 b2 b2 b2 b2
(a) , , ...... (b) , , ......
and also no incorrect answer is marked then score
D 3D 5D 2 D 4D 5D
b2 b2 b2
= 2 × number of correct answers marked ÷ actual
(c) , , ...... (d) None of the above number of correct answers. If any wrong option is
3D 5D 6D
marked or if any combination including a wrong
Category II (Q. Nos. 31 to 35) option is marked, the answer will be considered

)
Carry 2 marks each and only one option is correct. wrong, but there is no negative marking for the

be
In case of incorrect answer or any combination of same and zero marks will be awarded.
more than one answer, 1/2 mark will be deducted. 36. In which of the following situations are heavier

Tu
31. An inductor (L = 100 mH), a resistor of the two particles has smaller de-Broglie
(R = 100 Ω) and a battery (E = 100 V) are initially wavelength?
connected in series as shown in figure. After The two particles

ou
sometime, the battery is disconnected after (a) move with same speed
short circuiting the points A and B. The current (b) move with same KE
(c) move with same linear momentum
in the circuit 1 ms after the short circuit is

(Y
(d) have fallen through the same height
L 37. An ideal gas is taken from the state A(p , V) to
R the state B (p / 2, 2V) along a straight line path
on
as shown in figure. Select the correct statement
A B
from the following.
E
pi

p
1
(a) eA (b) 0.1 A (c) 1 A (d) A
e p
m

A
32. The diameter of a pipe at two points, where a
ha

venturimeter is connected is 8 cm and 5 cm; and p/2


B
the difference of levels in it is 4 cm. The volume
V V 2V V0
of water flowing through the pipe per second is
C

−1 −1
(a) 1889 ccs (b) 1520 ccs (a) Work done by the gas in going from A to B
(c) 1321 ccs −1 (d) 1125 ccs −1
exceeds the work done in going from A to B under
dy

33. A rectangular block is heated from 0°C to 100°C. isothermal conditions


The percentage increase in its length is 0.2%. (b) In the T-V diagram, path AB would become a
What is the percentage increase in its volume? parabola
u

(a) 0.6% (b) 0.10% (c) 0.2% (d) 0.4% (c) In the p-T diagram, path AB would be part of
hyperbola
St

34. The incident intensity on a horizontal surface at (d) In going from A to B, the temperature T of gas first
a sea level from Sun is about 1 kW m −2. increases to a maximum value 1 and then
Assuming that 50 per cent of this intensity is decreases
reflected and 50 per cent is absorbed, determine
the radiation pressure on this horizontal surface. 38. Two particles are projected in air with speed v 0
− 11
(a) 5 × 10 Pa −6
(b) 5 × 10 Pa at angles θ1 and θ 2 (both acute) to the
(c) 1 × 10 −6 Pa (d) 1 × 10 −11 Pa horizontal, respectively. If the height reached
by the first particle is greater than that of the
35. A 20 W, 50 V filament is connected in series to second, then tick the right choices.
an AC mains of 250 V-Hz. Calculate the value of (a) Angle of projection : θ 1 > θ 2
the capacitor required to run the lamp. (b) Time of flight : T1 > T2
(a) 6.6 × 10 −5 F . × 10 −6 F
(b) 51 (c) Horizontal range : R1 > R 2
−6
(c) 4.3 × 10 F (d) 302 × 10 −5 F (d) Total energy : U1 > U 2
WB JEE (Engineering) Practice Set 5 65
(b) VB − VD = 0
39. Figure shows a network of resistances and
(c) VB − VD = 2 V
batteries. Select the correct statements out of
(d) No current flows in the branch BD
the following.
40. Consider a wire carrying a steady current
B
I placed in a uniform magnetic field B
4Ω 4Ω perpendicular to its length. Consider the
+ C
charges inside the wire. It is known that
A
– 2V magnetic forces do no work. This implies that,
(a) motion of charges inside the conductor is

)
2Ω 2Ω unaffected by B, since they do not absorb energy

be
D (b) some charges inside the wire move to the surface
as a result of B
+ –
(c) if the wire moves under the influence of B, no work

Tu
6V is done by the force
(d) if the wire moves under the influence of B, no work
(a) The circuit satisfies the condition of a balanced is done by the magnetic force on the ions,
Wheatstone bridge assumed fixed within the wire

ou
Chemistry
Category I (Q. Nos. 41 to 70)
(Y
45. What is the value of ∆H for a reversible
on
Carry 1 marks each and only one option is isothermal evaporation of 180 g of water
correct. In case of incorrect answer or any at 100°C ? Latent heat of evaporation of water
= 539.7 cal K −1 g −1 .
pi

combination of more than one answer, 1/4 mark


will be deducted. (a) 97.146 kcal (b) 5.397 kcal
m

10 × 373 × 5397
. . × 373
5397
41. A compound with haemoglobin like structure (c) kcal (d) kcal
1000 1000
contain one atom of iron per molecule. If it
ha

contain 4.6% iron, its approximate molecular 46. The right order of the solubility of sulphates of
mass is alkaline earth metals in water is
−1 −1 (a) Be > Ca > Mg > Ba > Sr
C

(a) 100 g mol (b) 1200 g mol


(c) 1600 g mol −1 (d) 1400 g mol −1 (b) Mg > Be > Ba > Ca > Sr
(c) Be > Mg > Ca > Sr > Ba
42. Sulphur oxides which are responsible for major
dy

(d) Mg > Ca > Ba > Be > Sr


air pollution are caused by
(a) burning of coal and refining of petroleum 47. The correct order of stability of conformations
of NH2  CH2  CH2  OH is
u

(b) using indoor combustion devices like cooking gas


(a) gauche > eclipsed > anti
(c) combustion of fuels containing carbon and
St

(b) gauche > anti > eclipsed


hydrogen (c) anti > eclipsed > gauche
(d) burning of fuels in automobiles (d) eclipsed > gauche > anti
43. 1, 2-benzpyrene is 48. The rise in boiling point of a solution containing
(a) a polynuclear hydrocarbon 1.8 g glucose in 100 g of solvent is 0.1°C. The
(b) carcinogenic in nature
molal elevation constant of the liquid is
(c) an aromatic hydrocarbon
(d) Both (a) and (b) (a) 1.0 (b) 2.0 (c) 3.0 (d) 2.5

44. The bond angles of NH3, NH+4 and NH−2 are in 49. 1 mole heptane (v.p. = 92 mm of Hg) is mixed
with 4 mol. Octane (v.p. = 31 mm of Hg), form
the order
an ideal solution. What will be the vapour
(a) NH −2 > NH 3 > NH+4 (b) NH+4 > NH 3 > NH −2 pressure of the solution?
(c) NH 3 > NH 2− > NH+4 (d) NH > NH+4 > NH −2 (a) 41.2 mm of Hg (b) 43.2 mm of Hg
(c) 40.2 mm of Hg (d) 23.2 mm of Hg
66 WB JEE (Engineering) Practice Set 5

50. If the standard half cell reduction potentials are 56. Calcium crystallises in a face centred cubic unit
0.522 V for Cu + /Cu and 0.3402 V for Cu 2+ /Cu. cell with a = 0.556 nm. Calculate the density if it
The standard half cell reduction potential for contained 0.1% Schottky defects.
Cu + /Cu 2+ is (a) 1.5463 g/cm3 (b) 14962
. g/cm3
(a) 0.158 V (b) 0.20 V (c) 0.80 V (d) 0.40 V (c) 15448
. g/cm3 (d) 15943
. g/cm3

51. Identify Z in the following reaction series, 57. Limiting molar conductivity for some ions is
NaOH( ) Al 2 O3 given below (in S cm2 mol −1 ) Na+ = 50.1,
CH3CH2CH2Br  → X → Y
aq
∆ Cl − = 76.3, H+ = 349.6, CH3COO − = 40.9,

)
Ca2+ = 119.0.
HOCl
→ Z

be
Cl OH What will be the limiting molar conductivities
(a) Mixture of H3C and H3C (λ°m) of CaCl 2, CH3COONa and NaCl respectively?
(a) 2716 . and 126.4 S cm2 mol −1
. , 910

Tu
Cl Cl . and 242.8 S cm2 mol −1
(b) 97.65, 1110
(c) 119.0, 1024.5 and 9.2 S cm2 mol −1
OH (d) 111.0, 97.65 and 119.0 S cm2 mol −1

ou
(b) H3C 58. The decreasing order of boiling points of the
following hydrides is

(Y
Cl (a) H 2O > PH 3 > AsH 3 > SbH 3 > NH 3
(b) H 2O > SbH 3 > NH 3 > AsH 3 > PH 3
Cl (c) H 2O > NH 3 > SbH 3 > AsH 3 > PH 3
(c) H3C (d) H 2O > SbH 3 > AsH 3 > PH 3 > NH 3
on
59. Consider the following bromides :
HO Me Me
Me Br Me
pi

Cl
(A) Br Br
(d) H3C (B) (C)
m

Cl
The correct order of SN1 reactivity is
ha

(a) C > B > A (b) A > B > C


52. In the reaction, (c) B > C > A (d) B > A > C
4NH3( g )+ 5O 2( g ) → 4NO ( g )+ 6H2O ()
l 60. What is the reagent X in the following
C

reaction?
When 1 mol of ammonia and 1 mol of O 2 are
made to react to completion then C 6H5 N(CH3)2 + X → (CH3)2N  C 6H4
 CO  C 6H4 N(CH3)2
dy

(a) all the oxygen will consumed


(b) all the ammonia will be consumed (a) CO (b) CO 2
(c) 1.0 mol of NO will be produced (c) COCl 2 (d) OC(OC 2H 5 )2
(d) 1.0 mol of H 2O is produced
u

Br Br
53. Which of the following finds use as a superior  
St

thermometer liquid for high temperature 61. CH2  CH == CH CH2 Zn


→ A
(dust)

measurement? Above reaction is an example of


(a) Thallium (b) Gallium
(c) Mercury (d) Arsenic
1, 4-elimination, predict the product.
(a) CH 3  CH == C == CH 2
54. The correct order of increasing oxidising power (b) CH 3  C ≡≡ C  CH 3
is (c) CH 3  CH 2 C ≡≡ CH
(a) I 2 < Br2 < Cl 2 < F2 (b) Cl 2 < Br2 < F2 < I 2 (d) CH 2 == CH  CH == CH 2
(c) F2 < Br2 < Cl 2 < I 2 (d) F2 < Cl 2 < Br < I 2
62. Which one of the following is not an example
55. Choose the incorrect statement. Roasting is of chain growth polymer?
done to (a) Neoprene (b) Buna-S
(a) make the ore lumpy (b) make the ore porons (c) PMMA (d) Glyptal
(c) remove volatile impurities(d) reduce the oxide to metal
WB JEE (Engineering) Practice Set 5 67

63. 2, 2, 6, 6-tetramethyl cyclohexanol is treated 70. The rate constant of a reaction will be equal to
with an acid. An alkene is formed after the pre-exponential factor when
rearrangement. The structure of the alkene is (a) the absolute temperature is infinity
(b) the absolute temperature is zero
(c) temperature in centigrade is zero
(d) None of the above
(a) (b)

Category II (Q. Nos. 71 to 75)


Carry 2 marks each and only one option is

)
correct. In case of incorrect answer or any

be
(c) (d)
combination of more than one answer, 1/2 mark
will be deducted.

Tu
64. 0.5 mole of each H2, SO 2 and CH4 are kept in a
71. Which of the following is most effective in
container. A hole was made in container. After coagulating ferric hydroxide sol.
3 hours, the order of partial pressure in the (a) KCl (b) FeCl 3

ou
container will be
(c) Na 2SO 4 (d) K 3[Fe(CN)6 ]
(a) pSO 2 > pCH 4 > pH 2 (b) pH 2 > pSO 2 > pCH 4
(c) pCH 4 > pSO 2 > pH 2 (d) pH 2 > pCH 4 > pSO 2 72. Select the incorrect statement.
65. Give the composition of tincture of iodine.
(a)
(b)
2-3% solution of iodine in alcohol water mixture
A mixture of iodine in chloronylenol
(Y (a) Water is considered pure if it has BOD less than 5
ppm
(b) In COD determination, the pollutants resistant to
microbial oxidation are not oxidised by oxidising
on
(c) A mixture of 0.2% phenol and 2-3% iodine in water agent like K 2Cr2O 7
(d) 2-3% solution of iodine in potassium iodide (c) The lower concentration of DO dissolves oxygen,
the more polluted is the water sample
pi

66. Phosphorus pentachloride dissociates as follows (d) The tolerable limit of lead in drinking water is
in a closed reaction vessel, 50 ppm
m

PCl 5(g) - PCl 3(g) + Cl 2(g) 73. Identify the correct statement regarding
If total pressure at equilibrium of the reaction enzymes.
ha

mixture is p and degree of dissociation of PCl 5 is (a) Enzymes are specific biological catalysts that can
X, the partial pressure of PCl 3 will be normaly function at very high temperature
 X   X 
(a)   p (b)   p (T ~ 100 K)
C

 X + 1  X − 1 (b) Enzyme are normally heterogenous catalysts that


 2X   X  are very specific in their action
(c)   p (d)   p
1 − X  1 − X 
dy

(c) Enzymes are specific biological catalysts that


cannot poisoned
67. The isoelectric point of a colloidally dispersed
(d) Enzyme are specific biological catalysts that
material is the pH value at which
u

possess well defined active sites


(a) the dispersed phase does not migrate in an
St

electric field 74. Chloramphenicol is a


(b) the dispersed phase migrate in an electric field (a) narrow spectrum bactericidal antibiotic
(c) the dispersed phase has pH equal to 14 (b) narrow spectrum bacteriostatic antibiotic
(d) the dispersed phase has pH equal to zero (c) broad spectrum bactericidal antibiotic
(d) broad spectrum bacteriostatic antibiotic
68. If the radius of an atom of an element is 75 pm
and the lattice type is body centred cubic, what 75. In the reaction,
is the edge length of the unit cell?
[Al(H2O)6]3+ + H2O - [Al(H2O)5OH]2+
(a) 170 pm (b) 175 pm (c) 178 pm (d) 173.2 pm
+ H3O+
69. Calculate the EMF of the following half cells : 3+
(a) [Al(H 2O 6 )] is an acid
Pt,H2(2atm) | HCl(0.02 M) E È = 0 V
(b) [Al(H 2O)6 ]3+ is a base
(a) − 010
. 5V (b) − 0109
. V (c) Both (a) and (b)
(c) 0109
. V (d) 0104
. V (d) None of the above
68 WB JEE (Engineering) Practice Set 5

Category III (Q. Nos. 76 to 80) (a) (NH 2 )2C == O (b) (NH 2 )2 C == S
(c) P − NH 2C 6H 4SO 3H (d) C 6H 5SO 3H
Carry 2 marks each and one or more option(s)
78. Pottassium cyanide is used for separating
is/are correct. If all correct answers are not marked
(a) Ba 2+ and Ca 2+ (b) Co 2+ and Ni 2+
and also no incorrect answer is marked then score (c) Cu2+ and Cd 2+
= 2 × number of correct answers marked ÷ actual (d) Mn2+ and Zn2+
number of correct answers. If any wrong option is
79. Proteins can be classified into two types on the
marked or if any combination including a wrong
basis of their molecular shape, i.e., fibrous
option is marked, the answer will considered

)
proteins and globular proteins. Examples of
wrong, but there is no negative marking for the

be
globular proteins are
same and zero marks will be awarded. (a) insulin (b) keratin
(c) myosin (d) albumin
76. For particles having same K.E., the de-Broglie

Tu
wavelength is 80. Which of the following reactions should be
(a) unpredictable balanced in basic medium?
(b) inversely proportional to its velocity 4 → MnO 2 + NO 2
(a) NH 3 + MnOs

ou
(c) independent of its mass and velocity
(b) Cr(OH) + I → Cr(OH) + 2Is
2 2 2
(d) directly proportional to its velocity
(c) HNO 3 + Fe 2 + → Fe 3 + + NO 2
77. Which of the following compounds may give
(d) H 2O 2 + Fe 3+ → O 2 + Fe 2+

(Y
blood red colouration while performing
Lassaigne’s test for nitrogen
on
Mathematics
pi

Category I (Q. Nos. 1 to 50) (a) 7 y = 5 x (b) x = 5 y


m

(c) 7 x = 5 y (d) y = 5 x
Only one answer is correct. Correct answer will
5. There are 7 seats in a row. three persons take
fetch full marks 1. Incorrect answer or any
ha

seats at random. The probability that the


combination of more than one answer will fetch middle seat is always occupied and no two
−1/4 marks. persons are consecutive is
C

6 9 4 8
 a −1 + n
b (a) (b) (c) (d)
1. The value of lim   a, b > 0 is 35 22 35 21
n→∞  a  π cos 2 x
dy

(a) 2 a (b) a b (c) b a (d) 4 b 6. The value of ∫ dx, a > 0 is


−π 1 + ax
x
2. The point of intersection of f1 (x) = ∫ (2t − 5) dt 2 π π a
u

2 (a) (b) (c) (d)


x π a 2 π
and f 2(x) = ∫ 2t dt is
St

0 ax + b
7. If S.D of variable x is ∀x, then the SD of ,
(a)  ,
6 36 
(b)  ,  (c)  ,  (d)  , 
2 4 1 1 1 1
 p
 5 25   3 9  3 9  5 25 
∀ a, b, p ∈ R is
3. If the normal at point P(θ) to the ellipse
p a p a
5 x 2 + 14 y 2 = 70 intersects it again at the point (a) σx (b) σx (c) σx (d) σx
Q (2θ), then cosθ is equal to a p a p
2 2 1 1
(a) (b) − (c) (d) − 8. The complex numbers z1 , z2 and z3 satisfying
3 3 3 3
z1 − z3 1 − 3i
4. In ∆ ABC, equation of the right bisectors of the = are the vertices of a triangle
z2 − z3 2
sides AB and AC are x + y = 0 and x − y = 0,
respectively. If A ≡ (5, 7), then equation of side which is
BC is (a) equilateral (b) right angled
(c) right angled isosceles (d) obtuse angled isosceles
WB JEE (Engineering) Practice Set 5 69

9. If a || b × c then (a × b) ⋅ (a × c) is equal to 17. The length of the chord of the parabola y 2 = x


(a) b ⋅ (a ⋅ b )
2
(b) a ⋅ (b ⋅ c )
2 which is bisected at the point (2, 1) is
(c) c 2 ⋅ (a ⋅ b ) (d) (a ⋅ b )2c (a) 4 3 (b) 3 2 (c) 2 3 (d) 2 5

10. Let f (x + y) + f (x − y) = 2 f (x) ⋅ f (y), ∀ x , y ∈ R 18. In a ∆ABC, ∠B = 90°, AC = h and the length of
the perpendicular from B to AC is p such that
f (0) ≠ 0, then f (x) is
h = 4 p. If AB < BC, then ∠C has the measure
(a) periodic (b) odd π 7π π π
(c) even (d) None of these (a) (b) (c) (d)
4 12 12 6

)
11. Solution of the differential equation

be
19. If (x 2 + y 2)dy = xydx and y(x 0) = e , y()
1 = 1, then
 xf (y / x) 
xdy =  y + dx is x 0 is
 f ′ (y / x)  (a) e 3 (b) 2e 2 − 1 / 2

Tu
(a) f( y / x ) = c x , c > 0
e2 + 1
(b) f( y / x ) = x + c, c < 0 (c) e 2 − 1 / 2 (d)
2
(c) f( x / y) = x + c, c > 0

ou
(d) f( x / y) = c x , c < 0 20. If A and B are different matrices satisfying
A 3 = B 3 and A 2B = B 2 A , then
12. Median of 2n C 0 , 2nC1 , 2nC 2 ,....,2n C n (a) at least one det ( A 2 + B2 ) or det ( A − B) must be
(When n is odd) is

(a)
1  2n
 C n −1 +
2 
2n

2 

C n +1 

(b) 2n
C n/ 2 (Y zero.
(b) det ( A 2 + B2 ) as well as det ( A − B) must be zero
(c) det ( A 2 + B2 ) must be zero.
on
2

(c) 2n
Cn (d) None of these (d) det ( A 2 − B2 ) must be zero.

13. If the sum of the coefficients in the expansion 21. If a line is tangent to one point and normal at
pi

of (x − 2 y + 3z) is 128, then the greatest


n another point on the curve x = 4 t 2 + 3,
coefficient in the expansion of (1 + x)n is y = 8t 3 − 1, then slope of each line is
m

(a) 35 (b) 20 (c) 15 (d) 10 (a) ± 2 (b) ± 3 (c) ±1 (d) ± 6


ha

22. If the total number of m elements subsets of


cot −1 ( x + 1 − x) the set A = {a1 , a2 , a3.... an} is λ times the
14. lim is equal to
x →∞  x
−1  2 x + 1 
number of elements subsets containing a4 , then
sec   
C

n is
 x − 1   (a) ( m − 1)λ (b) mλ
(a) 1 (b) 0 (c) ( m + 1)λ (d) 0
dy

π sin x ∞
(c) (d) does not exists
2 23. If = Σ ar x r , then
x r=0
(a) a0 = 1, a1 = 0 (b) a0 = 1, a1 = 0
u

15. The plane passing through the points (5, 1, 2)


(c) a0 = 1, a1 = 1 (d) a0 = 0, a1 = 0
perpendicular to the line 2(x − 2) = y − 4 = z − 5
St

will meet the line on the point 24. The number of negative integral solution of the
(a) (1, 2, 3) (b) (2, 3, 1) equation x1 + x 2 + x 3 + x 4 + x 5 + 10 = 0
(c) (1, 3, 2) (d) (3, 2, 1)
(a) 9C 5 (b) 10C 5
16. If f (x), g(x) and h(x) are three polynomials of (c) 12C 6 (d) 6C 4
degree 2 and
π
25. Suppose, f (x) = x sin x − sin 2 x , x ∈  0,  then
1
f (x) g(x) h(x) 
2 2
∆(x) = f ′ (x) g′ (x) h′ (x) range of f (x) is
f ′′(x) g′′ (x) h′′ (x) (a) ( 0, 1)
π
(b)  0, 
 2
then ∆(x) is a polynomial of degree π − 1 π + 1
(a) 2 (b) 3 (c)  0,  (d)  0, 
 2   2 
(c) atmost 2 (d) atmost 3
70 WB JEE (Engineering) Practice Set 5
π P Q The value of f (7) is
26. In ∆PQR, R = . If tan and tan are the roots
2 2 2 (a) 4 (b) 11 (c) 44 (d) 0
of the equation ax 2 + bx + c = 0, then 2n + 1
k −1
36. Σ (−1) 2
. k equals
(a) a = b + c (b) b = c + a k =1
(c) c = a + b (d) b = c (a) ( n − 1) (2 n − 1) (b) ( n + 1) (2 n + 1)
(c) ( n + 1) (2 n − 1) (d) ( n − 1) (2 n + 1)
27. log 2(x 2 − 3 x + 18) < 4, then x belongs to
(a) (1, 2) (b) (2, 16) 37. If the plane x + y + z = 1 is rotated through an
(c) (1, 16) (d) None of these angle 90° about its line of intersection with the

)
28. The solution set of the inequality plane x − 2 y + 3z = 0, then the new position of

be
 the plane is
 x2 + 4 
log 0. 8 log 6    < 0 is (a) x − 5 y + 4 z = − 1 (b) x − 8 y + 7 z = − 2
  x + 4  (c) x − 5 y − 4 z = + 1 (d) x − 8 y + 7 z = 2

Tu
(a) ( −4, − 3) ∪ ( 8, ∞ ) (b) (4, −3) ∪ ( 8, ∞ )  0 3 3  x
(c) ( −4, ∞ ) (d) None of these 38. If A = −3 0 −4  and B =  y , then B′(AB) is
   
29. Out of thirty points in a plane, eight of them −3 4

ou
0  z 
are collinear. The number of straight lines that
(a) null matrix (b) symmetric matrix
can be formed by joining these point is (c) singular matrix (d) unit matrix
(a) 348 (b) 408 (c) 540 (d) 296

(Y
2
 x2 x4 x6 
30. If a line makes α , β, γ with the coordinate axes, 39. The expression 1 + + + + ..... ∞ will
then  2! 4! 6! 
(a) cos 2α + cos 2β + cos 2 γ − 1 = 0
on
be represented in ascending power of x as
(b) cos 2α + cos 2β + cos 2 γ − 2 = 0
(c) cos 2α + cos 2β + cos 2 γ + 1 = 0 2 2 x2 2 4 x4
(a) 1 + + + ... ∞
(d) None of the above 2! 4!
pi

(2 x )2 2 2 x4
x 49 tan −1 (x 50) (b) 1 + + + .... ∞
31. If ∫ dx = K [tan −1 (x 50)]2 + C, then k 2! 4!
(1 + x100)
m

(2 x )2 2x 4
equals (c) 1 + + + ... ∞
2.2 ! 4!
1 1 1 1
ha

(a) (b) (c) (d) (2 x )2 (2 x )4


100 20 50 200 (d) 1 + + + ... ∞
2.2 ! 2.4 !
32. If f (x) = f (a − x) and g(x) + g(a − x) = 2, then the
1+ 2+ x − 3
C

a
value of ∫ f (x) g(x) dx is 40. lim equals
0 x→ 2 x−2
a a
(a) ∫ f( x ) dx (b) ∫ g ( x ) dx 1
dy

0 0 (a) (b) 3
a a 8 3
(c) ∫ [g ( x ) − f( x )] dx (d) ∫ g ( x ) + f( x )] dx 1
0 0 (c) 2 3 (d) −
π 3
33. Value of θ lying between θ = 0 and θ = and
u

satisfying 2 π xdx
∫0 a2 cos 2 x + b2 sin 2 x is equal to
St

41.
1 + sin θ 2
cos θ 4 sin 4θ
2

π π2 π2 π2
sin θ
2
1 + cos θ2
4 sin 4θ (a) (b) (c) (d)
ab 2 ab ab a + b2
2
sin 2 θ cos 2 θ 1 + 4 sin 4θ
π 5π 9π 7π 42. The locus of the point which divides the
(a) (b) (c) (d)
24 24 24 24 x2 y 2
∞ double ordinates of the ellipse + = 1 in
2n a2 b2
34. The sum of series Σ is
n =1 (2n + 1)! the ratio 1 : 2 internally, is
(a) e −1 (b) e (c) 2e (d) e 3 x2 y2 x2 9 y2
(a) 2
+ 2
=1 (b) 2
+ =1
a b a b2
35. The function f satisfies the functional equation
x2 y2 9 x2 y2
 x + 59 (c) + =9 (d) + =1
3 f (x) + 2 f   = 10 x + 30 for all real x ≠ 1. a2 b2 a2 b2
 x −1 
WB JEE (Engineering) Practice Set 5 71
(a) 5 x 2 − 5 y 2 − 72 x + 54 y + 225 = 0
43. The point on the axis of X, whose perpendicular
x y (b) 5 x 2 + 5 y 2 − 72 x + 54 y + 225 = 0
distance from the straight line + = 1 is a, are (c) 5 x 2 + 5 y 2 + 72 x − 54 y + 225 = 0
y b
(d) 5 x 2 + 5 y 2 − 72 x − 54 y − 225 = 0
(b)  ( b ± a 2 + b 2 ), 0
b a
(a) ( a ± a 2 + b 2 , 0)
a b 
b a Category II (Q. No. 51 to 65)
(c) ( a + b, 0) (d) ( a ± a 2 + b 2 , 0)
a b
Carry 2 marks each and only one option is correct.
44. The Length of major axis of ellipse In case of incorrect answer or any combination of

)
(3 x − 4 y + 7)2 more than one answer, 1/2 mark will be deducted.

be
(5 x − 10)2 + (5 y + 15)2 = is
4
20 10 5 51. The three vectors $i + $j, $j + k,
$ k$ + $i taken two at
(a) (b) (c) (d) 2
a time form three planes. The three unit vectors

Tu
3 3 3
drawn perpendicular to three planes form a
45. If f (x) = (x + 1)2 − 1, (x ≥ −1) Then, the set parallelopiped of volume.
S = { x : f (x) = f −1 (x)} is (a)
4
cubic unit (b) 4 cubic unit

ou
 −3 + i 3 −3 − i 3  3 3
(a)  0, − 1, ,  1 3 3
 2 2  (c) cubic unit (d) cubic unit
2 4

(Y
(b) { 0, 1, − 1}
(c) { 0, − 1} 52. The normal at a variable point A on the ellipse
(d) Empty set
x2 y 2
+ = 1 of eccentricity e meets the axes of
on
46. The equation of tangent to the curve a2 b2
n n
 x  y the ellipse at S and T . Then locus of midpoint of
  +   = 2 at (a, b) is
 a  b ST is a conic with eccentricity e′ such that
pi

x y x y 1 (a) e ′ = e 2 (b) e ′ = e
(a) + =2 (b) + =
a b a b 2 1
(c) e ′ = (d) None of these
m

x y
(c) − = 2 (d) ax + by = 2 e
b a
53. Let x , y , z be such that y(x + 3) ≠ 0. If
ha

47. If A (z1), B(z2) and C(z3) are the vertices of ∆ABC x x + 1 x −1 x +1 y +1 z −1


π AB −y y + 1 y −1 + x −1 y −1 z +1
in which ∠ABC = and = 2 then z2
C

4 BC
z z − 1 z + 1 (−1)n + 2 x (−1)n + 1 y (−1)n z
equals
(a) z3 + i ( z1 + z3 ) (b) z3 − i ( z1 − z3 ) = 0, then n equals
dy

(c) z3 + i ( z1 − z3 ) (d) None of these (a) zero (b) Any integer


r −1 r −1 r −1
(c) Any odd integer (d) Any even integer
2 2.3 4.5 n
48. If D r = α β γ , then ∑ D r
54. If z1 and z2 are two complex numbers such that
u

r =1 z1 = z2 + z1 − z2 , then
2n − 1 3n − 1 5n − 1
St

z  z  z 
(a) Re  1  = 0 (b) Re  1  = Im 1 
equals  z2   z2   z2 
(a) 0 (b) αβγ  z1 
(c) α + β + γ (d) α.2 n + β.3 n + r.4 n (c) Im  = 0 (d) None of these
 z2 
49. The numbers of terms in the expansion of
55. Suppose a, b and c are in AP and a2 , b2 and c 2
(a + b + c)n will be
3
(a) n + 1 (b) n + 3 are in GP. If a < b < c and a + b + c = , then a
( n + 1) ( n + 2 ) z
(c) (d) None of these
2 equals
1 1
(a) (b)
50. The locus of a point P which moves such that 2 3 3 2
2PA = 3PB where coordinates of points A and B 1 1 1 1
(c) − (d) −
are (0, 0) and (4, −3) is 2 2 3 2
72 WB JEE (Engineering) Practice Set 5

56. The vertices of a variable triangle are (3, 4), 63. If α and β are the roots of ax 2 + bx + c = 0, then
(5 cos θ , 5sin θ) and (5sin θ , − 5 cos θ), where θ ∈ R. the equation ax 2 − bx(x − 1) + c(x − 1)2 = 0 has
The Locus of its othocenter is
roots
(a) ( x + y − 7 )2 + ( x − y + 1)2 = 100
α β 1− α 1− β
(b) ( x − y + 7 )2 + ( x − y − 1)2 = 100 (a) , (b) ,
1− α 1− β α β
(c) ( x + y − 7 )2 + ( x + y − 1)2 = 25
α β α +1 β+1
(d) ( x + y − 1)2 + ( x − y − 7 )2 = 25 (c) , (d) ,
α +1 β+1 α β
rα rα
57. If zr = cos + i sin 2 , where r = 1, 2, 3... n , then

)
n2 n 64. The value of

be
lim z1 z2... zn equals
n→∞
2(x)1 / 2 + 3(x)1 / 3 + 4(x)1 / 4 + ....+ n(x)1 / n
lim is
α α x →∞ (2 x − 3)1 / 2 + (2 x − 3)1 / 3 + ....+ (2 x − 3)1 / n
(b) cos   − i sin 
3
(a) e iα
 z  z

Tu
(a) 2 (b) 2 (c) −1 (d) 0
(c) cos α − i sinα (d) e iα / 2
65. tanα and tanβ are the roots of the equation
π π
58. If f (x) = sin 2 x + sin 2  x +  + cos x cos  x +  x 2 + ax + b = 0, then the value of

ou
 3  3
sin 2(α + β) + a sin(α + β)cos(α + β)
and g(5 / 4) = 1, then gof (x) is equal to
(a) 1 (b) −1 (c) 2 (d) −2 + b cos 2(α + β) is equal to
59. The maximum value of the function.
f (x) =
(1 + x)0. 6

1 + x 0. 6
in the interval [0, 1] is
(Y (a) ba (b) a

Category III (Q. Nos. 66 to 75)


(c) b 2a (d) b
on
Carry 2 marks each and one or more option(s)
(a) 2 0. 6 (b) 2 −0. 4 (c) 2 0 (d) 0
is/are correct. If all correct answers are not marked
60. A man is moving away from a tower 41.8m and also no incorrect answer is marked then score
pi

high at a rate of 2m / s. If the eye level of the = 2 × number of correct answers marked ÷ actual
man is 1.8 m above the ground, then the rate at
m

number of correct answer. If any wrong option is


which the angle of elevation of the top of the marked or if, any combination including a wrong
tower changes, when he is 40 m from the foot option is marked, the answer will be considered
ha

of the tower is wrong, but there is no negative marking for the


−1 −2
(a) rad/s (b) rad/s same and zero marks will be awarded.
40 625
C

−4
(c) rad/s (d) None of these 66. A(z1), B(z2), C(z3) and D(z4) are four complex
125
numbers representing the vertices of a rhombus
dy

61. Let A (1, 1) and B(3, 2) be two points. If C is a on the complex plane, then
point on x-axis such that AC + BC is minimum, z1 − z4 z − z4
(a) amp = amp 2
then the coordinates of C are z2 − z4 z3 − z4
u

(a)  , 0 (b)  , 0 z1 −


5 1 z3
3  3  (b) is purely imaginary
St

z2 − z4
(c) ( 3, 0) (d) None of these z − z4
(c) 1 is purely real
62. If a, b, c are the pth, qth, rth terms of an HP and z2 − z3
(d) Not necessary that z1 − z3 ≠ z2 − z4
u = (q − r)$i + (r − p)$j + (p − q )k$ and
$i $j k$ n n +1 n+2
v = + + , then 67. Let f (x) = n Pn n+1
Pn + 1 n+ 2
Pn + 2
a b c
n+1 n+ 2
(a) uxv = $i + $j + k$ n
Cn Cn + 1 Cn + 2
(b) u,v are parallel vectors
(c) u,v are orthogonal vectors Then f (x) is divisible by
(d) u.v = 1 (a) n2 + n + 1 (b) ( n + 1)!
(c) n! (d) None of these
WB JEE (Engineering) Practice Set 5 73

x2 y 2 72. The Eq. of two equal sides PQ , PR of an isoceles


68. If foci of − = 1 coincide with the foci of
a2 b2 triangle PQR are x + y = 5 and 7 x − y = 3. If area
x2 y 2 of ∆PQR is 5, then the equation of BC is
+ = 1 and eccentricity of the hyperbola is (a) 3 x + y − 12 = 0
25 9
(b) 3 x + y + 2 = 0
2, then (c) x − 3 y + 1 = 0
(a) a 2 + b 2 = 16 (d) x − 3 y − 21 = 0

(b) there is no director circle of the hyperbola 73. Let f (x) + f (y) = f (x 1 − y 2 + y 1 − x 2). Then,

)
(c) centre of the director circle is (0, 0) (a) f(2 x 1 − x 2 ) = 2 f( x )

be
(d) Length of latusrectum of the hyperbola = 12 (b) f( 4 x 3 − 3 x ) + 3f( x ) = 0
69. The parabolas y 2 = 4 x and x 2 = 4 y divide (c) f( 4 x 3 − 3 x ) = 3f( x )
the square region bounded by the lines x = 4, (d) f(2 x 1 − x 2 ) + 2 f( x ) = 0

Tu
y = 4 and the coordinate axes. If A1 , A2 and A3
74. If the tangent at any point A (4 m2 , 8m3) of
are respectively, the areas of these parts
x 3 − y 2 = 0 is also a normal to the curve
numbered from top to bottom, then

ou
A2 A1 1 x 3 − y 2 = 0, then m equals
(a) =1 (b) = −3
A3 A2 2 (a)
A 1 A 2

(Y
(c) 3 = (d) 1 = 1 3
A2 2 A2 (b)
2
− 2
70. The roots of the equation (c)
3
on
x 5 − 40 x 4 + Px 3 + Qx 2 + Rx − S = 0 are in GP If 2
(d)
the sum of their reciprocals is 10, then the 3
value of S can be equal to
pi

1 75. If a1 , a2 , ... an are in AP with common difference


(a) −32 (b)
32 d, then
m

−1
(c) (d) 32 1 + a1 a2 1 + a2a3
32 cot −1 + cot −1
d d
ha

71. f (x) = x 2 − 3 x + 2 , then which of the 1 + a a 1 + an an − 1 


+ cot −1 3 4
+ ... + cot −1  
following is/are true ? d  d 
(a) f ′ ( x ) = 2 x − 3 for x ∈ ( 0, 1) ∪ (2, ∞ )
C

is equal to
(b) f ′ ( x ) = 2 x + 3 for x ∈ ( −∞, − 2 ) ∪ ( −1, 0) (a) tan−1 an − tan−1 a1
(c) f ′ ( x ) = − 2 x − 3 for x ∈ ( −2, − 1)
(b) cot −1 a1 + cot −1 an
(d) None of the above
dy

(c) cot −1 a1 − cot −1 an


(d) None of these
u
St
Answers
Physics
1. (a) 2. (d) 3. (b) 4. (a) 5. (d) 6. (a) 7. (c) 8. (a) 9. (a) 10. (a)
11. (d) 12. (c) 13. (c) 14. (a) 15. (a) 16. (c) 17. (b) 18. (a) 19. (a) 20. (b)
21. (c) 22. (b) 23. (b) 24. (a) 25. (c) 26. (c) 27. (b) 28. (d) 29. (a) 30. (a)
31. (d) 32. (a) 33. (a) 34. (b) 35. (b) 36. (a,b, d) 37. (a, b) 38. (a, b) 39. (a, c) 40. (b, d)

)
be
Chemistry
41. (b) 42. (a) 43. (d) 44. (b) 45. (a) 46. (c) 47. (b) 48. (a) 49. (b) 50. (a)
51. (b) 52. (a) 53. (b) 54. (a) 55. (d) 56. (c) 57. (a) 58. (b) 59. (c) 60. (c)

Tu
61. (d) 62. (d) 63. (a) 64. (a) 65. (a) 66. (a) 67. (a) 68. (d) 69. (b) 70. (a)
71. (d) 72. (b) 73. (d) 74. (d) 75. (a) 76. (d) 77. (b, c) 78. (b, c) 79. (a, d) 80. (a, b)

ou
Mathematics
1. (b) 2. (a) 3. (b) 4. (a) 5. (c) 6. (c) 7. (b) 8. (a) 9. (b) 10. (c)
(a) (a) (a) (a) (a) (c) (d) (c) (a) (a)

(Y
11. 12. 13. 14. 15. 16. 17. 18. 19. 20.
21. (a) 22. (b) 23. (a) 24. (a) 25. (c) 26. (c) 27. (a) 28. (a) 29. (b) 30. (c)
31. (a) 32. (a) 33. (d) 34. (a) 35. (a) 36. (b) 37. (b) 38. (a) 39. (d) 40. (a)
41. (c) 42. (b) 43. (b) 44. (b) 45. (c) 46. (a) 47. (c) 48. (a) 49. (c) 50. (b)
on
51. (a) 52. (b) 53. (c) 54. (c) 55. (c) 56. (a) 57. (d) 58. (a) 59. (c) 60. (a)
61. (a) 62. (c) 63. (c) 64. (b) 65. (d) 66. (a, b, c, d) 67. (a, c) 68. (a, b, d) 69. (a, d) 70. (a, d)
71. (a, b, c) 72. (a, b, c, d) 73. (a, b) 74. (c, d) 75. (a, c)
pi

* For detailed solutions visit http://tinyurl.com/y45jxqcn or scan


m
ha
C
u dy
St

You might also like